You are on page 1of 504

www.jeeneetbooks.

in
www.jeeneetbooks.in

www.crackjee.xyz
www.jeeneetbooks.in

Mathematics
ALGEBRA
for IIT-JEE
VOL. 1
www.jeeneetbooks.in
www.jeeneetbooks.in

Mathematics
ALGEBRA
for IIT-JEE
VOL. 1

Dr. G.S.N. Murti


Reader and HOD of Maths (Retd.)
Rajah R. S. R. K. R. R. College, Bobbili, Andra Pradesh, India

Dr. U.M. Swamy


Professor of Mathematics (Retd.), Andhra University, India

Wiley India Pvt. Ltd.


www.jeeneetbooks.in

Mathematics
ALGEBRA
for IIT-JEE
VOL. 1

Copyright © 2010 by Wiley India Pvt. Ltd., 4435-36/7, Ansari Road, Daryaganj, New Delhi-110002.
All rights reserved. No part of this book may be reproduced, stored in a retrieval system, or transmitted in any form or by
any means, electronic, mechanical, photocopying, recording or scanning without the written permission of the
publisher.
Limits of Liability: While the publisher and the author have used their best efforts in preparing this book, Wiley and the
author make no representation or warranties with respect to the accuracy or completeness of the contents of this book,
and specifically disclaim any implied warranties of merchantability or fitness for any particular purpose. There are no
warranties which extend beyond the descriptions contained in this paragraph. No warranty may be created or extended
by sales representatives or written sales materials.
Disclaimer: The contents of this book have been checked for accuracy. Since deviations cannot be precluded entirely,
Wiley or its author cannot guarantee full agreement. As the book is intended for educational purpose, Wiley or its author
shall not be responsible for any errors, omissions or damages arising out of the use of the information contained in the
book. This publication is designed to provide accurate and authoritative information with regard to the subject matter
covered. It is sold on the understanding that the Publisher is not engaged in rendering professional services.
Other Wiley Editorial Offices:
John Wiley & Sons, Inc. 111 River Street, Hoboken, NJ 07030, USA
Wiley-VCH Verlag GmbH, Pappellaee 3, D-69469 Weinheim, Germany
John Wiley & Sons Australia Ltd, 42 McDougall Street, Milton, Queensland 4064, Australia
John Wiley & Sons (Asia) Pte Ltd, 2 Clementi Loop #02-01, Jin Xing Distripark, Singapore 129809
John Wiley & Sons Canada Ltd, 22 Worcester Road, Etobicoke, Ontario, Canada, M9W 1L1
First Edition: 2010
ISBN: 978-81-265-2182-1
ISBN: 978-81-265-8125-2 (ebk)
www.wileyindia.com
Printed at: Sanat Printers, New Delhi
www.jeeneetbooks.in

Dedication

Dedicated to
my mother
Smt. Ganti Balamma
for her untiring efforts to bring up the family to a respectable stage in the
society after our father's premature demise.

Dr. G. S. N. Murti
www.jeeneetbooks.in
www.jeeneetbooks.in

Acknowledgments

Dr. G. S. N. Murti would like to extend his thanks to the following:


1. Mr. U. V. Chalapati Rao, M. Sc. of erstwhile Gangadhar Tutorials, a pioneer
in IIT-JEE coaching, for giving him the opportunity to teach for IIT
coaching in 1985.
2. Dr. P. Narayana, Chairman, Narayana Group of Educational Institutions.
3. Dr. U.M. Swamy, research advisor and co-author of this book, for
immediately accepting his request.
4. Last but not least, his wife Smt. Balamba for her cooperation and advise.

Dr. U. M. Swamy would like to thank the following:


1. His wife Mrs. U. Lakshmi and daughters Sowmya and Mythri for their
excellent support in completing this project.
2. The co-author Dr. G. S. N. Murti for his collaboration in this work.
www.jeeneetbooks.in

Features and Benefits


at a Glance
Feature Benefit to student
Chapter Opener Peaks the student’s interest with the chapter opening vignette, definitions
of the topic, and contents of the chapter.
Clear, Concise, and Inviting Students are able to Read this book, which reduces math anxiety and
Writing Style, Tone and Layout encourages student success.
Theory and Applications Unlike other books that provide very less or no theory, here theory is
well matched with solved examples.
Theorems Relevant theorems are provided along with proofs to emphasize
conceptual understanding.
Solved Examples Topics are followed by solved examples for students to practice and
understand the concept learned.
Examples Wherever required, examples are provided to aid understanding of
definitions and theorems.
Quick Look Formulae/concepts that do not require extensive thought but can be
looked at the last moment.
Try It Out Practice problems for students in between the chapter.
Worked Out Problems Based on IIT-JEE pattern problems are presented in the form of
Single Correct Choice Type Questions
Multiple Correct Choice Type Questions
Matrix-Match Type Questions
Comprehension-Type Questions
Assertion–Reasoning Type Questions
Integer Answer Type Questions
In-depth solutions are provided to all problems for students to
understand the logic behind.
Summary Key formulae, ideas and theorems are presented in this section in
each chapter.
Exercises Offer self-assessment. The questions are divided into subsections as per
requirements of IIT-JEE.
Answers Answers are provided for all exercise questions for student’s to validate
their solution.
www.jeeneetbooks.in

Note to the Students


The IIT-JEE is one of the hardest exams to crack for students, for a very simple
reason – concepts cannot be learned by rote, they have to be absorbed, and IIT
believes in strong concepts. Each question in the IIT-JEE entrance exam is meant
to push the analytical ability of the student to its limit. That is why the questions
are called brainteasers!
Students find Mathematics the most difficult part of IIT-JEE. We understand that
it is difficult to get students to love mathematics, but one can get students to love
succeeding at mathematics. In order to accomplish this goal, the book has been
written in clear, concise, and inviting writing style. It can be used as a self-study
text as theory is well supplemented with examples and solved examples. Wher-
ever required, figures have been provided for clear understanding.
If you take full advantage of the unique features and elements of this textbook,
we believe that your experience will be fulfilling and enjoyable. Let’s walk
through some of the special book features that will help you in your efforts to
crack IIT-JEE.
To crack mathematics paper for IIT-JEE the five things to remember are:
1. Understanding the concepts
2. Proper applications of concepts
3. Practice
4. Speed
5. Accuracy

About the Cover Picture


The Mandelbrot set is a mathematical set of points in the complex plane,
the boundary of which forms a fractal. It is the set of complex values of c
for which the orbit of 0 under iteration of the complex quadratic polynomial
zn+1 = zn2 + c remains bounded. The Mandelbrot set is named after Benoît
Mandelbrot, who studied and popularized it.
www.jeeneetbooks.in
A. PEDAGOGY

CHAPTER OPENER
Quadratic Equations
4 Each chapter starts with an opening vignette, defini-
tion of the topic, and contents of the chapter that give
you an overview of the chapter to help you see the
big picture.
Contents
4.1 Quadratic Expressions
and Equations

Worked-Out Problems
Summary
Exercises
Answers

A polynomial equation of
the second degree having
the general form
Quadratic Equations

<0 ax2 + bx + c = 0
=0 is called a quadratic equation.
Here x represents a variable,
and a, b, and c, constants,
with a ¹ 0. The constants a, b,
and c are called, respectively,
the quadratic coefficient, the
linear coefficient and the
constant term or the free
term.
The term “quadratic” comes
from quadratus, which is the
>0 Latin word for “square”.
Quadratic equations can be
solved by factoring,completing
the square, graphing, Newton’s
method, and using the
quadratic formula (explained
in the chapter).

CLEAR, CONCISE, AND INVITING WRITING


Special attention has been paid to present 4.1 | Quadratic Expressions and Equations
an engaging, clear, precise narrative in the In this section, we discuss quadratic expressions and equations and their roots. Also, we derive various properties
of the roots of quadratic equations and their relationships with the coefficients.
layout that is easy to use and designed to
DE F I NI T I O N 4. 1 A polynomial of the form ax2 + bx + c, where a, b and c are real or complex numbers and
reduce math anxiety students may have. a ¹ 0, is called a quadratic expression in the variable x. In other words, a polynomial f (x)
of degree two over the set of complex numbers is called a quadratic expression. We often
write f ( x) º ax2 + bx + c to denote a quadratic expression and this is known as the standard
form. In this case, a and b are called the coefficients of x2 and x, respectively, and c is called
the constant term. The term ax2 is called the quadratic term and bx is called the linear term.

D E F I NI T I O N 4. 2 If f ( x) º ax2 + bx + c is a quadratic expression and a is a complex number, then we write


f (a) for aa 2 + ba + c. If f (a) = 0, then a is called a zero of the quadratic expression f (x).

DEFINITIONS Examples
(1) Let f (x) º x2 - 5x - 6. Then f (x) is a quadratic expres- (3) Let f ( x) º 2 x2 - ix + 1 be a quadratic expression. In
Every new topic or concept starts with de- sion and 6 and –1 are zeros of f (x). this case i and −i/2 are zeros of f (x).
(2) Let f (x) º x2 + 1. Then f (x) is a quadratic expression (4) The expression x2 + x is a quadratic expression and
fining the concept for students. Related ex- and i and –i are zeros of f (x). 0 and –1 are zeros of x2 + x.

amples to aid the understanding follow the DE F I NI T I O N 4. 3 If f (x) is a quadratic expression, then f (x) = 0 is called a quadratic equation. If a is a zero
of f (x), then a is called a root or a solution of the quadratic equation f (x) = 0. In other
definition. words, if f ( x) º ax2 + bx + c, a ¹ 0, then a complex number a is said to be a root or a solution
of f (x) = 0, if aa 2 + ba + c = 0. The zeros of the quadratic expression f (x) are same as the roots
or solutions of the quadratic equation f (x) = 0. Note that a is a zero of f (x) if and only if x − a
is a factor of f (x).

Examples

(1) 0 and –i are the roots of x2 + ix = 0. (3) i and –i are the roots of x2 + 1 = 0.
(2) 2 is the only root of x2 - 4 x + 4 = 0. (4) i is the only root of x2 - 2ix - 1 = 0.
www.jeeneetbooks.in
Example 4.1 EXAMPLES
Find the quadratic equation whose roots are 2 and –i. ( x - 2)[ x - (-i)] = ( x - 2)( x + i) = x2 + (i - 2) x - 2i
Hence the equation is x2 + (i - 2) x - 2i = 0. Examples pose a specific problem
Solution: The required quadratic expression is

Example 4.2 using concepts already presented


Find the quadratic equation whose roots are 1 + i and 3 [ x - (1 + i)]( x - (1 - i)) = 3 [( x - 1) - i)][( x - 1) + i] and then work through the solution.
1 – i and in which the coefficient of x2 is 3.
= 3 [( x - 1)2 + 1]
These serve to enhance the students'
Solution: The required quadratic expression is = 3 x2 - 6 x + 6
Hence the equation is 3x - 6x + 6 = 0.
2 understanding of the subject matter.
Example 4.3

If a and b are roots of the quadratic equation ax2 + bx + 0 = ( x - za )( x - zb )


c = 0 and z is any complex number, then find the quadratic
= x2 - (za + zb ) x + za ´ zb
equation whose roots are za and zb .
= x2 + z[-(a + b )]x + z2ab
Solution: We have
æ bö c
-b c = x2 + z ç ÷ x + z2
a+b= and ab = è aø a
a a
that is,
The equation whose roots are za and zb is
ax2 + zbx + z2 c = 0

Example 4.4

If a and b are the roots of a quadratic equation Therefore, the required equation is
ax2 + bx + c = 0, then find the quadratic equation whose
0 = a[ x - (a + z)] ´ [ x - (b + z)]
roots are a + z and b + z, where z is any given complex
number. = ax2 + a[-(a + z) - (b + z)]x + a(a + z)(b + z)
æb ö æc b ö
Solution: We have = ax2 + a ç - 2z÷ x + a ç - z + z2 ÷
èa ø èa a ø

THEOREMS THEOREM 4.5 If a, b and c are real numbers and a ¹ 0, then (4ac - b2 )/ 4a is the maximum or minimum value of
quadratic equation of f ( x) º ax2 + bx + c according as a < 0 or a > 0, respectively.
PROOF We have
Relevant theorems are provided along
æ b cö
f ( x) º ax2 + bx + c º a ç x2 + x + ÷
with proofs to emphasize conceptual un- è a aø

derstanding rather than rote learning. éæ bö


º a êç x + ÷ +
4ac - b2 ù
2
æ bö
ú º aç x + ÷ +
2
4ac - b2
êëè 2a ø 4a2 úû è 2a ø 4a

If a < 0, then
4ac - b2 æ -b ö
f ( x) £ =fç ÷ for all x Î
4a è 2a ø

Hence (4ac - b )/ 4a is the maximum value of f ( x).


2

If a > 0, then
æ -b ö 4ac - b
2
fç ÷= £ f ( x) for all x Î
è 2a ø 4a
Hence (4ac - b )/ 4a is the minimum value of f ( x).
2

QUICK LOOK 2 QUICK LOOK


Let f ( x) º ax + bx + c = 0 be a quadratic equation and
2
3. f (- x) = 0 is an equation whose roots are -a and -b.
a and b be its roots. Then the following hold good. 4. If ab ¹ 0 and c ¹ 0, f(1/x) = 0 is an equation whose
1. f (x - z) = 0 is an equation whose roots are a + z and roots are 1/a and 1/b .
Some important formulae and con-
b + z, for any given complex number z. 5. For any complex numbers z1 and z2 with z1 ¹ 0, cepts that do not require exhaustive
2. f ( x / z) = 0 is an equation whose roots are za and zb f [( x - z2 )/z1 ] = 0 is an equation whose roots are
for any non-zero complex number z. z1a + z2 and z1 b + z2. explanation, but their mention is im-
portant, are presented in this section.
These are marked with a magnifying
glass.
www.jeeneetbooks.in
TRY IT OUT Try it out Verify the following properties:
1. ((a, b) + (c, d)) + (s, t) = (a, b) + ((c, d) + (s, t))
Within each chapter the stu- 2. (a, b) + (c, d) = (c, d) + (a, b)
dents would find problems 3. (a, b) + (0, 0) = (a, b)
4. (a, b) + (-a, -b) = (0, 0)
to reinforce and check their 5. (a, b) + (c, d) = (s, t) Û (a, b) = (s, t) - (c, d)
understanding. This would Û (c, d) = (s, t) - (a, b)
help build confidence as one
DEF I NI TI O N 3 . 2 For any complex numbers (a, b) and (c, d), let us define
progresses in the chapter.
(a, b) × (c, d) = (ac - bd, ad + bc)
These are marked with a
This is called the product of (a, b) and (c, d) and the process of taking products is called
pointed finger. multiplication.

Try it out Verify the following properties for any complex numbers (a, b), (c, d) and (s, t).
1. [(a, b) × (c, d)] × ( s, t ) = (a, b) × [(c, d) × ( s, t )]
2. (a, b) × (c, d) = (c, d) × (a, b)
3. (a, b) × [(c, d) + ( s, t )] = (a, b) × (c, d) + (a, b) × ( s, t )
4. (a, b) × (1, 0) = (a, b)
5. (a, 0) × (c, d) = (ac, ad)
6. (a, 0) × (c, 0) = (ac, 0)
7. (a, 0) + (c, 0) = (a + c, 0)

SUMMARY SUMMARY
4.1 Quadratic expressions and equations: If a, b, c a At the end of every
ax2 + bx + c = (a ¢x2 + b¢ x + c ¢)
are real numbers and a ≠ 0, the expression of the a¢
form ax2 + bx + c is called quadratic expression and chapter, a summary is
4.8 Cube roots of unity: Roots of the equation x - 1 = 0
3
ax2 + bx + c = 0 is called quadratic equation. presented that organ-
are called cube roots of unity and they are
4.2 Let f (x) º ax2 + bx + c be a quadratic expression izes the key formulae
and a be a real (complex) number. Then we write -1 3
1, ±i and theorems in an
f (a) for aa2 + ba + c. If f(a) = 0, the a is called a zero 2 2
of f(x) or a root of the equation f(x) = 0. -1/ 2 ± i 3 / 2 are called non-real cube roots of unity. easy to use layout. The
Further each of them is the square of the other and related topics are indi-
4.3 Roots: The roots of the quadratic equation ax2 +
the sum of the two non-real cube roots of unity is
bx + c = 0 are cated so that one can
equal to -1. If w ≠ 1 is a cube root of unity and n is
- b + b2 - 4ac - b - b2 - 4ac any positive integer, then 1 + wn + w2n is equal to 3 quickly summarize a
and or 0 according as n is a multiple of 3 or not.
2a 2a chapter.
4.9 Maximum and minimum values: If f(x) º ax2 +
4.4 Discriminant: b2 - 4ac is called the discriminant of
bx + c and a ≠ 0, then
the quadratic expression (equation) ax2 + bx + c = 0.
æ - b ö 4ac - b
2

4.5 Sum and product of the roots: If a and b are roots of fç ÷=


è 2a ø 4a
the equation ax2 + bx + c = 0, then
is the maximum or minimum value of f according
-b c as a < 0 or a > 0.
a+b= and a b =
2a a
4.10 Theorems (change of sign of ax + bx + c): Let f(x) º
2

4.6 Let ax + bx + c = 0 be a quadratic equation and


2
ax + bx + c where a, b, c are real and a ≠ 0. If
2

Δ = b2 - 4ac be its discriminant. Then the following a and b are real roots of f(x) = 0 and a < b, then
hold good. (1) (i) f(x) and a (the coefficient of x ) have the
2

(1) Roots are equal Û Δ = 0 (i.e., b2 = 4ac). same sign for all x < a or x > b.
(2) Roots are real and distinct Û Δ > 0. (ii) f(x) and a will have opposite signs for all x
(3) Roots are non-real complex (i.e., imaginary) Û such that a < x < b.
Δ > 0. (2) If f (x) = 0 has imaginary roots, then f(x) and a
will have the same sign for all real values of x.
4.7 Theorem: Two quadratic equations ax + bx + c = 0
2

and a ¢x2 + b¢ x + c ¢ = 0 have same roots if and only 4.11 If f(x) is a quadratic expression and f (p)f (q) < 0
if the triples (a, b, c) and (a¢, b¢, c¢ ) are proportional for some real numbers p and q, then the quadratic
and in this case equation f (x) = 0 has a root in between p and q.
www.jeeneetbooks.in
B. WORKED-OUT PROBLEMS AND ASSESSMENT – AS PER IIT-JEE PATTERN
Mere theory is not enough. It is also important to practice and test what has been
proved theoretically. The worked-out problems and exercise at the end of each
chapter are in resonance with the IIT-JEE paper pattern. Keeping the IIT-JEE
pattern in mind, the worked-out problems and exercises have been divided into:
1. Single Correct Choice Type Questions
2. Multiple Correct Choice Type Questions
3. Matrix-Match Type Questions
4. Comprehension-Type Questions
5. Assertion–Reasoning Type Questions
6. Integer Answer Type Questions

WORKED-OUT PROBLEMS
In-depth solutions are provided to all worked-out problems for students to understand the logic behind and
formula used.

WORKED-OUT PROBLEMS
SINGLE CORRECT
Single Correct Choice Type Questions CHOICE TYPE
1. If the equations m < 0 and 3m2 + 4 m - 4 > 0
Þ m < 0 and (3m - 2)(m + 2) > 0
QUESTIONS
x2 + ax + 1 = 0 and x2 - x - a = 0
have a real common root, then the value of a is This gives m < -2 and so
(A) 0 (B) 1 (C) −1 (D) 2 x2 - 5 x + 6 < 0 Þ ( x - 2)( x - 3) < 0 Þ x Î (2, 3) These are the regular mul-
Answer: (C)
Solution: Let a be a real common root. Then tiple choice questions with
a 2 + aa + 1 = 0 4. If p is prime number and both the roots of the equation
x2 + px - (444) p = 0 are integers, then p is equal to four choices provided. Only
a -a -a=0
2
(A) 2 (B) 3 (C) 31 (D) 37
Therefore one among the four choices
Solution: Suppose the roots of x2 + px - (444) p = 0 are
a (a + 1) + (a + 1) = 0 integers. Then the discriminant will be the correct answer.
(a + 1)(a + 1) = 0 p2 + 4(444) p = p{ p + 4 ´ (444)}
If a = - 1, then the equations are same and also cannot must be a perfect square. Therefore p divides p + 4 ´
have a real root. Therefore a + 1 ¹ 0 and hence a = - 1, (444). This implies
so that a = 2.
p divides 4 ´ (444) = 24 ´ 3 ´ 37
Answer: (D)
Th f

MULTIPLE Multiple Correct Choice Type Questions


CORRECT CHOICE 1. Suppose a and b are integers and b ¹ -1. If the quadratic
equation x2 + ax + b + 1 = 0 has a positive integer root,
Solution:
Case 1: Suppose b is even, that is, b = 2 m. Then b2 - 4ac =
TYPE QUESTIONS then 4(m2 - ac) = 4k.
Case 2: Suppose b is odd, that is, b = 2 m - 1. Then
(A) the other root is also a positive integer
(B) the other root is an integer
b2 - 4ac = (2 m - 1)2 - 4ac
Multiple correct choice type (C) a2 + b2 is a prime number
(D) a2 + b2 has a factor other than 1 and itself = 4 m2 + 4 m + 1 - 4ac
questions have four choices Solution: Let a and b be the roots and a be a positive = 4(m2 + m - ac) + 1
provided, but one or more of integer. Then
= 4k + 1
the choices provided may be a + b = -a and ab = b + 1 Answers: (A), (B)
b = -a - a implies b is an integer and
correct. 3. If a and b are roots of the equation x2 + ax + b = 0,
a2 + b2 = (a + b )2 + (ab - 1)2 then
= a2 + b2 + a2 b2 + 1 (A) a = 0, b = 1 (B) a = 0 = b
= (a 2 + 1)(b 2 + 1) (C) a = 1, b = - 1 (D) a = 1, b = - 2
Solution: If a + b = -a and ab = b, then a = 0 = b or a = 1,
Since a 2 + 1 > 1 and b 2 + 1 > 1, it follows that a 2 + 1 is a
b = -2.
factor of a2 + b2 other than 1 and itself.
Answers: (B), (D)
Answers: (B), (D)
www.jeeneetbooks.in
MATRIX-MATCH TYPE QUESTIONS
These questions are the
regular “Match the Follow- Matrix-Match Type Questions
ing” variety. Two columns 1. Match the items in Column I with those in Column II z4 - 4z3 + 7z2 - 6z + 3 = z2 - 2z + 3

each containing 4 subdivi- = (z - 1)2 + 2 = i2 + 2 = 1


Column I Column II
4(z4 - 4z3 + 7z2 - 6z + 3) = 4
sions or first column with (A) If z = x + iy, z1/ 3 = a - ib and (p) 10
x y Answer: (D) Æ (s)
four subdivisions and sec- - = l (a2 - b2 ), then l is
a b (q) 14
2. Match the items in Column I with those in Column II.
ond column with more sub- (B) If | z - i | < 1, then the value of In the following, w ¹ 1 is a cube root of unity.
divisions are given and the | z + 12 - 6i | is less than (r) 1
(C) If | z1 | = 1 and | z2 | = 2, then Column I Column II
student should match ele- | z1 + z2 |2 + | z1 - z2 |2 is equal to
(s) 4
(A) The value of the determinant (p) 3w (1 - w )
ments of column I to that (D) If z = 1 + i, then (t) 5
4 (z4 - 4z3 + 7z2 - 6z + 3) is equal to 1 1 1
of column II. There can be
1 - 1 - w2 w2 is
one or more matches. Solution: 1 w2 w4 (q) 3w(w - 1)
(A) x + iy = z = (a - ib)3 = a3 - 3a2 bi + 3a(ib)2 - i3 b3
(B) The value of 4 + 5w 2002
+ 3w2009

= (a3 - 3ab2 ) + i(b3 - 3a2 b) is


(r) -i 3
Comparing the real parts we get (C) The value of the determinant
x = a3 - 3ab2 = a(a2 - 3b2 ) 1 1 + i + w2 w2
x 1- i -1 w2 - 1 is (s) i 3
= a2 - 3b2
a -i -i + w 2 + 1 -1
Comparing the imaginary parts we get (D) w2 n + wn + 1 (n is a positive integer (t) 0
d l i l f 3) i

COMPREHENSION-TYPE QUESTIONS
Comprehension-type questions consist
Comprehension-Type Questions
of a small passage, followed by three
1. Passage: 4 Indians, 3 Americans and 2 Britishers are (ii) The number of ways in which all the four prizes can
to be arranged around a round table. Answer the be given to any one of the 6 students = 6. Therefore multiple choice questions. The ques-
following questions. the required number of ways is 6 4 - 6 = 1290.
(i) The number of ways of arranging them is Answer: (B) tions are of single correct answer type.
1 1 (iii) Give a set of two prizes to the particular student.
(A) 9! (B) 9! (C) 8! (D) 8!
2 2 Then the remaining 2 can be distributed among
5 students in 52 ways. There are 4 C 2 sets, each
(ii) The number of ways arranging them so that the
containing 2 prizes. Therefore the required number
two Britishers should never come together is
of ways of distributing the prizes is
(A) 7 ! ´ 2 ! (B) 6 ! ´ 2 ! (C) 7! (D) 6 ! 6 P2
(iii) The number of ways of arranging them so that 52 ´ 4C 2 = 25 ´ 6 = 150
the three Americans should sit together is Answer: (C)
(A) 7 ! ´ 3! (B) 6 ! ´ 3! (C) 6 ! 6 P3 (D) 6 ! 7 P3
3. Passage: A security of 12 persons is to form from a
Solution: group of 20 persons. Answer the following questions.
(i) n distinct objects can be arranged around a circular (i) The number of times that two particular persons
table in (n - 1)! ways. Therefore the number of ways are together on duty is
of arranging 4 + 3 + 2 people = 8!.
20 ! 18 ! 20 ! 20 !
Answer: (C) (A) (B) (C) (D)
12 ! 8 ! 10 ! 8 ! 10 ! 8 ! 10 ! 10 !
(ii) First arrange 4 Indians and 3 Americans around a
round table in 6! ways. Among the six gaps, arrange (ii) The number of times that three particular
the two Britishers in 6 P2 ways. Therefore the total persons are together on duty is
number of arrangements in which Britishers are 17 ! 17 ! 20 ! 20 !
(A) (B) (C) (D)
separated is 6 ! ´ 6 P2 . 8! 9! 8! 8! 17 ! 3! 9! 8!
Answer: (D) (iii) The number of ways of selecting 12 guards such
(iii) Treating the 3 Americans as a single object, 7 (= 4 + that two particular guards are out of duty and
1 + 2) objects can be arranged cyclically in 6! ways. three particular guards are together on duty is
In each of these, Americans can be arranged among (20)! (18)! (15)! (15)!
themselves in 3! ways. Therefore, the number of (A) (B) (C) (D)
(15)! 5! 9 ! 3! 9! 6! 5! (10)!
required arrangements is 6 ! ´ 3!.
www.jeeneetbooks.in
ASSERTION–REASONING TYPE QUESTIONS
These questions check the
Assertion–Reasoning Type Questions
analytical and reasoning
2. Statement I: If P( x) = ax + bx + c and Q( x) = - ax +
2 2
In the following set of questions, a Statement I is given
skills of the students. Two and a corresponding Statement II is given just below it. dx + c , where ac ¹ 0, then the equation P ( x) Q ( x) = 0
Mark the correct answer as: has at least two real roots.
statements are provided –
(A) Both I and II are true and II is a correct reason for I Statement II: A quadratic equation with real coeffi-
Statement I and Statement ( B ) Both I and II are true and II is not a correct reason cients has real roots if and only if the discriminant is
for I greater than or equal to zero.
II. The student is expected ( C ) I is true, but II is false Solution: Let px2 + qx + r = 0 be a quadratic equation.
( D) I is false, but II is true
to verify if (a) both state- The roots are

ments are true and if both 1. Statement I: Let a, b and c be real numbers and - q ± q2 - 4 pr
a ¹ 0. If 4a + 3b + 2c and a have same sign, then not 2p
are true, verify if statement both the roots of the equation ax2 + bx + c = 0 belong
to the open interval (1, 2). These are real Û q2 - 4 pr ³ 0. Therefore Statement II
I follows from statement is true.
Statement II: A quadratic equation f ( x) = 0 will have In Statement I, ac ¹ 0. Therefore ac > 0 or ac < 0. If
II; (b) both statements are a root in the interval (a, b) if f (a) f (b) < 0 . ac < 0, then b2 - 4ac > 0, so that P(x) = 0 has two real roots.
If ac > 0, then d2 + 4ac > 0 so that Q(x) = 0 has two real
true and if both are true, Solution: Let f ( x) = px2 + qx + r . If f (a) and f (b) are
roots. Further, the roots of P(x) = 0 and Q(x) = 0 are also
of opposite sign, the curve (parabola) y = f ( x) must
verify if statement II is not the roots of P(x)Q(x) = 0. Therefore, Statement I is true
intersect x-axis at some point. This implies that f (x) has a
and Statement II is a correct reason for Statement I.
the correct reasoning for root in (a, b). Therefore, the Statement II is true.
Let a and b be roots of ax2 + bx + c = 0 . Then, Answer: (A)
statement I; (c), (d) which -b c 3. Statement I: If a, b and c are real, then the roots of the
a+b= and ab = equation (x - a)(x - b) + (x - b)(x - c) + (x - c)(x - a) = 0
of the statements is untrue. a a
are imaginary.
By hypothesis,
Statement II: If p, q and r are real and p ¹ 0 , then
4a + 3b + 2c the roots of the equation px2 + qx + r = 0 are real or
>0 imaginary according as q2 - 4 pr ³ 0 or q2 - 4 pr < 0.
a

INTEGER-TYPE QUESTIONS
Integer Answer Type Questions The questions in this section are nu-
The answer to each of the questions in this section is 2. The number of negative integer solutions of x2 ´ 2x + 1 + merical problems for which no choices
a non-negative integer. The appropriate bubbles below 2| x - 3|+2 = x2 ´ 2| x - 3|+ 4 + 2x - 1 is .
the respective question numbers have to be darkened. are provided. The students re required
3. If (a + 5i)/ 2 is a root of the equation 2 x - 6 x + k = 0,
2
For example, as shown in the figure, if the correct answer
to the question number Y is 246, then the bubbles under then the value of k is . to find the exact answers to numerical
Y labeled as 2, 4, 6 are to be darkened.

X Y Z W
4. If the equation x - 4 x + log1/ 2 a = 0 does not have
2
problems and enter the same in OMR
distinct real roots, then the minimum value of 1/a
0 0 0 0 is . sheets. Answers can be one-digit or
1 1 1 1
2 2 2 5. If a is the greatest negative integer satisfying two-digit numerals.
3 3 3 3 x2 - 4 x - 77 < 0 and x2 > 4
4 4 4
simultaneously, then the value of | a | is .
5 5 5 5
6 6 6 6. The number of values of k for which the quad-
7 7 7 7 ratic equations (2k - 5)x2 - 4x - 15 = 0 and (3k - 8)
8 8 8 8 x2 - 5x - 21 = 0 have a common root is .
9 9 9 9
7. The number of real roots of the equation 2 x2 - 6 x -
5 x2 - 3 x - 6 = 0 is .
1. The integer value of k for which
www.jeeneetbooks.in
EXERCISES EXERCISES
Single Correct Choice Type Questions
1. The roots of the equation For self-assessment, each chapter has
(C) ln = m2 + c / a (D) mn = l2 + bc / a
17
(10)2 /x
+ (25) = (50)1/x
1 /x
adequate number of exercise prob-
4 3. If x is real, then the least value of
are 6 x2 - 22 x + 21
lems where the questions have been
(A) 2, 1/2 (B) -2, 1/2 (C) 2, -1/2 (D) 1/2, -1/2 5 x2 - 18 x + 17 subdivided into the same categories as
2. If a ¹ 0 and a(l + m) + 2blm + c = 0 and a(l + n) + is
2 2

2 bln + c = 0, then (A) 5/4 (B) 1 (C) 17/4 (D) -5/4


asked in IIT-JEE pattern.
(A) mn = l2 + c / a (B) lm = n2 + c / a

Multiple Correct Choice Type Questions


2
+ log2 x - ( 5 / 4 )
1. The equation x
( 3 / 4 )(log2 x )
= 2 has (A) a + b (B) a − b
(A) atleast one real solution (C) ( a + b )2 (D) ( a - b )2
(B) exactly three solutions
(C) exactly one irrational solution 8. If the product of the roots of the equation
(D) complex roots x2 - 4 mx + 3e2 log m - 4 = 0
is 8, then the roots are
2. If S is the set of all real values of x such that
(A) real (B) non-real
2x - 1 (C) rational (D) irrational
>0
2 x + 3 x2 + x
3
- log1/ 9 [ x2 - ( 10 / 3) x + 1]
9. If 3 £ 1, then x belongs to
then S is a superset of (A) [0, 1/3) (B) (1/3, 1)
(A) (-¥, - 3 / 2) (B) (-3/2, -1/4) (C) (2, 3) (D) (3, 10/3]
(C) (-1/ 4, 1/ 2) (D) (1/2, 3)
10. If every pair of the equations x2 + ax + bc = 0, x2 + bx +
Matrix-Match Type Questions
In each of the following questions, statements are given
in two columns, which have to be matched. The state- Column I Column II
ments in Column I are labeled as (A), (B), (C) and
(D), while those in Column II are labeled as (p), (q), (A) The equation (p) cx2 + bx + a = 0
(r), (s) and (t). Any given statement in Column I can whose roots are
have correct matching with one or more statements in a + b and ab is
(B) The equation (q) a2 x2 + (2ac - b2 ) x + c2 = 0
Column II. The appropriate bubbles corresponding to
the answers to these questions have to be darkened as whose roots are a 2
illustrated in the following example. and b 2 is (r) a2 x2 + a(b - c) x - bc = 0
(C) The equation
Example: If the correct matches are (A) ® (p), (s); whose roots are
(B) ® (q), (s), (t); (C) ® (r); (D) ® (r), (t); that is if the (s) ax2 + (2ac + b) x + ac2 +
1/a and 1/b is
matches are (A) ® (p) and (s); (B) ® (q), (s) and (t); bc + c = 0
(D) The equation
(C) ® (r); and (D) ® (r), (t), then the correct darkening
whose roots are
of bubbles willComprehension-Type
look as follows: Questions a - c and b - c is (t) cx2 - bx + a = 0
1. Passage: Let A be a square matrix. Then (A) idempotent matrix
(A) A is called idempotent matrix, if A2 = A. (B) involutory
(B) A is called nilpotent matrix of index k, if Ak = O (C) nilpotent matrix of index 2
and Ak-1 ¹ O. (D) AAT = I.
(C) A is called involutory matrix if A2 = I.
2. Passage: Let A be 3 ´ 3 matrix and B is adj A. Answer
(D) A is called periodic matrix with least periodic k, if the following questions:
Ak + 1 = A and Ak ¹ A.
é0 1 1ù
(i) If A = êê 1 2 0 úú , then A-1 is equal to
Answer the following questions:

0 - 1ù êë 3 - 1 4 úû
(i) The matrix éê ú is
ë-1 0 û
é 8 -5 -2ù é- 8 5 2ù
1 ê
- 4 - 3 1 úú
1 ê
(A) idempotent (B) involutory (A) (B) 4 3 - 1úú
Assertion–Reasoning Type Questions 11 ê 11 ê
In each of the following, two statements, I and II, are given Statement II: If f ( x) º ax2 + bx + c > 0 for all x > 5,
and one of the following four alternatives has to be chosen. then the equation f ( x) = 0 may not have real roots or
(A) Both I and II are correct and II is a correct reasoning will have real roots less than or equal to 5.
for I.
(B) Both I and II are correct but II is not a correct 2. Statement I: If a, b and c are positive integers and
reasoning for I. ax2 - bx + c = 0 has two distinct roots in the integer
(0, 1), then log5 (abc) ³ 2.
(C) I is true, but II is not true.
(D) I is not true, but II is true. Statement II: If a quadratic equation f ( x) = 0 has
roots in an interval (h, k), then f (h), f (k ) > 0
1. Statement I: If f ( x) º ax + bx + c is positive for all x
2

greater than 5, then a > 0, but b may be negative or 3. Statement I: There are only two values for sin x satis-
2 2
may not be negative. fying the equation 2sin x
+ 5 ´ 2cos x
= 7.

Integer Answer Type Questions


The answer to each of the questions in this section is 2. The number of negative integer solutions of x2 ´ 2x + 1 +
a non-negative integer. The appropriate bubbles below 2| x - 3|+2 = x2 ´ 2| x - 3|+ 4 + 2x - 1 is .
the respective question numbers have to be darkened.
3. If (a + 5i)/ 2 is a root of the equation 2 x - 6 x + k = 0,
2
For example, as shown in the figure, if the correct answer
to the question number Y is 246, then the bubbles under then the value of k is .
Y labeled as 2, 4, 6 are to be darkened.
4. If the equation x2 - 4 x + log1/ 2 a = 0 does not have
X Y Z W distinct real roots, then the minimum value of 1/a
0 0 0 0 is .
1 1 1 1
2 2 2 5. If a is the greatest negative integer satisfying
3 3 3 3 x2 - 4 x - 77 < 0 and x2 > 4
4 4 4
simultaneously, then the value of | a | is .
5 5 5 5
www.jeeneetbooks.in
ANSWERS
The Answer key at the end of each chapter contains answers to all exercise problems.

ANSWERS
Single Correct Choice Type Questions
1. (D) 14. (B)
2. (B) 15. (C)
3. (C) 16. (A)
4. (C) 17. (A)
5. (A) 18. (B)
6. (D) 19. (B)
7. (D) 20. (D)
8. (A) 21. (C)
9. (D) 22. (D)
10. (D) 23. (A)
11. (C) 24. (A)
12. (B) 25. (C)
13. (A)

Multiple Correct Choice Type Questions


1. (B), (C) 9. (A), (B), (C), (D)
2. (B), (D) 10. (B), (D)
3. (B), (C) 11. (A), (B), (C)
4. (A), (B) 12. (A), (B), (C), (D)
5. (B), (D) 13. (A), (B)
6. (A), (B), (C) 14. (A), (B), (C), (D)
7. (A), (B), (C), (D) 15. (A), (D)
8. (A), (B), (C), (D)

Matrix-Match Type Questions


1. (A) ® (p), (B) ® (p), (C) ® (r), (D) ® (r) 4. (A) ® (r), (B) ® (r), (C) ® (q), (D) ® (p)
2. (A) ® (p), (B) ® (q), (C) ® (p), (D) ® (q) 5. (A) ® (q), (r) , (s) (B) ® (s), (C) ® (p), (D) ® (q),(s)
3. (A) ® (q), (B) ® (s), (C) ® (p), (D) ® (r)

Comprehension-Type Question
1. (i) (B); (ii) (A); (iii) (C) 3. (i) (A); (ii) (B); (iii) (A)
2. (i) (B); (ii) (A); (iii) (C) 4. (i) (D); (ii) (C); (iii) (D)

Assertion–Reasoning Type Questions


1. (A) 4. (C)
2. (A) 5. (A)
3. (D)

Integer Answer Type Questions


1. 2 4. 16
2. 3 5. 0
3. 6
www.jeeneetbooks.in
www.jeeneetbooks.in

BOOK FEEDBACK FORM

WE WOULD LIKE TO HEAR FROM YOU

Please complete this form. Your feedback concerning "Mathematics for IIT-JEE" will be appreci-
ated. If you do want to complete and submit the form online, you may visit us at www.wileyindia.com/
murti. You can also fill-in the form and send it as an attachment via E-mail or Fax it to +91-11-23275895.
Our e-mail address is acadmktg@wiley.com and mailing address is Wiley India Pvt. Ltd, Attn:
Marketing Department, 4436/7, Ansari Road, Daryaganj, New Delhi- 110002, India.

* Fields marked are compulsory

* Name

* Email Address

Address

Phone Mobile
School/Institute/College
Cut from here

* Name of Course Instructor

* Competitive Exam(s) Studying for

How did you hear about this book? Bookseller Classmate Teacher Advertisement
Other
(please mention)

Would you be interested in further readings by the authors? Yes No

Would you be interested in our other bestsellers from the Wiley MAESTRO series? Yes No

Your Comments

May we use your comments in future promotions? Yes No

Send us the form & get a chance to enter




Wiley India’s Monthly Student raffle! Win exciting Prizes!


Visit www.wileyindiablog.blogspot.com or www.wileyindia.com
to check the list of monthly winners.
www.jeeneetbooks.in
www.jeeneetbooks.in

Contents
1 Sets, Relations and Functions 1
1.1 Sets: Definition and Examples ........................................................................................................................ 2
1.2 Set Operations ............................................................................................................................................... 5
1.3 Venn Diagrams ............................................................................................................................................. 13
1.4 Relations ....................................................................................................................................................... 25
1.5 Equivalence Relations and Partitions ............................................................................................................ 33
1.6 Functions ...................................................................................................................................................... 38
1.7 Graph of a Function ...................................................................................................................................... 49
1.8 Even Functions and Odd Functions ............................................................................................................. 53
Worked-Out Problems .................................................................................................................................. 58
Summary ....................................................................................................................................................... 68
Exercises ....................................................................................................................................................... 73
Answers ........................................................................................................................................................ 83

2 Exponentials and Logarithms 85


2.1 Exponential Function .....................................................................................................................................86
2.2 Logarithmic Function .................................................................................................................................... 88
2.3 Exponential Equations .................................................................................................................................. 89
2.4 Logarithmic Equations .................................................................................................................................. 90
2.5 Systems of Exponential and Logarithmic Equations .................................................................................... 91
2.6 Exponential and Logarithmic Inequalities .................................................................................................... 92
Worked-Out Problems .................................................................................................................................. 93
Summary ..................................................................................................................................................... 100
Exercises ..................................................................................................................................................... 100
Answers ...................................................................................................................................................... 104

3 Complex Numbers 105


3.1 Ordered Pairs of Real Numbers .................................................................................................................. 106
3.2 Algebraic Form a + ib ................................................................................................................................. 108
3.3 Geometric Interpretation ............................................................................................................................ 112
3.4 The Trigonometric Form ............................................................................................................................. 128
3.5 De Moivre’s Theorem ................................................................................................................................. 131
3.6 Algebraic Equations ................................................................................................................................... 136
Worked-Out Problems ................................................................................................................................ 140
Summary ..................................................................................................................................................... 157
Exercises ..................................................................................................................................................... 161
Answers ...................................................................................................................................................... 166
www.jeeneetbooks.in
xxii Contents

4 Quadratic Equations 169


4.1 Quadratic Expressions and Equations ........................................................................................................ 170
Worked-Out Problems ................................................................................................................................ 180
Summary ..................................................................................................................................................... 197
Exercises ..................................................................................................................................................... 197
Answers ...................................................................................................................................................... 204

5 Progressions, Sequences and Series 207


5.1 Sequences and Series ................................................................................................................................ 208
5.2 Arithmetic Progressions .............................................................................................................................. 211
5.3 Geometric Progressions ............................................................................................................................. 217
5.4 Harmonic Progressions and Series .............................................................................................................. 221
5.5 Some Useful Formulae ............................................................................................................................... 225
Worked-Out Problems ................................................................................................................................ 227
Summary ..................................................................................................................................................... 264
Exercises ..................................................................................................................................................... 266
Answers ...................................................................................................................................................... 274

6 Permutations and Combinations 277


6.1 Factorial Notation ....................................................................................................................................... 278
6.2 Permutations .............................................................................................................................................. 278
6.3 Combinations ............................................................................................................................................. 286
Worked-Out Problems ................................................................................................................................ 297
Summary ..................................................................................................................................................... 312
Exercises ..................................................................................................................................................... 314
Answers ...................................................................................................................................................... 319

7 Binomial Theorem 321


7.1 Binomial Theorem for Positive Integral Index ............................................................................................ 322
7.2 Binomial Theorem for Rational Index ......................................................................................................... 329
Worked-Out Problems ................................................................................................................................ 333
Summary ..................................................................................................................................................... 352
Exercises ..................................................................................................................................................... 353
Answers ...................................................................................................................................................... 357

8 Matrices, Determinants and System of Equations 359


8.1 Matrices ...................................................................................................................................................... 360
8.2 Determinants .............................................................................................................................................. 395
8.3 Solutions of Linear Equations ..................................................................................................................... 412
www.jeeneetbooks.in
Contents xxiii

Worked-Out Problems ................................................................................................................................ 418


Summary ..................................................................................................................................................... 443
Exercises ..................................................................................................................................................... 449
Answers ...................................................................................................................................................... 456

9 Partial Fractions 459


9.1 Rational Fractions ....................................................................................................................................... 460
9.2 Partial Fractions .......................................................................................................................................... 462
Worked-Out Problems ................................................................................................................................ 466
Summary ..................................................................................................................................................... 470
Exercises ..................................................................................................................................................... 471
Answers ...................................................................................................................................................... 473

Index 475
www.jeeneetbooks.in
www.jeeneetbooks.in

Sets, Relations and


Functions 1
Contents
1.1 Sets: Definition and
Examples
1.2 Set Operations
1.3 Venn Diagrams
1.4 Relations
Sets, Relations and Functions

A AÇB B 1.5 Equivalence Relations


and Partitions
AÇBÇC 1.6 Functions
AÇC BÇC 1.7 Graph of a Function
1.8 Even Functions and
Odd Functions
C
Worked-Out Problems
Summary
Exercises
Answers

f(x) = x2 Sets: Any collection of well-


defined objects.
function name input what to output
Relations: For any two sets A
and B, any subset of A ´ B is
called a relation from A to B.
Input Relationship Output Functions: A relation f from
a set A to a set B is called a
f, g, h, ...
x, t,q, ... f(x), g(q),... function from A to B if for
each a ÎA, there exists unique
Domain Range Image
b ÎB such that (a, b) Îf.
Domain Elements Range Elements
Independent Variable Dependent Variable
Argument Value of Function
www.jeeneetbooks.in
2 Chapter 1 Sets, Relations and Functions

Mankind has been using the number concept as an abstraction without expressely formulating what, in precise terms,
a number is. The first precise formulation was made by the Swiss mathematician George Cantor during the years
1874 –1897 while working on number aggregates. To start with one has to realize that the abstraction that is the number
“five”‚ say, is the commonality that exists between all sets which can be put into one-to-one correspondence with the
set of fingers on a normal human hand. In olden days a shepherd would carry a bag of pebbles just to say that he has
that many sheep with him or, equivalently, there is a one-to-one correspondence between the pebbles in the bag and
the sheep he possesses. The concept of set and the concept of one-to-one correspondence of sets were introduced
by George Cantor for the first time into the world of mathematics. For a number like five or for any finite number,
Cantor’s approach through one-to-one correspondence of sets may appear to be a triviality. But if we turn to infinite
sets, we feel the difference. First of all, what is a set? The precise mathematical definition of a set had to wait for
more than three decades after Cantor’s proposal: It is a collection of objects and several paradoxes that followed the
Cantor’s viewpoint.

1.1 | Sets: Definition and Examples


For our present discussion we can be content with what most introductory mathematics texts are content with: the
intuitive concept of a set. A set is just a well-defined collection of objects, well-defined in the sense that given any object
in the world, one can say this much: Either the object belongs to the set or it does not. It cannot happen both ways. Let
us consider a counterexample first and an example of a set later.

Counter Example

Let X be the collection of all sets A such that A is not an If X does not belong to X, then X belongs to X. Either
object in A or, A does not belong to A. We shall argue way, we get a contradiction. Therefore, we cannot decide
that X is not a set. Suppose, on the contrary, that X is a set. whether X is an object in X. Thus, X is not a well-defined
If X belongs to X, then X does not belong to X. collection of objects and hence X is not a set.

Example

A positive integer greater than one is called a prime whether it is a prime number or not. For example, consider
number if it has exactly two positive divisors, namely 1 the number 2009. We may not be able to answer whether
and itself. Let P be the collection of all prime numbers. it is a prime number or not. But this much is certain that
This is a well-defined collection of objects. For, given any either 2009 is a prime or it is not. It can never be both.
object in the world, the question whether it belongs to This is the property of being a well-defined collection.
this set or not has a unique answer. First recognize that if
the given object is other than a positive integer, one can
answer the question in the negative without any think-
ing. If the object is a positive integer, the question arises

DEF IN IT ION 1 . 1 Set Any well-defined collection of objects is called a set.

DEF IN IT ION 1 . 2 Element Let X be any set. The objects belonging to X are called elements of X, or members
of X. If x is an element X, then we say that x belongs to X and denote this by x Î X. If x does
not belong to X, then we write x Ï X.

The sets are usually denoted by capital letters of English alphabet while the elements are denoted in general by small
letters. A set is represented by listing all its elements between the brackets { } and by separating them from each
other by commas, if there are more than one element. Here are some examples of sets and the usual notations used
to denote them.
www.jeeneetbooks.in
1.1 Sets: Definition and Examples 3

QUICK LOOK 1

1. The set of all natural numbers (i.e., the set of all 5. The set of all real numbers is denoted by .
positive integers) is denoted by  or +. That is, 6. The set of all positive real numbers is denoted by +.
 = {1, 2, 3, 4, ¼}.
7. The set of all positive rational numbers is denoted
2. The set of all non-negative integers is denoted by W; by +.
that is W = {0, 1, 2, 3, ¼}.
8.  denotes the set of all complex numbers.
3.  denotes the set of all integers.
4.  denotes the set of all rational numbers.

Example 1.1

Verify whether the following are sets: Note that the collections given in (1) and (4) are not
(1) The collection of all intelligent persons in Visakha- sets because, if we select a person in Visakhapatnam,
patnam. we cannot say with certainty whether he/she belongs to
the collection or not, as there is no stand and scale for the
(2) The collection of all prime ministers of India.
evaluation of intelligence or for being tall. However, the
(3) The collection of all negative integers. collections given in (2) and (3) are sets.
(4) The collection of all tall persons in India.

A set may be represented with the help of certain property or properties possessed by all the elements of that set.
Such a property is a statement which is either true or false. Any object which does not possess this property will not be
an element of that set. In order to represent a set by this method we write between the brackets { } a variable x which
stands for each element of the set. Then we write the property (or properties) possessed by each element x of the set.
We denote this property by p(x) and seperate x and p(x) by a symbol: or |, read as “such that”. Thus, we write
{ x | p(x)} or { x : p(x)}
to represent the set of all objects x such that the statement p(x) is true. This representation of a set is called “set builder
form” representation.

Examples

(1) Let P be the collection of all prime numbers. Then it (3) Let X be the set given above in (2) and
can be represented in the set builder form as
ì 1 ü
P = { x | x is a prime number} Y = í y| y = 0 or Î X ý
î y þ
(2) Let X be the set of all even positive integers which
are less than 15. Then Then
X = { x | x is even integer and 0 < x < 15} ì 1 1 1 1 1 1 1ü
Y = í0, , , , , , , ý
= {2, 4, 6, 8, 10, 12, 14} î 2 4 6 8 10 12 14 þ

DEF IN IT ION 1 . 3 Empty Set The set having no elements belonging to it is called the empty set or null set and
is denoted by the symbol f.

Examples

(1) Let X = {x | x is an integer and 0 < x < 1} . Then X is a (2) Let X = { a | a is a rational number and a2 = 2}. Then X
set and there are no elements in X, since there is no is the empty set, since there is no rational number a for
integer x such that 0 < x < 1. Therefore, X is the which a2 = 2 .
empty set.
www.jeeneetbooks.in
4 Chapter 1 Sets, Relations and Functions

Notation: The symbol Þ is read as “implies”. Thus a Þ b is read as “a implies b”. The symbol Û is read as “implies
and is implied by” or as “if and only if”. Thus a Û b is read as “a implies and implied by b” or “a if and only if b”.

Examples

(1) x is an integer and 0 < x < 2 Û x = 1. (2) a is an integer and a2 = a Û a = 0 or a = 1.

DE FIN IT ION 1 . 4 Equal Sets Two sets A and B are defined to be equal if they contain the same elements, in the
sense that,
x Î A Û x ÎB
In this case, we write A = B. If A and B are not equal, then we denote it by A ¹ B.

Examples

(1) Let A = {1, 2, 3, 4} and B = {4, 2, 3, 1}. Then A = B. and Z = { n | n Î + and 1 £ n2 £ 16}
(2) Let Then Y = Z and X ¹ Y, since -1 ÎX and -1 ÏY. Note that
X = {-4, -3, -2, -1, 1, 2, 3, 4}.
X = {n | n Î  and 1 £ n2 £ 16}
Y = { n | n Î  and 1 £ n £ 4}

DE FIN IT ION 1 . 5 Finite and Infinite Sets A set having a definite number of elements is called a finite set. A set
which is not finite is called an infinite set.

Examples

(1) The set + of positive integers is an infinite set. (3) The set  of real numbers is an infinite set.
(2) {a, b, c, d} is a finite set, since it has exactly four (4) { x | x Î  and 0 < x £ 100} is a finite set.
elements. (5) { x | x Î  and 0 < x < 1} is an infinite set.

DE FIN IT ION 1 . 6 Family of Sets A set whose members are sets is called a family of sets or class of sets.

Note that a family of sets is also a set. Usually families of sets are denoted by script letters Ꮽ, Ꮾ, Ꮿ, Ᏸ, etc.

Examples

(1) For any integer n, let An = { x | x is an integer and Xh = The set of persons belonging to the house h
x ³ n}. Then { An | n is an integer} is a family of sets.
Then {Xh | h is a house in Visakhapatnam} is a family of
(2) For any house h, let sets.

DE FIN IT ION 1 . 7 Indexed Family of Sets A family Ꮿ of sets is called an indexed family if there exists a set I
such that for each element i Î I, there exists a unique member Ai in Ꮿ associated with i and
Ꮿ = {Ai : i Î I}. In this case, the set I is called the index set.

For example, the family of sets + of positive integers is an indexed family of sets, the index set being , the set of
integers. In the example Xh = The set of persons belonging to the house h where {Xh | h is a house in Visakhapatnam}
also we have an indexed family of sets, where the index set is the set of houses in Visakhapatnam. If Ꮽ is an indexed
family of sets with the index set I, then we usually write
Ꮽ = {Ai}i ÎI or {Ai | i Î I }
www.jeeneetbooks.in
1.2 Set Operations 5

DEF IN IT ION 1 . 8 Intervals in  For any real numbers a and b, we define the intervals as the sets given below:
1. (a, b) = { x | x Î  and a < x < b}
2. (a, b] = { x | x Î  and a < x £ b}
3. [a, b) = { x | x Î  and a £ x < b}
4. [a, b] = { x | x Î  and a £ x £ b}

Examples

(1) [2, 4] = {x | x Î  and 2 < x < 4} (2) [0, 1] = { x | x Î and 0 £ x £ 1}

Note that, for any two real numbers a and b, the intervals [a, b] or [a, b) or (a, b] is empty if and only if a ³ b. Also
(a, b) is empty if and only if a > b. Further [a, b] has exactly one element if and only if a = b. Thus these intervals
become non-trivial only if a < b. Usually (a, b) is called an open interval, (a, b] is called left open and right closed inter-
val, [a, b) is called the left closed and right open interval and [a, b] is called a closed interval.

1.2 | Set Operations


We define certain operations between sets. These are closely related to the logical connectives “and”, “or” and “not”.
To begin with, we have the following.

DEF IN IT ION 1 . 9 Subset For any two sets A and B, we say that A is a subset of B or A is contained in B if every
element of A is an element of B; in this case we denote it by A Í B. A is not a subset of B is
denoted by A Í/ B .

If A Í B we also say that B is a super set of A or B contains A or B is larger than A or A is smaller than B. Sometimes,
we write B Ê A instead of A Í B. If A is a subset of B and A ¹ B, then we say that A is a proper subset of B and denote
this by A Ì B. Note that, for any sets A and B, A = B if and only if A Í B and B Í A.

QUICK LOOK 2

1. The set + of positive integers is a proper subset of 4.  is a proper subset of the set  of complex numbers.
the set  of integers. 5. The set of Indians is a subset of the set of human beings.
2.  is a proper subset of the set  of rational numbers. 6. If A = {1, 2, 3, 4, 5} and B = { x | x Î  and x2 − 5x +
3.  is a proper subset of the set  of real numbers. 6 = 0}, then B Ì A.

DEF IN IT ION 1 . 10 Power Set For any set X, the collection of all subsets of X is also a set and is called the
power set of X. It is denoted by P(X ).

Note that the empty set f and the set X are always elements in the power set P(X ). Also, X = f if and only if P(X ) has
only one element. Infact, X has exactly n elements if and only if P(X ) has exactly 2n elements, as proved in Theorem 1.1.
First, let us consider certain examples.

Examples

(1) If X = {a}, then P( X ) = {f, X } (4) If X = {1, 2, 3, 4, 5}, then P(X ) has 32 (= 25 ) elements
(2) If X = {a, b}, then P( X ) = {f, {a}, {b}, X } (5) If X is a set such that P(X ) has 128 elements then X
(3) If X = {1, 2, 3}, then has 7 elements, since 27 = 128

P( X ) = {f, {1}, {2}, {3}, {1, 2}, {2, 3}, {3, 1}, X }
www.jeeneetbooks.in
6 Chapter 1 Sets, Relations and Functions

DEF IN IT ION 1 . 11 Cardinality If X is a finite set, then the number of elements in X is denoted by | X | or n(X )
and this number is called the cardinality of X.

T H E O R E M 1 .1 Let X be any set. Then X is a finite set with n elements if and only if the power set P(X ) is a finite
set with 2n elements.
PROOF Suppose that X is a finite set with n elements. We apply induction on n. If n = 0, then X = f and
P(X ) = {f} which is a set with 1 (= 20) element. Now, let n > 0 and assume that the result is true
for all sets with n - 1 elements; that is, if Y is a set with n - 1 elements, then P(Y ) has exactly 2n-1
elements.
Since n > 0, X is a non-empty set and hence we can choose an element a in X. Let Y be the set of
all elements in X other than a. Then | Y | = n - 1 and therefore | P(Y )| = 2n - 1. Clearly P(Y ) Í P( X ).
Also, if A Î P(X ) and A Ï P(Y ), then A Í X and A Ë Y and hence a Î A. Therefore, the number of
subsets of X which are not subsets of Y is equal to the number of subsets of X containing a which
in turn coincides with |P(Y)|. Hence,
| P( X )| = | P(Y )| + | P(Y )| = 2n - 1 + 2n - 1 = 2n
Converse is clear; since each element x Î X produces an element { x } Î P(X ), therefore X must
be finite if P(X ) is finite. Also, note that, for non-negative integers n and m, 2n = 2m if and only if
n = m. ■

C O R O L L A RY 1.1 For any finite set X, | X | < | P(X ) |.

DE FIN IT ION 1 . 12 Intersection of Sets For any two sets A and B, we define the intersection of A and B to be
the set of all elements belonging to both A and B. It is denoted by A Ç B. That is,
A Ç B = { x | x Î A and x Î B}

Example 1.2

Let A = { x | x is an odd prime and x < 20} and B = { x | x is A = {3, 5, 7, 11, 13, 17, 19} and B = {7, 8, 9, 10, 11, 12, ...}
an integer and x > 6}. Find A Ç B.
Therefore
Solution: By hypothesis A Ç B = {7, 11, 13, 17, 19}

Example 1.3

Let X = The set of all circles in the plane whose radii is Solution: X Ç Y = f, the empty set, since no circle of
5 cm and Y = The set of all line segments of length 5 cm positive radius can be a line segment.
in the plane. Find X Ç Y.

Example 1.4

Let F = The set of all boys in a school who can play Solution: F Ç C = The set of all boys in the school who
football and C = The set of all boys in the school who can can play both football and cricket.
play cricket. Find F Ç C.

Example 1.5

Let A = The set of all non-negative integers and B = The Solution: A Ç B = {x | x is an integer, x ≥ 0 and x ≤ 0} = {0}.
set of all non-positive integers. Find A Ç B.
www.jeeneetbooks.in
1.2 Set Operations 7

The following can be proved easily.

Try it out
T H E O R E M 1 .2 The following hold for any sets, A, B and C.
1. A Í B Û A = A Ç B
2. A Ç A = A
3. A Ç B = B Ç A
4. (A Ç B) Ç C = A Ç (B Ç C)
5. A Ç f = f, where f is the empty set.
6. For any set X, X Í A Ç B if and only if X Í A and X Í B.

In view of (4) above, we write simply A Ç B Ç C for (A Ç B) Ç C or A Ç (B Ç C). In general, if A1, A2, ¼, An are sets,
we write
n

∩A
i =1
i for A1 Ç A2 Ç Ç An

More generally, for any indexed family {Ai}i ÎI of sets, we write ∩A i for the set of all elements common to all Ai’s,
i Î I and express this by i ÎI

∩ A = { x | x Î A for all i Î I }
i ÎI
i i

DEF IN IT ION 1 . 13 Disjoint Sets Two sets A and B are called disjoint if A Ç B is the empty set. In this case we
say that A is disjoint with B or B is disjoint with A.

Examples

(1) Let E be the set of even integers and O the set of all æ


¥
odd integers. Then E and O are disjoint sets. (4) ÷ =f
çè 0,
n=1

(2) Let A = { p | p is a prime number}. Then A Ç  = f, since, for any given a > 0, we can find an integer n
where  is the set of rational numbers, since it is such that 0 < 1/n < a and hence a Ï(0, 1/n).
known that p is an irrational number for any prime p.

é 1ù

¥
(3) n=1 êë0, n úû = {0}

DEF IN IT ION 1 . 14 Union of Sets For any two sets A and B, we define the union of A and B as the set of all
elements belonging to A or B and denote this by A È B; that is,
A È B = {x | x Î A or x Î B}
Note that the statement “x Î A or x Î B” does not exclude the case “x Î A and x Î B”.
Therefore
A È B = {x | x Î A or x Î B or both}

Example

Let E be the set of even integers and O the set of all odd E and O are disjoint and hence we do not come across
integers. Then E È O = , the set of integers. In this case, the case “x Î E and x ÎO ”.
www.jeeneetbooks.in
8 Chapter 1 Sets, Relations and Functions

Example 1.6

Let A be the interval [0, 1] and B the interval [1/2, 2]. = {x | x Î  and 0 £ x £ 2}
Then find A È B and A Ç B.
= [0, 2]
Solution: We have Also,
A È B = {x | x Î A or x Î B} é1 ù
A Ç B = ê , 1ú
ì 1 ü ë2 û
= í x | x Î  and ‘0 £ x £ 1 or £ x £ 2 ’ý
î 2 þ

Example 1.7

Let A = [0, 1] Ç  and B = (1, 2) Ç . Find A È B . = { x | x Î  and 0 £ x < 2}


= [0, 2) Ç 
Solution: A È B = { x | x Î A or x Î B}
= { x | x Î  and ‘x Î[0, 1] or x Î(1, 2)’}

Example 1.8
Let A be the set of all even primes and B the interval = {x | x Î  and 2 £ x < 3}
(2, 3). Find A È B.
= [2, 3)
Solution: A È B = {x | x is an even prime or x Î(2, 3)}
= {x | x = 2 or x Î  such that 2 < x < 3}

The following can be easily proved.

Try it out
T H E O R E M 1 .3 For any sets A, B and C the following hold.
1. A Ç B Í A È B
2. For any set X, A È B Í X if and only if A Í X and B Í X
3. A È A = A
4. A È B = B È A
5. (A È B) È C = A È (B È C)
6. A Í B Û A È B = B
7. A È f = A
8. A = A Ç B Û A Í B Û A È B = B
9. A Ç (A È B) = A
10. A È (A Ç B) = A

T H E O R E M 1 .4 The following hold for any sets A, B and C.


DISTRIBUTIVE 1. A Ç (B È C) = (A Ç B) È (A Ç C)
LAWS
2. A È (B Ç C) = (A È B) Ç (A È C)
These are called the distributive laws for intersection Ç and union È.
www.jeeneetbooks.in
1.2 Set Operations 9

PROOF 1. x Î A Ç (B È C) Þ x Î A and x ÎB È C
Þ x Î A and ( x Î B or x ÎC )
Þ ( x Î A and x Î B) or ( x Î A and x ÎC )
Þ x Î A Ç B or x ÎA Ç C
Þ x Î( A Ç B) È ( A Ç C )
Therefore
A Ç ( B È C ) Í ( A Ç B) È ( A Ç C ) (1.1)
On the other hand, we have
x Î( A Ç B) È ( A Ç C ) Þ x Î A Ç B or x Î A ÇC
Þ ( x Î A and x Î B) or ( x Î A and x ÎC )
Þ x Î A and (x Î B or x ÎC )
Þ x Î A and x Î B ÈC
Þ x Î A Ç (B È C )
Therefore
( A Ç B) È ( A È C ) Í A Ç ( B È C ) (1.2)
From Eqs. (1.1) and (1.2), we have A Ç ( B È C ) = ( A Ç B) È ( A Ç C ).
2. It can be proved similarly and is left as an exercise for the reader. ■

Try it out A È (B Ç C) = (A È B) Ç (A È C)

T H E O R E M 1 .5 For any sets A, B and C,

A Ç B = A Ç C and AÈ B = A ÈC imply B=C


PROOF Suppose that A Ç B = A Ç C and A È B = A È C . Consider
B = B Ç ( A È B) [by part (9) of Theorem 1.3]
= B Ç (A ÈC) (since A È B = A È C)
= ( B Ç A) È ( B Ç C ) (by the distributive laws)
= (C Ç A) È (C Ç B) (since A Ç B = A Ç C)
= C Ç ( A È B) (by the distributive laws)
= C Ç (A ÈC) (since A È B = A È C)
=C [by part (9) of Theorem 1.3]
Therefore B = C.
Since (A È B) È C = A È (B È C) for any sets A, B and C, we simply write A È B È C without
bothering about the brackets. In general, if A1, A2, …, An are any sets, then we write
n

∪A
i =1
i for A1 È A2 È È An

For any indexed family { Ai }iÎI of sets, we write ∪ i ÎI


Ai for the set of all elements belonging to at
least one Ai and express this by

∪ A = { x | x Î A for some i Î I }
i i
i ÎI ■
www.jeeneetbooks.in
10 Chapter 1 Sets, Relations and Functions

Examples

(1) For any positive integer n, let (3) For any positive real number a, let
An = (- n, n) = {x | x Î  and - n < x < n} Aa = The set of human beings on the Earth whose
height is less than or equal to a cm
Then
Then
¥

∪A
n=1
n
+
= {x | x Î  and -n < x < n for some n Î  } =  ∪ A = The set of all human beings on the Earth
a
aÎ+

since, for any real number x, there exists a positive (4) For any positive integer n, let
integer n such that | x | < n and hence -n < x < n, so
that x Î An. æ 1 1ö ì 1 1ü
Xn = ç - , ÷ = í x | x Î  and - < x < ý
è n nø î n nþ
(2) For any positive integer n, let
Pn = { p | p is a prime number and p < n} Then
¥ ¥
Note that P1 = f = P2, P3 = {2} and P4 = {2, 3}. Now
¥ ∩ X = {0}
n and ∪X n = (-1, 1)
∪ P = The set of all prime numbers
n=1 n=1
n
n=1
since Xn Í X1 for all n Î +.
since, for any prime p, we have p Î Ap +1 .

DEF IN IT ION 1 . 15 For any two sets A and B, the difference of A and B is defined as the set
A - B = { x | x Î A and x Ï B}

Example 1.9

Find the difference of the following sets. Now


(1) A = (0, 1) = {x | x Î  and 0 < x < 1} and A - B = {x | x Î A and x Ï B}
ì 1 ü
B = í x | x Î + and Î  ý ì 1 ü
î x þ = í x | x Î , 0 < x < 1 and Ï  ý
(2)  -  where the symbols have there usual meaning. î x þ
(3) A = The set of all students in a school and B = The ¥
æ 1 1ö
set of all girls = ∪ç , ÷
n=1 è n + 1 n ø
(4)  - + where the symbols have the usual meaning.
(2)  -  = {x | x Î  and x Ï }
Solution: = {x | x is a real number and not an integer}
(1) By hypothesis
A = (0, 1) = {x | x Î  and 0 < x < 1} and B = {x | x Î+
= ∪ (n, n + 1)
n Î
and 1/x Î}. We have
= È (-2, - 1) È (-1, 0) È (0, 1) È (1, 2) È
ì 1 1 1 ü (3) A - B = The set of all boys in the school
B = í1, , , , ý
î 2 3 4 þ (4)  - + = The set of all non-positive integers
= {x | x Î  and x £ 0}

T H E O R E M 1 .6 For any sets A, B and C, the following hold:


DE MORGAN'S 1. A - (B È C) = (A - B) Ç (A - C)
LAWS
2. A - (B Ç C) = (A - B) È (A - C)

PROOF 1. x Î A - ( B È C ) Þ x Î A and x ÏB È C
Þ x Î A and ( x Ï B and x ÏC )
www.jeeneetbooks.in
1.2 Set Operations 11

Þ ( x Î A and x Ï B) and ( x Î A and x ÏC )


Þ x Î A - B and x ÎA - C
Þ x Î( A - B) Ç ( A - C )
and therefore, A - (B È C) Í (A - B) Ç (A - C). Also,
x Î( A - B) Ç ( A - C ) Þ x Î A - B and x ÎA - C
Þ ( x Î A and x Ï B) and ( x Î A and x ÏC )
Þ x Î A and ( x Ï B and x ÏC )
Þ x Î A and x ÏB È C
Þ x Î A - (B È C )

and therefore (A - B) Ç (A - C) Í A - (B È C). Thus


A - (B È C) = (A - B) Ç (A - C)
2. It can be similarly proved and is left as an exercise for the reader. ■

Try it out
T H E O R E M 1 .7 The following hold for any sets A, B and C.
1. B Í C Þ A - C Í A - B
2. A Í B Þ A - C Í B - C
3. (A È B) - C = (A - C) È (B - C)
4. (A Ç B) - C = (A - C) Ç (B - C)
5. (A - B) - C = A - (B È C) = (A - B) Ç (A - C)
6. A - (B - C) = (A - B) È (A Ç C)

T H E O R E M 1 .8 Let { Ai }i ÎI be any family of sets and B and C any sets. Then the following hold:
GENERALIZED
æ ö
DE MORGAN'S 1. B - ç ∪ Ai ÷ = ∩ ( B - Ai )
LAWS è iÎI ø iÎI
æ ö
2. B - ç ∩ Ai ÷ = ∪ ( B - Ai )
è iÎI ø iÎI
æ ö
3. ç ∪ Ai ÷ - B = ∪ ( Ai - B)
è iÎI ø iÎI

æ ö
4. ç ∩ Ai ÷ - B = ∩ ( Ai - B)
è i ÎI ø i ÎI

PROOF These follow from the facts that

x Î ∪ Ai Û x Î Ai for some i Î I
i ÎI

x Î ∩ Ai Û x Î Ai for all i Î I
i ÎI

x Ï∪ Ai Û x Ï Ai for all i Î I
i ÎI

and x Ï∩ Ai Û x Ï Ai for some i Î I


i ÎI ■
www.jeeneetbooks.in
12 Chapter 1 Sets, Relations and Functions

Examples
æ ö
(1)  -  =  - ç ∪ {n}÷ (3) For any integer n,
è nÎ ø
 - (n, n + 1) = (-¥, n] È [n + 1, ¥)
= ∩ ( - {n})
n Î
Here (-¥, n) stands for the set of real numbers x
such that x £ n and [n + 1, ¥) for the set of real num-
æ ö
(2)  -  = ç ∪ [n, n + 1]÷ -  bers x such that n + 1 £ x .
è nÎ ø
(4) Let
= ∪ ([n, n + 1] - )
n Î
A = {x Î + | x < 10} = {1, 2, 3, 4, 5, 6, 7, 8, 9}
and B = The set of all prime numbers
= ∪ (n, n + 1)
n Î
Then
Note that, here we have used the fact that, for any A - B = {1, 4, 6, 8, 9}
integer n, there is no integer m such that n < m < n + 1.

It is convenient to write B ¢ for the set of all elements not belonging to B and to write A - B as A Ç B ¢. But the
problem here is that B ¢ may not be a set at all. However, if X is a superset of B, then certainly X - B is a set, which can
be imagined as B ¢. For any two sets A and B, we can take X = A È B and then
A - B = A Ç ( X - B) = A Ç B ¢
When we are dealing with a family { Ai }i ÎI of sets (or set of sets), we can assume that each Ai is a subset of some set X;
for example, we can take X = ∪ i ÎI Ai . This common superset is called a universal set. Therefore, when we discuss about
difference set A - B, we can treat A and B as subsets of a universal set X and treat A - B as A Ç B ¢, where
B ¢ = {x | x Î X and x Ï B}
B ¢ is certainly a set, since X and B are sets and so is X - B . This B ¢ is called the complement of B in X or, simply, the comple-
ment of B, when there is no ambiguity about X. Note that A - B = A - ( A Ç B) and A Ç B is a subset of A. Therefore, we
can call A - B is the complement of B in A. With this understanding, the properties proved above can be restated as follows:
A - B = A Ç B¢
A - B = A - ( A Ç B)
( B È C )¢ = B ¢ Ç C ¢ [Part (1), Theorem 1.6]
( B Ç C )¢ = B ¢ È C ¢ [Part (2), Theorem 1.6]
B Í C Þ C ¢ Í B¢ [Part (1), Theorem 1.7]
æ ö¢
çè ∪ Ai ÷ø = ∩ Ai¢ [Part (1), Theorem 1.8]
iÎI iÎI

æ ö¢
çè ∩ Ai ÷ø = ∪ Ai¢ [Part (2), Theorem 1.8]
iÎI iÎI

A - ( A - B) = A Ç B
B Í A Þ A - ( A - B) = B or ( B ¢)¢ = B
A Ç A¢ = f
A È A¢ = X, the universal set

DEF IN IT ION 1 . 16 Symmetric Difference For any sets A and B, the symmetric difference of A and B is defined
as the set
A D B = ( A - B) È ( B - A) = ( A Ç B ¢) È ( B Ç A¢)
That is, A D B is the set all elements belonging to exactly one of A and B.
www.jeeneetbooks.in
1.3 Venn Diagrams 13

Example 1.10

Find the symmetric difference of the following: Therefore


(1) A = {1, 2, 3, 4} and B = {4, 5, 6} A D B = {1, 2, 3} È {5, 6} = {1, 2, 3, 5, 6}
(2) A = {a, b, c, d, e} and B = {b, c, f, g}
(2) From the given sets we have
Solution: A – B = {a, d, e} and B – A = { f, g}
(1) We have A = {1, 2, 3, 4} and B = {4, 5, 6}. Then
Therefore
A - B = {1, 2, 3} and B - A = {5, 6} A D B = {a, d, e} È { f, g} = {a, d, e, f, g}

T H E O R E M 1 .9 The following hold for any sets A, B and C.


1. A D B = B D A
2. ( A D B) D C = A D ( B D C )
3. A D f = A
4. A D A = f

PROOF 1. A D B = ( A - B) È ( B - A)
= ( B - A) È ( A - B)
= BD A
2. ( A D B) D C = [( A D B) Ç C ¢] È [C Ç ( A D B)¢ ]

= [{( A Ç B¢) È ( B Ç A¢)} Ç C ¢] È [C Ç {( A Ç B¢) È ( B Ç A¢)}¢ ]


= [( A Ç B¢ Ç C ¢) È ( B Ç A¢ Ç C ¢)] È [C Ç ( A¢ È B) Ç ( B¢ È A)]
= ( A Ç B¢ Ç C ¢) È ( B Ç A¢ Ç C ¢) È [C Ç {( A¢ Ç B¢) È ( A¢ Ç A) È ( B Ç B¢) È ( B Ç A)}]
= ( A Ç B¢ Ç C ¢) È ( B Ç A¢ Ç C ¢) È [{C Ç ( A¢ Ç B¢) È ( A Ç B)}]
= ( A Ç B¢ Ç C ¢) È ( B Ç A¢ Ç C ¢) È (C Ç A¢ Ç B¢) È (C Ç A Ç B)
Therefore, we have
( A D B) D C = ( A Ç B ¢ Ç C ¢) È ( A¢ Ç B Ç C ¢) È ( A¢ Ç B ¢ Ç C ) È ( A Ç B Ç C )
This is symmetric in A, B and C; that is, if we take B, C and A for A, B and C, respectively, the
resultant is same. Therefore,
( A D B) D C = ( B D C ) D A = A D ( B D C )
3. A D f = ( A - f ) È (f - A) = A È f = A
4. A D A = ( A - A) È ( A - A) = f È f = f ■

1.3 | Venn Diagrams


A set is represented by a closed curve, usually a circle, and its elements by points within it. This facilitates better
understanding and a good insight. A statement involving sets can be easily understood with pictorial representation of
the sets. The diagram showing these sets is called the Venn diagram of that statement, named after the British logician
John Venn (1834 –1883).
Usually the universal set is represented by a rectangle and the given sets are represented by circles or closed
geometrical figures inside the rectangle representing the universal set. An element of set A is represented by a point
within the circle representing A.
www.jeeneetbooks.in
14 Chapter 1 Sets, Relations and Functions

In Figure 1.1, the rectangle represents the universal set S, A and B represent two disjoint sets contained in S and
a and b represent arbitrary elements in A and B, respectively.

a b

A B

FIGURE 1.1 A Venn diagram.

In Figure 1.2, two intersecting sets A and B are represented by the intersecting circles, indicating that the common
area of the circles represents the intersection A Ç B . Figure 1.3 represents the statement “A is a subset of B”.
The shaded parts in Figures 1.4 –1.6 represent the union of two sets A and B, namely A È B in the cases
A Ç B = f , A Ç B ¹ f and A Í B , respectively. Figures 1.7–1.9 represent the intersection A Ç B in these cases.

A B

FIGURE 1.2 Two intersecting sets A and B.

S
B

FIGURE 1.3 Representation of “A is a subset of B”.

A B

FIGURE 1.4 Representation of A È B when A Ç B = f.


www.jeeneetbooks.in
1.3 Venn Diagrams 15

A B

FIGURE 1.5 Representation of A È B when A Ç B ¹ f.

S
B

FIGURE 1.6 Representation of A È B when A Í B.

A B

FIGURE 1.7 Representation of A Ç B when A Ç B = f.

A B

FIGURE 1.8 Representation of A Ç B when A Ç B ¹ f.

FIGURE 1.9 Representation of A Ç B when A Í B.


www.jeeneetbooks.in
16 Chapter 1 Sets, Relations and Functions

A
B

FIGURE 1.10 Representation of A - B when B Í A.

S
B

FIGURE 1.11 Representation of A - B when A Í B. In this case A – B = f.

A B

FIGURE 1.12 Representation of A - B when A Ç B = f.

A B

FIGURE 1.13 Representation of A - B when A Ë B and B Ë A.

The shaded parts in Figures 1.10 –1.13 represent the difference A - B in various cases. The symmetric differences
A D B [= (A - B) È (B - A)] are represented by the shaded parts in the Figures 1.14 –1.17 in these cases.

A
B

FIGURE 1.14 Representation of A D B when B Í A.


www.jeeneetbooks.in
1.3 Venn Diagrams 17

B
A

FIGURE 1.15 Representation of A D B when A Í B.

A B

FIGURE 1.16 Representation of A D B when A Ç B = f.

A B

FIGURE 1.17 Representation of A D B when A Ë B and B Ë A.

Figure 1.18 represents the complement of a set A in a universal set S. Figures 1.19 –1.21 illustrate the cases A D B,
(A D B) - C and C - (A D B), respectively. (A D B) D C is represented by Figure 1.22. From this one can easily see that
(A D B) D C = (A D B) D C.

A A

FIGURE 1.18 Complement of a set A.


www.jeeneetbooks.in
18 Chapter 1 Sets, Relations and Functions

S
A B

FIGURE 1.19 Representation of A D B.

S
A B

FIGURE 1.20 Representation of (A D B) - C.

S
A B

FIGURE 1.21 Representation of C - (A D B).

S
A B

FIGURE 1.22 Representation of (A D B) D C.

Figures 1.23 and 1.24 represent the property


A - ( B È C ) = ( A - B) Ç ( A - C )
www.jeeneetbooks.in
1.3 Venn Diagrams 19

S
A B

FIGURE 1.23 Representation of A - ( B È C ).

S
A B

FIGURE 1.24 Representation of ( A - B) Ç ( A - C ).

In the following, we derive certain formulas for the number of elements in the intersection, union, difference and
symmetric difference of two given finite sets. First, recall that, for any finite set A, n(A) or |A| denotes the number of
elements in A.

Examples

(1) Let A = {a, b, c, d}, then n(A) = 4. (4) If X = {m | m Î Z and m2 = 1}, then n(X) = 2, since
(2) If A = {2, 3, 5, 7}, then n(A) = 4. X = {1, -1}.
(3) If X is a finite set and n(X ) = m, then n[ P( X )] = 2m,
where P(X) is the set of all subsets of X.

T H E O R E M 1 .10 For any two disjoint sets A and B,


n( A È B) = n( A) + n( B)
PROOF Any element of A È B is in exactly one of A and B and therefore n( A È B) = n( A) + n( B)
In Figure 1.25, the shaded part represents A È B when A and B are disjoint sets.

A B

FIGURE 1.25 Representation of A È B when A and B are disjoint sets. ■


www.jeeneetbooks.in
20 Chapter 1 Sets, Relations and Functions

C O R O L L A RY 1.2 If A1 , A2 , … , An are pairwise disjoint sets, then

n ( A1 È A2 È È An ) = n ( A1 ) + n ( A2 ) + + n ( An )

T H E O R E M 1 .11 For any finite sets A and B,

n ( A È B) = n ( A) + n ( B) - n ( A Ç B)

PROOF Let A and B be finite sets, n ( A) = a, n ( B) = b and n ( A Ç B) = m. If A Ç B is empty then m = 0


and, by Theorem 1.10,
n ( A È B) = n ( A) + n ( B) = n ( A) + n ( B) - n ( A Ç B) ■

Suppose that A Ç B ¹ f . Then A - B, B - A and A Ç B are pairwise disjoint sets (Figure 1.26) and hence we have

n ( A È B) = n [( A - B) È ( B - A) È ( A Ç B)] = n ( A - B) + n ( B - A) + n ( A Ç B)
= n ( A) + n ( B) - n( A Ç B)

since n ( A) = n ( A - B) + n ( A Ç B) and n ( B) = n ( B - A) + n ( A Ç B).


We have earlier proved that n ( A È B) = n ( A) + n ( B), if A and B are disjoint sets. The converse of this is also true.

A B

FIGURE 1.26 Representation of pairwise disjoint sets.

C O R O L L A RY 1.3 If A and B are finite sets such that n ( A È B) = n ( A) + n ( B), then A and B are disjoint.

PROOF If n ( A È B) = n ( A) + n ( B) , then by Theorem 1.11 n( A Ç B) = 0 and hence A Ç B = f . ■

C O R O L L A RY 1.4 For any finite sets A and B,


n ( A - B) = n ( A) - n ( A Ç B)

C O R O L L A RY 1.5 If A is a subset of a finite set B, then

n ( B) = n ( A) + n ( B - A)

T H E O R E M 1 .12 For any finite sets A, B and C,


n ( A È B È C ) = n( A) + n( B) + n(C ) - n( A Ç B) - n( B Ç C ) - n(C Ç A) + n ( A Ç B Ç C )
PROOF Let A, B and C be any finite sets. Then
n( A È B È C ) = n( A È B) + n(C ) - n[( A È B) Ç C ]
= n( A) + n( B) - n( A Ç B) + n(C ) - n[( A Ç C ) È ( B Ç C )]
www.jeeneetbooks.in
1.3 Venn Diagrams 21

= n( A) + n( B) + n(C ) - n( A Ç B) - [n( A Ç C ) + n( B Ç C ) - n( A Ç C Ç B Ç C )]
= n( A) + n( B)) + n(C ) - n( A Ç B) - n( B Ç C ) - n(C Ç A) + n( A Ç B Ç C )

S
A B

T H E O R E M 1 .13 Let A, B and C be finite sets. Then the number of the elements belonging to exactly two of the sets
A, B and C is
n( A Ç B) + n( B Ç C ) + n(C Ç A) - 3n( A Ç B Ç C )
PROOF The required number is
n[( A Ç B) - C ] + n[( B Ç C ) - A] + n[(C Ç A) - B] = [n( A Ç B) - n( A Ç B Ç C )]
+ [n( B Ç C ) - n( B Ç C Ç A)] + [n(C Ç A) - n(C Ç A Ç B)]
= n( A Ç B) + n( B Ç C ) + n(C Ç A) - 3n( A Ç B Ç C ) ■

T H E O R E M 1 .14 Let A, B and C be any finite sets. Then the number of elements belonging to exactly one of the
sets A, B and C is
n( A) + n( B) + n(C ) - 2 n( A Ç B) - 2 n( B Ç C ) - 2 n(C Ç A) + 3n( A Ç B Ç C )
PROOF The number of elements belonging only to A is
n[ A - ( B È C )] = n( A) - n[ A Ç ( B È C )]
= n( A) - n[( A Ç B) È ( A Ç C )]
= n( A) - [n( A Ç B) + n( A Ç C ) - n( A Ç B Ç A Ç C )]
= n( A) - n( A Ç B) - n( A Ç C ) + n( A Ç B Ç C )
Similarly, the number of elements belonging only to B is
n( B) - n( B Ç C ) - n( B Ç A) + n( A Ç B Ç C )
Also, the number of the elements belonging only to C is
n(C ) - n(C Ç A) - n(C Ç B) + n( A Ç B Ç C )
Thus the number of elements belonging to exactly one of the sets A, B and C is
[n( A) - n( A Ç B) - n( A Ç C ) + n( A Ç B Ç C )] + [n( B) - n( B Ç C ) - n( B Ç A) + n( A Ç B Ç C )]
+ [n(C ) - n(C Ç A) - n(C Ç B) + n( A Ç B Ç C )]
= n( A) + n( B) + n(C ) - 2 n( A Ç B) - 2n( B Ç C ) - 2 n(C Ç A) + 3n( A Ç B Ç C ) ■
www.jeeneetbooks.in
22 Chapter 1 Sets, Relations and Functions

QUICK LOOK 3

Summary of the formulas 6. The number of elements belonging to exactly one of


Let A, B and C be given finite sets and S a universal A, B and C is
finite set containing A, B and C. Then the following
n( A) + n( B) + n(C ) - 2 n( A Ç B) - 2 n( B Ç C )
hold:
- 2 n(C Ç A) + 3n( A Ç B Ç C )
1. n(A È B) + n(A Ç B) = n(A) + n(B)
2. n(A È B) = n(A - B) + n(B - A) + n(A Ç B) 7. The number of elements belonging to exactly two of
3. n(A È B) = n(A) + n(B) Û A Ç B = f A, B and C is
4. n(A) = n(A - B) + n(A Ç B) n(A Ç B) + n(B Ç C) + n(C Ç A) - 3n(A Ç B Ç C)
5. The number of the elements belonging to exactly 8. n(A¢ È B¢) = n(S) - n(A Ç B)
one of A and B is
9. n(A¢ Ç B¢) = n(S) - n(A È B)
n( A D B) = n( A - B) + n( B - A)
= n( A) + n( B) - 2 n( A Ç B)
= n( A È B) - n( A Ç B)

Example 1.11

If A and B are sets such that n(A) = 9, n(B) = 16 and Therefore, substituting the values we get
n(A È B) = 25, find A Ç B.
25 = 9 + 16 - n( A Ç B)
Solution: We have = 25 - n( A Ç B)
0 = n( A Ç B)
n( A È B) = n( A) + n( B) - n( A Ç B)
Hence A Ç B = f.

Example 1.12

If A and B are sets such that n( A) = 14, n( A È B) = 26 Solution: We have


and n( A Ç B) = 8, then find n( B) .
n( B) = n( A È B) + n( A Ç B) - n( A)
= 26 + 8 - 14 = 20

Example 1.13

If A, B, C are sets such that n(A) = 12, n(B) = 16, n(C) = 18, n( A) + n( B) + n(C ) - 2 n( A Ç B) - 2 n( B Ç C )
n(A Ç B) = 6, n(B Ç C) = 8, n(C Ç A) = 10 and n(A Ç B Ç - 2 n(C Ç A) + 3n( A Ç B Ç C )
C) = 4, then find the number of elements belonging to exa-
ctly one of A, B and C. = 12 + 16 + 18 - 2 ´ 6 - 2 ´ 8 - 2 ´ 10 + 3 ´ 4
= 10
Solution: The number of elements belonging to exactly
one of A, B and C is

Example 1.14

In Example 1.13, find the number of elements belonging Solution: The number is
to exactly two of A, B and C.
n( A Ç B) + n( B Ç C ) + n(C Ç A) - 3n( A Ç B Ç C )
= 6 + 8 + 10 - 3 ´ 4 = 12
www.jeeneetbooks.in
1.3 Venn Diagrams 23

Example 1.15

If A, B and C are sets defined as A = { x | x Î + and x £ Now


16}, B = { x | x Î and -3 < x < 8} and C = { x | x is a prime
A Ç B = {1, 2, 3, 4, 5, 6, 7}
number}, then find the number of elements belonging to
exactly two of A, B and C, even though C is an infinite set. B Ç C = {2, 3, 5, 7}
C Ç A = {2, 3, 5, 7, 11, 13}
Solution: We have
and A Ç B Ç C = {2, 3, 5, 7}
n( A) = 16, n( B) = 10 and n(C ) = ¥
Therefore, the required number is
n( A Ç B) + n( B Ç C ) + n(C Ç A) - 3n( A Ç B Ç C )
=7+4+6-3´4=5

Example 1.16

In a group of 80 students, 50 play football, 45 play cricket n(F Ç C ) = n(F ) + n(C ) - n(F È C )
and each student plays either football or cricket. Find the
number of students who play both the games. = 50 + 45 - 80 = 15

Solution: Let F be the set of the students who play


football and C be the set of students who play cricket.
Then n(F) = 50 and n(C) = 45. FÇC
Since each of the 80 students play at least one of the
two games, we have n(F È C) = 80. Therefore,
F C

Example 1.17

If 65% of people in a town like apples and 78% like n( A Ç M ) = n( A) + n(M ) - n( A È M )


mangoes, then find out the percentage of people who like
both apples and mangoes and the percentage of people = 65 + 78 - 100 = 43
who like only mangoes. Hence 43% of people like both apples and mangoes.
Also,
Solution: Let the total number of people in the village be
100. Let A be the set of people who like apples and M the n(M ) - n( A Ç M ) = 78 - 43 = 35
set of people who like mangoes. Then n(A) = 65, n(M) = 78 Therefore, 35% of people like only mangoes.
and n(A È M) = 100. Therefore

Example 1.18

The total number of students in a school is 600. If 150 n(A Ç P) = 100. Then
students drink apple juice, 250 students drink pineapple
n( A È P ) = n( A) + n( P ) - n( A Ç P )
juice and 100 students drink both apple juice and
pineapple juice, then find the number of students who = 150 + 250 - 100 = 300
drink neither apple juice nor pineapple juice.
Let S be the set of all students in the school, then S is
the universal set containing A and P. We are given that
Solution: Let
n(S) = 600. Now,
A = The set of students who drink apple juice
and P = The set of students who drink pineapple juice n [S - ( A È P )] = n(S) - n( A È P )
= 600 - 300 = 300
We are given that n(A) = 150, n(P) = 250 and
Therefore 300 students drink neither apple juice nor
pineapple juice.
www.jeeneetbooks.in
24 Chapter 1 Sets, Relations and Functions

Example 1.19

In a class there are 400 students. Following is a table We have,


showing the number of students studying one or more of
the subjects mentioned: n[M - ( P È C )] = n(M ) - n[M Ç ( P È C )]
= n(M ) - n[(M Ç P ) È (M Ç C )]
= n(M ) - [n(M Ç P ) + n(M Ç C )
Mathematics 250
- n(M Ç P Ç M Ç C )]
Physics 150
= n(M ) - n(M Ç P ) - n(M Ç C )
Chemistry 100
+ n(M Ç P Ç C )
Mathematics and Physics 100
= 250 - 100 - 60 + 30
Mathematics and Chemistry 60
= 120
Physics and Chemistry 40
Mathematics, Physics and Chemistry 30 Therefore 120 students study only Mathematics. Also
Only Mathematics n[ P - (M È C )] = n( P ) - n[ P Ç (M È C )]
Only Physics = 150 - n[( P Ç M ) È ( P Ç C )]
Only Chemistry
= 150 - n( P Ç M ) - n( P Ç C )
None of Mathematics, Physics and Chemistry
+ n( P Ç M Ç C )
Fill in the empty places in the above table. = 150 - 100 - 40 + 30
= 40
Solution: Let M, P and C stand for the set of students
studying Mathematics, Physics and Chemistry. Let S be the Therefore 40 students study only Physics. Similarly,
set of all students in the class. The Venn diagram is as follows:
n[C - (M È P )] = n(C ) - n[C Ç (M È P )]
S = 100 - n(C Ç M ) - n(C Ç P )
P C
+ n(C Ç M Ç P )
= 100 - 60 - 40 + 30
= 30
Therefore 30 students study only Chemistry. Again
n(M È P È C ) = n(M ) + n( P ) + n(C ) - n(M Ç P )
M
- n( P Ç C ) - n(C Ç M ) + n(M Ç P Ç C )
= 250 + 150 + 100 - 100 - 40 - 60 + 30
We are given that = 330
n(S) = 400, n(M ) = 250, n( P ) = 150, n(C ) = 100 n[S - (M È P È C )] = n(S) - n(M È P È C )
Also, from the table, = 400 - 330 = 70
n(M Ç P) = 100, n(M Ç C) = 60, n(P Ç C) = 40, Therefore 70 students study none of Mathematics, Physics
n(M Ç P Ç C) = 30 and Chemistry.

Example 1.20

Let X1 , X2 , …, X30 be 30 sets each with five elements and Suppose that each element of S belongs to exactly 10 of
Y1 , Y2 , …, Ym be m sets each with 3 elements. Let Xi’s and exactly 9 of Yj’s. Then find m.
30 m

∪ Xi = ∪ Yj = S
i =1 j =1
www.jeeneetbooks.in
1.4 Relations 25

Solution: Let n(S) = s. Since each element of S belongs Therefore, 10 s = 150 and hence s = 15. Similarly
to exactly 10 of Xi’s, so m

30
3m = å n(Yj ) = 9 ´ s = 9 ´ 15 = 135
å n( X ) = 10 s
i =1
i
j =1

Therefore, m = 45.
Since each Xi contains 5 elements, therefore
30

å n( X ) = 30 ´ 5 = 150
i =1
i

1.4 | Relations
Let A be the set of all straight lines in the plane and B the set of all points in the plane. For any L Î A and x Î B, let
us write L R x if the line L passes through the point x. This is a relation defined between elements of A and elements
of B. Here L R x can be read as “L is related to x” and R denotes the relation “is passing through”. Therefore L R x
means “L is passing through x” . We can also express this statement by saying that the pair of L and x is in relation R
or that the ordered pair (L, x) Î R. This pair is ordered in the sense that L and x cannot be interchanged because the
first coordinate L represents a straight line and the second coordinate represents a point and because the statement
“x passes through L” has no sense. Therefore, we can think of R as a set of ordered pairs (L, x) satisfying the property
that L passes through x. We formalize this in the following.

DE F IN IT ION 1 . 17 Ordered Pairs A pair of elements written in a particular order is called an ordered pair. It
is written by listing its two elements in a particular order, separated by a comma and enclos-
ing the pair in brackets. In the ordered pair (L, x), L is called the first component or the first
coordinate and x is called the second component or the second coordinate.

The ordered pairs (3, 4) and (4, 3) are different even though they consist of same pair of elements; for example these
represent different points in the Euclidean plane.

DE FIN IT ION 1 . 18 The Cartesian Product Let A and B be any sets. The set of all ordered pairs (a, b) with
a Î A and b Î B is called the Cartesian product of A and B and is denoted by A ´ B; that is,
A ´ B = {(a, b)| a Î A and b Î B}

Examples

(1) Let A = {a, b, c} and B = {1, 2}. Then (2) If A = {x, y, z} and B = {a}, then
A ´ B = {(a, 1), (a, 2), (b, 1), (b, 2), (c, 1), (c, 2)} A ´ B = {( x, a), ( y, a), (z, a)}
and B ´ A = {(1, a), (2, a), (1, b), (2, b), (1, c), (2, c)} and B ´ A = {(a, x), (a, y), (a, z)}

QUICK LOOK 4

1. For any sets A and B, 3. For any non-empty sets A and B,


A ´ B = f Û A = f or B = f A´ B= B´ AÛ A= B
2. If one of A and B is an infinite set and the other is a
non-empty set, then the Cartesian product A ´ B is
an infinite set.
www.jeeneetbooks.in
26 Chapter 1 Sets, Relations and Functions

DEF IN IT ION 1 . 19 If A1, A2, ¼, An are sets, then their Cartesian product is defined as the set of n-tuples (a1, a2,
n n
¼, an) such that ai Î Ai for 1 £ i £ n. This is denoted by A1 ´ A2 ´ ´ An or X Ai or
i =1
ÕA .
i =1
i

That is,

A1 ´ A2 ´ ´ An = {(a1 , a2 , …, an )| ai Î Ai for 1 £ i £ n}

If A1 = A2 = = An = A, say, then the Cartesian product A1 ´ A2 ´ ´ An is denoted by An ;


that is,

A1 = A, A2 = A ´ A = {(a, b)| a, b Î A}

A3 = A ´ A ´ A = {(a, b, c)| a, b, c Î A}

An = {(a1, a2, …, an )| ai Î A for 1 £ i £ n}

Examples

(1) If A = {a, b, c}, then (2) If A = {1, 2}, then


A = {(a, a), (a, b), (a, c), (b, a), (b, b),
2
A3 = {(1, 1, 1), (1, 1, 2), (1, 2, 1), (1, 2, 2),
(b, c), (c, a), (c, b), (c, c)} (2, 1, 1), (2, 1, 2), (2, 2, 1), (2, 2, 2)}

T H E O R E M 1 .15 For any finite sets A and B,


n( A ´ B) = n( A) × n( B)
PROOF Let A and B be finite sets such that n(A) = m and n(B) = n. Then A = {a1, a2, ¼, am} and B = {b1, b2, ¼,
bn} where ai’s are distinct elements of A and bj’s are distinct elements of B. In such case
m
A ´ B = ∪ ({ai } ´ B)
i =1

Since {ai } ´ B = {(ai , bj )| 1 £ j £ n} , we get that n({ai } ´ B) = n( B) = n. Also, for any i ¹ k, ai ¹ ak and
hence
({ai } ´ B) Ç ({ak } ´ B) = f

Therefore,

æm ö
n( A ´ B) = n ç ∪ ({ai } ´ B)÷
è i =1 ø
m
= å n({ai } ´ B)
i =1

m
= å n( B)
i =1

m
= ån
i =1

= m × n = n( A)× n( B) ■

C O R O L L A RY 1.6 If A1, A2 , … , Am are finite sets, then A1 ´ A2 ´ ´ Am is also finite and

n( A1 ´ A2 ´ ´ Am ) = n( A1 ) ´ n( A2 ) ´ ´ n( Am )
www.jeeneetbooks.in
1.4 Relations 27

C O R O L L A RY 1.7 If A is a finite set and m is any positive integer, then


n( Am ) = [n( A)]m
In particular, n( A2 ) = n( A)2 .

QUICK LOOK 5

Let A, B, C and D be any sets. Then the following hold. 6. ( A Ç B) ´ (C Ç D) = ( A ´ C ) Ç ( B ´ D)


1. (A È B) ´ C = (A ´ C) È (B ´ C) = ( A ´ D) Ç ( B ´ C )
2. A ´ (B È C) = (A ´ B) È (A ´ C) 7. ( A - B) ´ C = ( A ´ C ) - ( B ´ C )
3. A ´ (B Ç C) = (A ´ B) Ç (A ´ C) 8. A ´ ( B - C ) = ( A ´ B) - ( A ´ C )
4. (A Ç B) ´ C = (A ´ C) Ç (B ´ C)
5. ( A È B) ´ (C È D) =
( A ´ C ) È ( A ´ D) È ( B ´ C ) È ( B ´ D)

Try it out Prove the equalities in Quick Look 5.

Examples

(1) If A = {a, b, c, d} and B = {1, 2, 3}, then Then


n( A ´ B) = n( A) ´ n( B) = 4 ´ 3 = 12 S = {(1, 1), (1, 2), (1, 3), (1, 4), (1, 5), (1, 6)} È {(2, 2),
(2) If A = {a, b, c, d}, then (2, 4), (2, 6)} È {(3, 3), (3, 6), (4, 4), (5, 5), (6, 6)}
2

n(A2) = n(A)2 = 42 = 16 (6) If A is a finite set and n(A) = m, then n[P(A ´ A)] = 2m

and n(A3) = n(A)3 = 43 = 64 (7) If A has 3 elements,


2
then the number of subsets of
A ´ A is 23 = 29 , since A ´ A has 9 elements.
(3) For any sets A and B, we have
(8) If A has only one element, then An also has one
A´B= ∪ ({a} ´ B) = ∪ ( A ´ {b}) element and P( An ) has two elements for any posi-
aÎA bÎB tive integer n.
+
(4) Let S = {(a, b)| a, b Î  and a + 2b = 7} . Then (9) For any non-empty finite sets A and B,

S = {(1, 3), (3, 2), (5, 1)} n( A ´ B) n( A ´ B)


n( A) = and n( B) =
n( B) n( A)
(5) Let
A = {1, 2, 3, 4, 5, 6} and S = {(a, b) | a, b ÎA and a
divides b}

Example 1.21

If A and B are sets such that n(A ´ B) = 6 and A ´ B Since (1, 2), (2, 1) and (3, 2) Î A ´ B, 1, 2, 3 Î A and
contains (1, 2), (2, 1) and (3, 2), then find the sets A, B hence n(A) ³ 3. Also, 2, 1 ÎB and hence n(B) ³ 2. Thus
and A ´ B. n(A) = 3 and n(B) = 2. Therefore, A = {1, 2, 3} and B =
{1, 2}, so that
Solution: Since n(A) × n(B) = n(A ´ B) = 6, n(A) and
A ´ B = {(1, 1), (1, 2), (2, 1), (2, 2), (3, 1), (3, 2)}
n(B) are divisors of 6. Hence n(A) = 1 or 2 or 3 or 6.
www.jeeneetbooks.in
28 Chapter 1 Sets, Relations and Functions

Graphical Representation of Cartesian Product

(d, 5)
5

(a, 4)
4

(c, 3)
3

(e, 2)
2

(b, 1)
1

0
a b c d e X
FIGURE 1.27 Graphical representation of Cartesian product.

Let A and B be non-empty sets. The Cartesian product A ´ B can be represented graphically by drawing two
perpendicular lines OX and OY. We represent elements of A by points on OX and those of B by points on OY.
Now draw a line parallel to OY through the point representing a on OX and a line parallel to OX through the point
representing 4 on OY. The point of intersection of these lines represents the ordered pair (a, 4) in A ´ B. Figure 1.27
represents graphically the Cartesian product A ´ B where A = {a, b, c, d, e} and B = {1, 2, 3, 4, 5}.

DE F IN IT ION 1 . 20 For any sets A and B, any subset of A ´ B is called a relation from A to B.

Examples

(1) {(a, 2), (b, 1), (a, 4), (c, 3)} is a relation from A to B, (2) For any sets A and B, the empty set f and A ´ B are
where A = {a, b, c, d} and B = {1, 2, 3, 4}. also relations from A to B.

DE F IN IT ION 1 . 21 Let R be a relation from a set A into a set B. That is, R Í A ´ B. If (a, b) Î R, then we say that
“a is R related to b” or “a is related to b with respect to R” or “a and b have relation R”. It is
usually denoted by a R b.

DE FIN IT ION 1 . 22 Domain Let R be a relation from A to B. Then the domain of R is defined as the set
of all first components of the ordered pairs belonging to R and is denoted by Dom (R).
Mathematically,
Dom(R) = {a | (a, b) Î R for some b Î B}

Note that Dom(R) is a subset of A and that Dom(R) is non-empty if and only if R is non-empty.

D E F I N I T I O N 1 . 2 3 Range Let R be a relation from A to B. Then the range of R is defined as the set of all
second components of the ordered pairs belonging to R and is denoted by Range(R).
Mathematically,
Range(R) = {b | (a, b) Î R for some a Î A}

Note that Range(R) is a subset of B and that it is non-empty if and only if R is non-empty.
www.jeeneetbooks.in
1.4 Relations 29

Examples

(1) Let A = {1, 2, 3, 4}, B = {a, b, c, d, e}, and R = {(1, a), (3) Let R = {(a, b) Î + ´ + | 2a = b}. Then R is a relation
(2, c), (3, a), (2, a)}. Then from + to + and is given by
Dom(R) = {1, 2, 3} and Range(R) = {a, c} R = {(a, 2a) | a is a positive integer}
(2) Let A = {2, 3, 4}, B = {2, 3, 4, 5, 6, 7, 8} and R = {(a, b) Î Then
A ´ B | a divides b}. Then
Dom(R) = +
R = {(2, 2), (2, 4), (2, 6), (2, 8), (3, 3), (3, 6), (4, 4), (4, 8)} and Range(R) = The set of all positive even integers
Dom(R) = {2, 3, 4} and Range(R) = {2, 4, 6, 8, 3}

T H E O R E M 1 .16 Let A and B be non-empty finite sets with n(A) = m and n(B) = n. Then the number of relations
from A to B is 2mn.

PROOF It is known that the number of subsets of an n-element set is 2n. Since the relations from A to B
are precisely the subsets of A ´ B and since n(A ´ B) = n(A) × n(B) = mn, it follows that there are
exactly 2mn relations from A to B. ■

Examples

(1) Let A = {1, 2, 3} and B = {a, b}. Then n(A) = 3, n(B) = 2 (2) Let A and B be two finite sets and K be the number
and n(A ´ B) = n(A) × n(B) = 3 × 2 = 6. Therefore there of relations from A to B. Then K is not divisible by
are exactly 64 (=26) relations from A to B. any odd prime number, since K = 2n ( A)× n ( B) and 2 is
the only prime dividing 2m for any positive integer m.

Representations of a Relation
A relation can be expressed in many forms such as:
1. Roster form: In this form, a relation R is represented by the set of all ordered pairs belonging to R. For example,
R = {(1, a), (2, b), (3, a), (4, c)} is a relation from the set {1, 2, 3, 4} to the set {a, b, c}.
2. Set-builder form: Let A = {2, 3, 4, 5} and B = {2, 4, 6, 8, 10}. Let R = {(a, b) ÎA ´ B | a divides b}. Then R is a relation
from A to B. This is known as the set-builder form of a relation. Note that
R = {(2, 2), (2, 4), (2, 6), (2, 8), (2, 10), (3, 6), (4, 4), (4, 8), (5, 10)}
3. Arrow-diagram form: In this form, we draw an arrow corresponding to each ordered pair (a, b) in R from the first
component a to the second component b. For example, consider the relation R given in (2) above. Then R can be
represented as shown in Figure 1.28. There are nine arrows corresponding to nine ordered pairs belonging to the
relation R.

2
2
4
3 6

4 8

5 10

A B
FIGURE 1.28 Representation of arrow-diagram form.
www.jeeneetbooks.in
30 Chapter 1 Sets, Relations and Functions

4. Tabular form: To represent a given relation R, sometimes it is convenient to look at it in a tabular form. Suppose
R is a relation from a finite set A to a finite set B. Let
A = {a1, a2, …, an} and B = {b1, b2, …, bm}
Write the elements b1, b2, ..., bm (in this order) in the top row of the table and the elements a1, a2, …, an (in this order)
in the leftmost column. For any 1 ≤ i ≤ n and 1 ≤ j ≤ m, let us define

ìï 1 if (ai , bj ) Î R
rij = í
ïî0 if (ai , aj ) Ï R

Write rij in the box present in the ith row written against ai and in the jth column written against bj. This is called the
tabular form representation of the relation R.

Examples

Tabular Form R 2 4 6 8 10
Let us consider sets A = {2, 3, 4, 5}, B = {2, 4, 6, 8, 10}, and 2 1 1 1 1 1
relation R given by 3 0 0 1 0 0
R = {(a, b) Î A ´ B | a divides b} 4 0 1 0 1 0
5 0 0 0 0 1
That is
R = {(2, 2), (2, 4), (2, 6), (2, 8), (2, 10), (3, 6), (4, 4), Instead of writing 1 and 0, we can write T and F
(4, 8), (5, 10) signifying whether ai Rbj is true or false.
This relation R is represented in the following tabular form.

Among all four representations of a relation, the set-builder form is most popular and convenient. The roster form,
the arrow-diagram form and the tabular form can represent a relation R from A to B only when both the sets A and B
are finite. The set-builder form is more general and can represent a relation even when A or B or both are infinite sets.

Examples

Let R = {(a, b) Î + ´ +| a divides b}. Then R is a relation form or set-builder form or tabular form. Note that
from + to + . This cannot be represented by the roster
Dom(R) = + = Range(R)

DE FIN IT ION 1 . 24 Binary Relation Any relation from a set A to itself is called a binary relation on A or
simply a relation on A.

For example, the relation R given in the above example is a relation on +.
2 2
Remark: For any n-element set A, there are 2n relations on A. For example, if A = {a, b, c}, then there are 512 (= 23 )
relations on A.

DEF IN IT ION 1 . 25 Composition of Relations Let A, B and C be sets, R a relation from A to B and S a relation
from B to C. Define
S R = {(a, c) Î A ´ C | there exists b Î B such that (a, b) Î R and (b, c) Î S}
Then S R is a relation from A to C. In other words for any a Î A and c ÎC,
a(S R)c Û aRb and bSc for some b Î B
S R is called the composition of R with S.

Note that, for any relations R with S, R S may not be defined at all even when S R is defined. Also even when both
R S and S R are defined, they may not be equal.
www.jeeneetbooks.in
1.4 Relations 31

Examples

(1) Let R = {(a, b) Î + ´ + | b = 2a} and S = {(a, b) Î Note that (3, 8) ÏR S and (3, 10) ÏS R. Therefore
+ ´ + | b = a + 2}. Then both R and S are relations S R Ë R S and R S Ë S R.
from + to + and hence both R S and S R are (2) Let A = {1, 2, 3, 4}, B = {a, b, c, d}, and C = {x, y, z}. Let
defined. For any positive integers a and c, we have
R = {(1, c), (2, d), (2, a), (3, d)}
a(R S)c Û aSb and bRc for some b Î +
and S = {(a, y), (b, x), (b, y), (a, z)}
Û b = a + 2 and c = 2b for some b Î +
Then R is a relation from A to B and S is a relation
Û c = 2(a + 2) = 2a + 4 from B to C.
and Dom(R) = {1, 2, 3} and Range(R) = {a, c, d}
a(S R)c Û aRb and bSc for some b Î  + Dom(S) = {a, b} and Range(S) = {x, y, z}

Û b = 2a and c = b + 2 for some b Î + R S is not defined. However S R is defined and

Û c = 2a + 2 S R = {(2, y), (2, z)}


Since (2, a) ÎR and (a, y) ÎS, we have (2, y) ÎS R
For example, (3, 10) ÎR S since (3, 5) ÎS and (5, 10)
Since (2, a) ÎR and (a, z) ÎS, we have (2, z) ÎS R
ÎR. Also, (3, 8) ÎS R since (3, 6) ÎR and (6, 8) ÎS.

T H E O R E M 1 .17 Let A, B and C be sets, R a relation from A to B and S a relation from B to C. Then the following
hold:
1. S R ¹ f if and only if Range(R) Ç Dom(S) ¹ f
2. Dom(S R) Í Dom(R)
3. Range(S R) Í Range(S)
PROOF 1. Suppose that S R ¹ f. Choose(a, c) Î S R. Then there exists b Î B such that (a, c) Î R and
(b, c) Î S and hence b Î Range(R) and b Î Dom(S). Therefore b Î Range(R) Ç Dom(S). Thus
Range(R) Ç Dom(S) is not empty.
Conversely, suppose that Range(R) Ç Dom(S) ¹ f. Choose b Î Range(R) Ç Dom(S). Then
there exist a Î A and c Î C such that (a, b) Î R and (b, c) Î S and hence (a, c) Î S R. Thus S R
is not empty.
2. a Î Dom(S R) Þ (a, c) Î S R for some c Î C
Þ (a, b) Î R and (b, c) Î S for some b Î B
Þ a Î Dom(R)
Therefore Dom(S R) Í Dom(R).
3. c Î Range(S R) Þ (a, c) Î S R for some a Î A
Þ (a, b) Î R and (b, c) Î S for some b Î B
Þ c Î Range(S)
Therefore Range (S R) Í Range(S). ■

Example 1.22

Find S R, Dom(S R), Range(S R) for the following: (2) The sets are the same as above. The relations are
(1) A = {1, 2, 3, 4}, B = {a, b, c} and C = {x, y, z}. The rela- R = {(1, a), (2, b), (2, c), (4, a)}
tions are R = {(2, a), (3, b), (2, b), (3, c)} and S = {(a,
y), (b, x), (b, y)}. and S = {(b, x), (b, y), (d, z)}
www.jeeneetbooks.in
32 Chapter 1 Sets, Relations and Functions

Solution: (2) Using the given data we have


(1) From the given data, we have S R = {(2, x), (2, y)}
Dom(R) = {2, 3} and Range(R) = {a, b, c} Dom(S R) = {2} Ì {1, 2, 4} = Dom(R)
Dom(S) = {a, b} and Range(S) = {x, y} Range(S R) = {x, y} Ì {x, y, z} = Range(S)
(a) S R = {(2, y), (3, x), (3, y), (2, x)}
(b) Dom(S R) = {2, 3} = Dom(R)
(c) Range(S R) = {x, y} = Range(S)

T H E O R E M 1 .18 Let A, B, C and D be non-empty sets, R Í A ´ B, S Í B ´ C and T Í C ´ D. Then


(T S) R = T (S R)
PROOF For any a Î A and d Î D,
(a, d) Î (T S) R Þ (a, b) Î R and (b, d) Î T S for some b Î B
Þ (a, b) Î R, (b, c) Î S and (c, d) Î T for some b Î B and c Î C
Þ (a, c) Î S R and (c, d) Î T, c Î C
Þ (a, d) Î T (S R)
Therefore,
(T S) R Í T (S R)
Similarly
T (S R) Í (T S) R
Thus,
(T S) R = T (S R) ■

DEFIN IT ION 1 . 26 Inverse of a Relation Let A and B be non-empty sets and R a relation from A to B. Then
the inverse of R is defined as the set
{(b, a) Î B ´ A| (a, b) Î R}
and is denoted by R-1.

Note that, if R is a relation from A to B, then R-1 is a relation from B to A and that R R-1 is a relation on B and R-1 R
is a relation on A.

Examples

Let A = {1, 2, 3, 4} and B = {a, b, c, d, e}. R R-1 = {(a, a), (b, b),(b, c),(a, e), (d, d),
Let R = {(1, a), (2, b), (3, a), (4, d), (2, c), (3, e)}. Then (c, b), (c, c), (e, a), (e, e)}

R-1 = {(a, 1), (b, 2), (a, 3), (d, 4), (c, 2), (e, 3)} and R-1 R = {(1, 1), (2, 2), (3, 3), (4, 4)} = DA
(the diagonal of A).

T H E O R E M 1 .19 Let A, B and C be non-empty sets and R a relation from A to B and S a relation from B to C. Then
the following hold.
1. (S R)-1 = R-1 S-1
2. (R-1)-1 = R
www.jeeneetbooks.in
1.5 Equivalence Relations and Partitions 33

PROOF 1. S R is relation from A to C and therefore (S R)-1 is relation from C to A. Now consider
(c, a) Î (S R)-1 Û (a, c) Î S R
Û (a, b) Î R and (b, c) Î S for some b Î B
Û (c, b) Î S-1 and (b, a) Î R-1 for some b Î B
Û (c, a) Î R-1 S-1
Therefore (S R)-1 = R-1 S-1.
2. It is trivial and left as an exercise for the reader. ■

1.5 | Equivalence Relations and Partitions


A partitioning of a set is dividing the set into disjoint subsets as shown in the Venn diagram in Figure 1.29. In this
section we discuss a special type of relations on a set which induces a partition of the set and prove that any such
partition is induced by that special type of relation. Let us begin with the following.

FIGURE 1.29 Partitioning of a set.

DEF IN IT ION 1 . 27 Let X be a non-empty set and R a (binary) relation on X. Then,


1. R is said to be reflexive on X if (x, x) Î R for all x Î X.
2. R is said to be symmetric if (x, y) Î R Þ (y, x) Î R
3. R is said to be transitive if (x, y) Î R and (y, z) Î R Þ (x, z) Î R.
4. R is said to be an equivalence relation on X if it is a reflexive, symmetric and transitive
relation on X.

Examples

(1) Let X = {1, 2, 3, 4} and R = {(1, 2), (2, 1), (1, 1), (2, 2)}. (4) For any set X, let
Then R is a relation on X. R is not reflexive on X,
DX = {(x, x)| x ÎX}
since 3 ÎX and (3, 3) ÏR. However R is symmetric
and transitive. You can easily see that R is reflexive Then DX is reflexive, symmetric and transitive rela-
on a smaller set, namely {1, 2}. Therefore R is an equ- tion on X and hence an equivalence relation on X.
ivalence relation on {1, 2}. DX is called the diagonal on X.
(2) Let R = {(a, b) Î + ´ + | a divides b}. Then R is a (5) For any positive integer n, let
reflexive and transitive relation on the set + of posi-
Rn = {(a, b) Î  ´  | n divides a - b}
tive integers. However, R is not symmetric, since
(2, 6) ÎR and (6, 2) ÏR. Note that a relation R on a For any a Î , n divides 0 = a - a and hence (a, a) Î Rn.
set S is symmetric Û R = R-1. Therefore Rn is reflexive on . For any a, b Î ,
(3) Let X = {1, 2, 3, 4} and R = {(1, 1), (2, 2), (3, 3), (4, 4), (a, b) ÎRn Þ n divides (a - b)
(2, 3), (3, 2), (3, 4), (4, 3)}. Then R is a reflexive and
Þ n divides - (a - b)
symmetric relation on X. But R is not transitive, since
(2, 3) ÎR and (3, 4) ÎR, but (2, 4) ÏR. Þ n divides (b - a)
Þ (b, d) ÎRn
www.jeeneetbooks.in
34 Chapter 1 Sets, Relations and Functions

Therefore Rn is symmetric. Also, for any a, b and Therefore Rn is transitive also. Thus Rn is an equiva-
c Î , lence relation on  and is called the congruence relation
modulo n.
(a, b) ÎRn and (b, c) ÎRn Þ n divides (a - b) and (b - c)
(6) Let A and B be subsets of a set X such that A Ç B = f
Þ n divides (a - b) + (b - c) and A È B = X. Define
Þ n divides (a - c) R = {(x, y) ÎX ´ X | either x, y ÎA or x, y ÎB}
Þ (a, c) ÎRn Then R is an equivalence relation on X.

T H E O R E M 1 .20 Let R be a symmetric and transitive relation on a set X. Then the following are equivalent to each
other.
1. R is reflexive on X.
2. Dom(R) = X.
3. Range(R) = X.
4. R is equivalence relation on X.
PROOF Since R is already symmetric and transitive, (1) Û (4) is clear.
Also, since (a, b) Î R if and only if (b, a) Î R, it follows that (2) Û (3).
If R is reflexive on X, then (x, x) Î R for all x Î X and hence Dom(R) = X. Therefore (1) Û (2)
is clear.
Finally, we shall prove (2) Þ (1). Suppose that Dom(R) = X. Then,
x Î X Þ x Î Dom(R)
Þ (x, y) Î R for some y Î X
Þ (x, y) Î R and (y, x) Î R (since R is symmetric)
Þ (x, x) Î R (since R is transitive)
Therefore (x, x) Î R for all x Î X. Thus R is reflexive on X. ■

DEF IN IT ION 1 . 28 Partition Let X be a non-empty set. A class of non-empty subsets of X is called a partition
of X if the members of the class are pairwise disjoint and their union is X. In other words, a
class of sets {Ai}iÎI is called a partition of X if the following are satisfied:
1. For each i Î I, Ai is a non-empty subset of X
2. Ai Ç Aj = f for all i ¹ j Î I
3. ∪A = X
i ÎI
i

Examples

(1) For any set X, the class {{x}}x ÎX is a partition of X; (3) For any non-empty proper subset A of a set X, the
that is, the class of all singleton subsets of X is a class {A, X - A} is a partition of X. Note that X - A
partition of X. is not empty since A is a proper subset of X.
(2) Let E = the set of all even integers and O = the set of
all odd integers. Then the class {E, O} is a partition of .

DEF IN IT ION 1 . 29 Let R be an equivalence relation on a set X and x Î X. Then define


R(x) = {y Î x | (x, y) Î R}
R(x) is a subset of X and is called the equivalence class of x with respect to R or the
R-equivalence class of x or simply the R-class of x.
www.jeeneetbooks.in
1.5 Equivalence Relations and Partitions 35

Examples

(1) Let X = {1, 2, 3, 4} and R = {(1, 1), (2, 2), (3, 3), (4, 4), Rn(a) = {y ÎX | (a, y) ÎRn}
(2, 3), (3, 2)}. Then R is an equivalence relation on X
= {y ÎX | n divides a - y}
and the R-classes are as follows:
= {y ÎX | a - y = nx for some x Î }
R(1) = {x ÎX | (1, x) ÎR} = {1}
= {a + nx | x Î }
R(2) = {x ÎX | (2, x) ÎR} = {2, 3}
We can prove that Rn(0), Rn(1), …, Rn(n - 1) are all
R(3) = {x ÎX | (3, x) ÎR} = {2, 3}
the distinct Rn-classes in . If a ³ n or a < 0, we can
R(4) = {x ÎX | (4, x) ÎR} = {4} write by the division algorithm that
(2) Let n be a positive integer and a = qn + r
Rn = {(a, b) Î ´  | n divides a - b} where q, r Î  and 0 £ r < n. Hence Rn(a) = Rn(r),
0 £ r < n.
Then Rn is an equivalence relation on the set  of
integers. For any a Î , the Rn-class of “a” denoted
by Rn (a) is given by

T H E O R E M 1 .21 Let R be an equivalence relation on a set X and a, b Î X. Then the following are equivalent to each other:
1. (a, b) Î R
2. R(a) = R(b)
3. R(a) Ç R(b) ¹ f
PROOF (1) Þ (2): Suppose that (a, b) Î R. Then (b, a) Î R (since R is symmetric) and
x Î R(a) Þ (a, x) Î R
Þ (b, a) Î R and (a, x) Î R
Þ (b, x) Î R (since R is transitive)
Þ x Î R(b)
Therefore R(a) Í R(b). Similarly R(b) Í R(a). Thus R(a) = R(b).
(2) Þ (3) is trivial, since a Î R(a) and if R(a) = R(b), then a Î R(a) Ç R(b).
(3) Þ (1): Suppose that R(a) Ç R(b) ¹ f. Choose an element c Î R(a) Ç R(b). Then (a, c) Î R and
(b, c) Î R and hence (a, c) Î R and (c, b) Î R. Since R is transitive, we get that (a, b) Î R. ■

T H E O R E M 1 .22 Let R be an equivalence relation on a set X. Then the class of all distinct R-classes forms a partition
of X; that is,
1. R(a) is a non-empty subset of X for each a Î X.
2. Any two distinct R-classes are disjoint.
3. The union of all R-classes is the whole set X.
PROOF 1. By definition of the R-class R(a), we have
R(a) = { x Î X | (a, x) Î R}
Therefore R(a) is a subset of X. Since (a, a) Î R we have a Î R(a). Thus R(a) is a non-empty
subset of X for each a Î X.
2. This is a consequence of (2) Û (3) of Theorem 1.21.
3. Since a Î R(a) for all a Î X, we have
∪ R(a) = X
a ÎX ■
www.jeeneetbooks.in
36 Chapter 1 Sets, Relations and Functions

Examples

Let X = {1, 2, 3, 4, 5, 6, 7, 8} and R = {(x, y) Î X ´ X | both and R(2) = {2, 4, 6, 8} = R(4) = R(6) = R(8)
x and y are either even or odd}. Then
Therefore, there are only two distinct R-classes, namely
R(1) = {1, 3, 5, 7} = R(3) = R(5) = R(7) R(1) = {1, 3, 5, 7} and R(2) = {2, 4, 6, 8} and these two form
a partition of X.

In Theorem 1.22, we have obtained a partition from a given equivalence relation on set a X. Infact, for any given
partition of X, we can define an equivalence relation on X which induces the given partition. This is proved in the
following.

T H E O R E M 1 .23 Let X be a non-empty set and {Ai}iÎI a partition of X. Define


R = {(x, y) Î X ´ X | both x and y belong to same Ai , i Î I }
Then R is an equivalence relation whose R-classes are precisely Ai ’s .
PROOF We are given that {Ai}iÎI is a partition of X, that is,
1. Each Ai is a non-empty subset of X.
2. Ai Ç Aj = f for all i ¹ j Î I .
3. ∪ A = X.
i ÎI
i

For any x Î X, there exists only one i Î I such that x Î Ai and hence (x, x) Î R. This means that R
is reflexive on X; clearly R is symmetric. Also, (x, y) Î R and (y, z) Î R Þ x, y Î Ai and y, z Î Aj for
some i, j Î I. This implies
Ai Ç Aj ¹ f and hence i = j and Ai = Aj
Þ x, z Î Ai , i Î I
Þ (x, z) Î R
Thus R is transitive also. Therefore R is an equivalence relation on X. For any i Î I and x Î Ai, we
have
y Î Ai Û (x, y) Î R Û y Î R(x)
and have Ai = R(x). This shows that Ai ’s are all the R-classes in X. ■

Theorems 1.22 and 1.23 imply that we can get a partition of X from an equivalence relation on X and conversely
we can get an equivalence relation from a partition of X and that these processes are inverses to each other.

Examples

For any i = 0, 1 or 2, let R = {(a, b) Î + ´ + | a, b Î A0 or a, b Î A1


Ai = {a Î + | on dividing a with 3, the remainder is i} or a, b Î A2 }
That is, = {(a, b) Î + ´ + | The remainders are same
when a and b are divided by 3}
A0 = {3, 6, 9, 12, …} = {3n | n Î + }
= {(a, b) Î + ´ + | 3 divides a - b}
+
A1 = {1, 4, 7, 10, …} = {3n + 1| 0 £ n Î  }
In this case, R(1) = A1, R(2) = A2 and R(3) = A0 and these
A2 = {2, 5, 8, 11, …} = {3n + 2 | 0 £ n Î + } three are the only R-classes in +.

Then {A0, A1, A2} is a partition of +. The equivalence


relation corresponding to this partition is
www.jeeneetbooks.in
1.5 Equivalence Relations and Partitions 37

T H E O R E M 1 .24 Let R and S be two equivalence relations on a non-empty set X. Then R Ç S is also an equivalence
relation on X and, for any x Î X,
(R Ç S)(x) = R(x) Ç S(x)
PROOF For any x Î X, (x, x) Î R and (x, x) Î S (since R and S are reflexive on X ). Hence (x, x) Î R Ç S.
Therefore R Ç S is reflexive on X. Also,
(x, y) Î R Ç S Þ (x, y) Î R and (x, y) Î S
Þ (y, x) Î R and (y, x) Î S
Þ (y, x) Î R Ç S
Therefore R Ç S is symmetric. Further
(x, y), (y, z) Î R Ç S Þ (x, y), (y, z) Î R and (x, y), (y, z) Î S
Þ (x, z) Î R and (x, z) Î S
Þ (x, z) Î R Ç S
Therefore R Ç S is an equivalence relation. For any x Î X, we have
(R Ç S)(x) = { y Î X | ( x, y) Î R Ç S}
= { y Î X | ( x, y) Î R} Ç { y Î X | ( x, y) Î S}
= R(x) Ç S(x) ■

We have proved in Theorem 1.24 that the intersection of equivalence relations on a given set X is again an
equivalence relation. This result cannot be extended to the composition of equivalence relations. In this direction, we
have the following theorem that gives us several equivalent conditions for the composition of equivalence relations to
again become an equivalence relation.

T H E O R E M 1 .25 Let R and S be equivalence relations on a set X. Then the following are equivalent to each other.
1. R S is an equivalence relation on X.
2. R S is symmetric.
3. R S is transitive.
4. R S = S R.
PROOF (1) Þ (2) is clear.
(2) Þ (3): Suppose that R S is symmetric. Then
R S = (R S)–1 = S–1 R–1 = S R
and (R S) (R S) = R (S R) S
= R (R S) S
= (R R) (S S)
= (R S)
Since R and S are reflexive, we get that R DX = R = DX R and S DX = S = DX S. Also,
since R and S are transitive, R R Í R = R DX Í R R-1 so that R R = R. Similarly, S S = S.
Therefore, R S is transitive.
(3) Þ (4): Suppose that R S is transitive. Then (R S) (R S) = R S. Now, consider
S R = (DX S) (R DX ) Í (R S) (R S) = R S
and R S = R-1 S-1 = (S R)-1 Í (R S)-1 = S-1 R-1 = S R
www.jeeneetbooks.in
38 Chapter 1 Sets, Relations and Functions

Therefore
R S = S R
(4) Þ (1): Suppose that R S = S R. Then
(R S)-1 = S-1 R-1 = S R = R S
Hence R S is symmetric and transitive also. Further DX = DX DX Í R S and therefore R S is
reflexive on X. Thus, R S is an equivalence relation on X. ■

1.6 | Functions
Functions are a special kind of relations from one set to another set. The concept of a function is an important tool
in any area of logical thinking, not only in science and technology but also in social sciences. The word “function”
is derived from a Latin word meaning operation. For example, when we multiply a given real number x by 2, we
are performing an operation on the number x to get another number 2x. A function may be viewed as a rule which
provides new element from some given element. Function is also called a map or a mapping. In this section, we discuss
various types of functions and their properties. The following is a formal definition of a function.

DEF IN IT ION 1 . 30 Function A relation R from a set A to a set B is called a function (or a mapping or a map)
from A into B if the following condition is satisfied:
For each element a in A there exists one and only one element b in B such that (a, b) Î R.
That is, R Í A ´ B is called a function from A into B if the following hold:
1. For each a Î A, there exists b Î B such that (a, b) Î R.
2. If (a, b) Î R and (a, c) Î R, then b = c.

ALTERNATE DEFINITION A relation R from A to B is a function from A into B if Dom(R) = A and whenever
the first components of two ordered pairs in R are equal, then the second components
are also equal.

Examples

(1) Let R = {(x, 2x) | x Î }. Then R is a function from  (4) Let A and B be as in (3) above and R = {(1, a), (2, b),
into . (3, c), (3, a), (4, a)}. Then R is not a function from A
(2) Let R = {(x, | x |) | x Î }. Then R is a function from into B, since we have two ordered pairs (3, c) and (3, a)
the real number system  into itself. in R whose first components are equal and the
second components are different. Also, if S = {(1, a),
(3) Let A = {1, 2, 3, 4} and B = {a, b, c}. Let R = {(1, a),
(2, b), (4, c)}, then S is not a function of A into B,
(2, a), (3, b), (4, b)}. Then R is a function from A into B.
since Dom(S) ¹ A.

Notation
1. If R is a function from A into B and a Î A, then the unique element b in B such that (a, b) Î R is denoted by R(a).
2. Usually functions will be denoted by lower case letters f, g, h, ….
3. If f is a function from A into B, then we denote this by f : A ® B.
4. If f : A ® B is a function and a Î A, then there exists a unique element b in A such that (a, b) Î f. This unique
element is denoted by f (a). We write f (a) = b to say that (a, b) Î f or a f b. Some authors also write (a)f = b or simply
bb

af = b to say that (a, b) Î f . However in this chapter we prefer to use f (a) = b.

DEF IN IT ION 1 . 31 Let f : A ® B be a function. Then A is called the domain of f and is denoted by Dom( f ). B is
called the co-domain of f and is denoted by codom( f ). The range of f is also called the image
of f or the image of A under f and is denoted by Im( f ). That is,
Im( f ) = { f (a) | a Î A}
www.jeeneetbooks.in
1.6 Functions 39

Note that Im( f ) is a subset of B and may not be equal to B. If f (a) = b, then b is called the image of a under f and a is
called a pre-image of b. Note that for any a Î A, the image of a under f is unique. But, for b Î B, there may be several
pre-images of b or there may not be any pre-image of b at all. To describe a function f : A ® B it is enough if we pre-
scribe the image f (a) of each a Î A under f.

Examples

(1) Define a function f :  ®  by f (x) = x2 for all x Î. (2) Define f :  ®  by f (x) = x / 2 for all x Î. Then the
That is, f = {(x, x2) | x Î }. Here x2 is the image of domain of f is  and the co-domain of f is . Also
any x Î. Note that x2 is always non-negative for
any x Î and hence a negative real number has no ìx ü
Im( f ) = { f ( x) | x Î } = í | x Î  ý
pre-image under f. For example, there is no x Î such î2 þ
that f (x) = -1. Here both the domain and co-domain
of the function are  and the image of f (or range Here note that every integer n has a pre-image, namely
of f ) is equal to the set of non-negative real numbers. 2n, since f (2n) = n. The real number 1/3 has no pre-image.

Quite often a function is given by an equation of type f (x) = y without specifically mentioning the domain and co-
domain. We can identify the domain and co-domain by looking at the validity of the equation. The following examples
illustrate these.

Example 1.23

Let f be the function defined by The expression of the right-hand side has meaning for
all real numbers except when x = 6 or x = 2. Therefore,
x2 + 2 x + 1 the domain of f is the set at all real number other than 6
f ( x) =
x2 - 8 x + 12 and 2, that is,
Find out the domain of f. Dom ( f ) =  - {2, 6}

Solution: We are given that


x2 + 2 x + 1
f ( x) =
x2 - 8 x + 12

Example 1.24

Consider a function defined by Suppose


x2
ïìæ x2 ö üï y = f ( x) =
f = íç x, 2÷
x Î ý 1 + x2
ïîè 1 + x ø ïþ Then
Then f is a function from  into . Find the range of f. y + yx 2 = x 2 or x 2 (1 - y) = y
Therefore
Solution: We have
y y
x2 x2 = or x=±
f ( x) = for all x Î 1- y 1- y
1 + x2
provided y /(1 - y) ³ 0; that is, 0 £ y < 1. Thus the range
of f is [0, 1).

DEF IN IT ION 1 . 32 Let f : A ® B and g : B ® C be functions. Then the composition of f with g is defined as the
function g f : A ® C given by
(g f )(a) = g( f (a)) for all a Î A
www.jeeneetbooks.in
40 Chapter 1 Sets, Relations and Functions

Note that g f is defined only when the range of f is contained in the domain of g. If f : A ® B is a function and g : D ® C is
another function such that Range( f ) Í D = Dom(g), then g f can be defined as a function from A into C. When we regard
functions as relations, then the composition of functions is same as that of the relations as given in Definition 1.25. That is,
(a, c) Î g f Û (a, b) Î f and (b, c) Î g for some b Î B
Û f (a) = b and g(b) = c
Û g( f (a)) = c
Û (g f )(a) = c

Example 1.25

Let f :  ®  and g :  ®  be defined by [( x + 2)/ 3]2 - 1


=
x+2 [( x + 2)/ 3]2 + 1
f ( x) = for all x Î
3
( x + 2)2 - 9
=
x -1
2
( x + 2)2 + 9
and g( x) = for all x Î
x2 + 1
x2 + 4 x - 5
Find (g f )(x). =
x2 + 4 x + 13
Solution: We have

æ x + 2ö
( g f )( x) = g( f ( x)) = g ç
è 3 ÷ø

Example 1.26

Let A = {1, 2, 3, 4}, B = {a, b, c} and C = {x, y, z}. Let Solution: We have f : A ® B and g : B ® C are func-
tions. Then g f : A ® C is given by
f = {(1, a), (2, c), (3, b), (4, a)}
g f = {(1, y), (2, x), (3, z), (4, y)}
and g = {(a, y), (b, z), (c, x)}
Find g f.

Try it out
T H E O R E M 1 .26 Let f : A ® B and g : B ® C be functions. Then Dom(g f ) = Dom( f ) and codom(g f ) =
codom(g).

Two functions f and g are said to be equal if their domains are equal and f(x) = g(x) for all elements x in Dom ( f ).
For any functions f and g, even when both g f and f g are defined, g f may be different from f g, as seen in the
following example.

Example 1.27

Let f :  ®  and g :  ®  be defined by ( f g )( x) = f ( g( x))


f (x) = x 2
and g(x) = x + 2 for all x Î = f ( x + 2) = ( x + 2)2
Show that g f ¹ f g. = x2 + 4 x + 4
Solution: We have Therefore g f ¹ f g.
( g f )( x) = g( f ( x)) = g( x ) = x + 2
2 2
www.jeeneetbooks.in
1.6 Functions 41

The following is an easy verification and is a direct consequence of Theorem 1.18.

Try it out
T H E O R E M 1 . 27 Let f : A ® B, g : B ® C and h : C ® D be functions. Then
h ( g f ) = (h g ) f

In the following we discuss certain special types of functions. If f : A ® B is a function, a1 and a2 are elements of A
and b1 and b2 are elements of B such that f (a1) = b1 and f (a2) = b2 and if a1 = a2, then necessarily b1 = b2. In other words,
two elements of B are equal if their pre-images are equal. It is quite possible that two distinct elements of A may have
equal images under f. A function f : A ® B is called an injection if distinct elements of A have distinct images under f.
The following is a formal definition.

DEF IN IT ION 1 . 32 Injection A function f : A ® B is called an injection or “one-one function” if f (a1) ¹ f (a2)
for any a1 ¹ a2 in A; in other words,
f (a1) = f (a2) Þ a1 = a2
for any a1, a2 Î A.

Examples

(1) Let f :  ®  be defined by Then f is not an injection, since two distinct elements
f (x) = x + 2 for all x Î  have the same image; for example, 1 ¹ -1 but f(1) = 12
= (–1)2 = f(–1).
Then f is an injection, since, for any x, y, Î ,
f ( x) = f ( y) Þ x + 2 = y + 2 Þ x = y
(2) Let f :  ®  be defined by
f (x) = x2 for all x Î 

T H E O R E M 1 .28 Let f : A ® B and g : B ® C be functions. Then the following hold.


1. If f and g are injections, then so is g f.
2. If g f is an injection, then f is an injection.
PROOF 1. Suppose that both f and g are injections. For any a1, a2 Î A, we have
( g f )(a1 ) = ( g f )(a2 )
Þ g( f (a1 )) = g( f (a2 ))
Þ f (a1 ) = f (a2 ) (since g is an injection)
Þ a1 = a2 (since f is an injection)
Therefore, g f is an injection.
2. Suppose that g f is an injection. Then, for any a1, a2 Î A, we have
f (a1 ) = f (a2 ) Þ g( f (a1 )) = g( f (a2 )) (∵ g is a function)
Þ ( g f )(a1 ) = ( g f )(a2 )
Þ a1 = a2 (since g f is an injection)
Therefore f is an injection. ■

Note that g f can be an injection without g being an injection. An example of this case is given in the following.
www.jeeneetbooks.in
42 Chapter 1 Sets, Relations and Functions

Example

Define Þ (x + 2)2 = (y + 2)2


f : + ®  by f(x) = x + 2 for all x Î+ Þx+2=y+2 (since x and y are positive)
and g :  ®  by g(x) = x2 for all x Î Þx=y
+
Then g f :  ®  is given by Therefore g f is an injection. However, g is not an injec-
tion, since
(g f )(x) = g( f(x)) = g(x + 2) = (x + 2)2 for all x Î+
g(2) = 22 = (–2)2 = g(–2)
Now, for any x, y Î+,
(g f )(x) = (g f )(y)

Next we discuss functions under which every element in the codomain is the image of some element in the domain.

DEF IN IT ION 1 . 33 Surjection A function f : A ® B is called a surjection or “onto function” if the range of f is
equal to the co-domain B; that is, for each b Î B, b = f(a) for some a Î A.

Examples

(1) Let f :  ®  be defined by f (–1) = |–1| = 1 = f (1) and –1 ¹ 1


f(x) = 2x + 1 for all x Î (3) Define f :  ®  by f (x) = x + 1 for all x Î. Then
2

f is neither an injection nor a surjection. It is not an


Then, for any element y in the co-domain , we have
injection, since
(y - 1)/2 is in the domain  and
f (–1) = (-1)2 + 1 = 2 = 12 + 1= f (1) and –1 ¹ 1
æ y - 1ö 2( y - 1)
fç = + 1= y
è 2 ÷ø 2 f is not a surjection, since we cannot find an element
x in  such that x2 + 1 = 0; that is f (x) = 0.
Therefore f is a surjection. Note that f is an injection (4) Define f :  ®  by f(x) = x + 2 for all x Î . Then
also, since f is an injection and it is not a surjection, since we
f ( x) = f ( y) Þ 2 x + 1 = 2 y + 1 Þ x = y cannot find an integer x such that f(x) = 1/2. Note that
f (x) = x + 2 is always an integer for any integer x.
(2) Let  be the set of all non-negative integers. Define
f :  ®  by f (x) = | x | for all x Î. Then f is a
surjection, since f (x) = x for all x Î and  Í .
However, f is not an injection since

T H E O R E M 1 .29 Let f : A ® B and g : B ® C be functions. Then the following hold:


1. If f and g are surjections, then so is g f.
2. If g f is a surjection, then g is a surjection.
PROOF 1. Suppose that f and g are surjections. Also g f is a function from A into C. The domain of g f is
A and the co-domain of g f is C. Now,
c ÎC Þ c = g(b) for some b Î B (since g is a surjection)
Þ f (a) = b and g(b) = c fo
or some a Î A and b Î B (since f is a surjection)
Þ a Î A and ( g f )(a) = g( f (a)) = g(b) = c
Þ ( g f )(a) = c for some a Î A
Thus g f is a surjection.
www.jeeneetbooks.in
1.6 Functions 43

2. Suppose g f is a surjection. To prove that g : B ® C is a surjection, let c Î C. Since g f : A ® C


is a surjection, there exists a Î A such that (g f )(a) = c. Then f(a) Î B and
g( f (a)) = (g f )(a) = c
Thus g is a surjection. ■

Note that g f can be a surjection without f being a surjection. This is substantiated in the following.

Example

Define f :  ®  by f(x) = [2x] for x Î and g :  ®  by In this case g f is a surjection, since, for any n Î, n/2 Î
g(x) = [x] for all x Î, where [x] is the integral part of x and
(i.e., [x] is the largest integer £ x). Then g f :  ®  is
given by æ nö é n ù
( g f ) ç ÷ = ê 2 × ú = [n] = n
è 2ø ë 2 û
( g f )( x) = g( f ( x)) = [[2 x]] = [2 x]
However f is not a surjection, since f(x) is always an
integer and we cannot find x Î such that f (x) = 1/2.

It is a convention that, when f : A ® B is a surjection, we often denote this by saying “f is a function of A onto B”
or f is a surjection of A onto B. We use the word onto only in the case of surjections. Whenever we want to mention that
f : A ® B is a surjection, we say that f is a surjection (or surjective function or onto function) of A onto B.

DEFIN IT ION 1 . 34 Bijection A function f : A ® B is said to be a bijection or a one-one and onto function or a
one-to-one function if f is both injective and surjective.

Examples

(1) For any set X, define I : X ® X by I(x) = x for all Also, f is surjective, since, for any y Î,
x ÎX. Then clearly I is an injection and a surjection, y-b æ y - bö æ y - bö
Î  and fç = aç +b= y
è a ÷ø è a ÷ø
and hence a bijection. This is called the identity
a
function on X or identity map on X. To specify the set
X also, we denote the identity function I on X by IX. Thus, f is a bijection of  onto itself.
(2) Define f :  ®  by f (x) = x + 3 for all x Î. Then f is (4) Let E be the set of all even integers and  the set of
a bijection of  onto  (the term “onto” is used, since all integers. Define f : E ®  by
any bijection is necessarily a surjection).
ìï2 y if x = 4 y
(3) For any real numbers a and b with a ¹ 0, define f ( x) = í
f :  ®  by îï y if x = 2 y and y is odd
f (x) = ax + b for all x Î Then f is a bijection. One can verify that
Then f is an injection, since f (0) = 0 f (- 2) = - 1
f ( x) = f ( y) Þ ax + b = ay + b Þ ax = ay f ( 2) = 1 f (- 4) = - 2
Þ x = y (since a ¹ 0) f ( 4) = 2
f (6) = 3 f (- n) = - f (n)
f (8) = 4

Try it out
T H E O R E M 1 .30 Let f : A® B be any function. Then
IB f = f = f IA
www.jeeneetbooks.in
44 Chapter 1 Sets, Relations and Functions

T H E O R E M 1 .31 If f : A ® B and g : B ® C are bijections, then g f : A ® C is a bijection.


PROOF This is an immediate consequence of Theorems 1.28 [part (1)] and 1.29 [part (1)], since a bijection
is both an injection as well as a surjection. ■

In the following, we give a characterization property for bijections.

T H E O R E M 1 .32 Let f : A ® B be a mapping. Then f is a bijection if and only if there exists a function g : B ® A
such that
g f = IA and f g = IB
that is, g( f (a)) = a for all a Î A and f (g(b)) = b for all b Î B.
PROOF If there is a function g : B ® A such that
g f = IA and f g = IB
then, by Theorem 1.28 [part (2)], f is an injection (since g f = IA which is an injection. Also, by
Theorem 1.29 [part (2)], f is a surjection (since f g = IB which is a surjection). Thus f is a bijection.
Conversely suppose that f is a bijection. Define g : B ® A as follows:
g(b) = The pre-image of b under f
That is, if f (a) = b, then g(b) is defined as a. First observe that every element b Î B has a
pre-image a Î A under f (since f is a surjection). Also, this pre-image is unique (since f is an injection).
Therefore g is properly defined as a function from B into A. Now, for any a Î A and b Î B, we have
( g f )(a) = g( f (a)) = a
since a is the pre-image of f(a) and
( f g )(b) = f ( g(b)) = b
since g(b) = a if f(a) = b. Thus g f = IA and f g = IB . ■

DEF IN IT ION 1 . 35 Inverse of a Bijection Let f : A ® B and g : B ® A be functions such that g f = IA and
f g = IB. Then both f and g are bijections (by the above theorem). Also, g is unique such that
g f = IA and f g = IB, since, for any a Î A and b Î B, we have
f (a) = b Û g( f(a)) = g(b) Û a = g(b)
The function g is called the inverse function of f and f is called the inverse function of g. Both
f and g are interrelated by the property
f (a) = b Û a = g(b)
for all a Î A and b Î B. The inverse function of f is denoted by f -1. When we look at f as a
relation, then f -1 is precisely the inverse relation as defined in Definition 1.26.

To confirm that f is a bijection, the existence of g satisfying both the properties g f = IA and f g = IB are necessary.
Just g f = IA may not imply that f is a bijection. In this context, we have the following two results.

T H E O R E M 1 .33 Let f : A ® B be a function. Then f is an injection if and only if there exists a function g : B ® A
such that g f = IA.

PROOF If g : B ® A is a function such that g f = IA, then by Theorem 1.28 [part (2)], f is an injection, Conversely
suppose that f is an injection. Choose an arbitrary element a0 Î A and define g : B ® A as follows:
ìïa if b = f (a) for some a Î A
g(b) = í
îïa0 if b Î
/ Range ( f )
www.jeeneetbooks.in
1.6 Functions 45

Recall that Range( f ) = { f(a) | a Î A} Í B. Since f is an injection, there can be at most one a Î A for
any b Î B such that f (a) = b. Therefore, g is a well-defined function from B into A. Also, for any a Î A,
( g f )(a) = g( f (a)) = a
and hence g f = IA. ■

T H E O R E M 1 .34 Let f : A ® B be a function. Then f is a surjection if and only if there exists a function g : B ® A
such that f g = IB.
PROOF If there is a function g : B ® A such that f g = IB, then, by Theorem 1.29 [part (2)], f is a surjection.
Conversely, suppose that f is a surjection. Then each element b in B has a pre-image a in A [i.e.,
a is an element in A such that f (a) = b]. Now, for each b Î B, choose one element ab in A such that
f (ab) = b. Define g : B ® A by
g(b) = ab for each b Î B

Then g is a function from B into A and, for any b Î B, we have


( f g )(b) = f ( g(b)) = f (ab ) = b
Therefore f g = IB. ■

DEF IN IT ION 1 . 36 Real-Valued Function If f : A ® B is a function and a Î A then the image f(a) is also called
a value of f at a. If the value of f at each a Î A is a real number, then f is called a real-valued
function on A; that is, any function from a set A into a subset of the real number system  is
called a real-valued function on A.

If f : A ® B is a function and B Í C, then f can be treated as a function from A into C as well. Therefore, a real-valued
function on A is just a function from A into .

QUICK LOOK 6

Let f and g be real valued functions on a set A. Then we 4. ( f × g)(a) = f (a)f (b)
define the real-valued functions f + g, - f, f - g and f × g
on A as follows: Note that the operation symbols are those in the real
number system . Also, if g(a) ¹ 0 for all a ÎA, then
1. ( f + g)(a) = f (a) + g(a) the function f /g is defined as follows:
2. ( -f )(a) = - f (a)
5. ( f /g)(a) = f (a)/g(a) for all a ÎA
3. ( f - g)(a) = f (a) - g(a)

Examples

(1) Let f be a polynomial over , that is Then f is a real-valued function on .


f = a0 + a1 x + a2 x2 + + an xn (3) Define f : [0, 2p ] ®  by
f (a) = sin a for all a Î 
where a0, a1, a2, ¼, an are all real numbers. For any
a Î, let us define Then f is a real-valued function defined on [0, 2p]
and is denoted by sin.
f (a) = a0 + a1a + a2 a + + an a
2 n

(4) Define f : + ®  by
Then f :  ®  is a real-valued function on  and is
called a polynomial function. f (a) = a for all a Î +
(2) Define f :  ®  by This is a real-valued function defined on +. Here a
f (a) = ea for all a Î  stands for the positive square root of a.
www.jeeneetbooks.in
46 Chapter 1 Sets, Relations and Functions

We have earlier made use of the notation [x] to denote the largest integer ≤ x and called it the integral part of x.
Now, we shall formally define this concept before going on to prove certain important properties.

DEF IN IT ION 1 . 37 For any real number x, the largest integer less than or equal to x is called the integral part of x
and in denoted by [x]. The real number x - [ x] is called the fractional part of x and is denoted
by { x }.

Note that, for any real number x, [x] is an integer and { x } is a real number such that
x = [ x] + {x} and 0 £ {x} < 1
Also, this expression of x is unique in the sense that, if n is an integer and a is a real number such that x = n + a
and 0 £ a < 1 , then n = [x] and a = {x}.

Examples
é5ù ì5ü 5 é - 11 ù ì - 11 ü 9
(1) ê ú = 0 and í ý = (4) ê ú = - 2 and í ý=
ë6û î6 þ 6 ë 10 û î 10 þ 10
(2) For any 0 £ a < 1, [a] = 0 and {a} = a
é 1ù ì -1 ü 3
(3) ê - ú = - 1 and í ý =
ë 4û î4þ 4

T H E O R E M 1 .35 The following hold for any real number x.


1. [x] £ x < [x] + 1
2. x - 1 < [x] £ x
3. 0 £ { x } = x - [x] < 1
4. [ x] = å 1£ i £ x i, if x > 0
5. [ x] = x Û x Î  Û {x} = 0
6. {x} = x if and only if [ x] = 0
ìï 0 if x is an integer
7. [ x] + [- x] = í
îï- 1 if x is not an integer
PROOF (1) through (6) are all straight-forward verifications using the definition that [x] is the largest
integer n such that n £ x and that x - [ x] = {x}.
To prove (7), let [ x] = n. Then n £ x < n + 1 and therefore
-n - 1 < -x £ -n

If x is an integer, then so is –x and hence [ x] + [-x] = x + (-x) = 0. If x is not an integer, then –x is


also not an integer and therefore
-n - 1 < -x -n
So [-x] = -n - 1 and hence [ x] + [-x] = n + (-n - 1) = -1. ■

Examples

é 9ù é9ù é 6 ù é -6 ù
(1) ê - ú + ê ú = - 2 + 1 = - 1 (3) ê ú + ê ú = 1 + (- 2) = - 1
ë 5û ë5û ë5û ë 5 û

(2) [- 3] + [3] = - 3 + 3 = 0 é -7 ù é 7 ù
(4) ê ú + ê ú = - 1 + 0 = - 1
ë 8 û ë8û
www.jeeneetbooks.in
1.6 Functions 47

T H E O R E M 1 .36 The following hold for any real numbers x and y:


ìï[ x] + [ y] if {x} + { y} < 1
1. [ x + y] = í
îï[ x] + [ y] + 1 if {x} + {y} ³ 1
2. [ x + y] ³ [ x] + [ y] and equality holds if and only if {x} + { y} < 1
3. If x or y is an integer, then [ x + y] = [ x] + [ y]
PROOF 1. Let x = n + r and y = m + s, where n and m are integers, 0 £ r < 1 and 0 £ s < 1. Then [x] = n,
{ x } = r, [y] = m and {y} = s. Now,
x + y = [ x] + [ y] + ({x} + { y})
and 0 £ {x} + { y} < 2
Therefore
ìï[ x] + [ y] if {x} + { y} < 1
[ x + y] = í
ïî[ x] + [ y] + 1 if {x} + {y} ³ 1
2. This is a consequence of (1).
3. This is a consequence of (2) and the fact that x is an integer if and only if { x } = 0. ■

Examples

é8ù é9ù Note that


(1) ê ú + ê ú = 1 + 1 = 2
ë5û ë5û ì 7 ü ì 6 ü 3 1 19
í ý+ í ý = + = <1
é 8 9 ù é 17 ù é8ù é9ù î 4 þ î 5 þ 4 5 20
and êë 5 + 5 úû = êë 5 úû = 3 = êë 5 úû + êë 5 úû + 1
é8 ù é 23 ù é8ù
(3) ê + 5ú = ê ú = 7 = 2 + 5 = ê ú + [5]
Note that ë3 û ë3û ë3û
é 7 17 ù é 24 ù é 7 ù é 17 ù
ì8 ü ì9 ü 3 4 7 (4) ê + ú = ê ú = 4 = 1 + 2 + 1 = ê ú + ê ú + 1
í ý+ í ý = + = > 1 ë6 6 û ë 6 û ë6û ë 6 û
î5þ î5þ 5 5 5
Note that
é 7 6 ù é 51 ù é7ù é6ù
(2) ê + ú = ê ú = 2 = 1 + 1 = ê ú + ê ú ì 7 ü ì 17 ü 1 5
ë 4 5 û ë û
20 ë4û ë5û í ý+ í ý = + = 1
î6 þ î 6 þ 6 6

T H E O R E M 1 .37 The following hold for any real number x and any non-zero integer m:
é x ù é [ x] ù
1. ê ú = ê ú
ëmû ë m û
2. If n and k are positive integers and k > 1, then

é n ù é n + 1 ù é 2n ù
êë k úû + êë k úû £ êë k úû

PROOF 1. Let [x] = n. Then x = n + r, 0 £ r < 1 (where r = { x }). Let m > 0. By division algorithm, we have
n = qm + s, q, s Î  and 0£s<m
Alternately
n s s
=q+ , 0£ <1
m m m
www.jeeneetbooks.in
48 Chapter 1 Sets, Relations and Functions

Therefore,
é [ x] ù é n ù
êë m úû = êë m úû = q

Also,
x n+r n r s+r
= = + = q+ , 0 £ s+r < s+1£ m
m m m m m
and therefore
éxù é [ x] ù
êë m úû = q = êë m úû

Similar technique proves this when m < 0 also.


2. Let n and k be positive integers and k > 1. Let
énù
êë k úû = m

Then
n
= m + r, 0 £ r < 1
k
Therefore
n+1 1 2n
= m+r + and = 2 m + 2r
k k k
Now,
é n ù é n + 1 ù ì2m if r + (1/ k ) < 1
êë k úû + êë k úû = í2 m + 1 if r + (1/ k ) ³ 1
î
é 2n ù ì2m if 2r < 1
and êë k úû = í2 m + 1 if 2r ³ 1
î
Note that, if r + (1/ k ) ³ 1, then 2r ³ 2 - (2/k ) ³ 1 (∵ k ³ 2). Thus
é n ù é n + 1 ù é 2n ù
êë k úû + êë k úû £ êë k úû

Examples
é 16 ù é 17 ù é 2 ´ 16 ù é 13 ù é 14 ù é 2 ´ 13 ù
(1) ê ú + ê ú = 0 + 0 = 0 < 1 = ê (3) ê ú + ê ú = 0 + 0 = 0 < 1 = ê
ë 25 û ë 25 û ë 25 úû ë 15 û ë 15 û ë 15 úû
é7ù é8ù é2 ´ 7ù é -21 ù é -21 + 1 ù é -21 ´ 2 ù
(2) ê ú + ê ú = 0 + 1 = 1 = ê (4) ê +ê = (-1) + (-1) = -2 = ê
ë8 û ë8û ë 8 úû ú
ë 29 û ë 29 û ú ë 29 úû

DEF IN IT ION 1 . 38 Let A be a subset of  and f : A ®  be a function. A positive real number p is called a period
of f if f (x) = f (x + p) whenever x and x + p Î A. A function with a period is called a periodic
function.

Note that, if p is a period of a function f :  ® , then np is also a period of f for any positive integer n, since for any x Î,
f ( x) = f ( x + p) = f ( x + 2 p) =
www.jeeneetbooks.in
1.7 Graph of a Function 49

Examples

(1) Define the function f :  ®  by (3) The function f :  ® , defined by f (x) = c, for all
x Î, where c is a given constant, is a periodic func-
f ( x) = {x}, the fractional part of x tion. Infact, every positive real number is a period
Note that any real number x can be uniquely of this.
expressed as x = n + r, where n is an integer and (4) The function f :  ®  defined by
0 £ r < 1 and this r is the fractional part of x denoted
by {x} and this n is the integral part of x denoted by f (x) = [x] (the integral part of x)
[x]. If x = n + r, then x + 1 = (n + 1) + r and hence
is not a periodic function. Note that
f ( x) = {x} = {x + 1} = f ( x + 1)
[x + n] = [x] + n
for all x Î. Thus, 1 is a period of f and hence every
for all x Î and for all integers n.
positive integer n is a period of f. Therefore, f is a
periodic function.
(2) We will be learning later in Vol. II that functions like
sin x, cos x, cosec x, etc. are all periodic functions and
2p is a period of all these.

1.7 | Graph of a Function


A function f from a set A into a set B is a relation from A to B; that is, f Í A ´ B and hence it can be represented as
a subset of the Cartesian product A ´ B graphically. In particular, when the function is a real-valued function defined
on the real number system or a subset of , we can plot the point (a, f (a)) on the coordinate plane by treating the
x-axis as the domain and the y-axis as the co-domain of the function. This type of representation facilitates a better
insight into understanding various properties of the function. First, let us have the formal definition of the graph in
the following.

DEF IN IT ION 1 . 39 Graph of a Function Let f : A ® B be a function. Then the graph of f is defined as the set
of all ordered pairs whose first coordinate is an element a of A and the second coordinate is
the image of a under f. This is denoted by Graph ( f ). That is,

Graph ( f ) = {(a, f (a)) | a Î A}

Note that the graph of a function f : A ® B is a subset of the Cartesian product A ´ B. For each a Î A, there is exactly
one ordered pair in Graph ( f ) with a as the first coordinate. In the following, we shall provide graphs of certain
important functions and draw diagrams of these.

Example 1.28

Let f :  ®  be defined by f(x) = x for all x Î. (Recall Y=


that f is called the identity function on  and is denoted
by IR .) What is the graph of f ? Sketch the same.
b (b, b)
Solution: The graph of f is

{(a, f (a)) | a Î } = {(a, a) | a Î } a


(a, a)
This is known as the diagonal relation on . As shown in
Figure 1.30, it is a straight line passing through the origin, X=
contained in the first and third quadrants and bisecting O a b
the right angle XOY.
FIGURE 1.30 Example 1.28.
www.jeeneetbooks.in
50 Chapter 1 Sets, Relations and Functions

Example 1.29

For any given real numbers m and c, let us define the Y=
function f :  ®  by
f (x) = mx + c for all x Î
Sketch the graph of the same.
Solution: The graph of f is
{(x, mx + c) | x Î} (0, c)

As shown in Figure 1.31, this is a straight line whose


X=
slope is m and the intercept on the y-axis is c. If we take O
m = 1 and c = 0, we get the identity function given in
Example 1.28.
FIGURE 1.31 Example 1.29.

Example 1.30

Sketch the graph for m = 0 in Example 1.29. Y=

Solution: If m = 0 in Example 1.29, then we get


f (x) = c for all x Î
This is called the constant function with image c. (0, c)
The graph of f is a straight line parallel to the x-axis
(Figure 1.32). X=
O

FIGURE 1.32 Example 1.30.

Example 1.31

Define f :  ®  by Y=

f (x) = | x | for all x Î


y=

x
y=
-x

Sketch the graph of f.

Solution: The given function is


X⬘ X=
ìï x if x ³ 0 O
f ( x) = í
îï- x if x < 0
The graph of f is
{( x, x)| x ³ 0} È {( x, - x)| x < 0}
FIGURE 1.33 Example 1.31.
This is the combination of two straight lines: one passing
and contained in the
through the origin, bisecting XOY
first quadrant and the second passing through the origin,
and contained in the second quadrant
bisecting X ¢OY
(Figure 1.33).
www.jeeneetbooks.in
1.7 Graph of a Function 51

Example 1.32

Let f :  ®  be defined by f(x) = [x] for all x Î, where Y= 


[x] is the largest integer ≤ x. For example
æ 1ö
f ç 1 ÷ = 1, f (-2.5) = - 3, f (2.5) = 2
è 2ø
f (2) = 2, f (-4.2) = - 5, f (5.01) = 5
X =
O
f (3.9) = 3, f (-8.9) = - 9, f (-6.01) = - 7
Sketch the graph of f.

Solution: The graph of f is ∪ nÎ ([n, n + 1) ´ {n}) and


is given in Figure 1.34. This function is called the step
function. The graph of f restricted to an interval [n, n + 1), FIGURE 1.34 Example 1.32.
with n an integer, is a line segment parallel to the x-axis.

Example 1.33

Let f :  ®  be defined by This function is called the signum function. The graph
of this f is in three parts: one is the line y = 1 which is
ì 0 if x = 0 parallel to x-axis and contained in the first quadrant;
ï
f ( x) = í | x | the second is the origin (0, 0) and the third is the line
ï if x ¹ 0 y = –1 which is parallel to the x-axis and contained in the
îx third quadrant (Figure 1.35).
Sketch the graph of this function. Y=

Solution: We have
y =1
ì 0 if x = 0
ï
f ( x) = í | x |
ï if x ¹ 0 X=
îx O
y = -1
Then
ì-1 if x < 0
ï
f ( x) = í 0 if x = 0
ï
î 1 if x > 0 FIGURE 1.35 Example 1.33.

Example 1.34

Let f :  ®  be defined by Y =

ì1 - x if x < 0
ï
f ( x) = í 1 if x = 0
ï
î1 + x if x > 0 M L
P ( 0, 1 )
Sketch the graph for this function.
O X =
Solution: Note that f (x) = 1 + | x | for all x Î. The graph
of f is given by
FIGURE 1.36 Example 1.34.
{( x, 1 + x) x > 0} È {(0, 1)} È {( x, 1 - x)| x < 0}
www.jeeneetbooks.in
52 Chapter 1 Sets, Relations and Functions

This is in three parts: one is the straight line bisecting bisecting MPY and contained in the second quadrant
and contained in the first quadrant, the second
LPY (Figure 1.36).
is the point P = (0, 1) and the third is the straight line

DEF IN IT ION 1 . 40 Let A be a subset of  and f : A ®  be a function. Then we say that f is increasing if
f (x) £ f (y) whenever x £ y. f is said to be decreasing if f (x) ³ f (y) whenever x £ y.

Example 1.35

Let 1 < a Î and define f :  ®  by f (x) = ax for all x Î. Y =


Sketch the graph of f.

Solution: Since a > 1, f is an increasing function.


The graph of f is a curve which goes upward when x y = a x, a > 1
increases [i.e., f (x) increases when x increases] and goes
downwards when x decreases [i.e., f (x) decreases when
x decreases]. Also, since a > 1, a is positive and hence
ax is positive for all x. This implies that the graph of (0, 1)
f (x) = ax is contained in the first and second quadrants
X =
(Figure 1.37). O

FIGURE 1.37 Example 1.35.

Example 1.36

Let 0 < a < 1 and define f :  ®  by f (x) = ax for all x Î. y = a x, 0 < a < 1 Y =
Sketch the graph of f. Here, f(x) decreases as x increases
(since 0 < a < 1) and hence f is a decreasing function. The
graph of f is the curve shown in Figure 1.38. The curve
cuts the y-axis at (0, 1). Also, since a > 0, ax > 0 for all
x Î. Therefore, the graph of f is contained in the first
and second quadrants only.

(0, 1)

X =
O

FIGURE 1.38 Example 1.36.

Example 1.37

Let f :  ®  be a periodic function with a period p. Y= 


What would the graph of this function look like?

Solution: In this case, the graph of f between the lines


x = 0 and x = p is similar to that between the lines x = p X= 
and x = 2p. For example, consider the function f :  ®  -2 -1 0 1 2 3
defined by
f (x) = {x}, the fractional part of x
This is a periodic function with 1 as a period. The graph FIGURE 1.39 Example 1.37.
of this function is as shown in Figure 1.39. Note that
0 £ f (x) < 1 for all real numbers x.
www.jeeneetbooks.in
1.8 Even Functions and Odd Functions 53

1.8 | Even Functions and Odd Functions


If we consider the function f :  ®  defined by f (x) = x2, then we have f (x) = f (-x). Functions satisfying this property
are called even functions. If f is a real-valued function such that f (x) = -f (x) for all x, then f is called an odd function.
In this section we discuss certain elementary properties of even and odd functions. We shall begin with a formal defini-
tion in the following.

Even Functions
DEF IN IT ION 1 . 41 Symmetric Set A subset X of the real number system  is said to be a symmetric set if
x Î X Û -x Î X

Examples

(1) The interval [–1, 1] is a symmetric set, since -1 £ x £ 1 (4) The sets {0}, {–1, 1}, {–1, 0, 1} are symmetric.
if and only if -1 £ -x £ 1. (5) [-2, -1] È [1, 2] is a symmetric set.
(2) The interval [0, 1] is not symmetric.
(3) The set  of integers, the set  of rational numbers
and the whole set  are all symmetric sets.

DEF IN IT ION 1 . 42 Even Function Let X be a symmetric set and f : X ®  a function. Then f is said to be an
even function if
f (-x) = f (x) for all x Î X

Examples

(1) If f :  ®  is the function defined by f (x) = x 2 for all (3) Any constant function f :  ®  is even, that is, for
x Î, then f is an even function, since, for any x Î, any c Î, the function f :  ® , defined by f (x) = c
for all x Î, is even.
f (-x) = (-x)2 = x2 = f (x)
(4) The function f : [-p, p] ® , defined by f (x) = cos x
(2) The function f :  ® , defined by f (x) = | x | for all for all -p £ x £ p, is an even function, since cos(-x) =
x Î, is even, since cos x.
f (-x) = | -x | = | x | = f (x) for all x Î

Graphs of Even Functions


The graph of an even function is symmetric about the y-axis, in the sense that, when y-axis is assumed as plane mirror, the
graph in the left part is the image of the right part. Equivalently, if the graph is rotated through 180o about the y-axis, we
get the appearance of the graph as original. Figure 1.40 shows the graphs of the even functions given in the example above.

Odd Functions

DE F IN IT ION 1 . 41 Odd Function Let X be a symmetric set. A function f : X ®  is said to be an odd function if
f (-x) = - f (x) for all x Î X
www.jeeneetbooks.in
54 Chapter 1 Sets, Relations and Functions

Y= Y=

O X= O X=
(a) (b)

Y= Y=

(0, 1)

X=
(0, 0)
(0, c)
(0, -1)

O X=
(c) (d)
FIGURE 1.40 Graphs of the functions: (a) f (x) = x ; (b) f (x) = | x |; (c) f (x) = c; (d) f (x) = cos x.
2

Examples

(1) The identity function f :  ® , defined by f (x) = x (3) Define f : [-p, p] ®  by f (x) = sin x for all -p £ x £
for all x Î, is an odd function, since f (-x) = -x = p. Then f is an odd function, since f (-x) = sin(-x) =
-f (x) for all x Î. -sin x = -f (x) for all x Î[-p, p].
(2) In general, for any integer n, the function f :  ® , (4) Define f : (-p / 2, p / 2) ®  by f (x) = tan x for all
defined by f (x) = x2n+1, is an odd function, since -p / 2 < x < p / 2. Then f is an odd function, since
f (-x) = (-x)2n+1 = -x2n+1 = -f (x) for all x Î. tan(-x) = -tan x for all x Î(-p / 2, p / 2).

Note: If f is an odd function defined on a symmetric set S containing 0, then necessarily f (0) = 0, for f (0) = f (-0) = -f (0).
Hence 2 f (0) = 0, so that f (0) = 0.

Graphs of Odd Functions


The graph of an odd function is symmetric about the origin. If the graph is rotated through 180o, either clockwise or
anticlockwise, about the origin, the resulting figure gives the same appearance as original. Figure 1.41 gives the graphs
of the odd functions given in the above example.
www.jeeneetbooks.in
1.8 Even Functions and Odd Functions 55

Y Y

X X
O O

(a) (b)

( p /2, 1)

X
p -p /2 0 p /2 p

( -p /2, 1)

(c)

X
-p /2 0 p /2

(d)
FIGURE 1.41 Graphs of the functions: (a) f ( x) = x; (b) f ( x) = x3 ; (c) f(x) = sin x; (d) f(x) = tan x.
www.jeeneetbooks.in
56 Chapter 1 Sets, Relations and Functions

Remark: Unlike in integers, a function can be neither even nor odd. For example, consider the function f :  ® 
defined by f (x) = x2 + x + 1 for all x Î . Then f (-1) = 1 and f (1) = 3 and hence
f (–1) ¹ f (1) and f(–1) ¹ – f (1)
Therefore f is neither even nor odd. Next, note that a function f is both even and odd if and only if f ( x) = 0 for all x.

Examples

(1) Define f :  ®  by f (x) = ex + e-x for all x Î . Then = 4 1 + x3 - 4 1 - x3


f (-x) = e-x + e-(-x) = ex + e-x = f (x) for all x Î  and
therefore f is an even function. = - ( 4 1 - x3 - 4 1 + x3 )
(2) Define f : [-1, 1] ®  by f ( x) = 4 1 - x3 - 4 1 + x3 for = - f ( x)
all -1 £ x £ 1. Then
for all x Î[-1, 1]. Therefore f is an odd function.
f (- x) = 1 - (- x) - 1 + (- x)
4 3 4 3

T H E O R E M 1 .38 Let X be a symmetric set and f and g functions of X into . Then, the product fg is an even function
if both f and g are even or both f and g are odd.
PROOF Suppose that both f and g are even functions. Then, for any x Î X, we have
( fg )( x) = f ( x)g( x) = f (- x)g(- x) = ( fg )(- x)
and hence fg is an even function. One the other hand, suppose that both f and g are odd functions.
Then, for any x Î X, we have
( fg)(- x) = f (- x)g(- x) = (- f ( x))(- g( x)) = f ( x)g( x) = ( fg)( x)
and therefore fg is an even function. ■

T H E O R E M 1 .39 For any real-valued functions f and g defined on a symmetric set X, the product fg is an odd
function if one of f and g is odd and the other is even.
PROOF Note that fg = gf, since rs = sr for any real numbers r and s. Without loss of generality, we can
suppose that f is even and g is odd. Then, for any x Î X, we have
( fg )(- x) = f (- x)g(- x) = f ( x)(- g( x)) = -( f ( x)g( x)) = -( fg )( x)
Therefore fg is an odd function. ■

T H E O R E M 1 .40 Let f be a real-valued function on a symmetric set X. Then the following hold:
1. f is even if and only if af is even for any 0 ¹ a Î .
2. f is odd if and only if af is odd for any 0 ¹ a Î .
3. f is even (odd) if and only if –f is even (odd).
PROOF 1. Let us recall that for any a Î  the function af is defined by (af )(x) = af (x) for all x Î X. Suppose
that f is even. Then, for any a Î  and x Î X,
(af )(-x) = af (-x) = af (x) = (af )(x)
and hence af is even. Conversely, suppose that 0 ¹ a Î  such that af is even. Then, for any x Î X,
we have
af (- x) = (af )(- x) = (af )(x) = af (x)
Now, since a ¹ 0, f (-x) = f (x). Therefore, f is even.
www.jeeneetbooks.in
1.8 Even Functions and Odd Functions 57

2. It can be proved similarly.


3. It is a simple consequence of (1) and (2); take a = 1 in (1) and (2). ■

T H E O R E M 1 .41 If f and g are even (odd), then so is f ± g.


PROOF Suppose that f and g are even. Then, for any x Î X, we have
( f + g )(- x) = f (- x) + g(- x) = f ( x) + g( x) = ( f + g )( x)
Therefore f + g is even. This together with the above theorem implies that f - g is also even.
Similarly, we can prove that, if f and g odd, then so is f ± g. ■

T H E O R E M 1 .42 Any function can be expressed as a sum of an even function and an odd function.

PROOF Let f : X ®  be a function whose domain X is a symmetric set. Define g : X ®  and h : X ®  by

f ( x) + f (- x) f ( x) - f (- x)
g( x) = and h( x) =
2 2
for all x Î X . Then
f (- x) + f (-(- x)) f ( x) + f (- x)
g(- x) = = = g( x)
2 2
f (- x) = f (-(- x)) f (- x) - f ( x)
and h(- x) = = = - h( x)
2 2
for all x Î X . Therefore, g is an even function and h is an odd function. Also, for any x Î X,
f ( x) + f (- x) f ( x) - f (- x)
g( x) + h( x) = + = f ( x)
2 2
and hence f = g + h. ■

Note: The above representation of f is unique in the sense that if g + h = f = a + b, where g and a are even and h and
b are odd, then g = a and h = b; for, in this case g - a = b - h, which is both even and odd. Therefore, g - a = 0 = b - h
or g = a and h = b.
The unique functions g and h given in the proof of Theorem 1.42 are called the even extension of f and odd
extension of f, respectively.

Examples

(1) Let f :  ®  be defined by Then g is even, h is odd and f = g + h. Note that

f ( x) = x2 + 2 x + 1 = ( x + 1)2 f ( x) + f (- x) f ( x) - f (- x)
= g( x) and = h( x)
2 2
Note that f is neither even nor odd, since
(2) Consider the function f :  ®  defined by
f (-1) = (-1)2 + 2(-1) + 1 = 0
f (x) = ex for all x Î
and f (1) = (1)2 + 2(1) + 1 = 4
Then f = g + h, where
Therefore f (-1) ¹ f (1) and f (-1) ¹ -f (1). However,
consider the functions g and h defined by ex + e- x ex - e- x
g( x) = and h( x) =
2 2
g(x) = x2 + 1 and h(x) = 2x
Note that g is even and h is odd.
www.jeeneetbooks.in
58 Chapter 1 Sets, Relations and Functions

Example 1.38

Determine the even and odd extensions of the function and the odd extension of f is given by
f :  ®  given by f (x) = e-x.
f ( x) - f (- x) e- x - ex
h( x) = =
Solution: The even extension of f is given by 2 2

f ( x) + f (- x) e- x + ex
g( x) = =
2 2

WORKED-OUT PROBLEMS
Single Correct Choice Type Questions
1. If A is the set of positive divisors of 20, B is the set of Therefore,
all prime numbers less than 15 and C is the set of all
A6 Ç A10 = A30
positive even integers less than 11, then (A Ç B) È C is
(A) {2, 3, 5, 7, 8, 10} (B) {2, 4, 5, 7, 8, 10} since LCM {6, 10} = 30.
(C) {2, 4, 5, 6, 7, 8, 10} (D) {2, 4, 5, 6, 8, 10} Answer: (C)

Solution: It is given that 4. Let A = {a, b, c, d} and B = {a, b, c}. Then the number
A = {1, 2, 4, 5, 10, 20} of sets X contained in A and not contained in B is
B = {2, 3, 5, 7, 11, 13} (A) 8 (B) 6 (C) 16 (D) 12
C = {2, 4, 6, 8, 10} Solution: If X Í A and X Í B, then necessarily d Î X
Í A and hence X = Y È { d }, where Y is any subset of
Therefore
B. The number of subsets of B is 23 and therefore the
A Ç B = {2, 5} and ( A Ç B) È C = {2, 4, 5, 6, 8, 10} required number is 8.
Answer: (A)
Answer: (D)
5. Let A, B and C be three sets and X be the set of all
2. Which of the following sets is empty?
elements which belong to exactly two of the sets A, B
(A) { x Î  | x2 = 9 and 2x = 6} and C. Then X is equal to
(B) { x Î  | x2 = 9 and 2x = 4} (A) ( A Ç B) È ( B Ç C ) È (C Ç A)
(C) { x Î  | x + 4 = 4} (B) A D ( B D C )
(D) { x Î  | 2x + 1 = 3} (C) ( A È B) Ç ( B È C ) Ç (C È A)
Solution: We have x2 = 9 only if x = ±3. For this value (D) ( A È B È C ) - [ A D ( B D C )]
of x the equation 2x = 4 is not satisfied. Sets in (A), (B),
Solution: We have
and (D) are non-empty.
Answer: (B) x Î X Û x Î A Ç B and x ÏC
or x Î B Ç C and x ÏA
3. For each positive integer n, let
or x ÎC Ç A and x ÏB
An = The set of all positive multiples of n
Therefore
Then A6 Ç A10 is
X = [(A Ç B) - C] È [(B Ç C) - A] È [(C Ç A) - B]
(A) A10 (B) A20 (C) A30 (D) A60
= ( A È B È C ) - [ A D ( B D C )]
Solution: Given that An = {a Î + | n divides a}. Now
since
a Î An Ç Am Û Both n and m divide a
Û The LCM of {n, m} divides a A D ( B D C ) = ( Ac Ç Bc Ç C ) È ( A Ç Bc Ç C c ) È
Û a Î Ar , where r = LCM {n, m} ( Ac Ç B Ç C c ) È ( A Ç B Ç C )
Answer: (D)
www.jeeneetbooks.in
Worked-Out Problems 59

6. Let Ã( x) denote the power set of X. If A = {a, b, c, d, e} 10. Let S = {1, 2, 3, 4, 5, 6, 7, 8, 9} and A = {2, 3, 4, 5, 6}. Then
and B = {a, c, d, x, y}, then Ã( A Ç B) = the number of subsets B of S such that A D B = {5} is
(A) {f , {a, c}, {c, d}, {a, c, d}, {a}, {c}, {d}} (A) 1 (B) 2 (C) 3 (D) 0
(B) {f , {a}, {c}, {a, c}, {c, d}, {a, d}, {a, c, d}} Solution: For any subsets X, Y and Z of S, we have
(C) Ã( A È B) X D X = f, X D f = X
(D) Ã( A) Ç Ã( B)
and X DY =Z ÛY = X D Z
Solution: We have X Í A Ç B Û X Í A and X Í B.
Now,
Answer: (D)
A D B = {5} Û B = A D {5} = {2, 3, 4, 6}
7. Let A and B be finite sets with n(A) = m and n(B) = n.
Answer: (A)
If the number of elements in Ã(A) is 56 more than
those in Ã(B), then
11. Let A, B and C be finite sets such that A Ç B Ç C = f
(A) m = 6, n = 4 and each one of the sets A D B, B D C and C D A has
(B) m = 6, n = 3 100 elements. The number of elements in A È B È C is
(C) m = 7, n = 4 (A) 250 (B) 200 (C) 150 (D) 300
(D) m = 5, n = 3 Solution: Let n(X ) denote the number of elements in X.
Solution: It is given that Then,
n( A È B È C ) = n( A) + n( B) + n(C ) - n( A Ç B)
n(Ã( A)) = 2m = 56 + n(Ã( B)) = 56 + 2n
- n( B Ç C ) - n(C Ç A) + n( A Ç B Ç C )
Now 2m - 2n = 56 and m > n. Hence we get
= å n( A) - å n( A Ç B)
2n (2m - n - 1) = 56 = 8 ´ 7 = 23 (23 - 1)
(since A Ç B Ç C = f )
Therefore n = 3 and m - n = 3 and hence m = 6 and n = 3.
Answer: (B) Now,
A D B = ( A - B) È ( B - A) = ( A È B) - ( A Ç B)
8. If A and B are two subsets of a universal set X, then
Ac - Bc = Therefore
(A) A - B (B) (A - B) c
n( A D B) = n( A È B) - n( A Ç B)
(C) B Ç Ac (D) (B - A)c = n( A) + n( B) - 2 n( A Ç B)
Solution: We have and
A - Bc = Ac Ç ( Bc )c = Ac Ç B = B Ç Ac
c
300 = å n( A D B) = å [n( A) + n( B) - 2 n( A Ç B)]
Answer: (C)
= 2 éë å n( A) - å n( A Ç B)ùû
9. If A = {1, 2, 3, 4}, B = {1, 2, 5, 6}, C = {2, 7, 8, 9} and
D = {2, 4, 8, 9}, then (A D B) D (C D D) = Therefore
(A) {3, 4, 5, 6, 7} (B) {3, 4, 5, 7} n( A È B È C ) = å n( A) - å n( A Ç B) = 300/2 = 150
(C) {3, 5, 7, 8} (D) {3, 5, 6, 7}
Answer: (C)
Solution: We have Alternate Method
A D B = ( A - B) È ( B - A) = {3, 4, 5, 6} Draw the Venn diagram as follows:

C D D = (C - D) È (D - C ) = {7, 4} A B

and ( A D B) D (C D D) = ( A D B) - (C D D) a x b

È [(C D D) - ( A D B)]
z y
= {3, 5, 6} È {7} = {3, 5, 6, 7}}
Answer: (D) c
C
www.jeeneetbooks.in
60 Chapter 1 Sets, Relations and Functions

The shaded part is A Ç B Ç C which is given to be empty. with n + 1 elements, n > 0. If A is a non-empty subset of
Let a, b, c denote n[A – (B È C )], n[B – (C È A)], n[C – æ n + 1ö
X with K-elements (such sets are ç in number),
(A È B)] respectively. Let x, y, z denote the number è K ÷ø
of elements in (A Ç B) - C, (B Ç C) - A, (C Ç A) - B then the number of partitions of the set X - A is P( n + 1) - K .
respectively. Then For each f ¹ A Í X and for each partition of X - A,
we get a partition of X. Conversely, any partition of X
n( A È B È C ) = a + b + c + x + y + z
corresponds to a non-empty subset A of X and a parti-
We are given that tion of X - A. Therefore

100 = n( A D B) = (a + z) + (b + y)
n+1
æ n + 1ö n
æ nö
P( n + 1) = å çè
K =1 K ÷
ø
P( n + 1) - K = å ç ÷ Pr
r =0 è r ø
100 = n( B D C ) = (b + x) + (c + z)
Answer: (C)
and 100 = n(C D A) = (c + y) + (a + x)
Note: If n is a positive integer and 0 £ r £ n is an integer,
Adding the above three, we get that æ nö
then ç ÷ denotes the number of selections of n distinct
è rø
300 = 2(a + b + c) + 2( x + y + z) = 2 n( A È B È C )
objects taken r at a time (see Chapter 6).
and hence n( A È B È C ) = 150.
14. The number of equivalence relations on a five element
Answer: (C)
set is
12. Let n be a positive integer and (A) 32 (B) 42 (C) 50 (D) 52
R = {(a, b) Î  ´  | a - b = nm for some 0 ¹ m Î } Solution: Note that equivalence relations and partitions
are same in number. By Problem 13, we have
Then R is
(A) Reflexive on 
4
æ 4ö æ 4ö æ 4ö æ 4ö æ 4ö æ 4ö
P5 = å ç ÷ Pr = ç ÷ P0 + ç ÷ P1 + ç ÷ P2 + ç ÷ P3 + ç ÷ P4
(B) Symmetric r=0 è rø è 0ø è 1ø è 2ø è 3ø è 4ø
(C) Transitive Now,
(D) Equivalence relation on  æ 1ö æ 1ö
P0 = 1, P1 = 1, P2 = ç ÷ P0 + ç ÷ P1 = 1 + 1 = 2
Solution: R is not reflexive, since (2, 2) ÏR. R is sym- è 0ø è 1ø
metric, since
æ 2ö æ 2ö æ 2ö
(a, b) Î R Þ a - b = nm for some 0 ¹ m Î  P3 = ç ÷ P0 + ç ÷ P1 + ç ÷ P2 = 1 + 2 × 1 + 1× 2 = 5
è 0ø è 1ø è 2ø
Þ b - a = n(- m) and 0 ¹ - m Î 
æ 3ö æ 3ö æ 3ö æ 3ö
P4 = ç ÷ P0 + ç ÷ P1 + ç ÷ P2 + ç ÷ P3
Þ (b, a) Î R è ø
0 è ø
1 è ø
2 è 3ø
R is not transitive, since (2, n + 2) ÎR and (n + 2, 2) Î R, = 1× 1 + 3× 1 + 3× 2 + 1× 5 = 15
but (2, 2) ÏR.
æ 4ö æ 4ö æ 4ö æ 4ö æ 4ö
Answer: (B) P5 = ç ÷ P0 + ç ÷ P1 + ç ÷ P2 + ç ÷ P3 + ç ÷ P4
è 0ø è 1øø è 2ø è 3ø è 4ø
13. Let P0 = 1 and Pn be the number of partitions on = 1× 1 + 4 × 1 + 6 × 2 + 4 × 5 + 1× 155 = 52
a finite set with n elements. For n ≥ 1, a recursion
formula for Pn is given by Answer: (D)

(A) Pn = Pn - 1 + Pn - 2 for n ≥ 2 15. Which one of the following represents a function?


n-1
æ n - 1ö (A)
(B) Pn = å ç P
r =1 è r ÷ø r 1 a

n
æ nö 2 b
(C) Pn + 1 = å ç ÷ Pr
r =0 è r ø 3 c
(D) Pn + 1 = Pn + nPn - 1 4 d
Solution: We are given that P0 = 1. If X is a set with
only one element, then clearly P1 = 1. Now, let X be a set
www.jeeneetbooks.in
Worked-Out Problems 61

(B) 17. Let f :  ®  be the function defined by


1 a
ì x2 - 4 x + 3 if x < 2
f ( x) = í
2 b îx - 3 if x ³ 2

3 c Then number of real numbers x for which f ( x) = 3 is


(A) 1 (B) 2 (C) 3 (D) 4
4 d
Solution: We have
x < 2 and f ( x) = 3 Þ x2 - 4 x + 3 = 3
(C)
Þ x ( x - 4) = 0
1 a
Þ x = 0 (since x < 2)
2 b
Also x ³ 2 and f ( x) = 3 Þ x - 3 = 3 Þ x = 6. Therefore,
3 c only x = 0 or 6 satisfy f ( x) = 3.
4 d
Answer: (B)

18. Let

(D) ax
f ( x) = for x ¹ -1
1 a x+1

2 b Then the value of a such that ( f f )( x) = x for all


x ¹ -1 is
3 c (A) -1 (B) 2 (C) - 2 (D) 1
4 d Solution: We have
æ ax ö a[ax /( x + 1)]
x = ( f f )( x) = f ç ÷ =
Solution: In (A), 3 ® b and 3 ® d. It does not represent è x + 1ø [ax /( x + 1)] + 1
a function, since one element in the domain cannot be
Therefore
sent to two elements in the codomain. Similarly (B) and
(C) do not represent functions. But (D) represents a func- a2 x
x= for all x ¹ - 1
tion f, where f (1) = a, f (2) = b, f (3) = b and f (4) = d. ax + x + 1
Answer: (D)
(a + 1) x2 + (1 - a2 ) x = 0 for all x ¹ -1
16. Let A be the set of all men living in a town. Which This is a quadratic equation which is satisfied by more
one of the following relations is a function from than two values of x (infact, for all x ¹ -1). Therefore, the
A to A? coefficients of x2 and x must be both zero. Hence
(A) {(a, b) Î A ´ A | b is the son of a} a + 1 = 0 and 1 - a2 = 0
(B) {(a, b) Î A ´ A | b is the father of a}
and so
(C) {(a, b) Î A ´ A | a and b are same}
a = -1
(D) {(a, b) Î A ´ A | a is the grandfather of b}
Answer: (A)
Solution: Here (B) is not a function, since for any a Î A,
there should be exactly one b such that b is the father 19. If f (x) is a polynomial function satisfying the relation
of a. Then again there should be c Î A such that c is the
æ 1ö æ 1ö
father of b and so on. This chain breaks at some stage, f ( x) + f ç ÷ = f ( x) f ç ÷ for all x ¹ 0
where there is man a whose father is not in that town. è xø è xø
Therefore, not every element in A has an image. In (A) and f (4) = 65, then f (2) =
and (D) an element can have more than one images and
(A) 7 (B) 4 (C) 9 (D) 6
hence they do not represent a function. However, (C) is a
function; in fact, it is the identity function on A. Solution: Since f (4) = 65, f (x) must be a non-zero poly-
Answer: (C) nomial. Let
f ( x) = a0 + a1 x + a2 x2 + + an xn, an ¹ 0
www.jeeneetbooks.in
62 Chapter 1 Sets, Relations and Functions

Suppose that 9x 9
= +
æ 1ö æ 1ö 9 + 3 3(3 + 9x )
x

f ( x) + f ç ÷ = f ( x) f ç ÷ for all x ¹ 0
è xø è xø 3 × 9x + 9
= =1
Then 3(3 + 9x )

n n
ar æ n öæ n a ö Therefore,
åa x + å x
r
r
r
= ç å ar xr ÷ ç å rr ÷
è r=0 ø è r=0 x ø
r=0 r=0 2008
æ r ö é æ 1 ö æ 2008 ö ù
n
å f çè 2009 ÷ø = êë f çè 2009 ÷ø + f çè 2009 ÷ø úû + …
r =1
Multiplying throughout by x , we get that
æ n öæ n ö
1004
é æ r ö æ 2009 - r ö ù
å êë f çè 2009 ÷ø + çè
n n
=
å
r=0
ar xn + r + å ar xn - r = ç å ar xr ÷ ç å ar xn - r ÷
r=0 è r=0 ø è r=0 ø r =1
÷
2009 ø úû

æ r ö æ r ö
1004
That is, = å f çè 2009 ÷ø + f çè 1 - 2009 ÷ø
r =1
(a0 xn + a1 xn + 1 + + an x2 n ) + (a0 xn + a1 xn - 1 + + an - 1 x + an ) 1004

= (a0 + a1 x + + an x )(a0 x + a1 x n n n-1


+ + an - 1 x + an )
= å 1 = 1004
r =1

Equating the corresponding coefficients of powers of x, Answer: (A)


we have
Note: If a is any positive integer and f ( x) = a2 x/(a2 x + a),
an = a0 an, an - 1 = a0 an - 1 + a1an
then
an - 2 = a2 an + a1an - 1 + an - 2 a0
f ( x) + f (1 - x) = 1
2a0 = a02 + an2
21. Let [x] and { x } denote the integral part and fractional
an = a0 an Þ a0 = 1 (since an ¹ 0) part of x, respectively. Then the number of solutions
an - 1 = a0 an - 1 + a1an Þ a1an = 0 Þ a1 = 0 of the equation 4{ x } = x + [x] is
(A) 1 (B) 2 (C) 0 (D) infinite
an - 2 = a2 an + a1an - 1 + an - 2 a0 Þ an - 2 = a2 an + an - 2 Þ a2 = 0
Solution: Let 4{ x } = x + [x] = 2[ x ] + { x }. Therefore 3{ x }
Continuing this process, we get that an- 1 = 0 and 2 = 1 + an2 . = 2[x]. Since 0 £ { x } < 1, we have 0 £ 3{ x } < 3 and there-
Hence an = ±1. Therefore fore 0 £ 2{ x } < 3. Since 2[x] is even integer,
f ( x) = 1 ± xn 2
[ x] = 0 or 1 and {x} = 0 or
Since we are given that f (4) = 65 we have 3

65 = 1 ± 4n Therefore

Therefore f (x) cannot be 1 - xn. Thus, f (x) = 1 + xn and x = 0 or [ x] = 1


65 = 1 + 4n and hence n = 3. So f (x) = 1 + x3 and f (2) = 9. æ 2ö
x = 0 or 4 ç ÷ = x + 1
Answer: (C) è 3ø
5
20. Let f ( x) = 9 /(9 + 3 ) for all x Î. Then the value of
x x x = 0 or
3
å
2008
r =1
f (r / 2009) is Answer: (B)
(A) 1004 (B) 1005 (C) 1004.5 (D) 1005.5
22. If the function f :  ®  satisfies the relation f (x) +
Solution: Consider f (x + 4) = f (x + 2) + f (x + 6) for all x Î, then a period
of f is
9x 91- x
f ( x) + f (1 - x) = + 1- x (A) 3 (B) 7 (C) 5 (D) 8
9 +3 9 +3
x

Solution: The given relation is


9x 9
= +
9 + 3 9 + 3 × 9x
x f ( x) + f ( x + 4) = f ( x + 2) + f ( x + 6) (1.3)
www.jeeneetbooks.in
Worked-Out Problems 63

Replacing x with x - 2, we get that Therefore

f ( x - 2) + f ( x + 2) = f ( x) + f ( x + 4) (1.4) [ f ( x + a) - 1]2 = 2 f ( x) - [ f ( x)] 2 (1.5)

From Eqs. (1.3) and (1.4) we get Replacing x with x + a, we get


[ f (x + 2a) - 1]2 = 2f (x + a) - [ f (x + a)]2
f ( x - 2) = f ( x + 6) for all x Î 
or f ( x) = f ( x + 8) for all x Î  = -[ f ( x + a) - 1]2 + 1
= -[2 f ( x) - { f ( x)}2 ] + 1 [by Eq. (1.5)]
Answer: (D)
= [ f ( x) - 1] 2

23. Let A =  ´ ,  the real number system and Therefore,


R = {(( x, y), (a, b)) Î A ´ A | either x < a f ( x + 2a) - 1 = f ( x) - 1 [since f ( x + a), f ( x) ³ 1]
or x = a and y > b} f ( x + 2a) = f ( x) for all x Î 
Then which one of the following is true, if ((x, y), Thus 2a is a period of f.
(a, b)) Î R and ((a, b), ( p, q)) Î R? Answer: (A)
(A) (( x, y), ( p, q)) ÎR (B) (( x, y), (q, p)) ÎR
(C) (( x, y), ( y, q)) ÎR (D) (( y, x), ( p, q)) ÎR 25. The range of the function f defined by

Solution: Suppose that ((x, y), (a, b)), ((a, b), (p, q)) Î R. ex - e | x|
f ( x) =
Then ex + e | x|
either x < a or x = a and y > b is
and either a < p or a = p and b > q (A) [0, 1] (B) (–1, 0] (C) (0, 1) (D) [–1, 0]
If x < a and a < p, then x < p and hence ((x, y), (p, q)) Î R. Solution: Here f (x) is defined for all real x, since
Same is the case when x < a and a = p and also when x = a ex + e | x| ¹ 0 for all x Î . Also
and a < p. If x = a, y > b, a = p and b > q, then x = p and
ì0 for x ³ 0
y > b > q. Therefore ((x, y), (p, q)) Î R. ï x -x
f ( x) = í e - e e -1
2x

ï ex + e- x = e2 x + 1 for x < 0
Answer: (A)
î
24. Let a be a positive real number and f : ®  a func- Therefore
tion such that
2
f ( x + a) = 1 + 2 f ( x) - f 2 ( x) for all x Î  f ( x) = 1 - for all x < 0
e2 x + 1
Then a period of f is
For x < 0,
(A) 2a (B) 3a (C) 4a (D) 5a
2
Solution: Given f ( x + a) = 1 + 2 f ( x) - f 2( x) for all y = f ( x) Û 0 ³ y = 1 - > -1
e2 x + 1
x Î . Replacing x with x - a we get
From this it follows that the range of f is (–1, 0].
f ( x) = 1 + 2 f ( x - a) - ( f ( x - a)) and 1 £ f ( x) £ 2
2
Answer: (B)

Multiple Correct Choice Type Questions


1. Let A and B be two sets. If X is any set such that = ( B Ç A) È ( X Ç A)
A Ç X = B Ç X and A È X = B È X , then
= ( B Ç A) È ( X Ç B)
(A) B Í A (B) A Í B (C) A = B (D) A D B = f
= B Ç (A È X )
Solution: We have
= B Ç (B È X ) = B
A = (A È X ) Ç A Therefore A = B and hence all are correct answers.
= (B È X ) Ç A Answers: (A), (B), (C) and (D)
www.jeeneetbooks.in
64 Chapter 1 Sets, Relations and Functions

2. S is a set and the Cartesian product S ´ S has 9 elem- - x2 [{ f ( x)/ x2 } + 1]


ents of which two elements are (-2, 1) and (1, 2). Then = +1
( x + 1)2
(A) (2, - 2) Î S ´ S (B) (-2, - 2) Î S ´ S
-[ f ( x) + x2 ]
(C) (-2, 2) Ï S ´ S (D) S = {-2, 1, 2} = +1
( x + 1)2
Solution: S ´ S has 9 = 32 elements and hence S must
have 3 elements. Since (-2, 1) and (1, 2) Î S ´ S, we have Therefore
-2, 1, 2 Î S and therefore S = {-2, 1, 2}. Therefore (2, æ 1 ö ( x + 1)2 - x2 - f ( x)
-2) Î S ´ S and (-2, -2) Î S ´ S. fç = (1.7)
è x + 1÷ø ( x + 1)2
Answers: (A), (B) and (D)
From Eqs. (1.6) and (1.7), we get
3. Let f :  ®  be a function satisfying the following:
f ( x) + 1 = ( x + 1)2 - x2 - f ( x) = 2 x + 1 - f ( x)
(a) f (- x) = - f ( x)
Therefore, 2 f ( x) = 2 x and hence f ( x) = x for all x Î .
(b) f ( x + 1) = f ( x) + 1
Answers: (A), (B), (C) and (D)
æ 1 ö f ( x)
(c) f ç ÷ = 2 for all x ¹ 0
è xø x 4. If a ¹ b Î  and f :  ®  is a function such that

Then æ 1ö
af ( x) + bf ç ÷ = x - 1 for all 0 ¹ x Î 
(A) f ( x) = x for all x Î  è xø
(B) f (x + y) = f (x) + f (y) for all x, y Î  Then
(C) f (xy) = f (x)f (y) for all x, y Î  2a + b
(A) f (2) = (B) f (1) = 0
æ x ö f ( x) 2(a2 - b2 )
(D) f ç ÷ = for all x, y Î  with y ¹ 0
è y ø f ( y) (C) f (-1) = -2/(a + b) (D) f (-1) = 2(a - b)
Solution: We shall prove that f (x) = x for all x Î  and Solution: We are given that
hence (A), (B), (C) and (D) are all true. By (a), f is an odd
function and hence f (0) = 0. æ 1ö
a f ( x) + b f ç ÷ = x - 1 (1.8)
è xø
0 = f (0) = f (-1 + 1) = f (-1) + 1 [by (b)]
Replacing x with 1/x, we get
Therefore f (-1) = - 1. For any x ¹ 0 and -1, we have
æ 1ö 1
æ 1 ö f ( x + 1) f ( x) + 1 b f ( x) + a f ç ÷ = - 1 (1.9)
fç = = (1.6) è xø x
è x + 1÷ø ( x + 1)2 ( x + 1)2
From Eqs. (1.8) and (1.9), we have
Also, æ1 ö
(a2 - b2 ) f ( x) = a( x - 1) - b ç - 1÷
æ 1 ö æ -x ö èx ø
fç ÷ = fç + 1÷
è x + 1ø èx+1 ø Therefore
æ -x ö
=fç +1 a + b/ 2 2a + b
è x + 1÷ø f ( 2) = =
a -b
2 2
2(a2 - b2 )
æ x ö
= -f ç +1 f (1) = 0
è x + 1÷ø
-2a + 2b -2
æ 1 ö and f (-1) = =
= -f ç +1 a2 - b2 a+b
è ( x + 1) x ÷ø
Answers: (A), (B) and (C)
- f [( x + 1)/ x]
= +1
[( x + 1)/ x]2 5. Let P(x) be a polynomial function of degree n such that
- f [1 + (1/ x)]
= +1 k
[( x + 1)/ x] 2 P (k ) =
k+1
- x2 [ f (1/ x) + 1]
= +1
( x + 1) 2
www.jeeneetbooks.in
Worked-Out Problems 65

for k = 0, 1, 2, …, n. Then P(n + 1) is equal to Therefore


(A) -1 if n is even (B) 1 if n is odd
1 é (-1)n + 1 x( x - 1)( x - 2) ( x - n) ù
n n P ( x) º ê x + ú
(C) if n is even (D) if n is odd x + 1ë (n + 1)! û
n+2 n +2

Solution: Consider the polynomial 1


P(n + 1) = [(n + 1) + (-1)n + 1 ]
n+2
Q ( x) º P ( x)( x + 1) - x
Then Q(x) is a polynomial of degree n + 1 and 0, 1, 2, …, n ì 1 if n is odd
ï
are the roots of the equation Q(x) = 0. Therefore =í n
ï n + 2 if n is even
î
Q ( x) = Ax( x - 1)( x - 2) ( x - n)
where A is a non-zero constant. Substituting x = -1, we Answers: (B) and (C)
get that
1 = Q(-1) = A(-1)n + 1 (n + 1)!

Matrix-Match Type Questions


+
1. If A = {1, 2, 4, 5}, B = {2, 3, 4, 5} and C = {4, 5, 6, 7}, then 3. Let P : [0, ¥) ®  be defined as
match the items in Column I with those in Column II.
ì 13 if 0 £ x < 1
P ( x) = í +
î13 + 15n if n £ x < n + 1, n Î 
Column I Column II
Then match the items in Column I with those in
(A) (A - B) È C (p) {1, 2, 3} Column II.
(B) (A - B) È (B - C) (q) {1, 3, 4, 5, 6, 7}
(C) (A È B) - C (r) {1, 4, 5, 6, 7}
Column I Column II
(D) (A D B) D C (s) {1, 2, 3, 4}
(A) P(3 × 01) (p) 68
Solution: This can be solved by simple checking. (B) P(4 × 9) (q) 63
Answer: (A)Æ(r), (B)Æ(p), (C)Æ(p), (D)Æ(q) (C) P(3 × 999) (r) 73
(D) P([4 × 99]) (s) 58
2. Let A, B and C be subsets of a finite universal set X. Let
n(P) denote the number of elements in a set P. Then Solution: Given that P(x) = 13 + 15[x] for all x ³ 0,
match the items in Column I with those in Column II. where [x] is the integral part of x. Then
P(3 × 01) = 13 + 15 ´ 3 = 58
Column I Column II
Remaining parts can be solved similarly.
(A) n(A - B) (p) n(X) - n(A Ç B) Answer: (A)Æ(s), (B)Æ(r), (C)Æ(s), (D)Æ(r)
(B) n(A D B) (q) n(C) - n(C Ç B)
Note: Functions of this type are called Postage-stamp
(C) n(Ac È Bc) (r) n(A) - n(A Ç B)
functions.
(D) n(C Ç Bc) (s) n(A) + n(B) - 2n(A Ç B)

Solution: This can be solved by simple checking.


Answer: (A)Æ(r), (B)Æ(s), (C)Æ(p), (D)Æ(q)

Comprehension-Type Questions
1. In a group of 25 students aged between 16 years and ball, 9 play both cricket and tennis, 4 play tennis and
18 years, it was found that 15 play cricket, 12 play football and 3 play all the three games. Based on this,
tennis, 11 play football, 5 play both cricket and foot answer the following questions.
www.jeeneetbooks.in
66 Chapter 1 Sets, Relations and Functions

(i) The number of students in the group who play (i) f (x) is equal to
only football is
1é 1 x - 1ù
(A) 2 (B) 3 (C) 4 (D) 5 (A) êx + - ú
2ë 1- x x û
(ii) The number of students in the group who play
only cricket is 1é 1 x - 1ù
(B) êx - + ú
(A) 1 (B) 2 (C) 3 (D) 4 2ë 1- x x û
(iii) The number of students in the group who play
1é 1 x - 1ù
only tennis is (C) êx - - ú
2ë 1- x x û
(A) 1 (B) 2 (C) 3 (D) 4
(iv) The number of students who do not play any of 1é 1 x - 1ù
(D) êx + + ú
the three games is 2ë 1- x x û
(A) 1 (B) 2 (C) 3 (D) 4
(ii) f (-1) is equal to
Solution: Let C, T and F denote the sets of students in (A) 3/4 (B) -3/4 (C) 5/4 (D) -5/4
the group who play cricket, tennis and football, respec-
tively. Consider the Venn diagram. (iii) f (1/2) is equal to
C T (A) 5/4 (B) -7/4 (C) 7/4 (D) 9/4
Solution: Given that
x a y
æ x - 1ö
f ( x) + f ç =x (1.10)
3 è x ÷ø
c b
for all x ¹ 0, 1. Replacing x with ( x - 1)/ x both sides, we
z get that
F
æ x - 1ö æ [( x - 1)/ x] - 1ö x - 1
fç ÷ + fç =
We are given that
è x ø è ( x - 1)/ x ÷ø x

n(C ) = x + a + c + 3 = 15 That is,


n(T ) = y + b + a + 3 = 12 æ x - 1ö æ 1 ö x-1
fç + fç = (1.11)
è x ÷ø è 1 - x ÷ø x
n(F ) = z + c + b + 3 = 11
Then Again replacing x with ( x - 1)/ x in this, we get

n (C Ç T ) = a + 3 = 9 æ 1 ö 1
fç ÷ + f ( x) = (1.12)
n (T Ç F ) = b + 3 = 4 è 1 - xø 1- x

n (C Ç F ) = c + 3 = 5 Then by taking Eq. (1.10) + Eq. (1.12) - Eq. (1.11), we get that

n (C Ç T Ç F ) = 3 1 x-1
2 f (x) = x + -
1- x x
and by solving these, we get a = 6, b = 1, c = 2, x = 4, y = 2
and z = 5. The number of students who do not play any of 1é 1 x - 1ù
these games is 25 - (a + b + c + x + y + z + 3) = 2. or f (x) = êx + - ú (1.13)
2ë 1- x x û
Answer: (i) Æ (D); (ii) Æ (D); (iii) Æ (B); (iv) Æ (B)
Substituting the values x = -1 and 1/2 in Eq. (1.13) we get
2. Let f :  - {0, 1} ®  be a function satisfying the relation the solution for (ii) and (iii).
æ x - 1ö Answer: (i) Æ (A); (ii) Æ (D); (iii) Æ (C)
f ( x) + f ç =x
è x ÷ø

for all x Î  - {0, 1}. Based on this, answer the follo-


wing questions.
www.jeeneetbooks.in
Worked-Out Problems 67

Assertion–Reasoning Type Questions


In the following question, a Statement I is given and a Solution: Note that, for any sets A, B, C and D,
corresponding Statement II is given just below it. Mark
the correct answer as: ( A ´ B) Ç (C ´ D) = ( A Ç C ) ´ ( B Ç D)
(A) Both I and II are true and II is a correct reason for I and hence
(B) Both I and II are true and II is not a correct reason for I ( A ´ B) Ç ( B ´ A) = ( A Ç B) ´ ( B Ç A)
(C) I is true, but II is false
= ( A Ç B) ´ ( A Ç B)
(D) I is false, but II is true
Therefore
1. Statement I: If A = {1, 2, 3, 4} and B = {2, 3, 5, 6, 7},
then n((A ´ B) Ç (B ´ A)) = 4. n(( A ´ B) Ç ( B ´ A)) = [n( A Ç B)]2

Statement II: If two sets A and B have n elements in Answer: (A)


common, then the sets A ´ B and B ´ A have n2 elements
in common.

Integer Answer Type Questions


1. Let f :  ®  be a function such that f (1) = f (0) = 0 Therefore, we have 1 = f (1) > f (2) > f (3) > and hence
and | f(x) - f( y)| < |x - y| for all x ¹ y in [0, 1]. If f (n) ¹ n for all n > 1. Thus 1 is the only positive integer n
2| f(x) - f( y)| < K for all x, y Î [0,1], then K can be such that f (n) = n.
. Answer: 1
Solution: Let 0 < x < y < 1. Then
3. Let f :  ®  be a function such that f (2 + x) = f (2 - x)
| f ( x) - f ( y)| £ | f ( x)| + | f ( y)| and f (7 + x) = f (7 - x) for all real numbers x. If f (0) = 0
and there are atleast m number of integer solutions
= | f ( x) - f (0)| + | f ( y) - f (1)|
for f (x) = 0 in the interval [–2010, 2010], then m can
< | x - 0 | + | y - 1| be .

=x+1-y (1.14) Solution: For all x Î , we have

Also, f (2 + x) = f (2 - x) = f [7 - (5 + x)]

| f ( x) - f ( y)| < | x - y | = y - x (1.15) = f [7 + (5 + x)] = f (12 + x)

By adding Eqs. (1.14) and (1.15), we have By replacing x with x – 2 we get that

2 | f ( x) - f ( y)| < 1 f ( x) = f ( x + 10) for all x Î  (1.16)

Answer: 1 Now,

2. Let f :  ®  be a function such that f (x + y) = f (x) + 0 = f (0) = f (2 - 2) = f (2 + 2) = f (4) (1.17)


f (y) - xy - 1 for all x, y Î  and f (1) = 1. Then the From Eqs. (1.16) and (1.17), we have f (4 + 10 n) = 0 for
number of positive integers n such that f (n) = n is all integers n. Also, since f (0) = 0, we have f (10 n) = 0
. for all integers n. There are 403 integers of the form
Solution: By taking x = 0 = y, we get that f (0) = 1. By 10n and 402 integers of the form 10 n + 4 in the interval
hypothesis, f (1) = 1. For any integer n > 1, [–2010, 2010]. Therefore, there are atleast 805 integers n
in [–2010, 2010] for which f (n) = 0.
f (n) = f [(n - 1) + 1] = f (n - 1) + f (1) - (n - 1)1 - 1 Answer: 805
= f (n - 1) - (n - 1) < f (n - 1)
www.jeeneetbooks.in
68 Chapter 1 Sets, Relations and Functions

SUMMARY
1.1 Set: Any collection of well-defined objects. 1.13 If | X | = n, then |P(X)| = 2n.

1.2 Elements: Objects belonging to a set. 1.14 Intersection of sets: For any two sets A and B, the
intersection of A and B is the set of all elements
1.3 Empty set: Set having no elements and is denoted Ø. belonging to both A and B and is denoted by

1.4 Equal sets: Two sets A and B are said to be equal, if A Ç B = {x | x Î A and x Î B}
they contain same elements or every element of A
1.15 Theorem: The following hold for any sets A, B and C.
belong to B and vice-versa.
(1) A Í B Û A = A Ç B
1.5 Finite set: A set having definite number of elements is (2) A Ç A = A
called finite set. A set which is not a finite set is called (3) A Ç B = B Ç A (Commutative law)
infinite set.
(4) (A Ç B) Ç C = A Ç (B Ç C) (Associative law)
1.6 Family or class of sets: A set whose numbers are (5) A Ç Ø = Ø, where Ø is the empty set.
family of sets or class of sets. Family of sets or class (6) For any set X, X Í A Ç B Û X Í A and X Í B.
of sets are denoted by script letters Ꮽ, Ꮾ, Ꮿ, Ᏸ, P etc. (7) In view of (4) we write A Ç B Ç C for A Ç (B Ç C).


n
1.7 Indexed family of sets: A family C of sets is called (8) For any sets A1, A2, ¼, An we write Ai for
i=1
indexed family if there exists a set I such that for A1 Ç A2 Ç A3 Ç ¼ Ç An.
each element i Î I, there exists unique member
A Î C associated with i. In this case the set I is called 1.16 Disjoint sets: Two sets A and B are said to be
index set, C is called indexed family sets and we disjoint sets if A Ç B = Ø.
write C = {Ai : i Î I}.
1.17 Union of sets: For any two sets A and B, their union
1.8 Intervals: Let a, b be real numbers and a < b. Then is defined to be the set of all elements belonging to
(a, b) = {x Î  | a < x < b} either A or to B and this set is denoted by A È B.
That is A È B = {x| x Î A or x Î B} .
[a, b) = {x Î  | a £ x < b}
1.18 Theorem: For any sets A, B and C the following
(a, b] = {x Î  | a < x £ b} hold.
[a, b] = {x Î | a £ x £ b} (1) A Ç B Í A È B

(-¥, +¥) or (-¥, ¥) is  (2) For any set X, A È B Í X Û A Í X and B Í X


(3) A È A = A
1.9 Subset and superset: A set A is called a subset of a set (4) A È B = B È A (Commutative law)
B, if every element of A is also an element of B. In this
(5) (A È B) È C = A È (B È C) and we write A È B È C
case we write A Í B. If A is a subset of B, then B is for (A È B) È C
called superset of A. If A is not a subset of B, then we
(6) A Ç B = A Û A È B = B
write A Í B.
(7) A È Ø = A
1.10 Proper subset: Set A is called a proper subset of a (8) A Ç (A È B) = A
set B if A is a subset of B and is not equal to B. (9) A È (A Ç B) = A

1.11 Powerset: If X is a set, then the collection of all 1.19 Theorem (Distributive laws): If A, B and C are
subsets of X is called the powerset of X and is three sets, then
denoted by P(X).
(1) A Ç (B È C) = (A Ç B) È (A Ç C)
1.12 Cardinality of a set: If X is a finite set having n (2) A È (B Ç C) = (A È B) Ç (A È C)
elements, then n called cardinality of X and is
denoted by |X | or n(X ). 1.20 Theorem: For any sets A, B and C, A Ç B = A Ç C and
A È B = A È C Þ B = C.
www.jeeneetbooks.in
Summary 69

∪ i ÎI Ai is
1.21 If {Ai}iÎI is an indexed family of sets then (3) A D Ø = A
the set of all elements x where x belongs to atleast (4) A D A = Ø
one Ai.
1.31 Theorem: If A and B are disjoint sets, then
1.22 Set difference: For any two sets A and B, A – B =
(1) n(A È B) = n(A) + n(B)
{ x Î A| x Ï B} = A – (A Ç B)
(2) If A1, A2, ¼, Am are pairwise disjoint sets, then
1.23 De Morgan’s laws: If A, B and C are any sets, then æm ö
n ç ∪ Ai ÷ = n( A1 ) + n( A2 ) + + n( Am )
(1) A - (B È C) = (A - B) Ç (A - C) è i =1 ø
(2) A - (B Ç C) = (A - B) È (A - C)
Recall that for any finite set P, n(P) denotes the
number of elements in P.
1.24 Theorem: Let A, B and C be sets. Then
(1) B Í C Þ A - C Í A - B 1.32 Theorem: For any finite sets A and B, n(A È B) =
(2) A Í B Þ A - C Í B - C n(A) + n(B) - n(A Ç B).
(3) (A È B) - C = (A - C) È (B - C)
1.33 Theorem: For any finite sets A, B and C,
(4) (A Ç B) - C = (A - C) Ç (B - C)
(5) (A - B) - C = A - (B È C) = (A - B) Ç (A - C) n(A È B È C) = n(A) + n(B) + n(C) - n(A Ç B)
-n(B Ç C) - n(C Ç A) + n(A Ç B Ç C)
(6) A - (B - C) = (A - B) È (A Ç C)
1.34 Theorem: If A, B and C are finite sets, then the
1.25 Universal set: If {Ai}iÎI is a class of sets, then the set
number of elements belonging to exactly two of the
X = ∪ i ÎI Ai is called universal set. In fact the set X
sets is
whose subsets are under our consideration is called
universal set. n(A Ç B) + n(B Ç C) + n(C Ç A) - 3n(A ÇB Ç C)
Caution: Do not be mistaken that universal set
1.35 Theorem:
means the set which contains all objects in the
universe. Do not be carried away with word universal. (1) If A, B and C are finite sets, then the number of
In fact, the fundamental axiom of set theory is: elements belonging to exactly one of the sets is
Given any set, there is always an element which does n(A) + n(B) + n(C) - 2n(A Ç B) - 2n(B Ç C)
not belong to the given set. - 2n(C Ç A) + n(A Ç B Ç C)
(2) If A and B are finite sets, then the number of
1.26 Complement of a set: If X is an universal set and
A Í X then the set X - A is called complement of elements belonging to exactly one of the sets
A and is denoted by A¢ or Ac. equals
n(A D B) = n(A) + n(B) - 2n(A Ç B)
1.27 Relative complement: If X is an universal set and
= n(A È B) - n(A Ç B)
A, B are subsets of X, then A - B = A Ç B¢ is called
relative complement of B in A.
Relations
1.28 De Morgan’s laws (General form): If A and B are
two sets, then 1.36 Ordered pair: A pair of elements written in a
particular order is called an ordered pair and is
(1) (A È B)¢ = A¢ Ç B ¢ written by listing its two elements in a particular
(2) (A Ç B)¢ = A¢ È B ¢ order, separated by a comma and enclosing the pair
in brackets. In the ordered pair (x, y), x is the first
1.29 Symmetric difference: For any two sets A and B, element called first component and y is the second
the set (A - B) È (B - A) is called symmetric differ- element called second component. Also x is called
ence of A and B and is denoted by A D B. Since A first coordinate and y is called second coordinate.
- B = A Ç B ¢ and B - A = B Ç A¢, A D B = (A Ç B ¢)
È (B Ç A¢). 1.37 Cartesian product: If A and B are sets, then the
set of all ordered pairs (a, b) with a Î A and b Î B
1.30 Theorem: The following hold for any sets A, B and C. is called the Cartesian product of A and B and is
(1) A D B = B D A (Commutative law)
denoted by A × B (read as A cross B). That is
(2) (A D B) D C = A D (B D C) (Associative law) A ´ B = {(a, b) | a Î A and b Î B}
www.jeeneetbooks.in
70 Chapter 1 Sets, Relations and Functions

1.38 Let A, B be any sets and Ø is the empty set. Then 1.45 Range: If R is a relation from a set A to a set B,
(1) A ´ B = Ø Û A = Ø or B = Ø. then the set of all second components of the
ordered pairs belonging to R is called range of R
(2) If one of A and B is an infinite set and the
and is denoted by Range(R).
other is a non-empty set, then A ´ B is an infi-
nite set.
1.46 Theorem: If A and B are finite non-empty sets
(3) A ´ B = B ´ A Û A = B. such that n(A) = m and n(B) = n, then the number
of relations from A to B is 2mn which include the
1.39 Cartesian product of n sets (n is a finite positive empty set and the whole set A ´ B.
integer greater than or equal to 2): Let A1, A2, A3,
¼, An be n sets. Then their Cartesian product is 1.47 Relation on a set: If A is a set, then any subset of
defined to be the set of all n-tuples (a1, a2, ¼, an) A ´ A is called a binary relation on A or simply a rela-
such that ai ÎAi for i = 1, 2, 3, ¼, n and is denoted by tion on A.
n n
A1 ´ A2 ´ A3 ´ ´ An or X Ai
i =1
or ÕA
i =1
i 1.48 Composition of relations: Let A, B and C be sets, R
is a relation from A to B and S is a relation from B
That is, to C. Then, the composition of R and S denoted by
S R defined to be
A1 ´ A2 ´ L ´ An = {(a1, a2, K, an )| ai Î Ai for 1 £ i £ n}
The Cartesian product of a set A with itself n times S R = {(a, c) Î A ´ C | there exist b Î B
is denoted by An.
such that (a, b) Î R and (b, c) Î S}
1.40 Theorem: If A and B are finite sets, then n(A ´ B) =
n(A) · n(B). In general, if A1, A2, ¼, Am are infinite 1.49 Theorem: Let A, B and C be sets, R a relation from
sets, then n(A1 ´ A2 ´ ´ Am) = n(A1) ´ n(A2) ´ A to B and S a relation from B to C. Then the
´ n(Am). In particular, n(Am) = (n(A))m where A following hold:
is a finite set. (1) S R ¹ Ø if and only if Range(R) Ç Dom(S) ¹ Ø
(2) Dom(S R) = Dom(R)
1.41 Theorem: Let A, B, C and D be any sets. Then
(3) Range(S R) Í Range(S)
(1) A ´ (B È C) = (A ´ B) È (A ´ C)
(2) (A È B) ´ C = (A ´ C) È (B ´ C) 1.50 Theorem: Let A, B, C and D be non-empty sets,
(3) A ´ (B Ç C) = (A ´ B) Ç (A ´ C) R Í A ´ B, S Í B ´ C and T Í C ´ D. Then
(4) (A Ç B) ´ C = (A ´ C) Ç (B ´ C) (T S) R = T (S R) (Associative law)
(5) (A È B) ´ (C È D) = (A ´ C) È (A ´ D) È (B ´
C) È (B ´ D) 1.51 Inverse relation: Let A and B be non-empty sets
(6) (A Ç B) ´ (C Ç D) = (A ´ C) Ç (B ´ D) = (A ´ D) and R a relation from A to B. Then the inverse of R
Ç (B ´ C) is defined as the set {(b, a) Î B ´ A | (a, b) Î R} and
is denoted by R–1.
(7) (A - B) ´ C = (A ´ C) - (B ´ C)
(8) A ´ (B - C) = (A ´ B) - (A ´ C) 1.52 Theorem: Let A, B and C be non-empty sets, R a
relation from A to B and S a relation from B to C.
1.42 Relation: For any two sets A and B, any subset of Then the following hold:
A ´ B is called a relation from A to B. -1 -1 -1
(1) (S R) = R S
1.43 Symbol aRb: Let R be a relation from a set A to a (2) (R-1)-1 = R
set B (R Í A ´ B). If (a, b) Î R, then a is said to be
R related to b or a is said to be related to b and we
write aRb.
Types of Relations
1.53 Reflexive relation: Let X be a non-empty set and R
1.44 Domain: Let R be a relation from a set A to a set B.
relation from X to X. Then R is said to be reflexive
Then the set of all first components of the ordered
on X if (x, x) Î R for all x Î X.
pairs belonging to R is called the domain of R and
is denoted by Dom(R).
1.54 Symmetric relation: A relation R on a non-empty
set X is called symmetric if (x, y) Î R Þ (y, x) Î R.
www.jeeneetbooks.in
Summary 71

1.55 Transitive relation: A relation R on a non-empty set for each a Î A, there exists unique b Î B such that
X is called transitive if (x, y) Î R and (y, z) Î R Þ (a, b) Î f. That is f Í A ´ B is called a function from
(x, z) Î R. A to B, if
(1) Dom ( f ) = A
1.56 Equivalence relation: A relation R on a non-empty
(2) (a, b) Î f and (a, c) Î f Þ b = c
set X is called an equivalence relation if it is reflexive,
symmetric and transitive. If f is a function from A to B, then we write f : A ® B
is a function and for (a, b) Îf, we write b = f(a) and b
1.57 Partition of a set: Let X be a non-empty set. A is called f-image of a and a is called f-preimage of b.
class of subsets of X is called a partition of X if they
are pairwise disjoint and their union is X. 1.65 Domain, codomain and range: Let f : A ® B be a
function. Then A is called domain, B is called codo-
1.58 Equivalence class: Let X be a non-empty set and R main and Range of f denoted by f(A) = { f(a) | a ÎA}.
an equivalence relation on X. If x Î X, then the set f (A) is also called the image set of A under the
{ y Î X |( x, y) ÎR} is called the equivalence class of x function f.
with respect to R or the R-equivalence of x or simply
the R-class of x and is denoted by R(x). 1.66 Composition of functions: Let f : A ® B and g :
B ® C be functions. Then the composition of f with
1.59 Theorem: Let R be an equivalence relation on a set g denoted by g f is defined as g f : A ® C given by
X and a, b Î X. Then the following statements are (g f ) (a) = g(f (a)) for all a ÎA
equivalent:
(1) (a, b) Î R 1.67 Theorem: Let f : A ® B, g : B ® C and h : C ® D be
(2) R(a) = R(b) functions. Then
(3) R(a) Ç R(b) ¹ Ø (h g) f = h (g f)

1.60 Theorem: Let R be an equivalence relation on X. 1.68 One-one function or injection: A function f : A ® B
Then the class of all R-classes form a partition of X. is called “one-one function” if f (a1) ¹ f (a2) for any
a1 ¹ a2 in A.
1.61 Theorem: Let X be a non-empty and {Ai}i ÎI a parti-
tion of X. Then 1.69 Theorem: If f : A ® B and g : B ® C be functions.
Then the following hold:
R = {( x, y) Î X ´ X | both x and y
(1) If f and g are injections, then so is g f .
belong to the same Ai, i Î I }
(2) If g f is an injection, then f is an injection.
is an equivalence relation on X, whose R-classes are
precisely Ai’s. 1.70 Onto function or surjection: A function f : A ® B
is called “onto function” if the range of f is equal to
1.62 Theorem: Let R and S be equivalence relations on the codomain B. That is, to each b Î B, there exists
a non-empty X. Then R Ç S is also an equivalence a Î A such that f (a) = b.
relation on X and for any x Î X, (R Ç S)(x) = R(x)
Ç S(x). 1.71 Theorem: Let f : A ® B and g : B ® C be functions.
Then, the following hold:
1.63 Theorem: Let R and S be equivalence relations (1) If f and g are surjections, then so is g f.
on a set X. Then the following statements are
(2) If g f is a surjection, then g is a surjection.
equivalent.
(1) R S is an equivalence relation on X 1.72 Bijection or one-one and onto function: A function
(2) R S is symmetric f : A ® B is called “bijection” if f is both an injection
(3) R S is transitive and a surjection.
(4) R S = S R
1.73 Theorem: If f : A ® B and g : B ® C are bijections,
then g f : A ® C is a bijection.
Functions
1.74 Identity function: A function f : A ® A is called
1.64 Function: A relation f from a set A to a set B is an identity function if f (x) = x for all x Î A and is
called a function from A into B or simply A to B, if denoted by IA.
www.jeeneetbooks.in
72 Chapter 1 Sets, Relations and Functions

1.75 Theorem: If f: A ® B is a function, then IB f = f = f IA. QUICK LOOK

0 ≤ {x} < 1 for any real number x.


QUICK LOOK

Identity function is always a bijection.


1.81 Theorem: The following hold for any real numbers
x and y.
1.76 Theorem: Let f : A ® B be a function. Then, f is
ìï[ x] + [ y] if {x} + { y} < 1
a bijection if and only if there exists a function (1) [ x + y] = í
g : B ® A such that îï[ x] + [ y] + 1 if {x} + { y} ³ 1
g f = IA and f g = IB (2) [x + y] ≥ [x] + [y] and equality holds if and only
if { x } + {y} < 1.
That is
(3) If x or y is an integer, then [x + y] = [x] + [y].
g( f(a)) = a for all a Î A
(4) é x ù = é [ x] ù for any real number x and
and f (g(b)) = b for all b Î B êë m úû êë m úû
non-zero-integer m.
1.77 Inverse of a bijective function: Let f : A ® B and (5) If n and k are positive integers and k > 1, then
g : B ® A be functions such that g f = IA and f g =
IB. Then f and g are bijections. Also g is unique such é n ù é n + 1 ù é 2n ù
that g f = IA and f g = IB. g is called the inverse of f êë k úû + êë k úû £ êë k úû
and f is called the inverse of g. The inverse function
of f is denoted by f –1. 1.82 Periodic function: Let A be a subset of  and
f : A ®  a function. A positive real number p is
QUICK LOOK called a period of f if f (x + p) = f (x) whenever x and
x + p belong to A. A function with a period is called
If f : A ® B is a bijection, then f –1 : B ® A is also a periodic function. Among the periods of f, the least
bijection and f –1(b) = a Û f (a) = b for b Î B. one (if it exists) is called the least period.

1.83 Step function (greatest integer function): Let


1.78 Real-valued function: If the range of a function is f :  ®  be defined by f(x) = [x] for all x Î  where
a subset of the real number set , then the function [x] is the largest integer less than or equal to x. This
is called a real-valued function. function f is called step function.

1.79 Operations among real-valued functions: Let f 1.84 Signum function: Let f :  ®  be defined by
and g be real-valued functions defined on a set A.
Then we define the real-valued functions f + g, -f, ì- 1 if x < 0
f - g and f × g on the set A as follows: ï
f ( x) = í 0 if x = 0
(1) ( f + g)(a) = f (a) + g(a) ï 1 if x > 0
î
(2) (-f )(a) = -f (a)
(3) ( f - g)(a) = f (a) - g(a) is called Signum function and is written as sign(x).
(4) ( f · g)(a) = f (a) g(a)
1.85 Increasing and decreasing functions: Let A be a
(5) If g(a) ¹ 0 for all a Î A, then
subset of  and f : A ®  a function. Then, we say
æfö f (a) that f is an increasing function if f(x) ≤ f(y) whenever
çè g ÷ø (a) = g(a) x ≤ y. f is said to be decreasing function if f(x) ≥ f(y)
whenever x ≤ y.
(6) If n is a positive integer, then f (a) = ( f (a)) .
n n

1.86 Symmetric set: A subset X of  is called a symmetric


1.80 Integral part and fractional part: If x is a real set if x Î X Û -x Î X.
number, then the largest integer less than or equal
to x is called the integral part of x and is denoted by 1.87 Even function: Let X be a symmetric set and
[x]. x - [x] is called the fractional part of x and will f : X ®  a function. Then f is said to be even func-
be denoted by { x }. tion if f(-x) = f(x) for all x Î X.
www.jeeneetbooks.in
Exercises 73

1.88 Odd function: Let X be a symmetric set and 1.91 Theorem: If f and g are even (odd) functions then so
f : X ®  a function. Then f is said to be odd is f ± g.
function if f (-x) = -f(x ) for all x Î X.
1.92 Theorem: Every real-valued function can be
QUICK LOOK uniquely expressed as a sum of an even function
and an odd function. The representation is
If f is an odd function on a symmetric set X and 0
1 1
belongs to X, then f(0) is necessarily 0. f ( x) = [ f ( x) + f (- x)] + [ f ( x) - f (- x)]
2 2

1.93 Number of partions of a finite set: Let P0 = 1 and


1.89 Theorem: Let X be a symmetric set and f, g be func-
Pn be the number of partions on a finite set with n
tions from X to . Then, the following hold:
elements. Then for n ≥ 1,
(1) f · g is even if either both f and g are even or both
are odd. n æ nö
Pn + 1 = å ç ÷ Pr
(2) f · g is odd if one of them is odd and the other r =1 è r ø
is even.
æ nö
1.90 Theorem: Let f be a real valued function defined where ç ÷ is the number of selections of r objects
on a symmetric set X. Then the following hold: èr ø
(0 ≤ r ≤ n) from n distinct objects and this number
(1) f is even if and only if af is even for any non-zero
is equal to
a Î .
(2) f is odd if and only if af is odd for any non-zero n!
a Î . r ! ( n - r )!
(3) f is even (odd) if and only if -f is even (odd).

EXERCISES
Single Correct Choice Type Questions
1. Let U = {1, 2, 3, 4, 5, 6, 7, 8, 9}, A = {1, 2, 3, 4}, B = {2, 4, (C) X
6, 8} and C = {3, 4, 5, 6}. Then
(A) (B Ç C)c = {2, 4, 5, 6, 7}
(B) (A Ç C)c = {1, 2, 3, 4, 5, 8, 9}
(C) (B È C)c = {1, 7, 8, 9}
(D) (A Ç B)c = {1, 3, 5, 6, 7, 8, 9} A B

2. If A and B are two non-empty subsets of a set X, then


which one of the following shaded diagrams represent
(D)
the complement of B - A in X? X
(A) X

A B

A B

3. Let A Δ B denote the symmetric difference of A and B.


(B) X Then, for any sets, A, B and C, which one of the following
is not correct?
(A) A D B = C Û A = B D C
(B) A D B = C D B Û A = C
(C) ( A D B) D ( B D A) = f
A B (D) A D B = f Û A Í B
www.jeeneetbooks.in
74 Chapter 1 Sets, Relations and Functions

4. A, B, C are three finite sets such that A Ç B Ç C has (A)


10 elements. If the sets A D B, B D C and C D A have 1
100, 150 and 200 elements, respectively, then the
1
number of elements in A È B È C is
2
(A) 325 (B) 352 (C) 235 (D) 253
2

5. In a class of 45 students, it is found that 20 students liked 3


apples and 30 liked bananas. Then the least number of
students who liked both apples and bananas is
(A) 5 (B) 10 (C) 15 (D) 8 (B)
1
6. In a class of 45 students, 25 play chess and 26 play 1
cricket. If each student plays chess or cricket, then 2
the number of students who play both is 2
3
(A) 5 (B) 6 (C) 7 (D) 4 3
4
7. The number of subsets of the empty set is
(A) 1 (B) 2 (C) 0 (D) 3
(C)
8. The number of non-empty subsets of the set {1, 2, 3,
4, 5} is 1
1
(A) 30 (B) 32 (C) 31 (D) 33 2
2
9. The number of subsets of a set A is of the form 10 3
n + 4, where n is a single-digit positive integer. Then 3
4
n is equal to
(A) 8 (B) 4 (C) 5 (D) 6
(D)
10. If A and B are sets such that n(A È B) = 40, n(A) = 25
1
and n( B) = 20 , then n( A Ç B) = 1
(A) 1 (B) 2 (C) 5 (D) 4 2
2
3
11. Let  be the set of all natural numbers and 3
4
R = {(a, b) Î  ´  | g.c.d. of {a, b} = 1}
Then R is
(A) reflexive on  14. Let A = {1, 2, 3, 4}, B = {5, 6, 7} and c = {a, b, c, d, e}. If
(B) symmetric f = {(1, 5), (2, 5), (3, 6), (4, 7)} and g = {(5, a), (6, d),
(C) transitive (7, c)} are functions from A to B and from B to C,
(D) an equivalence relation respectively, then
(A) (g f ) (4) = d (B) ( g f )(3) = a
12. Let  denote the set of non-zero rational numbers
*
(C) ( g f )(2) = c (D) ( g f )(1) = a
and
15. Which one of the following diagrams does not repre-
R = {(a, b) Î * ´ * | ab = 1}
sent a function?
Then R is (A)
(A) symmetric
1
(B) reflexive on * a
(C) an equivalence relation 2
(D) transitive b
3
c
13. Which one of the following diagrams represents a 4
function?
www.jeeneetbooks.in
Exercises 75

(B) (C) 

1
a
2
b
3 
c O
4

(C)
1
a (D) 
2
b
3
c
4

O

(D)
1
a
2
b 17. Let f : [1, ¥) ® [2, ¥) be the function defined by
3
c 1
4 f ( x) = x +
x
If g : [2, ¥) ® [1, ¥), is a function such that (g f )(x) = x
for all x ³ 1, then g(t ) =
16. Which one of the following graphs does not represent
1 1
a function from the real number set  into ? (A) t + (B) t -
t t
(A) 
(C ) t + t - 4 (D) t - t - 4
2 2

2 2

18. Let f and g be the functions defined from  to  by

 ì- 2 if x < 0
O ï
f ( x) = í 0 if x = 0 and g( x) = 1 + {x}
ï 2 if x > 0
î
where {x} is the fractional part of x. Then, for all x Î ,
f (g(x)) is equal to
(B) 
(A) –2 (B) 0 (C) x (D) 2

19. The number of surjections of {1, 2, 3, 4} onto {x, y} is


(A) 16 (B) 8 (C) 14 (D) 6

O 20. If f (x) is a polynomial function satisfying the relation

æ 1ö æ 1ö
f ( x) + f ç ÷ = f ( x) f ç ÷
è xø è xø
for all 0 ¹ x Î  and if f (2) = 9, then f (6) is
(A) 216 (B) 217 (C) 126 (D) 127
www.jeeneetbooks.in
76 Chapter 1 Sets, Relations and Functions

21. Let a be positive real number and n a positive 27. The number of solutions of the equation 2x + {x + 1} =
integer. If f ( x) = (a - xn )1/ n, then ( f f )(5) is 4[x + 1] – 6 is
(A) 5 (B) 2 (C) 3 (D) 4 (A) 1 (B) 2 (C) 3 (D)

22. For any 0 ≤ x ≤ 1, let f (x) = max {x , (1 - x) , 2x(1 -


2 2
28. Let [x] denote the integral part of x. If a is a positive real
x)}. Then which one of the following is correct? number and f :  ®  is defined by f (x) = x - [x - a],
then a period of f is
ì2 x(1 - x), 0 £ x £ 1/ 3
ïï (A) 1 (B) a (C) 2[a] (D) 2a
(A) f ( x) = í (1 - x)2 , 1/ 3 < x £ 2 / 3
ï 29. If f (x) = k (constant) for all x Î , then the least
ïî x2 , 2 / 3 < x £ 1
period of f is
ì (1 - x)2 , 0 £ x £ 1/ 3 (A) 1/3 ( B) 1/2
ïï (C ) 2/3 (D) does not exist
(B) f ( x) = í 2 x(1 - x), 1/ 3 < x £ 2 / 3
ï
ïî x2 , 2 / 3 < x £ 1 30. Let a > 0 and f :  ®  a function satisfying

ì x , 0 £ x £ 1/ 3
2 f ( x + a) = 1 + [2 - 3 f ( x) + 3 f ( x)2 - f ( x)3 ]1/ 3
ïï
(C) f ( x) = í 2 x(1 - x), 1/ 3 < x £ 2 / 3 for all x Î . Then a period of f ( x) is ka where k is
ï a positive integer whose value is
ïî (1 - x) , 2 / 3 < x £ 1
2
(A) 1 (B) 2 (C) 3 (D) 4
ì (1 - x)2 , 0 £ x £ 1/ 3
ïï 31. Let a < c < b such that c - a = b - c . If f : ®  is a
(D) f ( x) = í x2 , 1/ 3 < x £ 2 / 3 function satisfying the relation
ï
ïî2 x(1 - x), 2 / 3 < x £ 1 f ( x + a) + f ( x + b) = f ( x + c) for all x Î
then a period of f is
23. Let [x] denote the greatest integer £ x. Then the
number of ordered pair (x, y), where x and y are (A) (b - a) ( B) 2(b - a)
positive integers less than 30 such that (C ) 3(b - a) (D) 4(b - a)

é x ù é 2 x ù é y ù é 4 y ù 7 x 21y 32. If f :  - {0, 1} ®  is a function such that


êë 2 úû + êë 3 úû + êë 4 úû + êë 5 úû = 6 + 20
æ 1 ö 2(1 - 2 x)
f ( x) + f ç = for all x ¹ 0, 1
is è 1 - x ÷ø x(1 - x)
(A) 1 (B) 2 (C) 3 (D) 4
then the value of f(2) is
24. Let P : [0, ¥) ®  be defined by (A) 1 (B) 2 (C) 3 (D) 4

ì 13 if 0 £ x < 1 33. If f :  ®  is a function satisfying the relations


P ( x) = í f (2 + x) = f (2 – x) and f (7 + x) = f (7 – x) for all x Î 
î13 + 15n if n £ x < n + 1, n Î 
then a period of f is
Then P is (A) 5 (B) 9 (C) 12 (D) 10
(A) an injection
( B) a surjection 34. If f :  ®  is defined by
(C ) a surjection but not an injection
é 1ù é 2ù
(D) neither an injection nor a surjection f ( x) = [ x] + ê x + ú + ê x + ú - 3 x + 5
ë 2û ë 3û
25. If [x] and {x} denote the integral part and the fractional where [x] is the integral part of x, then a period of f is
part of a real number x, then the number of negative (A) 1 (B) 2/3 (C) 1/2 (D) 1/3
real numbers x for which 2[ x] - {x} = x + {x} is
(A) 0 (B) 2 (C) 3 (D) infinite 35. If a function f :  ®  satisfies the relation

26. The number of real numbers x ³ 0 which are solutions f ( x + 1) + f ( x - 1) = 3 f ( x) for all x Î
of [ x] + 3{x} = x + {x} is
then a period of f is
(A) 1 (B) infinite (C) 0 (D) 2
(A) 10 (B) 12 (C) 6 (D) 4
www.jeeneetbooks.in
Exercises 77

36. The domain of f ( x) = 1/ | x | - x is If f is to be a surjection, then A should be


(A) [0, 1) (B) (0, ¥) (C) (- ¥, 0) (D) (1, ¥) (A) é0, 1ù ( B) é - 1 , 0 ù
êë 3 úû êë 3 úû
37. The domain of the function defined by f (x) = min{1 + x,
1 - x} is (C ) é - 1 , 1ù (D) é - 1 , 2 ù
êë 3 úû êë 3 úû
(A) (1, ¥) (B) (-¥, ¥) (C) [1, ¥) (D) (- ¥, 1]
44. Let f : [0, 1] ®  be defined by f (x) = 1 + 2x. If g :
38. The domain of definition of the function f ( x) = y  ®  is an even function such that g( x) = f ( x) for
given by the equation 2x + 2y = 2 is all x Î[0, 1], then, for any x Î, g( x) is equal to
(A) (- ¥, 1) (B) (- ¥, 1) (C) (- ¥, 0) (D) (0, 1) (A) 1 - 2 x ( B) 2 x - 1
(C ) 1 - 2 | x | (D) 1 + 2 | x |
39. The function f : [1, ¥) ® [2, ¥) is defined by f (x) = x +
(1/x). Then f -1 ( x) is equal to 45. Let  be the set of natural numbers and  the set of
x 1 real numbers. Let f :  ®  be a function satisfying
(A) ( B) ( x + x2 - 4 )
1 + x2 2 the following:
(i) f (1) = 1
(C ) 1 ( x - x2 - 4 ) (D) 1 + x2 - 4
2 n

40. Let 0 ¹ a Î  and f ( x) = ax/( x + 1) for all -1 ¹ x Î .


(ii) å r f (r) = n(n + 1) f (n) for all n ³ 2
r =1
If f ( x) = f -1 ( x) for all x, then the value of a is
Then the integral part of f (2009) is
(A) 1 (B) 2 (C) –1 (D) –2
(A) 0 (B) 1 (C) 2 (D) 3
41. If f (x) = k (constant) for all real numbers x, then the
46. A school awarded 22 medals in cricket, 16 medals in
least period of f is
football and 11 medals in kho-kho. If these medals
(A) 1/6 ( B) 1/4 (C ) 1/3 (D) does not exist went to a total of 40 students and only two students got
medals in all the three games, then how many received
42. Let f ( x) = ( x + 1) for all x ³ - 1. If g( x) is the
2
medals in exactly two of the three games.
function whose graph is the reflection of the graph
(A) 7 (B) 6 (C) 5 (D) 4
of f ( x) with respect to the line y = x , then g( x) is
equal to
47. Let P( x) be a polynomial of degree 98 such that
(A) x + 1 ( B) x - 1 P(K) = 1/K for K = 1, 2, 3, … , 99 . Then (50)P (100)
(C ) x + 1 (D) 1 equals
( x + 1)2 (A) 1 (B) 2 (C) 3 (D) 4
43. Let f :  ® A is defined by
48. For any positive integer K, let f1 (K ) denote the
x-1
f ( x) = 2 square of the sum of the digits in K. For example
x - 3x + 3 f1 (12) = (1 + 2)2 = 9 . For n ³ 2, let fn (K ) = f1 ( fn - 1 (K )).
Then f2010 (11) is equal to
(A) 1005 (B) 256 (C) 169 (D) 201

Multiple Correct Choice Type Questions


1. Let Ã( X ) denote the power set of a set X. For any (C ) B = {a, b, x, y}
two sets A and B, if Ã( A) = Ã( B), then (D) A ´ B = {( x, a), ( x, b), ( y, a), ( y, b), (z, a), (z, c)}
(A) A È B = A D B ( B) A = B
(C ) A Ç B = f (D) A D B = f 3. Let A = {1, 2, 3}, B = {3, 4} and C = {1, 3, 5}. Then
(A) n( A ´ ( B È C )) = 12 ( B) n( A ´ ( B Ç C )) = 3
2. Let A and B be two sets such that the number of (C ) n( A ´ ( B - C )) = 3 (D) n ( B ´ ( A - C )) = 2
elements in A ´ B is 6. If three elements of A ´ B are
(x, a), (y, b) and (z, b) then 4. For any three sets A, B and C,
(A) A = {x, y, z} (A) A ´ ( B È C ) = ( A ´ B) È ( A ´ C )
( B) B = {a, b} ( B) A ´ ( B Ç C ) = ( A ´ B) Ç ( A ´ C )
www.jeeneetbooks.in
78 Chapter 1 Sets, Relations and Functions

(C ) A ´ ( B - C ) = ( A ´ B) - ( A ´ C ) Then R is
(D) A ´ ( B D C ) = ( A ´ B) D ( A ´ C ) (A) transitive
(B) an equivalence relation on  ´ 
5. Let A be the set of all non-degenerate triangles in (C ) symmetric
the Euclidean plane and (D) reflexive on  ´ 
R = {( x, y) Î A ´ A | x is congruent to y}
11. Let M2 be the set of square matrices of order 2 over
Then R is the real number system and
(A) reflexive on A R = {( A, B) Î M2 ´ M2 | A = PT BP for some
(B) transitive
(C ) symmetric non-singular P Î M }
(D) an equivalence relation on A Then R is
(A) symmetric
6. Let n be a positive integer and
(B) transitive
R = {(a, b) Î  ´  | n divides a - b} (C ) reflexive on M2
(D) not an equivalence relation on M2
Then R is
(A) transitive 12. Let L be the set of all straight lines in the space and
(B) reflexive on 
(C) symmetric R = {(l, m) Î L ´ L | l and m are coplanar}
(D) an equivalence relation on  Then R is
(A) reflexive on L
7. Let A be the set of all human beings in a particular
( B) not an equivalence relation on L
city at a given time and
(C ) symmetric
R = {( x, y) Î A ´ A | x and y live in the same locality} (D) transitive

Then R is 13. Let * be the set of all non-zero rational numbers and
(A) symmetric
(B) reflexive on A R = {(a, b) Î * ´ * | ab = 1}
(C) transitive Then R is
(D) not an equivalence relation
(A) reflexive on * ( B) not reflexive on *
8. For any integer n, let In be the interval (n, n + 1).
(C ) symmetric (D) not symmetric
Define
14. Let  be the set of all rational numbers,  the set of
R = {( x, y) Î  |both x, y Î In for some n Î } all integers and
Then R is R = {(a, b) Î  ´  | a - b Î }
(A) reflexive on  Then which of the following are true?
(B) symmetric
(A) ( x, 2 x) Î for all x Î
(C) transitive
( B)  ´  Í R
(D) an equivalence relation
(C ) (3 × 5, 4 × 5) Î
(D) (6 × 3, 7 × 2) Î
9. Let  be the set of all real numbers and S = {(a, b) Î
 ´  | a - b £ 0}. Then S is
15. Let A = {1, 2, 3, 4}, B = {5, 6, 7} and C = {a, b, c, d, e}.
(A) reflexive on  Define mappings f : A ® B and g : B ® C by
(B) transitive
(C) symmetric f = {(1, 5), (2, 6), (3, 5), (4, 7)} and g = {(5, b),
(D) an equivalence relation on  (6, c), (7, a)}

10. For any ordered pairs (a, b) and (c, d) of real numbers,
Then which of the following are true?
define a relation, denoted by R, as follows: (A) ( g f )(2) = c ( B) ( g f )(4) = b
(C ) ( g f )(3) = b (D) ( g f )(1) = a
(a, b) R (c, d) if a < c or (a = c and b ≤ d)
www.jeeneetbooks.in
Exercises 79

16. Let A = {1, 2, 3, 4} and f : A ® A and g : A ® A 23. If f :  ®  is defined by f ( x) = ax + b, where a and


be mappings defined by f (1) = 2, f (2) = 3, f (3) = 4, b are given real numbers and a ¹ 0, then
f (4) = 1; g (1) = 1, g (2) = 3, g (3) = 4 and g (4) = 2. (A) f is an injection ( B) f is a surjection
Then which of the following are true? (C ) f is not a bijection (D) f is a bijection
(A) f is a bijection ( B) g is an injection
(C ) g is a surjection (D) f is an injection 24. If f : [0, ¥) ® [0, ¥) is the function defined by
x
17. Let f :  ®  and g :  ®  be mappings defined by f ( x) =
f ( x) = x2 + 3 x + 2 and g( x) = 2 x - 3 . Then which of x+1
the following are true? then
(A) ( f g )(1) = 0 ( B) ( g f )(1) = 9 (A) f is an injection but not a surjection
(C ) ( f g )(3) = 20 (D) ( g f )(3) = 20 ( B) f is a bijection
(C ) Each 0 £ y < 1 has an inverse image under f
18. Let f : ®  and g : ®  be mappings defined by (D) f is a surjection
f ( x) = x2 and g (x) = 2x + 1. If ( f g )( x) = ( g f )( x),
then x is equal to 25. Let f be a real-valued function defined on the inte-
1 rval [–1, 1]. If the area of the equilateral triangle with
(A) -2 + ( B) –2 (0, 0) and (x, f (x)) as two vertices is 3/4, then f (x)
2
is equal to
1
(C ) -2 - (D) 0
2 (A) 1 - x2 ( B) 1 + x2
19. Let f : [–1, ¥) ®  be defined by f (x) = (x + 1) – 1. If2

( f f )( x) = x , then the value of x is (C ) - 1 - x2 (D) - 1 + x2


(A) 1 (B) 0 (C) –1 (D) –2
26. Consider the equation x + y = 1 . Then
2 2

20. Let f :  ®  be a function such that f (x + y) = f (x) + (A) y in terms of x is a function with domain [–1, 1]
f (y) for all x, y Î. Then which of the following hold? (B) y = + 1 - x2 is a function with domain [–1, 1]
(A) f (0) = 0
( B) f is an odd function (C) y = + 1 - x2 is an injection of [0,1] into [0, 1]
(C ) f (n) = nf (1) for n Î (D) y = + 1 - x2 is a bijection of [0,1] onto [0, 1]
(D) f is an even function
27. Let f ( x) = x for all x Î[- 2, 2]. Then f is
2

21. If f :  ®  is a function such that f(0) = 1 and f(x + f(y)) =


(A) an even function
f(x) + y for all x, y Î , then
( B) not an even function
(A) 1 is a period of f (C ) a bijection
( B) f (n) = 1 for all integers n (D) not an injection
(C ) f (n) = n for all integers n
(D) f (–1) = 0

22. Let f, g :  ®  be functions defined by f(x) = ax + b


and g(x) = cx + d, where a, b, c, d are given real
numbers and c ¹ 0. If ( f g )( x) = g( x), then
(A) a = 1 ( B) b = 0
(C ) ab = 1 (D) f (4) = 4

Matrix-Match Type Questions


In each of the following questions, statements are given in matching with one or more statements in Column II. The
two columns, which have to be matched. The statements appropriate bubbles corresponding to the answers to
in Column I are labeled as (A), (B), (C) and (D), while these questions have to be darkened as illustrated in the
those in Column II are labeled as (p), (q), (r), (s) and following example.
(t). Any given statement in Column I can have correct
www.jeeneetbooks.in
80 Chapter 1 Sets, Relations and Functions

Example: If the correct matches are (A) ® (p), (s); where [x] is the largest integer £x. Then match the items
(B) ® (q), (s), (t); (C) ® (r); (D) ® (r), (t) that is if the given in Column I with those in Column II.
matches are (A) ® (p) and (s); (B) ® (q), (s) and (t); (C)
® (r); and (D) ® (r), (t) then the correct darkening of
bubbles will look as follows: Column I Column II

(A) ( g f ) æç ö÷
p q r s t 1 (p) 3
A è 2ø
(q) 0
( B) ( f g ) æç ö÷
B 3
C è 2ø (r) –1
D
(C ) ( f g f ) æç ö÷
3
è 4ø (s) 1
1. Let X be the universal set and A and B be subsets of X.
(D) ( g f g ) æç ö÷
2
Then match the items in Column I with Column II. (t) 2
è 3ø

Column I Column II 5. Let S be the set of all square matrices of order 3 over
(A) A - B = A Û A Ç B = (p) f the real number system. For A Î S, | A | is the determi-
(q) A = B nant value of A. Define f : S ®  by f ( A) = | A. for all
(B) ( A - B) Ç B = A Î S . Then match the items in Column I with those
(r) A – B
(C) ( A - B) È ( B - A) = in Column II.
(s) B Í A
(D) A D B = f Û (t) ( A È B) - ( A Ç B)
Column I Column II
2. Let A, B and C be sets. Then match the items in (A) If (p) 1
Column I with those in Column II.
éa b c ù
A = êêb c a úú
Column I Column II
êë c a búû
(A) A D B = C Û (p) A Ç B = f
with a + b + c = 0, then f (A) = (q) –1
(B) A - ( B È C ) = (q) ( A Ç B) - ( A Ç C )
(r) B D C = f (B) If w ¹ 1 is a cube root of unity
(C) A Ç ( B - C ) =
(s) A = B D C é1 w w2 ù
(D) A D B = A È B Û (t) ( A - B) Ç ( A - C ) ê ú
and A= êw w2 1ú
êw2 1 w úû (r) 3abc - a3 -b3 - c3
3. Let A, B, C and D be sets. Then match the items in ë
Column I with those in Column II.
then f (A)=
(C) If
Column I Column II
é0 5 -7 ù
(A) A ´ ( B È C ) (p) ( A ´ B) Ç ( A ´ C )
A = ê -5 0 11 úú
ê (s) 2
( B) ( A È B) ´ (C È D) (q) ( A ´ C ) Ç ( B ´ D)
êë 7 -11 0 úû
(r) ( A ´ B) È ( A ´ C )
(C ) ( A Ç B) ´ (C Ç D)
(s) ( A ´ C ) È ( B ´ D) then f(A) =
(D) A ´ ( B Ç C ) (t) ( A ´ C ) È ( A ´ D) (D) If A Î S and AAT = I (the unit
È ( B ´ C ) È ( B ´ D) matrix) then f ( A) = (t) 0

4. Let f , g :  ®  be the functions defined by

f (x) = x2 + 1 and g(x) = 2[x] - 1


www.jeeneetbooks.in
Exercises 81

6. Match the items in Column I with those in Column II


Column I Column II

æ k ö
1997
Column I Column II (p) 998.5
(A) å
k =1
f9 ç
è 1998 ÷ø
=
(A) If f is a function such (p) 4
1997
æ k ö (q) 994
that f (0) = 2, f (1) = 3 and
f ( x + 2) = 2 f ( x) - f ( x + 1), (B) å
k =1
f4 ç
è 1998 ÷ø
=

then f(5) is equal to (r) 993


æ k ö
2009

(B) If
(q) 3
(C) å
k =1
f16 ç
è 2010 ÷ø
=

ì x2 , for x ³ 0 (s) 1004


æ k ö
2008
f (x) = í
î x, for x < 0
(D) å
k =1
f25 ç
è 2009 ÷ø
=
(t) 1004.5
(r) 12
then f ( 13 ) =
(C) If f (x) + 2 f (1 - x) = x2 + 2 for 8. Consider the following graphs G1, G2, G3 and G4 and
all x Î, then f(5) is match the items in Column I with those in Column II.
(s) 11
x
4
(D) If f ( x) = for all Column I Column II
4 +2x

x Î , then å f æç k ö÷ =
6
(A) G1 (p) Does not represent a function
(t) 13
k =1
è 7ø (q) Represents an increasing function
(B) G2
(r) Represents an increasing injection
7. For any 0 < a Î  , let (C) G3
(s) Represents a periodic function
ax (D) G4 (t) Represents a bijection
fa ( x) =
ax + a
for all x Î . Then match the items in Column I with
those in Column II.

Y Y

(0,1)
X
X O
O

Group G1 Group G2

Y
Y

X X
O p 2p 3p 4p O 1 2

Group G3 Group G4
www.jeeneetbooks.in
82 Chapter 1 Sets, Relations and Functions

Comprehension-Type Questions
1. Passage: f is a real-valued function satisfying the (ii) The sum of all positive possible values of x such
functional relation: that f(x) = 1 is
(A) 4 (B) 6 (C) 8 (D) 5
æ 2 x + 29 ö
2 f ( x) + 3 f ç = 100 x + 80 for all x ¹ 2 (iii) The number of values of x such that f ( x) = 3 is
è x - 2 ÷ø
(A) 1 (B) 0 (C) 3 (D) 2
Answer the following questions:
4. Passage: Let f(x) = x + |x|. Answer the following ques-
(i) f (0) is equal to
tions.
(A) 754 (B) –754 (C) 854 (D) –854
(i) The range of f (x) is
(ii) f æç -29 ö÷ is equal to (A) [0, ¥) (B) (-¥, 0] (C) (0, ¥) (D) 
è 2 ø (ii) The number of values of x such that f ( x) = x is
(A) 659 (B) –596 (C) 596 (D) –659 (A) 0 (B) 1 (C) 2 (D) infinite
(iii) f (–4) is equal to (iii) The number of values of x such that f ( x) = 0 is
(A) 34 (B) –34 (C) 43 (D) –43 (A) 0 (B) 1 (C) 2 (D) infinite
2. Passage: Let f :  - {0} ®  be a function satisfying 5. Passage: Let f :  ®  be a function satisfying the
functional relation
æ 1ö
f ( x) + 2 f ç ÷ = 3 x ( f ( x))y + ( f ( y))x = 2 f ( xy)
è xø
for all 0 ¹ x Î . Answer the following questions. for all x, y Î and it is given that f (1) = 1/ 2. Answer
the following questions.
(i) xf (x) =
(i) f ( x + y) =
(A) 2 - x2 (B) x2 - 2 (C) x2 - 1 (D) 1 - x2
(A) f ( x) + f ( y) ( B) f ( x) f ( y)
(ii) The number of solution of the equation f (x) =
f (-x) is
y x
(C ) f ( x y ) (D) f ( x)
(ii) f ( xy) = f ( y)
(A) 1 (B) 2 (C) 3 (D) 0
(iii) The number of solutions of the equation f (-x) = (A) f ( x) f ( y) ( B) f ( x) + f ( y)
-f (x) is (C ) ( f ( x))y (D) ( f ( xy))xy
(A) 1 (B) 2 (C) 0 (D) Infinite ¥

3. Passage: It is given that f ( x) = 2 - | 2 x - 5 | .


(iii) å f (k ) =
k =0

Answer the following questions. (A) 5/2 (B) 3/2 (C) 3 (D) 2
(i) The range of the function f is
(A) (-¥, - 1) (B) (-¥, 2)
(C) (-¥, 2] (D) (2, ¥)

Assertion–Reasoning Type Questions


Statement I and statement II are given in each of the drink both coffee and tea. Then the number of adults
questions in this section. Your answers should be as per who drink neither coffee nor tea is 380.
the following pattern:
Statement II: If A and B are two finite sets, then
(A) If both statements I and II are correct and II is a correct
reason for I n( A È B) + n( A Ç B) = n( A) + n( B)
( B) If both statements I and II are correct and II is not a 2. Statement I: In a class of 40 students, 22 drink Sprite,
correct reason for I 10 drink Sprite but not Pepsi. Then the number of
(C ) If statement I is correct and statement II is false students who drink both Sprite and Pepsi is 15.
(D) If statement I is false and statement II is correct Statement II: For any two finite sets A and B,
1. Statement I: In a survey of 1000 adults in a village, it n( A) = n( A - B) + n( A Ç B)
is found that 400 drink coffee, 300 drink tea and 80
www.jeeneetbooks.in
Answers 83

3. Statement I: In a class of 60, each student has to enroll n( A È B È C )


for atleast one of History, Economics and Political
Science. 20 students have enrolled for exactly two of = n[ A - ( B È C )]
these subjects and 8 enrolled for all the three. Then + n[ B - (C È A)] + n[C - ( A È B)]
the number of students who have enrolled for exactly
one subject is 32. + n[( A Ç B) - C ] + n[( B Ç C ) - A]
Statement II: For any three finite sets A, B and C. + n[(C Ç A) - B] + n( A Ç B Ç C )

ANSWERS
Single Correct Choice Type Questions
1. (D) 25. (A)
2. (B) 26. (B)
3. (D) 27. (B)
4. (C) 28. (A)
5. (A) 29. (D)
6. (B) 30. (B)
7. (A) 31. (C)
8. (C) 32. (C)
9. (D) 33. (D)
10. (C) 34. (A)
11. (B) 35. (B)
12. (A) 36. (C)
13. (C) 37. (D)
14. (D) 38. (A)
15. (A) 39. (B)
16. (B) 40. (C)
17. (C) 41. (D)
18. (D) 42. (B)
19. (C) 43. (C)
20. (B) 44. (D)
21. (A) 45. (A)
22. (B) 46. (C)
23. (D) 47. (A)
24. (D) 48. (C)

Multiple Correct Choice Type Questions


1. (B), (D) 15. (A), (C)
2. (A), (B), (D) 16. (A), (B), (C), (D)
3. (A), (B), (C), (D) 17. (A), (B), (C)
4. (A), (B), (C), (D) 18. (B), (D)
5. (A), (B), (C), (D) 19. (B), (C)
6. (A), (B), (C), (D) 20. (A), (B), (C)
7. (A), (B), (C) 21. (A), (B)
8. (B), (C) 22. (A), (B), (D)
9. (A), (B) 23. (A), (B), (D)
10. (A), (D) 24. (A), (C)
11. (A), (B), (C) 25. (A), (C)
12. (A), (B), (C) 26. (B), (C), (D)
13. (B), (C) 27. (A), (D)
14. (B), (C)
www.jeeneetbooks.in
84 Chapter 1 Sets, Relations and Functions

Matrix-Match Type Questions


1. (A) ® (p), (B) ® (p), (C) ® (t), (D) ®(q) 6. (A) ® (t), (B) ® (t), (C) ® (q), (D) ® (q)
2. (A) ® (s), (B) ® (t), (C) ® (q), (D) ® (p) 7. (A) ® (p), (B) ® (p), (C) ® (t), (D) ® (s)
3. (A) ® (r), (B) ® (t), (C) ® (q), (D) ® (p) 8. (A) ®(q), (r), (t), (B) ® (p), (C) ® (s),
4. (A) ® (s), (B) ® (t), (C) ® (t), (D) ® (p) (D) ® (s)
5. (A) ® (r), (t), (B) ® (t), (C) ® (t),
(D) ® (p), (q)

Comprehension-Type Questions
1. (i) (D); (ii) (C); (iii) (B) 4. (i) (A); (ii) (B); (iii) (D)
2. (i) (A); (ii) (B); (iii) (D) 5. (i) (B); (ii) (C); (iii) (D)
3. (i) (C); (ii) (D); (iii) (B)

Assertion–Reasoning Type Questions


1. (A) 3. (A)
2. (D)
www.jeeneetbooks.in

Exponentials and
Logarithms 2
Contents
2.1 Exponential Function
2.2 Logarithmic Function
2.3 Exponential Equations
2.4 Logarithmic Equations
2.5 Systems of Exponential
Exponentials and Logarithms

and Logarithmic
Equations
2.6 Exponential and
Logarithmic
Inequalities

2×2×2=8 Worked-Out Problems


log (8) = 3 Summary
2
3 Exercises
Answers
base

23 = 8

Exponential Function: For


log (8) = 3 any positive real number a,
2
the function f(x) = ax for x Î 
is called exponential function
with base a.
Logarithmic Function: Let
a > 0 and a ¹ 1. Consider the
function g : + ®  defined
by g( y) = x Û y = ax for all
y Î + and x Î  . The func-
tion g is the logarithmic
function denoted by loga.
www.jeeneetbooks.in
86 Chapter 2 Exponentials and Logarithms

In this chapter, we will discuss various properties of exponential and logarithmic functions which are often used in
solving equations, systems of equations, and inequalities containing these functions.

2.1 | Exponential Function


For any positive real number a, we can define ax for all real numbers x. This function is called an exponential function,
whose domain is the set of all real numbers and codomain is also the set of real numbers.

DEF IN IT ION 2 . 1 Let a be any positive real number. Then the function f :  ® , defined by f(x) = ax for all real
numbers x, is called the exponential function with base a.

As usual, we simply say that ax is the exponential function with base a, with the idea that, as x varies over the set of
real numbers, we get a function mapping x onto ax. Note that a must be necessarily positive for ax to be defined for all
x Î. For example (-1)1 2 is not defined in ; for this reason, we take a to be positive.

Examples

(1) 2x is the exponential function with base 2. (4) The constant map which maps each x onto the real
(2) (0.02)x is the exponential function with base 0.02. number 1 is also an exponential function with base 1,
(3) (986)x is the exponential function with base 986. since 1x = 1 for all x Î.

The following theorems are simple verifications and give certain important elementary properties of exponential
function.

T H E O R E M 2.1 Let a be a positive real number. Then the following hold for all real numbers x and y:

1. ax ay = ax + y
2. ax > 0
3. ax / ay = ax - y
4. (ax )y = axy
1
5. a- x =
ax
6. a0 = 1
7. a1 = a
8. 1x = 1

T H E O R E M 2.2 1. If a > 1, then ax is an increasing function; that is, x £ y Þ ax £ ay.


2. If 0 < a < 1, then ax is a decreasing function; that is, x £ y Þ ax ³ ay.
3. If a > 0 and a ¹ 1, ax is an injection; that is, ax ¹ ay for all x ¹ y.
4. For any a > 0 and a ¹ 1, ax = 1 if and only if x = 0.
www.jeeneetbooks.in
2.1 Exponential Function 87

Examples

(1) The function y = 2x is increasing and its graph is given in Figure 2.1.
(2) The function y = (1/ 2)x is decreasing and its graph is given in Figure 2.2.

Y=

y = 2x

(0,1)

X=
0

FIGURE 2.1 Graph of the function y = 2x.

Y=

x
æ 1ö
y =ç ÷
è 2ø

(0,1)

X=
0

FIGURE 2.2 Graph of the function y = (1/2)x.


www.jeeneetbooks.in
88 Chapter 2 Exponentials and Logarithms

2.2 | Logarithmic Function


We have observed in the previous section that, when a > 0 and a ¹ 1, the exponential function with base a is an injection
of  into  and its range is + = (0, + ¥). Therefore the function f :  ® (0, +¥), defined by f(x) = ax, is a bijection and
hence f has an inverse. This implies that there exists a function g : (0, +¥) ®  such that
f ( x) = y Û x = g( y) or y = ax Û g( y) = x
for any x Î and 0 < y Î . This function g is called the logarithmic function with base a. Formally, we have the
following definition.

DEF IN IT ION 2 . 2 Let 0 < a Î  and a ¹ 1. Then the function g : + ® , defined such that
g( y) = x Û y = ax

for all y Î + and x Î , is called the logarithmic function with base a and is denoted by log a.

It is a convention to write log a y instead of log a ( y). Note that log a y is defined only when a > 0, a ¹ 1 and y > 0
and that
loga y = x Û y = ax
for any y Î + and x Î .
The following are easy verifications and these are the working tools for solving exponential and logarithmic
equations and inequalities.

T H E O R E M 2 .3 Let 0 < a Î , a ¹ 1. Then the following hold for any y, y1, y2 Î + and x, x1, x2 Î :

1. aloga y = y
2. loga ax = x
3. loga y = x Û y = ax
4. log a ( y1 y2 ) = log a y1 + log a y2
5. log a (1/y) = - log a y
6. log a ( y1 /y2 ) = log a y1 - log a y2
7. log a ( yz ) = z log a y for all z Î 
8. log a a = 1 and log a 1 = 0

FORMULA FOR 1. For any a, b Î + - {1} and for any y Î +,


TRANSITION TO
A NEW BASE
loga y
logb y = or loga y = loga b logb y
loga b
1
2. logaa ( y) = loga y for any a ¹ 0.
a
PROOF 1. Let loga y = x, logb y = t and log a b = z. Then ax = y, bt = y and az = b and hence
azt = (az )t = bt = y

Therefore loga y = zt = loga b × logb y.


2. For a ¹ 0, (aa )(1/ a )loga y = aloga y = y and therefore
1
logaa ( y) = loga y
a ■
www.jeeneetbooks.in
2.3 Exponential Equations 89

T H E O R E M 2 .4 1. If a > 1, then log a x is an increasing function.


2. If 0 < a < 1, then log a x is a decreasing function.
PROOF This is a consequence of Theorem 2.2 and the fact that, where two functions f and g are inverses to
each other and one function is increasing (decreasing), then so is the other. ■

T H E O R E M 2 .5 For any a > 0 and a ¹ 1, the function log a x is a bijection from the set  + onto .

PROOF This follows from the fact that ax and loga x are functions which are inverses to each other. ■

2.3 | Exponential Equations


It is known from the previous two sections that, for any a > 0, a ¹ 1, the equation ax = b possesses a solution for any
b > 0 and that the solution is unique. In general, the solution is written as x = loga b. If a = 1, then the equation 1x = b
has a solution for b = 1 only. Any real number x can serve as a solution for 1x = 1. Further, for any a > 0, a ¹ 1, the equa-
tion loga x = b has a solution for any b Î and the solution is unique and is written as x = a b. Since the exponential
function ax and the logarithmic function loga x are inverses to each other, the exponential function is often called the
antilogarithmic function.
We often make use of the two transformations, taking logarithms and taking antilogarithms for solving exponential
and logarithmic equations. Taking logarithms to the base a > 0, a ¹ 1 is a transition from the equality
x=y (2.1)
to the equality
loga x = loga y (2.2)
(x and y here can be numbers or the expressions containing the variables). If Eq. (2.1) is true and both sides are
positive, then Eq. (2.2) is also true. Taking antilogarithms to the base a > 0, a ¹ 1, is similar as transition from Eq. (2.2)
to Eq. (2.1). If Eq. (2.2) is true, then Eq. (2.1) is true as well.

Example 2.1

Solve the equation Put 5x-1 = t. The equation reduces to


5x -1 + 5(0.2)x - 2 = 26 t + 25t- 1 = 26
t2 - 26t + 25 = 0
Solution: First observe that
(t - 1)(t - 25) = 0
2
0.2 = = 5- 1 t = 1 or 25
10
and hence Since 5x - 1 = t. We get 5x - 1 = 1 or 52.
(0.2)x - 2 = 5- ( x - 2 ) = 52 - x Solving we get x = 1 or 3.
Therefore, the given equation reduces to
5x - 1 + 53 - x = 26

Example 2.2

Solve the equation which has the same solutions as the original equation.
Since log2 3 = (1/ 2) log2 9, we get that
4 ´ 9x - 1 = 3 22 x + 1
3
Solution: First note that both sides of the given equa- x(log2 9 - 1) = (log2 9 - 1)
2
tion are positive. Taking logarithms with base 2, we get the
equation Since log2 9 ¹ 1, it follows that x = 3 / 2.
1
2 + ( x - 1) log2 9 = log2 3 + (2 x + 1)
2
www.jeeneetbooks.in
90 Chapter 2 Exponentials and Logarithms

Example 2.3

Find the solution(s) of the equation Then

5x ´ 2( 2 x - 1)/( x + 1) = 50 x=2
-1
Solution: The given equation is equivalent to or 1= log5 2
x+1
5x - 2 = 2[ 1-( 2 x - 1)/( x + 1)] 1
x + 1 = log5
5x - 2 = 2( 2 - x )/( x + 1) 2
1 1
By transforming this into logarithmic equation (taking x = log5 - log5 5 = log5
2 10
logarithms with base 5), we get
-( x - 2) Therefore the given equation has two solutions, namely,
x-2= log5 2 2 and log5(1/10).
x+1

2.4 | Logarithmic Equations


Transforming a given logarithmic equation into an exponential equation, we can find solutions of the equations. For
any a > 0, a ¹ 1, the logarithmic equation
loga x = loga y
is equivalent to x = y, where x and y are positive real numbers or expressions containing the variable. We simply write
log x for log10 x or loge x. One has to take it depending on the context. Since
1
log10 x = (loge x)
loge 10
it is easy to pass from logarithms with base 10 to those with base e.

Example 2.4

Find the solution(s) of the equation Note that this is meaningful for all x ¹ 0, whereas the
given equation is valid only when x > 0. It follows that
2 log(2 x) = log( x2 + 75) (2.3)
4 x2 = x2 + 75
Solution: The equation is meaningful only when x > 0. x2 = 25
The given equation can be transformed into
Therefore x = 5 or -5. Equation (2.3) has only 5 as a solu-
log(4 x2 ) = log( x2 + 75) (2.4) tion, whereas Eq. (2.4) has two solutions, namely 5 and –5.

Example 2.5

Find the solution(s) of the equation log2 x + log2 (x -1) = 1. or (x - 2) (x + 1) = 0


Now we have
Solution: The equation is meaningful only when x > 1.
Transforming the sum of logarithms to the logarithm of a x-2=0Þx=2
product, we have x + 1 = 0 Þ x = -1
log2 [ x( x - 1)] = 1 = log2 2
Therefore this has two solutions, namely, 2 and –1.
Therefore However, for the given equation to be meaningful, we
x( x - 1) = 2 should have x > 1. Therefore, 2 is the only solution of
the given equation.
or x2 - x - 2 = 0
www.jeeneetbooks.in
2.5 Systems of Exponential and Logarithmic Equations 91

Example 2.6

Find the solution(s) of the equation This gives


log3 (3 - 8) = 2 - x
x 3x = 9 or 3x = - 1
The equation 3x = -1 has no solution and the equation
Solution: Taking antilogarithms with the base 3 of the
3x = 9 has unique solution, namely 2. Thus, 2 is the only
given equation, we get
solution of the given equation.
3x - 8 = 32 - x
32x - 8 ´ 3x - 9 = 0
(3x - 9)(3x + 1) = 0

Example 2.7

Find the solution(s) of the equation 1


log x = (- log 2 ± log2 2 + 4 log 5 )
2
xlog 2 x = 5
Since log 5 = 1 - log 2, we find that
Solution: By taking logarithms with base 10, we get an
equation log2 2 + 4 log 5 = (log 2 - 2)2
log 2 x ´ log x = log 5 and therefore,
log x(log 2 + log x) = log 5 1
This gives log x = [- log 2 ± (log 2 - 2)]
2
log2 x + log 2 ´ log x - log 5 = 0 Therefore, log x = -1 or 1 - log 2(= log 5). Thus 1/10 and
This is equivalent to the original equation and is mean- 5 are solutions of the given equation.
ingful only when x > 0. Also, the above equation is a
quadratic equation with respect to log x. Therefore

2.5 | Systems of Exponential and Logarithmic Equations


In this section we consider finding solutions simultaneously satisfying a given system of exponential and logarithmic
equations.

Example 2.8

Solve the simultaneous equations Therefore t = 2 or 1/2. Now


logx y + logy x = 2.5 t = 2 Þ logx y = 2 Þ y = x2

xy = 27 t = 1/ 2 Þ logy x = 2 Þ x = y2

Solution: We have to find a common solution to both From the equation xy = 27, it follows that when y = x2
the above equations. Note that 0 < x ¹ 1 and 0 < y ¹ 1. By we get
taking logx y = t in the first equation we get that x3 = 27 Þ x = 3
1 5
t+ = Substituting this value of x we get y = (3)2 = 9. Therefore
t 2 (3, 9) is one solution. Similarly (9, 3) is another solution.
2t2 + 2 = 5t Therefore, (3, 9) and (9, 3) are common solutions for the
given two equations.
(t - 2)(2t - 1) = 0
www.jeeneetbooks.in
92 Chapter 2 Exponentials and Logarithms

Example 2.9

Solve the simultaneous equations From Eq. (2.6), we get

xlog3 y = 27 y (1 + log3 x) log3 x = 4 + log3 x

ylog3 x = 81x (log3 x)2 = 4


log3 x = ± 2
Solution: Taking logarithms with base 3, these equations
can be transformed into Now two situations occur:

log3 y log3 x = 3 + log3 y (2.5) (1) log3 x = 2 Þ x = 9, log3 y = 3 , y = 27


-2 1 1
log3 x log3 y = 4 + log3 x (2.6) (2) log3 x = -2 Þ x = 3 = , log3 y = -1, y =
9 3
Comparing Eqs. (2.5) and (2.6) we get Thus, (9, 27) and (1/ 9, 1/ 3) are the solutions of the given
3 + log3 y = 4 + log3 x system of equations.
log3 y = 1 + log3 x

Example 2.10

Solve the system of equations s + t = 3st


log8 ( xy) = 3 (log8 x × log8 y ) 4( s - t ) = s / t

æ x ö log8 x By solving these two equations, we get that t = 1/2 or 1/6.


4 log8 ç ÷ = Therefore, we have
è y ø log8 y
t = 1/ 2 Þ log8 y = 1/ 2 Þ y = 2 2
Solution: This system of equations can be transformed to
s = 1 Þ log8 x = 1 Þ x = 8
log8 x + log8 y = 3 log8 x ´ log8 y
t = 1/ 6 Þ log8 y = 1/ 6 Þ y = 81/ 6 = 2
log8 x
4(log8 x - log8 y) =
log8 y s = -1/ 3 Þ log8 x = -1/ 3 Þ x = 8- 1/ 3 = 2- 1 = 1/ 2

By putting s = log8 x and t = log8 y, we get Therefore, (8, 2 2 ) and (1/ 2, 2 ) are the solutions of the
given system of equations.

2.6 | Exponential and Logarithmic Inequalities


Let us recall that, if a > 1, the function ax increases and that, 0 < a < 1, the function ax decreases. Also, the function loga x
increases if a > 1, and decreases if 0 < a < 1. These properties can be used to solve some exponential and logarithmic
inequalities.

Example 2.11

Solve the inequality These expressions are meaningful only when 2x/(x + 1) > 0.
Also, the function log9 x is increasing and hence the inequ-
1 2x ality (2.7) is equivalent to the inequality
< log 9 (2.7)
2 x+1
2x
3< (2.9)
Solution: This can be written as x+1

2x Now x cannot be positive [for, if x > 0, then x + 1 > 0 and


log9 3 < log9 (2.8) hence, by Eq. (2.9), 3( x + 1) < 2 x and hence x + 3 < 0, a
x+1
www.jeeneetbooks.in
Worked-Out Problems 93

contradiction to the fact that 2 x /( x + 1) > 0 ]. Therefore and therefore, x > - 3. Thus, the interval (-3, - 1) is the
x < 0. Then x + 1 < 0 and hence x < - 1. Again by Eq. (2.9) set of solutions of the given inequality.
3( x + 1) > 2 x

Example 2.12

Solve the inequality x2 - 2.5 x + 1 ³ 1üï


ý (2.12)
( x2 - 2.5 x + 1)x + 1 £ 1 (2.10) x + 1 £ 0 þï
The system Eq. (2.11) of inequalities has solutions
Solution: This is equivalent to the collection of two 0 £ x < 0.5 and 2 < x £ 2.5. The system Eq. (2.12) has
systems of inequalities solutions x £ - 1. Therefore, the set of solutions of the
0 < x2 - 2.5 x + 1 £ 1üï inequality Eq. (2.10) is
ý (2.11)
x + 1 ³ 0 þï éæ 1 ö æ 5 ö ù
[-¥, - 1] È êç 0, ÷ È ç 2, ÷ ú
ëè 2 ø è 2 ø û

Example 2.13

Solve the inequality This implies


1
2x < 31/ x x< log2 3
(2.13) x
x2 - log2 3
Solution: First note that both sides of this inequality <0 (2.15)
are positive for all x ¹ 0 and therefore, their logarithms x
are defined with respect to any base. In particular, since If x is a solution of Eq. (2.15) and x > 0, then x2 - log2 3 < 0
the function log2 x is increasing, the inequality (2.13) is and hence 0 < x < log2 3 . If x < 0 and is a solution of
equivalent to the inequality
Eq. (2.15), then x2 - log2 3 > 0 and hence x < - log2 3 .
log2 (2x ) < log2 31/ x (2.14) Therefore, the set of solutions of the inequality (2.13) is
(-¥, - log2 3 ) È (0, log2 3 )

WORKED-OUT PROBLEMS
Single Correct Choice Type Questions
log2.5 [( 1 / 3) + ( 1 / 32 ) + +¥ ] -2 log2.5 ( 2 ) log5/ 2 ( 4 )
1. (0.16) = æ 2ö æ 5ö
=ç ÷ =ç ÷ =4
è 5ø è 2ø
(A) 2 2 (B) 2 (C) 4 2 (D) 4
Answer: (D)
Solution: We know that, for any -1 < r < 1,
a 2. If log 12 27 = a, then log 6 16 =
a + ar + ar2 + + ¥ =
1- r æ3+aö æ3-aö
(A) 4 ç ÷ (B) 4 ç ÷
Therefore è3-aø è3+aø
1 1 1/ 3 1 æ3-aö æ3+aö
+ + + ¥ = = (C) 2 ç ÷ (D) 2 ç ÷
3 32 1 - 1/ 3 2 è3+aø è3-aø
Finally we have Solution:
2 log2.5 ( 1 / 2 )
2 æ 2ö log 6 16 = 4 log 6 2 =
4
=
4
(0.16)log2.5 [(1/ 3) + (1/ 3 ) + +¥ ]
=ç ÷ (2.16)
è 5ø log 2 6 1 + log 2 3
www.jeeneetbooks.in
94 Chapter 2 Exponentials and Logarithms

Now, 5. If log3 2 + log3 (2 - 7 / 2) = 2 log3 (2 - 5), then the value


x x

of x is
3 3
a = log12 27 = 3 log12 3 = = (A) 3 (B) 2 (C) 1 (D) 4
log3 12 1 + 2 log3 2
Solution: First note that 2x > 7 / 2 and 2x > 5. Therefore
Therefore
x > 2. From the hypothesis, we have
a(1 + 2 log3 2) = 3
3 3-a 2(2x - 7 / 2) = (2x - 5)2
2 log3 2 = - 1 =
a a Therefore
2 3-a
= 2 ´ 2x - 7 = 22 x - 10 ´ 2x + 25
log2 3 a
2a Put a = 2x . Then 2a - 7 = a2 - 10a + 25 . Therefore
log2 3 =
3-a a2 - 12a + 32 = 0
Substituting in Eq. (2.16), we get that (a - 8)(a - 4) = 0
4 æ 3 - aö Now a = 4 or 8. That is
log6 16 = = 4ç
1 + [2a /(3 - a)] è 3 + a ÷ø
2x = 4 or 8
Answer: (B) x = 2 or 3

3. If log(a + c) + log(a - 2b + c) = 2 log(a - c), then But x > 2. Therefore x = 3.


(A) 2b = a + c (B) a2 + c2 = 2b2 Answer: (A)
2ac
(C) b2 = ac (D) =b 6. If log( 2 x + 3) (6 x2 + 23 x + 21) = 4 - log( 3 x + 7 ) (4 x2 + 12 x + 9),
a+c
then the value of -4x is
Solution:
(A) 0 (B) 1 (C) 2 (D) -1/4
log[(a + c)(a - 2b + c)] = log(a - c) 2
Solution: First note that 2x + 3 > 0 and 2x + 3 ¹ 1, that
is, x > -3 / 2 and x ¹ -1. Also, 3 x + 7 > 0 and 3x + 7 ¹ 1,
(a + c)(a + c - 2b) = (a - c)2
that is, x > -7 / 3 and x ¹ -2. Suppose x > -3 / 2, x ¹ -1.
(a + c)2 - 2b(a + c) = (a - c)2 Then the given equation can be written as

2ac log[(2 x + 3)(3 x + 7)] 2 log(2 x + 3)


b= = 4-
a+c log(2 x + 3) log(3 x + 7)
Answer: (D) log(3 x + 7) 2 log(2 x + 3)
1+ = 4-
log(2 x + 3) log(3 x + 7)
4. The solution of the equation log7 log5( x + 5 + x ) = 0 is
Put
(A) 2 (B) 3 (C) 4 (D) 1
log(3 x + 7)
Solution: =y
log(2 x + 3)
log7 log5 ( x + 5 + x ) = 0 Then
log5 ( x + 5 + x ) = 7 = 10
2
1 1+ y = 4 -
x+5+ x = 5 = 5 y
x + 5 = 25 - 10 x + x Therefore
10 x = 20 2
y=3-
y
x=2
y2 - 3 y + 2 = 0
x=4
( y - 1)( y - 2) = 0
Therefore, x = 4 satisfies the given equation.
Answer: (C) This gives y = 1 or 2.
www.jeeneetbooks.in
Worked-Out Problems 95

Case 1: Suppose that y = 1. Then Þ x ¹ 2, x ¹ 3 and [ x = 2 or 4 or ( x - 3)( x - 5) = 0]

log(3 x + 7) = log(2 x + 3) Þ x = 4 or x=5


3x + 7 = 2 x + 3 Therefore, the number of the solutions of the given
equation is 2.
x = -4
Answer: (B)
This is rejected because x > -3/2.
Alternative Method
Case 2: Suppose that y = 2. Then 2
- 8 x + 15)/( x - 2 )
| x - 3 |( x =1
log(3 x + 7) = 2 log(2 x + 3) = log(2 x + 3) 2
Þ x ¹ 2, x ¹ 3 and | x - 3 | = 1 or x2 - 8 x + 15 = 0
Therefore Þ x ¹ 2, x ¹ 3 and ( x = 4 or 2 or x = 3 or 5)
3 x + 7 = 4 x2 + 12 x + 9
Þ x = 4 or x=5
4 x2 + 9 x + 2 = 0
(4 x + 1)( x + 2) = 0 9. If (x1, y1) and (x2, y2) are solutions of the system of
simultaneous equations
x = - 1/ 4 or - 2
log8 ( xy) = 3 log8 x × log8 y
Here x = -1 / 4 (since x > -3 / 2). So
æ x ö log8 x
-4x = 1 4 log8 ç ÷ =
è y ø log8 y
Answer: (B) then x1 x2 + y1 y2 equals to
(A) 4 (B) 6 (C) 2 (D) 8
7. The number of the solutions of the equation log(x2 -
6x + 7) = log(x - 3) is Solution: Clearly x > 0, y > 0 and y ¹ 1, so as to make
(A) 6 (B) 5 (C) 7 (D) 4 the equations meaningful. The given equations are
equivalent to
Solution: We have, for the term in parentheses on the
RHS of the given equation, log8 x + log8 y = 3 log8 x log8 y
4(log8 x - log8 y) = log8 x /log8 y
x2 - 6 x + 7 = ( x - 3)2 - 2 > 0 Û | x - 3 | > 2
Also, log(x - 3) is defined for all x > 3. From the given Put log8 x = m and log8 y = n ¹ 0. Then the equivalent
equation, x2 - 6 x + 7 = x - 3 , x > 3. Therefore system is
m + n = 3mn ü
x2 - 7 x + 10 = 0, x > 3 ý (2.17)
4(m - n) = m / n þ
( x - 2)( x - 5) = 0, x > 3
Multiplying both the equations of the equivalent system
x=5 we get
Answer: (B)
4(m2 - n2 ) = 3m2
8. The number of solutions of the equation Therefore

m2 = 4 n2 or m = ±2 n
2
- 8 x + 15)/( x - 2 )
| x - 3 |( x =1
is Putting m = 2 n in Eq. (2.17), we get that
(A) 1 (B) 2 (C) 0 (D) 4
1
3n = 6 n2 or n = (since n ¹ 0) and m = 1
Solution: 2
2
- 8 x + 15)/( x - 2 )
| x - 3 |( x =1 Now

x2 - 8 x + 15 m = 1 Þ log8 x = 1 Þ x = 8
Þ x ¹ 3, x ¹ 2 and log | x - 3 | = 0
x-2 1 1
n= Þ log8 y = Þ y = 2 2
Þ x ¹ 2, x ¹ 3 and | x - 3 | = 1 or x - 8 x + 15 = 2
2
2 2
www.jeeneetbooks.in
96 Chapter 2 Exponentials and Logarithms

Therefore Solution: The given inequality is meaningful for x > 0


and is equivalent to
x1 = 8, y1 = 2 2
2
1 é 1 ù
Again by taking m = -2 n, we get that log2 x - 2 ê - log2 x ú + 1 > 0
2 ë 2 û
n = 6 n2 or n = 1/ 6 and m = - 1/3 1 1
log2 x - (log2 x)2 + 1 > 0
1 2 2
-1/ 3 = m = log8 x Þ x = 8-1/ 3 = (23 )-1/ 3 =
2 (log2 x)2 - log2 x - 2 < 0
1/ 6 = n = log8 y Þ y = 81/ 6 = (23 )1/ 6 = 2 (log2 x - 2)(log2 x + 1) < 0

For x2 = 1/ 2 and x2 = 2 . Therefore - 1 < log2 x < 2


1
1 < x < 22
x1 x2 + y1 y2 = 8 ´ +2 2 ´ 2 = 4+4 = 8 2
2
Answer: (C)
Answer: (D)
12. If log3 x( x + 2) = 1, then x is equal to
10. If
(A) 3 or -1 (B) 1 or -4
æ 1 ö (C) -3 or -1 (D) 1 or -3
log10 ç x = x(log10 5 - 1)
è 2 + x - 1÷ø
Solution: log3 x( x + 2) = 1 is meaningful if x( x + 2) ¹ 0
then x is equal to and x( x + 2) > 0 . Also, this equation implies
(A) 1 (B) 2 (C) 3 (D) 0 x( x + 2) = 3
Solution: Given equation is equivalent to x + 2x - 3 = 0
2

æ 1 ö ( x + 3)( x - 1) = 0
log10 ç x = x(log10 5 - log10 10)
è 2 + x - 1÷ø x = -3 or 1
æ 5ö Answer: (D)
= x log10 ç ÷
è 10 ø
13. A solution of the equation
1
= log10 x 1
2 log(2 x) = log( x - 15)4
4
Therefore is
(A) 4 (B) 5 (C) 2 (D) -15
1 1
= Solution: The given equation is meaningful if x > 0 and
2x + x - 1 2x
x ¹ 15. If x > 15, then the given equation is equivalent to
This gives x – 1 = 0 or x = 1 which satisfies the equation. log 2 x = log( x - 15)
Answer: (A)
and hence 2 x = x - 15 and therefore x = -15, which is
11. The set of all values of x satisfying the inequality false (since x > 0). Therefore 0 < x < 15. Then, from the
log2 x - 2(log1/ 4 x)2 + 1 > 0 is the interval given equation

(A) (0, 1) (B) (4, ¥) 1


log(2 x) = log(15 - x)4 = log(15 - x)
4
(C) æç , 4ö÷ (D) æç ,
1 1 1ö
è2 ø è4 ÷
2ø and hence 2 x = 15 - x, so that x = 5.
Answer: (B)
www.jeeneetbooks.in
Worked-Out Problems 97

Multiple Correct Choice Type Questions


1. Which of the following are true? (C) 34 log3 5 + (27)log9 36 = 54 + (33 )(1/ 2 ) log3 ( 36 )
1 1 1
(A) + + + = logn (43)! = 625 + (36)3 / 2 = 625 + 216 = 841
log2 n log3 n log43 n
3 1/ 3
121 + ( 1/ 3) + 1/ 3]
(D) 8log2 = 23[log2 (121)
1 1 1
(B) + + =2
logxy ( xyz) logyz ( xyz) logzx ( xyz) = 2log2 121+ 1 = 121 ´ 2 = 242
(C) If n = (2009)!, then Answers: (A), (B), (D)

1 1 1 3. If x, y, z simultaneously satisfy the equations


+ + + =1
log2 n log3 n log2009 n log2 x + log4 y + log4 z = 2
loga n log3 y + log9 z + log9 x = 2
(D) = loga b - 1
logab n
log4 z + log16 x + log16 y = 2

Solution: then which of the following is (are) true?


43
1 43 43 (A) xy = 9 / 4 (B) yz = 36
(A) å log
k=2 n
= å logn k = å logn k = logn (43)!
k=2 k=2
(C) zx = 64 / 9 (D) x + y + z = xyz
k

1 1 1 Solution: First observe that


(B) + +
logxy ( xyz) logyz ( xyz) logzx ( xyz)
log2 x = log4 ( x2 )
= logxyz ( xy) + logxyz ( yz) + logxyz (zx)
log3 y = log9 ( y2 )
= logxyz ( xy × yz × zx) = 2
log4 z = log16 (z2 )
(C) By (A), the given sum is logn (2009)! = logn n = 1.
loga n logn ab logn a + logn b From log2 x + log4 y + log4 z = 2, we get that
(D) = =
logab n logn a logn a
log4 x2 yz = 2
logn b
= 1+ = 1 + loga b and hence
logn a
x2 yz = 42 = 16 (2.18)
Answers: (A), (B), (C)
Similarly,
2. Which of the following are correct?
y2 zx = 92 = 81 (2.19)
(A) logb a × logc b × logd c × loga d = 1
z2 xy = 162 = 256 (2.20)
4
(B) 22 × 2- log2 5 =
5 From Eqs. (2.18) – (2.20), we get that x y z = 16 ´ 81 ´ 256. 4 4 4

(C) 34 log3 5 + (27)log9 36 = 741 Therefore


3
121 + ( 1 / 3)
(D) 8log2 = 242 xyz = 2 ´ 3 ´ 4 = 24

Solution: Since x2 yz = 16 and xyz = 24, we get that


(A) logb a × logc b × logd c × loga d = logc a × logd c × loga d 16 2
x= =
= logd a × loga d = 1 24 3
-1 4 Similarly, y = 27 / 8 and z = 32 / 3 . Therefore, xy = 9 / 4, yz = 36
(B) 22 × 2- log2 5 = 4 × 2log2 ( 5 )
=
5 and zx = 64 / 9.
Answers: (A), (B), (C)
www.jeeneetbooks.in
98 Chapter 2 Exponentials and Logarithms

Matrix-Match Type Questions


1. Match the items in Column I with those in Column II. Therefore

Column I Column II x = 2, 2-2, 2-1/ 3

(A) The number of real (p) 3 Answer: (B) Æ (p)


solutions of the equation (C)
log4 ( x - 1) = log2 ( x - 3) is
(q) 0 1
(B) The number of solutions of the log0.3( x - 1) < log0.09( x - 1) = log( 0.3)2 ( x - 1) = log0.3( x - 1)
equation 2
2
x[ 3/ 4(log2 x ) + log2 x - 5/ 4 ] = x 2 is (r) 2 Therefore
(C) The smallest positive 2 log10 ( x - 1) log10 ( x - 1)
integer x such that <
(s) 4 log10 (0.3) log10 (0.3)
log0.3 ( x - 1) < log0.09 ( x - 1) is
(D) The minimum value of (t) 1 2 log10 ( x - 1) > log10 ( x - 1)
loga x + logx a, where log10 ( x - 1) > 0
1 < a < x is
x - 1 > 1 or x>2
Solution:
(A) Therefore, the smallest integer x satisfying the given equa-
tion is 3.
log4 ( x - 1) = log2 ( x - 3)
Answer: (C) Æ (p)
1 (D)
Þ log2 ( x - 1) = log2 ( x - 3)
2
1 < a £ x Þ loga x > 0, logx a > 0
Þ x - 1 = ( x - 3)2
Therefore
Þ x2 - 7 x + 10 = 0
Þ x = 2 or 5 loga x + logx a ³ 2(loga x × logx a)1/2 = 2

But the given equation is defined for x > 3. and equality occurs if and only if x = a. Therefore mini-
Therefore x = 5. mum value is 2.
Answer: (A) Æ (t) Answer: (D) Æ (r)
(B)
2. Match the items in Column I with those in Column II.
[ 3 / 4 (log2 x )2 + log2 x - 5 / 4 ]
x = 2
Column I Column II
Taking logarithms on both sides to the base 2,
(A) log2 (log3 81) = (p) 0
é3 5ù 1
êë 4 (log2 x) + log2 x - 4 úû log2 x = 2
2
(q) 1
(B) 34 log9 7 = 7k , then k =
(r) 3
Put log2 x = t. Then (C) 2log3 5 - 5log3 2 = (s) 2
é3 2 5ù 1 (D) log3 [log2 (512)] = (t) 4
êë 4 t + t - 4 úû
t=
2
Solution:
Therefore
(A) log2 (log3 81) = log2 (log3 34 ) = log2 4 = 2
3t + 4t - 5t - 2 = 0
3 2
Answer: (A) Æ (s)
Clearly, t = 1 is root of this equation. Now, (B)

(t - 1)(3t2 + 7t + 2) = 0 34 log9 7 = 7k
t = 1, - 2, - 1/ 3 Þ 34 ´(1/ 2 ) log3 7 = 7k
www.jeeneetbooks.in
Worked-Out Problems 99

Þ (3log3 7 )2 = 7k (D)

Þ 72 = 7k log 3 [log 2 (512)] = log 3 (log 2 2 9 )

Þ k= 2 = log 3 9 = 2
Answer: (B) Æ (s) Answer: (D) Æ (s)
(C)

2log3 5 - 5log3 2 = 2log2 5×log3 2 - 5log3 2


= (2log2 5 )log3 2 - 5log3 2
= 5log3 2 - 5log3 2 = 0
Answer: (C) Æ (p)

Comprehension-Type Questions
1. Passage: It is given that æ ö
(iii) logp ç logp p p p
p p ÷ (n radicals) =
loga (bc) = loga b + loga c, a ¹ 1, a > 0, b > 0, c > 0 è ø
æbö (A) np (B) –n (C) –np (D) n
loga ç ÷ = loga b - loga c, a ¹ 1, a > 0, b > 0, c > 0
ècø
Solution:
n (i)
logam b = loga b, a ¹ 1, a > 0, b > 0, m ¹ 0
n

m
a2 + b2 = 7ab
loga b = logc b /logc a, a ¹ 1, c ¹ 1, a > 0, b > 0, c > 0
(a + b)2 = 9ab
1
loga b = , a ¹ 1, b ¹ 1, a > 0, b > 0 2 log(a + b) = 2 log 3 + log(ab)
logb a

Answer the following questions: æ a + bö


2 log ç = log(ab)
è 3 ÷ø
(i) If a > 0, b > 0 and a2 + b2 = 7ab, then
æ a + bö Answer: (A)
(A) 2 log ç = log(ab)
è 3 ÷ø (ii) The given number can be written as
æ a + bö log3 (135) log3 (15) - log3 5 × log3 405
(B) log ç = log(ab)
è 3 ÷ø
2
= (log3 5 + 3)(1 + log3 5) - (log3 5)(log3 5 + 4) = 3
æ a + bö æ aö
(C) log ç = log ç ÷
è 3 ÷ø
Answer: (C)
è bø
æ ö
æ | a - b |ö
(iii) logp ç log p p p p p ÷ = logp logp ( p1/p )
n
(D) log ç = log a + log b
è 3 ÷ø ç p
 ÷
è n ø
(ii) log3 135 - log3 5 is equal to æ 1ö
log15 3 log405 3 = log p ç n ÷ = -n
èp ø
(A) 4 (B) 5 (C) 3 (D) 0
Answer: (B)

Assertion–Reasoning Type Questions


1. Statement I: If a, b, c are the sides of a right-angled Statement II: a2 = c2 - b2
triangle with c as the hypotenuse and both c + b and
(A) Both Statements I and II are correct and State-
c - b are not equal to unity, then
ment II is a correct explanation of Statement I.
logc + b a + logc - b a = 2 logc + b a ´ logc - b a
www.jeeneetbooks.in
100 Chapter 2 Exponentials and Logarithms

(B) Both Statements I and II are correct and loga (c + b) + loga (c - b)


Statement II is not a correct explanation of =
loga (c + b) loga (c - b)
Statement I.
(C) Statement I is true, but Statement II is false. loga (c2 - b2 )
=
(D) Statement I is false, but Statement II is correct. loga (c + b) loga (c - b)

Solution: In a right-angled triangle, it is known that loga a2


=
the square of the hypotenuse is equal to the sum of the loga (c + b) loga (c - b)
squares of the other two sides. Therefore Statement II is
correct. Also, = 2 logc + b a ´ logc - b a
1 1 Answer: (A)
logc + b a + logc - b a = +
loga (c + b) loga (c - b)

SUMMARY
2.1 Exponential function: For any positive real number a, 2.4 Properties of logarithmic function:
the function f(x) = ax for x Î is called exponential (1) a =y
loga y
function with base a.
(2) log a(ax) = x
2.2 Properties of ax: (3) loga y = x Û y = ax
(1) ax·ay = ax+y (4) loga(y1y2) = logay1 + logay2
x
(2) a > 0 (5) loga (1/ y) = - loga y
ax (6) loga( y1/y2) = logay1 - logay2
(3) y = ax - y
a (7) loga ( yz ) = z loga y for all z Î
(4) (ax)y = axy (8) logaa = 1 and loga1 = 0
(5) a-x = 1/ax
0
(6) a = 1 2.5 Some more important formulae:
1
(7) a = a (1) Change of base: If a, b are both positive and dif-
ferent from 1, and y is positive, then
(8) 1x = 1
x y
(9) For a > 1, if x ≤ y, then a ≤ a (i.e., a is an
x logay = logby × logab
increasing function). 1
(2) logb a ´ loga b = 1 or logba =
(10) If 0 < a < 1, then x ≤ y Þ ax ≥ ay (i.e., ax is a loga b
decreasing function). (3) logx y = log y /log x where both numerator and
(11) If a > 0, then ax is an infection. denominator have common base.
(12) For a > 0 and a ≠ 1, then ax = 1 Û x = 0.
1
(4) logan ( y) = loga y
2.3 Logarithmic function: Let a > 0 and a ≠ 1. Consider n
the function g: + ®  defined by g(y) = x Û y = ax (5) If 0 < a < 1, then loga x is a decreasing function.
for all y Î+ and x Î . This function g is denoted (6) If a > 1, then loga x is an increasing function.
by loga meaning that loga y = x Û y = ax. Note that
loga y is defined only 0 < a ≠ 1 and y > 0.

EXERCISES
Single Correct Choice Type Questions
1. If a > 0, b > 0 and a2 + 4b2 = 12ab, then log(a + 2b) - (A) log a + log b (B) 2(log a + log b)
2 log 2 is equal to 1
(C) 3(log a + log b) (D) (log a + log b)
2
www.jeeneetbooks.in
Exercises 101

2. If 1 < a £ b, then (A) (- ¥, - 3) (B) (2, ¥)


(C) (-2, -1) (D) ( - 2, 0) È (0, 1)
2[ loga 4 ab + logb 4 ab
10. If |log2 ( x2 / 2)| £ 1, then x lies in
- loga 4 b / a + logb 4 a / b ] loga b = (A) (0, 1) (B) [ - 2, - 1] È [1, 2]
(C) (3, ¥) (D) (-¥, - 2)
(A) 1 (B) 2 (C) 3 (D) 4
11. The domain of the function
3. log3 2 × log4 3 × log5 4 × log6 5 × log7 6 × log8 7 =
log2 ( x + 3)
f ( x) =
(A) 1 (B) 3 (C) 1 (D) 2 x2 + 3 x + 2
2 3
is
logx (log2 x + 1) 1
4. log2(2x2) + (log2 x) × x + (log4 x4)2 + 2-3 log1/2 log2 x = (A)  - { - 1, - 2} (B) (-2, ¥)
2 (C)  - {- 1, - 2, - 3} (D) (- 3, ¥) - {- 1, - 2}
(A) (1 + log2 x)3 (B) 1 + log2 x
x ( x - 1)
(C) (1 + log2 x) 2
(D) (1 + log2 x)4 12. Let f : [1, ¥) ® [1, ¥) be defined by f ( x) = 2 .
Then f -1 ( x) is equal to
5. The number of pairs (x, y) satisfying the equations 1
(A) 2- x ( x - 1) (B) (1 + 1 + 4 log2 x )
logy x + logx y = 2 and x = 20 + y is
2 2
1
(A) Infinite (B) 2 (C) 0 (D) 1 (C) (1 - 1 + 4 log2 x ) (D) f -1 ( x) does not exist
2

6. The set of solutions of the inequality logx (2x - 3 / 4) > 13. Let f ( x) = x2 + x + log(1 + | x |) for 0 £ x £ 1. If F(x)
2 is is defined on [-1, 1] such that F(x) is odd and
æ 1ö æ 1 ö æ3 1ö æ 3ö F(x) = f(x) for 0 £ x £ 1, then
(A) ç 0, ÷ È ç , 1 ÷ (B) ç , ÷ È ç 1, ÷
è 2ø è2 ø è8 2ø è 2ø ì f ( x) for 0 £ x £ 1
æ 3ö æ 3ö æ 3ö (A) F ( x) = í 2
(C) ç 0, ÷ È ç 1, ÷ (D) (0, 1) È ç 1, ÷ î- x + x - log(1 + | x |) for - 1 £ x £ 0
è 8ø è 2ø è 2ø (B) F ( x) = x2 + x - log(1 + | x |) for -1 £ x £ 0
(C) F ( x) = - f ( x) for -1 £ x £ 0
7. The set of solutions of the inequality 2 log2 (x - 1) >
log2 (5 - x) + 1 is (D) F ( x) = - x2 + x + log(1 + | x |) for -1 £ x £ 0
(A) (1, 5) (B) (5, ¥)
14. Let W be the set of whole numbers and f : W ® W
(C) (3, 5) (D) (-¥, -3)
be defined by
8. If loga 2 = m and loga 5 = n, where 0 < a ¹ 1, then ìæ é x ùö [log10 x ] æ é x ùö
loga 500 = ïç x - 10 ê ú÷ø 10 + f ç ê ú÷ if x>0
f ( x) = íè ë 10 û è ë 10 ûø
(A) 2m + 3n (B) 3m + 2n ï
(C) 3m + 3n (D) 2m + 2n î0 iff x = 0
where [ y] denotes the largest integer £ y. Then
9. The domain of the function f (x) = [1 / log10 (1 - x)] + f (7752) =
x + 2 is
(A) 7527 (B) 5727 (C) 7257 (D) 2577

Multiple Correct Choice Type Questions


1. If logx (6 x - 1) > logx (2 x), then x belongs to x( y + z - x) y(z + x - y) z( x + y - z)
2. If = = then
log x log y log z
(A) æç ,

(B) æç , +¥ ö÷
1 1
è6 ÷ (A) xy yx = yz zy (B) yz zy = xz zx
4ø è6 ø
(C) xz zy = yz zx (D) xy yx = zx xz
(D) ç , +¥ ö÷
æ 1
(C) (1, + ¥)
è8 ø
www.jeeneetbooks.in
102 Chapter 2 Exponentials and Logarithms

3. A solution of the equation x


log 2 x
= 5 is 6. If f ( x) = log10 (3 x - 4 x + 5), then
2

(A) 0.2 (B) 0.1 (C) 5 (D) 4 (A) Domain of f is 


4. A solution of the system of equations
(B) Range of f is [log10 (11/ 3), + ¥)
(C) f is defined in (0, + ¥)
xx - y = yx + y and x ×y = 1
(D) Range of f is (-¥, log10 (11/ 3)]
is
g (x)
7. If e + e = e, then
x
(A) (1, 1) (B) (1, 3 3 )
(C) (1/ 3 9 , 1) (D) (1/ 3 9 , 3 3 ) (A) Domain of g is (-¥, 1)
(B) Range of g is (-¥, 1)
5. A solution of the inequality log0.2 ( x2 - 4) ³ - 1 satisfies
(C) Domain of g is (-¥, 0]
(A) 1 < x < 2 (B) 2 < x £ 3
(D) Range of g is (-¥, 1]
(C) 3 < x £ 4 (D) 1 < x £ 3

Matrix-Match Type Questions


In each of the following questions, statements are given in 2. Match the items in Column I with those in Column II.
two columns, which have to be matched. The statements in
Column I are labeled as (A), (B), (C) and (D), while those Column I Column II
in Column II are labeled as (p), (q), (r), (s) and (t). Any
given statement in Column I can have correct matching (A) The number of solutions of the
(p) 0
with one or more statements in Column II. The appropriate equation log10(3x2 + 12x + 19) -
bubbles corresponding to the answers to these questions log10(3x + 4) = 1 is
have to be darkened as illustrated in the following example. (q) 3
(B) log 5 (4x - 6) - log 5 (2x - 2) = 2 is
Example: If the correct matches are (A) ® (p), (s); satisfied by x whose number is
(B) ® (q), (s), (t); (C) ® (r), (D) ® (r), (t); that is if the (r) 2
matches are (A) ® (p) and (s); (B) ® (q), (s) and (t); (C) The number of solutions of the
(C) ® (r); and (D) ® (r), (t); then the correct darkening equation log3 (3x - 8) = 2 - x is
(s) 4
of bubbles will look as follows: (D) The number of values of
p q r s t x that satisfy the equation
A 2 log3 ( x - 2) + log3 ( x - 4)2 = 0 is (t) 1
B
C
D 3. Match the items in Column I with those in Column II.

1. Match the items in Column I with those in Column II. Column I Column II

Column I Column II log0.3 x - 2


(A) f ( x) = is defined for x (p) [1, 2)
x
(A) The number of solutions (p) 3 belonging to
of the equation (q) (–2, 1)
2 - x + 3 log5 2 = log5 (3x - 52 - x ) is (B) Domain of the function
(B) The number of values of (q) 1 f ( x) = log[1 - log10 ( x2 - 5 x + 16)] is (r) (2, 3)
x satisfying the equation
(log2 x)2 - 5(log2 x) + 6 = 0 is (r) 4 (C) f ( x) = ( log0.5 ( x2 - 7 x + 13))-1 is (s) (3, 4)
(C) The number of roots of the equation defined for x belonging to
1 (s) 0 (D) Domain of the function
log10 x - 1 + log10 (2 x + 15) = 1 is (t) (2, 3]
2 æ 4 - x2 ö
(D) The number of solutions of the (t) 2 f ( x) = log ç ÷ is
è 1- x ø
equation log7 ( x + 2) = 6 - x is
www.jeeneetbooks.in
Exercises 103

Assertion–Reasoning Type Questions


Statement I and Statement II are given in each of the Statement II: aloga x = x where 0 < a ¹ 1 and x > 0
questions in this section. Your answers should be as per
the following pattern: 3. Statement I: The equation log(2 + x )-1 (5 + x2 ) =
(A) If both Statements I and II are correct and II is a log3 + x2 (15 + x ) has no solution.
correct reason for I
1
(B) If both Statements I and II are correct and II is not a Statement II: logbm a = logb a
m
correct reason for I
(C) If Statement I is correct and Statement II is false 4. Statement I: The equation 9log3(log2 x) = log2 x - (log2 x)2 + 1
(D) If Statement I is false and Statement II is correct. has only one solution.
Statement II: aloga x = x and logaxn = nlogax, where
1. Statement I: If a = x2, b = y2 and c = z2, where x, y, x > 0.
z are non-unit positive reals, then 8(loga x3)(logb y3)
(logc z3) = 27. 5. Statement I: If n is a natural number greater than 1
such that n = p1a1 p2a2 pkak, where p1 , p2 , ¼, pk are dis-
Statement II: logb a × loga b = 1 tinct primes and a 1 , a 2 ,… , a k are positive integers,
2
then log n ³ k log 2.
2. Statement I: If xlogx (1- x ) = 9, then x = 3.
Statement II: loga x > loga y when x > y and a > 1.

Integer Answer Type Questions


The answer to each of the questions in this section is a 3. The value of x satisfying the equation 62x+4 = (33x) (2x+8)
non-negative integer. The appropriate bubbles below the is .
respective question numbers have to be darkened. For 2
-1
example, as shown in the figure, if the correct answer to 4. The number of solutions of the equation | x - 2 |10 x =
the question number Y is 246, then the bubbles under Y | x - 2 |3 x is .
labeled as 2, 4, 6 are to be darkened.
5. The number of ordered pairs (x, y) satisfying the
X Y Z W
two equations 8( 2 )x - y = (0.5)y - 3 and log3 (x - 2y) +
0 0 0 0
log3 (3x + 2y) = 3 is .
1 1 1 1
6. If (x1, y1) and (x2, y2) are the solutions of the simultane-
2 2 2
ous equations x + y = 12 and 2(2 logy2 x - log1/ x y) = 5,
3 3 3 3
then x1 x2 - y1 y2 is equal to .
4 4 4
7. The number of solutions of the system of equations
5 5 5 5
y = 1 + log4 x, xy = 46 is .
6 6 6
7 7 7 7 8. The number of integers satisfying the inequality
8 8 8 8
3( 5 / 2 )log3 (12 - 3 x ) - 3log2 x > 83 is .
9 9 9 9 9. The number of integer values of x satisfying the
inequality 2 x + 1 < 2 log2 ( x + 3) is .

æ 4 ö æ 1 ö
1. 5log1/5 (1/ 2 ) + log 2 ç + log1/ 2 ç =
÷
è 3 + 7ø è 10 + 2 21 ÷ø
.

( 81)1/ log 9 + 3
5 3 / log 6
3

[( 7 )2 / log25 7 - ( 125)
log25 6
2. ]= .
409
www.jeeneetbooks.in
104 Chapter 2 Exponentials and Logarithms

ANSWERS
Single Correct Choice Type Questions
1. (D) 8. (A)
2. (B) 9. (D)
3. (C) 10. (B)
4. (A) 11. (D)
5. (D) 12. (B)
6. (B) 13. (A)
7. (C) 14. (D)

Multiple Correct Choice Type Questions


1. (A), (C) 5. (B), (D)
2. (A), (B), (D) 6. (A), (B), (C)
3. (B), (C) 7. (A), (B)
4. (A), (D)

Matrix-Match Type Questions


1. (A) ® (q), (B) ® (t), (C) ® (q), (D) ® (q) 3. (A) ® (p), (r), (t); (B) ® (r),
2. (A) ® (r), (B) ® (t), (C) ® (t), (D) ® (t) (C) ® (s), (D) ® (q)

Assertion–Reasoning Type Questions


1. (A) 4. (A)
2. (D) 5. (A)
3. (A)

Integer Answer Type Questions


1. 6 6. 0
2. 1 7. 2
3. 4 8. 2
4. 2 9. 4
5. 1
www.jeeneetbooks.in

Complex Numbers
3
Contents
3.1 Ordered Pairs of Real
Numbers
3.2 Algebraic Form a + ib
3.3 Geometric
Interpretation
3.4 The Trigonometric
Form
A: What do you mean?
B: Well, what if we make up a 3.5 De Moivre’s Theorem
number, say ‘i’, so that 3.6 Algebraic Equations
i × i = -1
A: Can we do that?
Worked-Out Problems
B: Why not!
A: But there is no such number Summary
that has that size. Exercises
B: I know, but the idea can exist in Answers
our imagination! I think we should
call it an imaginary number.
Complex Numbers

Any ordered pair (a, b) where


a and b are real numbers is
Firsts Lasts called a complex number.
The set of all complex num-
bers is denoted by  which is
(a+bi)(c+di)  ´ .
Inners
Outers
www.jeeneetbooks.in
106 Chapter 3 Complex Numbers

It is well known that there is no real number a for which a2 = -1. In other words, the equation x2 + 1 = 0 has no root
in the real number system . Likewise, the equation x2 + x + 1 = 0 has no root in . For this reason, the real number
system  is enlarged to a system  in such a way that every polynomial equation, with coefficients in , has a root in .
The members of  are called complex numbers. Infact, the system  of complex numbers is the smallest expansion of
the real number system  satisfying the above property. In this chapter we will discuss the construction and several
properties of the system of the complex numbers.

3.1 | Ordered Pairs of Real Numbers


A complex number can be defined as an ordered pair of real numbers. Let  denote the set of real numbers and
=´
That is,  is the set of all ordered pairs (a, b) such that a and b are real numbers. We will introduce all the
arithmetical concepts of addition, subtraction, multiplication, and division among members of . The members of  are
called complex numbers. First let us recall that two ordered pairs (a, b) and (c, d) are said to be equal if a = c and b = d.

Mathematical Operations on Complex Numbers


DEF IN IT ION 3 . 1 For any complex numbers (a, b) and (c, d), let us define

(a, b) + (c, d) = (a + c, b + d)
(a, b) - (c, d) = (a - c, b - d)
(a, b) + (c, d) is called the sum of (a, b) and (c, d) and the process of taking sum is called the
addition. Similarly (a, b) - (c, d) is called the difference of (c, d) with (a, b) and the process of
taking difference is called the subtraction.

Try it out Verify the following properties:


1. ((a, b) + (c, d)) + (s, t) = (a, b) + ((c, d) + (s, t))
2. (a, b) + (c, d) = (c, d) + (a, b)
3. (a, b) + (0, 0) = (a, b)
4. (a, b) + (-a, -b) = (0, 0)
5. (a, b) + (c, d) = (s, t) Û (a, b) = (s, t) - (c, d)
Û (c, d) = (s, t) - (a, b)

DEF IN IT ION 3 . 2 For any complex numbers (a, b) and (c, d), let us define

(a, b) × (c, d) = (ac - bd, ad + bc)


This is called the product of (a, b) and (c, d) and the process of taking products is called
multiplication.

Try it out Verify the following properties for any complex numbers (a, b), (c, d) and (s, t).
1. [(a, b) × (c, d)] × ( s, t ) = (a, b) × [(c, d) × ( s, t )]
2. (a, b) × (c, d) = (c, d) × (a, b)
3. (a, b) × [(c, d) + ( s, t )] = (a, b) × (c, d) + (a, b) × ( s, t )
4. (a, b) × (1, 0) = (a, b)
5. (a, 0) × (c, d) = (ac, ad)
6. (a, 0) × (c, 0) = (ac, 0)
7. (a, 0) + (c, 0) = (a + c, 0)
www.jeeneetbooks.in
3.1 Ordered Pairs of Real Numbers 107

Properties 6 and 7 in “Try it out” suggest that, when we identify any real number a with the complex number
(a, 0), then the usual arithmetics of real numbers are carried over to the complex numbers of the form (a, 0). Further
one can easily observe that the mapping a  (a, 0) is an injection of  into . Therefore, we can identify  with the
subset  ´ {0} of . This also suggests that any real number a can be considered as a complex number (a, 0). Thus  is
an enlargement of  without disturbing the arithmetics in .

Examples

æ1 2ö æ 1 2ö
Let z1 = (2, 3) and z2 = ç , ÷ , then (3) z1 × z2 = (2, 3) × ç , ÷
è2 3ø è 2 3ø
æ 1 2 2 1ö
æ1 2ö æ 1 2 ö æ 5 11ö = ç2 ´ - 3 ´ , 2 ´ + 3 ´ ÷
(1) z1 + z2 = (2, 3) + ç , ÷ø = çè 2 + , 3 + ÷ =ç , ÷ è 2 3 3 2ø
è2 3 2 3ø è 2 3 ø
æ 4 3ö
= ç 1 - 2, + ÷
æ1 2ö æ 1 2ö æ 3 7ö è 3 2ø
(2) z1 - z2 = (2, 3) - ç , ÷ø = çè 2 - , 3 - ÷ =ç , ÷
è2 3 2 3ø è 2 3ø æ 17 ö
= ç -1, ÷
è 6ø

Zero and Unity in Complex Numbers

DEF IN IT ION 3 . 3 The complex numbers (0, 0) and (1, 0) are called the zero and unity, respectively, and are
simply denoted by 0 and 1. Note that these are the real numbers 0 and 1 also, since, for any
real number a, we identify a with the complex number (a, 0).

T H E O R E M 3 .1 For any non-zero complex number z, there exists a unique complex number s such that z × s = 1 [= (1, 0)].

PROOF Let z = (a, b) be a non-zero complex number; that is, z ¹ (0, 0) and hence either a ¹ 0 or b ¹ 0 so
that a2 + b2 is a positive real number. Put

æ a -b ö
s=ç 2 , 2
è a + b a + b2 ÷ø
2

Then
æ a -b ö
z × s = (a, b) × ç 2 , 2
è a + b a + b2 ÷ø
2

æ a2 b(- b) a(- b) ba ö
=ç 2 - 2 , 2 + 2
èa + b2
a +b a +b
2 2
a + b2 ÷ø
= (1, 0) = 1
If (c, d) is any complex number such that
(a, b) × (c, d) = (1, 0)
then ac - bd = 1 and ad + bc = 0. From these we can derive that
a -b
c= and d=
a + b2
2
a + b2
2

Thus, s is the unique complex number such that z × s = 1. ■

Multiplicative Inverse
DEF IN IT ION 3 . 4 The unique complex number s such that z × s = 1 is called the multiplicative inverse of z and
is denoted by 1/z or z-1. Also, z1 × (1/z2) will be simply expressed as z1/z2.
www.jeeneetbooks.in
108 Chapter 3 Complex Numbers

C O R O L L A RY 3.1 For any complex numbers z1 and z2,


z1 × z2 = 0 Û z1 = 0 or z2 = 0

Examples

(1) If z = (2, 3), then (3) If z = (0, 1), then


1 æ 2 -3 ö æ 2 -3 ö 1
=ç 2 , 2 =ç , ÷ = (0, - 1)
z è 2 + 3 2 + 32 ÷ø è 13 13 ø
2
z
(2) If z = (4, 0), then Infact, if z = (0, b), then
1 æ 4 -0 ö æ 1 ö 1 æ - 1ö
=ç 2 , 2 = ç , 0÷ = ç 0, ÷
z è 4 + 0 4 + 02 ÷ø è 4 ø
2
z è bø
Infact, if z = (a, 0), then (4) (0, 1) × (0, 1) = (-1, 0)
1 æ1 ö
= ç , 0÷
z èa ø

3.2 | Algebraic Form a + ib


Even though there is no real number a such that a2 = -1, there is a complex number z such that z2 (= z × z) = - 1; for
consider the complex number (0, 1). We have
(0, 1) × (0, 1) = (-1, 0) = -1
Also,
(0, -1) × (0, -1) = (-1, 0) = -1
Infact, (0, 1) and (0, -1) are the only complex numbers satisfying the equation z2 = -1. For if z = (a, b) and z2 = - 1, then
(-1, 0) = -1 = (a, b) × (a, b) = (a2 - b2 , 2ab)
and hence a2 - b2 = -1 and 2ab = 0. Since b ¹ 0 (for, if b = 0, then a is a real number such that a2 = -1), it follows that
a = 0 and b = ± 1 and hence z = (0, 1) or (0, -1).
Note: We will denote the complex number (0, 1) by the symbol i (indicating that it is an imaginary number). By the
above discussion, we have i2 = -1 = (-i)2. Recall that we are identifying a real number a with the complex number
(a, 0). With this notation, we have the following theorem.

T H E O R E M 3 .2 Any complex number z can be uniquely expressed as


z = a + ib
where a and b are real numbers and i = (0, 1). This expression is called the algebraic form of z.

PROOF Let z be a complex number. Then z = (a, b) where a and b are real numbers. Now consider
z = (a, b) = (a, 0) + (0, 1)(b, 0) = a + ib
Clearly a and b are unique real numbers such that z = a + ib. ■

Note: We can perform the algebraic operations addition and multiplication with much ease when we consider the
complex numbers in the form a + ib. We can sum or multiply as in the real number system by substituting -1 for i2.

DEFINITION 3.5 Let z be a complex number and z = a + ib, where a and b are real numbers. Then a is called the real
part of z and is denoted by Re(z). Also, b is called the imaginary part of z and is denoted by Im(z).
www.jeeneetbooks.in
3.2 Algebraic Form a + ib 109

By the uniqueness of the real and imaginary parts of a complex number, it follows that, for any complex numbers
z1 and z2,
z1 = z2 Û Re(z1 ) = Re(z2 ) and Im(z1 ) = Im(z2 )

Example 3.1

Write (2 + 3i)2 (3 + 2i) in the form a + ib. = (-5 + 12i)(3 + 2i)

Solution: Consider = (-15 - 24) + (-10 + 36)i


= -39 + 26i
(2 + 3i)(2 + 3i)(3 + 2i) = (4 - 9 + 12i)(3 + 2i)

Example 3.2

Find the real and imaginary parts of = 21 - 20 + (21 + 20)i


z = (1 + i)(5 + 2i)2
= 1 + 41i

Solution: Consider Therefore, Re(z) = 1 and Im(z) = 41.

z = (1 + i)(5 + 2i)2 = (1 + i)(25 - 4 + 20i)


= (1 + i)(21 + 20i)

Example 3.3

Find the real and imaginary parts of 25 - 1 - 10i


=
(1 + i)(2 - 3i) 25 + 1
z=
(1 - i)(2 + 3i) 24 æ -10 ö 12 æ -5 ö
= +ç ÷ i= +ç ÷i
26 è 26 ø 13 è 13 ø
Solution: Consider
(1 + i)(2 - 3i) (2 + 3) + (2 - 3)i Therefore Re(z) = 12 / 13 and Im(z) = -5 / 13.
z= =
(1 - i)(2 + 3i) (2 + 3) + (- 2 + 3)i
5-i (5 - i)2
= =
5 + i (5 + i)(5 - i)

Example 3.4

Cube roots of unity


Þ z = 1 or (- 8a3 = 1 and b = ± 3a2 )
Compute all the complex numbers z such that z3 = 1.
æ -1 3ö
Solution: Let z = a + ib. Then Þ z = 1 or ç a = and b = ± ÷
è 2 2 ø
z3 = 1 Þ (a + ib)2 (a + ib) = 1
-1 3 -1 æ 3 ö
Þ z = 1; or z = + i; or z = - i
Þ (a2 - b2 + 2abi)(a + ib) = 1 2 2 2 çè 2 ÷ø
Þ (a2 - b2 )a - 2ab2 + (2a2 b + a2 b - b3 )i = 1
Therefore
ab2 ) + (3a2 b - b3 )i = 1 + 0i
Þ (a3 - 3a
Þ a3 - 3ab2 = 1 and 3a2 b - b3 = 0 -1 + 3i -1 - 3i
1, and
2 2
Þ a(a2 - 3b2 ) = 1 and b(3a2 - b2 ) = 0
Þ (b = 0 and a = 1) or [b2 = 3a2 and a(a2 - 3b2 ) = 1] are all the complex numbers z for which z3 = 1.
www.jeeneetbooks.in
110 Chapter 3 Complex Numbers

Aliter: Now
z3 - 1 = 0 Û (z - 1)(z2 + z + 1) = 0 -1 + i 3 -1 - i 3
and
Ûz=1 or z + z+ 1= 0
2 2 2

-1 ± i 3 are having the property that each is the square of the


Ûz=1 or z= other. If we denote one of them as w, then the other will
2 be w2 and, further, 1 + w + w2 = 0.
Thus, cube roots of unity are

-1 ± i 3
1,
2

Example 3.5

Express the complex number 3+i


=
3+i 1 + 2 + i - 2i
z=
(1 + i)(1 - 2i) 3+i (3 + i)2
= =
3 - i (3 - i)(3 + i)
in the algebraic form.
9 - 1 + 6i 8 + 6i 4 + 3i
= = =
Solution: Consider 9+1 10 5
3+i =
4
+i
3
z=
(1 + i)(1 - 2i) 5 5

QUICK LOOK 1

Let us summarize and record the arithmetical opera- 1 a b


4. = -i 2
tions on the complex numbers in algebraic form. a + ib a2 + b2 a + b2
1. (a + ib) + (c + id) = (a + c) + i(b + d) a + ib 1
5. = (c - id)(a + ib)
c + id c2 + d2
2. (a + ib) - (c + id) = (a - c) + i(b - d)
1
3. (a + ib) × (c + id) = (ac - bd) + i(ad + bc) = [(ac + bd) + i(bc - ad)]
c2 + d 2

DEF IN IT ION 3 . 6 A complex number z is called purely real if Im(z) = 0 and is called purely imaginary if
Re(z) = 0.

Note: A complex number is both purely real and purely imaginary if and only if it is 0 (= 0 + i0).

Examples

(1) If x is a positive real number such that (x + i)2 is (2) If x is a real number such that (2x + i)2 is purely real,
purely imaginary, then then

0 = Re( x + i)2 = Re[ x2 - 1 + 2 xi] = x2 - 1 0 = Im(2 x + i)2 = Im[4 x2 - 1 + 4 xi] = 4 x


and hence x = 1 (since x > 0). and hence x = 0.
www.jeeneetbooks.in
3.2 Algebraic Form a + ib 111

QUICK LOOK 2

Let us turn our attention to all the integral powers of i. Infact, for any integer n,
Recall that i [= (0, 1)] is a complex number such that
i2 = - 1. Now, ì 1 if n is a multiple of 4
ï i n - 1 is a multiple of 4
ï if
i 0 = 1, i 1 = i, i 2 = - 1, i 3 = - i, i 4 = 1 in = í
ï-1 if n - 2 is a multiple of 4
Also, ïî -i if n - 3 is a multiple of 4
æ 1ö
i -1ç = ÷ = - i, i -2 = - 1, i -3 = i, i -4 = 1, …
è iø

T H E O R E M 3 .3 The sum of any four complex numbers which are consecutive powers of i is zero.

PROOF Let z1, z2 , z3, z4 be any four consecutive powers of i. Then, there is an integer n such that

z1 = in, z2 = in + 1, z3 = in + 2 and z4 = in + 3
Among the powers of i, 1, i, -1, -i occur cyclically and hence z1 + z2 + z3 + z4 = 0. ■

Examples

(1) i2009 = i4( 502 ) + 1 = (i4 )502 × i1 = 1502 × i = i


å i = i + i2 + å n = 3 in = i - 1 + 0 = - 1 + i
2010 n 2010
(3) n=1

(2) i1947 + i1948 + i1949 + i1950 = i3 + i4 + i5 + i 6


å i n = å n = 1 i n - å n = 1 i n = i - (i + i 2 ) = 1
3005 3005 1002
(4)
= -i + 1 + i - 1 = 0 n = 1003

DE F IN IT ION 3 . 7 For any complex number z = a + ib (a and b are real numbers), the conjugate of z is defined as

z = a - ib

In the following theorem, whose proof is a straight forward verification, we list several properties of the conjugates of
complex numbers.

QUICK LOOK 3

The following hold for any complex numbers z, z1 and z2. 8. z1 × z2 = z1 × z2


1. (z ) = z
æz ö z
9. If z2 ¹ 0, ç 1 ÷ = 1
z+z è z2 ø z2
2. Re(z) =
2
10. z × z is a non-negative real number
z-z
3. Im(z) = 11. zz = 0 Û z = 0 Û z = 0
2i
4. z = z Û z is purely real 12. z1z2 + z1z2 = 2 Re(z1z2 ) = 2 Re(z1z2 )

5. z = - z Û z is purely imaginary 13. z1z2 - z1z2 = - 2i Im(z1z2 ) = 2i Im(z1z2 )


14. For any polynomial f(x) with real coefficients, f (z) =
6. z1 + z2 = z1 + z2
f (z )
7. z1 - z2 = z1 - z2
www.jeeneetbooks.in
112 Chapter 3 Complex Numbers

T H E O R E M 3 .4 For any complex numbers z and w, with w ¹ 0, there exists a complex number z1 such that
wz1 = z
This z1 is unique and is denoted by z/w.

PROOF Let z = a + ib and w = c + id, where a, b, c and d are real numbers such that c2 + d2 > 0. Put
1
z1 = zw
c2 + d 2
Then
1 1
wz1 = w × zw = [(c + id)(c - id)z] 2 =z
c +d
2 2
c + d2
Also, for any complex number z2,
wz2 = z Þ wwz2 = wz
1
Þ z2 = wz = z1
c2 + d 2 ■

Example 3.6

Find a complex number z such that (2 + 3i)z = 3 - i. Then


1
Solution: Take (2 + 3i)z = (2 - 3i)(2 + 3i)(3 - i)
2 + 32
2

1
z= (2 - 3i)(3 - i) 22 + 32
2 + 32
2
= (3 - i) = 3 - i
22 + 32

Example 3.7

4 + 3i 1
Express in the form a + ib. = (8 + 3 + 6 i - 4 i )
2+i 22 + 12
Solution: Consider 11 2
= + i
4 + 3i (4 + 3i) (2 - i) 5 5
=
2+i (2 + i) (2 - i)

3.3 | Geometric Interpretation


We have introduced the concept of a complex number as an ordered pair of real numbers that can be viewed as a point
in the plane with respect to a given coordinate system. Infact, given a coordinate system in the plane, there is a one-
to-one correspondence between the complex numbers and the points in the plane. This makes it possible to consider
a complex number a + ib as the point (a, b) in the coordinate plane. For this reason, the plane is called ARGAND’S
plane or complex plane. The abscissa axis is called the real axis or the axis of real numbers, containing the points
of the form (a, 0), where a is a real number. The ordinate axis is called the imaginary axis or axis of imaginaries,
containing the points of the form (0, b), where b is a real number. 
For any complex number z = a + ib, it is often convenient to represent z by the vector OM, where M is the point
(a, b) in the plane and O is the origin. Also, every vector in the plane begining at the origin O(0, 0) and terminating
at the point M(a, b) can be associated with the complex number a + ib. The origin O(0, 0) is associated with the zero
vector (Figure 3.1).
www.jeeneetbooks.in
3.3 Geometric Interpretation 113

M
b z = a + ib

x
O a

FIGURE 3.1 Graphical representation of a complex number of the form z = a + ib.

Representation of complex numbers as vectors facilitates a simple geometrical interpretation of operations on


complex numbers. First, let us consider the addition of complex numbers. Let z1 = a1 + ib1 and z2 = a2 + ib2 be two
complex numbers represented by the points M1 and M2 in the plane as shown in Figure 3.2.

z1 +z2 = (a1 +a2)+i(b1 +b2)


b1 +b2 M

b1 M1
z2 = a2 +ib2 z1 = a1 +ib1
M2 b2

x
a2 O a1 +a2 a1

FIGURE 3.2 Geometrical interpretation of operations on complex numbers.

When z1 and z2 are added, their real and imaginary parts are added up (see Figure 3.2). When adding up vectors
 
OM 1 and OM 2 corresponding to z1 and z2, their coordinates are added. Therefore, with the correspondence which we
have established between complex numbers and vectors, the sum z1 + z2 of the numbers z1 and z2 will be associated with
  
the vector OM which is equal to the sum of the vectors OM 1 and OM 2 . Thus, a sum of complex numbers can be inter-
preted in terms of geometry as a vector equal to the sum of the vectors corresponding to the complex numbers (in other
 
words, it also corresponds to the fourth vertex of the parallelogram
constructed
 with OM 1 and OM 2 as adjacent sides).
For any complex number z = a + ib, the length of the vector OM corresponding to z has special importance. This
is same as the distance of the point (a, b) from the origin O in the plane. This is termed as modulus of z and is denoted
by | z |. The concept of the modulus of a complex number plays a vital role in the analysis of complex numbers. By the
Pythagorean Theorem, it follows that the modulus of a + ib is a2 + b2 . The following is a formal definition of the
modulus of a complex number.

Modulus of z

DE FIN IT ION 3 . 8 Let z = a + ib be a complex number, where a and b are real numbers. The modulus of z is
defined as a2 + b2 , the non-negative square root of a2 + b2 and is denoted by | z |. That is,

| z |2 = a2 + b2 = [Re(z)]2 + [Im(z)]2
www.jeeneetbooks.in
114 Chapter 3 Complex Numbers

Try it out It can be easily seen that zz = (a + ib)(a - ib) = a2 + b2 = | z | 2

In the following theorem, we list various properties of the modulus of a complex number and the proofs of these
are straight forward routine verifications.

QUICK LOOK 4

8. | z | = | z | for all integers n


n n
The following hold for any complex numbers z, z1 and z2:
1. | z | is a real number and | z | ³ 0 9. |z1 + z2 | 2 = (z1 + z2 )(z1 + z2 ) = | z1 | 2 + | z2 | 2 + (z2 z1 + z2 z1)
2. | z | = 0 if and only if z = 0 = | z1 | 2 + | z2 | 2 + 2 Re(z1z2 )
3. | z | = | -z | = | z | = | -z | 10. |z1 - z2 | = (z1 - z2 )(z1 - z2 ) = | z1 | + | z2 | + (z2 z1 + z2 z1)
2 2 2

4. | z1z2 | = | z1 || z2 | = | z1 | 2 + | z2 | 2 - 2 Re(z1z2 )
5. | z | = zz
2
11. | z1 + z2 | 2 + | z1 - z2 | 2 = 2[| z1 | 2 + | z2 | 2 ]
z1 | z1 |
6. = , if z2 ¹ 0 12. || z1 | - | z2 || £ | z1 ± z2 | £ | z1 | + | z2 |
z2 | z2 |
Note that || z1 | - | z2 || = | z1 - z2 | if and only if z1, z2 are
7. | z1 ± z2 | £ | z1 | + | z2 | collinear with the origin on the same side of the origin.
Note that | z1 + z2 | = | z1 | + | z2 | if and only if the points
z1, z2 are collinear with the origin and lie on the same
side of the origin.

Property 12 above says that | z1 | + | z2 | is the greatest possible value of | z1 ± z2 | and || z1 | - | z2 || is the least possible
value of | z1 ± z2 |.

Unimodular Complex Number

DEF IN IT ION 3 . 9 A complex number z is said to be unimodular if its modulus is 1, that is, | z | = 1.

Note that, for any non-zero complex number z, z / | z | is always unimodular and
z
z = | z|×
| z|

This implies that z can be expressed as


z = rw
where 0 < r Î  and | w | = 1. Moreover, this expression is unique, since
1 z
| z | = | rw | = | r || w | = r × 1 = r and w = z =
r | z|

Example 3.8

If z1 and z2 are non-zero complex numbers such that Therefore


(z1 - z2)/(z1 + z2) is unimodular, then prove that iz1/z2 is
a real number. (z1 /z2 ) - 1
=1
(z1 / z2 ) + 1
Solution: We are given that
2 2
æ z1 ö æ zö
z1 - z2
=1 çè z2 ÷ø - 1 = çè z2 ÷ø + 1
z1 + z2
www.jeeneetbooks.in
3.3 Geometric Interpretation 115

By properties 9 and 10 of Quick Look 4, we have This implies

z1
2
æz ö z
2
æz ö z1
+ 1 - 2 Re ç 1 ÷ = 1 + 1 + 2 Re ç 1 ÷ = ia
z2 z z z2
è 2ø 2 èz ø 2
where a is a real number and
Therefore
z1
i = -a
æz ö z1 z2
Re ç 1 ÷ = 0 or
è z2 ø z2
which is a real number.
is purely imaginary

The complex numbers z having the same modulus | z | = r evidently correspond to the points of the complex plane
located on the circle of radius r with center at the origin. If r > 0, then there are infinitely many complex numbers with
the given modulus r. If r = 0, then there is only one complex number, namely z = 0, whose modulus is 0.

z = a+ib
b

q
x
O a r

FIGURE 3.3 Geometrical determination of z using the angle q and the modulus a2 + b2 .

From the geometrical point of view, it is evident that the complex number z ¹ 0 is not completely determined
by its modulus and depends on the direction also; for example, in Figure 3.3, z is determined by the angle q and the
modulus a2 + b2 . Next, we introduce another important concept which, together with the modulus, completely
determines a complex number.

Argument of z

DEF IN IT ION 3 . 10 Let z ¹ 0 be a complex number and OM be the vector in the plane representing z. Then the
argument of z is defined to be
the magnitude of the angle between the positive direction of
the real axis and the vector OM, measured in counterclockwise sense. The angle will be con-
sidered positive if we measure counterclockwise and negative if we measure clockwise.

Note: For the complex number z = 0 the argument is not defined, and in this and only this case the number is specified
exclusively by its modulus. Specification of the modulus and argument results in a unique representation of any
non-zero complex number.
Unlike the modulus, the argument of a non-zero complex number is not defined uniquely. For example, the
arguments of the complex number z = a + ib shown in Figure 3.4 are the angles q1, q2 and q3. Note that
q2 = 2p + q1 and q3 = q1 - 2p
In general, q is an argument of z if and only if q = q1 + 2np for some integer n, where q1 is also an argument of z;
that is, any two arguments of a complex number differ by a number which is a multiple of 2p. The set of all arguments
of z will be denoted by arg z or arg(a + ib). That is, if q is an argument of z, then
arg z = {q + 2 np | n is an integer}
www.jeeneetbooks.in
116 Chapter 3 Complex Numbers

y y y

M M M
b
(z =a +ib) (z =a +ib) (z = a + ib)

q1 q2
x x x
O a q3

FIGURE 3.4 Different arguments of the complex number z = a + ib.

However, there is a unique q such that -p < q £ p and arg z = {q + 2np | n is an integer}. This q is called the principal
argument of z and is denoted by Arg z (note that A here is uppercase). Note that
-p < Arg z £ p
Also arg z and Arg z are related to each other by the relation
arg z = {Arg z + 2 np | n is an integer}
Frequently, we denote arg z by Arg z + 2np, where Arg z is the principal argument of z.

Example 3.9

Find the arguments of the complex numbers z1 = -i, Therefore


z2 = 1 and z3 = -1 + i.
-p -p
Arg(- i) = and arg (- i) = + 2 np
Solution: From Figure 3.5, we have 2 2

-p 3p Arg (1) = 0 and arg (1) = 2np


q1 = , q2 = 0 and q 3 =
2 4 3p 3p
Arg (- 1 + i) = and arg (- 1 + i) = + 2 np
4 4

y y y
M3 (-1+i )
1

q3
q1
M2
x x x
1 -1

M1 -i

FIGURE 3.5 Example 3.9.

The real and imaginary parts of the complex number z = a + ib can be expressed in terms of the modulus | z | = r
and argument q as follows:
a = r cos q and b = r sin q
(see Figure 3.6) and hence
z = r(cosq + i sinq)
www.jeeneetbooks.in
3.3 Geometric Interpretation 117

z =a +ib
b

q
r x
O a

FIGURE 3.6 Geometrical interpretation of z in polar form.

Therefore, the arguments q of a complex number a + ib can be easily found from the following system of equations:
a b
cos q = and sin q = (3.1)
a +b
2 2
a + b2
2

Example 3.10

Find the arguments of the complex number z = - 1 - i 3 . Solving these we find that
-2p
Solution: In this case, we have a = -1 and b = - 3. Arg z =
Equation (3.1) takes the form 3

-1 - 3 and hence
cos q = and sin q =
2 2 - 2p
arg z = + 2 np, n Î 
3

The arguments of a complex number can be found by another method. It can be seen from formula (3.1) that each
of the arguments satisfies the equation
b
tanq =
a
This equation is not equivalent to the system of equations (3.1). It has more solutions, but the selection of the required solu-
tions (the arguments of the complex number) does not present any difficulties since it is always clear from the algebraic
notation of the complex number in what quadrant of the complex plane it is located. This is elaborated in the following.

Key Points
Let z = a + ib and q = Arg z, the principal argument of z. Note that z is necessarily non-zero for the arg z to be
defined.
1. If a = 0 and b > 0, then
p p
Arg z = and arg z = + 2 np , n Î 
2 2
If a = 0 and b < 0, then
-p -p
Arg z = and arg z = + 2 np , n Î 
2 2
www.jeeneetbooks.in
118 Chapter 3 Complex Numbers

If b = 0 then z = a lies on the x-axis and hence


Arg z = 0 or p and arg z = 2 np or (2 n + 1)p , n Î 

y y

-p
q=
2
x x
-p
=q
2

2. Let (a, b) belong to the first quadrant of the complex plane, that is, a > 0 and b > 0. Then the principal argument of
z is given by

æ bö
Arg z = q = tan-1 ç ÷
è aø
where tan q = b/a. This is an acute angle 0 < q < p/2 and positive. Therefore,

æ bö
arg z = 2 np + tan-1 ç ÷ , n Î 
è aø
y

b z = a + ib

q
x
a a

3. Let (a, b) belong to the second quadrant of the complex plane, that is, a < 0 and b < 0. Then the principal argument
of z is given by
æ bö
Arg z = q = p - tan-1 ç ÷
è | a |ø

This is an obtuse angle and is positive. Therefore,


æ -bö
arg z = (2 n + 1)p - tan-1 ç ÷ , n Î 
è a ø
y
z = a +ib
M b

b q

x
a |a |
www.jeeneetbooks.in
3.3 Geometric Interpretation 119

4. Let (a, b) lie in the third quadrant of the complex plane, that is, a < 0 and b < 0. Then the principal argument of z is
given by

æ bö
Arg z = q = -p + tan-1 ç ÷
è aø
This is an obtuse angle and negative. Therefore

æ bö
arg z = (2 n - 1)p + tan-1 ç ÷ , n Î 
è aø

2p -q
a
x

|b| q

|a|
M b
z =a +ib

5. Let (a, b) lie in the fourth quadrant of the complex plane, that is, a > 0 and b < 0. Then the principal argument of z
is given by

æ | b|ö
Arg z = q = - tan-1 ç ÷
è aø

This is an acute angle and negative. Therefore


æ | b|ö æ -bö
arg z = 2 np - tan-1 ç ÷ = 2 np - tan-1 ç ÷ , n Î 
è aø è a ø

a
x
q
|b|

b M
a z =a +ib

 
Note: Arg z is the smallest angle of rotation
 of OX (positive x-axis) to fall on the vector OM [M = (a, b)]. Arg z >
<0
according to whether the rotation of OX is anticlockwise or clockwise, respectively.

Example 3.11

Find the arguments of the complex number z = - 3 + i. æ bö æ 1 ö p 5p


Arg z = p - tan-1 ç ÷ = p - tan-1 ç ÷ =p - =
è | a |ø è 3ø 6 6
Solution: In this case z = a + ib, where a = - 3 and b = 1.
Therefore z lies in the second quadrant of the complex Therefore
plane and hence the principal argument is
5p
arg (- 3 + i) = + 2 np, n Î 
6
www.jeeneetbooks.in
120 Chapter 3 Complex Numbers

Next we will discuss the geometrical constructions of difference, product and quotient of two complex numbers
z1 and z2.

Construction of z2 − z1
Let us construct the vector z2 - z1 as the sum of the vectors z2 and −z1 (Figure 3.7). By thedefinition
 of the modulus, the
real number | z 2 - z1 | is the length of the vector z2 - z1; that is, the length of the vector OM , where M , M 1 , M 2 and N1
represent the points in the complex plane corresponding to the complex numbers z2 - z1, z1, z 2 and -z1, respectively.
 
The congruence of the triangles OMN1 and M1 M2O yields |OM | = | M1 M2 |. Therefore the length of the vector z2 - z1 is
equal to the distance between the points z1 and z2. Thus we can say that the modulus of the difference of two complex
numbers is the distance between the points of the complex plane corresponding to those complex numbers. This important
geometrical interpretation of the modulus of the difference between two complex numbers makes it possible to use
simple geometrical facts in solving certain problems. See examples given in Section 3.3.
y
M1

z1 M2
z2

O
x
-z1
z2 -z1
M

N1

FIGURE 3.7 Construction the vector z 2 - z1 as the sum of the vectors z2 and −z1.

Before going to illustrate the construction of the product and quotient of complex numbers, we present the following
definition:

DEFIN IT ION 3 . 11 Two triangles ABC and A¢B¢C¢ are said to be directly similar if ÐA = ÐA¢, ÐB = ÐB¢ and
ÐC = ÐC¢ and the ratios of the sides opposite to equal angles are equal.

Note that directly similar triangles are similar and not vice-versa. For example, if D ABC and D A¢ B ¢C ¢ are directly
similar, then D ABC and D B ¢A¢C ¢ are not directly similar (unless they are equilateral triangle).

Construction of z1 z2 and z1/z2 (z2 π 0)


Step 1: Let z1 and z2 be complex  and P and Q the points representing them, respectively. Let O be the origin
 numbers
so that the vectors OP and OQ represent z1 and z2, respectively. Let A be the point (1, 0). Join A and P, and
on the base OQ, construct the triangle OQR directly similar to the triangle OAP (Figure 3.8). Then
ÐQOR = ÐAOP, ÐOQR = ÐOAP, ÐQRO = ÐAPO
and further,
OR OQ QR
= =
OP OA AP
Therefore
OR = OP × OQ (∵ OA = 1) (3.2)
Let ÐXOP = q1 and ÐXOQ = q2 . Then
ÐXOR = ÐXOQ + ÐQOR
= ÐXOQ + ÐAOP (3.3)
= q2 + q1 = q1 + q2
www.jeeneetbooks.in
3.3 Geometric Interpretation 121

From Eqs. (3.2) and (3.3),

z1z2 = r1 r2 [cos(q 1 + q 2 ) + i sin(q 1 + q 2 )]

where r1 = OP and r2 = OQ. Therefore R represents z1 × z2.

y
R(z1z2)

Q(z2)

P(z1)
x
O A(1, 0)

FIGURE 3.8 Step 1.

Step 2: Draw a triangle OPR directly similar to the triangle OQA. By the above construction, if R is represented by z,

then z × z2 = z1 (Figure 3.9). Notice
 that Ð QOP = arg( z1 / z2 ) is the angle through which OQ must be rotated

in order that it may lie along OP and arg(z1 /z2 ) is positive or negative according as the rotation of OQ is
anticlockwise or clockwise.

y
P(z1)

z1
r1 R z
2

r2 Q(z2)
x
O A(1, 0)

FIGURE 3.9 Step 2.

In the following theorem, an important consequence of arg(z1/z2) is derived. This can help the reader in solving
many problems in the geometry of complex numbers.

T H E O R E M 3 .5 Let z1, z2 and z3 be three complex numbers represented by P, Q and R, respectively. If a is the angle
ÐPRQ , then
z2 - z3 RQ
= (cos a + i sin a )
z1 - z3 RP

PROOF Let the points A and B represent z1 - z3 and z2 - z3, respectively, so that RP = OA, RQ = OB
and PQ = AB (Figure 3.10). Therefore DPQR and DABO are congruent and hence ÐAOB = a.
By Step 2 above,
æ z2 - z3 ö
a = arg ç ÷
è z1 - z3 ø
Therefore
z2 - z3 | z2 - z3 |
= (cos a + i sin a )
z1 - z3 | z1 - z3 |
RQ
= (cos a + i sin a )
RP
www.jeeneetbooks.in
122 Chapter 3 Complex Numbers

y
Q(z2)

a P(z1)
R(z3)
B(z2 -z3)

a A(z1 -z3)
x
O

FIGURE 3.10 Theorem 3.5. ■

QUICK LOOK 5

æz -z ö  2. For any four points z1, z2, z3 and z4, the angle of
1. arg ç 2 3 ÷ is the angle of rotation of the vector RP inclination of the line joining z1 to z2 with the line
è z1 - z3 ø
 joining z3 to z4 is
to fall along RQ.
æz -z ö
z3 z1 arg ç 3 4 ÷
è z1 - z2 ø

3. The lines joining z1 to z2 and z3 to z4 are at right angles


if and only if
æz -z ö p
arg ç 3 4 ÷ = ±
è z1 - z2 ø 2
and hence
z2 z4
z3 - z4
= ±li
z1 - z2

where l > 0.

Example 3.12

Determine the sets of complex numbers defined by each (2) We can give a different formulation of the problem,
of the following conditions. using the geometrical interpretation of the modulus of
the difference between two complex numbers. We are
(1) | z - i | = 1
asked to determine the set of points in the complex
(2) | 2 + z | < | 2 - z| plane that are located closer to the point z = -2 than to
(3) 2 £ | z - 1 + 2 i | < 3 the point z = 2. It is clear that this property is possessed
by all the points of the plane that lie to the left of the
Solution: imaginary axis and only by those points. In the figure
given below, the shaded portion of the complex plane
(1) | z - i | = 1 is satisfied by those and only those points of the
represents the set of points satisfying | 2 + z | < | 2 - z |.
complex plane which are at a distance equal to 1 from
the point i. Therefore, the set of complex numbers z satis- y
fying the condition | z - i | = 1 is precisely the circle of unit
radius with center at the point i (see the figure below).

x
–2 O

x
O
www.jeeneetbooks.in
3.3 Geometric Interpretation 123

(3) The given condition is y

2 £ | z - (1 - 2i)| < 3

A complex number z satisfies the given condition if x


and only if its distance from the point 1 - 2i is greater
than or equal to 2 but less than 3. Such points lie in
r =2
the interior and on the inner boundary of the ring
formed by two concentric circles with centers at the l =1-2i
point 1 - 2i and the radii r = 2 and R = 3. The required
R =3
set is indicated by the shaded portion of the figure at
the right side.

Next, we will turn our attention to general equations of certain geometrical figures in the complex plane, in terms
of a complex variable.

T H E O R E M 3 .6 The general equation of a straight line in the complex plane is


lz + lz + m = 0
where l is a non-zero complex number and m is a real number.
PROOF Let l = a + ib be a non-zero complex number and m a real number. Consider the equation
lz + lz + m = 0
Let z = x + iy be an arbitrary point on this curve. Then
(a + ib)( x + iy) + (a + ib)( x + iy) + m = 0
Therefore
(a - ib)( x + iy) + (a + ib)( x - iy) + m = 0
Solving we get
2 ax + 2 by + m = 0, a ¹ 0 or b ¹ 0 (since l ¹ 0)

This represents a straight line in the plane. Conversely, if px + qy + r = 0 is a straight line, where
p, q, r are reals and p ¹ 0 or q ¹ 0 and if z = x + iy is a point on this line, then
æz+zö æz-zö
pç ÷ + qç ÷+r=0
è 2 ø è 2i ø
Therefore
pz + pz - qiz + qi z + 2r = 0
( p - qi)z + ( p + qi) z + 2r = 0
By taking l = p + qi and m = 2r, the above equation takes the form

lz + lz + m = 0
Note that l ¹ 0, since p ¹ 0 or q ¹ 0. ■

T H E O R E M 3 .7 In the complex plane the equation of the line joining the points z1 and z2 is

z z 1
z1 z1 1 =0
z2 z2 1
www.jeeneetbooks.in
124 Chapter 3 Complex Numbers

PROOF Let the points z1 and z2 be A and B, respectively. Then P(z) is a point on the line AB if and only if
A, P and B are collinear which implies
æ z - zö
arg ç 1 = 0 or p
è z2 - z ÷ø
z1 - z
Û is pure real
z2 - z

z1 - z z1 - z
Û =
z2 - z z2 - z
Û (z1 - z)(z2 - z ) = (z2 - z)(z1 - z )
z z 1
Û z1 z1 1 =0
z2 z2 1 ■

QUICK LOOK 6

1. The complex number (z1 - z2 )/(z1 - z2 ) is called the and m is a real number), the complex number (z1 -
complex slope of the line joining z1 and z2. z2)/(z1 - z2 ) is equal to - l / l and hence - l / l is the com-
2. For any two points z1 and z2 on the straight line lz + plex slope of the line lz + lz + m = 0.
lz + m = 0 (where l is a non-zero complex number

T H E O R E M 3 .8 The equation of the perpendicular bisector of the line segment joining the points z1 and z2 is

(z1 - z2 )z + (z1 - z2 )z + z2 z2 - z1z1 = 0


PROOF Let A(z1) and B(z2) be the given points and L be the perpendicular bisector of the line segment
AB. Then P(z) is point on L. This implies
PA = PB
Û | z - z1 | = | z - z2 |
Û (z - z1 )(z - z1 ) = (z - z2 )(z - z2 )
Û (z1 - z2 ) z + (z1 - z2 ) z + z2 z2 - z1z1 = 0 ■

In the following theorem we obtain a necessary and sufficient condition for two points in the complex plane to be
images of each other in a given straight line in the same plane.

T H E O R E M 3 .9 Two points z1 and z2 are images of each other in the line lz + lz + m = 0 (0 ¹ l Î  and m Î ) if
and only if lz1 + lz2 + m = 0.

PROOF Suppose that z1 and z2 are images of each other in the line lz + lz + m = 0. Then this line is the
perpendicular bisector of the line segment joining z1 and z2. By Theorem 3.7, the equation of the
perpendicular bisector is
(z1 - z2 )z + (z1 - z2 )z + z2 z2 - z1 z1 = 0

Therefore
l l m
= = = k (say)
z1 - z2 z1 - z2 z2 z2 - z1 z1
www.jeeneetbooks.in
3.3 Geometric Interpretation 125

Now,
lz1 + lz2 + m = k(z1 - z2 )z1 + k(z1 - z2 )z2 + (z2 z2 - z1z1 )k
= k [z1z1 - z2 z1 + z1z2 - z2 z2 + z2 z2 - z1z1 ]
= k(0) = 0
Conversely, suppose that lz1 + lz2 + m = 0 . Let z be any point on the given line. Then
lz + lz + m = 0
and therefore
l (z - z1 ) + l(z - z2 ) = 0

which implies that


| l (z - z1 )| = | - l(z - z2 )|
and hence
| z - z1 | = | z - z2 | = | z - z2 |

That is, z is equidistant from both the points z1 and z2. Therefore the line lz + lz + m = 0 is the
perpendicular bisector of the line segment joining z1 and z2. ■

T H E O R E M 3 .10 The perpendicular distance of the straight line lz + lz + m = 0 (0 ¹ l Î  and m Î ) from a given
point z0 is
lz0 + lz0 + m
2l

PROOF Let z = x + iy, so that the equation of the given line is


( l + l ) x + i( l - l ) y + m = 0
which is a first degree equation in x and y with real coefficients. Therefore, the distance of the line
from the point z0 = a + ib is

( l + l )a + i( l - l ) b + m l (a + ib) + l(a - ib) + m


=
( l + l ) 2 - ( l - l )2 4l l
lz0 + z0 + m
=
2l ■

T H E O R E M 3 .11 The general equation of a circle in the complex plane is


zz + bz + bz + c = 0

where b is a complex number and c is a real number.

PROOF Let z0 be a fixed point in the complex plane and r a non-negative real number. Then the equation

| z - z0 | = r
represents the locus of the point z whose distance from the point z0 is the constant r. We know that
this locus is a circle with centre at z0 and radius r. This equation is equivalent to
(z - z0 )(z - z0 ) = r2
www.jeeneetbooks.in
126 Chapter 3 Complex Numbers

That is, zz + (- z0 )z + (- z0 )z + (z0 z0 - r2 ) = 0 which is of the form zz + bz + bz + c = 0, where


b = - z0 and c = zz0 - r2. On the other hand, any equation zz + b z + bz + c = 0 can be written as
(z + b)(z + b ) = bb - c
That is,
| z + b | = bb - c

which represents a circle with center at -b and radius bb - c . Note that bb and c are real
numbers and b b - c > 0 or = 0 or < 0. ■

QUICK LOOK 7

1. Note that the circle zz + bz + bz + c = 0 is real or mz2 + nz1


z=
point circle or imaginary circle according as bb - c m+n
is a positive real number or bb = c or negative real
number, respectively. 3. If A(z1), B(z2) and C(z3) are the vertices of a triangle,
then the complex number (z1 + z2 + z3) / 3 represents
2. If A(z1) and B(z2) are points in the complex plane
the centroid of the triangle ABC.
and P(z) is a point on the line joining A(z1) and
B(z2) dividing the line segment AB in the ratio
m : n (m + n ¹ 0), then

Example 3.13

Find the center and radius of the circle where b = -(2 + 3i) and c = -3. Therefore -b(= 2 + 3i) is
zz - (2 + 3i)z - (2 - 3i)z - 3 = 0 the center of the circle and bb - c [= (2 + 3i)(2 - 3i) + 3
= 16 = 4] is the radius.
Solution: This equation is of the form
zz + bz + bz + c = 0

Example 3.14

If 2 + i and 4 + 3i represent the extremities A and C, In Figure 3.11 DEAB is right angled at E. If z represents B,
respectively, of a diagonal of a square ABCD, described then
in counterclock sense, then find the other two vertices
z - (3 + 2i) p p
B and D. = cos + i sin = i
(2 + i) - (3 + 2i) 2 2
Solution: Let E be the intersection of the diagonals.
and therefore, z = i(-1 - i) + 3 + 2i = 4 + i. Similarly, from
Then E is represented by
DECD, if z¢ represents D, then
(2 + i) + (4 + 3i)
= 3 + 2i z¢ - (3 + 2i)
1+ 1 =i
(4 + 3i) - (3 + 2i)
D C
and hence z¢ = 2 + 3i .

A B

FIGURE 3.11 Example 3.14.


www.jeeneetbooks.in
3.3 Geometric Interpretation 127

Example 3.15

If z1, z2, z3 and z4 are the vertices of a square described in z1 - z2 p p


the counterclock sense, then express z2 and z4 in terms of = cos + i sin = i
z3 - z2 2 2
z1 and z3, and z1 and z3 in terms of z2 and z4 (Figure 3.12).
z1 - z2 = i(z3 - z2 )
D(z4) C(z3)
z1 - iz3 = (1 - i)z2
90° 90° 1
z2 = [(1 + i)z1 + (1 - i)z3 ]
2
Similarly
90° 90°
1
A(z1) B(z2) z4 = [(1 - i)z1 + (1 + i)z3 ]
2
FIGURE 3.12 Example 3.15. 1
z3 = [(1 + i)z2 + (1 - i)z4 ]
2
Solution: Rotate BC about B through 90° in anticlock-
1
wise sense. Then z1 = [(1 - i)z2 + (1 + i)z4 ]
2

Example 3.16

Let l1 z + l 1 z + m1 = 0 and l2 z + l 2 z + m2 = 0 be two strai- and ( l2 + l 2 ) x + i( l2 - l 2 ) y + m2 = 0


ght lines in the complex plane. Then prove that
which are first degree equations with real coefficients
(1) the lines are parallel if and only if l1 l2 = l 2 l1 . [recall that z + z and i(z - z) are always real numbers
(2) the lines are perpendicular if and only if l1 l2 + l 2 l1 = 0. for all complex numbers z]. Therefore, we can use the
conditions for parallelness and perpendicularity as in
Solution: Writing z = x + iy ( x and y real) , the equa- two-dimentional geometry.
tions of the given straight lines are transformed into Calculations are left for students as an exercise.
( l1 + l 1 ) x + i( l1 - l 1 ) y + m1 = 0

Example 3.17

Let lz + lz + m = 0 be a straight line in the complex plane Solution: Let Q(z) be any point on the given line.
and P(z0) be a point in the plane. Then prove that
(1) We have
(1) the equation of the line passing through P(z0) and -l z - z0
parallel to the given line is = slope of the line =
l z - z0
l ( z - z0 ) + l ( z - z 0 ) = 0 which gives the required equation.
(2) the equation of the line passing through P(z0) and (2) If Q(z) is any line passing through P(z0) and is
perpendicular to the given line is perpendicular to the given line, then

l (z - z0 ) - l(z - z0 ) = 0 z - z0 l
= (see Example 3.16)
z - z0 l
which gives the required equation.

Example 3.18

Find the foot of the perpendicular drawn from a point Solution: The given line is
P(z0) onto to a line lz + lz + m = 0. lz + lz + m = 0 (3.4)
www.jeeneetbooks.in
128 Chapter 3 Complex Numbers

The line passing through P(z0) and perpendicular to the Eqs. (3.4) and (3.5), we have
given line is
lz0 - lz0 - m
l (z - z0 ) - l(z - z0 ) = 0 (3.5) z=
2l
The foot of the perpendicular from P(z0) satisfies which is the foot of the perpendicular.
both Eqs. (3.4) and (3.5). Therefore, eliminating z from

Example 3.19

Find the radius and the center of the circle This equation represents a circle with center at
zz + (2 - 3i) z + (2 + 3i) z + 4 = 0 -b (= -2 -3i) and radius bb - 4 (= 4 + 9 - 4 = 3).

Solution: If b = 2 + 3i, then the given equation is


zz + bz + bz + 4 = 0

Example 3.20

Prove that the equation | z + 1| = 2 | z + 1| represents a That is,


circle and find its center and radius.
zz + (-3)z + (-3)z + 1 = 0
Solution: The given equation is equivalent to which represents a circle with centre at 3 [= (3, 0)] and
(z + 1)(z + 1) = 2(z - 1)(z - 1) radius 32 - 1 = 2 2 .

3.4 | The Trigonometric Form


In the previous section, we have noted that the real and imaginary parts of a complex number z = a + ib can be
expressed in terms of the modulus | z | = r and argument q as
a = r cos q and b = r sin q
Therefore, any non-zero complex number z can be expressed as
z = r (cos q + i sin q )
where r is the modulus of z and q is an argument of z. This expression of a complex number is called the trigonometric
notation or trigonometric form or polar form of z.
Let us recall that the expression z = a + ib, where a and b are real numbers and i2 = -1, is called the algebraic form
of z. To pass from algebraic form to trigonometric form, it is sufficient to find the modulus of a complex number and
one of its arguments. Let us consider certain examples.

Example 3.21

Express the following complex numbers in trigonometric é æ -3p ö æ -3p ö ù


z1 = 2 êcos ç ÷ + i sin ç
form:
ë è 4 ø è 4 ÷ø úû
(1) z1 = -1 - i
(2) | z2 | = 2 and Arg z2 = p and hence
(2) z2 = -2
z2 = 2(cos p + i sin p )
(3) z3 = i
(3) | z3 | = 1 and Arg z3 = p / 2 and hence
Solution:
æpö æpö
z3 = cos ç ÷ + i sin ç ÷
(1) | z1 | = 2 and Arg z1 = -3p / 4 and hence è 2ø è 2ø
www.jeeneetbooks.in
3.4 The Trigonometric Form 129

Example 3.22

Express the following complex numbers in trigonometric Now, we have


form: é æ -p ö æ -p ö ù
z1 = 2 êcos ç ÷ + i sin ç
æ 7p ö æpö ë è 4 ø è 4 ÷ø úû
(1) z1 = 2 cos ç ÷ - 2 i sin ç ÷
è 4 ø è 4ø
æpö æpö
z2 = - cos ç ÷ + i sin ç ÷
æpö æpö è 17 ø è 17 ø
(2) z2 = - cos ç ÷ + i sin ç ÷
è 17 ø è 17 ø æ pö æ pö
= cos ç p - ÷ + i sin ç p - ÷
Solution: Note that in these cases, we need not find the è 17 ø è 17 ø
modulus and arguments, although it is easy to find these. æ 16p ö æ 16p ö
= cos ç + i s in ç
Instead, we will make use of the facts that è 17 ÷ø è 17 ÷ø
æ 7p ö æ pö æ -p ö
cos ç ÷ = cos ç 2p - ÷ = cos ç
è 4 ø è 4ø è 4 ÷ø

æpö æ -p ö
and - sin ç ÷ = sin ç
è 4ø è 4 ÷ø

The operations of multiplication and division of complex numbers can be easily performed by transforming the given
complex numbers into trigonometric form. We have already noted that the modulus of the product (quotient) of any
two complex numbers is the product (quotient) of their moduli.
Now, let us turn our attention to the arguments of products and quotients.

T H E O R E M 3.1 2 The following hold for any two non-zero complex numbers z1 and z2.
1. z1 = z2 Û | z1 | = | z2 | and arg z1 = arg z2

2. arg(z1 z2 ) = Arg z1 + Arg z2 + 2 np , n Î 

æ 1ö
3. arg ç ÷ = - Arg z2 + 2np , n Î 
è z2 ø
æ z1 ö
4. arg ç ÷ = Arg z1 - Arg z2 + 2 np , n Î 
è z2 ø

PROOF First let us express the given non-zero complex numbers z1 and z2 in trigonometric form. Let
z1 = r1 (cos q 1 + i sin q 1 ), r1 > 0, -p < q 1 £ p and z2 = r2 (cos q 2 + i sin q 2 ), r2 > 0, -p < q 2 £ p . That is, |z1| =
r1, | z2 | = r2 , Arg z1 = q 1 and Arg z2 = q 2 .

1. This part is clear since arg z1 = Arg z1 + 2 np , n Î  .


2. Consider the product,

z1 z2 = r1 (cos q 1 + i sin q 1 )r2 (cos q 2 + i sin q 2 )


= r1 r2 [(cos q 1 cos q 2 - sin q 1 sin q 2 ) + i(cos q 1 sin q 2 + sin q 1 cos q 2 )]
= r1 r2 [cos(q 1 + q 2 ) + i sin(q 1 + q 2 )]

and therefore
| z1 z2 | = r1 r2 and arg(z1 z2 ) = q 1 + q 2 + 2 np = Arg z1 + Arg z2 + 2 np , n Î 
www.jeeneetbooks.in
130 Chapter 3 Complex Numbers

3. This follows from the fact that


1 cos q - i sin q
=
cos q + i sin q (cos q + i sin q )(cos q - i sin q )
cos q - i sin q
=
cos 2 q + sin 2 q
= cos(-q ) + i sin(-q )

Therefore,
æ 1ö
arg ç ÷ = -Arg z2 + 2 np , n Î 
è z2 ø

4. It follows from (2) and (3). ■

Example 3.23

Let Therefore
é æ 11p ö æ 11p ö ù é æ - 7p ö æ - 7p ö ù
z1 = 2 êcos ç ÷ø + i sin çè ÷ z1z2 = 4 êcos ç ÷ø + i sin çè
ë è 4 4 ø úû ë è 8
÷
8 ø úû
é æ 3p ö æ 3p ö ù é æ 7p ö æ 7p ö ù
and z2 = 8 êcos ç ÷ + i sin ç ÷ ú = 4 êcos ç ÷ - i sin ç ÷ ú
ë è 8ø è 8 øû è ø è 8 øû
ë 8
Find z1z2 and z1/z2.
Also,
Solution: First note that
11p 3p z1 z1 2 1
= 2p + = = =
4 4 z2 z2 8 2

Now, | z1 | = 2 and | z2 | = 8 , therefore æ z1 ö


arg ç ÷ = Arg z1 - Arg z2 + 2 np , n Î 
3p 3p è z2 ø
Arg z1 = and Arg z2 =
4 8 3p 3p
= - + 2 np , n Î 
4 8
Therefore, | z1z2 | = | z1 || z2 | = 2 8 = 4. Now
3p
= + 2 np , n Î 
arg (z1 z2 ) = Arg z1 + Arg z2 + 2 np , n Î  8
3p 3p Therefore,
= + + 2 np , n Î 
4 8 æ z1 ö 3p
9p Arg ç ÷ =
= + 2 np , n Î  è z2 ø 8
8
and hence
-7p
= + 2(n + 1)p , n Î 
8 z1 1 é æ 3p ö æ 3p ö ù
= cos ç ÷ + i sin ç ÷ ú
-7p z2 2 êë è 8ø è 8 øû
= + 2 mp , m Î 
8
and hence
-7p
Arg(z1 z2 ) =
8
www.jeeneetbooks.in
3.5 De Moivre’s Theorem 131

3.5 | De Moivre’s Theorem


In the previous section, we have derived formulas for the product and quotient of two complex numbers in
trigonometric form. The formula for the product of two complex numbers can be extended to the case of n factors by
mathematical induction. As a special case, we have the following.

T H E O R E M 3.13 For any real number q and any positive integer n,


(D E M O I V R E ’ S
THEOREM) (cos q + i sin q ) n = cos(n q ) + i sin (n q )

PROOF We prove this by induction on n. If n = 1, this is trivial. Now, let n > 1 and assume that

(cos q + i sin q ) n - 1 = cos[(n - 1) q ] + i sin[(n - 1) q ]

Now, consider
(cos q + i sin q ) n = (cos q + i sin q ) n - 1 (cos q + i sin q )
= [cos{(n - 1)q } + i sin {(n - 1)q }](cos q + i sin q )
= [cos{(n - 1) q }cos q - sin {(n - 1) q }sin q ]
+ i[cos{(n - 1) q }sin q + cos q sin{(n - 1) q }]
= cos[(n - 1) q + q ] + i sin[(n - 1) q + q ]
= cos(n q ) + i sin(n q ) ■

C O R O L L A R Y 3 .2 For all real numbers q and for all integers n,

(cos q + i sin q ) n = cos(n q ) + i sin(n q )

PROOF For n = 0, this is obvious. Let n < 0. First observe that


1 cos q - i sin q
=
cos q + i sin q (cos q + i sin q )(cos q - i sin q )
cos(- q ) + i sin(- q )
=
cos 2 q + sin 2 q
= cos(- q ) + i sin(- q )
Now,

(cos q + i sin q ) n = [(cos q + i sin q ) - n ]- 1


1
=
(cos q + i sin q ) - n
1
=
cos(- n q ) + i sin(- n q )
= cos(n q ) + i sin(n q )

since -n > 0 and by Theorem 3.13. ■

In the following we demonstrate the use of De Moivre’s Theorem in expressing certain powers of complex numbers
with natural exponents in algebraic form.
www.jeeneetbooks.in
132 Chapter 3 Complex Numbers

Example 3.24

Express the number z = (i - 3 )13 in algebraic form. é æ 65p ö æ 65p ö ù


= 213 ê cos ç ÷ + i sin ç
ë è 6 ø è 6 ÷ø úû
Solution: First we write the given number in trigonomet-
ric form and then pass to the algebraic form. Let w = i - 3. é æ 5p ö æ 5p ö ù
= 213 êcos ç ÷ + i sin ç ÷ ú
Then | w | = 1 + 3 = 2 and Arg w = 5p / 6. Therefore ë è 6 ø è 6 øû

é æ 5p ö æ 5p ö ù Thus
w = 2 êcos ç ÷ + i sin ç ÷ ú
ë è 6 ø è 6 øû æ 3 1 ö
(i - 3 )13 = 2 13 ç - + i÷ = - 2 12 3 + 2 12 i
and hence è 2 2 ø

é æ 5p ö æ 5p ö ù
z = w 13 = 2 13 êcos ç 13 × ÷ + i sin ç 13 × ÷ ú
ë è 6 ø è 6 øû

Next, let us find the root of a given degree of a complex number.

Roots of Degree n

DEF IN IT ION 3 . 12 If z and w are complex numbers and n a positive integer such that zn = w, then z is called a
root of degree n or nth root of the number w and is denoted by n w . Roots of degree 2 or 3
are called square roots or cube roots, respectively.

For example, i and -i are both square roots of -1. In general, to extract a root of degree n of a complex number w, it
is sufficient to solve the equation zn = w. If w = 0, then the equation zn = w has exactly one solution, namely z = 0. The
case w ¹ 0 is dealt with in the following.

T H E O R E M 3 .14 Let w be a non-zero complex number and n a positive integer. Then the equation zn = w has n
solutions.

PROOF First we represent z and w in the trigonometric form. Let


z = r (cos q + i sin q ) and w = s (cos a + i sin a )
The equation z = w takes the form
n

rn (cos(n q ) + i sin (n q )) = s (cos a + i sin a )


Two complex numbers are equal if and only if their moduli are equal and argument differ by an
integral multiple of 2p. Therefore,

rn = s and n q = a + 2 mp , m Î 
a 2p
or r=ns and q = + n m, m Î 
n
Thus, all the solutions of the equation zn = w can be written as follows:

é æ a 2p ö æ a 2p ö ù
zm = n s êcos ç n + m÷ + i sin ç n + n m÷ ú , m Î 
ë è n ø è øû

It can be easily seen that zm for m = 0, 1, … , n - 1 are different. For m ³ n, we cannot obtain any
other complex numbers different from z0 , z1 , … , zn - 1. For example, for m = n, we get
www.jeeneetbooks.in
3.5 De Moivre’s Theorem 133

é æa ö æa öù
zn = n s êcos ç + 2p ÷ + i sin ç n + 2p ÷ ú
ë è n ø è øû

é æaö æaöù
= n s êcos ç ÷ + i sin ç n ÷ ú = z0
ë è ø
n è øû
It can be seen that zn + k = zk for all k ³ 0. Thus, these are exactly n roots of degree n of the number
w and they are all obtained from the formula
é æ a 2p ö æ a 2p ö ù
zm = n s êcos ç + m÷ + i sin ç + m÷ , for m = 0, 1, 2, …,, n - 1
ë è n n ø èn n ø úû ■

It can be seen from the above formula that all the roots of degree n of the number w have one and the same
moduli but distinct arguments differing from each other by (2p / n)m, where m is some integer.

QUICK LOOK 8

1. All the roots of degree n of the complex number w - 1 is understood to be the set consisting of
correspond to the points of the complex plane lying
two complex numbers i and -i. Sometimes, n w is
at the vertices of a regular n-gon inscribed in a circle
understood as a root of degree n of w. In such
of radius n | w | with centre at the point z = 0.
cases, it must be indicated what value of the root
2. Usually the expression n w is to be understood as is meant.
the set of all roots of degree n of w. For example,

Theorem 3.13 paves a way to formulate and prove the most general version of the De Moivre’s Theorem in the
following. If z0 is a solution of the equation zn = w, then let us agree to write z0 as w1/n. Therefore w1/n has n values. In
particular, if w is any complex number and r = m/n, where m and n are integers and n > 0, then w1/n has n values.

T H E O R E M 3 .15 For any real number q and any rational number r,


(DE MOIVRE’S
THEOREM FOR (cos q + i sin q ) r = cos(r q ) + i sin (r q )
R AT I O N A L
INDEX)
PROOF Let q be a real number and r = n/m, where n and m are integers and m > 0. Then
m
é æn ö æ n öù
[cos(r q ) + i sin(r q )]m = êcos ç q ÷ + i sin ç m q ÷ ú
ë è m ø è øû
= cos(n q ) + i sin (n q ) (by Theorem 3.13)
= (cos q + i sin q ) n (by Corollary 3.2)

Therefore cos(r q ) + i sin(r q ) is the mth root of (cos q + i sin q ) n or is a value of [(cosq + i sinq )n]1/m.
Thus cos(r q ) + i sin(r q ) is a value of (cos q + i sin q ) r. ■

Example 3.25

Find all the squares of the roots of the equation root of unity; that is, z = (- 1)1/ 7 or z = (1)1/ 4. Therefore,
x11 - x7 + x4 - 1 = 0 z2 = (- 1)2 / 7 or z2 = (1)2 / 4 = (1)1/ 2 = 1 or -1. That is
z2 = 1 or -1 or (1)1/ 7
Solution: We have x11 - x7 + x4 - 1 = x7 ( x4 - 1) + x4 - 1 =
( x7 + 1)( x4 - 1). If z is a root of x11 - x7 + x4 - 1 = 0, This implies that z2 is either a square root of 1 or a
then z must be either a seventh root of –1 or a fourth seventh root of 1.
www.jeeneetbooks.in
134 Chapter 3 Complex Numbers

Example 3.26

Find all the values of 6 - 64 . é æ 3p ö æ 3p ö ù


z4 = 2 êcos ç ÷ + i sin ç ÷ ú = - 2i
ë è 2 ø è 2 øû
Solution: First, we should express w = - 64 in trigono-
é æ 11p ö æ 11p ö ù
metric form: z5 = 2 ê cos ç ÷ø + i sin çè ÷ = 3 -i
ë è 6 6 ø úû
w = - 64 = 64(cos p + i sin p )
These lie on the circle of radius 2 with center at z = 0 and
Now, if zm are the values of 6 - 64 , then form vertices of a regular hexagon.

é æ p 2p ö æ p 2p ö ù y
zm = 6 64 êcos ç + m÷ + i sin ç + m÷
ë è 6 6 ø è6 6 ø úû z1

for m = 0, 1, 2, 3, 4 and 5. Therefore,


z2 z0 = √3 +i
æp pö
z0 = 2 cos ç + i sin ÷ = 3 + i
è6 6ø
é æpö æpöù x
z1 = 2 êcos ç ÷ + i sin ç ÷ ú = 2 i
ë è 2 ø è 2øû O

é æ 5p ö æ 5p ö ù z5
z2 = 2 êcos ç ÷ + i sin ç ÷ ú = - 3 + i z3
ë è 6 ø è 6 øû

é æ 7p ö æ 7p ö ù
z3 = 2 êcos ç ÷ + i sin ç ÷ ú = - 3 - i z4
ë è 6 ø è 6 øû
FIGURE 3.13 Example 3.27.

In the following, we express the square roots of a given complex number and nth roots of unity in algebraic form.
These are straight forward verifications.

Square Root of a Complex Number


The square roots of z = a + ib are given as
é | z| + a | z| - a ù
±ê +i ú if b > 0 (3.6a)
êë 2 2 úû
é | z| + a | z| - a ù
and ±ê -i ú if b < 0 (3.6b)
êë 2 2 úû

QUICK LOOK 9

1. The square roots of i are ± (1 + i / 2 ) 3. The square roots of -7 -24i are


é 25 - 7 25 + 7 ù
2. The square roots of - i are ± (1 - i / 2 ) ±ê -i ú = ± (3 - 4 i)
ë 2 2 û

Cube Roots of Unity


The cube roots of unity (solutions of z3 = 1) are
-1 + i 3 -1 - i 3
1, and
2 2
Usually (- 1 + i 3 )/ 2 is denoted by w. Note that 1, w, w are the cube roots of unity.
2
www.jeeneetbooks.in
3.5 De Moivre’s Theorem 135

Properties of Cube Roots of Unity


Let w ¹ 1 be a cube root of unity; that is
1
w = (- 1 ± i 3 )
2
Then the following properties are satisfied by w.

QUICK LOOK 10

1. 1 + w + w2 = 0 5. The values 1, w, w2 represent the vertices of an equi-


lateral triangle inscribed in a circle of radius 1 with
2. w3 n = 1, w3 n + 1 = w and w3 n + 2 = w2 for any integer n
center at z = 0, one vertex being on positive real axis.
3. w = w2
6. For any real numbers a, b and c,
4. (w)2 = w
a + bw + cw2 = 0 Û a = b = c

In the following we list certain important relations concerning the cube roots 1, w and w2 of unity.

Relations Concerning the Cube Roots of Unity


Let w ¹ 1 be a cube root of unity. The following relations hold good. Here x is any real or complex variable.
1. 1 + x + x2 = ( x - w)( x - w2 )
2. 1 - x + x2 = ( x + w)( x + w2 )
3. x2 + xy + y2 = ( x - yw)( x - yw2 )
4. x2 - xy + y2 = ( x + yw)( x + yw2 )
5. x3 + y3 = ( x + y)( x + yw)( x + yw2 )
6. x3 - y3 = ( x - y)( x - yw)( x - yw2 )
7. x2 + y2 + z2 - xy - yz - zx = ( x + yw + zw2 )( x + yw2 + zw)
8. x3 + y3 + z3 - 3 xyz = ( x + y + z)( x + yw + zw2 )( x + yw2 + zw)

Example 3.27

If a , b and g are roots of x3 - 3 x2 + 3 x + 7 = 0 , then find x-1


= 1, w, w2
the value of -2
a -1 b -1 g -1 which are the cube roots of unity. Therefore -1, 1 - 2w,
+ +
b -1 g -1 a -1 1 - 2w2 are the roots of the given equation. Let a = -1,
b = 1 - 2w and g = 1 - 2w2. Then a - 1 = -2, b - 1 = -2w
in terms of a cube root of unity. and g - 1 = -2w2. Hence

Solution: The given equation x3 - 3 x2 + 3 x + 7 = 0 can a -1 b -1 g -1 -2 -2w -2w2


be expressed as + + = + +
b - 1 g - 1 a - 1 -2w -2w2 -2
( x - 1)3 + 8 = 0
1 1
That is, = + + w2
w w
( x - 1)3 = (-2)3 = w2 + w2 + w2 = 3w2
3
æ x - 1ö
çè ÷ =1
-2 ø
www.jeeneetbooks.in
136 Chapter 3 Complex Numbers

Properties of nth Roots of Unity


Let n be a positive integer and
2p 2p
a = cos + i sin
n n
Then all the properties in “Quick Look 11” hold.

QUICK LOOK 11

1. 1, a , a 2 ,… , a n- 1 are all the nth roots of unity and and hence


n-1
é æ 2p ö æ 2p öù
r æ 2p
a = cos ç
ö æ 2p
r ÷ + i sin ç
ö
r÷ for 0 £ r < n å êëcos çè n r ÷ + i sin ç n
ø è
r÷ ú = 0
øû
è n ø è n ø r =0

1 - an and therefore
2. 1 + a + a 2 + + a n - 1 =
1-a n-1
æ 2p ö
n
æ 2p ö
1 - [cos(2p ) + i sin(2p )]
å cos çè
r =0
r = 0 = å sin ç
n ÷ø r =0
r
è n ÷ø
=
1-a
4. The terms 1, a , a 2 , … , a n- 1 represent the vertices of a
and therefore regular n-gon inscribed in the unit circle with center
at the origin, one vertex being on the positive real
1 + a + a 2 + + a n-1 = 0 axis.
3. The summation
n-1

åa
r =0
r
=0

Example 3.28

If 1, w, w2, …, wn-1 are all the nth roots of unity, find the Therefore
value of the product
xn - 1
(5 - w)(5 - w2 ) (5 - wn - 1 ) = ( x - w)( x - w2 ) ( x - wn - 1 )
x-1
Solution: The polynomial xn - 1 has n roots, namely This is true for all numbers x ¹ 1. Substituting x = 5, we
1, w, w2 , … , wn - 1 and hence get that
xn - 1 = ( x - 1)( x - w)( x - w2 ) ( x - wn - 1 ) 5n - 1
(5 - w)(5 - w2 ) (5 - wn - 1 ) =
4

3.6 | Algebraic Equations


Most gratifying fact about complex numbers is that any polynomial (algebraic) equation with complex numbers as
coefficients has a solution. We will discuss the same in this section.

DEF IN IT ION 3 . 13 Let f (z) = a0 + a1z + a2 z2 + + an zn, an ¹ 0 and a0 , a1 , … , an complex numbers. Then
f(z) = 0
is called an algebraic equation of degree n. Any algebraic equation of degree 2 is called a
quadratic equation. A complex number z0 is called a solution or root of the equation f(z) = 0 if
f(z0) = 0; that is,
a0 + a1z0 + a2 z02 + + an z0n = 0
www.jeeneetbooks.in
3.6 Algebraic Equations 137

QUICK LOOK 12

1. 2 + i + z = 0 is an algebraic equation of degree 1 and 4. 32(1 - i)z + iz7 = 0 is an algebraic equation of degree
z0 = -2 -i is the only root of this. 7 and z0 = 0 is a root of this equation. In addition to
z0 = 0, any root of the equation z6 = 32(1 + i) must be
2. z2 - 1 = 0 is an algebraic equation of degree 2 and z0 = 1
a root of the given equation and hence there must be
and z1 = -1 are the roots of the equation z2 - 1 = 0. six more roots for the given equation.
3. i + iz2 + z3 + z5 = 0 is an algebraic equation of degree
5 and z0 = i is a root of this equation.

The general form of an algebraic equation of the first degree is

a0 + a1z = 0, a1 ¹ 0

Such an equation possesses exactly one solution z0 = -a0/a1. An equation of the second degree is generally written as
a0 + a1z + a2 z2 = 0, a2 ¹ 0
To solve this, we transform the equation as follows:

æ a a ö
a2 ç z2 + a1 z + a0 ÷ = 0
è 2 2ø

éæ a ö
2
a a2 ù
a2 êç z + 1 ÷ + 0 - 1 2 ú = 0
êëè 2a2 ø a2 4a2 ú
û
éæ a ö
2
a2 - 4a0 a2 ù
a2 êç z + 1 ÷ - 1 ú=0
êëè 2a2 ø 4a22 úû

and find the roots of


2
æ a1 ö a12 - 4a0 a2
çè z + =
2a2 ÷ø 4a2

as

-a1 a12 - 4a0 a2


z= +
2a2 2a2
that is,

-a1 + D
z=
2a2

where D = a12 - 4a0 a2 . D is called the discriminant of the equation a0 + a1z + a2z2 = 0. D is to be understood as all the
values of the square root of D. The formula

-a1 + D
z=
2a2
for the roots of a quadratic equation has the same form as in the case when the coefficients of the equation are real
numbers and the solutions are thought in the set of real numbers. But in as much as in the set of complex numbers the
operation of extracting a square root is meaningful for any complex number, the restriction D > 0 becomes superflu-
ous. Moreover, the restriction loses sense since the discriminant D may prove to be not a real number, and the concepts
of “greater than” and “less than” are not defined for such numbers. Thus, in the set of complex numbers, any quadratic
equation is always solvable. If the discriminant D is zero, then the equation has one root. If D ¹ 0, the equation has two
roots that are given by the formula
www.jeeneetbooks.in
138 Chapter 3 Complex Numbers

-a1 + D
z0 =
2a2
This is known as the standard formula for the roots of a quadratic equation.

Example 3.29

Solve the equations: are the solutions of the given equation. To find all
the values of 3 - 4i, we can use the formula given
(1) z2 + 3z + 3 = 0
in Eqs. (3.6a) and (3.6b). But another technique is
(2) z2 - 8z - 3iz + 13 + 13i = 0 much simpler. Let us put

Solution: 3 - 4i = x + iy

(1) By the formula for the roots of a quadratic equation, Then 3 - 4i = x2 - y2 + i(2 xy) and therefore
the roots of z2 + 3z + 3 = 0 are given by x2 - y2 = 3 and xy = - 2
-3 + 9 - 12 -3 + -3 x and y being real numbers. This system of simultane-
z= =
2 2 ous equations has two real solutions, x = 2, y = -1 and
x = -2, y = 1. Therefore
Since -3 = ± i 3 , it follows that
3 - 4i = 2 - i or - 2 + i
-3 + i 3 -3 - i 3
z1 = and z2 = Thus,
2 2
8 + 3i + 2 - i
are the solutions of the equation z2 + 3z + 3 = 0. z1 = =5+i
2
(2) The given equation can be written as
8 + 3i - 2 + i
(13 + 13i) + (-8 - 3i)z + z2 = 0 and z2 = = 3 + 2i
2
By the standard formula for the roots of a quadratic
equation, we get that are the solutions of the given quadratic equation.

8 + 3i + (8 + 3i)2 - 4(13 + 13i) 8 + 3i + 3 - 4i


z= =
2 2

Solving algebraic equations of degree n > 2 is much more difficult. However, the great German mathematician
Carl Gauss proved the following celebrated theorem in 1799. In view of its importance and in honor of Gauss, the
theorem is named after Gauss and is popularly known as the Fundamental Theorem of Algebra. Its proof is beyond
the scope of this book and hence not given here.

Fundamental Theorem of Algebra


Every algebraic equation has atleast one root in the set of complex numbers.
The following theorem is an important consequence of the fundamental theorem of algebra.

T H E O R E M 3 .16 Every algebraic equation of degree n has exactly n roots, including the repeatitions (multiplicities)
of the roots, in the set of complex numbers.
PROOF Let
f (z) = a0 + a1z + a2 z2 + + an zn , an ¹ 0
where a0 , a1 , a2 , … , an are all complex numbers. Then f(z) = 0 is an algebraic equation of degree n.
It can be proved that, for any complex number w,
f (z) = (z - w) g(z)
www.jeeneetbooks.in
3.6 Algebraic Equations 139

for some polynomial g(z) with complex coefficients if and only if w is a root of the equation
f(z) = 0; that is, f(w) = 0. This, together with the fundamental theorem of algebra, gives us that
f (z) = an (z - z1 )r1 (z - z2 )r2 (z - zk )rk

where z1, z2, ¼, zk are distinct complex numbers and r1, r2, ¼, rk are positive integers such that
r1 + r2 + + rk = n
Therefore, if follows that z1, z2, ¼, zk are all the distinct roots of the equation f(z) = 0. Here we
say that zi is a root of multiplicity ri. If we agree to count the root of the equation as many
times as is its multiplicity, then we get that the equation f(z) = 0 has r1 + r2 + + rk (= n) roots in
the set of complex numbers. ■

Theorem 3.16 and the fundamental theorem of algebra are both typical theorems of existence. They both present
a comprehensive solution of the problem on the existence of roots of an arbitrary algebraic equation; but, unfortu-
nately they do not say how to find these roots. The root of the first-degree equation
a0 + a1z = 0
is determined by the formula
a0
z=-
a1
and the roots of the second-degree equation
a0 + a1z + a2 z2 = 0
are determined by the formula

-a1 + D
z=
2a2
where D is the determinant defined by
D = a12 - 4a0 a2

The analogous formulae for the roots of third- and fourth-degree equations are so cumbersome that they are avoided.
There is no general method for finding the roots of algebraic equations of degree greater than 4. The absence of a
general method does not prevent us, of course, from finding all the roots in certain special cases, depending on the
specific nature of the equation. For example, in Theorem 3.14, we discussed a method to find all the roots of the
equation
a0 + an zn = 0
The following theorem often helps us in solving algebraic equations with integral coefficients.

T H E O R E M 3 .17 Let f (z) = a0 + a1z + a2 z2 + + an zn , an ¹ 0, where a0, a1, a2, ¼, an are all integers. If k is an integer
and is a root of f(z) = 0, then k is a divisor of a0.

PROOF Let k be an integer and f(k) = 0. That is, a0 + a1k + a2 k 2 + + an k n = 0, and hence a0 = k(-a1 -
a2k - - ankn-1). Since k and a1, a2, ¼, an are integers, so is -a1 - a2 k - - an k n - 1. Therefore k is a
divisor of a0. ■

Example 3.30

Solve the equation Solution: Note that all the coefficients are integers. By
considering the divisors of the constant term -6 and by
z3 - z - 6 = 0
www.jeeneetbooks.in
140 Chapter 3 Complex Numbers

using Theorem 3.17, we get that 2 is the only integral root are
of z3 - z - 6 = 0. By the usual division of z3 - z - 6 by
z - 2, we get that -2 ± 4 - 12
2
(z - 2)(z2 + 2z + 3) = z3 - z - 6
Thus, z1 = 2, z2 = - 1 + 2i and z3 = - 1 - 2i are all the
Therefore, the roots of z3 - z - 6 = 0 are precisely the
roots of the equation z3 - z - 6 = 0.
roots of z2 + 2z + 3 = 0 and 2 . The roots of z2 + 2z + 3 = 0

Example 3.31

Solve the equation f (z) = (z2 - 4)(-18 + 9z + 2z2 - z3 )


72 - 36z - 26z2 + 13z3 + 2z4 - z5 = 0 Again –3 and 3 are roots of -18 + 9z + 2z2 - z3 and
-18 + 9z + 2z2 - z3 = (z2 - 9)(z - 2). Therefore,
Solution: Let f (z) = 72 - 36z - 26z2 + 13z3 + 2z4 - z5. Note
that all the coefficients are integers. Consider the constant f (z) = (z - 2)(z + 2)(z - 3)(z + 3)(z - 2)
term 72. Testing the divisors of the constant term 72, we find = (z - 2)2 (z + 2)(z - 3)(z + 3)
that z1 = 2 and z2 = -2 are roots of the given equation. By
dividing f (z) with (z - 2)(z + 2) = z2 - 4, we get that Thus the roots of f (z) = 0 are 3, - 3, - 2 and 2 and the
root 2 is of multiplicity 2.

WORKED-OUT PROBLEMS
Single Correct Choice Type Questions
1. If 4 1
= =
8-4 3 2- 3
3+i a+i
= Answer: (D)
2 a-i
and a is a real number, then a is 2. If z1, z2 are complex numbers such that Re(z1 ) = | z1 - 2|,
(A) 1/2 + 3 (B) 1/2 - 4 3 Re(z2 ) = | z2 - 2| and arg (z1 - z2 ) = p / 3, then Im(z1 - z2 ) =
(C) 2 - 3 (D) 1/2 - 3 (A) 2 / 3 (B) 4 / 3 (C) 2 3 (D) 3
Solution: The equation Solution: Let z1 = x1 + iy1 and z2 = x2 + iy2 . Then

3+i a+i x12 = ( x1 - 2)2 + y12 and x22 = ( x2 - 2)2 + y22


=
2 a-i Therefore
implies that 4 x1 = y12 + 4 and 4 x2 = y22 + 4
( 3 + i)(a - i) = 2a + 2i On subtraction we get
that is, a( 3 - 2 + i) = ( 3 + 2)i - 1. Therefore 4( x1 - x2 ) = y12 - y22 = ( y1 + y2 )( y1 - y2 )
( 3 + 2)i - 1 Hence
a=
3-2+i 4( x1 - x2 )
y1 + y2 = (3.7)
[( 3 + 2)i - 1][( 3 - 2) - i] y1 - y2
=
[( 3 - 2) + i][( 3 - 2) - i] Also arg (z1 - z2 ) = p / 3. Therefore
(3 - 4))i - 3 + 2 + i + 3 + 2 p y1 - y2
= tan =
( 3 - 2) + 1
2 3 x1 - x2
y1 - y2
3= (3.8)
x1 - x2
www.jeeneetbooks.in
Worked-Out Problems 141

From (3.7) and (3.8), we have Û 12 < 4 x

Im (z1 + z2 ) = y1 + y2 =
4 Û3< x
3 Answer: (D)
Answer: (B)
6. If
3. The smallest positive integer n for which [(1 + i)/
3
(1 - i)]n = 1 is x + iy =
2 + cos q + i sin q
(A) 2 (B) 4 (C) 6 (D) 7
Solution: We have then x2 + y2 =
(A) 4x - 3 (B) 3x - 4 (C) 4x + 3 (D) 3x + 4
1 + i (1 + i)2
= = i and in = 1 for n = 4, 8, 12, … Solution:
1- i 2
3(2 + cos q - i sin q )
Therefore, the smallest positive integer n for which x + iy =
(2 + cos q )2 + sin2 q
n
æ 1 + iö
çè 1 - i ÷ø = 1 is 4 =
3(2 + cos q ) + i(-3 sin q )
5 + 4 cosq
Answer: (B)
Comparing the real and imaginary parts we get
4. Let C be the set of all complex numbers and 3(2 + cos q ) -3 sin q
x= , y=
ì z -z ü 5 + 4 cos q 5 + 4 cos q
R = í(z1 , z2 ) ÎC ´ C : 1 2 is real ý
î z1 + z2 þ Squaring and adding values of x and y, we get
Then, on C, R is a 9(2 + cos q )2 + 9 sin2 q
(A) reflexive relation (B) symmetric relation x2 + y2 =
(5 + 4 cos q )2
(C) transitive relation (D) equivalence relation
9(5 + 4 cos q ) 9
Solution: Since (0, 0) ÏR, R is not reflexive, we have = =
(5 + 4 cos q )2
5 + 4 cosq
z1 - z2
(z1 , z2 ) Î R Þ is real Also
z1 + z2
z -z 12(2 + cos q ) 9
Þ 2 1 is real 4x - 3 = -3=
z1 + z2 5 + 4 cos q 5 + 4 cos q
Þ (z2 , z1 ) Î R Therefore
Therefore R is symmetric. x2 + y2 = 4 x - 3
Since (0, z) Î R and (z, 0) Î R, but (0, 0) Ï R, there-
fore R is not transitive. Hence R is not an equivalence Answer: (A)
relation.
7. If
Answer: (B)
3+i
5. If z = x + iy is such that z - 4 < z - 2 , then x + iy =
1 + 3i
(A) x > 0, y > 0
(B) x < 0, y > 0 then ( x2 + y2 )2 equals
(C) x > 2, y > 3 (A) 0 (B) 2 (C) 3 (D) 1
(D) x > 3 and y is any real number Solution:
Solution: We have 3 + i (3 + i)(1 - 3i)
x2 - y2 + 2ixy = =
2 2 1 + 3i 1+ 9
z-4 < z-2 Û z-4 < z-2
Comparing the real and imaginary parts we get
Û ( x - 4)2 + y2 < ( x - 2)2 + y2
6 -8
x2 - y2 = and 2 xy =
10 10
www.jeeneetbooks.in
142 Chapter 3 Complex Numbers

Now is equal to
(x + y ) = (x - y ) + 4x y
2 2 2 2 2 2 2 2 (A) 1 (B) –1 (C) 0 (D) 1/2
2 2 Solution: We have
æ 6 ö æ -8 ö 9 16
=ç ÷ +ç ÷ = + =1
è 10 ø è 10 ø 25 25 (k - w)(k - w2 ) = k 2 + k + 1
Answer: (D) Therefore

8. If a is a positive real number, z = a + 2i and z| z | - æ 10 p ö æ 10 p ö


cos ç å (k - w)(k - w2 ) ÷ = cos ç å (k 2 + k + 1)
az + 1 = 0, then è k =1 450 ø è k =1 450 ÷ø
(A) z is pure imaginary æ p ö
= cos ç 450 × ÷
(B) a2 = 2 è 450 ø
(C) a2 = 4 = cos p = - 1
(D) no such complex number exists Answer: (B)
Solution:
11. If a = - 1 + i 3 and n is a positive integer which is
z | z | - az + 1 = 0 not a multiple of 3, then
(a + 2i) a2 + 4 = a(a + 2i) - 1 a 2 n + 2n a n + 2 2 n =
(A) 1 (B) −1 (C) 0 (D) a 2
= a2 - 1 + 2ai
Solution: We have
This implies
éæ a ö 2 n æ a ö n ù
a a2 + 4 = a2 - 1 and 2 a2 + 4 = 2a a 2 n + 2n a n + 22 n = 22 n êç ÷ + ç ÷ + 1ú
êëè 2 ø è 2ø úû
which gives a2 = a2 - 1, which is absurd.
Answer: (D) é a -1 i 3 æaö
3
ù
= 22 n (w2 n + wn + 1) ê∵ = + and ç ÷ = 1ú
êë 2 2 2 è 2ø úû
9. If | z1 + z2 | = | z1| + | z2 |, then one of the values of
arg(z2/z1) is = 22 n (0) = 0 (since 3 does not divide n)
(A) 0 (B) p (C) p/2 (D) 3p Answer: (C)
Solution: If z1 + z2 = z1 + z2 , then z1, z2 and origin
12. If arg (z) < 0, then arg ( - z) - arg(z) =
are collinear and z1, z2 lie on same side to origin and
hence arg(z2/z1) = 2np. Then 0 is one of the values of (A) p (B) -p (C) p/2 (D) -p/2
arg(z2/z1). Solution: Let arg (z) = q < 0. Then -p < q < 0 and there-
Answer: (A) fore 0 < q + p < p. Hence
Alternate Method:
arg ( - z) = p + q
Let z1 = r1(cosq1 + i sinq1) and z2 = r2(cosq2 + i sinq2). Then
z1 + z2 = z1 + z2 implies arg ( - z) - arg (z) = p
Answer: (A)
(r1 cos q1 + r2 cos q2 )2 + (r1 sin q1 + r2 sin q2 )2 = (r1 + r2 )2
13. Let w ¹ 1 be a cube root of unity and
That is
E = 2(1 + w)(1 + w2 ) + 3(2w + 1)(2w2 + 1)
cos(q1 - q2 ) = 1
+ 4(3w + 1)(3w2 + 1) +
Therefore
+ (n + 1)(nw + 1)(nw2 + 1)
q1 - q2 = 2 np
Then E is equal to
10. If w is a cube root of unity and w ¹ 1, then n2 (n + 1)2 n2 (n + 1)2
(A) (B) +n
4 4
æ 10 p ö
cos ç å (k - w)(k - w2 ) n (n + 1)2
2
n2 (n + 1)2
è k =1 450 ÷ø (C) -n (D) - (n + 1)
4 4
www.jeeneetbooks.in
Worked-Out Problems 143

  
Solution: We have by a + b + c and hence by z1 + z2 + z3. Note that the origin
(k + 1)(kw + 1)(kw2 + 1) = (k + 1)(k 2 - k + 1) = k 3 + 1 is the circumcenter.
Answer: (B)
Therefore,
n n
n2 (n + 1)2 16. If z1, z2 and z3 are the vertices of an equilateral triangle
E = å (k 3 + 1) = å k3 + n = +n
k =1 k =1 4 and z0 be its orthocenter, then z12 + z22 + z32 = kz 02 , where
k is equal to
Answer: (B)
(A) 3 (B) 2 (C) 6 (D) 9
14. If z - 3 + 2i £ 4, then the absolute difference between Solution: In an equilateral triangle, the circumcenter,
the maximum and minimum values of | z | is the centroid and the orthocenter are one and the same
(A) 2 11 (B) 3 11 (C) 2 13 (D) 3 13 point. Therefore

Solution: Let C = 3 - 2i be the center of the circle z1 + z2 + z3


z0 =
z - 3 + 2i = 4. Join the origin to C and let it meet the 3
circle in A and B (see figure). 9z 02 = z12 + z22 + z32 + z(z1 z2 + z2 z3 + z3 z1 )
Least value of | z | = OB = 3(z 12 + z 22 + z 23)
[since z1 + z2 + z3 = å z1 z2 (by Problem 7 of Multiple
2 2 2
= CB - OC
Correct Choice Type Questions in Worked-Out Problems
= 4 - 32 + 22 section)]. Therefore
= 4 - 13 z12 + z22 + z32 = 3z02 and k = 3
Maximum value of | z | = OA = 4 + 13 Answer: (A)
The absolute difference between the maximum and 17. Let z1 = 10 + 6i and z2 = 4 + 6i. If z is any complex
minimum values of | z | is 2 13. number such that the argument of (z - z1)/(z - z2) is
p/4, then | z - 7 - 9i | is equal to
(A) 2 3 (B) 3 2 (C) 3 (D) 2
B
3
O Solution: Let z = x + iy, x, y Î  . Then z - z1 = ( x - 10) +
2
C i( y - 6) and z - z2 = ( x - 4) + i( y - 6). Therefore
z - z1 ( x - 10) + i( y - 6)
A =
z - z2 ( x - 4) + i( y - 6)

Answer: (C) [( x - 10) + i( y - 6)][( x - 4) - i( y - 6)]


=
( x - 4)2 + ( y - 6)2
15. If z1, z2 and z3 represent the vertices of a triangle
whose circumcenter is at the origin, then the complex Therefore
number representing the orthocenter of the triangle is z - z1 ( x - 10)( x - 4) + ( y - 6)2
1 1 1 Real part of =
(A) + + (B) z1 + z2 + z3 z - z2 ( x - 4)2 + ( y - 6)2
z1 z2 z3
z - z1 ( x - 4)( y - 6) - ( x - 10)( y - 6)
1 1 1 Imaginary part of =
(C) + + (D) z1z2 + z2 z3 + z3 z1 z - z2 ( x - 4)2 + ( y - 6)2
z1z2 z2 z3 z3 z1
6( y - 6)
Solution: It is known that every complex number can =
( x - 4)2 + ( y - 6)2
be represented by means of a vector in the Argand’s
plane. If A and B represent the complex numbers z1 Now,
and z2, respectively, then the vector AB represents the
complex number z2 - z1 . (These matters will be discussed p æ z - z1 ö é 6( y - 6) ù
   = arg ç ÷ = tan-1 ê 2 ú
in detail later in Volume II) Correspondingly, if a, b, c 4 è z - z2 ø ë ( x - 10)( x - 4) + ( y - 6) û
are the position vectors of the points A(z1 ), B(z2 ), C (z3 ),
then the orthocenter of the triangle ABC is represented
www.jeeneetbooks.in
144 Chapter 3 Complex Numbers

Therefore (A) 27 (B) 18 (C) 54 (D) -27


( x - 10)( x - 4) + ( y - 6)2 = 6( y - 6) Solution: x + x + 1 = 0 Þ x is a non-real cube root of
2

unity. Let x = w ¹ 1 be a cube root of unity. Then w3 = 1


x + y - 14 x - 18 y + 112 = 0
2 2
and 1 + w + w2 = 0. The given equation, thus, becomes
Now, 2 2 2 2
æ 1ö æ 2 1 ö æ 3 1 ö æ 27 1 ö
| z - 7 - 9 i | = ( x - 7) + ( y - 9 )
2 2 2 çè w + ÷ø + çè w + 2 ÷ø + çè w + 3 ÷ø + çè w + 27 ÷ø
w w w w
2 2
= x2 - 14 x + y2 - 18 y + 130 æ w2 + 1ö æ w4 + 1ö æ 1ö
2

=ç ÷ +ç ÷ + (1 + 1)2 + ç w + ÷ +
= - 112 + 130 = 18 è w ø è w ø è wø
+ (1 + 1)
2
Therefore

| z - 7 - 9i | = 18 = 3 2 éæ -w ö 2 æ -w2 ö 2 ù
= 9 êç ÷ + ç 2 ÷ + (1 + 1)2 ú
Answer: (B) êëè w ø è w ø úû
= 9(1 + 1 + 4) = 54
18. If x = cos a + i sin a and y = cos b + i sin b, then(x − y)/
(x + y) is equal to Answer: (C)

æa - bö æa - bö 21. If z is a complex number and i = -1, then the


(A) i tan ç (B) - i tan ç
è 2 ÷ø è 2 ÷ø minimum possible value of | z | 2 + | z - 3 | 2 + | z - 6i | 2 is
æa + bö æa + bö (A) 15 (B) 30 (C) 20 (D) 45
(C) i tan ç (D) -i tan ç
è 2 ÷ø è 2 ÷ø Solution: Let z = x + iy. Then
Solution: We know that
| z2 | + | z - 3 | 2 + | z - 6i | 2 = x 2 + y 2 + ( y - 3)2 + y2 + x2 + ( y - 6)2
x - y (cos a - cos b ) + i(sin a - sin b ) = 3( x2 + y2 ) - 6 x - 12 y + 45
=
x + y (cos a + cos b ) + i(sin a + sin b ) = 3[( x - 1)2 + ( y - 2)2 ] + 30 ³ 30
- 2 sin[(a + b )/ 2]sin[(a - b )/ 2] (equality holds when z = 1 + 2i). Therefore, the minimum
+ 2i cos[(a + b )/ 2]sin[(a - b )/ 2] value is 30.
=
2 cos[(a + b )/ 2]cos[(a - b )/ 2] Answer: (B)
+ 2i sin[(a + b )/ 2]cos[(a - b )/ 2]
22. The curve in the complex plane given by the equa-
i sin[(a - b ) / 2]{cos[(a + b )/ 2] + i sin[(a + b )/ 2]} tion Re(1/z) = 1/4 is a
=
cos[(a - b )/ 2]{cos[(a + b )/ 2] + i sin[(a + b )/ 2]} (A) vertical line intersecting with the x-axis at (4, 0)
æa - bö (B) a circle with radius 2 and centre at (2, 0)
= i tan ç
è 2 ÷ø (C) circle with unit radius
Answer: (A) (D) straight line not passing through the origin
Solution: Let z = x + iy, where x and y are reals. Then
19. If | z1 - 1 | < 1, | z2 - 2 | < 2 and | z3 - 3 | < 3, then | z1 +
z2 + z3 | æ 1ö 1 æ x - iy ö 1
Re ç ÷ = Þ Re ç 2 =
(A) is less than 6 (B) is greater than 6 è zø 4 è x + y2 ÷ø 4
(C) is less than 12 (D) lies between 6 and 12 x 1
Þ =
Solution: We have x +y
2 2
4

| z1 + z2 + z3 - 6 + 6 | £ | z1 + z2 + z3 - 6 | + 6 Þ x2 + y2 = 4 x
= |(z1 - 1) + (z2 - 2) + (z3 - 3)| + 6 Þ ( x - 2)2 + y2 = 4 = 22
£ | z1 - 1| + | z2 - 2 | + | z3 - 3 | + 6 This is the equation of the circle with radius 2 and center
< 1 + 2 + 3 + 6 = 12 at (2, 0).
Answer: (C) Answer: (B)

20. If x2 + x + 1 = 0 then the value of 23. The origin and the points represented by the roots of
2 2 2 2
the equation z2 + mz + n = 0 form the vertices of an
æ 1ö æ 2 1 ö æ 3 1 ö æ 27 1 ö equilateral triangle if and only if
çè x + ÷ø + çè x + 2 ÷ø + çè x + 3 ÷ø + + çè x + 27 ÷ø is
x x x x
www.jeeneetbooks.in
Worked-Out Problems 145

(A) m2 = 3n (B) n2 = 3m Then | z1 + z2 | is equal to


(C) 3m2 = n (D) 3n2 = m æ 2ö æ 2ö
(A) 2 cos ç 20 cos-1 ÷ (B) 2 sin ç 10 cos-1 ÷
è 3ø è 3ø
Solution: The points z1, z2 and z3 are the vertices of an
equilateral triangle if and only if æ 2ö æ 2ö
(C) 2 cos ç 10 cos-1 ÷ (D) 2 sin ç 20 cos-1 ÷
z + z + z = z1 z2 + z2 z3 + z3 z1
2 2 2 è 3ø è 3ø
1 2 3

(see Problem 7 of Multiple Correct Choice Type Questions Solution: Adding the two we get
in Worked-Out Problems section). Let z1 and z2 be the
(2 + i 5 ) 20 + (2 - i 5 ) 20
roots of z2 + mz + n = 0 . Therefore z1 + z2 = - m, z1 z2 = n . z1 + z2 =
Now z1, z2 and the origin form an equilateral triangle if 9 10
and only if Suppose 2 + i 5 = r (cos q + i sin q ), so that r = 22 + 5 = 3,
z12 + z22 = z1 z2 r cos q = 2 and r sin q = 5. Therefore

Û (z1 + z2 )2 = 3z1 z2 2 5
cos q = and sin q =
3 3
Û (- m)2 = 3n
Answer: (A) In this case
1 20
24. Let z = x + iy, where x and y are real. The points z1 + z2 = [r {cos(20 q ) + i sin(20 q )
910
(x, y) in the plane, for which (z + i)/(z - i) is purely + cos(20 q ) - i sin(20 q )}]
imaginary, lie on
r20 320 é æ 2ö ù
(A) a straight line = 2 cos(20 q ) = 10 2 cos ê 20 cos-1 ç ÷ ú
910
9 ë è 3ø û
(B) a circle
é æ 2ö ù
(C) a curve whose equation is of the form = 2 cos ê 20 cos-1 ç ÷ ú
ë è 3ø û
x2 y2
+ = 1, a ¹ 1, b ¹ 1 Answer: (A)
a2 b2
(D) a curve whose equation is of the form 26. If (1 + z)n = a0 + a1z + a2z2 + + anzn, where a0, a1,
a2, …, an, are real, then
x2 y2
- =1 (a0 - a2 + a4 - a6 + )2 + (a1 - a3 + a5 - a7 + )2 =
a2 b2
(A) 2n (B) a02 + a12 + a22 + + an2
Solution: We have 2
(C) 2n (D) 2 n2
z + i x + i( y + 1)
= Solution: Substitute z = i on both sides. Then
z - i x + i( y - 1)
[ x + i( y + 1)][ x - i ( y - 1)] (1 + i)n = (a0 - a2 + a4 - a6 + ) + i(a1 - a3 + a5 - a7 + )
=
x2 + ( y - 1)2 Therefore
This is pure imaginary if and only if | 1 + i |2 n = (a0 - a2 + a4 - a6 + )2 + (a4 - a3 + a5 - a7 + )2
æ z + iö 2n = (a0 - a2 + a4 - a6 + ))2 + (a1 - a3 + a5 - a7 + )2
Re ç =0
è z - i ÷ø Answer: (A)
x + ( y - 1)
2 2
Û =0 27. Let z1 and z2 be roots of the equation z + pz + q = 0,
2
x2 + ( y - 1)2
where p, q may be complex numbers. Let A and B
Û x2 + y2 = 1 represent z1 and z2 in the complex plane. If ÐAOB = a ¹ 0
and OA = OB, where O is the origin, then
Therefore (x, y) lie on the circle | z | = 1.
æaö æaö
Answer: (B) (A) p2 = 4 q cos2 ç ÷ (B) p2 = 4 q sin2 ç ÷
è 2ø è 2ø
25. Let z1 and z2 be given by æaö æaö
(C) p2 = - 4 q cos2 ç ÷ (D) q2 = 4 p sin2 ç ÷
10 10 è 2ø è 2ø
æ 2 + i 5ö æ 2 - i 5ö
z1 = ç ÷ and z2 = ç ÷ Solution: z1 and z2 are roots of z2 + pz + q = 0. This
è 2 - i 5ø è 2 + i 5ø
implies z1 + z2 = -p and z1z2 = q. Now
www.jeeneetbooks.in
146 Chapter 3 Complex Numbers

z2 - 0 OB Therefore
= (cos a + i sin a )
z1 - 0 OA é æ pö 3 æ p ö 3ù
z3 / 4 = 23 / 8 êcos ç 2kp + ÷ + i sin ç 2kp + ÷ ú
y ë è ø
4 4 è 4ø 4û
for k = 0, 1, 2, 3. The product of the values of this is equal
B(z2)
to

é æ p 9p 17p 25p ö 3ù æ 52p 3 ö


A(z1) 23 / 2 êcis ç + + + ÷ = 23 / 2 cis ç ×
ë è4 4 4 4 ø 4 úû è 4 4 ÷ø
a
39p
x = 23 / 2 cis
O 4
æ 3p ö
= 23 / 2 cis ç 9p + ÷
è 4ø
Therefore
æ pö
z2 = 23 / 2 cis ç 10p - ÷
= cos a + i sin a è 4ø
z1
z2 - z1 é æ pö æ pöù
= - 1 + cos a + i sin a = 23 / 2 êcos ç 10p - ÷ + i sin ç 10p - ÷ ú
z1 ë è 4 ø è 4øû

This gives é p pù
= 23 / 2 êcos - i sin ú
2 ë 4 4û
é a a aù
(z2 - z1 )2 = z12 ê -2 sin2 + 2 i sin cos ú
ë 2 2 2û æ 1 i ö
= 23 / 2 ç - ÷
è 2 2ø
2 2
æ aö é a aù
= z12 ç 2i sin ÷ êcos + i sin ú = 2(1 - i)
è 2ø ë 2 2û
Answer: (B)
a
= - 4z sin (cos a + i sin a )
2
1
2

2 29. If z1, z2 and z3 are the vertices of a right-angled isos-


celes triangle, right-angled at the vertex z2 (see
z2 a a figure), then z12 + 2z22 + z32 = kz2 (z1 + z3 ), where the value
= - 4z12 sin2 = - 4q sin2
z1 2 2 of k is
Hence, (A) 0 (B) 1 (C) -2 (D) 2
p2 = (z1 + z2 )2 = (z1 - z2 )2 + 4z1 z2 A(z1)
a
= - 4q sin2 + 4q
2
æ aö æaö
= 4q ç 1 - sin2 ÷ = 4q cos2 ç ÷
è 2 ø è 2ø
Answer: (A)
90°
28. The continued product of all the four values of the
complex number (1 + i)3 / 4 is B(z2) C(z3)
(A) 2 (1 + i)
3
(B) 2(1 - i) Solution: Let A, B and C represent z1, z2 and z3, respec-
(C) 2(1 + i) (D) 23(1 - i) tively, described in counterclock sense. Therefore
Solution: Let z1 - z2 BA æ p ö
= cis ç ÷ = i
æ p pö z3 - z2 BC è 2 ø
z = 1 + i = 2 ç cos + i sin ÷
è 4 4ø (z1 - z2 )2 = - (z3 - z2 )2
www.jeeneetbooks.in
Worked-Out Problems 147

z12 + z22 - 2z1z2 = - z32 - z22 + 2z2 z3 æz ö æz -z ö


arg ç 3 ÷ = 2q = 2 arg ç 3 1 ÷
z + 2z + z = 2z2 (z1 + z3 )
2
1
2
2
2
3
è z2 ø è z2 - z1 ø
This gives k = 2. Therefore k = 2.
Answer: (D) Answer: (C)

30. Let z1, z2 and z3 be vertices of a triangle and | z1 | = a, 31. Let z = ( 3 / 2) - (i / 2). Then the smallest positive
| z2 | = b and | z3 | = c such that integer n such that (z95 + i 67 )94 = zn is
a b c (A) 12 (B) 10 (C) 9 (D) 8
b c a =0 Solution: From the hypothesis we have
c a b
Then 3 i æ 1 i 3ö
z= - = iç- - = iw
æz ö æz -z ö 2 2 è 2 2 ÷ø
arg ç 3 ÷ = k arg ç 3 1 ÷
è z2 ø è z2 - z1 ø where w = (- 1/ 2) - (i 3 / 2) which is a cube root of unity.
where k is Now, z95 = (iw)95 = -iw2 (since w3 = 1) and i67 = i3 = -i.
Therefore,
(A) 0 (B) 1 (C) 2 (D) 3
Solution: We have z95 + i67 = - i(1 + w2 ) = (- i)(- w) = iw
(z95 + i67 )94 = (iw)94 = i2 w = - w
a b c
b c a =0 Now
c a b - w = zn = (iw)n

Þ 3abc - a3 - b3 - c3 = 0 Þ in × wn - 1 = - 1
Þ n = 2, 6, 10, 14, … and n - 1 = 3, 6, 9, …
Þ a3 + b3 + c3 - 3abc = 0
Therefore n = 10 is the required least positive integer.
Þ (a + b + c)(a2 + b2 + c2 - ab - bc - ca) = 0
Answer: (B)
1
Þ (a + b + c) ((a - b)2 + (b - c)2 + (c - a)2 ) = 0
2 32. The number of complex numbers z satisfying the
conditions |(z / z ) + (z / z)| = 1, | z| = 1 and arg z Î(0, 2p) is
Therefore (a - b)2 = 0 = (b - c)2 = (c - a)2 and hence a = b = c
(since a, b, c are positive). This implies that z1, z2 and z3 rep- (A) 1 (B) 2 (C) 4 (D) 8
resent points on a circle with center at the origin. Suppose Solution: It is given that | z | = 1 which implies that z =
A, B and C represent z1, z2 and z3, respectively, described cosq + i sinq, 0 £ q < 2p :
in counterclock sense (see figure). If ÐBAC = q , then
ÐBOC = 2q. In such case z z
+ =1
z z
Þ 2 |cos 2q | = 1
B(z2)
1 -1
Þ cos 2q = or cos 2q =
2 2
q A(z1)
Now
O
1 p 5p 7p 11p
cos 2q = Þq = , , ,
2 6 6 6 6
1 p 2p 4p 5p
and cos 2q = - Þq = , , ,
C(z3) 2 3 3 3 3
1 Answer: (D)
www.jeeneetbooks.in
148 Chapter 3 Complex Numbers

Multiple Correct Answer Type Questions


1. The complex number z that satisfies simultaneously z1 z2 and z1/z2 are pure imaginary.
the equations is Answers: (A), (B), (C), (D)
z - 4i z - 8 + 3i 3
= 1 and = 3. If z1 and z2 are two complex numbers, then
z - 2i z + 3i 5
(A) 2(| z1 |2 + | z2 |2 ) = | z1 + z2 |2 + | z1 - z2 |2
(A) 3 + 8 i (B) 8 + 3 i (C) 3 + 17 i (D) 17 + 3 i
(B) | z1 + z12 - z22 | + | z1 - z12 - z22 | = | z1 + z2 | + | z1 - z2 |
Solution:
z1 + z2 z +z
z - 4i (C) + z1z2 + 1 2 - z1 z2 = | z1 | + | z2 |
= 1 Þ | z - 4i | = | z - 2i | 2 2
z - 2i
(D) | z1 + z2 | 2 - | z1 - z2 | 2 = 2(z1z2 + z1z2 )
Therefore, the point representing z in the Argand’s plane
Solution: | z1 + z2 | 2 = (z1 + z2 )(z1 + z2 )
is equidistant from the points (0, 2) and (0, 4). Hence, z
lies on the line y = 3 and so = | z1 | 2 + | z2 | 2 + z1z2 + z1z2
z = x + yi = x + 3i and | z1 - z2 | 2 = (z1 - z2 )(z1 - z2 )
Substituting z = x + 3i in the second equation, we get that Therefore
x + 3i - 8 + 3i 3 | z1 + z2 | 2 + | z1 - z2 | 2 = 2(| z1 | 2 + | z2 | 2 ) (A is true)
=
x + 3i + 3i 5
| z1 + z2 | - | z1 - z2 | = 2(z1z2 + z1z2 )
2 2
(D is true)
x - 8 + 6i 3
= Now
x + 6i 5
Therefore (| z1 + z12 - z22 | + | z1 - z12 - z22 |) 2

25 [( x - 8)2 + 36] = 9( x2 + 36) = | z1 + z12 - z22 | 2 + | z1 - z12 - z22 | 2 + 2 | z12 - (z12 - z22 )|

16 x2 - 400 x + 2176 = 0 = 2(| z1 |2 + | z12 - z22 |) + 2 | z2 |2


x2 - 25 x + 136 = 0 = 2(| z1 |2 + | z2 |2 ) + 2 | z12 - z22 |
( x - 8)( x - 17) = 0 = | z1 + z2 |2 + | z1 - z2 |2 + 2 | z1 + z2 || z1 - z2 |
x = 8, 17 = (| z1 + z2 | + | z1 - z2 |)2
Hence
Therefore
z = 8 + 3i, 17 + 3 i
Answers: (B), (D) | z1 + z12 - z22 | + | z1 - z12 - z22 | = | z1 + z2 | + | z1 - z2 |

Hence (B) is true. Also


2. If z1 and z2 are complex numbers such that | z1 + z2 |2 =
| z1 | 2 + | z2 | 2 , then z1 + z2 z +z
+ z1z2 + 1 2 - z1z2
(A) z1z2 is pure imaginary (B) z1z2 + z1z2 = 0 2 2
æz ö p æz ö p 1 1
(C) Arg ç 1 ÷ = ± (D) Arg ç 1 ÷ = ± = | z1 + z2 | 2 + | z1 - z2 | 2
è 2ø
z 2 z
è 2ø 2 2 2
Solution: 1
= [2 | z1 | 2 +2 | z2 | 2 ]
2
| z1 + z2 |2 = | z1 |2 + | z2 |2
= | z1 | + | z2 |
(z1 + z2 )(z1 + z2 ) = z1z1 + z2 z2
z1z2 + z2 z1 = 0 Therefore (C) is true.
z1 æz ö Answers: (A), (B), (C), (D)
= - ç z1 ÷
z2 è 2ø
www.jeeneetbooks.in
Worked-Out Problems 149

4. If x and y are real numbers and Solution: Let

(1 + i) x - 2 i (2 - 3i) y + i [sin( x / 2) + cos( x / 2)] + i tan x


+ =i
3+ i 3-i 1 + 2i sin( x / 2)
then Then
(A) x = 3 (B) y = 1 (C) y = -1 (D) x = -3 [sin( x / 2) + cos( x / 2) + i tan x][1 - 2i sin( x / 2)]
z=
Solution: From the given equation, we get that 1 + 4 sin2 ( x / 2)
(3 - i)[(1 + i) x - 2 i] + (3 + i)[(2 - 3i) y + i] = 10i Suppose that z is real. Then Im(z) = 0. Therefore
Therefore x é æ xö æ xö ù
tan x - 2 sin êsin çè 2 ÷ø + cos çè 2 ÷ø ú = 0
4 x - 2 + i(2 x - 6) + 9 y - 1 + i (3 - 7 y) = 10i 2 ë û
4 x + 9 y - 3 + (2 x - 7 y - 13)i = 0 x æ x xö
sin x - 2 sin cos x ç sin + cos ÷ = 0
4 x + 9 y = 3 an nd 2 x - 7 y = 13 2 è 2 2ø
The two equations give x = 3 and y = -1. æ xö
sin x - sin x cos x - 2 sin2 ç ÷ cos x = 0
Answers: (A), (C) è 2ø
sin x(1 - cos x) - (1 - cos x)cos x = 0
5. The complex number(s) satisfying the equations
(1 - cos x)(sin x - cos x) = 0
z - 12 5 z- 4
= and = 1 is (are) cos x = 1 or tan x = 1
z - 8i 3 z-8
(A) 6 - 8i (B) 6 + 17i (C) 6 + 8i (D) 6 - 17i Therefore

Solution: Let z = x + iy p
x = 2 np , x = np + , n is an integer
4
z-4
=1 Since 0 £ x £ 2p , x = 0, p /4, 2p , 5p /4 .
z-8
Answers: (A), (B), (C), (D)
( x - 4)2 + y2 = ( x - 8)2 + y2
x=6 7. If z1, z2 and z3 represent the vertices A, B and C, respec-
Therefore tively, of a triangle (see figure), then the triangle ABC
is equilateral if and only if
z = 6 + iy
(A) z12 + z22 + z32 = z1z2 + z2 z3 + z3 z1
Now 1 1 1
(B) + + =0
z - 12 5 z1 - z2 z2 - z3 z3 - z1
=
z - 8i 3 (C) | z1 + z2 + z3 | = 3
9(36 + y ) = 25 [36 + ( y - 8) ]
2 2
(D) | z1z2 + z2 z3 + z3 z1 | = 3
( y - 8)( y - 17) = 0
y = 8, 17 A(z1)

Therefore
60°
z = 6 + 8i, 6 + 17i
Answers: (B), (C)
60°
6. If x is a real number such that 0 £ x £ 2p and B(z2) C(z3)

[sin( x / 2) + cos( x / 2)] + i tan x Solution: Suppose that triangle ABC is equilateral.
1 + 2i sin( x / 2) Then
is real, then the possible value(s) of x is (are) z3 - z1 p p z1 - z2 p p
= cos + i sin and = cos + i sin
(A) 0 (B) 2p (C) p /4 (D) 5p /4 z2 - z1 3 3 z3 - z2 3 3
www.jeeneetbooks.in
150 Chapter 3 Complex Numbers

Therefore Then
(z3 - z1 )(z3 - z2 ) = (z2 - z1 )(z1 - z2 ) a+b 1
=-
z - z3 z2 - z1z3 + z1z2 = z2 z1 - z22 - z12 + z1z2
2
3 ab g
z12 + z22 + z32 = z1z2 + z2 z3 + z3 z1 Therefore
Conversely, suppose that - g 2 = - ab (since a + b = - g )
z12 + z22 + z32 = z1z2 + z2 z3 + z3 z1 g 3 = abg
Then, Similarly
z1 (z1 - z2 ) + z2 (z2 - z3 ) + z3 (z3 - z1 ) = 0 b 3 = abg = a 3
Therefore This gives a3 = b 3 = g 3 and therefore | a | = | b | = | g |. That is,
z1 (z1 - z2 ) + z2 (z2 - z1 + z1 - z3 ) + z3 (z3 - z1 ) = 0
| z1 - z2 | = | z2 - z3 | = | z3 - z1 |
(z1 - z2 )2 - (z2 - z3 )(z3 - z1 ) = 0
Therefore DABC is equilateral.
That is Answers: (A) and (B)
(z1 - z2 )2 = (z2 - z3 )(z3 - z1 )
8. If c ³ 0, then the equation | z | - 2iz + 2c (1 + i) = 0
2
(z1 - z2 )3 = (z1 - z2 )(z2 - z3 )(z3 - z1 )
(z is complex) has
Similarly, (A) infinitely many solutions if c < 2 - 1
(z2 - z3 )3 = (z1 - z2 )(z2 - z3 )(z3 - z1 ) (B) has unique solution if c = 2 - 1
(C) finite number of solutions if c > 2 - 1
and (z3 - z1 )3 = (z1 - z2 )(z2 - z3 )(z3 - z1 )
(D) no solutions if c > 2 - 1
Therefore
Solution: Let z = x + iy. Then
(z1 - z2 )3 = (z2 - z3 )3 = (z3 - z1 )3
( x2 + y2 ) - 2i( x + iy) + 2c(1 + i) = 0
| z1 - z2 | = | z2 - z3 | = | z3 - z1 |
Therefore
Therefore AB = BC = CA. That is DABC is equilateral.
x2 + y2 + 2 y + i(2c - 2 x) + 2c = 0
Answer: (A)
x2 + y2 + 2 y + 2c = 0 (3.9)
We will prove that (B) is also correct. Suppose that
DABC is equilateral. Then and 2c - 2 x = 0 or x=c (3.10)
| z1 - z2 | = | z2 - z3 | = | z3 - z1 | = k (say) Substituting x = c in Eq. (3.9), we get that
Let a = z1 - z2 , b = z2 - z3 and g = z3 - z1. Then a + b + g = 0 c2 + y2 + 2 y + 2c = 0 (3.11)
and hence a + b + g = 0. That is Equation (3.11) has solutions if 4 - 4 (c2 + 2c) ³ 0 , that is
k2 k2 k2 1 - c2 - 2c ³ 0. Therefore
+ + = 0 (since aa = |a |2 = k2 )
a b g (c + 1)2 £ 2 or - 2 £ c + 1 £ 2
Therefore - 2 - 1£ c £ 2 -1
1 1 1 It is given that c ³ 0. Therefore 0 £ c £ 2 - 1.
+ + =0
a b g (i) If c < 2 - 1, then z = c + (- 1 ± 1 - 2c - c2 )i.
1 1 1 (ii) If c = 2 - 1, then z = ( 2 - 1) - i.
+ + =0
z1 - z2 z2 - z3 z3 - z1 (iii) If c > 2 - 1, the equation has no solutions.
Conversely, suppose that Answers: (B), (D)
1 1 1
+ + =0 9. If z1, z2, z3 are complex numbers such that
a b g z12 z2 z2
| z1 | = | z2 | = | z3 | = 1 and + 2 + 3 = -1
z2 z3 z3 z1 z1z2
www.jeeneetbooks.in
Worked-Out Problems 151

then the value of | z1 + z2 + z3 | can be 10. If


(A) 0 (B) 1 (C) 2 (D) 3/2
arg (z3 / 8 ) = (1/ 2) arg (z2 + zz1 / 2 )
Solution: Let z = z1 + z2 + z3. Then
then which of the following is (are) true?
1 1 1 z z + z2 z3 + z3 z1 (A) | z | = 1 (B) z is real
z = z1 + z2 + z3 = + + = 1 2
z1 z2 z3 z1z2 z3 (C) z is pure imaginary (D) z1/2 = 1
Therefore zb= z1z2 + z2z3 + z3z1, where b = z1z2z3. Hence Solution: The given relation is
2 2 2
z z z
+
1
+ 2
= - 1 Þ z13 + z23 + z33 = - z1z2 z3
3
2 arg (z3 / 8 ) = arg (z 2 + zz1 / 2 )
z2 z3 z3 z1 z1z2
æ z3 / 4 ö
Þ z13 + z23 + z33 - 3z1z2 z3 = - 4b Þ arg ç 2 =0
è z + zz1/ 2 ÷ø
Now
z3/ 4
(z1 + z2 + z3 ) [(z1 + z2 + z3 )2 - 3(z1z2 + z2 z3 + z3 z1 )] = - 4b Þ is purely real
z + zz1/ 2
2

That is z2 + zz1/ 2
Þ is purely real
z(z - 3zb) = - 4b
2
z3/ 4
Therefore Þ z 5 / 4 + zz - 1/ 4 is purely real
z - 3 | z | b + 4b = 0
3 2
Þ z5 / 4 + zz-1/ 4 = z5 / 4 + zz - 1/ 4
z = (3 | z | - 4)b
3 2
Þ ((z )5 / 4 + z(z ) - 1/ 4 ) = z5 / 4 + zz - 1/ 4
| z | = | 3 | z | - 4 | (since | b | = | z1z2 z3 | = 1)
3 2
(z ) 5 / 4 - z5 / 4
Þ (z )5 / 4 - z5 / 4 = zz - 1/ 4 - z(z ) - 1/ 4 =
Case 1: Suppose that 3 | z | ³ 4. Then 2
(zz )1/ 4
| z|3 = 3| z|2 - 4 é 1 ù
Þ [(z )5 / 4 - z5 / 4 ] ê1 - =0
| z| - 3| z| + 4 = 0
3 2
ë ( zz )1/ 4 úû
(| z| - 2)(| z |2 - | z | - 2) = 0 1
Þ z = z or =1
(| z | - 2)(| z | - 2)(| z | + 1) = 0 | z|2
| z| = 2 Þ z = z or | z | = 1

Case 2: Suppose that 3 | z |2 < 4 . Then Answers: (A) and (B)

| z|3 = | 3| z|2 - 4 | = 4 - 3| z|2 11. The vertices A and C of a square ABCD (see figure)
| z| + 3| z| - 4 = 0
3 2 are 2 + 3i and 3 - 2i, respectively. If z1 and z2 repre-
sent the other two vertices B and D respectively,
(| z | - 1)(| z | + 4 | z | + 4) = 0
2
then
(| z | - 1)(| z | + 2) 2 = 0 (A) z1 = 0 (B) z2 = 5 - i
| z| = 1 (C) z1 = 1 + i (D) z2 = 5 + i
Answers: (B) and (C)
B (z1) A (2+ 3i )
Note that, in case 2, z, z1, z2, and z3 lie on the circle with
radius 1 and center at the origin. Therefore, origin is the
circumcenter of the triangle with z1, z2 and z3 as verti-
ces. Hence, z1 + z2 + z3 (= z) represents the orthocenter. 90°
Thus z1, z2 and z3 form a right-angled triangle because
M
the distance between the orthocenter and circumcenter
is equal to the radius of the circumcircle. Hence two of
z1, z2, and z3 are the reflections of each other, through the
center of the circle. Since z1, z2, z3 satisfy the condition
å z12 / z2 z3 = -1, it implies that two are real and the third is C (3-2i ) D
the reflection of them in the origin.
www.jeeneetbooks.in
152 Chapter 3 Complex Numbers

Solution: Let M be the center of the square. Then (C) D is the reflection of the orthocenter in the
side BC
5 i
M= + (D) If H is the orthocenter, then HD is perpendicular
2 2
to the side BD
Let z1 denote the point B. Then ÐCMD = 90°. Therefore
A(z1)
z1 - (5 + i)/ 2
=i
2 + 3i - (5 + i)/ 2
5+i æ 5 + iö
z1 = + i ç 2 + 3i - ÷
2 è 2 ø 90°
C(z3)
5+i æ - 1 + 5i ö B(z2)
= + iç
2 è 2 ÷ø
5+i-i-5
= =0
2
Therefore, D(z)

D = 3 - 2i + 2 + 3i - 0 Solution: AD is perpendicular to BC and therefore


z2 = 5 + i
Answers: (A) and (D) æ z - z1 ö p
arg ç =±
è z3 - z2 ÷ø 2
12. For any complex number z = x + iy, define
This implies that (z - z1)/(z3 - z2) is pure imaginary.
(z) = | x | + | y | Therefore
If z1 and z2 are any complex numbers, then
z - z1 æ z - z1 ö
(A) (z1 + z2 ) £ (z1 ) + (z2 ) = -ç
z3 - z2 è z3 - z2 ÷ø
(B) (z1 + z2 ) = (z1 ) + (z2 )
(1/z) - (1/z1 ) æ z - z1 ö
(C) (z1 + z2 ) ³ (z1 ) + (z2 ) = -ç
(1/z3 ) - (1/z2 ) è z3 - z2 ÷ø
(D) |(z1 + z2 )| £ |(z1 )| + |(z2 )|
æ z1 - z ö æ z2 z3 ö æ z - z1 ö
Solution: Let z1 = x1 + iy1 and z2 = x2 + iy2 . Then z1 + z2 = çè z - z ÷ø çè zz ÷ø = - çè z - z ÷ø
2 3 1 3 2
( x1 + x2 ) + i( y1 + y2 ). Now
z2 z3 -z z
= - 1 or z = 2 3
(z1 + z2 ) = | x1 + x2 | + | y1 + y2 | zz1 z1
£ | x1 | + | x2 | + | y1 | + | y2 |
This implies (A) is correct.
= (z1 ) + (z2 ) Also, since the orthocenter H is z1 + z2 + z3, we have
|(z1 + z2 )| = || x1 + x2 | + | y1 + y2 || BH = | z1 + z2 + z3 - z2 | = | z1 + z3 |
= | x1 + x2 | + | y1 + y2 |
£ | x1 | + | x2 | + | y1 | + | y2 | z2 z3 | z2 |
and BD = z2 + = | z1 + z3 | = | z1 + z3 |
z1 | z1 |
= |(z1 )| + |(z2 )|
Answers: (A) and (D) (since | z1 | = 1 = | z2 |)
Therefore, B is equidistant from H and D. Similarly, C
13. Let z1, z2 and z3 be complex numbers representing is equidistant from H and D. This gives that BC is the
three points A, B and C, respectively, on the unit perpendicular bisector of HD and so H, D are reflections
circle | z | = 1 (see figure). Let the altitude through A of each other through the side BC.
meet the circle in D(z). Then Answers: (A) and (C)
- z2 z3 - z1
(A) z = (B) z =
z1 z2 z3
www.jeeneetbooks.in
Worked-Out Problems 153

14. Let a, b be real numbers such that | b | £ 2a2 . Let æ 5p 5p ö


III. z2 - z1 = (z3 - z1 ) ç cos + i sin ÷
è 3 3ø
X = {z : | z - a | = 2a2 + b }
æ 2p 2p ö
Y = {z : | z + a | = 2a2 - b } IV. z1 + z2 ç cos + i sin ÷
è 3 3ø
S = {z : | z2 - a2 | = | 2az + b |}
æ 4p 4p ö
+ z3 ç cos + i sin ÷ = 0
Then which of the following is (are) true? è 3 3ø
(A) X is a subset of S (B) Y is a subset of S Then which one is correct:
(C) S = X È Y (D) S = X Ç Y (A) I Þ II (B) II Þ III
Solution: Let z Î S. Therefore | z2 - a2 | = | 2az + b |. This (C) III Þ IV (D) IV Þ I
relation is equivalent to
Solution:
| z2 - a2 |2 = | 2az + b|2 
I. Suppose D ABC is equilateral (see figure). Rotating AB
(z2 - a2 )(z 2 - a2 ) = (2az + b)(2az + b) about A through the angle p/3 in anticlocksense, we get
| z|4 - a2 (z2 + z 2 ) + a4 = 4a2 | z|2 + 2ab (z + z ) + b2 z3 - z1 p p
= cos + i sin
| z|4 - a2 [(z + z )2 - 2 | z|2 ] + a4 = 4a2 | z|2 + 2ab (z + z ) + b2 z2 - z1 3 3
| z|4 - 2a2 | z|2 + a4 = a2 (z + z )2 + 2ab (z + z ) + b2
Therefore, I Þ II. This implies (A) is true.
Hence, (| z | 2 - a2 ) 2 = [a(z + z ) + b] 2 . Therefore C(z3)
| z|2 - a2 = ± [a(z + z ) + b]
p
Therefore 3

| z|2 - a2 - a(z + z ) - b = 0 p p
3 3
or | z|2 - a2 + a (z + z ) + b = 0
A(z1) B(z2)
This is equivalent to II. Assume that
(z - a)(z - a) = 2a + b 2
æ p pö
z3 - z1 = (z2 - z1 ) ç cos + i sin ÷
or (z + a)(z + a) = 2a - b 2
(3.12) è 3 3ø

Hence, Therefore
| z - a | = 2a + b or | z + a | = 2a - b
2 2 z3 - z1 p p
= cos + i sin
z2 - z1 3 3
Since | b | £ 2a2 , both 2a2 + b and 2a2 - b are non-negative.
From Eq. (3.12), if we retrace the steps backwards, then p
| z3 - z1 | = | z2 - z1 | and ÐBAC =
we get z satisfying the relation 3
| z2 - a2 | = | 2az + b | This implies DABC is equilateral. Therefore, II Þ I.

Therefore Now rotate AC about A through angle 5p /3 in anti-
clock sense so that
S = X ÈY
Answers: (A), (B), (C) æ 5p 5p ö
z2 - z1 = (z3 - z1 ) ç cos + i sin ÷
è 3 3ø
15. Let z1, z2, z3 be the complex numbers representing
the vertices A, B, C of a triangle described in coun- This means II Û III.
terclocksense. Consider the following statements. Similarly we can see that III Û IV and IV Û I.
I. D ABC is equilateral Answers: (A), (B), (C), (D)
æ p pö
II. z3 - z1 = (z2 - z1 ) ç cos + i sin ÷
è 3 3ø
www.jeeneetbooks.in
154 Chapter 3 Complex Numbers

Matrix-Match Type Questions


1. Match the items in Column I with those in Column II z4 - 4z3 + 7z2 - 6z + 3 = z2 - 2z + 3
= (z - 1)2 + 2 = i2 + 2 = 1
Column I Column II
4(z4 - 4z3 + 7z2 - 6z + 3) = 4
(A) If z = x + iy, z1/ 3 = a - ib and (p) 10
x y Answer: (D) Æ (s)
- = l (a2 - b2 ), then l is
a b (q) 14
2. Match the items in Column I with those in Column II.
(B) If | z - i | < 1, then the value of In the following, w ¹ 1 is a cube root of unity.
| z + 12 - 6i | is less than (r) 1
(C) If | z1 | = 1 and | z2 | = 2, then (s) 4 Column I Column II
| z1 + z2 |2 + | z1 - z2 |2 is equal to
(A) The value of the determinant (p) 3w (1 - w )
(D) If z = 1 + i, then (t) 5
4 (z4 - 4z3 + 7z2 - 6z + 3) is equal to 1 1 1
1 - 1 - w2 w2 is
Solution: 1 w2 w4 (q) 3w(w - 1)
(A) x + iy = z = (a - ib)3 = a3 - 3a2 bi + 3a(ib)2 - i3 b3
(B) The value of 4 + 5w2002 + 3w2009
= (a3 - 3ab2 ) + i(b3 - 3a2 b) is
(r) -i 3
Comparing the real parts we get (C) The value of the determinant
x = a3 - 3ab2 = a(a2 - 3b2 ) 1 1 + i + w2 w2
x 1- i -1 w2 - 1 is (s) i 3
= a2 - 3b2
a -i -i + w 2 + 1 -1
Comparing the imaginary parts we get (D) w2 n + wn + 1 (n is a positive integer (t) 0
y = b - 3a b = b(b - 3a )
3 2 2 2 and not a multiple of 3) is
y
= b2 - 3a2 Solution:
b
(A) 1 1 1 3 1 + w + w2 1 + w2 + w
Therefore 1 - 1 - w2 w2 = 1 w w2
x y 1 w2 w4
1 w2 w
- = 4(a2 - b2 )
a b
3 0 0
l=4
= 1 w w2
Answer: (A) Æ (s)
1 w2 w
(B) | z - 12 - 6i | = |(z - i) + (12 - 5i)|
= 3(w2 - w4 ) = 3w(w - 1)
£ | z - i | + | 12 - 5i | < 1 + 13 = 14 Answer: (A) Æ (q)
Answer: (B) Æ (q) (B) 4 + 5w 2002
+ 3w
2009
= 4 + 5w + 3w 2
(∵ w3 = 1)
(C) | z1 + z2 |2 + | z1 - z2 |2 = 2(| z1 |2 + | z2 |2 ) = 2(1 + 4) = 10 = 1 + 2w + 3 (1 + w + w2 )
Answer: (C) Æ (p) = 1 + 2w
(D) If z = 1 + i, then Since
(z - 1) = i
4 4

-1 i 3
Therefore w= ±
2 2
z4 - 4z3 + 6z2 - 4z + 1 = 1 we get
(z - 4z + 7z - 6z + 3) - z + 2z - 2 = 1
4 3 2 2

æ -1 i 3 ö æ -1 i 3 ö
4 + 5w2002 + 3w2009 = 1 + 2 ç + or 1 + 2 ç -
è 2 2 ÷ø è 2 2 ÷ø
www.jeeneetbooks.in
Worked-Out Problems 155

= 1 - 1 + i 3 or 1 - 1 - i 3 Solution: We have w3 = 1 and 1 + w + w2 = 0.


=±i 3 (A) We have
Answers: (B) Æ (r), (s) 1 1
(1 - w)(1 - w2 )(1 - w4 )(1 - w8 ) = (1 - w)2 (1 - w2 )2
3 3
(C) 1 1+ i + w 2
w 2
1 -w + i w 2

1- i -1 w2 - 1 = 1 - i -1 w2 - 1 1
= [(1 - w)(1 - w2 )]2
3
-i -i + w - 1 -1 -i -i + w -1 -1
1
1 -w + i -1 = (1 - w - w2 + w3 )2
3
= 0 0 0 =0 1
-i -i + w - 1 -1 = (1 + 1 + 1)2 = 3
3
Answer: (C) Æ (t) Answer: (A) Æ (t)
(B) We have
(D) Let n > 0 and n ¹ 3m for all integers m. Then
n = 3m + 1 or 3m + 2 w(1 + w - w2 )7 = w(-w2 - w2 )7
n = 3m + 1 Þ w2 n + wn + 1 = w6 m + 2 + w3 n + 1 + 1 = w[2(-w2 )]7
= w2 + w + 1 = 0
-27 w15 = -128
6 m+ 4 3m+ 2
n = 3m + 2 Þ w + w + 1 = w
2n n
+w +1 Answer: (B) Æ (p)
=w + w + 1= 02 (C) We have
Answer: (D) Æ (t) (1 + w2 )n = (1 + w4 )n
3. Match the items in Column I with those in Column II. (1 + w2 )n = (1 + w)n
w ¹ 1 is a cube root of unity.
(-w)n = (-w2 )n

Column I Column II wn = w2 n
(A) The value of (p) -128 The least such positive n is 3.
1 Answer: (C) Æ (t)
(1 - w)(1 - w2 )(1 - w4 )(1 - w8 ) is (q) 6
3 (D) We have
(B) w (1 + w - w2 )7 is equal to (r) 0
1 1 1 1 1+ w - 2 - w
+ - = +
(C) The least positive integer n such that 1 + 2w 2 + w 1 + w 1 + 2w (2 + w)(1 + w)
(1 + w2 )n = (1 + w4 )n is (s) 128
1 1
1 1 1 = -
(D) + - is equal to (t) 3 1 + 2w 2 + 3w + w2
1 + 2w 2 + w 1 + w 1 1
= - =0
1 + 2w 1 + 2w
Answer: (D) Æ (r)

Comprehension-Type Questions
1. Passage: A complex number z is pure real if and only (A) 8 x - 17 y = 16 (B) 8 x + 17 y = 16
if z = z and is pure imaginary if and only if z = - z.
(C) 17 x - 8 y = 16 (D) 17 x - 8 y = - 16
Answer the following questions:
(ii) If
(i) If x and y are real numbers and the complex
number 3 + 2i sin q æ pö
z= çè 0 < q £ ÷ø
1 - 2i sin q 2
(2 + i) x - i (1 - i) y + 2i
+
4+i 4i is pure imaginary, then q is equal to
is pure real, the relation between x and y is (A) p/4 (B) p/6 (C) p/3 (D) p/12
www.jeeneetbooks.in
156 Chapter 3 Complex Numbers

(iii) If z1 and z2 are complex numbers such that 3


Û sinq = ±
z1 - z2 2
=1 p æ pö
z1 + z2 Ûq = çè since 0 < q £ ÷ø
3 2
then Answer: (C)
(A) z1/z2 is pure real
(iii) | z1 - z2 | = | z1 + z2 |
(B) z1/z2 is pure imaginary
(C) z1 is pure real Þ (z1 - z2 )(z1 - z2 ) = (z1 + z2 )(z1 + z2 )
(D) z1 and z2 are pure imaginary Þ z1z2 = - z1z2
Solution: z1 z æz ö
Þ = - 1 = -ç 1 ÷
(i) Let z2 z2 è z2 ø
(2 + i) x - i (1 - i) y + 2i z1
z= + Þ is pure imaginary
4+i 4i z2
2 x + ( x - 1)i y + (2 - y)i
= + Answer: (B)
4+i 4i
(2 x + ( x - 1))i)(4 - i) -iy + (2 - y) 2. Passage: Consider z = a + ib and z = a - ib, where a and
= +
17 4 b are real numbers, are conjugates of each other. Answer
8 x + x - 1 + i(4 x - 4 - 2 x) (2 - y) - iy the following three questions:
= +
17 4 (i) If the complex numbers -3 + i(x2y) and x2 + y + 4i,
9 x - 1 + i(2 x - 4) 2 - y - iy where x and y are real, are conjugate to each
= +
17 4 other, then the number of ordered pairs (x, y) is
(A) 1 (B) 2 (C) 3 (D) 4
Now
(ii) Let z = x 2
- 7 x - 9 yi such that z = y i + 20i - 12,
2

z is real Û z = z then the number of ordered pairs (x, y) is


Û Im z = 0 (A) 1 (B) 2 (C) 3 (D) 4
2x - 4 y (iii) The number of real values of x such that sin x +
Û - =0 i cos 2x and cos x - i sin 2x are conjugate to each
17 4
other is
Û 8 x - 16 = 17 y (A) 1 (B) 2 (C) >2 (D) 0
Û 8 x - 17 y = 16 Solution:
Answer: (A) (i) -3 + ix2 y = x2 + y - 4i implies
3 + 2i sin q x2 + y = -3 and x2 y = - 4 (3.13)
(ii) z =
1 - 2i sin q
Therefore
(3 + 2i sin q )(1 + 2i sin q ) 4
= x2 - = -3
1 + 4 sin2 q x2
(3 - 4 sin2 q ) + i(8 sin q ) x4 + 3 x2 - 4 = 0
=
1 + 4 sin2 q
( x2 + 4)( x2 - 1) = 0
Now,
This gives x2 = 1 (since x2 ¹ -4). Therefore x = ±1 and
z is pure imaginary Û z = - z y = -4. Hence the ordered pairs are (1, -4) and (-1, -4).
Û Re(z) = 0 Answer: (B)
3 - 4 sin2 q (ii) We have
Û =0
1 + 4 sin2 q z = x2 - 7 x - 9 yi
3 Þ z = x2 - 7 x + 9 yi
Û sin2 q =
4
Þ x2 - 7 x + 9 yi = y2 i + 20i - 12
www.jeeneetbooks.in
Summary 157

This implies that (iii) sin x + i cos 2 x = cos x + i sin 2 x


x2 - 7 x = - 12 (3.14) Þ sin x = cos x and cos 2 x = sin 2 x
and 9 y = y + 20
2
(3.15) Þ 2 cos2x - 1 = cos 2 x = sin 2 x = 2 sin x cos x = 2 cos2x
Solving Eq. (3.14) we get
Þ - 1 = 0, which is absurd
x = 3, 4
Therefore, there are no such real numbers x.
Solving Eq. (3.15) we get
Answer: (D)
y2 - 9 y + 20 = 0 Þ y = 4, 5
Therefore, the required ordered pairs are (3, 4), (3, 5),
(4, 4) and (4, 5).
Answer: (D)

Assertion–Reasoning Type Questions


In the following set of questions, a Statement I is given Let z = x + iy. Then
and a corresponding Statement II is given just below it.
æ z - zö p
Mark the correct answer as: arg ç 1 = Þ ( x - 9)( x - 3) + ( y - 5)2 = 6 y - 30
è z2 - z ÷ø 4
(A) Both I and II are true and II is a correct reason for I
(B) Both I and II are true and II is not a correct reason for I Þ x2 + y2 - 12 x - 16 y + 82 = 0
(C) I is true, but II is false Now
(D) I is false, but II is true
| z - 6 - 8i | 2 = ( x - 6) 2 + ( y - 8)2
1. Statement I: If z1 = 9 + 5i, z2 = 3 + 5i and arg [(z−z1)/
= x2 + y2 - 12 x - 16 y + 100
(z − z2)] = p / 4, then the values of | z - 6 - 8i | is 3 2 .
Statement II: In a circle, the angle made by a chord at = ( x2 + y2 - 12 x - 16 y + 82) + 18
the center is double the angle subtended by the same
= 0 + 18
chord on the circumference.
Solution: Let z be a point such that Therefore

æ z - zö p z - 6 - 8i = 3 2
arg ç 1 =
è z2 - z ÷ø 4 Answer: (B)

SUMMARY
Complex Number
3.1 Complex number: Any ordered pair (a, b) where a (2) If z = (a, b), then -z = (- a, - b).
and b are real numbers is called a complex number
(3) z1 = (a, b), z2 = (c, d), then z1 - z2 = z1 + (-z2) =
and the set of all complex numbers is denoted by 
(a - c, b - d).
which is  ´ .
(4) If z = (a, b), and l is real, then l z = (l a, l b).
3.2 Real number as a complex number: If a is a real
(5) Product: If z1 = (a, b), and z2 = (c, d), then z1 z2 =
number, we write a for the ordered pair (a, 0) so that
(ac - bd, ad + bc).
every real numbered is considered to be a complex
number.
3.4 Zero complex number and unit complex number:
(0, 0) is called zero complex number and is denoted
3.3 Algebraic operations:
by 0. (1, 0) is called unit complex number and is
(1) Addition: If z1 = (a, b) and z2 = (c, d), then denoted by 1.
z1 + z2 = (a + c, b + d)
www.jeeneetbooks.in
158 Chapter 3 Complex Numbers

3.5 Complex number i: The complex number (0, 1) is 3.11 Pure real and pure imaginary: A complex number z
such that (0, 1) (0, 1) = (-1, 0) = -1. (0, 1) is denoted is called pure real if Im(z) = 0 and pure imaginary if
by i with the convention that i2 = -1 or i = -1. If n is Re(z) = 0.
any positive integer, then in + i n+1 + i n+2 + i n+3 = 0.
3.12 Conjugate: For z = a + ib, the complex number
3.6 Quotient of complex numbers: Let z1 = (a, b) and z = a - ib is called conjugate of z.
z2 = (c, d) ≠ (0, 0). Then the unique complex number
z such that z · z2 = z1 is called quotient of z1 and z2 3.13 Properties of z:
and is denotes by z1/z2. In particular, if z = (a, b) ≠ (1) (z ) = z
(0, 0), then there exists z¢ = (c, d) such that zz¢ = z+z z-z
(1, 0) = 1 and (2) = Re(z) and = Im(z)
2 2i
a -b (3) If z = a + ib, there zz = a + b
2 2
c= , d= 2
a2 + b2 a + b2 (4) z is pure real Û z = z
(5) z is pure imaginary Û z = - z
3.7 Representation of (a, b) as a + ib:
(6) (z1 ± z2 ) = z1 ± z2
z = (a, b) = (a, 0) + (0, 1) (b, 0) = a + ib
(7) z1z2 = z1 z2
3.8 Real and imaginary parts: If z = a + ib (a, b are real),
æz ö z
then a is called real part of z denoted by Re(z) and (8) If z2 ≠ 0, then ç 1 ÷ = 1
b is called imaginary part denoted by Im(z). è z2 ø z2
(9) z1z2 + z1z2 = 2 Re(z1z2 ) = 2 Re(z1z2 )
3.9 Usual operations:
(10) z1z2 - z1z2 = 2i Im(z1z2 ) = - 2i Im(z1z2 )
(1) z1 = a + ib, z2 = c + id, then z1 + z2 = (a + c) + i(b + d)
and z1 - z2 = (a - c) + i(b - d) 3.14 Modulus and its properties: If z = a + ib, then
(2) z1 z2 = (ac - bd) + i(ad + bc) | z | = a2 + b2 and | z | = | z | = | -z|. Let z1 and z2 be
(3) If z2 ≠ 0, then complex numbers. Then
z1 ac + bd
= 2
(bc - ad )
+i 2 (1) | z1z2 | = | z1 || z2 |
z2 c +d 2
c + d2 z1 | z1 |
(2) = when z2 ¹ 0
z2 | z2 |
(4) If z = x + iy ¹ 0, then
(3) | z | 2 = zz (very useful)
1 x æ -y ö (4) | z1 + z2 | £ | z1 | + | z2 | (equality holds if and only if
= + iç 2
z x2 + y2 è x + y2 ÷ø z1 and z2 are collinear with origin and lie on the
same side of the origin)
3.10 Cube roots of unity: (5) | z1 - z2 | ³ | z1 | - | z2 | (equality holds if and only
(1) Roots of the equation z3 = 1 are called cube if z1, z2 are collinear with origin and lie on the
roots of unity and they are same side of origin)
(6) | zn | = | z |n for all positive integers.
-1 + i 3 -1 - i 3
1, and (7) | z1 + z2 | 2 = | z1 | 2 + | z2 | 2 + (z1z2 + z1z2 )
2 2
(8) | z1 - z2 | = | z1 | + | z | - (z1z2 + z1z2 )
2 2 2 2
2
(2) If w ≠ 1 is a cube root of unity, then w is also cube
root of unity and hence 1, w and w2 are cube roots
2
( 2
) (parallelogram
(9) | z1 + z2 | + | z1 - z2 | = 2 | z1 | + | z2 |
2 2

of unity having the relation 1 + w + w2 = 0. law)


(3) If w is a non-real cube root of unity and n is any (10) | z1 | + | z2 | is the greatest possible value of | z1 ± z2 |
positive integer, then
and || z1 | - | z2 || is the least possible value of | z1 ± z2 |
ì3 if n is a multiple of 3
1 + w n + w 2n = í 3.15 Unimodular complex number: If | z | = 1, then z is
î0 otherwise called unimodular complex number. If z ≠ 0, then
z/| z | is always unimodular.
www.jeeneetbooks.in
Summary 159

3.16 Geometric interpretation (Argand’s plane): sense, then Arg z is positive, otherwise Arg z < 0.
Consider a plane and introduce coordinate system Further, let z1 and z2 be complex numbers.
in the plane. Now, we can view any complex
number z = a + ib as the point (a, b) in the plane y
and any point (a, b) in the plane as the complex
number a + ib. Also if z = a + ib and P is the point
 P(z)
representing z, then one can view the vector OP
and conversely if P is a point with coordinates Arg z

 the complex number z = a + ib


(a, b), we can consider x
O
and the vector OP representing it. Hence there is Arg z < 0
a one-to-one correspondence between the set of
complex numbers, the points in the Argand’s plane
P(z)
and the vectors in the Argand’s plane.

3.17 Identification of complex number: We identify


complex number z = a + ib with the point P(a, b) (1) z1 = z2 Û | z1 | = | z2 | and Arg z1 = Arg z2.

and with the vector OP in the Argand’s plane (2) arg (z1z2) = Arg z1 + Arg z2 + 2np, n Î.
where O is the origin.
(3) arg (z1/z2) = Arg z1 - Arg z2 + 2np, n Î.
3.18 More about Argand’s plane: Let z, z1, z2 be complex (4) arg (1 / z) = - Arg + 2 np, n Î  for any complex
numbers and A, P and Q represent them in the number z ¹ 0.
Argand’s plane. Then
(1) z is presented by the reflection of the point A 3.22 Geometrical meaning of arg (z1/z2): Let P and Q
in the real axis (i.e., x-axis). represent z1 and z2 in the Argands plane and ‘O’
is the origin. Thus arg (z1/z2) is the angle of rota-

(2) -z is represented by the reflection of A through
tion of OQ about origin to fall on the vector OP .
the origin that is (-a, -b) represents -z.
arg (z1/z2) is positive, if the rotation is anticlock
(3) z1 + z2 is the fourth vertex of the parallelogram sense otherwise it is negative.
 
constructed on OP and OQ as adjacent sides.
(4) z1 - z2 is the fourth vertex of the parallelogram 3.23 Directly similar triangles: DABC and DA¢B¢C¢ are
   directly similar, if A = A¢, B = B ¢, C = C ¢ and the
constructed with OP and -OQ (i.e., QO) as adja-
cent sides. sides about equal angles are proportional. That is,
indirectly similar triangles, the angles at the vertices
3.19 Modulus and argument form (Trigonometric or
in the prescribed order are equal.
Polar form): Every complex number z can be Let z1, z2, z3 and z1¢, z2¢ and z3¢ represent the ver-
expressed as r(cos q + i sin q) where tices of two triangles. Then they are directly similar
r = | z | and if and only if
q is the angle made by the vector OP with real
axis and P represents z in the Argand plane. This z1 z1¢ 1
q is called argument of z and is denotes by arg z. z2 z2¢ 1 = 0
If q is an argument z, then q + 2np is also an argu- z3 z3¢ 1
ment of z.

3.20 Principal value of arg z (denoted by Arg z): If z is QUICK LOOK


a complex number, then there exists, unique q such
that -p < q ≤ p and z = | z | (cos q + i sin q). This q is Directly similar triangles are similar also.
called the principal value of arg z and is denoted
by Arg z.
3.24 Most useful formula: Let A, B and C be three
3.21 Geometrical meaning of Arg z and computing Arg z : points representing the complex numbers z1, z2
Arg z is the shortest turn taken by the position
 x-axis and z3 respectively and the points are described in
about the origin to fall on the vector OP where P counter clock sense and BAC = a.
represents z. If the shortest turn is anticlockwise
www.jeeneetbooks.in
160 Chapter 3 Complex Numbers

A(z1) 3.28 Line joining two points in the complex plane: The
equation of the line joining the points A(z1) and
a B(z2) is

C(z3) z z 1
B(z2)
z1 z1 1 =0
z2 z2 1
Then,
The complex number z1 - z2 / z1 - z2 is called com-
z3 - z1 æ CA ö plex slope of the line AB.
=ç ÷ (cos a + i sin a )
z2 - z1 è BA ø
3.29 General equation of a straight line: If l ¹ 0 is a
In particular the segments AB and AC are at right complex number and m is a real number then the
angles Û arg (z3 - z1 /z2 - z1 ) = ± p / 2 and in such a equation lz + lz + m = 0 represents a straight line in
case z3 - z1 /z2 - z1 is pure imaginary. the complex plane. The real slope of this line is

3.25 Equilateral triangles: Let A, B and C be the vertices æ l + lö


of a triangle represented by z1, z2 and z3 respectively. çè l - l ÷ø i
The following hold:
(1) D ABC is equilateral Û z12 + z22 + z32 = z1z2 + 3.30 Condition for parallel and perpendicular lines:
z2z3 +z3z1. Let l1z + l1z + m1 = 0, l2 z + l2 z + m2 = 0 where m1, m2
are real be two straight lines. Then
(2) D ABC is equilateral
(1) The two lines are parallel Û l1l 2 = l 1 l2 .
1 1 1
Û + + =0 (2) The two lines are perpendicular to each other if
z1 - z2 z2 - z3 z3 - z1
and only if l1l 2 + l 1 l2 = 0.
3.26 Orthocentre with reference to circumcentre: Let
A, B, C be the vertices of a triangle whose circum- 3.31 Equation of the perpendicular bisector of the segment
centre is at the origin. If z1, z2, z3 represent A, B, joining the points A(z1) and B(z2) is | z - z1 | = | z - z2 |.
C respectively, then the orthocentre of DABC is Equivalently
represented by z1 + z2 + z3.
(z1 - z2 )z + (z1 - z2 )z + z2 z2 - z1z1 = 0
3.27 Angle between two segments: A(z1), B(z2), C(z3)
and D(z4). There AB is inclined at an angle of 3.32 The points representing z1 and z2 in theArgand’s plane
arg (z4 - z3 / z2 - z1 ) to CD. The lines are at right angles are images of each other in the line lz + l z + m = 0
if and only if (m is real) if and only if lz1 + lz2 + m = 0.

æz -z ö p 3.33 Distance of a line from a point: The perpendicular


arg ç 4 3 ÷ = ±
è z2 - z1 ø 2 distance drawn from a point A(z0) onto a straight
line lz + lz + m = 0 (m is real) is
The points A, B, C are collinear
lz0 + lz0 + m
æz -z ö
Û arg ç 2 1 ÷ = 0 or p 2 |l |
èz -z ø
3 1

3.34 Let A(z1), B(z2) and C(z3) be the vertices of a triangle


C(z3)
whose circum centre is the origin. If the altitude drawn
A(z1)
from A onto the side BC, meets the circumcircle of
DABC in D, then D is represented by the complex
number -z2 z3 / z1. Also note that D is the reflection of
the orthocenter of DABC in the side BC.

Circle
3.35 Circle: Equation of the circle with centre at the
D(z4)
B(z2) point z0 and radius r (> 0) is | z - z0 | = r.
www.jeeneetbooks.in
Exercises 161

QUICK LOOK 3.36 General equation of a circle in the complex


plane: If a is complex number and b is real, then
1. | z - z0 | = r Û | z - z0 | = r
2 2
the equation zz + az + az + b = 0 represents circle
Û (z - z0 )(z - z0 ) = r 2 with centre at the point -a and radius aa - b . The
Û zz - z0 z - z0 z + z0 z0 - r 2 = 0 circle is real circle or point circle or imaginary circle
according as aa - b is positive or zero or negative.
2. If z0 = 0, then the equation of the circle with centre
at origin and radius r is | z | = r.

EXERCISES
Single Correct Choice Type Questions
1. If w ¹ 1 is a cube root of unity, then the value of the (A) only two roots (B) only four roots
expression (1 - w + w2) (1 - w2 + w4) (1 - w4 + w8) (C) no roots (D) infinite number of roots
upto 2n factors is
(A) 2n (B) 22n (C) 0 (D) 1 8. In D ABC, origin is the circumcenter, H is the ortho-
center and D is the midpoint of the side BC. If P is
å k =1[sin(2kp /11) - i cos(2kp /11)] is
10
2. The value of any point on the circumcircle other than the vertices
and T is the midpoint of PH, then the angle between
(A) 1 (B) –1 (C) i (D) –i AP and DT is
(A) p/4 (B) p/3 (C) p/6 (D) p/2
3. If z is a complex number and n is a positive integer
satisfying the equation (1 + z)n = (1 - z)n, then z lies on 9. The number of solutions of the equation z(z - 2i) =
(A) the line x = 0 (B) the line x = 1/2 2(2 + i) is
(C) the line y = 0 (D) the line x = -1/2 (A) 4 (B) 3 (C) 2 (D) 0

4. Let a and b be complex numbers representing the 10. If 0 < a, b < 1, z1 = a + i and z2 = 1 + ib and if the origin,
points A and B, respectively, in the complex plane. If z1 and z2 represent the vertices of an equilateral
(a/b) + (b/a) = 1 and O is the origin, then DOAB is triangle, then
(A) right angled (B) right-angled isosceles
3
(B) obtuse angled (D) equilateral (A) a = 3 - 1, b = (B) a = 2 - 3 = b
2
1 3 3 1
5. The complex numbers z1 , z2 , z3 and z4 represent the (C) a = ,b= (D) a = ,b=
vertices of a parallelogram in this order, if 2 4 4 2
(A) z1 + z2 = z3 + z4 (B) z1 + z3 = z2 + z4
11. If | z + 1| = | z - 1| and arg(z - 1)/(z + 1) = p / 4, then z
z+z z+z
(C) z1 + z4 = z2 + z3 (D) 1 3 = 1 4 is equal to
z2 z3 z1z4
(A) ( 2 + 1) + i (B) 1 + i 2
6. The area of the region in the complex plane satisfying (C) (1 ± 2 )i (D) ( 2 - 1)i
the inequality
é z-2 +5 ù 12. If z = (1 - t ) + i t + t + 2 , where t is a real parameter,
2

log cos(p / 6) ê ú<2 then z lies on the curve


ë4 z - 2 - 4û
is 2 2
æ 3ö 7 æ 3ö 7
(A) x2 + ç y + ÷ = (B) x2 - ç y + ÷ =
(A) 4p (B) 8p (C) 12p (D) 15p è 2ø 4 è 2ø 4
2 2
7. If z is a non-zero complex number, then the equation æ 3ö 7 æ 3ö 7
(C) y2 - ç x - ÷ = (D) y + ç x - ÷ =
2

z2 + | z | z + | z | 2 = 0 has è 2ø 4 è 2ø 4
www.jeeneetbooks.in
162 Chapter 3 Complex Numbers

13. z is a complex number and z ¹ i. If arg(z + i)/(z - i) = 21. The distance of the point z0 from the line az + az + b
p / 2, then z lies on the curve = 0 (b is real) is
(A) x + y = 1
2 2
(B) x - y = 1
2 2

(C) xy = 1 (D) y = x + 1 (A) az0 + az0 + b (B) a z0 + az0 + b


2a 2a
14. If z1 and z2 are complex nth roots of unity which sub-
(C) az0 + az0 + b (D) a z0 + az0 + b
tend right angle at the origin, then n must be of the form a a
(A) 4K + 1 (B) 4K + 2
(C) 4K + 3 (D) 4K 22. Let a, b be non-zero complex numbers and z1, z2 be
m the roots of the equation z2 + az + b = 0. If there exists
32
æ 10 æ æ 2 np ö æ 2 np ö ö ö
15. å (3m + 2) ç å ç sin ç ÷ø - i cos çè ÷ = l ³ 4 such that a2 = l b, then the points z1, z2 and the
m=1 è è è
n=111 11 ø ÷ø ÷ø origin
(A) 4(1 - i) (B) 12(1 + i) (A) form an equilateral triangle
(C) 12(1 - i) (D) 48(1 - i) (B) form a right-angled triangle, right angled at the
origin
16. The complex slope (see “Quick Look 6”) of the line (C) are collinear
joining the two points 1 - i and 2 - 5i is
(D) form an obtuse-angled triangle
1 - 4i 1 + 4i
(A) (B)
1 + 4i 1 - 4i 23. If (w - wz)/(1 - z) is purely real, where w = a + i b
1 + 2i 1 - 2i and z ¹ 1, then the set of values of z is
(C) (D) (A) {z : | z | = 1} (B) {z : z = z }
1 - 2i 1 + 2i
(C) {z : z ¹ 1} (D) {z : | z | = 1 and z ¹ 1}
17. If | z - i | £ 2 and z0 = 5 + 3i, then the maximum value
of | z0 + iz | is 24. If z1, z2 and z3 are distinct complex numbers such
that | z1 | = | z2 | = | z3 | = 1 and
(A) 7 (B) 7 (C) 5 (D) 9
z12 z2 z2
+ 2 + 3 = -1
18. If w ¹ 1 is a cube root of unity, x = a + b, y = aw + bw
2
z2 z3 z3 z1 z1z2
and z = aw2 + bw, then x3 + y3 + z3 is equal to
(A) 3ab (B) 0 (C) 3a3b3 (D) 3(a3 + b3) then the value of | z1 + z2 + z3 | can be
(A) 1/2 (B) 3 (C) 3/2 (D) 2
19. If a, b and c are integers not all simultaneously equal
and w ¹ 1 is a cube root of unity, then the minimum 25. If z1, z2 and z3 are the vertices of a right-angled isos-
value of | a + bw + cw2 | is celes triangle described in counter clock sense and
(A) 0 (B) 1 (C) 3/2 (D) 1/2 right angled at z3, then (z1 - z2)2 is equal to
(A) (z1 - z3 )(z3 - z2 ) (B) 2(z1 - z3 )(z3 - z2 )
20. The center and the radius of the zz + (2 - 3i)z + (2 + 3i) (C) 3(z1 - z3 )(z3 - z2 ) (D) 3(z3 - z1 )(z3 - z2 )
z + 4 = 0 are
(A) -2 - 3i, 3 (B) 2 - 3i, 3
(C) 2 + 3i , 3 (D) -2 + 3i, 3

Multiple Correct Choice Type Questions


In this section, each question has 4 choices (A), (B), (C), and 2. If z = -(1 + i), then
(D) for its answer, out of which one or more is/are correct.
p 5p
(A) arg z = (B) arg z =
1. Let z = 1 + cos(10p / 9) + i sin(10p / 9). Then 4 4
-3p
æ 2p ö 8p (C) Arg z = (D) | z | = 2
(A) | z | = 2 cos ç ÷ (B) arg z = 4
è 9 ø 9
3. If z1, z2 and z3 are the vertices of an equilateral triangle
æ 4p ö 5p described in counterclock sense and w ¹ 1 is a cube root
(C) | z | = 2 cos ç ÷ (D) arg z =
è 9 ø 9 of unity, then
www.jeeneetbooks.in
Exercises 163

(A) z1 - z3 = (z3 - z2 )w 7. ABCD is a rhombus. Its diagonals AC and BD inter-


(B) z1 + z2 w + z3w = 0
2 sect at M and satisfy BD = 2AC. If the points D and
M are represented by the complex numbers 1 + i and
1 1 1
(C) + + =0 2 – i, respectively, then A is represented by
z1 - z2 z2 - z3 z3 - z1
(A) 3 - i/2 (B) 3 + i/2 (C) 1 + 3i/2 (D) 1 - 3i/2
(D) z12 + z22 + z32 = z1z2 + z2 z3 + z3 z1
8. If the vertices of a square described in counter clock
4. Let z1 = 1 + i, z2 = -1 - i and z3 be complex numbers sense are represented by the complex numbers z1, z2,
such that z1, z2 and z3 form an equilateral triangle. z3 and z4, then
Then z3 is equal to
1 1
(A) 3 (1 + i) (B) 3 (1 - i) (A) z2 = (1 + i)z1 + (1 - i)z3
2 2
(C) 3 (i - 1) (D) 3 (-1 - i) 1 1
(B) z4 = (1 - i)z1 + (1 + i)z3
2 2
5. If cos a + cos b + cos g = 0 = sin a + sin b + sin g , then
1 1
(A) cos(2a ) + cos(2 b ) + cos(2g ) = 0 (C) z3 = (i - 1)z2 + (1 + i)z4
2 2
(B) sin(3a ) + sin(3b ) + sin(3g ) = 3 sin(a + b + g ) 1 1
(D) z1 = (i + 1)z2 + (1 - i)z4
(C) cos(3a ) + cos(3b ) + cos(3g ) = 3 cos(a + b + g ) 2 2
(D) sin(2a ) + sin(2 b ) + sin(2g ) = 0
9. Let p and q be positive integers having no positive
6. Let a > 0 and | z + (1/z)| = a (z ¹ 0 is a complex number). common divisors except unity. Let z1, z2, …, zq be the
Then the maximum and minimum values of | z | are q values of zp/q, where z is a fixed complex number. Then
the product z1z2 zq is equal to
a + a2 + 4 2a + a2 + 4 (A) zp, if q is odd (B) -zp, if q is even
(A) (B)
2 2 (C) zp, if q is even (D) -zp, if q is odd
a2 + 4 - a a2 + 4 - 2a
(C) (D)
2 2

Matrix-Match Type Questions


In each of the following questions, statements are given 1. In Column I equations are given and in Column II the
in two columns, which have to be matched. The state- number of ordered pairs (x, y) satisfying the equations
ments in Column I are labeled as (A), (B), (C) and (D), are given. Match them assuming that x and y are real
while those in Column II are labeled as (p), (q), (r), (s) numbers.
and (t). Any given statement in Column I can have cor-
rect matching with one or more statements in Column II. Column I Column II
The appropriate bubbles corresponding to the answers
to these questions have to be darkened as illustrated in (A) ( x + 2 y) + i(2 x - 3 y) = 5 - 4i (p) 1
the following example.
(q) 2
Example: If the correct matches are (A) ® (p), (s); (B) ( x + iy) + (7 - 5i) = 9 + 4i
(B) ® (q), (s), (t); (C) ® (r); (D) ® (r), (t); that is if the (r) 3
matches are (A) ® (p) and (s); (B) ® (q), (s) and (t); (C) x2 - y2 - i(2 x + y) = 2i
(C) ® (r); and (D) ® (r), (t); then the correct darkening (s) 4
of bubbles will look as follows:
(D) (2 + 3i) x2 - (3 - 2i) y = 2 x - 3 y + 5i (t) 0

p q r s t
A
B
C
D
www.jeeneetbooks.in
164 Chapter 3 Complex Numbers

2. Match the items in Column I with those in Column II. by equations with real coefficients are given. Match
the items in Column I with those in Column II.
Column I Column II
Column I Column II
(A) The number of values of q Î (-p , p )
æ iz + 1ö
for which 3 + 2i sin q is purely real is (p) 2 (A) If Re ç = 2, then z (p) 4 x2 + 4 y2 + x -
1 - 2i sin q è iz - 1 ÷ø
6y + 2 = 0
lies on the curve
(B) The number of values of q Î (-p , p )
(B) z1 = 6 + i, z2 = 4 - 3i and z
for which 3 + 2i sin q is purely (q) 3
is a complex number such
(q) x2 + y2 + 4 y + 3 = 0
1 - 2i sin q
imaginary is æ z - z1 ö p
that arg ç = ,
(C) The number of solutions of the è z2 - z ÷ø 2 (r) 3( x2 + y2 ) - 2 x - 4 y
(r) 4
equation then z lies on =0
( x + 2 xi) - (3 x + iy)
4 2
æ 2z + 1 ö
(C) If Im ç = 2, then z (s) x2 + y2 - x + 2 y - 1 = 0
= (1 + 2 yi) + (3 - 5i) (s) 0 è 1 + iz ÷ø
lies on
where x and y are positive real is
(D) The number of complex numbers z (t) 1 2z - i
(D) If = 1, then z lies on (t) ( x - 5)2 + ( y + 1)2 = 5
such that z = iz2 is z+1

3. In Column I equations which are satisfied by complex


number z are given. In Column II curves represented

Comprehension-Type Questions
1. Passage: If z1, z2 and z3 are three complex numbers represent the vertices of an equilateral triangle, then
representing the points A, B and C, respectively, in the (A) a12 = a3 (B) a12 = a2
Argands plane and ÐBAC = a, then
(C) a12 = a2 a3 (D) a13 = a2 a3
z3 - z1 æ AC ö
=ç ÷ (cos a + i sin a ) 2. Passage: Let X, Y and Z be the three sets of complex
z2 - z1 è AB ø
numbers defined as follows:
Answer the following three questions.
X = {z : Im(z) ³ 1}
(i) The four points 2 + i, 4 + i, 4 + 3i and 2 + 3i repre-
sent the vertices of Y = {z : | z - 2 - i | = 3}
(A) Square Z = {z : Re(z(1 - i)) = 2 }
(B) Rhombus but not a square
Answer the following questions.
(C) Rectangle but not a square
(i) The number of elements in the set X Ç Y Ç Z is
(D) Trapezium which is not rhombus/square/
(A) 0 (B) 1 (C) 3 (D) Infinite
rectangle
(ii) Let z be any point in X Ç Y Ç Z. Then | z + 1 - i |2 +
(ii) The roots of the equation z3 - 1 = 0 represent the
| z - 5 - i |2 lies between
vertices of
(A) 25 and 29 (B) 30 and 34
(A) An obtuse-angled triangle
(C) 35 and 39 (D) 40 and 44
(B) Isosceles but not an equilateral triangle
(iii) Let z be any point in X Ç Y Ç Z and w be any
(C) Equilateral triangle
point satisfying | w - 2 - i | < 3. Then | z | - | w | + 3
(D) Right-angled isosceles triangle lies between
(iii) If the roots of the equation (A) -6 and 3 (B) -3 and 6
z3 + 3a1z2 + 3a2 z + a3 = 0 (C) -6 and 6 (D) -3 and 9
www.jeeneetbooks.in
Exercises 165

Assertion–Reasoning Type Questions


In each of the following, two statements, I and II, are Statement II: Equation of the perpendicular bisec-
given and one of the following four alternatives has to tor of the segment joining two points z1 and z2 in the
be chosen. complex plane is z(z1 - z2 ) + z (z1 - z2 ) - z1z1 + z2 z2 = 0.
(A) Both I and II are correct and II is a correct reason-
ing for I. 5. Statement I: If a, b, c and u, v, w are complex numbers
(B) Both I and II are correct but II is not a correct representing the vertices of two triangles such that
reasoning for I. c = (1 - g )a + g b and w = (1 - g )u + g v, then the two
(C) I is true, but II is not true. triangles are similar.
(D) I is not true, but II is true. Statement II: Complex numbers z1 , z2 , z3 and z1¢, z2¢ , z3¢
1. Statement I: If p1 and p2 are distinct prime represent the vertices of directly similar triangles if
numbers and a complex number a ¹ 1 satisfies and only if the determinant
the equation zp1 + p2 - zp1 - zp2 + 1 = 0, then either z1 z1¢ 1
1 + a + a 2 + + a p1 - 1 = 0 or 1 + a + a 2 + + a p2 - 1 = 0
but not both. z2 z2¢ 1 = 0
z3 z3¢ 1
Statement II: For any two distinct prime numbers
p1 and p2, the two equations zp1 - 1 = 0 and zp2 - 1 = 0
cannot have common roots other than unity. 6. Statement I: If a and b are fixed complex numbers,
then the equation |(z - a)/(z - b)| = K(¹ 1) represents
2. Statement I: If a is a complex number satisfying the a circle whose radius and center are K |a - b|/|1 - K2|
equation (z + 1)8 = z8, then Re(z) = -1. and (a - K2 b)/(1 - K2).
Statement II: If z1 and z2 are fixed complex numbers Statement II: If a is a non-zero complex number
and z is any complex number such that |z - z1 | = |z - z2 |, and b is real such that | a |2 > b, then the equation
then z lies on the perpendicular bisector the segment zz + az + az + b = 0 represents a circle with center at
joining z1 and z2. -a and radius aa - b .

3. Let 7. Statement I: Let A, B and C be vertices of a triangle


x3 x6 described in counter clock sense and, respectively, be
a =1+ + + + ¥ represented by z1, z2 and z3. Then the area of DABC is
3 6
|Im(z1z2 + z2 z3 + z3z1 )/2 |.
x x4 x7
b= + + + + ¥ Statement II: The area of DABC is equal to the
1 4 7
absolute value of the number
x2 x5 x8
c= + + + + ¥, z1 z1 1
2 5 8 i
z2 z2 1
4
Statement I: a3 + b3 + c3 - 3abc = 1 . z3 z3 1
Statement II: a3 + b3 + c3 - 3abc = (a + b + c)(a + bw + cw2 ),
(a + bw2 + cw) where w ¹ 1 is a cube root of unity.

4. Statement I: Let lz + lz + m = 0 be a line in the


complex plane, where l ¹ 0 is a complex number and
m is a real number. If two points z1 and z2 are reflec-
tions of each other in the line, then lz1 + l z2 + m = 0.

Integer Answer Type Questions


The answer to each of the questions in this section is a example, as shown in the figure, if the correct answer to
non-negative integer. The appropriate bubbles below the the question number Y is 246, then the bubbles under Y
respective question numbers have to be darkened. For labeled as 2, 4, 6 are to be darkened.
www.jeeneetbooks.in
166 Chapter 3 Complex Numbers

X Y Z W 5. If z2/z1 is pure imaginary and a and b are non-zero real


0 0 0 0 numbers, then |(az1 + bz2 )/(az1 - bz2 )| is equal to .
1 1 1 1
2 2 2
6. If the points 1 + 2i and -1 + 4i are real reflections of
each other in the line z(1 + i) + z (1 - i) + K = 0, then
3 3 3 3
the value of K is .
4 4 4
5 5 5 5 7. If the straight lines ai z + ai z + bi = 0(i = 1, 2, 3), where
6 6 6 bi are real, are concurrent, then å b1 (a2 a3 - a2 a3 ) is
equal to .
7 7 7 7
8 8 8 8
8. The number of points z in the complex plane satisfying
9 9 9 9
both the equations |z - 4 - 8i| = 10 and |z - 3 - 5i| +
|z - 5 - 11i| = 4 5 is .
1. The number of common roots of the equations
x5 - x3 + x2 - 1 = 0 and x4 - 1 = 0 is . 9. If z = x + iy satisfies the equation z2 + z 2 = 2 then
x2 - y2 = K, where K is .
2. The quadratic equation z + (a + ib)z + (c + id) = 0 (a,
2

b, c, d) are real and (bd ¹ 0) has equal roots. Then the 10. If the area of a triangle with vertices Z1, Z2 and Z3 is
value of ab/d is . the absolute value of the number

Z1 Z1 1
3. If the equation z2 + (a + ib)z + (c + id) = 0 (a, b, c, d)
are real and (bd ¹ 0) has real root, where k is real, li Z2 Z2 1
then d2 - abd + bc is equal to . Z3 Z3 1

4. If z1 and z2 are complex numbers such that | z2 | ¹ 1 and then the value of 1/l is equal to .
|(z1 - 2z2 )/(2 - z1z2 )| = 1, then |z1 | is equal to .

ANSWERS
Single Correct Choice Type Questions
1. (B) 14. (D)
2. (C) 15. (D)
3. (A) 16. (A)
4. (D) 17. (A)
5. (B) 18. (D)
6. (D) 19. (B)
7. (D) 20. (A)
8. (D) 21. (A)
9. (C) 22. (C)
10. (B) 23. (D)
11. (C) 24. (D)
12. (C) 25. (B)
13. (A)

Multiple Correct Choice Type Questions


1. (C), (D) 6. (A), (C)
2. (B), (C), (D) 7. (A), (D)
3. (A), (B), (C), (D) 8. (A), (B)
4. (B), (C) 9. (A), (B)
5. (A), (B), (C), (D)
www.jeeneetbooks.in
Answers 167

Matrix-Match Type Questions


1. (A) ® (p), (B) ® (p), (C) ® (q), (D) ® (q) 3. (A) ® (q), (B) ® (t), (C) ® (p), (D) ® (r)
2. (A) ® (t), (B) ® (r), (C) ® (t), (D) ® (r)

Comprehension-Type Questions
1. (i) (A); (ii) (C); (iii) (B) 2. (i) (B); (ii) (C); (iii) (D)

Assertion–Reasoning Type Questions


1. (A) 5. (A)
2. (D) 6. (A)
3. (A) 7. (A)
4. (A)

Integer Answer Type Questions


1. 2 6. 6
2. 2 7. 0
3. 0 8. 2
4. 2 9. 1
5. 1 10. 4
www.jeeneetbooks.in
www.jeeneetbooks.in

Quadratic Equations
4
Contents
4.1 Quadratic Expressions
and Equations

Worked-Out Problems
Summary
Exercises
Answers

A polynomial equation of
the second degree having
the general form
Quadratic Equations

<0 ax2 + bx + c = 0
=0 is called a quadratic equation.
Here x represents a variable,
and a, b, and c, constants,
with a ¹ 0. The constants a, b,
and c are called, respectively,
the quadratic coefficient, the
linear coefficient and the
constant term or the free
term.
The term “quadratic” comes
from quadratus, which is the
>0 Latin word for “square”.
Quadratic equations can be
solved by factoring,completing
the square, graphing, Newton’s
method, and using the
quadratic formula (explained
in the chapter).
www.jeeneetbooks.in
170 Chapter 4 Quadratic Equations

In this chapter, we will discuss quadratic expressions and equations along with their roots. Numerous examples and
worked-out problems would help the readers understand the concepts. Exercises at the end of the chapters would help
evaluate your understanding.

4.1 | Quadratic Expressions and Equations


In this section, we discuss quadratic expressions and equations and their roots. Also, we derive various properties
of the roots of quadratic equations and their relationships with the coefficients.

DEF IN IT ION 4 . 1 A polynomial of the form ax2 + bx + c, where a, b and c are real or complex numbers and
a ¹ 0, is called a quadratic expression in the variable x. In other words, a polynomial f (x)
of degree two over the set of complex numbers is called a quadratic expression. We often
write f ( x) º ax2 + bx + c to denote a quadratic expression and this is known as the standard
form. In this case, a and b are called the coefficients of x2 and x, respectively, and c is called
the constant term. The term ax2 is called the quadratic term and bx is called the linear term.

DEF IN IT ION 4 . 2 If f ( x) º ax2 + bx + c is a quadratic expression and a is a complex number, then we write
f (a) for aa 2 + ba + c. If f (a) = 0, then a is called a zero of the quadratic expression f (x).

Examples
(1) Let f (x) º x2 - 5x - 6. Then f (x) is a quadratic expres- (3) Let f ( x) º 2 x2 - ix + 1 be a quadratic expression. In
sion and 6 and –1 are zeros of f (x). this case i and −i/2 are zeros of f (x).
(2) Let f (x) º x2 + 1. Then f (x) is a quadratic expression (4) The expression x2 + x is a quadratic expression and
and i and –i are zeros of f (x). 0 and –1 are zeros of x2 + x.

DEF IN IT ION 4 . 3 If f (x) is a quadratic expression, then f (x) = 0 is called a quadratic equation. If a is a zero
of f (x), then a is called a root or a solution of the quadratic equation f (x) = 0. In other
words, if f ( x) º ax2 + bx + c, a ¹ 0, then a complex number a is said to be a root or a solution
of f (x) = 0, if aa 2 + ba + c = 0. The zeros of the quadratic expression f (x) are same as the roots
or solutions of the quadratic equation f (x) = 0. Note that a is a zero of f (x) if and only if x − a
is a factor of f (x).

Examples

(1) 0 and –i are the roots of x2 + ix = 0. (3) i and –i are the roots of x2 + 1 = 0.
(2) 2 is the only root of x2 - 4 x + 4 = 0. (4) i is the only root of x2 - 2ix - 1 = 0.

T H E O R E M 4 .1 Let f (x) º ax2 + bx + c be a quadratic expression. Then the roots of the quadratic equation
f (x) = 0 are

-b ± b2 - 4ac
2a
that is,

-b + b2 - 4ac -b - b2 - 4ac
and
2a 2a

PROOF First note that for f ( x) º ax2 + bx + c to be a quadratic equation, it is necessary that a ¹ 0. Let a
be any complex number. Then
www.jeeneetbooks.in
4.1 Quadratic Expressions and Equations 171

a is a root of f ( x) = 0 Û aa 2 + ba + c = 0
Û 4a(aa 2 + ba + c) = 0
Û (2aa + b)2 - b2 + 4ac = 0
Û (2aa + b)2 = b2 - 4ac

Û 2aa + b = ± b2 - 4ac

-b ± b2 - 4ac
Ûa =
2a ■

Note that, in the above, b2 - 4ac denotes a square root of b2 - 4ac; that is, it is a complex number b such that
b = b2 - 4ac. From the above theorem, it follows that any quadratic equation has two roots, which are not neces-
2

sarily distinct. This is demonstrated in the examples described before. In the following some more examples are
considered.

Examples

(1) Consider the quadratic equation f(x) º x2 + x + 1 = 0. (3) Consider the equation 3 x2 + 2 x + 1 = 0. The roots of
Comparing with the standard form ax2 + bx + c, we this equation are
have a = 1 = b = c. Therefore, the roots of the given
equation are - 2 ± (2)2 - 4 ´ 3 ´ 1 1
= (- 1 ± i 2 )
2´3 3
-b ± b2 - 4ac - 1 ± 12 - 4 ´ 1 ´ 1 -1 ± i 3
= = (4) Consider the equation 3 ( x2 + 2) + 10( x - 3 ) = 0.
2a 2´1 2
To find its roots, we have to first transform this into
Recall from the previous chapter that these are the standard form ax2 + bx + c = 0. We thus obtain
precisely the cube roots of unity other than the unity.
(2) The roots of the quadratic equation x2 + 4ix - 4 = 0 3 ( x2 + 2) + 10( x - 3 ) = 3 x2 + 10 x - 8 3
are Therefore, the roots of the given equation are
-4i ± (4i) - (4(-4) ´ 1)
2
-4i ± - 16 + 16 - 10 ± (10)2 - 4 3(- 8 3 ) - 10 ± 14 2
= = - 2i = = , -4 3
2´1 2 2 3 2 3 3
-2i is a repeated root or a double root of the given
equation.

DEF IN IT ION 4 . 4 Let f ( x) º ax2 + bx + c, a ¹ 0. Then the discriminant of the quadratic expression f ( x) or the
quadratic equation f ( x) = 0 is defined as b2 - 4ac and is denoted by D[ f ( x)] or simply D.

It is evident that the roots of a quadratic equation f ( x) = 0 are real or imaginary according as the discriminant of
f ( x) is non-negative or negative, respectively. In the following we list the various natures of roots of a quadratic equa-
tion which mainly depend on the nature of the discriminant. The proof of the following theorem is a straight-forward
verification.

T H E O R E M 4 .2 Let a and b be the roots of the quadratic equation f ( x) º ax2 + bx + c = 0, where a, b and c are
real or complex numbers and a ¹ 0. Let D be the discriminant of f ( x), that is, D = b2 - 4ac. Then
the following hold good:
1. a = b Û D = 0 (i.e., b2 = 4ac), and in this case
-b
a= =b
2a
www.jeeneetbooks.in
172 Chapter 4 Quadratic Equations

2. If a, b and c are real numbers, then


(i) D > 0 Û a and b are real numbers and a ¹ b.
(ii) D < 0 Û a and b are non-real complex numbers which are conjugate to each other.

PROOF The proof is left as an exercise for the readers. ■

Examples

(1) The equation x2 + 5 x + 7 = 0 has no real roots, since (3) If a, b and c are rational numbers, then the roots of
the discriminant (5)2 - 4 ´ 7 ´ 1 = - 3 < 0. the equation
(2) Suppose that we wish to find the value of k such that x2 - 2ax + a2 - b2 + 2bc - c2 = 0
the equation x2 + 2(k + 2) x + 9k = 0 has equal roots.
The discriminant is given by are also rational, for the discriminant is given by

D = b2 - 4ac = [2(k + 2)]2 - 4 ´ 1 ´ 9k D = (- 2a)2 - 4 ´ 1 ´ (a2 - b2 + 2bc - c2 )


= 4a2 - 4a2 + 4b2 - 8bc + 4c2
= 4k2 + 16k + 16 - 36k
= 4b2 - 8bc + 4c2
= 4k2 - 20k + 16
Since the roots are equal, therefore the discriminant = 4(b - c)2
should be zero, that is Since b and c are rational numbers, (b - c)2 is a
D = 0 Û k - 5k + 4 = 0
2 non-negative rational number and hence D ³ 0, so that
the given equation has real roots. Also, the roots are
-(- 5) ± (- 5)2 - 4 ´ 1 ´ 4
Ûk= -(- 2a) ± 4(b - c)2
2 = a ± (b - c )
2
5±3
Ûk= = 4 or 1 which are rational numbers, since a, b and c are so.
2

T H E O R E M 4.3 Let a and b be the roots of the quadratic equation ax2 + bx + c = 0. Then
-b c
a+b= and ab =
a a
PROOF The values of a and b are given by

-b ± b2 - 4ac
2a
and hence
- b + b2 - 4ac -b - b2 - 4ac -b
a+b= + =
2a 2a a

-b + b2 - 4ac -b - b2 - 4ac b2 - (b2 - 4ac) c


and ab = ´ = =
2a 2a 4a2 a ■

Also, we can write down a quadratic equation if the roots are known. In other words, if a and b are any given
complex numbers, then a( x - a )( x - b ) = a[ x + (-a - b ) x + ab ] = 0 is a quadratic equation whose roots are a and b,
where a is an arbitrary non-zero real or complex number. This can also be verified by observing
æ b cö
ax2 + bx + c = a ç x2 + x + ÷
è a aø
= a( x2 - (a + b ) x + ab )
= a( x - a )( x - b )
www.jeeneetbooks.in
4.1 Quadratic Expressions and Equations 173

QUICK LOOK 1

If the coefficient of x2 in a quadratic equation is unity 2. The product of the roots is equal to the constant term.
(i.e., 1), then 3. The equation can be written as ( x - a )( x - b ) = 0,
1. The sum of the roots is equal to the coefficient of x where a and b are the roots.
with its sign changed; that is, a + b + b = 0, where b
is the coefficient of x.

Example 4.1

Find the quadratic equation whose roots are 2 and –i. ( x - 2)[ x - (-i)] = ( x - 2)( x + i) = x2 + (i - 2) x - 2i
Hence the equation is x2 + (i - 2) x - 2i = 0.
Solution: The required quadratic expression is

Example 4.2

Find the quadratic equation whose roots are 1 + i and 3 [ x - (1 + i)]( x - (1 - i)) = 3 [( x - 1) - i)][( x - 1) + i]
1 – i and in which the coefficient of x2 is 3.
= 3 [( x - 1)2 + 1]
Solution: The required quadratic expression is = 3 x2 - 6 x + 6
Hence the equation is 3x2 - 6x + 6 = 0.

Example 4.3

If a and b are roots of the quadratic equation ax2 + bx + 0 = ( x - za )( x - zb )


c = 0 and z is any complex number, then find the quadratic
= x2 - (za + zb ) x + za ´ zb
equation whose roots are za and zb .
= x2 + z[-(a + b )]x + z2ab
Solution: We have
æ bö c
-b c = x2 + z ç ÷ x + z2
a+b= and ab = è aø a
a a
that is,
The equation whose roots are za and zb is
ax2 + zbx + z2 c = 0

Example 4.4

If a and b are the roots of a quadratic equation Therefore, the required equation is
ax2 + bx + c = 0, then find the quadratic equation whose
0 = a[ x - (a + z)] ´ [ x - (b + z)]
roots are a + z and b + z, where z is any given complex
number. = ax2 + a[-(a + z) - (b + z)]x + a(a + z)(b + z)
æb ö æc b ö
Solution: We have = ax2 + a ç - 2z÷ x + a ç - z + z2 ÷
èa ø èa a ø
-b
(a + z) + (b + z) = (a + b ) + 2z = + 2z = ax2 + (b - 2az) x + (c - bz + az2 )
a
Therefore, the quadratic equation whose roots are a + z
c b and b + z is
and (a + z) × (b + z) = ab + (a + b )z + z2 = - z + z2
a a
ax2 + (b - 2az) x + (c - bz + az2 ) = 0

Example 4.5

Let a and b be the roots of a quadratic equation Then find the quadratic equation whose roots are pa + q
ax2 + bx + c = 0 and p and q be any complex numbers. and pb + q.
www.jeeneetbooks.in
174 Chapter 4 Quadratic Equations

Solution: Consider, Therefore, the required equation is


pb æ - pb ö æ p2 c pqb ö
( pa + q) + ( pb + q) = p(a + b ) + 2q = - + 2q x2 - ç + 2q÷ x + ç - + q2 ÷ = 0
a è a ø è a a ø
and ( pa + q) × ( pb + q) = p2 ab + pq(a + b ) + q2 ax2 + ( pb - 2aq) x + ( p2 c - pqb + q2 a) = 0
p2 c pqb
= - + q2
a a

Example 4.6

If a and b are roots of the quadratic equation æ 1ö æ 1ö 1 1 a


and çè ÷ø ç ÷ = = =
ax + bx + c = 0 and c ¹ 0
2 a è b ø ab c /a c

find the quadratic equation whose roots are 1/a and 1/b . Therefore, the required equation is

Solution: First, let us observe that a ¹ 0 and b ¹ 0, as a æ -b ö a


x2 - ç ÷ x + = 0
and b are roots of ax2 + bx2 + c = 0 and c ¹ 0. Now, consider è c ø c

1 1 a + b -b /a b That is,
+ = = =-
a b ab c /a c cx2 + bx + a = 0

The results obtained in the examples given above are summarized in the following and the reader can easily
supplement formal proofs of these.

QUICK LOOK 2

Let f ( x) º ax2 + bx + c = 0 be a quadratic equation and 3. f (- x) = 0 is an equation whose roots are -a and -b.
a and b be its roots. Then the following hold good. 4. If ab ¹ 0 and c ¹ 0, f(1/x) = 0 is an equation whose
1. f (x - z) = 0 is an equation whose roots are a + z and roots are 1/a and 1/b .
b + z, for any given complex number z. 5. For any complex numbers z1 and z2 with z1 ¹ 0,
2. f ( x / z) = 0 is an equation whose roots are za and zb f [( x - z2 )/z1 ] = 0 is an equation whose roots are
for any non-zero complex number z. z1a + z2 and z1 b + z2.

Note: If ax2 + bx + c = 0 is a quadratic equation, then for any non-zero complex number d, the equation
dax2 + dbx + dc = 0
has the same roots as ax2 + bx + c = 0. Therefore, given a and b , the quadratic equation whose roots are a and b is not
unique. However any two such equations are equivalent in the sense that their coefficients are proportional.

T H E O R E M 4 .4 Two quadratic equations


ax2 + bx + c = 0 and a ¢x2 + b¢ x + c ¢ = 0
have same roots if and only if the triples (a, b, c) and (a ¢, b¢, c ¢) are proportional and, in this case,
a
ax2 + bx + c = (a ¢x2 + b¢ x + c ¢)

PROOF Suppose that a and b are the roots of ax2 + bx + c = 0 and a ¢x2 + b¢ x + c ¢ = 0 simultaneously. Then
by Theorem 4.3 we have
-b b¢ c c¢
=a + b = - and = ab =
a a¢ a a¢
www.jeeneetbooks.in
4.1 Quadratic Expressions and Equations 175

Now
æ b cö æ b¢ c ¢ ö a
(a, b, c) = a ç 1, , ÷ = a ç 1, , ÷ = (a ¢, b¢, c ¢)
è a aø è a¢ a¢ ø a¢
Therefore, (a, b, c) and (a ¢, b¢, c ¢) are proportional and
a
ax2 + bx + c = (a ¢x2 + b¢ x + c ¢)

Conversely, suppose that (a, b, c) and (a ¢, b¢, c ¢) are proportional. Then, there is non-zero d such that
(a, b, c) = d(a ¢, b¢, c ¢)
and hence ax + bx + c = d(a ¢x + b¢ x + c ¢). Therefore, for any complex number a,
2 2

aa 2 + ba + c = 0 Û a ¢a 2 + b¢a + c ¢ = 0 ■

Example

The quadratic equations 2 x2 + 3 x + 1 = 0 and 6 x2 + 9 x + 3 = 0 have same roots since 3(2, 3, 1) = (6, 9, 3).

Example 4.7

Let a and b be the roots of the quadratic equation - b - 2c æb ö


= = - ç + 2÷
ax + bx + c = 0, c ¹ 0
2 c èc ø

Find the quadratic equation whose roots are Also

1-a 1- b æ 1 - a ö æ 1 - b ö 1 - (a + b ) + ab
and çè ÷ =
a b a ø çè b ÷ø ab

Solution: 1 + (b / a) + (c / a)
We have =
c /a
-b c
a+b= and ab = a+b+c
a a =
c
Now, consider
Therefore, the quadratic equation whose roots are (1 - a)/a
1 - a 1 - b b (1 - a ) + a (1 - b ) and (1 - b )/b is
+ =
a b ab
é æb öù a+b+c
(a + b ) - 2ab x2 - ê - ç + 2÷ ú x + =0
ë è c ø û c
=
ab
cx2 + (b + 2c) x + (a + b + c) = 0
(- b / a) - 2(c / a)
=
c /a

Example 4.8

If a and b are the roots of the quadratic equation (i) a 2 + b 2 = (a + b )2 - 2ab


ax2 + bx + c = 0 , then evaluate the following: 2
æ bö æ cö 1
(i) a 2 + b 2 (ii) a 3 + b 3 (iii) a 4 + b 4 = ç - ÷ - 2 ç ÷ = 2 (b2 - 2ac)
è aø è aø a
Solution: We know that (ii) a 3 + b 3 = (a + b )3 - 3ab (a + b )
-b c æ bö
3
æ c ö æ bö 1
a+b= and ab = = ç - ÷ - 3 ç ÷ ç - ÷ = 3 (3ab
bc - b3 )
a a è aø è aø è aø a
www.jeeneetbooks.in
176 Chapter 4 Quadratic Equations

(iii) a 4 + b 4 = (a 2 + b 2 )2 - 2a 2 b 2 1
= [(b2 - 2ac)2 - 2c2 a2 ]
2 2 a4
é1 ù æ cö
= ê 2 (b2 - 2ac)ú - 2 ç ÷ 1 4
ëa û è aø = [b + 2c2 a2 - 4ab2 c]
a4

Example 4.9

Find the quadratic equation whose roots are a and b, Therefore, the required equation is
where a + b = 1 and a 2 + b 2 = 13.
0 = x2 - (a + b ) x + ab = x2 - x - 6
Solution: We have
1 1
ab = [(a + b )2 - (a 2 + b 2 )] = (1 - 13) = - 6
2 2

Certain polynomial equations of degree greater than two can be reduced to quadratic equations by suitable substitutions.
These are demonstrated in the following examples.

Example 4.10

Find the solutions of the equation This gives y = 5, -1. Therefore


x - 4x - 5 = 0
4 2
x2 = 5, - 1

Solution: Put y = x2. Then the given equation is reduced to x = ± 5, ± i


y - 4y - 5 = 0
2 Hence 5 , - 5 , i and -i are the solutions of the given
equation.
( y - 5)( y + 1) = 0

Example 4.11

Solve the equation x4 - 3 x3 + 2 x2 - 3 x + 1 = 0. This gives y = 0, 3. When y = 0, we have


1
Solution: Since zero is not a solution of this equation, x+ = 0 Þ x2 + 1 = 0 Þ x = ± i
we can divide both sides of the equation by x2 and get x
an equation whose roots are same as that of the given When y = 3, we have
equation. That is
1
3 1 x+ = 3 Þ x2 - 3 x + 1 = 0
x2 - 3 x + 2 - + =0 x
x x2
1 æ 1ö -(-3) ± (-3)2 - 4 3± 5
x + 2 - 3ç x + ÷ + 2 = 0
2 Þx= =
x è xø 2 2

Putting y = x + (1/x), we get Thus i, - i, (3 + 5 )/ 2 and (3 - 5 )/ 2 are the solutions of


the given equation.
( y2 - 2) - 3 y + 2 = 0
y2 - 3 y = 0

Example 4.12

Solve x2 / 5 + 3 x1/ 5 - 4 = 0. y2 + 3 y - 4 = 0

Solution: By substituting y = x1/ 5, the given equation ( y + 4)( y - 1) = 0


reduces to a quadratic equation given by y = 1 or - 4
www.jeeneetbooks.in
4.1 Quadratic Expressions and Equations 177

Now Therefore −1024 and 1 are the solutions of the given


equation.
y = 1Þ x 1/ 5
= 1Þ x = 1
y = - 4 Þ x1/ 5 = - 4 Þ x = (-4)5 = - 1024

Example 4.13

Solve 4x + 3 × 4- x - 4 = 0. Now
y = 1 Þ 4x = 1 Þ x = 0
Solution: Substituting y = 4x , we get
y = 3 Þ 4x = 3 Þ x = log4 3
3
y+ -4=0 Therefore 0 and log4 3 are the solutions of the given
y
equation.
y2 - 4 y + 3 = 0
( y - 3)( y - 1) = 0
y = 3 or 1

Example 4.14

Solve the following: Now

x 1 - x 13 3 x 3
+ = y= Þ =
1- x x 6 2 1- x 2

Solution: Substituting y for x /(1 - x), we get Þ 4 x = 9(1 - x)

1 13 9
y+ = Þx=
y 6 13

6 y2 - 13 y + 6 = 0 2 x 2
y= Þ =
3 1- x 3
(2 y - 3)(3 y - 2) = 0
Þ 9x = 4(1 - x)
3 2
y= or
2 3 4
Þx=
13
Therefore 9/13 and 4/13 are the solutions of the given
equation.

Example 4.15

Find all pairs of consecutive positive odd integers such x2 + 2 x - 143 = 0


that the sum of their squares is 290.
( x + 13)( x - 11) = 0
Solution: Let x be a positive odd integer. Then x + 2 x = - 13 or x = 11
will be the next odd integer. We have to find all the value
of x for which But x is given to be odd positive integer. Therefore
x = 11 and x + 2 = 13. Thus, (11, 13) is the unique pair of
x2 + ( x + 2)2 = 290 consecutive positive odd integers such that the sum of
their squares is 290.
2 x2 + 4 x - 286 = 0
www.jeeneetbooks.in
178 Chapter 4 Quadratic Equations

Example 4.16

Derive a necessary condition that one root of the Therefore


quadratic equation ax2 + bx + c = 0, a ¹ 0 and c ¹ 0, 2
is n times the other, where n is a positive integer. -b æ -b ö c
a= and ç n=
a(n + 1) è a(n + 1) ÷ø a
Solution: Let a and na be roots of the equation ax2 +
bx + c = 0. Then Simplifying the second equation we get nab2 = (n + 1)2
a2c. Now since a ¹ 0, nb2 = (n + 1)2 ac.
b c
a + na = - and a × na =
a a

T H E O R E M 4 .5 If a, b and c are real numbers and a ¹ 0, then (4ac - b2 )/ 4a is the maximum or minimum value of
quadratic equation of f ( x) º ax2 + bx + c according as a < 0 or a > 0, respectively.
PROOF We have

æ b cö
f ( x) º ax2 + bx + c º a ç x2 + x + ÷
è a aø

éæ 4ac - b2 ù
2 2
bö æ bö 4ac - b2
º a êç x + ÷ + ú º aç x + ÷ +
êëè 2a ø è 2a ø
2
4a úû 4a

If a < 0, then
4ac - b2 æ -b ö
f ( x) £ =fç ÷ for all x Î
4a è 2a ø

Hence (4ac - b2 )/ 4a is the maximum value of f ( x).


If a > 0, then
æ - b ö 4ac - b
2
fç ÷= £ f ( x) for all x Î
è 2a ø 4a
Hence (4ac - b2 )/ 4a is the minimum value of f ( x). ■

QUICK LOOK 3

1. If a, b and c are real numbers and a < 0, then 2. If a, b and c are real numbers and a > 0, then f(-b/2a)
f(-b/2a) is the maximum value of f(x) º ax2 + bx + c. is the minimum value of f(x) º ax2 + bx + c.

Examples

(1) The maximum value of 2 x - x2 + 3 is (2) The minimum value of x2 + 3 x + 2 is


2 2
æ -2 ö æ -2 ö æ -3 ö æ -3 ö 9 9 1
2ç - + 3= 2 -1+ 3= 4 çè 2(1) ÷ø + 3 çè 2(1) ÷ø + 2 = 4 - 2 + 2 = - 4
è 2(-1) ÷ø çè 2(-1) ÷ø

T H E O R E M 4 .6 Let f ( x) = ax2 + bx + c, where a, b and c are real numbers and a ¹ 0.


1. If a and b are real roots of f ( x) = 0 and a < b , then
(i) f ( x) and a will have the same sign for all real x < a or x > b .
(ii) f ( x) and a will have opposite sign for all real x such that a < x < b .
2. If f ( x) = 0 has imaginary roots, then f ( x) and a will have the same sign for all real x.
www.jeeneetbooks.in
4.1 Quadratic Expressions and Equations 179

PROOF 1. It is given that f ( x) º a( x - a )( x - b ). Therefore


f ( x)
= ( x - a )( x - b )
a
(i) If x < a, then x < b also and hence x - a and x - b are both negative, so that f ( x)/ a > 0.
Similarly, if x > b, then x > a also and hence both x - a and x - b are positive, so that
f ( x)/ a > 0. Therefore, in either case, f ( x)/ a is positive, and hence f ( x) and a are either
both positive or both negative.
(ii) If a < x < b , then x - a > 0 and x - b < 0 and hence f ( x)/ a < 0 which implies that one of
f ( x) and a are positive and the other is negative.
2. Suppose that f ( x) = 0 has imaginary roots. Then b2 - 4ac < 0 and
2
f ( x) æ bö 4ac - b2
= çx + ÷ + >0
a è 2a ø 4a2
for all real x. Hence either both f ( x) and a are positive or both are negative. ■

QUICK LOOK 4

Let f ( x) º ax2 + bx + c, where a, b and c are real numbers and a ¹ 0. Consider the graph of the curve y = ax2 + bx + c.
Different cases considered in Theorem 4.6 are described next by means of the graph of y = ax2 + bx + c.
1.
y y

y = ax 2 +bx +c
y = ax 2 +bx +c

x x
a b a b

a<0 a>0

f ( x) < 0 for all x Ï[a , b ] f ( x) > 0 for all x Ï[a , b ]


f ( x) > 0 for all x Î(a , b ) f ( x) < 0 for all x Î(a , b )
2.
y y

y = ax 2 +bx +c

x x

y = ax 2 +bx +c
a>0
a<0

f ( x) = 0 has no real roots f ( x) = 0 has no real roots

Examples
(1) 2x2 - 11x + 15 > 0 for all x < 5/2 or x > 3 [by Theorem (2) -2x2 + x + 15 < 0 for all x < -5/2 or x > 3 and -2x2 + x +
4.6 (1(i))] and 2x2 - 11x + 15 < 0 for all 5/2 < x < 3 15 > 0 for all - 5 / 2 < x < 3 since - (5 / 2) and 3 are roots
[by Theorem 4.6 (1(ii))] since 5/2 and 3 are roots of of this quadratic expression in which the coefficient of
this quadratic expression in which the coefficient of x2 x2 = - 2 < 0.
is 2 > 0.
www.jeeneetbooks.in
180 Chapter 4 Quadratic Equations

WORKED-OUT PROBLEMS
Single Correct Choice Type Questions
1. If the equations m < 0 and 3m2 + 4 m - 4 > 0
x2 + ax + 1 = 0 and x2 - x - a = 0 Þ m < 0 and (3m - 2)(m + 2) > 0
have a real common root, then the value of a is This gives m < -2 and so
(A) 0 (B) 1 (C) −1 (D) 2 x2 - 5 x + 6 < 0 Þ ( x - 2)( x - 3) < 0 Þ x Î (2, 3)
Answer: (C)
Solution: Let a be a real common root. Then
a 2 + aa + 1 = 0 4. If p is prime number and both the roots of the equation
x2 + px - (444) p = 0 are integers, then p is equal to
a2 - a - a = 0 (A) 2 (B) 3 (C) 31 (D) 37
Therefore
Solution: Suppose the roots of x2 + px - (444) p = 0 are
a (a + 1) + (a + 1) = 0 integers. Then the discriminant
(a + 1)(a + 1) = 0 p2 + 4(444) p = p{ p + 4 ´ (444)}
If a = - 1, then the equations are same and also cannot must be a perfect square. Therefore p divides p + 4 ´
have a real root. Therefore a + 1 ¹ 0 and hence a = - 1, (444). This implies
so that a = 2.
p divides 4 ´ (444) = 24 ´ 3 ´ 37
Answer: (D)
Therefore
2. If the roots of the equation x + px + q = 0 are cubes
2 p = 2 or 3 or 37
of the roots of the equations x2 + mx + n = 0, then If p = 2 or 3 then p2 + 4 (444) p is not a perfect square and
(A) p = m3 + 3 mn (B) p = m3 - 3 mn when p = 37, it is a perfect square. Therefore, p = 37.
3
p æ mö Answer: (D)
(C) p + q = m3 (D) =ç ÷
q è nø 5. If a, b and c are distinct real numbers, then the number
Solution: Let a and b be the roots of the equation of real solutions of the equation
x2 + mx + n = 0. Therefore ( x - a)( x - b) ( x - b)( x - c) ( x - c)( x - a)
+ + + 1= 0
a + b = - m, ab = n (c - a)(c - b) (a - b)(a - c) (b - c)(b - a)

Also since a and b are the roots of the equation is


x2 + mx + n = 0, so that a 3 and b 3 are the roots of the (A) 0 (B) 1 (C) 2 (D) infinite
equation x2 + px + q = 0. Now, Solution: Let p( x) = 0 be the given equation. Then
a 3 + b 3 = - p and a 3 b 3 = q p(a) = p(b) = p(c) = 2
We have Since p( x) is a polynomial of degree 2 and a, b and c
-p = a + b
3 3 are distinct real numbers, it follows that p(x) º 2, that is
p( x) = 2 for all x.
= (a + b )3 - 3ab (a + b )
Answer: (A)
= - m3 - 3n (- m)
6. The number of real solutions of the equation
Therefore p = m3 - 3 mn.
Answer: (B) x + 14 - 8 x - 2 + x + 23 - 10 x - 2 = 3

is
3. If ( x - 5 x + 4) ( y + y + 1) < 2 y for all real numbers
2 2

y then x belongs to the interval (A) 2 (B) 4 (C) 8 (D) infinite


(A) (3, 4) (B) (3, 5) (C) (2, 3) (D) (–1, 2) Solution: The given equation is

Solution: Let m = x2 - 5 x + 4. Then my2 + (m - 2) y + ( x - 2 - 4)2 + ( x - 2 - 5)2 = 3


m < 0 for all real y. Therefore, m < 0 (by taking y = 0 )
and (m - 2)2 - 4 m2 < 0. Hence we have x-2-4 + x-2-5 =3
www.jeeneetbooks.in
Worked-Out Problems 181

Put x - 2 - 5 = y. Then, the given equation becomes exist and are in arithmetic progression is
| y + 1| + | y | = 3 (Note: p, q, r are said to be in arithmetic progression
if q - p = r - q.)
Case 1: Suppose y ³ 0. Then y + 1 + y = 3 or y = 1. Therefore (A) -33/4 (B) 33/4 (C) −12 (D) 12
x - 2 - 5 = 1 Þ x = 38 Solution: Put y = 5x + 5- x . Then 5 y, a / 2, y2 - 2 are in
Case 2: Suppose y £ -1. Then y + 1 £ 0. This implies AP. Therefore

-(y + 1) - y = 3 or y = -2 æ aö
5 y + y2 - 2 = 2 ç ÷ = a
è 2ø
Hence
x - 2 - 5 = - 2 Þ x = 11 This implies that y2 + 5 y - 2 - a = 0 has real solutions.
Hence
Note that -1 < y < 0 is impossible (for, otherwise, 3 =
|y + 1| + |y| = y + 1 - y). Thus, x = 38 or 11. 25 + 4 (a + 2) ³ 0
Answer: (A) 33
a³- (4.3)
4
7. If a and b are roots of the equation (x + c)(x + d) -
k = 0, then the roots of the equation( x - a)( x - b) + Also, since (5x / 2 - 5- x / 2 )2 ³ 0, we get that
k = 0 are
y = 5x + 5- x ³ 2
(A) c, d (B) −c, −d (C) −c, d (D) c, −d
Therefore
Solution: If a and b are roots of the equation ( x + c)
( x + d) - k, then a = y2 + 5 y - 2 ³ 12 (4.4)
( x + c)( x + d) - k = ( x - a)( x - b) From Eqs. (4.3) and (4.4), we get a ³ 12. Therefore the
and hence minimum value of a is 12 and for this value of a, we have

( x - a)( x - b) + k = ( x + c)( x + d) y2 + 5 y - 14 = 0
Therefore, - c, - d are the roots of ( x - a)( x - b) + k = 0. ( y + 7)( y - 2) = 0
Answer: (B) y = - 7 or 2
-x
8. The number of integer values of x satisfying But y = 5 + 5 > 0. Therefore y = 2. This implies
x

( x + 1) > 5 x - 1 and ( x + 1) < 7 x - 3


2 2
5x + 5- x = 2
simultaneously is 52 x - 2 ´ 5x + 1 = 0
(A) 1 (B) 2 (C) 4 (D) 0
(5x - 1)2 = 0
Solution: The first inequality,
5x = 1
( x + 1)2 > 5 x - 1 Þ x2 - 3 x + 2 > 0
x=0
Þ ( x - 1)( x - 2) > 0
Therefore the following are in arithmetic progression:
Þ x < 1 or x>2 (4.1)
a
The second inequality, 51+ x + 51- x = 10, = 6 and 52 x + 5-2 x = 2
2
( x + 1)2 < 7 x - 3 Þ x2 - 5 x + 4 < 0 Answer: (D)
Þ ( x - 1)( x - 4) < 0
Þ1< x < 4 (4.2) 10. Let a, b be positive real numbers. If the equations
x2 + ax + 2b = 0 and x2 + 2bx + a = 0 have real roots,
From Eqs. (4.1) and (4.2), we get 2 < x < 4 and that x is then minimum value of a + b is
an integer. Therefore x = 3.
(A) 4 (B) 6 (C) 8 (D) 2
Answer: (A)
Solution: We have
9. The minimum value of “a” for which the real values
if x such that x2 + ax + 2b = 0 has real roots Þ a2 ³ 8b (4.5)
a x2 + 2bx + a = 0 has real roots Þ b2 ³ a (4.6)
51+ x + 51- x , , 52 x + 5-2 x
2
www.jeeneetbooks.in
182 Chapter 4 Quadratic Equations

Therefore [Note that ( x2 - x + 1)( x2 + x + 1) = x4 + x2 + 1 ]. Therefore,


æ a2 ö a4
2
( x2 + 1)[a - 1 - (a + 1)] + x(a - 1 + a + 1) = 0
a£b £ç ÷ =
2

è 8ø 64 Þ - 2( x2 + 1) + 2ax = 0
64 £ a3 or a ³ 4 (4.7) Þ x2 - ax + 1 = 0
Now, This has distinct real roots if and only if a2 - 4 > 0, that
is, | a | > 2.
b2 ³ a and a ³ 4 Þ b ³ 2 (4.8)
Answer: (B)
From Eqs. (4.7) and (4.8), we have a + b ³ 6 and, for
values a = 4 and b = 2, the equations x2 + ax + 2b = ( x + 2)2 13. If a, b, c and d are distinct positive real numbers such
and x2 + 2 bx + a = ( x + 2)2 have real roots. that a and b are the roots of x2 - 10cx - 11d = 0 and
Answer: (B) c and d are the roots of x2 - 10ax - 11b = 0, then the
value of a + b + c + d is
11. Let a, b, c and d be non-zero real numbers. If c and d (A) 1110 (B) 1010 (C) 1101 (D) 1210
are roots of the equation x2 + ax + b = 0 and a and b
are roots of the equation x2 + cx + d = 0, then the Solution: Since a and b are the roots of x2 - 10cx - 11d = 0
value of -(a + b + c + d) is we have
(A) 1 (B) 2 (C) 3 (D) 4 (i) a + b = 10c and (ii) ab = - 11d (4.11)
Solution: Since c and d are roots of the equation Also since c and d are the roots of x - 10ax - 11b = 0,
2

x2 + ax + b = 0, we have we have
c + d = - a and cd = b (4.9) (i) c + d = 10a and (ii) cd = - 11b (4.12)
Since a and b are roots of the equation x + cx + d = 0,2 Adding part (i) of Eqs. (4.11) and (4.12), we get
we have a + b + c + d = 10(a + c) Þ b + d = 9(a + c) (4.13)
a + b = -c and ab = d (4.10) Multiplying part (ii) of Eqs. (4.11) and (4.12), we get
From Eqs. (4.9) and (4.10) we have abcd = 121 bd Þ ac = 121 (4.14)
a+b+c=0=a+c+d Also,
and cd = b, ab = d a2 - 10ca - 11d = 0 = c2 - 10ca - 11b
We thus have b = d ¹ 0. Therefore a = c = 1 and b = d = -2.
Þ a2 + c2 - 20ca - 11(b + d) = 0
Hence
From Eqs. (4.13) and (4.14), we have
a + b + c + d = 0 + d = d = -2
a2 + c2 - 20(121) - 99(a + c) = 0
- (a + b + c + d ) = 2
Answer: (B) (a + c)2 - 2 ´ 121 - 20 ´ 121 - 99(a + c) = 0
(a + c - 121)(a + c + 22) = 0
12. If (a - 1)(x2 + x + 1)2 - (a + 1)(x4 + x2 + 1) = 0 has distinct
a + c = 121
real roots, then
(A) | a | < 2 (B) | a | > 2 or a + c = - 22
(C) | a | = 2 (D) a is not a real number Since a, c are positive, a + c ¹ -22. Therefore a + c = 121
and
Solution: Consider
2 a + b + c + d = (a + c) + 9(a + c) = 1210
æ 1ö 3
x2 + x + 1 = ç x + ÷ø + > 0 (for all real x) Answer: (D)
è 2 4
Therefore, the given equation can be written as 14. The sum of all the real roots of the equation
| x - 2 |2 + | x - 2 | - 2 = 0 is
(a - 1)( x2 + x + 1) - (a + 1)( x2 - x + 1) = 0
(A) 1 (B) 2 (C) 3 (D) 4

Try it out If we drop the condition that a, b, c and d are positive and assume that they are distinct non-zero
real numbers, then also a + b + c + d value may be 1210 (Try!)
www.jeeneetbooks.in
Worked-Out Problems 183

Solution: a2 + b2 + c2
<2 (4.16)
Case 1: Suppose x ³ 2. Then the equation becomes ab + bc + ca
( x - 2)2 + ( x - 2) - 2 = 0 From Eqs. (4.15) and (4.16) we get
x2 - 3 x = 0 l<
2 2 4
+ =
3 3 3
Since x ³ 2, we get that x = 3.
Answer: (A)
Case 2: Suppose x < 2. Then the equation becomes
17. Let a and b be roots of the equation x2 - px + r = 0
(2 - x)2 + (2 - x) - 2 = 0
and a /2 and 2b be the roots of the equation
x2 - 5x + 4 = 0 x2 - qx + r = 0. Then the value of r is
( x - 1)( x - 4) = 0 2 2
(A) ( p - q)(2q - p) (B) (q - p)(2 p - q)
9 9
But x < 2. Therefore x = 1. Thus the real roots of the
equation are 1 and 3 and their sum is 4. 2 2
(C) (q - 2 p)(2q - p) (D) (2 p - q)(2q - p)
Answer: (D) 9 9
Solution: Since a and b are roots of the equation
15. If the product of the roots of the equation x2 - 3kx + x2 - px + r = 0, we have
2e2 log k - 1 = 0 is 7, then the roots are real for k = a + b = p and ab = r
(A) 0 (B) 1 (C) 2 (D) 3
Since a / 2 and 2b are the roots of the equation x2 - qx +
Solution: Observe that log k is defined when k > 0. r = 0, we have
The given equation is x2 - 3kx + 2k2 - 12 = 0. It is given a a
that the product of the roots is 7. That is + 2 b = q and ´ 2b = r
2 2
2k2 - 1 = 7 Therefore,
k =4
2
a + b = p and a + 4 b = 2q
k = ±2 Solving the two equations we get
Since k > 0, we get that k = 2. Further, for k = 2, the given 2q - p 2q - p 2
b= and a = p - b = p - = (2 p - q)
equation is x2 - 6 x + 7 = 0 whose roots are 3 ± 2 , which 3 3 3
are real. Therefore
Answer: (C)
2
r = ab = (2 p - q) (2q - p)
9
16. Let a, b and c be the sides of a triangle, where a, b, c
are distinct, and l be a real number. If the roots of the Answer: (D)
equation x2 + 2(a + b + c) x + 3l (ab + bc + ca) = 0 are
18. If sin2 a cos2 a = sin2 b , then the roots of the equa-
real, then
tion x2 + 2 x cot b + 1 = 0 are always
4 5
(A) l < (B) l > (A) equal (B) imaginary
3 3 (C) real and distinct (D) greater than 1
1 5 4 5 Solution: The discriminant of the given equation is
(C) <l< (D) <l<
3 3 3 3
4 cot2 b - 4 = 4(cot2 b - 1)
Solution: The given equation has real roots. Therefore, = 4(cosec2 b - 2)
4(a + b + c)2 - 12l (ab + bc + ca) ³ 0 4 4
= -8 = -8
a + b + c + (2 - 3l )(ab + bc + ca) ³ 0
2 2 2 sin2 b sin2 a cos2 a
æ 2 ö
= 8ç 2 - 1÷
a2 + b2 + c2 2 è sin 2a ø
l£ + (4.15)
3(ab + bc + ca) 3
2 - sin2 2a
=8 >0
Since a, b and c are sides of a triangle, (a - b)2 < c2 , sin2 2a
(b - c)2 < a2 and (c - a)2 < b2 , so that Answer: (C)
www.jeeneetbooks.in
184 Chapter 4 Quadratic Equations

19. If l is real and (l 2 + l - 2) x2 + (l + 2) x < 1 for all It is enough if we show that D(l ) > 0 for any l and hence
real x, then l belongs to the interval to prove that the discriminant of D(l ) is negative. The
(A) (-2, 1) (B) (-2, 2/5) discriminant of D(l ) is given by
(C) (2/5, 1) (D) (1, 2)
4(ab + ad + bc + cd - 2bd - 2ac)2 - 4(a - c)2 (b - d)2
Solution: Suppose that (l + l - 2) x + (l + 2) x - 1 < 0
2 2
= 4[ab + ad + bc + cd - 2bd - 2ac + (a - c)(b - d)]
for all real x. Then from Theorem 4.6,
´ [ab + ad + bc + cd - 2bd - 2ac - (a - c)(b - d)]
l 2 + l - 2 < 0 and (l + 2)2 + 4(l2 + l - 2 ) < 0
= - 16(b - a)(d - c)(c - b)(d - a)
(l + 2)(l - 1) < 0 and 5l 2 + 8 l - 4 < 0
- 2 < l < 1 and (l + 2)(5l - 2) < 0 This is less than 0 if a < b < c < d. If a < b < c < d, then
D(l ) > 0 for all real l.
2
- 2 < l < 1 and -2 < l < Answer: (A)
5
These inequalities imply 22. If a and b are the roots of x2 + bx + c = 0 and are
positive, then a + b is
æ 2ö
l Î ç - 2, ÷ (A) b + 2 c (B) -b + 2 c
è 5ø
(C) b + 2 c (D) 2b - c
Answer: (B)
Solution: Since a and b are the roots of x2 + bx + c = 0,
20. At least one of the equations x + ax + b = 0 and
2
we have
x2 + cx + d = 0 has real roots if
a + b = - b and ab = c
(A) ac = 2(b + d) (B) ad = 2(b + c)
(C) bc = 2(a + d) (D) ab = 2(c + d) Therefore
Solution: Suppose both the equations have imaginary
( a + b )2 = a + b + 2 ab = - b + 2 c
roots and ac = 2(b + d). Then a2 - 4b < 0 and c 2 - 4d < 0.
Therefore
a + b = -b + 2 c
a + c - 4(b + d) < 0
2 2
Answer: (B)
a + c - 2ac < 0
2 2

23. Let a be a root of ax2 + bx + c = 0 and b be a root


(a - c ) < 0
2
of -ax2 - bx + c = 0, where a, b and c are real numbers
which is impossible. Therefore ac = 2(b + d) implies that and a ¹ 0. Then the equation
at least one of a2 - 4b and c2 - 4d is greater than or equal a 2
to 0. x + bx + c = 0
2
Answer: (A)
has a root g such that
21. For any real l, the quadratic equation (x - a)(x - c) + (A) g < min{a , b }
l( x - b)( x - d) = 0 has always real roots if (B) g > max{a , b }
(A) a < b < c < d (B) a < c < b < d (C) g lies between a and b
(C) a < c < d < b (D) d < c < b < a (D) -g lies between a and b
Solution: The given equation is Solution: By hypothesis,
(1 + l ) x2 - (a + c + lb + ld) x + (ac + lbd) = 0 aa 2 + ba + c = 0 and ab 2 - bb - c = 0
This equation has real roots if the discriminant Let
D(l ) = (a + c + lb + ld) - 4(1 + l )(ac + lbd) ³ 0
2
a 2
f ( x) = x + bx + c
for any l. That is 2

D(l ) = (b - d)2 l 2 + 2(ab + ad + bc + dc - 2bd - 2ac)l Then


+ (a - c)2 ³ 0 for any real l a 2
f (a ) = a + ba + c
D(0) = (a - c) > 0 for a ¹ c
2
2
www.jeeneetbooks.in
Worked-Out Problems 185

1 26. The number of solutions of the equation


= (aa 2 + 2ba + 2c) | x |2 - 2 | x | - 8 = 0 which belong to the domain of the
2
function f ( x) = 5 - 2 x is
1 a
= (aa 2 - 2aa 2 ) = - a 2 (A) 0 (B) 1 (C) 2 (D) 3
2 2
Solution: The domain of f = {x | x £ 5 / 2}
a
and f (b ) = b 2 + bb + c | x |2 - 2 | x | - 8 = (| x | - 4)(| x | + 2) = 0
2
Þ | x | = 4 (since | x | + 2 > 0)
1
= (ab 2 + 2bb + 2c) Þ x = 4 or - 4
2
Now -4 < 5 / 2 and –4 belongs to the domain of f.
1
= (a b 2 + a b 2 ) = a b 2 Answer: (B)
2
Therefore, 27. The least integral value of k for which the quadratic
expression (k - 2) x2 + 8 x + k + 4 is positive for all
-a a b
2 2 2
real x is
f (a ) f (b ) = <0
2 (A) 4 (B) −6 (C) 5 (D) 6
Hence f (x) = 0 has a root in between a and b. Solution: Let f(x) = (k - 2)x2 + 8x + k + 4. From Theorem
4.6 we have
Answer: (C)
f ( x) > 0 for all real x Þ discriminant < 0 and coefficient
24. The number of equations of the form ax + bx + 1 = 0, 2
of x2 > 0
where a, b Î {1, 2, 3, 4}, having real roots is
This implies
(A) 15 (B) 9 (C) 7 (D) 8
64 - 4(k - 2)(k + 4) < 0 and k > 2
Solution: The roots are real Û b2 - 4a ³ 0 Û b2 ³ 4a. In
k2 + 2k - 24 > 0 and k > 2
tabular form
(k + 6)(k - 4) > 0 and k > 2
No. of required k > 4 and k is an integer
a 4a b b2 equations
Therefore the least integral value of k is 5.
1 4 2, 3, 4 4, 9, 16 3 Answer: (C)
2 8 3, 4 9, 16 2
3 12 4 16 1 28. If the roots of the quadratic equation ( p - 3) x2 -
2 px + 5 p = 0 are real and positive, then
4 16 4 16 1
Total 7 (A) p > 0 (B) 3 £ p £ 15 /4
(C) 3 < p £ 15 /4 (D) p > 15 /4
Answer: (C) Solution: Let f ( x) = ( p - 3) x2 - 2 px + 5 p. The roots of
f ( x) = 0 are real. This implies
25. If x2 + (a - b) x + 1 - a - b = 0 , where a and b are real
numbers, has distinct real roots for all values of b, 4 p2 - 20 p( p - 3) ³ 0
then 4 p2 - 15 p £ 0
(A) a < 1 (B) a > 1 p(4 p - 15) £ 0
(C) a < 0 (D) 0 < a < 1
15
Solution: We have 0£ p£ (4.17)
4
(a - b)2 - 4(1 - a - b) > 0 for all real b Now
Þ b + 2(2 - a)b + a - 4(1 - a) > 0 for all reeal b
2 2 1. p = 0 implies that the roots are 0, 0.
2. Roots are positive implies that f (0) and the coeffi-
Þ 4(2 - a)2 - 4(a2 - 4 + 4a) < 0
cient of x2 must have the same sign. f (0) = 5 p and
Þ - 16a + 16 + 16 - 16a < 0 p - 3 have the same sign and p ¹ 3.
Þa>1 By Eq. (4.17), we have 3 < p £ 15 /4.
Answer: (B) Answer: (C)
www.jeeneetbooks.in
186 Chapter 4 Quadratic Equations

29. If a and b are the roots of the equation (5 + 2 ) x2 - and coefficient of x2 = 1 > 0. Therefore a and b must lie
(4 + 5 ) x + 8 + 2 5 = 0, then between the roots and a < b. Hence one root is less than
a and another is greater than b.
2 Answer: (D)
(1/a ) + (1/b )
32. The number of real solutions of the equation
is equal to
(x2 - 5x + 7)2 - (x - 2)(x - 3) = 0 is
(A) 2 (B) 4 (C) 1/2 (D) 1/4
(A) 1 (B) 2 (C) 3 (D) 0
Solution: Since a and b are the roots of the given
Solution: The given equation is
equation we have
( x2 - 5 x + 7)2 - ( x2 - 5 x + 6) = 0
4+ 5 8+2 5
a+b= and ab = Put x2 - 5 x + 7 = t. Then
5+ 2 5+ 2 2
æ 5ö 3
t = çx - ÷ø + > 0 for all x Î 
Therefore è 2 4

a+b 4+ 5 1 The given equation is equivalent to


= =
ab 2(4 + 5) 2 t2 - (t - 1) = 0

2 2ab t2 - t + 1 = 0
= =2´2=4
(1/a ) + (1/b ) a + b 1± i 3
t=
Answer: (B) 2
Therefore there is no real root of the given equation.
30. If a < b are the roots of the equation x2 + bx + c = 0,
where c < 0 < b, then Answer: (D)
(A) 0 < a < b (B) a < 0 < b < |a |
33. The number of real values of x satisfying the equation
(C) a < b < 0 (D) a < 0 < |a | < b
1 1 æ 1ö
Solution: Since a < b are the roots of the equation x3 + + x2 + 2 - 6 ç x + ÷ -7=0
x3
x è xø
x2 + bx + c = 0,
a + b = -b < 0 (since 0 < b ) is
(A) 1 (B) 2 (C) 3 (D) 4
and ab = c < 0
Solution: Put x + (1/x) = t. Then we have
Since a < b and ab < 0, we get that
1 1
x2 + = t2 - 2 and x3 + = t3 - 3t
a <0<b (4.18) x2
x3

Also, b = - b - a < - a . Therefore b < -a and a < b and Therefore the given equation transforms into
hence (t3 - 3t ) + (t2 - 2) - 6t - 7 = 0
|a | > b (4.19) t3 + t2 - 9 t - 9 = 0
From Eqs. (4.18) and (4.19), we get (t + 1)(t2 - 9) = 0
a < 0 < b < |a | Equating t = -1 and substituting the value to t back, we
Answer: (B) get
1
31. If a < b, then the equation ( x - a)( x - b) - 1 = 0 has x+ = - 1 Þ x2 + x + 1 = 0 Þ x is not real
x
(A) both the roots in [a, b]
Now for t2 = 9 we get
(B) both the roots in (-¥, a)
(C) both the roots in (b, +¥) t = ± 3 Þ x2 ∓ 3 x + 1 = 0
(D) one root in (-¥, a) and another in (b, +¥) This gives
Solution: Let f ( x) = ( x - a)( x - b) - 1. Then
3± 5 -3 ± 5
x= and x=
f (a) = - 1 = f (b) 2 2
www.jeeneetbooks.in
Worked-Out Problems 187

Therefore four real values of x satisfy the given equation. x2 + x + 2 + x > 0


Answer: (D) ( x + 1)2 + 1 > 0

34. The number of quadratic equations, with coefficient of which is always true. Therefore any x Î (-¥, - 2) satisfies
2
x as 1, which are unaltered by squaring their roots is the given inequality.
(A) 2 (B) 4 Case 2: Suppose that x ³ -2. Then
(C) 6 (D) infinite x2 - ( x + 2) + x > 0 Û x2 - 2 > 0
Solution: Let a and b be the roots of x2 + bx + c = 0 Ûx<- 2 or x> 2
for which a 2 and b 2 are also roots. But the equation whose
roots are a 2 and b 2 is x2 - (b2 - 2c) x + c2 = 0. Therefore Therefore, the required set is (-¥, - 2 ) È ( 2 , ¥).
b2 - 2c = - b and c = c (i.e., c = 0 or 1).
2
Answer: (B)
(i) If c = 0, then b2 = - b and hence b = 0 or −1.
37. The smallest value of k for which both roots of
(ii) If c = 1, then b2 + b - 2 = 0 and hence b = 1 or −2. the equation x2 - 8kx + 16(k2 - k + 1) = 0 are real,
Therefore the required equations are distinct and have values at least k is
(A) 0 (B) 1 (C) 3/2 (D) 2
x2 = 0, x2 - x = 0, x2 + x + 1 = 0, x2 - 2 x + 1 = 0
Answer: (B) Solution: Roots are real and distinct. Therefore discrim-
inant > 0. That is
35. The quadratic equations x2 - 6 x + a = 0 and x2 - cx + 64k2 - 64(k2 - k + 1) > 0
6 = 0 have one root in common. The other roots of
k >1 (4.20)
the first equation and the second equation are inte-
gers in the ratio 4 : 3. Then the common root is For k = 1, the given equation has roots 4, 4. Values of the
(A) 4 (B) 3 (C) 2 (D) 1 roots at least 4 implies that the product of the roots ³ 16.
Therefore
Solution: Let a be the common root and let the other
16(k2 - k + 1) ³ 16
roots of the equations be 4b and 3b, respectively. Then
k(k - 1) ³ 0
a + 4 b = 6, 4ab = a
k £ 0 or k ³ 1 (4.21)
and a + 3b = c, 3ab = 6
Roots ³ 4 Þ f (4) ³ 0
This implies
Þ 16 - 32k + 16(k2 - k + 1) ³ 0
6
a = 4ab = 4 × = 8 Þ (k - 1)(k - 2) ³ 0
3
The first equation is x2 - 6 x + 8 = 0 whose roots are 2 Þ k £ 1 or k ³ 2 (4.22)
and 4. From Eqs. (4.20), (4.21) and (4.22), we get k ³ 2.
If a = 4, then Answer: (D)
6 1 3
b= = and 3b = 38. If
3a 2 2
x2 + mx + 1
which is not an integer, a contradiction to the hypothesis. -3 < <3
x2 + x + 1
Therefore, a = 2 is the common root, in which case the
equations are x2 - 6 x + 8 = 0 and x2 - 5 x + 6 = 0, whose for all real x, then
roots are 2, 4 and 2, 3, respectively. (A) m < -1 (B) - 1 < m < 6
Answer: (C) (C) - 1 < m < 5 (D) m > 6
Solution: We have
36. The set of all real values of x for which x2 - | x + 2 | +
2
x > 0 is æ 1ö 3
x2 + x + 1 = ç x + ÷ø + > 0
(A) (-¥, 2) È (2, ¥) (B) (-¥, - 2 ) È ( 2 , ¥) è 2 4
(C) (-¥, - 1) È (1, ¥) (D) ( 2 , ¥) Therefore
Solution: - 3( x2 + x + 1) < x2 + mx + 1 < 3( x2 + x + 1)
Case 1: Suppose that x < -2. Then the given inequality is
www.jeeneetbooks.in
188 Chapter 4 Quadratic Equations

4 x2 + (m + 3) x + 4 > 0 and 2 x2 + (3 - m) x + 2 < 0 Solution: x2 - 8 x - n(n - 10) = 0 has no real solutions.


for all real x This implies
(m + 3)2 - 64 < 0 and (3 - m)2 - 16 < 0 64 + 4 n(n - 10) < 0
(m + 3 + 8)(m + 3 - 8) < 0 and (3 - m + 4)(3 - m - 4) < 0 n2 - 10 n + 16 < 0
(m + 11)(m - 5) < 0 and (m - 7)(m + 1) < 0 (n - 8)(n - 2) < 0
- 11 < m < 5 and - 1 < m < 7 2<n<8
This gives - 1 < m < 5. Therefore, n = 3, 4, 5, 6, 7. Also, when n = 3, 4, 5, 6 or 7,
it can be seen that ( x - 8) x = n (n - 10) has no real solu-
Answer: (C)
tions. Therefore the number of such n is 5.
39. The number of natural numbers n for which the Answer: (D)
equation ( x - 8) x = n(n - 10) has no real solutions is
(A) 2 (B) 3 (C) 4 (D) 5

Multiple Correct Choice Type Questions


1. Suppose a and b are integers and b ¹ -1. If the quadratic Solution:
equation x2 + ax + b + 1 = 0 has a positive integer root, Case 1: Suppose b is even, that is, b = 2 m. Then b2 - 4ac =
then 4(m2 - ac) = 4k.
(A) the other root is also a positive integer Case 2: Suppose b is odd, that is, b = 2 m - 1. Then
(B) the other root is an integer
b2 - 4ac = (2 m - 1)2 - 4ac
(C) a2 + b2 is a prime number
(D) a2 + b2 has a factor other than 1 and itself = 4 m2 + 4 m + 1 - 4ac
Solution: Let a and b be the roots and a be a positive = 4(m2 + m - ac) + 1
integer. Then
= 4k + 1
a + b = -a and ab = b + 1 Answers: (A), (B)
b = -a - a implies b is an integer and
3. If a and b are roots of the equation x2 + ax + b = 0,
a2 + b2 = (a + b )2 + (ab - 1)2 then
= a2 + b2 + a2 b2 + 1 (A) a = 0, b = 1 (B) a = 0 = b
= (a + 1)(b + 1)
2 2 (C) a = 1, b = - 1 (D) a = 1, b = - 2
Solution: If a + b = -a and ab = b, then a = 0 = b or a = 1,
Since a + 1 > 1 and b 2 + 1 > 1, it follows that a 2 + 1 is a
2
b = -2.
factor of a2 + b2 other than 1 and itself.
Answers: (B), (D)
Answers: (B), (D)
Note: If b = -1, then a2 + b2 may be prime number; for 4. Let a, b and c be real numbers and a ¹ 0. Let a and b
example, the equation x2 - 2x = 0 has a positive root 2 and be the roots of ax2 + bx + c = 0 . If a ¢ and b ¢ are roots
the other root is 0. Here a2 + b2( = 5) is a prime number. of the equation a3 x2 + (abc) x + c3 = 0, then
In fact, a2 + b2 is prime implies a 2 + 1 = 1 or b 2 + 1 = 1. (A) a ¢ = a 3 b 2 (B) b ¢ = b 3a 2
This gives (C) a ¢ = a b
2
(D) b ¢ = ab 2
a = 0 or b=0 Solution: Since a and b are the roots of ax2 + bx + c = 0,
or b = -1 we have

b c
a+b=- and ab =
2. If a, b and c are integers, then the discriminant of a a
ax2 + bx + c is of the form (where k is an integer)
Also since a ¢ and b ¢ are roots of the equation a3x2 + (abc)
(A) 4k (B) 4k + 1
(C) 4k + 2 (D) 4k + 3 x + c3 = 0,
www.jeeneetbooks.in
Worked-Out Problems 189

-abc æ -b ö æ c ö Now
a¢+ b¢= = ç ÷ ç ÷ = (a + b )ab
a3 è a ø è aø (a + 1)(b + 1) = ab + a + b + 1
3
c = - ( p + q) + p + 1 = 1 - q
a ¢b ¢ = = (ab )3
a3
(a + 1)2 (b + 1)2 (a + 1)2
Now + =
(a + 1)2 + q - 1 (b + 1)2 + q - 1 (a + 1)2 - (a + 1)(b + 1)
(a ¢ - b ¢)2 = (a ¢ + b ¢)2 - 4a ¢b ¢
(b + 1)2
= (a + b ) a b - 4a b
2 2 2 3 3 +
(b + 1)2 - (a + 1)(b + 1)
= (ab )2 [(a + b )2 - 4ab ] a +1 b +1
= +
= (ab ) (a - b )
2 2
a - b b -a
Also (a + 1) - (b + 1)
= =1
|a ¢ - b ¢ | = |ab (a - b )| a-b
Therefore Answers: (A), (D)
a ¢ - b ¢ = ab (a - b ) Þ a ¢ = a 2 b and b ¢ = ab 2
7. Let a, b and c be real numbers and f ( x) = ax + bx + c.
2

a ¢ - b ¢ = - ab (a - b ) Þ a ¢ = ab 2 and b ¢ = a 2 b Suppose that whenever x is an integer, f ( x) is also an


Answers: (C), (D) integer. Then
(A) 2a is an integer (B) a + b is an integer
5. If a and b are roots of the equation x2 - 2ax + b2 = 0 (C) c is an integer (D) a + b + c is an integer
and g and d are the roots of the equation x2 - 2bx +
a2 = 0, then Solution: By hypothesis, f(-1), f(0) and f(1) are integers.
(A) a + b = 2 gd (B) a + b = 2(g + d ) Therefore a(-1)2 + b(-1) + c, a(0)2 + b(0) + c and a(1)2 +
(C) (g + d ) = 4ab
2
(D) (a + b )(g + d ) = 4gd b(1) + c are all integer. Hence

Solution: Since a and b are roots of the equation a - b + c, c and a + b + c are integers
x2 - 2ax + b2 = 0, we have Also
a + b = 2a and ab = b 2
a - b and a + b are integers Þ 2a is an integer
Since g and d are roots of the equation x - 2bx + a = 0, 2 2
Answers: (A), (B), (C), (D)
we have
g + d = 2b and gd = a2 8. If one root of the equation 3 x + px + 3 = 0 is the
2

square of the other, then p is equal to


Solving the two sets of equations we get
(A) 1/3 (B) 1 (C) 3 (D) −6
a + b = 2 gd
Solution: Let a and a be the roots of 3 x + px + 3 = 0.
2 2

(g + d )2 = 4b2 = 4ab Then


Answers: (A), (C)
-p
a + a2 = and a3 = 1
6. If a and b are the roots of x - p( x + 1) - q = 0, then
2 3
(A) (a + 1)(b + 1) = 1 - q Therefore
(B) (a + 1)(b + 1) = 1 + q
-1 + i 3 -1 - i 3
(a + 1)2 (b + 1)2 a = 1, a = w = or a=
(C) + =q 2 2
(a + 1) + q - 1 (b + 1)2 + q - 1
2

(i) If a = 1, then p = -6 so that the equation is 3x2 - 6x +


a 2 + 2a + 1 b 2 + 2 b + 1
(D) 2 + =1 3 = 0 whose roots are 1, 1.
a + 2a + q b 2 + 2 b + q
(ii) If a = w or w2, then p = -3(a + a2) = -3(w + w2) =
Solution: Since a and b are the roots of x2 - p( x + 1) - -3(-1) and hence p = 3 so that the equation is
q = 0, we have 3 x2 + 3 x + 3 = 0, whose roots are w and w2.
a + b = p and ab = - ( p + q) Answers: (C), (D)
www.jeeneetbooks.in
190 Chapter 4 Quadratic Equations

9. Suppose that the three quadratic equations ax2 -2bx + a2 = bc, b2 = ca and c2 = ab
c = 0, bx - 2cx + a = 0 and cx - 2ax + b = 0 all have
2 2
a3 = b3 = c3 = abc
only positive roots. Then
(A) b2 = ca (B) c2 = ab a=b=c
(C) a = bc
2
(D) a = b = c Answers: (A), (B), (C), (D)
Solution: Let a > 0 and b > 0 be the roots of ax - 2bx +
2
10. Let a and b be two real numbers. If the roots of the
c = 0. Then equation x2 - ax - b = 0 have absolute values less
c than 1, then
= ab > 0
a (A) | b | < 1 (B) a + b < 1
(C) b - a < 1 (D) a + b = 0
and therefore a and c have the same sign. Similarly,
b and c have the same sign and a and b have the same Solution: Let a and b be the roots of x2 - ax - b = 0.
sign. Therefore, a, b and c have the same sign and Then, |a | < 1 and | b | < 1. Also
hence ab > 0. Also (-2b)2 ³ 4ac, that is, b2 ³ ac. Similarly
| b | = | - b | = |ab | = |a || b | < 1
c2 ³ ab and a2 ³ bc. Hence
Since the roots a and b lie between –1 and 1, we have
b2 c2 ³ a2 bc, c2 a2 ³ b2 ca and a2 b2 ³ c2 ab f (- 1) > 0 and f (1) > 0. Therefore
which gives
1 + a - b > 0 and 1 - a - b > 0
bc ³ a2, ca ³ b2 and ab ³ c2 (since ab, bc
or b - a < 1 and a + b < 1
and ca are all positive)
Answers: (A), (B), (C)
But we have a2 ³ bc, b2 ³ ca and c2 ³ ab. Therefore

Matrix-Match Type Questions


1. Match the items in Column I with those in Column II. (a + 1)2 4(a - 1) (a - 1)2
- =
4 2 4
Column I Column II (a + 1)2 - 8(a - 1) - (a - 1)2 = 0

(A) If the difference of the roots of the (p) 1 a - 2(a - 1) = 0


equation 2 x2 - (a + 1) x + (a - 1) = 0 a=2
is equal to their product, then the
Answer: (A) Æ (r)
value(s) of a is (are) (q) 0 (B) Let a and b be the roots of x2 - 2a(x - 1) - 1 = 0.
(B) If the sum of the roots of the equation
Then a + b = 2a and ab = 2a - 1. Now
x2 - 2a( x - 1) - 1 = 0 is equal to the
sum of their squares, then a is a 2 + b 2 = a + b Þ (a + b )2 - 2ab = a + b
(r) 2
(C) If one root of the equation Þ (2a)2 - 2(2a - 1) = 2a
x2 - x - 3m = 0 (m ¹ 0) is twice one
of the roots of x2 - x - m = 0, then Þ 4a2 - 6a + 2 = 0
the value of m is (s) 1/2
Þ 2a2 - 3a + 1 = 0
(D) If the sum of the squares of the roots
Þ (2a - 1)(a - 1) = 0
of the equation x2 - 4 x + m = 0 is
equal to 16, then m is (t) –1 1
Þa= ,1
2
Solution: Answer: (B) Æ (p), (s)
(A) Let a and b be the roots of 2x2 - (a + 1)x + (a - 1) = 0. (C) Let a be one root of x2 - x - m = 0 and 2a be a root
Then of x2 - x - 3m = 0. Then
a+1 a-1 a 2 - a - m = 0 and (2a )2 - (2a ) - 3m = 0
a+b= and ab =
2 2 Eliminating m, we have a = 0, - 1. Also a = 0 Þ m = 0,
Now a contradiction to hypothesis. Therefore, a = -1 and
m = 2.
|a - b |2 = (ab )2 Þ (a + b )2 - 4ab = (ab )2 Answer: (C) Æ (r)
www.jeeneetbooks.in
Worked-Out Problems 191

(D) Let a and b be the roots of x2 - 4 x + m = 0. Then a+c 2c b


+ =-
a + b = 4 and ab = m. Now 2a a+c a

a 2 + b 2 = 16 Þ (a + b )2 - 2ab = 16 (a + c)2 + 4ac b


=-
Þ 16 - 2 m = 16 2a(a + c) a

Þm=0 a2 + c2 + 6ac = - 2bc - 2ab


Answer: (D) Æ (q) Adding b2 to both the sides and splitting 6ac, we get
a2 + b2 + c2 + 2ab + 2bc + 2ca = b2 - 4ca
2. Match the items in Column I with those in Column II.
(a + b + c)2 = b2 - 4ca
Answer: (A) Æ (s)
Column I Column II
(B) Let a and a be the roots of ax + bx + c = 0. Then
2 2

(A) If the roots of the equation 1


(p) (a2 + b2 ) -b c
ax2 + bx + c = 0 are of the form 2 a + a2 = and a3 =
(k + 1)/ k and (k + 2)/(k + 1), a a
then (a + b + c)2 = Therefore
(B) If one root of the equation (q) 2ac 2/3 1/ 3
ax2 + bx + c = 0 is the square of æ cö æ cö -b
çè ÷ø +ç ÷ =
the other, then b3 + ac2 + a2 c = a è aø a
(C) If the sum of the roots of the 2 2/3 1/ 3
éæ c ö 2 / 3 æ c ö 1/ 3 ù -b3
equation ax2 + bx + c = 0 æ cö c æ cö æ cö
(r) 3abc çè ÷ø + + 3 çè ÷ø çè ÷ø êç ÷ + ç ÷ ú = 3
is equal to the sum of their a a a a êëè a ø è a ø úû a
squares, then b(a + b) = 2
æ cö c c æ -b ö b3
(D) If the roots of the equation çè ÷ø + + 3 çè ÷ø = - 3
(s) b2 - 4ac a a a a a
1 1 1
+ = c2 + ca - 3bc -b3
x+a x+b c = 3
a2 a
are equal in magnitude, but
opposite in sign, then the 1 b3 + ac2 + ca2 = 3abc
(t) - (a2 + b2 )
product of the roots is 2 Answer: (B) Æ (r)
(C) Let a and b be the roots of ax + bx + c = 0, then 2

Solution:
b c
(A) Since (k + 1)/ k and (k + 2)/(k + 1) are the roots of a+b=- and ab =
the given equation we have a a
Therefore
k+1 k+2 b
+ =- (4.23)
k k+1 a b b2 2c
- = a + b = a 2 + b 2 = (a + b )2 - 2ab = 2 -
a a a
k+2 c
and = (4.24) This gives
k a
-ab = b2 - 2ac
From Eq. (4.24),
2ca = b2 + ab
c k+2 2
-1 = -1 = Answer: (C) Æ (q)
a k k
(D) The given equation is equivalent to
and hence
x2 + [a + b - 2c]x + ab - bc - ca = 0
2a If a and −a are the roots of this, then
k=
c-a
0 = a + (-a ) = 2c - a - b and - a 2 = ab - bc - ca
Substituting this value for k in Eq. (4.23), we get
Therefore
[2a /(c - a)] + 1 [2a /(c - a)] + 2 b a + b = 2c and - a 2 = ab - c(a + b)
+ =-
2a /(c - a) [2a /(c - a)] + 1 a
www.jeeneetbooks.in
192 Chapter 4 Quadratic Equations

The product of the roots is (C) We have


(a + b)2 æ a2 + b2 ö | x - 1 - x2 | £ | x2 - 3 x + 4 |
ab - c(a + b) = ab - =-ç
2 è 2 ÷ø
Þ | x2 - x + 1| £ | x2 - 3x + 4 |
Answer: (D) Æ (t)
Þ x2 - x + 1 £ x2 - 3x + 4 (since both are positive
3. Match the items in Column I with those in Column II. for all real x)
3
Column I Column II Þx£
2
(A) The values of k for which (p) (2, 5) Therefore
both the roots of the equation
x2 - 6kx + 2 - 2k + 9k2 = 0 are æ 3ö æ 3ö
(q) ç -¥, ÷ x Î ç -¥, ÷
greater than 3 belong to è 2ø è 2ø
(B) If log 0.1( x2 + x) > log 0.5( x3 - x) + Answer: (C) Æ (q)
log2 ( x - 1), then x belongs to (r) (1, +¥)
(D) We have
(C) If | x - 1 - x2 | £ | x2 - 3 x + 4 |, then x | x2 - 2 x - 3 | < 3 x - 3 Þ |( x - 3)( x + 1)| < 3 x - 3
(s) æç , + ¥ö÷
11
belongs to
è 9 ø Case 1: x < −1. Then
(D) If | x2 - 2 x - 3 | < 3 x - 3, then x lies
in the interval (t) [2, 5] ( x - 3)( x + 1) < 3 x - 3 Û x2 - 5 x < 0 Û 0 < x < 5
However, x < −1.
Solution: Case 2: −1 < x < 3. Then
(A) Let f(x) = x2 - 6kx + 2 - 2k + 9k2 and a and b be the
roots of f (x) = 0. Then, since 3 < a and 3 < b, we have (3 - x)( x + 1) < 3 x - 3 Û - x2 + 2 x + 3 < 3 x - 3
6k > 6 and therefore k > 1. Also Û 0 < x2 + x - 6
f (3) > 0 Û ( x + 3)( x - 2) > 0
9k - 20k + 11 > 0
2
Therefore, x > 2. Hence
(9k - 11)(k - 1) > 0 x Î (2, +¥)
11 Case 3: x ³ 3. Then
k < 1 or k >
9
( x - 3)( x + 1) < 3x - 3 Û x2 - 2 x - 3 < 3x - 3
Since k > 1, it follows that k > 11/9.
Û x2 - 5x < 0
Answer: (A) Æ (s)
Û0<x<5
(B) The inequality is defined for x > 1. Since
Û3£ x< 5 (∵ x ³ 3)
log 2 ( x - 1) = log ( 0.5)-1 ( x - 1) = - log 0.5( x - 1)
From the above two cases x Î (2, 5)
we have
Answer: (D) Æ (p)
log 0.1( x2 + x) > log 0.5( x3 - x) - log 0.5( x - 1)
4. Match the items in Column I with those in Column II.
Therefore
æ x3 - x ö Column I Column II
log 0.1( x2 + x) > log 0.5 ç = log 0.5( x2 + x)
è x - 1 ÷ø
x 1
(A) If £ , then x belongs to (p) (−¥, +¥)
which further gives x-3 x
x2 + x > 1 Þ x2 + x - 1 > 0 x2 + 6 x - 7 (q) (0, 2)
(B) £ 2 for all x belonging to
x2 + 1
æ 1 + 5ö 5-1 (r) [1, 6]
x < -ç or x> and x>1 ( x - 1)2 ( x + 1)3
÷ (C) £ 0 for all x in
è 2 ø 2 x4 ( x - 2)
(s) [−1, 0]
This implies x Î (1, ¥) 3 x2 - 7 x + 8
(D) 1 < £ 2 for all x in
Answer: (B) Æ (r) x2 + 1 (t) (0, 3)
www.jeeneetbooks.in
Worked-Out Problems 193

Solution: Now f ( x) = 0 Û x = 1, - 1. To determine the change


(A) Let of sign of f (x), we have to consider the points −1, 0, 2.
Case 1: x < -1 Þ (x + 1)3 < 0 and x - 2 < 0. Therefore
x 1 x2 - x + 3 f (x) > 0 for all x < −1.
f ( x) = - =
x-3 x x( x - 3) Case 2: - 1 < x < 0 Þ x - 2 < 0 and f (x) < 0.
Observe that Case 3: 0 < x < 2 Þ x - 2 < 0 Þ f(x) < 0. Therefore
2
f (x) £ 0 for all x Î (- 1, 0) È (0, 2).
æ 1ö 11
Answer: (C) Æ (q), (s)
x2 - x + 3 = ç x - ÷ + >0
è 2 ø 4
(D) We have
and hence f (x) ¹ 0 for all real x ¹ 0, 3. Therefore
3 x2 - 7 x + 8
f ( x) < 0 Û x( x - 3) < 0 Û 0 < x < 3 1< £2
x2 + 1
Answer: (A) Æ (t)
Û x2 + 1 < 3 x2 - 7 x + 8 £ 2( x2 + 1)
(B) Let
Û x2 + 1 < 3 x2 - 7 x + 8 and 3 x2 - 7 x + 8 £ 2 x2 + 2
x + 6x - 7
2
f ( x) = Û 2 x2 - 7 x + 7 > 0 and x2 - 7 x + 6 £ 0
x2 + 1
Note that x2 + 1 > 0 for all x. Now, Note that
2
æ 7ö 7
f ( x) £ 2 Û x2 + 6 x - 7 £ 2( x2 + 1) 2 x2 - 7 x + 7 = 2 ç x - ÷ + > 0 for all x
è 4ø 8
Û x2 - 6 x + 9 ³ 0
x2 - 7 x + 6 £ 0 Û ( x - 1)( x - 6) £ 0 Û 1 £ x £ 6
Û ( x - 3) ³ 02
Therefore, both the inequalities hold for 1 £ x £ 6.
which is true for all x. Also f (3) = 2. Also note that
Answer: (B) Æ (p) 3 x2 - 7 x + 8
=2
(C) Let x2 + 1

( x - 1)2 ( x + 1)3 when x = 1, 6.


f ( x) = Answer: (D) Æ (r)
x4 ( x - 2)

Comprehension-Type Questions
1. Passage: Let f ( x) = ax + bx + c, where a, b and c are
2
Solution:
real and a ¹ 0. Let a < b be the roots of f ( x) = 0. Then (i) f(x) = x2 - mx + 1 and a £ b are the roots of f(x) = 0.
(a) for all x such that a < x < b , f ( x) and a have Now a < b < 1 implies that f (1) and the coefficients
opposite signs. of x2 have the same sign. This gives
(b) for x < a or x > b , f (x) and a have the same sign. 1 - m + 1 = f (1) > 0
Based on this, answer the following three questions.
m<2 (4.25)
(i) If both the roots of the equation
x2 - mx + 1 = 0 are less than unity, then Also, discriminant is m - 4 ³ 0. Therefore
2

(A) m £ -2 (B) m > 2 m £ - 2 or m ³ 2 (4.26)


(C) - 1 £ m £ 3 (D) 0 £ m £ 5 / 2 From Eqs. (4.25) and (4.26), m £ -2 . Also, note that
(ii) If both the roots of the equation x2 - 6 mx + if m = -2, the roots are - 1, - 1.
9 m2 - 2 m + 2 = 0 are greater than 3, then Answer: (A)
(A) m < 0 (B) m > 1 (ii) Let f ( x) = x - 6 mx + 9 m - 2 m + 2. Let a > b > 3 be
2 2

(C) 0 < m < 1 (D) m > 11/ 9 the roots of f(x) = 0. Then 6 < a + b = 6 m and hence
(iii) If both the roots of the equation 4x2 - 2x +
m>1 (4.27)
m = 0 belong to the interval (-1,1), then
(A) - 3 < m < - 2 (B) 0 < m < 2 Also 9 m2 - 2 m + 2 = ab > 9. Therefore
(C) 2 < m < 5 / 2 (D) - 2 < m £ 1/ 4 9 m2 - 2 m - 7 > 0
www.jeeneetbooks.in
194 Chapter 4 Quadratic Equations

2. Passage: Let f (x) = ax + bx + c, where a, b and c are


2
(9 m + 7)(m - 1) > 0
real numbers and a ¹ 0. If b2 - 4 ac < 0, then for all
This gives real x, f (x) and a will have the same sign. If a < b are
-7 real roots of f ( x) = 0, then
m< or m > 1 (4.28)
9 (a) f ( x) and a are of opposite sign for all x, a < x < b.
(b) f ( x) and a are of same sign for all x such that
Also, f (3) and the coefficient of x2 have the same
x < a or x > b.
sign. Therefore, f (3) > 0. This gives
Answer the following questions.
9 - 18 m + 9 m2 - 2 m + 2 > 0
(i) If (a - 1) x2 - (a + 1) x + (a + 1) > 0 for all real x,
9 m2 - 20 m + 11 > 0 then
(9 m - 11)(m - 1) > 0 (A) a < -5 / 3 (B) - 5 / 3 < a < 5 / 3
(C) a < 5 / 3 (D) a > 5 / 3
11 (ii) If (a + 4) x - 2 ax + 2a - 6 < 0 for all real x, then
2
m < 1 or m > (4.29)
9
(A) a < -6 (B) - 6 < a < 0
From Eqs. (4.27)–(4.29), we get (C) - 6 < a < 6 (D) a > 6
11 (iii) If the roots of the equation (2 - x)(x + 1) = a are
<m real and positive, then
9
(A) a < -2 (B) - 2 < a < 2
Answer: (D) (C) 2 < a £ 9 / 4 (D) 9 / 4 < a < 17 / 4
(iii) Let a, b, where a £ b, be the roots of 4x 2 -
2x + m = 0. Then - 1 < a , b < 1 and Solution:
(i) Let f(x) º (a - 1)x2 - (a + 1)x + a + 1. Then the discri-
1 m minant is given by
a+b= , ab =
2 4 (a + 1)2 - 4 (a2 - 1) = - 3a2 + 2a + 5
Now f (-1) and the coefficient of x2 have the same f ( x) > 0 for all real x, this implies
sign. Therefore f (- 1) > 0 and hence 4 + 2 + m > 0,
- 3a2 + 2a + 5 < 0
that is
3a2 - 2a - 5 > 0
m > -6 (4.30)
(3a - 5)(a + 1) > 0
Also, f (1) > 0 Þ 4 - 2 + m > 0. This implies
5
m > -2 (4.31) a < - 1 or a > (4.33)
3
The discriminant is 4 - 16 m ³ 0. Therefore f ( x) and a - 1 are of same sign. This implies
1 a -1>0Þa >1 (4.34)
m£ (4.32)
4 From Eqs. (4.33) and (4.34), we have
From Eqs. (4.30)–(4.32), we get 5
a>
1 3
-2 < m £
4 Answer: (D)
If m = 1/ 4 , then the given equation is (ii) Let f ( x) = (a + 4) x - 2ax + 2a - 6. Now f ( x) < 0 for
2

all real x, implies


1 4a2 - 8 (a + 4)(a - 3) < 0 and a+4<0
4 x2 - 2 x + =0
4
a < -4 and a2 - 2 (a2 + a - 12) < 0
16 x - 8 x + 1 = 0
2
a < -4 and (a + 6)(a - 4) > 0
Therefore the roots are 1/4, 1/4. If the roots are dis- a < -6
tinct, then
Answer: (A)
1
-2 < m < (iii) The given equation is equivalent to x - x + a - 2 = 0.
2

4 Let f ( x) º x2 - x + a - 2 . Roots of f ( x) = 0 are real


Answer: (D) and positive. Therefore discriminant ³ 0. That is
www.jeeneetbooks.in
Worked-Out Problems 195

1 - 4 ( a - 2) ³ 0 9 - 4a < 1

9 a>2

4 Therefore,
Also, the roots of f ( x) = 0 are
9
2<a£
1 ± 1 - 4 (a - 2) 1 4
= (1 ± 9 - 4a )2
2 2 Note that, when a = 9 / 4, f ( x) = 0 takes the form
These are given to be positive. Therefore, 4 x2 - 4 x + 1 = 0 , whose roots are 1/ 2, 1/ 2 .
Answer: (C)
1 - 9 - 4a > 0

Assertion–Reasoning Type Questions


In the following set of questions, a Statement I is given 2. Statement I: If P( x) = ax2 + bx + c and Q( x) = - ax2 +
and a corresponding Statement II is given just below it. dx + c , where ac ¹ 0, then the equation P ( x) Q ( x) = 0
Mark the correct answer as: has at least two real roots.
(A) Both I and II are true and II is a correct reason for I Statement II: A quadratic equation with real coeffi-
( B ) Both I and II are true and II is not a correct reason cients has real roots if and only if the discriminant is
for I greater than or equal to zero.
( C ) I is true, but II is false Solution: Let px2 + qx + r = 0 be a quadratic equation.
( D) I is false, but II is true The roots are
1. Statement I: Let a, b and c be real numbers and - q ± q2 - 4 pr
a ¹ 0. If 4a + 3b + 2c and a have same sign, then not 2p
both the roots of the equation ax2 + bx + c = 0 belong
to the open interval (1, 2). These are real Û q - 4 pr ³ 0. Therefore Statement II
2

is true.
Statement II: A quadratic equation f ( x) = 0 will have In Statement I, ac ¹ 0. Therefore ac > 0 or ac < 0. If
a root in the interval (a, b) if f (a) f (b) < 0 . ac < 0, then b2 - 4ac > 0, so that P(x) = 0 has two real roots.
If ac > 0, then d2 + 4ac > 0 so that Q(x) = 0 has two real
Solution: Let f ( x) = px2 + qx + r . If f (a) and f (b) are
roots. Further, the roots of P(x) = 0 and Q(x) = 0 are also
of opposite sign, the curve (parabola) y = f ( x) must
the roots of P(x)Q(x) = 0. Therefore, Statement I is true
intersect x-axis at some point. This implies that f (x) has a
and Statement II is a correct reason for Statement I.
root in (a, b). Therefore, the Statement II is true.
Let a and b be roots of ax2 + bx + c = 0 . Then, Answer: (A)

-b c 3. Statement I: If a, b and c are real, then the roots of the


a+b= and ab = equation (x - a)(x - b) + (x - b)(x - c) + (x - c)(x - a) = 0
a a
are imaginary.
By hypothesis,
Statement II: If p, q and r are real and p ¹ 0 , then
4a + 3b + 2c the roots of the equation px2 + qx + r = 0 are real or
>0 imaginary according as q2 - 4 pr ³ 0 or q2 - 4 pr < 0.
a
æ bö æ cö Solution: Statement II is obviously true. In Statement I,
4 + 3ç ÷ + 2 ç ÷ > 0 the given equation is 3x2 - 2(a + b + c)x + ab + bc + ca = 0.
è aø è aø
The discriminant is
4 - 3 (a + b ) + 2 ab > 0
4(a + b + c)2 - 12(ab + bc + ca)
(ab - 2a - b + 2) + (ab - a - 2 b + 2) > 0
(a - 1)(b - 2) + (a - 2)(b - 1) > 0 (4.35) = 4[a2 + b2 + c2 - ab - bc - ca]

If 1 < a, b < 2, then (a - 1) (b - 2) + (a - 2) (b - 1) < 0 = 2 [(a - b)2 + (b - c) + (c - a)2 ] ³ 0


which is contradiction to Eq. (4.35). Therefore, at least Therefore, the equation has real roots. Statement I is
one root lies outside (1, 2). false and Statement II is true.
Answer: (B) Answer: (D)
www.jeeneetbooks.in
196 Chapter 4 Quadratic Equations

4. Statement I: Let f ( x) = x + ax + b , where a and b are


2
b2 - 4 ac q2 - 4 pr
integers. Then, for each integer n, there corresponds =
a2 p2
an integer m such that f (n) f (n + 1) = f (m).
Statement II: If a and b are roots of a quadratic
Statement II: If a and b are roots of x + px + q = 0 ,
2 equation f ( x) = 0, then the equation whose roots are
then x2 + px + q = ( x - a )( x - b ). a + h and b + h is f ( x - h) = 0 .
Solution: Let a¢ = a + d and b¢ = b + d. Then (a¢ - b¢ )2
Solution: Let a and b be roots of f ( x) = 0, where
= (a - b)2 . That is,
a and b may be imaginary. Then f ( x) º ( x - a )( x - b ),
a + b = - a and ab = b. Now, (a ¢ - b ¢)2 - 4a ¢b ¢ = (a - b )2 - 4 ab

f (n) f (n + 1) = (n - a )(n - b )(n + 1 - a )(n + 1 - b ) Therefore


2 2
= (n - a )(n + 1 - b )(n - b )(n + 1 - a ) æ -q ö æ r ö æ -b ö c
çè p ÷ø - 4 çè p ÷ø = çè a ÷ø - 4 a
= [n(n + 1) - n(a + b ) + ab - a ]
´ [n(n + 1) - n(a + b ) + ab - b ] q2 - 4 pr b2 - 4ac
=
p2 a2
= [n(n + 1) + an + b - a ][n(n + 1) + an + b - b ]
So, Statement I is true.
Put m = n(n + 1) + an + b. Then m is an integer and f (n) For Statement II, put y = x + h. Then x = y - h. Therefore
f (n + 1) = (m - a)(m - b) = f (m). a + h and b + h are the roots of f(y - h) = 0. By replacing y
Answer: (A) with x, Statement II is also true, but Statement II is not a
correct reason for Statement I.
5. Statement I: If one root of 2x2 - 2(2a + 1)x + a(a + 1) = 0 Answer: (B)
is less than a and the other root is greater than a, then
a Î (-¥ , -1) È (0, +¥). 7. Statement I: If a, b, c, d and p are distinct real numbers
such that
Statement II: If a < b are the roots of the equation
f ( x) º ax2 + bx + c = 0 , then for a < x < b , f (x) and a (a2 + b2 + c2 ) p2 - 2 (ab + bc + cd) p
have opposite signs.
+ (b2 + c2 + d2 ) £ 0
Solution: Roots are to be real and distinct. The discrim- then a, b, c and d are in geometric progression, that is
inant is
a b c
= =
4 (2a + 1) - 8a(a + 1) > 0
2
b c d
4a2 + 4a + 1 - 2a2 - 2a > 0 Statement II: Sum of squares of real numbers is always
2a + 2a + 1 = (a + 1) + a > 0
2 2 2 non-negative and equal to zero if and only if each of
the real numbers is zero.
Therefore a lies between the roots Þ f (a) and coefficient Solution: Statement II is obviously true. In Statement I,
of x2 are of opposite sign. Hence f (a) < 0 , which gives the given inequality can be written as
2a2 - 2(2a + 1) a + a(a + 1) < 0 (a2 p2 - 2 abp + b2 ) + (b2 p2 - 2 bcp + c2 )
a(a + 1) > 0 + (c2 d2 - 2 cdp + d2 ) £ 0
a < - 1 or a > 0 (ap - b)2 + (bp - c)2 + (cp - d)2 £ 0
a Î (- ¥, - 1) È (0, + ¥) and hence
Therefore both Statements I and II are correct and ap = b, bp = c, cp = d
Statement II is a correct reason for Statement I.
a b c 1
Answer: (A) or = = =
b c d p

6. Statement I: If a and b are roots of the equa tion Therefore, both Statements I and II are true and II is
ax + bx + c = 0 and a + d and b + d are roots of the
2 a correct reason for I.
equation px2 + qx + r = 0 , then Answer: (A)
www.jeeneetbooks.in
Exercises 197

SUMMARY
4.1 Quadratic expressions and equations: If a, b, c a
ax2 + bx + c = (a ¢x2 + b¢ x + c ¢)
are real numbers and a ≠ 0, the expression of the a¢
form ax2 + bx + c is called quadratic expression and
ax2 + bx + c = 0 is called quadratic equation. 4.8 Cube roots of unity: Roots of the equation x3 - 1 = 0
are called cube roots of unity and they are
4.2 Let f (x) º ax2 + bx + c be a quadratic expression
and a be a real (complex) number. Then we write -1 3
1, ±i
f (a) for aa2 + ba + c. If f(a) = 0, the a is called a zero 2 2
of f(x) or a root of the equation f(x) = 0. -1/ 2 ± i 3 / 2 are called non-real cube roots of unity.
Further each of them is the square of the other and
4.3 Roots: The roots of the quadratic equation ax + 2
the sum of the two non-real cube roots of unity is
bx + c = 0 are
equal to -1. If w ≠ 1 is a cube root of unity and n is
- b + b2 - 4ac - b - b2 - 4ac any positive integer, then 1 + wn + w2n is equal to 3
and or 0 according as n is a multiple of 3 or not.
2a 2a
4.9 Maximum and minimum values: If f(x) º ax2 +
4.4 Discriminant: b2 - 4ac is called the discriminant of
bx + c and a ≠ 0, then
the quadratic expression (equation) ax2 + bx + c = 0.
æ - b ö 4ac - b
2

4.5 Sum and product of the roots: If a and b are roots of fç ÷=


è 2a ø 4a
the equation ax2 + bx + c = 0, then
is the maximum or minimum value of f according
-b c as a < 0 or a > 0.
a+b= and a b =
2a a
4.10 Theorems (change of sign of ax + bx + c): Let f(x) º
2

4.6 Let ax + bx + c = 0 be a quadratic equation and


2
ax + bx + c where a, b, c are real and a ≠ 0. If
2

Δ = b2 - 4ac be its discriminant. Then the following a and b are real roots of f(x) = 0 and a < b, then
hold good. (1) (i) f(x) and a (the coefficient of x2) have the
(1) Roots are equal Û Δ = 0 (i.e., b2 = 4ac). same sign for all x < a or x > b.
(2) Roots are real and distinct Û Δ > 0. (ii) f(x) and a will have opposite signs for all x
(3) Roots are non-real complex (i.e., imaginary) Û such that a < x < b.
Δ > 0. (2) If f (x) = 0 has imaginary roots, then f(x) and a
will have the same sign for all real values of x.
4.7 Theorem: Two quadratic equations ax2 + bx + c = 0
and a ¢x2 + b¢ x + c ¢ = 0 have same roots if and only 4.11 If f(x) is a quadratic expression and f (p)f (q) < 0
if the triples (a, b, c) and (a¢, b¢, c¢ ) are proportional for some real numbers p and q, then the quadratic
and in this case equation f (x) = 0 has a root in between p and q.

EXERCISES
Single Correct Choice Type Questions
1. The roots of the equation
(C) ln = m2 + c / a (D) mn = l2 + bc / a
17
(10) 2 /x
+ (25) = (50)1/x
1 /x

4 3. If x is real, then the least value of


are 6 x2 - 22 x + 21
(A) 2, 1/2 (B) -2, 1/2 (C) 2, -1/2 (D) 1/2, -1/2 5 x2 - 18 x + 17
2. If a ¹ 0 and a(l + m) + 2blm + c = 0 and a(l + n) + is
2 2

2 bln + c = 0, then (A) 5/4 (B) 1 (C) 17/4 (D) -5/4


(A) mn = l + c / a
2
(B) lm = n + c / a
2
www.jeeneetbooks.in
198 Chapter 4 Quadratic Equations

4. The roots of the equation x2 - (m - 3) x + m = 0 are 14. A sufficient condition for the equation x2 + bx −
such that exactly one of them lies in the interval 4 = 0 to have integer roots is that
(1, 2). Then (A) b = 0, ±3 (B) b = 0, ±2
(A) 5 < m < 7 (B) m < 10 (C) b = 0, ±1 (D) b = 0, ±4
(C) 2 < m < 5 (D) m > 10
15. The quadratic expression ax2 + bx + c assumes both
5. If a and b are roots of the equation 2x + ax + b = 0, 2
positive and negative values if and only if
then one of the roots of the equation 2(ax + b)2 + (A) ab ¹ 0 (B) b2 − 4ac > 0
a(ax + b ) + b = 0 is (C) b − 4ac ³ 0
2
(D) b2 − 4ac < 0
(A) 0 (B) a + 2b
a2 16. If a > 0 and one root of ax2 + bx + c = 0 is less than –2
(C) aa + b (D) aa - 2b and the other is greater than 2, then
2a 2 2a 2 (A) 4a + 2 | b | + c < 0
(B) 4a + 2 | b | + c > 0
6. If a < b and x + (a + b)x + ab < 0, then (C) 4a + 2 | b | + c = 0
2

(A) a < x < b (B) −b < x < −a (D) a + b = c


(C) x < a or x > b (D) x < −b or x > −a
17. If b and c are real, then the equation x + bx + c = 0
2

7. If a and b are the roots of x − 2x + 4 = 0, then the


2
has both roots real and positive if and only if
value of a + b is
6 6
(A) b < 0 and c > 0
(A) 64 (B) 128 (C) 256 (D) 32 (B) bc < 0 and b ³ 2 c
(C) bc < 0 and b2 ³ 4c
8. The greatest value of the expression (D) c > 0 and b £ - 2 c

1
18. It is given that the quadratic expression ax + bx + c
2

4t + 2t + 1
2
takes all negative values for all x less than 7. Then
is (A) ax2 + bx + c = 0 has equal roots
(A) 4/3 (B) 5/2 (C) 13/14 (D) 14/13 (B) a is negative
(C) a and b are both negative
9. The roots of the equation 4x - 3 ´ 2x + 2 + 32 = 0 are (D) a and b are both positive
(A) 1, 2 (B) 1, 3 (C) 2, 3 (D) 2, 1/2
19. The value of a for which the equation cos x -
4

(a + 2)cos x - (a + 3) = 0 possesses solution, belongs


2
10. If the equations x - 3 x + a = 0 and x + ax - 3 = 0
2 2
to the interval
have a common root, then a possible value of a is
(A) (−¥, 3) (B) (2, +¥)
(A) 3 (B) 1 (C) –2 (D) 2
(C) [−3, −2] (D) (0, +¥)
11. If x2 - 1 £ 0 and x - x - 2 ³ 0 hold simultaneously
2

for a real x, then x belongs to the interval 20. If the expression ax + (1/ x) - 2 ³ 0 for all positive
values of x, then the minimum value of a is
(A) (–1, 2) (B) (–1, 1)
(C) [–1, 2) (D) x = –1 (A) 1 (B) 2 (C) 1/4 (D) 1/2

12. Let a ¹ 1 and a = 1. If a = a + a + a + a


13
+ a -3 +
3 4 -4 21. If a, b and c are real, a ¹ 0, b ¹ c and the equations
-1
a and b = a + a + a + a + a + a then the
2 5 6 -6 -5 -2 x2 + abx + c = 0 and x2 + cax + b = 0 have a common
quadratic equation whose roots are a and b is root, then
(A) x2 + x + 3 = 0 (B) x2 + x + 4 = 0 (A) a2 (b + c) = −1 (B) b2 (c + a) = 1
(C) x + x − 3 = 0
2
(D) x2 + x − 4 = 0 (C) c (a + b) = 1
2
(D) a2 (b + c) = 1

13. If ax - 2a x + 1 = 0 and x - 3ax + a = 0, a ¹ 0 , have 22. If ( x + x + 2) - (a - 3)( x + x + 2)( x + x + 1) + (a - 4) ´


2 2 2 2 2 2 2 2

3
a common root, then a is a root of the equation ( x2 + x + 1)2 = 0 has atleast one real root, then
(A) x2 − x − 1 = 0 (B) x2 + x − 1 = 0 (A) 0 < a < 5 (B) 5 < a £ 19 / 3
(C) x + x + 1 = 0
2
(D) x2 − x - 2 = 0 (C) 5 £ a < 7 (D) a ³ 7
www.jeeneetbooks.in
Exercises 199

Multiple Correct Choice Type Questions


( 3 / 4 )(log2 x )2 + log2 x - ( 5 / 4 )
1. The equation x = 2 has (A) a + b (B) a − b
(A) atleast one real solution (C) ( a + b )2 (D) ( a - b )2
(B) exactly three solutions
(C) exactly one irrational solution 8. If the product of the roots of the equation
(D) complex roots x2 - 4 mx + 3e2 log m - 4 = 0
is 8, then the roots are
2. If S is the set of all real values of x such that
(A) real (B) non-real
2x - 1 (C) rational (D) irrational
>0
2 x3 + 3 x2 + x
- log1/ 9 [ x2 - ( 10 / 3) x + 1]
9. If 3 £ 1, then x belongs to
then S is a superset of (A) [0, 1/3) (B) (1/3, 1)
(A) (-¥, - 3 / 2) (B) (-3/2, -1/4) (C) (2, 3) (D) (3, 10/3]
(C) (-1/ 4, 1/ 2) (D) (1/2, 3)
10. If every pair of the equations x2 + ax + bc = 0, x2 + bx +
3. If || x - 5 x + 4 | - | 2 x - 3 || = | x - 3 x + 1|, then x belongs
2 2 ca = 0 and x2 + cx + ab = 0 has a common root, then
to the interval (A) sum of these common roots is -(1/ 2)(a + b + c)
(A) (−¥, 1] (B) (1, 3/2) (B) sum of these common roots is (1/ 2)(a + b + c)
(C) [3/2, 4] (D) (4, +¥) (C) product of the common roots is abc
(D) product of the common roots is −(abc)
4. Let
11. If the equations 4 x2 - 11x + 2k = 0 and x2 - 3 x - k = 0
( x + 1)( x - 3) have a common root a, then
y=
x-2 (A) k = 0 (B) k = -17 / 36
(C) a = 0 (D) a = 17 / 6
Then the set of real values of x for which y is real is
(A) [−1, 2) (B) (2, 3) 12. If a and b are real and x2 + ax + b2 = 0 and x2 + ax + a2 = 0
(C) (−¥, −1) (D) [3, +¥) have a common root, then which of the following are
true?
5. Let a, b and c be distinct positive reals such that the (A) a = b
quadratics ax2 + bx + c, bx2 + cx + a and cx2 + ax + b (B) a + b is the common root
are all positive for all real x and (C) for real roots, a = b = 0
(D) no real values of a and b exist
a2 + b2 + c2
s=
ab + bc + ca 13. For a > 1, the equation

Then (a + a2 - 1)x
2
-2 x
+ (a - a2 - 1)x
2
-2x
= 2a
(A) s £ 1 (B) 1 < s < 4
(C) s Ï(-¥, 1) È (4, + ¥) (D) 0 < s < 1 has
(A) three real roots
6. If a and 1/a (a > 0) are roots of ax - bx + c = 0, then
2
(B) roots which are independent of a
(A) c = a (B) c ³ 2b (C) roots whose sum is 3
(C) b ³ 2a (D) a ³ 2b (D) roots whose product is –1

14. If a, b and c are positive real and a = 2b + 3c, then the


7. If
equation ax2 + bx + c = 0 has real roots for
k a b b c
= + (A) -4 ³2 7 (B) -4 ³2 7
2x x + c x - c c b
where c ¹ 0, a and b are positive, has equal roots, then a a 13
(C) - 11 ³ 4 7 (D) +4 ³2
the value of k is c b 3
www.jeeneetbooks.in
200 Chapter 4 Quadratic Equations

Matrix-Match Type Questions


In each of the following questions, statements are given
in two columns, which have to be matched. The state- Column I Column II
ments in Column I are labeled as (A), (B), (C) and
(D), while those in Column II are labeled as (p), (q), (A) The equation (p) cx2 + bx + a = 0
(r), (s) and (t). Any given statement in Column I can whose roots are
have correct matching with one or more statements in a + b and ab is
(B) The equation (q) a2 x2 + (2ac - b2 ) x + c2 = 0
Column II. The appropriate bubbles corresponding to
the answers to these questions have to be darkened as whose roots are a 2
illustrated in the following example. and b 2 is (r) a2 x2 + a(b - c) x - bc = 0
(C) The equation
Example: If the correct matches are (A) ® (p), (s); whose roots are
(B) ® (q), (s), (t); (C) ® (r); (D) ® (r), (t); that is if the (s) ax2 + (2ac + b) x + ac2 +
1/a and 1/b is
matches are (A) ® (p) and (s); (B) ® (q), (s) and (t); bc + c = 0
(D) The equation
(C) ® (r); and (D) ® (r), (t), then the correct darkening
whose roots are
of bubbles will look as follows:
a - c and b - c is (t) cx2 - bx + a = 0
p q r s t
A
3. Match the items in Column I with those in Column II.

B
Column I Column II
C
D (A) The maximum value of (p) 0
x2 - 6 x + 4
1. Match the items in Column I with those in Column II. x2 + 2 x + 4
(x is real) is
Column I Column II (q) 1
(B) The correct value of a
for which the equation
(A) If a and b are roots of (p) x2 + (a + 4) x + 2a = 0 (a2 + 4a + 3) x2 + (a2 - a - 2) x +
x2 + x + 1 = 0 and k is a a(a + 1) = 0 has more than two roots is
positive integer and not
a multiple of 3, then the (C) The number of real values of x (r) –1
equation whose roots satisfying 5x + 5- x = log10
25
is
are a k and b k is (q) x2 - x + 1 = 0
(D) If the ratio of the roots of the
(B) If a and b are roots of equation ax2 + bx + b = 0 (a and b
x2 + x + 1 = 0, then the positive) is in the ratio l : m (l and m
equation whose roots positive), then (s) -1/3
are a 2009 and b 2009 is
(r) x2 + x + 1 = 0 l m b
(C) If a and b are roots of + -
x2 + ax + b = 0, b ¹ 0, m l a
then the equation (t) 5
is equal to
whose roots are
1/a , 1/b is (s) bx2 - ax - 1 = 0 4. For the equation ( x2 - 6 x)2 = 81 + 2( x - 3)2, match the
(D) If a and b are roots items in Column I with those in Column II.
of x2 + ax - 4 = 0,
then the equation Column I Column II
whose roots are
a - 2 and b - 2 is (t) bx + ax + 1 = 0
2 (A) The number of rational roots is (p) 12
(B) The number of irrational roots is (q) 6
(r) 2
2. Let a and b be roots of the equation ax + bx + c = 0
2
(C) Sum of all the real roots is
and ab ¹ 0. Then match the items in Column I with (s) 99
those in Column II. (D) Product of the real roots is (t) –99
www.jeeneetbooks.in
Exercises 201

5. Let 6. Match the items in Column I with those in Column II.

x2 - 6 x + 5
f ( x) = Column I Column II
x2 - 5 x + 6
(A) If x is real, the expression (p) [1, 7]
Then match the items in Column I with those in
Column II. ( x + 3)2 - 24 ( x ¹ 2)
2 ( x - 2)
Column I Column II
admits all values except those in
(A) If -1 < x < 1, then f (x) satisfies (p) 0 < f ( x) < 1 the interval
(B) If 1 < x < 2, then f ( x) satisfies (q) f ( x) < 0
(B) If the expression (q) [-1/11, 1]
(r) f ( x) > 0
(C) If 3 < x < 5, then f ( x) satisfies (s) f ( x) < 1 px + 3 x - 4
2

(D) If x > 5, then f ( x) satisfies (t) f ( x) = 0 p + 3 x - 4 x2


( p + 3 x - 4 x2 ¹ 0) takes all real
values, then p lies in the interval (r) (−3, −2)
(C) If x is real, then
x
x2 - 5 x + 9
must lie in the interval (s) (4, 6)
(D) If x ¹ - 2 and x ¹ -3, then
x2 - 4 x + 5
<0
x2 + 5 x + 6
for all x in the interval (t) (3, 5)

Comprehension-Type Questions
1. Passage: To solve equations of the form (iii) The real solution set of the equation
(ax + bx + c)(ax + bx + d) = k
2 2
( x - 2)( x + 1)( x + 4)( x + 7) = 19
use the substitution ax + bx = y, so that the given equa-
2
contains
tion transforms into a quadratic equation in y which
can be solved. Answer the following three questions. (A) four elements (B) three elements
(C) two elements (D) no elements
(i) The number of real roots of the equation
2. Passage: Let f ( x) = ax + bx + c and a ¹ 0. If a and b
2
1 1 1
- = are roots of f (x) = 0, then a + b = - b /a and ab = c /a.
x( x + 2) ( x + 1)2 12
Further, if a and b are real roots with a < b, then f (x)
is and a have the same sign for all x < a or x > b , and f ( x)
(A) 2 (B) 1 (C) 0 (D) 4 and a have opposite sign for all a < x < b . Consider
the quadratic equation
(ii) The equation (1 + m) x2 - 2(1 + 3m) x + (1 + 8 m) = 0
24 15 Now, answer the following three questions.
- =2
x2 + 2 x - 8 x2 + 2 x - 3 (i) The number of real values m such that the roots
has of the given quadratic equation are in the ratio
2:3 is
(A) all positive solutions
(B) three positive and one negative solutions (A) 2 (B) 4
(C) two non-negative and two negative solutions (C) 0 (D) infinite
(D) two real and two imaginary solutions
www.jeeneetbooks.in
202 Chapter 4 Quadratic Equations

(ii) The set of values of m such that both the roots of Now, answer the following three questions.
the equation are positive is (i) If the equation (a2 + 1)x2 - (a + 1)x + (a2 - a - 2) = 0
(A) (−¥, + ¥) has one positive and one negative root, then which
(B) -1 < m < 3 one of the following is possible?
(C) 3 £ m < ¥ (A) a £ -1 (B) -1 < a < 2
(D) (-¥, - 1) È (3, +¥) È {0} (C) 2 £ a £ 5 (D) a > 5
(iii) The values of m for which the equation has equal (ii) If mx2 - (m + 1) x + 3 = 0 has roots belonging to
roots are (1, 2), then
(A) 0, 3 (B) −1, 3 (C) −1, 0 (D) 1, 3 (A) 0 < m < 1 (B) 1 £ m £ 2
(C) m < 0 (D) no real value for m
3. Passage: Let f ( x) º ax + bx + c, a ¹ 0. Let a and b be
2
exists
roots of f (x) = 0. Then the following hold good.
(iii) If x - (m + 1) x + m + m - 8 = 0 has one root
2 2

(a) a + h and b + h are roots of f(x - h) = 0 for all h. in the open interval (-¥, 1) and the other in
(b) la + h and lb + h are roots of f [(x - h)/l] = 0 for all (1, +¥), then
h and for all l ¹ 0.
(A) m < -2 2 (B) m > 2 2
Now, answer the following three questions.
(C) -2 2 < m < 2 2 (D) no real value for m
(i) If a and b are the roots of ax2 + bx + c = 0, then exists
the equation whose roots are
5. Passage: Let f ( x) º ax + bx + c, where a, b and c are
2
a +1 b +1
and real and a ¹ 0. Then f ( x) = 0 has real roots or imagi-
a-2 b-2
nary roots according as b2 - 4ac ³ 0 or b2 - 4ac < 0.
is Answer the following three questions.
(A) a( x + 1)2 + b( x + 1)( x - 2) + c( x - 2)2 = 0 (i) If the function
(B) a( x - 2)2 + b( x + 1)( x - 2) + c( x + 1)2 = 0
x2 - x
(C) a(2 x + 3) + b( x + 1)( x + 2) + c( x + 2) = 0 y=
2 2

(D) a(2 x + 1) + b(2 x + 1)( x - 1) + c( x - 1)2 = 0


2 1 - mx
(ii) If a and b are roots of the equation 2x2 + 4x - takes all real values for real values of x, then
5 = 0, then the equation whose roots are 2a - 3 (A) m < 0 (B) 0 < m < 1 (C) m > 0 (D) m > 1
and 2b - 3 is
(ii) If
(A) x2 + 10 x - 11 = 0
x2 + 2 x + c
(B) 11x2 + 10 x - 1 = 0 y=
(C) x2 + 10 x + 11 = 0 x2 + 4 x + 3c
(D) 11x - 10 x + 1 = 0
2
takes all real values, then
(iii) If a and b are roots of ax2 + bx + c = 0, then the (A) 0 < c < 1 (B) c < −1
equation whose roots are a + (c /a) and b + (c /a) is (C) c > 1 (D) c > 0
(A) a2 x2 - 2(ac + b) x + c(a + b) = 0 (iii) If
(B) a2 x2 - (ca + b) x + c(a + b) = 0
(C) a2 x2 + 2(ac + b) x - c(a + b + c) = 0 x2 + ax + 1
<3
(D) a x - a(b + 2c) x + c(a + b + c) = 0
2 2
x2 + x + 1

4. Passage: Let a, b and c be real numbers, a ¹ 0 and for all real x, then
f(x) º ax2 + bx + c. If a < b are roots of f(x) = 0, then it (A) a < 0 (B) a < −1
is known that (C) −1 < a < 7 (D) a > 7
(A) f ( x)× a < 0 for all x in the open interval (a, b).
(B) f ( x)× a > 0 for all x such that either x < a or x > b .

Assertion–Reasoning Type Questions


In each of the following, two statements, I and II, are given (C) I is true, but II is not true.
and one of the following four alternatives has to be chosen. (D) I is not true, but II is true.
(A) Both I and II are correct and II is a correct reasoning
1. Statement I: If f ( x) º ax + bx + c is positive for all x
2
for I.
(B) Both I and II are correct but II is not a correct greater than 5, then a > 0, but b may be negative or
reasoning for I. may not be negative.
www.jeeneetbooks.in
Exercises 203

Statement II: If f ( x) º ax2 + bx + c > 0 for all x > 5, c b


1+ + <0
then the equation f ( x) = 0 may not have real roots or a a
will have real roots less than or equal to 5.
Statement II: Let f (x) º ax2 + bx + c, where a, b and c
2. Statement I: If a, b and c are positive integers and are real numbers and a ¹ 0. If f ( x) = 0 has real roots,
ax2 - bx + c = 0 has two distinct roots in the integer then af ( x) < 0 for all real x lying between the roots
(0, 1), then log5 (abc) ³ 2. of f(x) = 0.
Statement II: If a quadratic equation f ( x) = 0 has
9. Statement I: Let
roots in an interval (h, k), then f (h), f (k ) > 0
x2 - x + 1
3. Statement I: There are only two values for sin x satis- y=
2 2 x2 + x + 1
fying the equation 2sin x
+ 5 ´ 2cos x
= 7.
Statement II: Maximum value of sin2 x is 1. where x is real. Then y cannot lie between 1/3 and 3.
Statement II: If a, b and c are real, then the quadratic
4. Statement I: If x = 1 is a root of the quadratic equa- equation ax2 + bx + c = 0 has real roots if and only if
tion ax2 + bx + c = 0, then the roots of the equation b2 - 4ac ³ 0.
4ax2 + 3bx + 2c = 0 are imaginary.
Statement II: For any polynomial equation, 1 is a root 10. Statement I: For all real values of x, the range of the
if and only if the sum of all the coefficients of the poly- function
nomial is zero. (ax - b)(cx - d)
y=
(bx - a)(dx - c)
5. Let a, b, c, p and q be real numbers and a and b be
roots of the equation x2 + 2 px + q = 0. Suppose a and is ¡ if a, b, c and d are real, a ¹ b, c ¹ d and (a2 - b2 )
1/b are roots of the equation ax2 + 2bx + c = 0 where (c2 - d2) > 0.
b 2 Ï{-1, 0, 1}.
Statement II: A quadratic equation will have real
Statement I: ( p - q)(b - ac) ³ 0
2 2
roots if its discriminant is greater than or equal to
Statement II: b ¹ pa or c ¹ qa zero.

6. Statement I: Let a, b, a and b be real numbers. If a + 11. Statement I: Suppose a, b and c are real, c > 0,
ib (a ¹ 0, b ¹ 0) is a root of the equation x3 + bx + c = 0, a + b + c > 0 and a - b + c > 0. Then both the roots
then 2a is a root of one of the following equations. of the equation ax2 + bx + c = 0 lie between –1 and 1.
(i) x3 - bx + c = 0 (ii) x3 - bx - c = 0 Statement II: For a quadratic expression f (x), if f (p)
(iii) x + bx - c = 0
3
(iv) x3 + bx - 2c = 0 and f (q) are of opposite sign, then f (x) = 0 has a root
Statement II: Complex roots occur in conjugate pairs in between p and q.
for any polynomial equation with real coefficients.
12. Statement I: Let f ( x) and g( x) be quadratic expres-
7. Statement I: The maximum value of sions with rational coefficients. Suppose they have
a common root of the form a + b where b is not a
3 x2 + 9 x + 17 perfect square of a rational number. Then g( x) = g f ( x)
( x is real)
3 x2 + 9 x + 7 for some rational number g.
is 8. Statement II: For a quadratic equation, with rational
Statement II: If a, b, and c are real numbers and a > 0, coefficients, if a + b (b is not a perfect square of
then the minimum value of ax2 + bx + c (x is real) is a rational number) is a root, then a - b is also a root.

4ac - b2 13. Statement I: If the equation x + px + q = 0 has rational


2

4a roots and p and q are integers, then the roots are


integers.
8. Statement I: Suppose a, b and c are real numbers and
a ¹ 0. If the equation ax2 + bx + c = 0 has two roots Statement II: A quadratic equation has rational roots
of which one is less than –1 and the other is greater if and only if its discriminant is a perfect square of a
than 1, then rational number.
www.jeeneetbooks.in
204 Chapter 4 Quadratic Equations

Integer Answer Type Questions


The answer to each of the questions in this section is 2. The number of negative integer solutions of x2 ´ 2x + 1 +
a non-negative integer. The appropriate bubbles below 2| x - 3|+2 = x2 ´ 2| x - 3|+ 4 + 2x - 1 is .
the respective question numbers have to be darkened.
3. If (a + 5i)/ 2 is a root of the equation 2 x - 6 x + k = 0,
2
For example, as shown in the figure, if the correct answer
to the question number Y is 246, then the bubbles under then the value of k is .
Y labeled as 2, 4, 6 are to be darkened.
4. If the equation x2 - 4 x + log1/ 2 a = 0 does not have
X Y Z W distinct real roots, then the minimum value of 1/a
0 0 0 0 is .
1 1 1 1
2 2 2 5. If a is the greatest negative integer satisfying
3 3 3 3 x2 - 4 x - 77 < 0 and x2 > 4
4 4 4
simultaneously, then the value of | a | is .
5 5 5 5
6 6 6 6. The number of values of k for which the quad-
7 7 7 7 ratic equations (2k - 5)x2 - 4x - 15 = 0 and (3k - 8)
8 8 8 8 x2 - 5x - 21 = 0 have a common root is .
9 9 9 9
7. The number of real roots of the equation 2 x2 - 6 x -
5 x2 - 3 x - 6 = 0 is .
1. The integer value of k for which
x2 - 2(4k - 1) x + 15k2 - 2k - 7 > 0
for all real x is .

ANSWERS
Single Correct Choice Type Questions
1. (D) 12. (C)
2. (A) 13. (A)
3. (B) 14. (A)
4. (D) 15. (B)
5. (A) 16. (A)
6. (B) 17. (D)
7. (B) 18. (B)
8. (A) 19. (C)
9. (C) 20. (A)
10. (D) 21. (A)
11. (D) 22. (B)

Multiple Correct Choice Type Questions


1. (A), (B), (C) 8. (A), (D)
2. (A), (D) 9. (A), (D)
3. (A), (C) 10. (A), (C), (D)
4. (A), (D) 11. (A), (B), (C), (D)
5. (B), (C) 12. (A), (B), (C)
6. (A), (C) 13. (A), (B), (C), (D)
7. (C), (D) 14. (A), (C)
www.jeeneetbooks.in
Answers 205

Matrix-Match Type Questions


1. (A) ® (r), (B) ® (r), (C) ® (t), (D) ® (p) 5. (A) ® (p), (r), (s), (B) ® (q), (s),
2. (A) ® (r), (B) ® (q), (C) ® (p), (D) ® (s) (C) ® (q), (s), (D) ® (p), (r), (s)
3. (A) ® (t), (B) ® (r), (C) ® (p), (D) ® (p) 6. (A) ® (s), (B) ® (p), (C) ® (q), (D) ® (r)
4. (A) ® (r), (B) ® (r), (C) ® (p), (D) ® (t)

Comprehension-Type Questions
1. (i) (A); (ii) (C); (iii) (A) 4. (i) (B); (ii) (D); (iii) (C)
2. (i) (A); (ii) (D); (iii) (A) 5. (i) (D); (ii) (A); (iii) (C)
3. (i) (D); (ii) (C); (iii) (D)

Assertion–Reasoning Type Questions


1. (A) 8. (A)
2. (A) 9. (D)
3. (A) 10. (A)
4. (D) 11. (A)
5. (B) 12. (A)
6. (A) 13. (A)
7. (D)

Integer Answer Type Questions


1. 3 5. 3
2. 0 6. 2
3. 17 7. 4
4. 16
www.jeeneetbooks.in
www.jeeneetbooks.in

Progressions,
Sequences and Series 5
Contents
Progressions, Sequences and Series

5.1 Sequences and Series


5.2 Arithmetic Progressions
5.3 Geometric Progressions
5.4 Harmonic Progressions

Worked-Out Problems
Summary
Exercises
Answers

Sequences: A sequence is an
+2 +2 +2 +2 ordered list of objects (or
events). It contains members
(also called elements or terms),
and the number of terms
0 1 2 3 4 5 6 7 8 9 10 (possibly infinite) is called the
length of the sequence. Order
matters and the exactly same
elements can appear multiple
times at different positions in
the sequence.
Series: The sum of terms of
a sequence is a series.
www.jeeneetbooks.in
208 Chapter 5 Progressions, Sequences and Series

We have defined the concept of a function and its domain, codomain and range in Chapter 1. A sequence is a function
whose domain is the set of natural numbers and codomain is a given set. In this chapter, we discuss various aspects of
sequences, in particular of sequences defined in certain recursive types.

5.1 | Sequences and Series


In this section, we will introduce the notion of a sequence and the corresponding series and their limits. Though the
concept of limit is discussed in another volume of this series, we assume a certain intuitive idea about the limit or the
approaching value. For example, the value of 1/n decreases as n increases and 1/n becomes nearer to zero (and it is
never zero) as we take bigger values for n. A naive idea like this is enough to understand the concepts introduced in
this chapter.

Sequence of Elements

DEF IN IT ION 5 . 1 Let + be the set of positive integers and X any set. Then a mapping a : + ® X is called a
sequence of elements in X or, simply, a sequence in X. For any n Î + , we prefer to write an for
the image a(n). This an is called the nth term of the sequence.

Usually a sequence is denoted by its range {an | n Î+} or simply {an } or {a1, a2, a3 , …}.

Examples

(1) {1/ n} is a sequence of real numbers. Here the sequence (4) {in} is a sequence of complex numbers. Here the
a : + ®  is given by an = 1/ n for any n Î +. sequence a : + ®  is given by an = in for any n Î+.
(2) {n2 } is a sequence of integers. Here an = n2 for all n Î +. Recall that in = 1 if n is a multiple of 4, in = i if n = 4m + 1,
in = -1 if n = 4m + 2 and in = -i if n = 4m + 3.
(3) {log2 n} is a sequence of real numbers. Here the sequence
a : + ®  is given by an = log2 n for any n Î + .

DEF IN IT ION 5 . 2 A sequence {an } is called finite if its range is a finite set. In other words, the set
{an | n Î+}
is a finite set. An infinite sequence is a sequence which is not finite.

Examples
(1) The sequence {in} is finite, since {1, i, -1, -i} is precisely (3) For any m > 1, {mn} is an infinite sequence.
the range. (4) The sequence {log2 n} is infinite.
(2) The sequence {(-1)n} is finite, since {1, -1} is its range.

DEF IN IT ION 5 . 3 A sequence {an } is called constant if a1 = a2 = (i.e., an = am for all n and m Î + ). {an } is called
ultimately constant if it is constant after a certain stage in the sense that, there is a positive
integer m such that
am = am+ k for all k Î +
or am = am+1 = am+ 2 =

Quite often, ultimately constant sequences are also called finite sequences for the simple reason that their ranges are
finite. As per our terminology, any ultimately constant sequence is finite and not vice-versa; for, consider the following
examples:
www.jeeneetbooks.in
5.1 Sequences and Series 209

Examples
(1) The sequence {(-1)n} is finite but not ultimately whether n is odd or even, respectively. In this case
constant, since an ¹ an+1 for all n Î +, where an = (-1)n, {an} is finite, but not ultimately constant. Here also,
an = 1 if n is even and an = -1 if n is odd. an ¹ an+1 for all n Î +.
(2) The sequence {an}, where an = [1/n] (the integral part of (4) The sequence {in } is also finite, but not ultimately
1/n), is ultimately constant, since [1/n] = 0 for all n > 1. constant.
(3) Define the sequence {an} by an = the remainder obtained
by dividing n with 2. Then an is 1 or 0 depending on

Quite often, sequences are defined recursively in the sense that an is defined in terms of an-1, an- 2 , …, a2 , a1 . Of
course, one has to define the first term a1 or the first few terms.

Try it out
1. Let a1 = 2 and an = an-1 + 2 for any n > 1. Then show that
a2 = a1 + 2 = 2 + 2 = 4
a3 = a2 + 2 = 6
a4 = a3 + 2 = 8, etc.

2. Let a1 = 1, a2 = 4 and an = a1 + a2 + + an-1 for n ³ 3. Then show that


a3 = a1 + a2 = 5
a4 = a1 + a2 + a3 = 10
a5 = a1 + a2 + a3 + a4 = 20 , etc.
Note that an = 2an-1 for n > 3.
3. Let a1 = 1, a2 = 2 and an = an-1 + an- 2 for any n > 2 . Then show that
a3 = a2 + a1 = 3
a4 = a3 + a2 = 5
a5 = a4 + a3 = 8
a6 = a5 + a4 = 13
a7 = a6 + a5 = 21, etc.
4. Let

1 an-1
a1 = and an = for all n > 1
2 1 + 2an-1

Then show that

1 1 1 1
a2 = , a3 = , a4 = , a5 = , etc.
4 6 8 10
Note that an = 1/2 n for all n Î +.

Series

DEF IN IT ION 5 . 4 If {an } is a sequence of real or complex numbers, then an expression of the form
a1 + a2 + a3 +
www.jeeneetbooks.in
210 Chapter 5 Progressions, Sequences and Series

is called a series. If sn is the sum of the first n-terms of the sequence {an } , that is,
sn = a1 + a2 + + an
then {sn } is again a sequence and sn is called the nth partial sum of the series.

Examples

(1) 1 + 2 + 3 + is a series and the partial sum is given by (2) 1 + (-1) + 1 + (-1) + is a series and the partial sum
is sn = 1 or 0 depending on whether n is odd or even.
n(n + 1)
sn = 1 + 2 + + n =
2

Note: An ultimately constant sequence {an } is some times referred as a finite sequence and is expressed as a1, a2, … , an
with the assumption that an = an +1 =

Limit
DEF IN IT ION 5 . 5 Let {an } be a sequence of real numbers and a be a real number. Then a is said to be limit of the
sequence {an } if, for each Î> 0 , there exists a positive integer n0 such that
| an - a | < Î for all n ³ n0
That is, a - Î< an < a + Î for all n ³ n0.

T H E O R E M 5 .1 Any sequence can have at most one limit.


PROOF Let {an } be a sequence and a and b be limits of {an }. Suppose that a ¹ b. Take
1
Î= | a - b |
2
Since a is a limit of {an }, there exists n0 Î + such that

| an - a | < Î for all n ³ n0

Similarly, there exists n1 Î + such that

| an - b | < Î for all n ³ n1

Choose n Î + such that n > max{n0 , n1 }. Then

| a - b | £ | a - an | + | an + b | < Î+ Î= | a - b |

which is a contradiction. Thus a = b. ■

DEF IN IT ION 5 . 6 If a is the limit of {an }, then we write lim an = a or, simply, lim an = a and denote it by an ® a.
n ®¥

Examples

(1) Consider the sequence {1/ n} . Then lim(1/ n) = 0. For, (2) The sequence {n} has no limit.
if Î> 0 is given, choose a positive integer n0 > 1/ Î so (3) If lim an = a and lim bn = b, then lim(am ± bn ) = a ± b
that, for any n ³ n0 and lim anbn = ab.
1 1 1 (4) If a is the limit of a sequence {an }, then a is the limit of
-0 = £ <Î any sequence obtained by omitting a finite number
n n n0
of consecutive terms from {an }.
www.jeeneetbooks.in
5.2 Arithmetic Progressions 211

Convergent and Divergent Series

DEF IN IT ION 5 . 7 Let {an} be any sequence of real numbers and sn = a1 + a2 + + an. If the sequence {sn} has limit s,
then we write
¥

åa
n= 1
n =s

¥ ¥
In this case, the infinite series å a is said to be convergent to s. If {sn} has no limit, then the series
n= 1 n å a is said to
n= 1 n
be divergent.

Examples

¥
(1) å
¥
n= 1
1
n2
is convergent. (3) If å a is convergent, then lim an = 0.
n= 1 n

¥
¥1 (4) If lim an = 0, then å n= 1 an may not be convergent (see
(2) å is divergent.
n= 1
n example given in point 2).

5.2 | Arithmetic Progressions


A sequence whose terms satisfy a specific condition is called a progression. In this section we discuss sequences in
which the difference between any two consecutive terms is a fixed constant. Such sequences are called arithmetic
progressions. We begin with the formal definition in the following.

DEF IN IT ION 5 . 8 A sequence {an } is called an arithmetic progression if an + 1 - an = an - an - 1 for all integers n > 1
and, in this case, an + 1 - an is called the common difference.

An arithmetic progression is also called an arithmetic sequence. Note that {an } is an arithmetic progression if and
only if
an + 1 + an - 1 = 2an
for all integers n > 1.
Before going for examples, let us have the following fundamental characterization of arithmetical progressions.

T H E O R E M 5 .2 Let {tn } be a sequence of real numbers. Then {tn } is an arithmetic progression if and only if there
exist unique real numbers a and d such that
tn = a + (n - 1)d
for all integers n ³ 1.
PROOF Suppose that {tn } is an arithmetic progression. Then
tn + 1 - tn = tn - tn - 1 for all n > 1
Take a = t1 and d = t2 - t1. Then
t1 = a + (1 - 1)d
t2 = t1 + (t2 - t1 ) = a + (2 - 1)d
t3 = t2 + (t3 - t2 ) = t2 + (t2 - t1 ) = a + d + d = a + 2d

and, in general, we can prove by induction that tn = tn - 1 + (tn - tn - 1 ) = a + (n - 2)d + d = a + (n - 1)d


for any positive integer n.
www.jeeneetbooks.in
212 Chapter 5 Progressions, Sequences and Series

Conversely, suppose that there are real numbers a and d such that tn = a + (n - 1)d for all
positive integers n. Then
tn + 1 - tn = a + (n + 1 - 1)d - (a + (n - 1)d) = d

for all n Î + and therefore {tn } is an arithmetic progression. The uniqueness of a and d follows
from the facts that t1 = a and t2 - t1 = d. ■

QUICK LOOK 1

1. In any arithmetic progression {tn} the first term t1 3. Any arithmetic progression must be of the form
and the common difference tn+1 - tn determine all the a, a + d, a + 2d, a + 3d, … where a is the first term
terms and, therefore, the first and second terms (t1 = a and d is the common difference. This is called the
and t2 - t1 = d) of an arithmetic progression deter- general form of an arithmetic progression.
mine the whole sequence. Also, by examining the
4. The nth term of an arithmetic progression is
first three terms (in fact any three consecutive terms)
we can get a clue that the given sequence is or is not tn = a + (n - 1)d
an arithmetic progression.
where a is the first term t1 and d is the common
2. A sequence {tn} is an arithmetic progression if and difference tn + 1 - tn (= t2 - t1 ).
only if twice of any term is equal to the sum of its
proceeding term and succeeding term.

Examples

(1) The sequence {n} is an arithmetic progression. Here (2) Any constant sequence is an arithmetic progression
the first term and the common difference are both the common difference being zero.
equal to 1.

QUICK LOOK 2

1. If {an} is an arithmetic progression, then for any real also an arithmetic progression whose common differ-
number k, {an + k} is also an arithmetic progression ence is kd.
with the same common difference as {an}, since 3. If {an} and {bn} are arithmetic progressions, then
(an+1 + k ) - (an + k ) = an+1 - an, for any n Î +. {an + bn} is also an arithmetic progression; however
2. If {an} is an arithmetic progression with common {anbn} is not so, in general. In this direction, we have
difference d and k is any real number, then {kan} is the following.

T H E O R E M 5 .3 Let {an} and {bn} be arithmetic progressions. Then {an bn} is an arithmetic progression if and only if
either {an} or {bn} is a constant sequence.
PROOF If {an} or {bn} is constant, then by point 2 of Quick look 2, {anbn} is an arithmetic progression.
To prove the converse, let d and e be the common differences of {an} and {bn}, respectively.
Then
{an bn } is an AP Þ an + 1bn + 1 - an bn = an bn - an - 1bn - 1
Þ (an + 1 - an )bn + 1 + (bn + 1 - bn )an = (an - an - 1 )bn + (bn - bn - 1 )an - 1
Þ dbn + 1 + e an = dbn + e an - 1
Þ d(b
bn + 1 - bn ) = - e(an - an - 1 )
Þ de = - de
www.jeeneetbooks.in
5.2 Arithmetic Progressions 213

Þ 2de = 0
Þ d = 0 or e = 0
Þ {an } is constant or {bn } is constant ■

DEF IN IT ION 5 . 9 Let a1, a2 , … , an be given numbers. Then a1, a2 , … , an are said to be in arithmetic progression if
these are, in this order, consecutive terms of an arithmetic progression.

T H E O R E M 5 .4 a1, a2 , … , an are in arithmetic progression (where n > 2) if and only if 2ar = ar + k + ar - k for all r and k
such that 1 £ r - k < r < r + k £ n.
PROOF If 2ar = ar + k + ar - k , then
ar + 1 - ar = ar - ar - 1 for all 1 < r < n
and hence ar = a1 + (r - 1)(a2 - a1 ) for all 1 £ r £ n. Therefore a1, a2, …, an are in
arithmetic progression. ■

Try it out The converse is clear. It is left for the reader as an exercise.

QUICK LOOK 3

1. Any two real numbers a1, a2 are in arithmetic progres- 6. In general, (2r + 1) numbers in AP can be taken as
sion whose common difference is a2 - a1. a - rd, a - (r - 1)d, … , a, a + r, … , a + rd.
2. a1, a2 , a3 are in AP if and only if 2a2 = a 1 + a3. 7. In general 2r numbers (r Î + ) in AP can be taken as
3. Three numbers in AP can be taken as a - d, a, a + d a - (2r - 1)d, a - (2r - 3)d, … , a - d, a + d, a + 3d, … ,
for some a and d. a + (2r − 1)d.
4. Four numbers in AP can be taken as a − 3d, a − d, 8. A sequence {an } is an AP if and only if the nth term
a + d, a + 3d. an is a linear expression in n.
5. Five numbers in AP can be taken as a − 2d, a − d,
a, a + d, a + 2d.

T H E O R E M 5 .5 The sum of the first n terms of an arithmetic progression is given by

é n-1 ù
sn = n êa1 + dú
ë 2 û

where a1 is the first term and d is the common difference.


PROOF Let {an } be an arithmetic progression and d the common difference. Then
an = a1 + (n - 1)d for all n Î +
Let sn be the sum of the first n terms in {an } . Then
sn = a1 + a2 + + an
n n
= å ai = å (a1 + (r - 1)d)
r =1 r =1

æ n ö
= n a1 + ç å (r - 1)÷ d
è r =1 ø
www.jeeneetbooks.in
214 Chapter 5 Progressions, Sequences and Series

= na1 + [0 + 1 + 2 + + (n - 1)]d

n(n - 1)
= na1 + d
2
æ n-1 ö
= n ç a1 + d÷
è 2 ø

The sum of the first n terms of an AP is given by

é n - 1 ù æ dö 2 æ dö
n êa1 + d ú = ç ÷ n + ç a1 - ÷ n
ë 2 û è 2ø è 2ø

which is a quadratic expression in n, with constant term zero. ■

Converse of Theorem 5.5 is proved in the following theorem.

T H E O R E M 5 .6 A sequence is an arithmetic progression if and only if the sum of the first n terms is a quadratic
expression in n with the constant term zero.
PROOF Let {an } be a sequence and sn = a1 + a2 + + an , for any n Î +. Then the nth term is given by
an = sn - sn -1
Now, suppose that sn is a quadratic expression in n with constant term zero, that is,
sn = an2 + bn
where a and b are real numbers. Then
an = sn - sn-1
= an2 + bn - [a(n - 1)2 + b(n - 1)]
= a[n2 - (n - 1)2 ] + b[n - (n - 1)]
= (2 n - 1)a + b
Therefore, the nth term is an = (2 n - 1)a + b and so, for any n > 1,
an - an -1 = (2 n - 1)a + b - [(2(n - 1) - 1)a + b]
= [2 n - 1 - (2 n - 3)]a = 2a
This shows that an - an -1 is a constant for all n and hence {an } is an arithmetic progression with a
common difference 2a and first term a + b. ■

QUICK LOOK 4

1. In the above, if sn = an2 + bn + c with c ¹ 0 , then {an} Therefore {an} is not an arithmetic progression.
is not an arithmetic progression. In this case, it can However, a2, a3, a4, a5, … are in AP, with common
be observed that difference 2a. That is, excluding a1, {an } is an AP.
an - an-1 = 2a 2. If the sum of the first n terms of a sequence is
for n > 2
an2 + bn for all n Î +, then the sequence is an AP
and a2 - a1 = ( s2 - s1 ) - a1 = s2 - 2 s1 whose first term is a + b and the common difference
is 2a.
= (4a + 2b + c) - 2(a + b + c)
= 2a - c ¹ 2a (since c ¹ 0)
www.jeeneetbooks.in
5.2 Arithmetic Progressions 215

Example 5.1

If the first, second and nth term of an arithmetic The sum of the first n terms is given by
progression are a, b and c respectively, then find the sum
of the first n terms of the sequence. æ n-1 ö æ c-a ö
nça + d÷ = n ç a + (b - a)÷
è 2 ø è 2(b - a) ø
Solution: The given sequence can be written as
a, b, b3 , b4 , …, bn-1 , c, bn+1 , … c + b - 2a æ c - aö
= ça + ÷
The common difference must be b - a. Also, c which is b-a è 2 ø
the nth term equals
(c + b - 2a)(a + c)
a + (n - 1)d = a + (n - 1)(b - a) =
2(b - a)
Therefore
c-a c + b - 2a
n= + 1=
b-a b-a

Arithmetic Mean

DEF IN IT ION 5 . 10 If three numbers a, b, c are in arithmetic progression, then b is called the arithmetic mean (AM)
between a and c. In general, if a, b1, b2, …, bn, c are in arithmetic progression, then b1, b2, …, bn
are called n arithmetic means (n AMs) between a and c.

T H E O R E M 5 .7 If A1, A2, …, An are n arithmetic means between a and c, then


k ( c - a)
Ak = a + for 1 £ k £ n
n+1

PROOF Let a, A1, A2, …, An, c be in arithmetic progression and d be the common difference. Then
A1 = a + d, A2 = a + 2d, … , Ak = a + kd, An = a + nd
and c = a + (n + 1)d
Therefore,
c-a c-a
d= and Ak = a + k
n+1 n+1 ■

QUICK LOOK 5

1. If b is the arithmetic mean between a and c then æ a + bö


A1 + A2 + + An = n ç
a+c è 2 ÷ø
b=
2 That is, the sum of n arithmetic means between two
2. For any real numbers a and b, given real numbers a and b is equal to n times of the
AM of a and b.
a+b 4. If a is the first term and b is the nth term in an AP,
a, ,b
2 then the sum of the first n terms is equal to
are in arithmetic progression. n
(a + b)
3. If A1, A2, …, An are n arithmetic means between a and 2
b, then
www.jeeneetbooks.in
216 Chapter 5 Progressions, Sequences and Series

T H E O R E M 5 .8 If Tn and Tn¢ are the nth terms of two arithmetic progression and sn and sn¢ are their sums of the
first n terms, respectively, then
Tn s2 n - 1
=
Tn¢ s2¢n - 1

PROOF Let the two AP’s be


a, a + d, a + 2d, … and b, b + e, b + 2e, …
Then, by Theorem 5.4, we have

s2 n-1 (2 n - 1)[a + {(2 n - 2)/ 2}d ]


=
s2¢n-1 (2 n - 1) [b + {(2 n - 2)/ 2}e ]

a + (n
n - 1)d Tn
= =
b + (n - 1)e Tn¢ ■

Example 5.2

The nth terms of two AP’s {an } and {bn } are 10 and 15, and hence
respectively. If sum of the first n terms of {an } is 30n, then
find the sum of the first 21 terms of {bn }. 30(2 n - 1) ´ 15
t2 n - 1 = = 45(2 n - 1)
10
Solution: If sn and tn are the sums of the first n terms
Thus, the sum of the first 21 terms in {bn } is given by
of {an } and {bn }, respectively, then by Theorem 5.7,
t21 = t2 ´ 11- 1 = 45(21) = 945
30(2 n - 1) s2 n - 1 an 10
= = =
t2 n - 1 t2 n - 1 bn 15

Example 5.3

The 22nd term and 46th term of an AP are 36 and 72, Then, solving the above two equations in two variables
respectively. Find the general term of the AP. we get

Solution: 3
The 22nd term is 36. This means 24d = 36 Þ d =
2
a + 21d = 36
Substituting this value of d in any one of the above equa-
The 46th term is 72 which implies tions gives
a + 45d = 72 9
a=
Here, a is the first term and d is the common difference 2
of the AP. Therefore, the nth term is given by
9 (n - 1)3
an = +
2 2

Example 5.4

The sum of four integers in AP is 24 and their product is (a - 3d) + (a - d) + (a + d) + (a + 3d) = 24


945. Find these integers.
and (a - 3d)(a - d)(a + d)(a + 3d) = 945
Solution: Let the four integers be a - 3d, a - d, a + d, Therefore, from the first equation we get
and a + 3d. Then, it is given that
4a = 24 or a = 6
www.jeeneetbooks.in
5.3 Geometric Progressions 217

Also from the second equation we have d4 - 40d2 + 39 = 0


(a - d )(a - 9d ) = 945
2 2 2 2
(d2 - 1)(d2 - 39) = 0
(36 - d2 )(36 - 9d2 ) = 945 Since the terms of the given AP are integers, so is d.
9d - 360d + (36 ´ 36) = 945
4 2 Therefore, d2 ¹ 39. This gives d2 = 1 or d = ±1. Hence, the
given integers are
d4 - 40d2 + 144 = 105
3, 5, 7, 9 or 9, 7, 5, 3

5.3 | Geometric Progressions


A sequence in which the ratio of any term, and its immediate predecessor term is constant is called a geometric
progression. In this section we will discuss various properties of geometric progressions.

DEF IN IT ION 5 . 11 A sequence {an} of non-zero real numbers is called a geometric progression (GP) if
an a
= n+1 for all n > 1
an-1 an

DEF IN IT ION 5 . 12 Let {an} be a geometric progression and r be the constant an+1/an. Then r is called the common
ratio.

Examples

(1) The sequence {1, 2, 22, 23, …} is a geometric progres- (3) The common ratio of a geometric progression is 1 if
sion with common ratio 2. and only if it is a constant sequence.
(2) {3, -3/2, 3/4, -3/8, 3/16, -3/32, …} is a geometric (4) {3, -3, 3, -3, 3, -3, …} is a geometric progression with
progression with common ratio -1/2. common ratio -1.

DEF IN IT ION 5 . 13 Non-zero real numbers t1, t2, …, tm are said to be in geometric progression (GP) if these are
consecutive terms of a geometric progression.

QUICK LOOK 6

i -1
1. Any geometric progression with first term a and æa ö
common ratio r can be expressed as ai = ç 2 ÷ a1 for all 1 £ i £ n
è a1 ø
a, ar, ar 2, …, ar n, …
4. The nth term of a GP with first term a and common
This is known as the general form of a GP. ratio r is given by
2. Three non-zero real numbers a, b and c are in GP if tn = r n-1a, for any n Î +
and only if b2 = ac.
3. In general, non-zero real numbers a1, a2, …, an are in
GP if and only if

T H E O R E M 5 .9 Let a1, a2, …, an be in GP with common ratio r.


1. For any non-zero constant l, la1, la2, …, lan are in GP with common ratio r.
2. For any real number b > 1, logb a1, logb a2, …, logb an are in AP with common difference logb r,
provided ai > 0 for 1 £ i £ n.
www.jeeneetbooks.in
218 Chapter 5 Progressions, Sequences and Series

PROOF 1. The first part is clear, since

lan a
= n = r for all n > 1
lan - 1 an - 1

2. The second part follows from the fact that

æ a ö
logb an - logb an - 1 = logb ç n ÷ = logb r for all n > 1
è an - 1 ø ■

T H E O R E M 5 .10 The sum of the first n terms of the GP with first term a and common ratio r ¹ 1 is given by

a(1 - r n )
sn =
1- r
PROOF The GP with first term a and common ratio r can be expressed as
a, ar, ar2, …
If sn is the sum of the first n terms, then
sn = a + ar + + ar n-1
sn (1 - r ) = a - ar n = a(1 - r n )
and therefore,

a(1 - r n )
sn = if r ¹ 1
1- r ■

QUICK LOOK 7

If the common ratio of a GP is 1, then the sum of the first n terms is na, where a is the first term.

DEF IN IT ION 5 . 14 Let

a(1 - r n )
sn =
1- r

be the sum of the first n terms of a GP with first term a and common ratio r. If | r | < 1, then
a
lim sn =
n®¥ 1- r
is called the sum to infinity of the GP and this will be generally denoted by s¥.

Example 5.5

Consider the sequence Solution: The nth term of the sequence is (1/4)n. The
sequence is a GP with first term 1/4 and common ratio
1 1 1 1 1/4. Therefore,
, , , ,…
4 16 64 256
1 - (1/ 4)n 4n - 1 1 4
Calculate the sum of first n terms and the sum to infinity. sn = = and s¥ = =
1 - (1/ 4) 3× 4n-1 1 - (1/ 4) 3
www.jeeneetbooks.in
5.3 Geometric Progressions 219

Geometric Mean
DEF IN IT ION 5 . 15 If three numbers a, b and c are in GP, then b is called the geometric mean (GM) of a and c or
geometric mean between a and c.

QUICK LOOK 8

1. A number b is the GM between a and c if and only if or, equivalently,


b c b2 = ac
=
a b 2. If x and y are any non-negative real numbers, then
x, xy , y are in GP.

DEF IN IT ION 5 . 16 If a, g1, g2, …, gn, b are in GP, then g1, g2, …, gn are called n geometric means or, simply, n GMs
between a and b.

In the following we discuss the insertion of n GM’s between two given non-zero real numbers, where n is a given posi-
tive integer.

T H E O R E M 5 .11 Let a and b be two given non-zero real numbers and n a positive integer. If
k /( n + 1)
æ bö
gk = a ç ÷ for 1 £ k £ n
è aø
then g1, g2, …, gn are n geometric means between a and b.
PROOF Let
k /( n + 1)
æ bö
gk = a ç ÷ for 1 £ k £ n
è aø

and consider a, g1, g2, …, gn, b. Then


1/( n + 1)
g1 æ b ö
=ç ÷
a è aø
1/( n + 1)
g2 a(b/a )2 /( n+1) æ b ö
= =ç ÷
g1 a(b/a )1/( n+1) è a ø
1/( n + 1)
gk a(b/a )k /( n+1) æ bö b
= =ç ÷ =
gk -1 a(b/a)( k - 1)/( n + 1)
è aø gn
Therefore a, g1, g2, …, gn, b are in GP with common ratio (b/a)1(n+1) and hence g1, g2, …, gn are the
n GMs between a and b. ■

QUICK LOOK 9

If g1, g2, …, gn are n geometric means between a and b, æ bö


( 1+ 2 + + n )/( n + 1)

then their product is given by = an ç ÷


è aø
g1 , g2 , …, gn = ( ab )n æ bö
n/2

= an ç ÷ = ( ab )n
since è aø
k /( n + 1)
n n
æ bö
Õ
k =1
gk = Õ
k =1
aç ÷
è aø
www.jeeneetbooks.in
220 Chapter 5 Progressions, Sequences and Series

Example 5.6

Insert 8 geometric means between 1 and 16. and hence gk = (16)k/9. Therefore,

Solution: Let 1, g1, g2, …, g8, 16 be in GP. Then (16)1/ 9, (16)2 / 9, …, (16)8 / 9

k /( 8 + 1) are the 8 GMs between 1 and 16.


æ 16 ö
gk = 1ç ÷ for 1 £ k £ 8
è 1ø

Arithmetic Geometric Progression


DEF IN IT ION 5 . 17 A sequence of the form
a, (a + d)r, (a + 2d)r 2, (a + 3d)r 3, …
is called arithmetic geometric progression (AGP) the nth term in AGP is [a + (n - 1)d]rn-1,
where d and r are non-zero real numbers.

T H E O R E M 5 .12 The sum of the first n terms of an AGP is given by

a dr(1 - r n-1 ) (a + (n - 1)d)r n


sn = + -
1- r (1 - r )2 1- r
If | r | < 1, the sum to infinity is
a dr
s¥ = +
1 - r (1 - r )2

PROOF Let a, (a + d)r, (a + 2d)r2, … be an AGP and sn be the sum of first n-term; that is,
sn = a + (a + d)r + (a + 2d)r 2 + + [a + (n - 1)d]r n-1
Then
rsn = ar + (a + d)r 2 + (a + 2d)r 3 + + [a + (n - 1)d]r n
Now using the two equation we get
dr(1 - r n-1 )
(1 - r )sn = sn - rsn = a + - [a + (n - 1)d]r n
1- r
a dr(1 - r n--1 ) [a + (n - 1)d]r n
sn = + -
1- r (1 - r )2 1- r
If | r | < 1, then
a dr
s¥ = lim sn = +
n®¥ 1 - r (1 - r )2 ■

Example 5.7

Find the sum to infinity of the series which is an arithmetic geometric progression with a = 1,
d = 3 and r = 1/5. Since | r | < 1, the sum to infinity of the
4 7 10
1+ + + + AGP is given by
5 52 53
a dr 1 3(1/ 5) 5 15 35
+ = + = + =
Solution: The given series is of the form 1 - r (1 - r )2 1 - (1/ 5) [1 - (1/ 5)]2 4 16 16
a, (a + d)r, (a + 2d)r 2, …
www.jeeneetbooks.in
5.4 Harmonic Progressions and Series 221

5.4 | Harmonic Progressions and Series


In this section we consider sequences in which the reciprocals of the terms form an arithmetic progression. Such
sequences are called harmonic progressions. Let us begin with the following.

DEF IN IT ION 5 . 18 A sequence of non-zero real numbers is said to be a harmonic progression (HP) if their
reciprocals form an arithmetic progression (AP). That is, {a1, a2, …} is said to be a HP if

(i) 0 ¹ an Î  for all n


(ii) {1/a1, 1/a2, …} is an AP

DEF IN IT ION 5 . 19 Non-zero real numbers a1, a2, …, an are said to be in HP if they are consecutive terms of a HP,
that is,
1 1 1 1
- = - for all 1 < i < n
ai ai-1 ai+1 ai

QUICK LOOK 10

The general form of an HP is where the nth term is


1 1 1 1
, , ,…
a a + d a + 2d a + (n - 1)d

Examples

(1) {1, 1/2, 1/3, 1/4, …} is an HP . is an HP, if a ¹ -nd for all n Î+.
(2) For any 0 < a Î and 0 ¹ d Î, (3) The numbers 1/3, 1/7, 1/11 are in HP .
ì1 1 1 ü
í , , , …ý
î a a + d a + 2d þ

Harmonic Mean
DEF IN IT ION 5 . 20 If a, b, c are in HP, then b is called the harmonic mean (HM) between a and c. Note that b is
the HM between a and c if and only if
2ac æ 1 1 1 1ö
b= çè i.e., - = - ÷
a+c b a c bø

Examples

(1) Note that 1/5 is the HM of 1/3 and 1/7. (2) If y is the AM of x and z, then 1/y is the HM of 1/x
and 1/z.

DEF IN IT ION 5 . 21 h1, h2, …, hn are said to be n harmonic means between two given real numbers a and b if a, h1,
h2, …, hn, b are in HP.

T H E O R E M 5 .13 If h1, h2, …, hn are n HMs between two non-zero real numbers a and b, then
ab(n + 1)
hK = for any 1 £ K £ n
b(n + 1) + K (a - b)
www.jeeneetbooks.in
222 Chapter 5 Progressions, Sequences and Series

PROOF Suppose that a, h1, h2, …, hn, b are in HP. Then


1 1 1 1 1
, , , …, ,
a h1 h2 hn b
are in AP. If d is the common difference of this AP, then
1 1 1 1 1
= + d, = + 2d, …, hn = + nd
h1 a h2 a a
1 1
and = + (n + 1)d
b a
and so

1 é 1 1ù a-b
d= ê - ú=
n + 1 ë b a û (n + 1)ab
Therefore

1 1 æ a - b ö b(n + 1) + K (a - b)
= + Kç =
hK a è (n + 1)ab ÷ø (n + 1)ab

(n + 1)ab
hK =
b(n + 1) + K (a - b) ■

T H E O R E M 5 .14 If A, G and H are the arithmetic, geometric and harmonic means, respectively, between two
positive real numbers a and b, then
AH = G2
that is, A, G, H are in GP or G is the GM of A and H.
PROOF Since A, G and H are the arithmetic, geometric and harmonic means, respectively, we have
a+b 2ab
A= , G = ab and H =
2 a+b
Therefore,
a + b 2ab
AH = × = ab = G2
2 a+b ■

Note: In Theorem 5.14, one has to take a and b to be non-zero, but in this theorem a and b must be positive. Also, note
that A ³ G ³ H and that the equality holds at the two places if and only if a = b. These are proved in the more general
cases later (see Theorem 5.15).
Some inequality problems and maxima and minima problems can be solved by using the inequalities A ³ G ³ H,
where A, G and H are arithmetic, geometric and harmonic means, respectively, of two positive real numbers. Let us
begin with the following.

DEF IN IT ION 5 . 22 Let a1, a2, …, an be positive real numbers (n ³ 2). Then

(i) (a1 + a2 + + an)/n is called the arithmetic mean (AM)


(ii) (a1a2 an)1/n is called the geometric mean (GM)
(iii) n/[(1/a1) + (1/a2) + + (1/an)] is called the harmonic mean (HM)

T H E O R E M 5 .15 If a1, a2, …, an (n ≥ 2) are positive real numbers and A and G be their AM and GM, respectively,
then A ³ G and the equality holds if and only if ai = aj for all 1 £ i, j £ n.
www.jeeneetbooks.in
5.4 Harmonic Progressions and Series 223

PROOF We will use mathematical induction on n. If a1, a2, …, an are all equal to each other, then clearly
A = G = a1. Suppose that, not all the ais are equal. For n = 2
a1 + a2
A= and G = a1a2
2
so that

( a1 - a2 )2
A-G= >0
2
and hence A > G. Now, consider the case n = 3. Let a1, a2, a3 be positive real numbers and let
x = (a1 )1/ 3, y = (a2 )1/ 3 and z = (a3 )1/ 3
Then x, y and z are positive and
a1 + a2 + a3
A-G= - (a1a2a3 )1/ 3
3
1
= ( x3 + y3 + z3 - 3 xyz)
3
1
= ( x + y + z)( x2 + y2 + z2 - xy - yz - zx)
3
1
= ( x + y + z)(( x - y)2 + ( y - z)2 + (z - x)2 ) ³ 0
6
Hence A ³ G. Also, A = G if and only if x = y = z and hence a1 = a2 = a3. Therefore the theorem is
valid for n = 2 and n = 3.
Now, let n > 3 and assume that the theorem is valid for any n - 1 positive real numbers. Let
a1, a2, …, an be any positive real numbers that are not all equal. We can suppose that a1 ³ a2 ³ ³ an
and a1 > an. Let
a1 + a2 + + an
A= and G = (a1a2 an )1/ n
n

Consider a2, a3, …, an-1, a1an/G (which are n - 1 in number). By the induction hypothesis,
1/( n - 1)
1 æ a1an ö æ a1an ö
çè a2 + a3 + + an-1 + ÷ø > çè a2 a3 an-1 ÷
n-1 G G ø
a1an æ aa ö
a2 + a3 + + an-1 + > (n - 1) ç a2 a3 an-1 1 n ÷ = (n - 1)G
G è G ø
Therefore
a1an
nG < G + a2 + a3 + + an - 1 +
G
a1an
= G + nA - (a1 + an ) +
G
G2 - (a1 + an )G + a1an
= nA +
G
(G - a1 )(G - an )
= nA +
G
< nA (since a1 > G > an )
Therefore G < A. ■
www.jeeneetbooks.in
224 Chapter 5 Progressions, Sequences and Series

Note: If any two of the ais are not equal, say a1 ¹ a2, then we can write
a1 + a2 + + an [(a1 + a2 )/ 2] + [(a1 + a2 )/ 2] + a3 + a4 + + an
=
n n
1/ n
ææ a + a ö2 ö
³ ç ç 1 2 ÷ a3 a4 an ÷
èè 2 ø ø

> (a1 a2 a3 a4 an )1/ n

C O R O L L A R Y 5 .1 If a1, a2, …, an are positive real numbers such that their sum is a fixed positive real numbers, then
their product is greatest when each of
s
ai = (i = 1, 2, …, n)
n

PROOF By Theorem 5.15,


a1 + a2 + + an ³ n(a1 a2 ¼ an )1/ n
where equality holds if
s
a1 = a2 = a3 = = an =
n
Therefore greatest value of a1a2a3 an is (s/n)n. ■

C O R O L L A R Y 5 .2 If a1, a2, …, an are positive real numbers such that their product is a fixed positive real number P,
then their sum is least when each of a1, a2, …, an is equal to P1/n.
PROOF The following equality
a1 + a2 + + an
³ (a1 a2 an )1/ n = P1/ n
n
holds when each ai = P1/n. Therefore, the least value of a1 + a2 + + an is nP1/n. ■

The following formulae and the methods of their derivation will help us in finding the sum to n terms of certain series.

Example 5.8

If a, b, c are in HP, then show that a : a - b = a + c : a - c. a-b b-c


=
ab bc
Solution: If a, b, c are in HP then 1/a, 1/b, 1/c are in AP.
a - b b - c (a - b) + (b - c) a - c
Therefore = = =
a c a+c a+c
1 1 1 1
- = - From the first and the last fractions we get
b a c b
a:a-b=a+c:a-c

Example 5.9

Find the harmonic mean of the roots of the quadratic 4+ 5 2(4 + 5 )


equation a+b= and ab =
5+ 2 5+ 2
(5 + 2 ) x2 - (4 + 5 ) x + 2(4 + 5 ) = 0 Therefore, the harmonic mean of a and b is

Solution: Let a and b be the roots of the given equa- 2ab


=4
tion. Then a+b
www.jeeneetbooks.in
5.5 Some Useful Formulae 225

5.5 | Some Useful Formulae


I. Telescopic Series: Suppose that we have to find the sum to n terms of a series u1 + u2 + u3 + . If a1 + a2 + a3 +
is another series such that
uK = aK - aK +1 for all K
then u1 + u2 + + un = a1 - an+1
1 1 1
For example, consider + + + . Here, we have
2 × 3 3× 4 4 × 5
1 1 1
= - for all K ³ 2
K (K + 1) K K + 1
II. Suppose that the nth term un of a given series is the product of r successive terms of an AP beginning with the nth
term of the AP; that is, suppose that
un = [a + (n - 1)d](a + nd) [a + (n + r - 2)d]
By choosing an = un[a + (n + r - 1)d], we can write
1
un = [an - an-1 ]
(r + 1)d
so that the sum to n terms is equal to
1
(an - a0 )
(r + 1)d
For example, consider
(i) 1× 2 × 3 + 2 × 3× 4 + 3× 4 × 5 +
(ii) 1× 3× 5× 7 + 3× 5× 7 × 9 + 5× 7 × 9 × 11 +
III. Suppose that the nth term of a series is the reciprocal of the nth term of the series given in II; that is,
1
un =
[a + (n - 1)d][a + nd] [a + (n + r - 2)d]

Then, we can choose


an = un [a + (n - 1)d]
so that
1
un = (an-1 - an )
(r - 1)d
and sum to n terms is given by
1
(a0 - an )
(r - 1)d
For example, consider
1 1 1
(i) + + +
1× 4 × 7 4 × 7 × 10 7 × 10 × 13
1 1 1
(ii) + + +
1× 3× 5× 7 3× 5× 7 × 9 5× 7 × 9 × 11
IV. Successive Differences Method: Suppose that in a given series, we cannot express the nth term by using induction.
But when we take the successive differences of the series, ultimately we may arrive at an AP or a GP. Then we can
find the nth term by the following method.
www.jeeneetbooks.in
226 Chapter 5 Progressions, Sequences and Series

Let the given series be u1 + u2 + u3 + . Let

Dun = (u2 - u1 ) + (u3 - u2 ) + (u4 - u3 ) +


     
v1 v2 v1
D2un = (v2 - v1 ) + (v3 - v1 ) + (v4 - v3 ) +

(i) Suppose that DKun becomes a series where all the terms are equal. Afterwards DK+1un becomes a series in
which each term is zero. We stop at DKun. Let the first terms of Dun, D2un, D3un, …, DKun be d1, d2, d3, …, dn,
respectively. Then

d1 (n - 1) d2 (n - 1)(n - 2) d3 (n - 1)(n - 2)(n - 3)


un = u1 + + + +
1! 2! 3!
DKun is called the Kth-order differences of the given series.

Example

Let us find the sum to n terms of the series 2, 10, 30, 68, Now,
130, 222, 350, …
8(n - 1) 12(n - 1)(n - 2) 6(n - 1)(n - 2)(n - 3)
Dun = 8, 20, 38, 62, 92, 128, … un = 2 + + +
1! 2! 3!
D2un = 12, 18, 24, 30, 36, … = 2 + 8 n - 8 + 6(n2 - 3n + 2) + (n2 - 3n + 2)(n - 3)
D3un = 6, 6, 6, 6, …
= n3 + n
D un = 0, 0, 0,…
4
One can check that n3 + n is the general term by giving
values 1, 2, 3, … for n.

(ii) Suppose that DK+1un forms a geometric progression. In this case, the nth term un is of the form
ar n-1 + a0 + a1 (n - 1) + a2 (n - 1)(n - 2) +
where r is the common ratio and the values of a, a0, a1, a2, … can be evaluated by giving values 1, 2, 3, … to n
and equating the terms to the corresponding terms of the given series.

Example 5.10

Consider the series 6 + 9 + 14 + 23 + 40 + . Find the nth Taking n = 3, we get


term and sum to n terms.
14 = u3 = 4a + a0 + 2a1 (5.3)
Solution: We have Dun = 3, 5, 9, 17, … . Therefore Solving Eqs. (5.1)-(5.3), we get that a = 2, a0 = 4 and
a1 = 1. Therefore
D2un = 2, 4, 8,…
un = 2n + 4 + (n - 1) = 2n + n + 3
which is a GP. Therefore
Hence the sum to n terms is given by
un = a 2n-1 + a0 + a1 (n - 1)
Taking n = 1, we get
n
n(n + 1)
å (2
K =1
K
+ K + 3) = (2 + 22 + + 2n ) +
2
+ 3n
6 = u1 = a + a0 (5.1)
n(n + 1)
Taking n = 2, we get = 2n+1 - 2 + + 3n
2
9 = u1 = 2a + a0 + a1 (5.2)
www.jeeneetbooks.in
Worked-Out Problems 227

WORKED-OUT PROBLEMS
Single Correct Choice Type Questions
1. Sum of the first 20 terms of the AP Solution: Using the formula for sum of an AP and
substituting the values, we get
1 1 3
2, 3 , 4 , 5 ,…
4 2 4 15
600 = (2a + 14 ´ 5)
is 2
1 1 a + 35 = 40
(A) 274 (B) 277 (C) 277 (D) 274
2 2
a=5
Solution: First term a and the common difference d Answer: (D)
are given by
1 5. If 17 arithmetic means are inserted between 7/2 and
a = 2 and d = 1 -83/2, then the 17th AM is
4
(A) -19 (B) -29 (C) -39 (D) -49
Therefore the sum of the first 20 terms is given by
Solution: Let d be the common difference. Then
20 æ 5ö 1
ç 4 + 19 ´ ÷ø = 277 (- 83 / 2) - (7 / 2) 5
2 è 4 2 d= =-
18 2
Answer: (B)
Therefore the 17th mean is given by (using Theorem 5.6)
2. The third term of an AP is 18 and the seventh term is
30, then the sum of the first 17 terms is 7 æ 5ö
+ 17 ç - ÷ = - 39
2 è 2ø
(A) 812 (B) 512 (C) 612 (D) 712
Answer: (C)
Solution: Let “a” be the first term and “d” the common
difference. Then
6. The number of terms of the series 26, 21, 16, 11, … to
a + 2d = 18 (5.4) be added so as to get the sum 74 is
a + 6d = 30 (5.5) (A) 5 (B) 4 (C) 3 (D) 6

Solving Eqs. (5.4) and (5.5), we get a = 12, d = 3. Therefore Solution: In the given series a = 26, d = -5. Suppose
sum of 17 terms is sum of the first n terms is 74. This implies
n
17
(24 + 16 ´ 3) = 17 ´ 36 = 612 74 = [52 + (n - 1)(- 5)]
2 2
Answer: (C) 148 = n(57 - 5n)
5n - 57 n + 148 = 0
2
3. The nth term of an AP is 4n + 1. The sum of the first
15 terms is 5n - 20 n - 37 n + 148 = 0
2

(A) 495 (B) 555 (C) 395 (D) 695 5n(n - 4) - 37(n - 4) = 0
Solution: The progression is: 5, 9, 13, 17, …. Here the n=4
first term is a = 5, the common difference is d = 4 and
Answer: (B)
the number of terms is n = 15. Then the sum of the first
15 terms is
7. The sum of the first n terms of two arithmetic series
15 are in the ratio (7n + 1) : (4n + 27). The ratio of their
(10 + 14 ´ 4) = 15 ´ 33 = 495 11th terms is
2
Answer: (A) (A) 2 : 3 (B) 3 : 2 (C) 3 : 4 (D) 4 : 3
Solution: It is given that
4. The sum of the first 15 terms of an AP is 600 and the
common difference is 5. The first term is equal to Sn 7n + 1
=
(A) 8 (B) 9 (C) 10 (D) 5 ¢
Sn 4 n + 27
www.jeeneetbooks.in
228 Chapter 5 Progressions, Sequences and Series

Therefore, using Theorem 5.8 we get Therefore


Tn 7(2 n - 1) + 1 14 n - 6 k (2 y - x) k ( y - 2 x)
= = Ak = Ak¢ Þ x + = 2x +
Tn¢ 4(2 n - 1) + 27 8 n + 23 n+1 n+1
Substituting n = 11 in this equation we get Þ k(2 y - x) = x(n + 1) + k( y - 2 x)
T11 148 4 Þ ky = ( n + 1 - k ) x
= =
T11¢ 111 3 Answer: (B)

Answer: (D)
10. Between two numbers whose sum is 2 61 , an even
number of arithmetic means is inserted; the sum of
8. If S1, S2, S3, … , Sp are the sums of first n terms of p
these means exceeds their number by unity. Then,
arithmetic progressions whose first terms are 1, 2,
the number of means is
3, …, p and whose common differences are 1, 3, 5, 7, …,
then the value of S1 + S2 + S3 + + Sp is (A) 8 (B) 10 (C) 12 (D) 16
(A) np(np + 1)/2 (B) np(np - 1)/2 Solution: Let x and y be the given numbers so that
(C) (np - 1) (np + 1)/2 (D) n(np + 1)
13
x+ y= (5.6)
Solution: We have 6
n n(n + 1) Let A1, A2, …, A2n be the means between x and y. Then
S1 = [2 + (n - 1)1] =
2 2
æ x + yö
(2 n) ç = Sum of the means = 2n + 1
n
S2 = [4 + (n - 1)3] =
n(3n + 1) è 2 ÷ø
2 2
Therefore by Eq. (5.6)
n n(5n + 1)
S3 = [6 + (n - 1)5] =
2 2 æ 13 ö
n ç ÷ = 2n + 1
è 6ø
n n[(2k - 1) n + 1]
Sk = [2k + (n - 1)(2k - 1)] =
2 2 This implies n = 6 and 2n = 12 is the number of means.
Therefore Answer: (C)
p
n
åS
k =1
k = [n {1 + 3 + 5 + + (2 p - 1)} + p]
2
11. The number of terms in an AP is even. The sum of
the odd terms is 24 and the sum of the even terms
n is 30. If the last term exceeds the first term by 10 21 ,
= (np2 + p) then the number of terms in the AP is
2
np (A) 6 (B) 8 (C) 10 (D) 12
= (np + 1)
2 Solution: Let a, a + d, …, a + (2n - 1)d be the 2n num-
Answer: (A) bers. Therefore, by hypothesis
n
9. A total of n arithmetic means are inserted between 24 = Sum of the odd terms = [2a + (n - 1) 2d]
2
x and 2y and further n arithmetic means are inserted
between 2x and y. If the kth arithmetic means of both This gives
sets are equal, then a relation between x and y is
n[a + (n - 1)d] = 24 (5.7)
(A) ky = (n - k)x (B) ky = (n + 1 - k)x
(C) k(y + 1) = (n - k)x (D) k(y + 1) = (n + 1 - k)x Also, again by hypothesis,
n
Solution: Let Ak and Ak¢ be the kth AMs between x and 30 = Sum of the even terms = [2(a + d) + (n - 1) 2d]
2y and 2x and y, respectively. Then 2

(2 y - x) This gives
Ak = x + k
n+1 n(a + nd) = 30 (5.8)
k ( y - 2 x) Now it is given that the last term exceeds the first term
and Ak¢ = 2 x + (Theorem 5.7)
n+1 by 10 21 , so
www.jeeneetbooks.in
Worked-Out Problems 229

21 Therefore the sum of the first (m + n) terms is given by


a + (2 n - 1)d = a +
2
m + n é ì b(m - 1) - a(n - 1) ü (a - b) ù
21 ê2 í ý + (m + n - 1) ú
(2 n - 1)d = (5.9) 2 ë î m-n þ m-n û
2
From Eqs. (5.7) and (5.8) we get m+né 1 ù
= ê [2b(m - 1) - 2a(n - 1) + (a - b)(m + n - 1)]ú
30 - nd = 24 2 ëm - n û
(5.10)
nd = 6 m+né 1 ù
= [bm - bn - b - an + am + a]ú
From Eqs. (5.9) and (5.10) we have 2 êë m - n û
21
12 - d = m+né 1 ù
2 = [m(a + b) - n(a + b) + (a - b)]ú
2 êë m - n û
3 æ 3ö
d= and 6 = nd = n ç ÷ Þ n = 4
2 è 2ø
m+né 1 ù
= ê [(a + b)(m - n) + (a - b)]ú
Therefore the number of terms is 2n = 8. 2 ëm - n û
Answer: (B)
m+né a-bù
= ê(a + b) +
12. The mth term of an AP is a and its nth term is b, and 2 ë m - n úû
m ¹ n. Then the sum of the first (m + n) terms of the AP is
Answer: (A)
m+né a-bù
(A) ê a+b+
2 ë m - n úû 13. The sum of the first and fifth terms of an AP is 26
and the product of the second and fourth is 160. Then
m+né a-bù the sum of the first six terms of the progression is
ê(a + b) -
m - n úû
(B)
2 ë (A) 59 or 69 (B) 69 or 87
m+né a+bù (C) 87 or 109 (D) -69 or 87
(a + b) +
2 êë m + n úû
(C)
Solution: Let a be the first term and d the common
m+né (a - b) ù difference, then
ê(a - b) -
m + n úû
(D)
2 ë a + (a + 4d) = 26
Solution: Let a be the first term and d the common Þ a + 2d = 13 (5.12)
difference. Then
(a + d)(a + 3d) = 160 (5.13)
a + (m - 1) d = a
(5.11) (13 - d)(13 + d) = 160 [from Eq. (5.12)]
a + (n - 1) d = b
Therefore 169 - d2 = 160

a-b d = ± 3 and a = 7, 19
d=
m-n Therefore the sum of the first six terms = 69, 87.
Substituting the value of d in the first equation of Eq. (5.11), Answer: (B)
we get
14. If the sum of first n terms of an AP is cn2, then the
(m - 1)(a - b)
a =a- sum of the squares of these terms is
m-n
(A) [n(4n2 - 1)c2]/6 (B) [n(4n2 + 1)c2]/3
a(m - n) - (m - 1)(a - b) (C) [n(4n2 - 1)c2]/3 (D) [n(4n2 + 1)c2]/6
=
m-n
Solution: Let an be the nth term. Therefore
am - an - am + a + bm - b
= an = (sum of first n terms) - [sum of first (n - 1) terms]
m-n
b(m - 1) - a(n - 1) = cn2 - c(n - 1)2
=
m-n = c(2 n - 1)
www.jeeneetbooks.in
230 Chapter 5 Progressions, Sequences and Series

This gives Solution: The sum s2n = 2 + 5 + 8 + upto 2n terms is


n n
2n
åa
k =1
2
k = c2 å (2k - 1)2
k =1 2
[4 + (2 n - 1)3] = n(6 n + 1)

n Now the sum sn¢ = 57 + 59 + 61 + upto n terms is


= c2 å (4k 2 - 4k + 1)
n
k =1 [114 + (n - 1)2] = n(n + 56)
2
é n n
ù
= c2 ê 4 å k 2 - 4 å k + n ú Therefore
ë k =1 K =1 û
s2 n = sn¢ Þ n(6 n + 1) = n(n + 56)
é 4 n(n + 1)(2 n + 1) 4 n(n + 1) ù
= c2 ê - + nú Þ 5n2 - 55n = 0
ë 6 2 û
Þ n = 11
é 2 n(n + 1)(2 n + 1) ù
= c2 ê - n(2 n + 1)ú Answer: (C)
ë 3 û
n(2 n + 1) 17. In an AP, if sn is the sum of the first n terms (n is odd)
= [2(n + 1) - 3]c2 and sn¢ is the sum of the first n odd terms, then sn / sn¢ =
3
(A) 2n/n + 1 (B) n/n + 1
n(4 n2 - 1)c2
= (C) n + 1/2n (D) n + 1/n
3
Answer: (C) Solution: Let a be the first term and d the common
difference. Then
15. If the numbers 32a -1, 14, 34-2a (0 < a < 1) are the first n
three terms of an AP, then its fifth term is equal to sn = [2a + (n - 1)d]
2
(A) 33 (B) 43 (C) 53 (D) 63
é æn+1 ö ù
sn¢ = a + (a + 2d) + (a + 4d) + + êa + ç - 1÷ 2d ú
Solution: By hypothesis 32a -1 + 34-2a = 28. Therefore ë è 2 ø û
9a 81 æ n + 1ö
+ = 28 =ç [2a + (n - 1)d]
3 9a è 4 ÷ø
Substituting 9a = x, we get Therefore
x 81 sn (n / 2)[2a + (n - 1)d] n 4 2n
+ = 28 = = × =
3 x + + - + +1
sn¢ [( n 1 )/ 4][ 2 a ( n 1)d ] 2 n 1 n
x2 - 84 x + 243 = 0 Answer: (A)
( x - 81)( x - 3) = 0 18. The series of natural numbers is divided into groups
x = 81 or x=3 (1), (2, 3, 4), (5, 6, 7, 8, 9)… and so on. The sum of the
numbers in the nth group is
This gives
(A) n3 + (n + 1)3 (B) (n - 1)3 + n3
9a = 81 or 9a = 3
(C) n + 1 + (n - 1)
3 3
(D) (n + 1)3 + (n - 1)3
1
a = 2 or Solution: Clearly the nth group contains 2n - 1 numbers.
2
The last terms of each group are 12, 22, 32, … and hence the
1 last term of nth group is n2. Also, the first term of each
0 < a < 1Þ a =
2 group is one more than the last term of its previous group.
Therefore, the numbers are 1, 14, 27, which are in AP with Therefore the first term of the nth group is
common difference 13. The fifth term is 1 + 4 ´ 13 = 53. (n - 1)2 + 1
Answer: (C) Hence the sum of the numbers in the nth group is
16. If the sum of the first 2n terms of the AP, 2, 5, 8, 11, … 2n - 1
[(n - 1)2 + 1 + n2 ] = (2 n - 1)(n2 - n + 1)
is equal to the sum of the first n terms of the AP. 2
57, 59, 61, 63, …, then n is equal to
= (n - 1)3 + n3
(A) 10 (B) 12 (C) 11 (D) 13
Answer: (B)
www.jeeneetbooks.in
Worked-Out Problems 231

19. If log10 2, log10 (2x - 1), log10 (2x + 3) are in AP, then Solution: Let a and b be the positive numbers. Then
(A) x = 1 (B) x = 2 a+b 3 75
= 18 Þ a + b =
(C) x = log2 5 (D) x = log10 5 2 4 2
Solution: By hypothesis log10 2, log10 (2x − 1), log10 (2x + 3) and ab = 15 Þ ab = 225
are in AP and so
Therefore
log10 2 + log10 (2x + 3) = 2 log10 (2x - 1)
2
log10 2(2x + 3) = log10 (2x - 1)2 æ 75 ö
a - b = ± ç ÷ - 4ab
è 2ø
2(2x + 3) = (2x - 1)2
2
22 x - 4 × 2 x - 5 = 0 75
=± - 4 ´ 152
4
a2 - 4 a - 5 = 0 (where a = 2x )
(a - 5)(a + 1) = 0 æ 75 ö æ 75 ö
= ± ç + 30÷ ç - 30÷
è 2 øè 2 ø
2x = - 1 or 2x = 5
But 2x cannot be negative. Therefore 135 ´ 15 45
=± =±
2 2
2x = 5
Case 1: a + b = 75/2 and a - b = 45/2
x = log2 5
Solving the two equations we get a = 30 and b = 15/2
Answer: (C)
Case 2: a + b = 75/2 and a - b = -45/2. Solving the two
equations we get
20. In a sequence a1, a2, a3, … of real numbers it is observed
that ap = 2 , aq = 3 and ar = 5 , where p, q, r are 15
a= and b = 30
positive integers such that 1 £ p < q < r. Then 2
(A) ap, aq, ar can be terms of an AP Therefore larger of the numbers is 30.
(B) 1/ap, 1/aq, 1/ar can be terms of an AP Answer: (D)
(C) ap, aq, ar can be terms of an AP if and only if
p, q, r are perfect-squares 22. The sum of the integers from 1 to 100 which are
(D) Neither ap, aq, ar are in AP nor 1/ap, 1/aq, 1/ar are divisible by exactly one of 2 and 5 is
in AP (A) 2505 (B) 1055 (C) 2550 (D) 3050
Solution: Suppose Solution: Let A, B and C be the set of all integers from
1 to 100 that are divisible by 2, 5 and 10, respectively.
2 = ap = a + (l - 1)d Therefore
3 = aq = a + (m - 1)d A = {2, 4, 6, … , 100}, B = {5, 10, 15, … , 100}

5 = ar = a + (n - 1)d and C = {10, 20, 30, … , 100}

where l, m, n are positive integers in the increasing order. Clearly, C = A Ç B. Therefore


Therefore (i) A contains 50 numbers which are in AP with first
term 2 and common difference 2.
(m - l )d = 3 - 2 and (n - m)d = 5 - 3
(ii) B contains 20 numbers that are in AP with first
and so term 5 and common difference 5.
m-l 5- 3 (iii) C = A Ç B contains 10 numbers which are in AP
= with common difference 10 and first term 10.
n-m 3- 2
which is absurd because left-hand side (LHS) is rational. Therefore the required sum is
Answer: (D) 50 20
å x + å y - åz =
x ÎA yÎB zÎC 2
(2 + 100) + (5 + 100)
2
21. The arithmetic mean of two numbers is 18 43 and the
positive square root of their product is 15. The larger 10
- (10 + 100) = 3050
of the two numbers is 2 Answer: (D)
(A) 24 (B) 25 (C) 20 (D) 30
www.jeeneetbooks.in
232 Chapter 5 Progressions, Sequences and Series

23. If the sum of the roots of the quadratic equation Since b is a root of the given equation we get
ax2 + bx + c = 0 is equal to the sum of the squares of
10 b 3 - ab 2 - 54 b - 27 = 0
their reciprocals, then
(A) bc2, ca2, ab2 are in AP Substituting the value of b we get
3 2
(B) bc2, ab2, ca2 are in AP æ 3ö æ 3ö æ 3ö
10 ç - ÷ - a ç - ÷ - 54 ç - ÷ - 27 = 0
(C) ca2, bc2, ab2 are in AP è 2ø è 2ø è 2ø
(D) ab, bc, ca are in AP æ 27 ö 9a
10 ç - ÷ - + 81 - 27 = 0
Solution: Let a and b be the roots of the equation. è 8ø 4
Then 9a 10 ´ 27
=- + 54
b c 4 8
a + b = - , ab =
a a æ 30 ö 18
a = 4ç - + 6÷ = =9
Now è 8 ø 2
Answer: (D)
1 1
a+b= 2
+ 2
a b 25. The fifth and 31st terms of an AP are, respectively,
b (b2 /a2 ) - (2c / a) 1 and -77. If kth term of the given AP is -17, then k is
Þ- = (A) 12 (B) 10 (C) 11 (D) 13
a c2 /a2
b b2 - 2ca Solution: The fifth term is 1; therefore,
- =
a c2 a + 4d = 1 (5.14)
- bc = ab - 2ca
2 2 2
The 31st term is -77; therefore,
2ca2 = ab2 + bc2 a + 30d = -77 (5.15)
2 2 2
Therefore, bc , ca , ab are in AP. Solving Eqs. (5.14) and (5.15) we get
Answer: (A) 26d = 78 Þ d = - 3
Substituting the value of d in either equation we get
24. If the reciprocals of the roots of the equation
10x - ax - 54x - 27 = 0 are in AP, then the value of a is
3 2
a = 13
(A) 6 (B) 8 (C) -9 (D) 9 Now the kth term of the given AP is -17; therefore,
Solution: Let a , b, g be the roots of the given equation. a + (k - 1)d = - 17
Then
Þ 13 - 3(k - 1) = - 17
a 54 27
a + b + g = , ab + bg + ga = - , abg = Þ 3k = 33
10 10 10
Þ k = 11
Now the reciprocals of the roots of the equation are in
AP, that is Answer: (C)

1 1 2 26. The sum of the four arithmetic means between 4 and


+ =
a g b 40 is
Adding 1/b to both the sides we get (A) 90 (B) 88 (C) 108 (D) 118
Solution: Sum of the four means is given by
1 1 1 3
+ + = (4 + 40)
a b g b 4 = 88
2
bg + ga + ab 3
= Answer: (B)
abg b
54 3 27. In an increasing arithmetic progression, the sum
- = of the first three terms is 27 and the sum of their
27 b
squares is 275. The common difference of the AP is
3 (A) 6 (B) 8 (C) 2 (D) 4
b=-
2
www.jeeneetbooks.in
Worked-Out Problems 233

Solution: Let a - d, a, a + d be the first three terms. 30. The ratio of sum of m terms to the sum of n
Since the terms are increasing, d > 0. By hypothesis terms of an AP is m2 : n2. If Tk is the kth term, then
T5 /T2 is
3a = 27 Þ a = 9
(A) 6 (B) 5 (C) 4 (D) 3
(a - d)2 + a2 + (a + d)2 = 275
Solution: By hypothesis
and 3a2 + 2d2 = 275
sm m2
d = 16 Þ d = ± 4
2 =
sn n2
Since d > 0, we get d = 4. Also
Answer: (D)
Tm sm - sm-1 m2 - (m - 1)2 2 m - 1
= = 2 =
28. If 52 × 54 × 56 52n = (0.04)-28, then n is equal to Tn sn - sn-1 n - (n - 1)2 2n - 1
(A) 7 (B) 5 (C) 6 (D) 3 Substituting m = 5 and n = 2 in this equation we get
Solution: The given equation can be written as T5 2(5) - 1 9
= = =3
(5 ) × (5 ) × (5 ) (5 ) = (5 )
2 2 2 2 3 2 n 2 28 T2 2(2) - 1 3
Answer: (D)
251+ 2 + 3+ + n = 2528
25[ n( n+1)/ 2 ] = 2528 31. The sum of the first eight terms of a GP whose nth
term is 2 × 3n(n = 1, 2, 3, …) is
Since the bases are the same, equating the powers we get
(A) 19880 (B) 19860 (C) 19660 (D) 19680
n(n + 1)
= 28 Solution: The terms of the GP are
2
2 × 3, 2 × 32, 2 × 33, …, 2 × 38
n(n + 1) = 56
First term is 6 and the common ratio is 3. Therefore the
(n + 8)(n - 7) = 0
sum of the first 8 terms is
n = - 8, 7 6(1 - 38 )
= 19680
Now n = -8 is not possible. Hence n = 7. 1- 3
Answer: (A) Answer: (D)

29. The interior angles of a polygon are in AP. The 32. The difference between the fourth and the first term
smallest angle is 120° and the common difference of a GP is 52 and the sum of the first three terms
is 5°. The number of sides of the polygon is is 26. Then the sum of the first six terms is
(A) 11 (B) 9 (C) 12 (D) 13 (A) 720 (B) 725 (C) 728 (D) 780
Solution: Sum of the interior angles of a polygon of Solution: Let the GP be a, ar, ar2, … . Then by hypothesis
n sides equals (2n - 4) right angles. Therefore
ar 3 - a = 52
n
[240 + 5(n - 1)] = (2 n - 4)90 a + ar + ar 2 = 26
2
r3 - 1 52
5n2 - 125n + 720 = 0 = =2
1 + r + r 2 26
n2 - 25n + 144 = 0 Therefore
(n - 9)(n - 16) = 0 r -1= 2Þr = 3
Now this gives two values of n = 9 and 16. We take n = 9, Using this value we get a = 2. Therefore the sum of the
as n = 16 is rejected because the last angle becomes first six terms is
120° + (15 ´ 5)° = 195° 2(36 - 1)
= 728
Therefore number of sides = 9. 3-1
Answer: (B) Answer: (C)
www.jeeneetbooks.in
234 Chapter 5 Progressions, Sequences and Series

33. The sequence {an} is a GP such that Solution: The given series is
a4 1 æ 1ö æ 1ö æ 1ö
= and a2 + a5 = 216 çè 1 - ÷ø + çè 1 - ÷ø + çè 1 - ÷ø +
a6 4 2 4 16
If the common ratio is positive, then the first term is Therefore the sum of the first n terms is
(A) 12 (B) 11 (C) 103/7 (D) 13
1æ 1 1 1 ö
(1 + 1 + + n times) - ç 1 + + 2 + + n-1 ÷
Solution: Let a1 = a, a2 = ar, a3 = ar2, a4 = ar3, a5 = ar4 and 2è 2 2 2 ø
a6 = ar5. Then
1 æ 1 - (1/ 2n ) ö
=n- ç = 2- n + n - 1
2 è 1 - (1/ 2) ÷ø
3
1 a4 ar 1
= = =
4 a6 ar 5 r 2
Answer: (C)
and hence r = 2 (since r > 0). Therefore
36. Let a and b be the roots of the quadratic equation
a2 + a5 = 216 Þ a(r + r 4 ) = 216 Þ a = 12
ax2 + bx + c = 0 and Δ = b2 − 4ac. If a + b, a 2 + b 2 and
Answer: (A) a 3 + b 3 are in GP, then
(A) Δ ¹ 0 (B) bΔ = 0 (C) cΔ = 0 (D) Δ = 0
34. a, b, c, d are in GP and are in ascending order such that
a + d = 112 and b + c = 48. If the GP is continued with Solution: Since a and b be the roots of the given
a as the first term, then the sum of the first six terms is quadratic equation, we have
(A) 1156 (B) 1256 (C) 1356 (D) 1456 -b c
a+b= and ab =
Solution: Let r be the common ratio so that b = ar, c = ar , 2 a a
and d = ar3. Therefore Now
a + ar 3 = 112 and ar + ar 2 = 48 b2 2c
a 2 + b 2 = (a + b )2 - 2ab = -
Dividing the first equation by the second and canceling a a2 a
we get -b3 3bc
a 3 + b 3 = (a + b )3 - 3ab (a + b ) = + 2
1 + r 3 112 7 a3 a
= =
r + r2 48 3 Suppose that a + b, a 2 + b 2, a 3 + b 3 are in GP. Then

(1 + r )(1 - r + r 2 ) 7 (a 2 + b 2 )2 = (a + b )(a 3 + b 3 )
=
r(1 + r ) 3 æ b2 2c ö
2
b æ b3 3bc ö
çè a2 - = - - + 2 ÷
3(1 - r + r 2 ) = 7r a ÷ø a çè a3 a ø
3r 2 - 10r + 3 = 0 (b2 - 2ac)2 = - b(- b3 + 3abc)
(3r - 1)(r - 3) = 0 4c2 a2 + b4 - 4b2 ac = b4 - 3ab2 c
1 b2 ca - 4c2 a2 = 0
r = 3 or
3
ca(b2 - 4ac) = 0
(i) r = 3 Þ a = 4
(ii) r = 1/3 Þ a = 108 Since a ¹ 0, cΔ = 0.
But, it is given that a < b < c < d. Therefore, the GP is 4, 12, Answer: (C)
36, 108, 324, 972, … . Hence the sum of the first 6 terms is
37. The first term of a GP a1, a2, a3, … is unity. The value
4 + 12 + 36 + 108 + 324 + 972 = 1456 of 4a2 + 5a3 is minimum when the common ratio is
Answer: (D) (A) 1/3 (B) -1/3 (C) 2/5 (D) -2/5
Solution: Let r be the common ratio. Then
35. The sum of the first n terms of the series
a1 = 1, a2 = r, a3 = r 2 , …, an = r n-1
1 3 7 15
+ + + + Now 4a2 + 5a3 = 4r + 5r2, which is minimum when
2 4 8 16
4 2
is equal to r=- =-
2´5 5
(A) 2n - n - 1 (B) 1 - 2-n
(C) 2-n + n - 1 (D) 2n + 1 Answer: (D)
www.jeeneetbooks.in
Worked-Out Problems 235

(Note that, if a > 0, then ax2 + bx + c assumes its minimum arl -1 = 27, ar m-1 = 8 and ar n-1 = 12
value at x = -b/2a.) 3 l -1
æ 3 ö 27 r
çè ÷ø = = = rl - m (5.17)
38. Three numbers are in AP. If 8 is added to the first 2 8 r m-1
number, we get a GP with sum of the terms is equal
-1
to 26. Then the common ratio of the GP when they æ 3ö 8 r m-1
çè ÷ø = = = r m- n (5.18)
are written in the ascending order, is 2 12 r n-1
(A) 3 (B) 1/3 (C) 2 (D) 1/2
From Eqs. (5.17) and (5.18), we have
Solution: Let the three numbers which are in AP be 3
a - d, a, a + d. Then a - d + 8, a, a + d are given to be in r = , l - m = 3 and m - n = - 1
2
GP. Therefore
Therefore
a2 = (a - d + 8)(a + d) = a2 - d2 + 8 (a + d)
l = m + 3 = n + 2 and m = n - 1
-d2 + 8a + 8d = 0 (5.16)
Each value of n determines the values of l and m.
Also (a - d + 8) + a + (a + d) = 26. Therefore
Therefore, there are infinitely many GP’s satisfying the
3a + 8 = 26 given conditions.
and hence a = 6. Answer: (D)
From Eq. (5.16), we get
41. If a < b < c are numbers lying between 2 and 18 such
d2 - 48 - 8d = 0 that
(d + 4)(d - 12) = 0 (i) a + b + c = 25
and hence d = 12 or - 4. (ii) 2, a, b are three consecutive terms of an AP in
that order
(i) If d = 12, the GP a - d + 8, a, a + d is 2, 6, 18
(iii) b, c, 18 are three consecutive terms of a GP in
(ii) If d = -4, the GP a - d + 8, a, a + d is 18, 6, 2 that order
When we write the GP in the ascending order, the then the product abc is equal to
common ratio is 3.
(A) 480 (B) 680 (C) 440 (D) 640
Answer: (A)
Solution: Given 2 < a < b < c < 18,
39. Three distinct numbers a, b, c form a GP in that order
a + b + c = 25 (5.19)
and the numbers a + b, b + c, c + a form an AP in that
order. Then the common ratio of the GP is 2+b
=a (5.20)
(A) 1/2 (B) -1/2 (C) -2 (D) 2 2

Solution: Let b = ar and c = ar2. Then c2 = 18b (5.21)

2(ar + ar 2 ) = (a + ar ) + (ar 2 + a) From Eqs. (5.19) and (5.20), we get

2r + 2r 2 = 2 + r + r 2 3a + c = 27 (5.22)
From Eqs. (5.20) and (5.21), we get
r2 + r - 2 = 0
(r + 2)(r - 1) = 0 c2 = 36(a - 1) (5.23)

r = - 2 or 1 From Eqs. (5.22) and (5.23), we get

Since a, b and c are distinct, r ¹ 1. Therefore r = -2. (27 - 3a)2 = 36(a - 1)


Answer: (C) (9 - a)2 = 4(a - 1)

40. The number of geometric progressions containing


a2 - 22a + 85 = 0
27, 8 and 12 as three of their terms, is (a - 5)(a - 17) = 0
(A) 1 (B) 2 a = 5 or 17
(C) 5 (D) infinite Case 1: Let a = 5. Then b = 2a - 2 = 8 and
Solution: Let a be the first term and r the common c2 = 18b = 18 ´ 8 = 9 ´ 16
ratio of a GP containing 27, 8 and 12 as lth, mth and nth
terms, respectively. Then implies c = 12
www.jeeneetbooks.in
236 Chapter 5 Progressions, Sequences and Series

Case 2: Let a = 17. Then b = 2a - 2 = 32 and From Eqs. (5.24) and (5.29), we get
c2 = 18b = 18 ´ 32 = 9 ´ 64 (2 x + 12)2 = x(3 x + 72)
implies c = 3 ´ 8 = 24 x2 - 24 x + 144 = 0
But 2 < a < b < c < 18. Therefore a = 5, b = 8, c = 12 and ( x - 12)2 = 0
abc = 480.
Therefore x = 12, y = 36, z = 108.
Answer: (A)
Answer: (B)
42. An infinite GP has first term x and sum 5. Then
44. Let a, b, c be in GP. If p is the AM between a and b
(A) x < -10 (B) -10 < x < 0 and q is the AM between b and c, then b is equal to
(C) 0 < x < 10 (D) x > 10 (A) p + q/2 (B) p + q/pq
Solution: Let r be the common ratio. Then | r | < 1. (C) p + q/2pq (D) 2pq/p + q
Therefore
Solution: Given that
x x a+b b+c
= 5 and r = 1 - b2 = ac, p = and q =
1- r 5 2 2
Since Therefore a = 2p - b and c = 2q - b. Hence
x
- 1 < r < 1, - 1 < 1 - < 1 b2 = ac = (2 p - b)(2q - b) = 4 pq - 2b( p + q) + b2
5
This gives
Therefore
2 pq
x b=
-2 < - <0 p+q
5
Answer: (D)
- 10 < - x < 0
0 < x < 10 45. If a1, a2, a3, … is a GP satisfying the relation ak + ak+2 =
3ak+1 for all k ³ 1, then common ratio is
Answer: (C)
(A) ( 3 ± 1)/ 2 (B) ( 5 ± 2)/ 2
43. If x, y, z are in GP, x - 2, y - 6, z - 58 are in AP and (C) (3 ± 5 )/ 2 (D) (4 ± 5 )/ 2
x - 1, y - 3, z - 9 are in GP, then the numbers x, y, z are
Solution: Let a be the first term and r the common
(A) 3, 9, 27 (B) 12, 36, 108
ratio. Then
(C) 9, 36, 144 (D) 27, 81, 243
ark -1 + ark +1 = 3ark
Solution: From the hypothesis, we have
Therefore
y2 = xz (5.24)
r 2 - 3r + 1 = 0
2( y - 6) = ( x - 2) + (z - 58)
3± 5
or 2 y = x + z - 48 (5.25) r=
2
and (y - 3)2 = (x - 1)(z - 9) (5.26) Answer: (C)
Using Eq. (5.24) and (5.26), we get
46. The value of
6y = 9x + z (5.27)
2 3
2n + 1 æ 2 n + 1ö æ 2 n + 1ö
From Eqs. (5.25) and (5.27), we get + 3ç + 5ç +
2n - 1 è 2 n - 1 ÷ø è 2 n - 1 ÷ø
6 x - 2z + 144 = 0
(5.28) upto n terms is
3 x - z = - 72
(A) n(2n + 1) (B) (n + 1)(2n + 1)
2 y = x + z - 48
(C) n(2n + 3) (D) (n + 1)(2n + 3)
= x + (3 x + 72) - 48
(5.29) Solution: Let
= 4 x + 24
2n + 1
x=
y = 2 x + 12 2n - 1
www.jeeneetbooks.in
Worked-Out Problems 237

and sn = x + 3 x2 + 5 x3 + + (2 n - 1) xn Dividing Eq. (5.30) by Eq. (5.31) we get

Then 1 + r 15
= =5
xsn = x + 3 x + + (2 n - 3) x + (2 n - 1) x
2 3 n n+1 1- r 3
2
Therefore r=
3
(1 - x)sn = x + 2 x2 + 2 x3 + + 2 xn - (2 n - 1) xn+1
Using this value of r in Eq. (5.30) we get a = 5. Also 3r = 2.
= 2( x + x2 + x3 + + xn ) - x - (2 n - 1) xn+1 Therefore
æ 1 - xö n-1
5
5
÷ sn = 2(1 + x + x + + x ) - 1 - (2 n - 1) x å [a + (k - 1)3r] = 2 [2a + (k - 1)(3r)] = 45
2 n
çè
x ø k =1

2(1 - x ) n
Hence, the required sum is 45.
= - 1 - (2 n - 1)xn
1- x Answer: (C)
Now,
48. The value of 0.423232323 (= 0.423) is
1 - x 1 - [(2n + 1)/(2 n - 1)] -2
= = (A) 419/423 (B) 419/990
x (2 n + 1)/(2 n - 1) 2n + 1
(C) 423/990 (D) 419/999
Therefore
Solution: We have
æ -2 ö 2[1 - {(2 n + 1)/(2 n - 1)} ] n

çè 2 n + 1÷ø sn = 1 - [(2 n + 1)/(2 n - 1)] - 1


4 23 23
0.423 = + + + + ¥
10 103 105
n
æ 2 n + 1ö 4 23 æ 1 1 ö
- (2n - 1) ç = + ç 1 + 2 + 3 + ÷ø
è 2 n - 1 ÷ø 10 103 è 10 10

é æ 2 n + 1ö n ù 4 23 æ 1 ö
= + 3ç 2 ÷
= - (2 n - 1) ê1 - ç ÷ ú-1 10 10 è 1 - (1/ 10 ) ø
êë è 2 n - 1 ø úû
4 23 æ 102 ö 419
n
= + 3ç =
æ 2 n + 1ö
- (2 n - 1) ç 10 10 è 99 ÷ø 990
è 2 n - 1 ÷ø Answer: (B)
= - (2 n - 1) - 1 = - 2 n
49. If x, y and z are, respectively, the fourth, seventh and
Hence sn = n(2n + 1). 10th terms of a GP, then
Answer: (A) (A) x2 = y2 + z2 (B) y2 = zx
(C) x2 = yz (D) z2 = xy
47. Sum to infinity of a GP is 15 and the sum to infinity
of their squares is 45. If a is the first term and r is the Solution: Let the first term be a and common ratio r.
common ratio, then the sum of the first 5 terms of Then
the AP with first term a and common difference 3r is
x = ar 3, y = ar6, z = ar9
(A) 25 (B) 35 (C) 45 (D) 55
Therefore,
Solution: By the hypothesis, we have
a y2 = a2 r12 = (ar 3 )(ar9 ) = xz
= 15 (5.30) Answer: (B)
1- r
a2
= 45 50. In a certain GP, if the first, second and eighth terms
1 - r2 are x-4, xK and x52, respectively, then the value of
a2 K is
= 45
(1 - r )(1 + r ) (5.31)
(A) 2 (B) 3 (C) 4 (D) 0
a Solution: Let a, ar, ar2, … be the terms of the GP. Then
= 3 [using Eq. (5.30)]
1+ r a = x-4, ar = xK and ar 7 = x52
www.jeeneetbooks.in
238 Chapter 5 Progressions, Sequences and Series

Therefore 53. Let A be the arithmetic mean of x and y. If p and q


are two GM’s between x and y and p3 + q3 = K(pq)A,
x-4 r = xK and x-4 r 7 = x52
then the value of K is
r = xK + 4 and r 7 = x56 (A) 1 (B) 2 (C) 3 (D) 4

x7( K + 4 ) = x56 Solution: It is given that A is the arithmetic mean of


x and y; that is
Equating the powers we get
x+y
A=
7(K + 4) = 56 2
K+4=8 Now x, p, q, y are in GP. Therefore
K=4 p = xr, q = xr 2 and y = xr 3
Answer: (C) where r = p/x. Then
1/ 3
51. Three numbers a, b, c are in GP. If a, b, c - 64 are in æ yö
r=ç ÷
AP and a, b - 8, c - 64 are in GP, then the sum of the è xø
numbers may be Hence
(A) 124 (B) 241 (C) 142 (D) 214 1/ 3
æ yö
Solution: Let b = ar and c = ar . Given that a, ar, ar − 4
2 2 p = xr = x ç ÷ = x2 / 3 × y1/ 3
è xø
are in AP. Therefore
2/3
æ yö
a + (ar - 64) = 2(ar )
2
q = xç ÷ = x1/ 3 × y2 / 3 and pq = xy
è xø
(5.32)
a(r 2 - 2r + 1) = 64 Now
Again a, ar - 8, ar - 64 are in GP. Therefore
2
p3 + q3 = x2 y + xy2
a(ar 2 - 64) = (ar - 8)2 = xy( x + y)
(5.33)
a(16r - 64) = 64 = ( xy)(2 A) = ( pq)(2 A)
From Eqs. (5.32) and (5.33), we get Therefore K = 2.
Answer: (B)
r 2 - 2r + 1 = 16r - 64

r 2 - 18r + 65 = 0 54. Let f :  ®  be a function such that f(1) = 2 and it


satisfies the relation f(x + y) = f(x)f(y) for all natural
(r - 5)(r - 13) = 0 numbers x and y. Then the value of the natural number
a such that
r = 5 or 13
n
If r = 5, then a(80 - 64) = 64 and hence a = 4. In this case å f (a + K ) = 16 (2 n
- 1)
the numbers are 4, 20, 100 and their sum is 124. K =1

Answer: (A) is
(A) 3 (B) 4 (C) 5 (D) 6
52. The product of nine GMs inserted between the
numbers 2/9 and 9/2 is Solution: f(1) = 2
(A) 9 (B) 1 (C) 3 (D) 3 3 f (2) = f (1 + 1) = f (1) f (1) = 22
Solution: The product of n GMs inserted between two f (3) = f (2 + 1) = f (2) f (1) = 23
positive real numbers a and b is ( ab )n. Here a = 2/9,
This implies
b = 9/2 and n = 9. Substituting these values we get the
required product as f(K) = 2K for any natural number K
9
æ 2 9ö Now
ç 9×2÷ = 1 n n
è ø
16(2n - 1) = å f (a + K ) = å f (a) f (K )
Answer: (B) K =1 K =1
www.jeeneetbooks.in
Worked-Out Problems 239

= 2a (2 + 22 + + 2n ) are in AP. Therefore

= 2a + 1 (2n - 1) b=
2ac
a+c
Therefore
Now
2a+1 = 16 = (2)4
1 1 1 1
Equating the powers we get a = 3. + = +
b - a b - c [2ac /(a + c)] - a [2ac /(a + c)] - c
Answer: (A)
a+c a+c
= +
55. In an HP, if the mth term is n and the nth term is m, ac - a2 ac - c2
then (m + n)th term is
é 1 1 ù
(A) m - n/m + n (B) mn/m + n = (a + c ) ê + ú
(C) m + n/mn (D) 2mn/m + n ë a(c - a) c(a - c) û
a+c
Solution: By hypothesis =
ac
1
n = mth term = Answer: (A)
a + (m - 1)d
1 57. If (m + 1)th term, (n + 1)th term and (r + 1)th terms
m = nth term =
a + (n - 1)d of an AP are in GP and m, n and r are in HP, then
the ratio of the common difference to the first term
Therefore of AP is
1 1 m-n (A) -1/n (B) 1/n (C) 2/n (D) -2/n
[a + (m - 1)d] - [a + (n - 1)d] = - =
n m mn
Solution: By hypothesis,
m-n
(m - n)d = 2 mr
mn (a + md)(a + rd) = (a + nd)2 and n =
1 m+r
d=
mn Therefore
Now a2 + ad(m + r ) + mrd2 = a2 + 2and + n2 d2
1 a(m + r ) + mrd = 2an + n2 d
a + (m - 1)d =
n
d(n2 - mr ) = a(m + r - 2 n)
m-1 1
a+ = d (m + r ) - 2 n (m + r ) - 2 n
mn n = = 2
1 m-1 1 a n2 - mr n - [n(m + r )/ 2]
a= - =
n mn mn æ n(m + r ) ö
çè∵ mr = ÷ø
Therefore the (m + n)th term is 2
-1 2[m + r - 2 n] - 2
æ 1 1 ö mn = =
çè + (m + n - 1) ÷ø = n(2 n - m - r ) n
mn mn m+n
Answer: (D)
Answer: (B)
58. Three numbers l, m and n are in GP. The lth, mth
56. If a, b, c are in HP (in this order), then and nth terms of an AP are in HP. Then the ratio of
1 1 c+a 1 1 c+a the first term of the AP to its common difference is
(A) + = (B) + =
b-a b-c ca a+b b+c ca (A) m : 1 (B) 1 : m
1 1 1 1 1 1 1 1 (C) 1 : m + 1 (D) m + 1 : 1
(C) + = + (D) + = +
c-a b-a a c a b b c Solution: Since l, m and n are in GP, we have m2 = ln.
Solution: We have Let the AP be a, a + d, a + 2d, … . Therefore
1 1 1 a + (l - 1)d, a + (m - 1)d and a + (n - 1)d
, ,
a b c are in HP. Hence
www.jeeneetbooks.in
240 Chapter 5 Progressions, Sequences and Series

2[a + (l - 1)d][a + (n - 1)d] 60. Two AMs A1 and A2, two GMs G1 and G2 and two
a + (m - 1)d = HMs H1 and H2 are inserted between two given
[a + (l - 1)d] + [a + (n - 1)d]
non-zero real numbers x and y. Then
2[a + (l - 1)d][a + (n - 1)d]
1 1
= [a + (m - 1)d][2a + d(l + m - 2)] + =
H1 H2
2ad(l + n - 2) + 2(l - 1)(n - 1)d2 1 1 1 1
(A) + (B) +
= ad(l + n - 2 + 2 m - 2) + (m - 1)(l + n - 2)d 2
A1 A2 G1 G2
2a(l + n - 2) + 2(l - 1)(n - 1)d G1G2 A + A2
(C) (D) 1
A1 + A2 G1 G2
= a(l + n + 2 m - 4) + (m - 1)(l + n - 2)d2
Solution: Given that
a[2l + 2 n - 4 - l - n - 2 m + 4]
= d[(m - 1)(l + n - 2) - 2(l - 1)(n - 1)] y - x 2x + y x + 2y
A1 = x + = and A2 =
3 3 3
a(l + n - 2 m) 1/ 3
æ yö
= d(m + 1)(l + n - 2 m) (∵ m = ln) 2
G1 = x ç ÷ = x2 / 3 y1/ 3 and G2 = x1/ 3 y2 / 3
è xø
a
=m+1 3 xy 3 xy 3 xy
d H1 = = and H2 =
3 y + ( x - y) x + 2 y 2x + y
Answer: (D)
Therefore
59. If a1, a2, a3, …, an are in HP and
1 1 ( x + 2 y) + (2 x + y)
+ =
æ n ö
f (K ) = ç å ar ÷ - aK H1 H2 3 xy
è r =1 ø
3( x + y) x + y A1 + A2
= = =
then 3 xy xy G1G2
a1 a a Answer: (D)
, 2 , …, n
f (1) f (2) f (n)
61. Let a1, a2, …, a10 be in AP and h1, h2, …, h10 be in HP. If
are in a1 = h1 = 2 and a10 = h10 = 3, then a4h7 =
(A) AP (B) GP (C) HP (D) AGP (A) 2 (B) 3 (C) 5 (D) 6
Solution: It is given that Solution: Given a1, a2, …, a9 are 8 AMs between 2 and 3
n
and h1, h2, …, h9 be 8 HMs between 2 and 3. Therefore
f (K ) + aK = å ar
r =1
æ 3 - 2ö 7
a4 = 2 + 3 ç =
1 1 1 è 9 ÷ø 3
and , , …, are in AP
a1 a2 an 6(9) 54
and h7 = =
Therefore 3(9) + 6(2 - 3) 21
n n n

åa åa r r åa r
Therefore
r =1
, r =1
, …, r =1
are in AP 7 54
a1 a2 an a4 h7 = × = 6
3 21
a1 + f (1) a2 + f (2) a + f (n) Answer: (D)
, , …, n are in AP
a1 a2 an
62. If a, b, c are distinct real numbers and are in AP and
f (1) f (2) f (n) a2, b2, c2 are in HP, then
, , …, are in AP
a1 a2 an (A) a, b, c/2 are in GP (B) a, b, -c/2 are in GP
Finally, (C) a/2, b, c are in GP (D) a, b/2, c/2 are in GP

a1 a a Solution: By hypothesis,
, 2 , …, n are in HP
f (1) f (2) f (n) 1 1 1 1
2b = a + c and - = -
Answer: (C) b2 a2 c2 b2
www.jeeneetbooks.in
Worked-Out Problems 241

Therefore Given that


H 4
(a - b)(a + b) (b - c)(b + c) =
= G 5
a2 b2 b2 c2
Therefore
a+b b+c
= 2 (since b - a = c - b)
a2 c 2ab 4
=
c a+c b=a b+a c
2 2 2 2 ab (a + b) 5

ac(c - a) + b(c2 - a2 ) = 0 10 ab = 4(a + b)


ac + b(c + a) = 0 (since c ¹ a) a æa ö
5 = 2 ç + 1÷
ac + 2b = 0 (since c + a = 2b)
2 b èb ø
Let x = a / b . Substituting this we get
Hence a, b, -c/2 are in GP.
Answer: (B) 5 x = 2( x2 + 1)
( x - 2)(2 x - 1) = 0
63. If the roots of the equation 10x3 - Kx2 - 54x - 27 = 0
1
are in HP, then K is equal to x = 2 or
2
(A) 3 (B) 6 (C) 9 (D) 12
(i) When x = 2, a/b = 4.
Solution: Let a, b, g be the roots of the given equation.
Then (ii) When x = 1/2, a/b = 1/4.
Answer: (A)
a + b + g = K, ab + bg + g a = - 54, abg = 27
65. If a, b, c are in GP and a - b, c - a, b - c are in HP,
Now a, b, g are in HP, and hence 1/a, 1/b, 1/g are in
then the value of a + 4b + c is equal to
AP. This gives
(A) 1 (B) 0 (C) 2abc (D) b2 + ac
- 54 ab + bg + ga 1 1 1 3
= = + + = Solution: It is given that b = ar and c = ar2. Also
27 abg a b g b
2 1 1
-3 = +
b= c-a a-b b-c
2
2 1 1
Since b is a root of the given equation, substituting the = +
ar 2 - a a - ar ar - ar 2
value of b in it we get
2 1 1 r+1
3 2 = + =
æ -3ö æ -3ö æ -3ö r 2 - 1 1 - r r(1 - r ) r(11 - r )
10 ç ÷ - K ç ÷ - 54 ç ÷ - 27 = 0
è 2ø è 2ø è 2ø
-2 r+1
=
- 270 9 K r+1 r
- + 81 - 27 = 0
8 4
- 2r = (r + 1)2
9 K 162
= r 2 + 4r + 1 = 0
4 8
K=9 ar 2 + 4ar + a = 0
Answer: (C) c + 4b + a = 0
Answer: (B)
64. Let G and H be, respectively, the GM and HM
between two numbers. If H : G = 4 : 5, then the ratio 66. If a, b, c are in AP, b, c, d are in GP and c, d, e are in
of the numbers can be HP, then a, c, e are in
(A) 1 : 4 (B) 5 : 9 (C) 9 : 2 (D) 3 : 4 (A) AP (B) GP (C) HP (D) AGP
Solution: Let a and b be the numbers. Then Solution: It is given that
2ab a+c 2 2ce
G = ab and H = b= , c = bd and d =
a+b 2 c+e
www.jeeneetbooks.in
242 Chapter 5 Progressions, Sequences and Series

Therefore
å
40
Then K =1
f (K ) =
a + c 2ce
c2 = bd = × (A) 365 (B) 366 (C) 364 (D) 363
2 c+e
Solution: Let x = 2 n + 1 and y = 2 n - 1 . Then
c2 (c + e) = ce(a + c)
x2 + y2 = 4 n
c2 = ea
x2 - y2 = 2
This implies a, c, e are in GP.
Answer: (B) xy = 4 n2 - 1
Therefore
67. If ax = by = cz and a, b, c are in GP, then x, y, z, are in
(A) AP (B) GP (C) HP (D) AGP x2 + y2 + xy x3 - y3 1
f (n) = = 2 = [(2 n + 1)3/ 2 - (2 n - 1)3/ 2 ]
x+y x - y2 2
Solution: From the hypothesis we have
Substituting n = 1 to 40 we get
a = by/ x , c = by/ z and b2 = ac
Therefore 1
f (1) = (33/ 2 - 13/ 2 )
2
b2 = by/ x , by/ z = b( y/ x )+( y/ z)
1
y y f (2) = (53/ 2 - 33/ 2 )
2= + 2
x z
1
2 xz = y( x + z) f (3) = (73/ 2 - 53/ 2 )
2
2 xz 
y=
x+z
1
Therefore x, y, z are in HP. f (40) = (813/ 2 - 793/ 2 )
2
Answer: (C)
Therefore
68. Sum to n terms of the series 1 + 5 + 19 + 65 + 211 + 40
1
is equal to å f (n) = 2 (81
n=1
3/ 2
- 13/ 2 ) = 364
(A) 1/2(3n+1 - 2n+2 + 1) (B) 1/2(3n + 2n - 1)
Answer: (C)
(C) 3n+1 - 2n+1 - 1 (D) 3n + 2n + 1
Solution: We have 70. Let

1 = 3 - 2, 5 = 32 - 22, 19 = 33 - 23, 1 1 1 1 1 1
x = 1+ 2
+ 2 + 1+ 2 + 2 + 1+ 2 + 2
65 = 3 - 2 , 211 = 3 - 2 ,…
4 4 5 5 1 2 2 3 3 4

Therefore sum to n terms is 1 1


+ + 1+ +
n 20092 20102
å (3
K =1
K
- 2K ) = (3 + 32 + + 3n ) - (2 + 22 + + 2n )
Then
3(3n - 1) 2(2n - 1) (2010) x - 2009
= - =
3-1 2-1 2010
3 (A) 2010 (B) 2009 (C) 1999 (D) 2000
= (3n - 1) - 2(2n - 1)
2 Solution: Let
1
= (3n+1 - 2n+ 2 + 1) 1 1 n2 (n + 1)2 + (n + 1)2 + n2
2 Tn = 1 + + =
n2
(n + 1)2 n2 (n + 1)2
Answer: (A)
Then
69. For any positive integer n, let
n2 + n + 1 1 1 1
Tn = =1+ =1+ -
4n + 4 n2 - 1 n(n + 1) n(n + 1) n n+1
f (n) =
2n + 1 + 2n - 1
www.jeeneetbooks.in
Worked-Out Problems 243

Therefore, which is an AP. Then, using Sec. 5.5, IV(i), we get


2009 2009
æ 1 1 ö 2(n - 1) 2(n - 1)(n - 2)
x = å Tn = å ç 1 + - un = 1 + +
n=1 n=1 è n n + 1÷ø 1! 2!
= 1 + 2 n - 2 + n2 - 3n + 2
æ 1 ö
= 2009 + ç 1 -
è 2009 + 1÷ø = n2 - n + 1
The sum of the numbers in the nth group is
(2010)2 - 1
=
2010 (n2 - n + 1) + (n2 - n + 3) + (n2 - n + 5) + +
2011× 2009 (n2 - n + 2 n - 1)
=
2010 = n(n2 - n) + [1 + 3 + 5 + + (2 n - 1)]
Therefore
= n3 - n2 + n2 = n3
2010 x - 2009 Answer: (B)
= 2009
2010
73. Sum to n terms of the series 1, 2, 3, 6, 17, 54, 171, … is
Answer: (B)
(A) 1/8(3n - 1) - n/12(2n2 - 9n - 2)
71. Sum to infinity of the series (B) 1/8(3n + 1) + n/12(2n2 - 9n + 2)
1 1 1 (C) 1/8(3n - 1) + n/12(2n2 - 9n + 2)
+ 2 + 2 +
2 -1 4 -1 6 -1
2
(D) 1/8(3n + 1) - n/12(2n2 + 9n - 2)
is Solution: Given series is 1, 2, 3, 6, 17, 54, 171, …
(A) 1/3 (B) 1 (C) 2/3 (D) 1/2 D un = 1, 1, 3, 11, 37, 117, …
Solution: The nth term D2un = 0, 2, 8, 26, 80, …
1 1æ 1 1 ö D3un = 2, 6, 18, 54, … ,
un = = ç -
4 n - 1 2 è 2 n - 1 2 n + 1÷ø
2
which is a GP with common ratio 3. By Sec. 5.5, IV(ii),
Therefore we get

1æ 1 ö un = a 3n-1 + a0 + a1 (n - 1) + a2 (n - 1)(n - 2)
sn = u1 + u2 + + un = ç 1 -
2è 2 n + 1÷ø Therefore
Now sum to infinity is given by 1 = u1 = a + a0
1 2 = u2 = 3a + a0 + a1
s¥ = lim sn =
n®¥ 2 3 = u3 = 9a + a0 + 2a1 + 2a2
Answer: (D)
6 = u4 = 27a + a0 + 3a1 + 6a2
72. Odd natural numbers are arranged in groups as (1), Solving the above set of equations for a, a0, a1, a2, we get that
(3, 5), (7, 9, 11), (13, 15, 17, 19), (21, 23, 25, 27, 29), …. 1 3 1 1
Then the sum of the natural numbers in the nth a = , a0 = , a1 = , a2 = -
4 4 2 2
group is
(A) n3 + n (B) n3 (C) (n + 1)3 (D) n3 - n Hence

Solution: The nth group consists of n natural odd 3n - 1 3 n - 1 (n - 1)(n - 2)


un = + + -
numbers. Let un be the first number in the nth group. We 4 4 2 2
thus have 1
sn = (1 + 3 + 32 + + 3n - 1 )
u1 = 1, u2 = 3, u3 = 7, u4 = 13, u5 = 21, u6 = 31,… 4
3n n(n - 1) 1 (n - 2)(n - 1)n
Now + + -
4 4 2 3
Dun = 3 - 1, 7 - 3, 13 - 7, 21 - 13, 31 - 21
1 n n
= 2, 4, 6, 8, 10 = (3 - 1) - (2 n2 - 9 n - 2)
8 12
D2un = 2, 2, 2,… Answer: (A)
www.jeeneetbooks.in
244 Chapter 5 Progressions, Sequences and Series

74. Sum to n terms of the series Therefore the sum of n terms is


14 24 34 sn = u1 + u2 + + un
+ + +
1× 3 3× 5 5× 7 1 n 2 n 1 æ 1 ö
= å
4 K =1
K + + ç
16 32 è
1-
2 n + 1÷ø
is
(A) n(n + 1)(2n2 + 1)/8(2n + 1) n(n + 1)(2 n + 1) n n
= + +
(B) n(n + 1)(n + n + 1)/6(2n + 1)
2
4×6 16 16(2 n + 1)
(C) (n + 1)[(2n + 1)2 + 1]/8(2n + 1) n(n + 1)(n2 + n + 1)
(D) n(n + 1)[(2n + 1)2 + 1]/16(2n + 1) =
6(2 n + 1)
Solution: The Kth term of the given series is Answer: (B)
K 4
K 12
1 æ 1 1 ö
uK = = + + -
(2 K - 1)(2 K + 1) 4 ç
16 32 è 2 K - 1 2 K + 1÷ø

Multiple Correct Choice Type Questions


1. Consider the AP When n = 31, Tn = 0. Therefore
1 2 31
20, 19 , 18 , 18, … s31 = (20 + 0) = 310
3 3 2
Answers: (A), (B), (C), (D)
Then
(A) sum of the first 25 terms is 300 2. The sum of the first 8 and 19 terms of an AP is 64 and
(B) sum of the first 36 terms is 300 361, respectively. Then
(C) sum of the terms from 26th term to 36th term is zero (A) common difference is 2
(D) the sum of all the non-negative terms is 310 (B) first term is 1
(C) sum of the first n terms is n2
Solution: The given series is an AP with first term
(D) nth term is 2n
a = 20 and the common difference d = - 2 / 3. Let
n Solution: The sum of first 8 terms is 64; therefore,
sn = [2a + (n - 1)d]
2 2a + 7d = 16

be the sum of the first n terms. The sum of the first The sum of first 19 terms is 361; therefore,
25 terms is given by 19a + 171d = 361
25 é æ 2ö ù Solving these two equations in two variables, we get
s25 = 40 + 24 ç - ÷ ú
2 êë è 3ø û d = 2, a = 1.
Answers: (A), (B), (C)
é æ 2ö ù
= 25 ê 20 + 12 ç - ÷ ú = 25 ´ 12 = 300 3. Let an + bn be the sum of the first n terms of an AP. Then
2
ë è 3ø û
(A) first term is a + b
Again sum of first 36 terms is (B) first term is a - b
36 é æ 2ö ù æ 50 ö (C) common difference is 2a
s36 = ê 40 + 35 ç - ÷ ú = 18 ç ÷ = 300 (D) common differences is b - a
2 ë è 3ø û è 3ø
Solution: By hypothesis
Now s36 = s25 implies that the sum of the terms from 26th
to 36th is zero. sn = an2 + bn
If Tn is the last non-negative term, then The nth term is given by
æ 2ö sn - sn -1 = a[n2 - (n - 1)2 ] + b[n - (n - 1)] = (2 n - 1) a + b
Tn = 20 + (n - 1) ç - ÷ ³ 0
è 3ø
Þ 60 - 2 n + 2 ³ 0 Therefore, the series is a + b, 3a + b, 5a + b, … . In this case
the first term is a + b and the common difference is 2a.
Þ n £ 31
Answers: (A), (C)
www.jeeneetbooks.in
Worked-Out Problems 245

4. The numbers a, b, c and A, B, C are in AP. The Substituting these values in Eq. (5.36), we get n = 59.
common difference of the second set is one more than Answers: (A), (C)
the common difference of the first. If
2 2 2
a + b + c = A + B + C = 15 6. If a , b , c are in AP, then
(A) 1/(b + c), 1/(c + a), 1/(a + b) are in AP
abc 7
and = (B) a /(b + c), b /(c + a), c /(a + b) are in AP
ABC 8
(C) (b + c - a)/ a, (c + a - b)/ b, (a + b - c)/ c are not in AP
then (D) 1/ a, 1/ b, 1/ c are in AP
(A) a = 7, A = 6 (B) B = b = 5 Solution: By hypothesis b2 - a2 = c2 - b2 . Therefore
(C) a = 5, A = 6 (D) a = 6, A = 5
(b - a)(b + a) = (c - b)(c + b) (5.37)
(It is given that the two sets of numbers are in the
descending order.) (A) We have

Solution: Let a = a - d, b = a , c = a + d. By hypothesis 1 1 b-a


- =
c + a b + c (c + a)(c + b)
A = b - (d + 1), B = b , C = b + (d + 1)
which gives b2 - a2
= (5.38)
(a + b)(b + c)(c + a)
3a = 15 = 3b Þ a = 5 = b
Now 1 1 c-b
- =
a + b c + a (a + b)(c + a)
7 abc a (a 2 - d2 )
= =
8 ABC b[b 2 - (d + 1)2 ] c2 - b2
= (5.39)
(a + b)(b + c)(c + a)
7[25 - (d + 1) ] = 8(25 - d )
2 2

Equations (5.38), (5.39) and b2 - a2 = c2 - b2 give


8d2 - (d + 1)2 7 = 25
1 1 1 1
d2 - 14d - 32 = 0 - = -
c+a b+c a+b c+a
d = - 2, 16 Therefore
Therefore if d = -2, then
1 1 1
, ,
a = a - d = 7, A = 5 - (- 2 + 1) = 6 and b = 5 = B b+c c+a a+b
Answers: (A), (B)
are in AP. Therefore (A) is true.
5. The second, 31st and last terms of an AP are, (B) We have
respectively, 31/4, 1/2 and -13/2. Then a b c
(A) first term is 8 (B) number of terms is 58 , , are in AP
b+c c+a a+b
(C) number of terms is 59 (D) first term is 6
a b c
Solution: Let a be the first term and d the common Û + 1, + 1, + 1 are in AP
b+c c+a a+b
difference. Also let the nth term be the last term. Then
31 a+b+c a+b+c a+b+c
a+d= (5.34) Û , , are in AP
4 b+c c+a a+b
1 1 1 1
a + 30d = (5.35) Û , , are in AP and this is true
2 b+c c+a a+b
13 Therefore
a + (n - 1)d = - (5.36)
2
a b c
From Eqs. (5.34) and (5.35) we get , ,
b+c c+a a+b
1
d=- , a=8 are in AP. Therefore (B) is true.
4
www.jeeneetbooks.in
246 Chapter 5 Progressions, Sequences and Series

(C) We have 8. The sum of the first two terms of an infinite GP is


equal to 5 and every term is three times the sum of all
b+c-a c+a-b a+b-c the terms that follow. If a and r are the first term and
, and
a b c common ratio, respectively, then
(A) a = 4 (B) a = 3
are in AP. This implies
(C) r = 1 / 4 (D) r = 2 / 3
b+c c+a a+b Solution: By hypothesis
, ,
a b c
æ ar ö
a + ar = 5 and a = 3 ç
are in AP and this is not true according to B. è 1 - r ÷ø
Therefore (C) is not true.
(D) We have Therefore 1 - r = 3r Þ r = 1/ 4 . This value of r gives a = 4.
Answers: (A), (C)
1 1 1 1 1 1 1
, , are in AP Û - = -
a b c b a c b 9. If x, y, z are in GP and x + y, y + z, z + x are in AP in
that order, then
a-b b-c
Û = (A) common ratio of the GP is 2
ab bc
(B) x = y = z
a-b a (C) common ratio of the GP is -2
Û =
b-c c (D) common ratio is 1/ 2
This is not true because from Eq. (5.37). Then Solution: Let y = xr and z = xr2. Then

a-b b+c x + y = x(1 + r ), y + z = xr (1 + r ), z + x = x(r2 + 1)


=
b-c b+a These are in AP. Therefore

Therefore (D) is not true. 2 xr(1 + r ) = x(1 + r ) + x(r2 + 1) = x(r2 + r + 2)


Answers: (A), (B) 2r(1 + r ) = 2 + r + r2

7. Consider an infinite geometric series with first term r2 + r - 2 = 0


a and common ratio r. If its sum is 4 and the second
(r + 2)(r - 1) = 0
term is 3/4, then
(A) a = 4 / 7, r = 3 / 7 (B) a = 1, r = 3 / 4 r = 1 or - 2
(C) a = 3 / 2, r = 1/ 2 (D) a = 3, r = 1/ 4 Answers: (B), (C)

Solution: By hypothesis 10. The first two terms of an infinitely decreasing GP are
3 a 3 and 2 /( 3 + 1). Then the
ar = and =4 (A) common ratio is ( 3 - 1)/ 3
4 1- r
(B) sum to infinity of the GP is 3 3
Dividing the first equation by the second we get
(C) common ratio is 1/ 3
3 1 3 (D) sum to infinity is 3
r(1 - r ) = × =
4 4 16
Solution: The common ratio is
16r2 - 16r + 3 = 0
2 3-1
(4r - 1)(4r - 3) = 0 =
( 3 + 1) 3 3
1 3
r= or The sum to infinity is
4 4

(i) r = 1/ 4 Þ a = 3 3
=
3
=3
(ii) r = 3 / 4 Þ a = 1 1 - [( 3 - 1)/ 3 ] 1/ 3
Answers: (B), (D) Answers: (A), (D)
www.jeeneetbooks.in
Worked-Out Problems 247

11. If d ¹ 0 and the sequence a(a + d), (a + d)(a + 2d), (i) When r = -1/2, a = 8
(a + 2d) a forms a GP, then (ii) When r = 3/2, a = 24/19
(A) common ratio of the GP is -2 Answers: (A), (C)
(B) 3a = - 2d
(C) a = -2d 13. The ratio of the sum of the cubes of an infinitely
decreasing GP to the sum of its squares is 12:13. The
(D) common ratio is 2
sum of the first and second terms is equal to 4/3. If
Solution: It is given that a, r and s¥ denote the first term, common ratio and
sum to infinity of the GP, then
(a + d)(a + 2d) (a + 2d)a
= (A) r = 1/ 3, a = 6 / 5, s¥ = 9 / 5
a(a + d) (a + d)(a + 2d)
(B) r = 1/ 3, a = 1
a + 2d a (C) r = - 4 / 3, a = - 1/ 3, s¥ = - 1/ 7
=
a a+d
(D) s¥ = 3 / 2
a2 + 3ad + 2d2 = a2 Solution: By hypothesis, | r | < 1 and
d(3a + 2d) = 0 a3 a2
: = 12 : 13
Therefore 3a = - 2d (since d ¹ 0). Hence, the common ratio 1 - r3 1 - r2
is
a(1 - r2 ) 12
a + 2d a - 3a =
= = -2 1 - r3 13
a a
Therefore
Answers: (A), (B)
13a(1 + r ) = 12(1 + r + r2 ) (5.42)
Note : If x, y, z are in AP with a non-zero common differ-
ence and xy, yz, zx are in GP, then common ratio of the Also, given that
GP is -2 and also 3 x = - 2(z - y).
4
a + ar = (5.43)
12. The sum of the first three terms of a GP is 6 and the 3
sum of its first three odd terms is 10.5. Then the first
From Eqs. (5.42) and (5.43), we get
term and the common ratio are
(A) 8, - 1/ 2 (B) 8, 1/ 2 4
13 ´ = 12(1 + r + r2 )
(C) 24 / 19, 3 / 2 (D) 24 / 29, 3 / 2 3

Solution: Let a, ar, ar , … be the GP. It is given that


2 13 = 9(1 + r + r2 )

a(1 + r + r2 ) = 6 (5.40) 9 r2 + 9 r - 4 = 0

21 (3r - 1)(3r + 4) = 0
a(1 + r2 + r4 ) = (5.41)
2 The values of r thus obtained are
Dividing Eq. (5.41) by Eq. (5.40) we get 1 -4
r= or
1+ r + r
2 4
21 7 3 3
= = (i) When r = 1/3, then a = 1 and
1+ r + r 2
12 4
4 + 4 r2 + 4 r4 = 7 + 7 r + 7 r2 1 3
s¥ = =
1 - (1/ 3) 2
4r4 - 3r2 - 7r - 3 = 0
(ii) When r = -4/3, then a = 4.
Now r = -1/ 2 is a solution. We have
Therefore (C) is not possible.
(2r + 1)(2r3 - r2 - r - 3) = 0
Answers: (B), (D)
(2r + 1)(2r - 3)(r2 + r + 1) = 0
Therefore 14. Let

-1 3 æ1 1 1 ö
r= , x = log 5 ç + + + ¥÷
2 2 è 4 8 16 ø
www.jeeneetbooks.in
248 Chapter 5 Progressions, Sequences and Series

Then Solution: Let b = ar, c = ar2 and d = ar3.


(A) x = -2 log5 2 (B) 5 = 2 x

(A) (a2 + b2 + c2 )(b2 + c2 + d2 )


(C) (0.2)x = 4 (D) (0.2)x = 2
= a2 (1 + r2 + r4 )a2 (r2 + r4 + r6 )
Solution: We have
= a4 r2 (1 + r2 + r4 )2
1 1 1 1/ 4 1
+ + + = = = (a2 r + a2 r3 + a2 r5 )2
4 8 16 1 - (1/ 2) 2
= (ab + bc + cd)2
Therefore
Therefore (A) is true.
æ 1ö
x = log 5 ç ÷ = - 2 log5 2 (B) (b - c) + (c - a) + (d - b)
2 2 2
è 2ø
-2 log5 2 = (ar - ar2 )2 + (ar2 - a)2 + (ar3 - ar )2
æ 1ö
(0.2) = ç ÷
x
=5 log5 4
=4
è 5ø = a2 [(r4 - 2rr3 + r2 ) + (r4 - 2r2 + 1)
1 + (r6 - 2r4 + r2 )]
5x = 5-2 log5 2 = 5log5 (1 / 4 ) =
4 = a2 (r6 - 2r3 + 1)
Answers: (A), (C)
= a2 (r3 - 1)2
15. If s1, s2 and s3 are, respectively, equal to the sums of = (d - a)2
the first n, 2 n and 3n terms of a GP, then
Therefore (B) is true.
(A) s1 ( s3 - s2 ) = ( s2 - s1 )2 (B) s12 + s22 = s1 ( s2 + s3 )
(C) s1 ( s2 + s3 ) = ( s1 + s2 )2
(D) s22 = s1 s3 (C) a2 - b2 = a2 (1 - r2 ), b2 - c2 = a2 (r2 - r4 ) = a2 r2 (1 - r2 )

Solution: Let the GP be a, ar, ar2, … . Given that and c2 - d2 = a2 (r4 - r6 ) = a2 r4 (1 - r2 )


So,
a(1 - rn ) a(1 - r2 n ) a(1 - r3 n )
s1 = ; s2 = ; s3 = (a2 - b2 )(c2 - d2 ) = a4 r4 (1 - r2 )2 = (b2 - c2 )2
1- r 1- r 1- r
Therefore (C) is true.
Then
(D) a2 + b2 = a2 (1 + r2 ), b2 + c2 = a2 r2 (1 + r2 )
a r2 n a(1 - rn ) 2 n
s3 - s2 = (r 2 n - r 3 n ) = = r s1
1- r 1- r and c2 + d2 = a2 r4 (1 + r2 )
2
é a(1 - rn ) n ù So,
s1 ( s3 - s2 ) = ê r ú
ë 1-r û (a2 + b2 )(c2 + d2 ) = a4 r4 (1 + r2 )2 = (b2 + c2 )2
arn (1 - rn ) Therefore (D) is true.
s2 - s1 =
1- r Answers: (A), (B), (C), (D)
( s2 - s1 ) = s1 ( s3 - s2 )
2
17. Let a, b, c be three distinct real numbers in GP. If x is
real and a + b + c = xb , then
Also,
(A) x < -1 (B) 0 < x < 1 (C) 2 < x < 3 (D) x > 3
( s2 - s1 )2 + 2 s1 s2 = s1 ( s3 - s2 + 2 s2 )
Solution: Let b = ar and c = ar2, and r ¹ 1. Then
and hence
s12 + s22 = s1 ( s3 + s2 ) a(1 + r + r2 ) = xar

Answers: (A), (B) Therefore


r2 + (1 - x)r + 1 = 0
16. If a, b, c and d are in GP, then (1 - x)2 - 4 ³ 0
(A) (a + b + c )(b + c + d ) = (ab + bc + cd)
2 2 2 2 2 2 2

x2 - 2 x - 3 ³ 0
(B) (a - d)2 = (b - c)2 + (c - a)2 + (d - b)2
(C) a2 - b2 , b2 - c2 , c2 - d2 are in GP ( x + 1)( x - 3) ³ 0
(D) a2 + b2 , b2 + c2 , c2 + d2 are in GP x £ - 1 or x³3
www.jeeneetbooks.in
Worked-Out Problems 249

(i) When x = -1, then r2 + 2r + 1 = 0 and hence r = -1, Therefore sum of n terms is
so that a = c , a contradiction to the hypothesis.
æ 1 ö 6n
(ii) When x = 3, then r2 - 2r + 1 = 0 and hence r = 1, so sn = u1 + u2 + + un = 6 ç 1 - =
that a = b = c. è n + 1÷ø n + 1
Therefore x < -1 or x > 3. and sum to infinity is
Answers: (A), (D)
æ 6 ö
s¥ = lim sn = lim ç =6
n ®¥ n ®¥ è 1 + (1/ n) ÷
ø
18. Let a and b be positive real numbers. If a, A1, A2 , b
are in AP, a, G1, G2 , b are in GP and a, H1, H2 , b are Answers: (B), (D)
in HP, then
G1G2 20. Let a1, a2 , a3, a4 , … be in GP. If the HM of a1 and a2 is
= 12 and that of a2 and a3 is 36, then
H1 H2
(A) a1 = 8 (B) a2 = 24
(A) (2a + b)(a + 2b)/ 9ab (B) ( H1 H2 )/( A1 A2 ) (C) a3 = 72 (D) a4 = 216
(C) ( A1 + A2 )/( H1 + H2 ) (D) ( H1 + H2 )/( A1 + A2 ) Solution: Let a2 = a1r, a3 = a1r2 , a4 = a1r3, … . Then
Solution: It is given that 2a1a2 2a12 r 2ra1
12 = = = (5.44)
a1 + a2 a1 (1 + r ) 1 + r
b - a 2a + b 2(b - a) a + 2b
A1 = a + = , A2 = a + =
3 3 3 3
2a2 a3 2a12 r3 2r2 a1
36 = = = (5.45)
æ bö
1/ 3
æ bö
2/3
a2 + a3 a1 (r + r2 ) 1 + r
G1 = a ç ÷ = a × b , G2 = a ç ÷
2/3 1/ 3
= a ×b
1/ 3 2/3
è aø è aø
From Eqs. (5.44) and (5.45), we get
3ab 3ab 3ab 3ab
H1 = = , H2 = = 12 36
3b + 1(a - b) a + 2b 3b + 2(a - b) 2a + b = 2
2ra1 2r a1

Therefore and hence r = 3.


From Eq. (5.44),
G1G2 ab(a + 2b)(2a + b) (a + 2b)(2a + b)
= = 6a1 3
H1 H2 9a2 b2 9ab 12 = = a1
1+ 3 2
-1
A1 + A2 æ 2a + b a + 2b ö æ 3ab 3ab ö and hence a1 = 8 . Therefore
=ç + ÷ø ç +
H1 + H2 è 3 3 è a + 2b 2a + b ÷ø
a2 = a1r = 24, a3 = a1r2 = 72, a4 = a1r3 = 216
(a + 2b)(2a + b)
= Answers: (A), (B), (C), (D)
9ab
Answers: (A), (C) 21. Let A1, A2 ; G1, G2 and H1, H2 be two AMs, GMs and
HMs, respectively, between two positive real numbers
19. Let sn be the sum to n terms of the series a and b. Then
(A) A1 H2 = ab (B) G1G2 = ab
3 5 7 9
+ + + + (C) A1 H2 = a2 b2 (D) A2 H1 = ab
12 12 + 22 12 + 22 + 32 12 + 22 + 32 + 42
Solution: By Problem 18 (solution)
Then
(A) sn = n / n + 1 (B) sn = 6 n / n + 1 2a + b a + 2b
A1 = , A2 =
(C) s¥ = 1 (D) s¥ = 6 3 3

Solution: Let G1 = a2 / 3 b1 / 3 , G2 = a1 / 3 b2 / 3
2K + 1 6 æ 1 1 ö 3ab 3ab
uK = = = 6ç - H1 = , H2 =
12 + 22 + + K 2 K (K + 1) è K K + 1÷ø a + 2b 2a + b
www.jeeneetbooks.in
250 Chapter 5 Progressions, Sequences and Series

Therefore 24. If a, b, c are in HP, then

2a + b 3ab (A) a/(b + c), b/(c + a), c/(a + b) are in AP


A1 H2 = × = ab (B) a/(b + c), b/(c + a), c/(a + b) are in HP
3 2a + b
(C) a/(b + c - a), b/(c + a - b), c/(a + b - c) are in HP
G1G2 = a2 / 3 b1 / 3 × a1 / 3 b2 / 3 = ab
(D) a/(a + b + c), b/(a + b + c), c/(a + b + c) are in HP
a + 2b 3ab Solution: Given that 1/a, 1/b, 1/c are in AP. Therefore
A2 H1 = × = ab
3 a + 2b
a+b+c a+b+c a+b+c
Answers: (A), (B), (D) , , are in AP
a b c
22. Consider four non-zero real numbers a, b, c, d (in this
a+b+c a+b+c a+b+c
order). If a, b, c are in AP and b, c, d are in HP, then Þ - 1, - 1, - 1 are in AP
a b c
(A) ac = bd (B) a / b = c / d
(C) a / c = b / d (D) a + b / a - b = c + d / c - d b+c c+a a+b
Þ , , are in AP
a b c
Solution: Since a, b, c are in AP,
a b c
2b = a + c Þ , , are in HP
b+c c+a a+b
Since b, c, d are in HP,
Therefore (B) is true. Also,
2bd
c= b+c c+a a+b
b+d - 1, - 1, - 1 are in AP
a b c
c(b + d) = 2bd = (a + c)d
a b c
Þ , , are in HP
therefore b+c-a c+a-b a+b-c
a b a+b c+d Therefore (C) is true. Also,
cb = ad and = , =
c d a-b c-d
a b c
Answers: (B), (C), (D) , , are in HP
a+b+c a+b+c a+b+c
23. Let a1, a2, a3, …, an be in AP and h1, h2 , h3, … , hn be in HP. Therefore (D) is true.
If a1 = h1 and an = hn, then Answers: (B), (C), (D)
(A) ar hn - r + 1 = a1an (B) an - r + 1 hr = a1an
(C) ar hn - r +1 is independent of r (D) ar hr = a1an 25. If a, b, c are in AP; b, c, d are in GP and c, d, e are in
HP, then
Solution: We have
(A) a, c, e are in GP (B) e = (2b - a)2 /a
æa -a ö (C) a, c, e are in HP (D) e = ab2 /(2a - b)2
ar = a1 + (r - 1) ç n 1 ÷
è n-1ø Solution: By hypothesis,
a1 n - a1 + (r - 1)an - a1r + a1 (5.46) 2ce
= 2b = a + c, c2 = bd and d =
n-1 c+e

a1 (n - r ) + an (r - 1) Therefore
= a + c 2ce
n-1 c2 = bd = ×
2 c+e
a1an (n - 1)
hn - r + 1 = and also
an (n - 1) + (n - r )(a1 - an )
(5.47) c(e + c) = e(a + e)
a1an (n - 1) This gives c = ae and hence (A) is true. Also,
2
=
an (r - 1) + a1 (n - r)
c2 (2b - a)2
From Eqs. (5.46) and (5.47), we get e= =
a a
ar hn - r + 1 = a1an = an - r + 1 hr and hence (B) is true.
Answers: (A), (B), (C) Answers: (A), (B)
www.jeeneetbooks.in
Worked-Out Problems 251

26. In Problem 25, if a = 2 and e = 18, then the possible (B) a = bx / y, c = bz / y and b2 = ac imply that
values of b, c, d are respectively,
(A) 4, 6, 9 (B) -2, -6, -18 b2 = b( x + z) / y
(C) 6, 4, 9 (D) 2, 6, 18 and hence

Solution: By Problem 25, c2 = ae = 2 ´ 18 and hence c = ± 6. 2y = x + z


Now, 2b = a + c = 2 ± 6 = 8 or −4 (b = 4 or −2). Therefore Thus (B) is true.
2
c 36 36 (C) We have
d= = or = 9 or - 18
b 4 -2
1 1 1
Hence b2 = ac, = logu a, = logu b, = logu c
loga u logb u logc u
b = 4, c = 6, d = 9
or b = - 2, c = - 6, d = - 18 Now 2 logu b = logu a + logu c . Therefore
Answers: (A), (B) 2 1 1
= +
27. Which of the following statement(s) is (are) true? logb u loga u logc u
(A) If ax = by = cz and a, b, c are in GP, then x, y, z are Thus (C) is true.
in HP.
(D) We have 2b = a + c. Therefore
(B) If a1 / x = b1 / y = c1/ z and a, b, c are in GP, then x, y, z
are in AP. 2ba
(C) If a, b, c are positive, each of them not equal c=
a+b
to 1, and are in GP, then, for any positive
u ¹ 1, loga u, logb u, logc u are in HP. (a + c)a
c=
(D) If a, b, c are in AP and b, c, a are in HP, then c, a+b
a, b are in GP. bc + ca = a2 + ac
Solution: (A) a = b , c = b
y/ x y/z
and b = ac imply that
2
a2 = bc
( y / x ) + ( y / z)
b =b
2

Thus (D) is true.


and hence
Answers: (A), (B), (C), (D)
æ 1 1ö
2 = yç + ÷
è x zø
Therefore x, y, z are in HP. Thus (A) is true.

Matrix-Match Type Questions


1. Match the items in Column I with those in Column II. Solution:
(A) We have a = 11/2, d = 5/4. Therefore
Column I Column II
n é æ 11ö 5ù
(A) If the sum of n terms of the series (p) 5 ê 2 ç ÷ + (n - 1) ú = 238
2ë è 2ø 4û
5(1/ 2), 6(3 / 4), 8, …
n é 44 + 5(n - 1) ù
is 238, then n is (q) -2(1/2) úû = 238
2 êë 4
(B) The first term of an AP is 5, the last
term is 45 and the sum of the terms
5n2 + 39 n - 8 ´ 238 = 0
is 400. The number of terms and the
(r) 2(2/3)
common difference are, respectively, 5n2 - 80 n + 119 n - 8 ´ 238 = 0
(C) The sum of three numbers which are
in AP is 27 and sum of their squares is 5n(n - 16) + 119(n - 16) = 0
293. Then the common difference is (s) 16
(D) The fourth and 54th terms of an AP n = 16
are, respectively, 64 and -61. The Answer: (A) Æ (s)
common difference is (t) -5
www.jeeneetbooks.in
252 Chapter 5 Progressions, Sequences and Series

(B) We have a = 5, d = common difference. Now 2. Match the items in Column I with those in Column II.

a + (n - 1)d = 45 Þ (n - 1)d = 40 (5.48)


Column I Column II
n
[10 + (n - 1)d] = 400
2 (A) If the pth, qth and rth terms of an (p) 15
n AP are a, b, c respectively, then the
(10 + 40) = 400 value of a (q - r) + b (r - p) +
2
c( p - q) is
800 (q) 27
n= = 16 (B) The sum of m terms of an AP is n
50 and the sum of n terms is m,
Substituting this value of n in Eq. (5.48) we get then the [sum of (m + n)
terms] + (m + n) is (r) -(m + n)
40 (C) If five arithmetic means are
d=
15 inserted between 2 and 4, then the
sum of the five means are
Finally, (s) 2(m + n)
(D) In an AP, if the sum of n terms is
2 3n2 and the sum of m terms is 3m2
n = 16, d = 2 (m ¹ n) then, the sum of the first
3
three terms is (t) 0
Answer: (B) Æ (r), (s)
(C) Let the three numbers be (a - d), a, (a + d). Then by Solution:
hypothesis
(A) Since a, b, c are pth, qth and rth terms of an AP, let
(a - d) + a + (a + d) = 27 a = a + ( p - 1)d (5.49)
3a = 27 b = a + (q - 1)d (5.50)
a=9 c = a + (r - 1)d (5.51)

Again by hypothesis, since sum of their squares is Solving Eqs. (5.49) and (5.50) we get
293 we have d( p - q) = a - b
(9 - d)2 + 92 + (9 + d)2 = 293 a-b
d=
p-q
243 + 2d2 = 293
Therefore
d2 = 25
(a - b)c
d = ±5 c( p - q) =
d
Answer: (C) Æ (p), (t)
Similarly,
(D) Since the fourth term is 64 we get
(c - a)b
b(r - p) =
a + 3d = 64 d
Since the 54th term is -61 we get a(b - c)
a(q - r ) =
d
a + 53d = - 61
Therefore
Solving the two equations we get
1 1
-50d = 125 å a(q - r ) = å a(b - c) = (0) = 0
d d
1 Answer: (A) Æ (t)
d = -2
2 (B) By hypothesis we get
Answer: (D) Æ (q) m
[2a + (m - 1)d] = n (5.52)
2
n
[2a + (n - 1)d] = m (5.53)
2
www.jeeneetbooks.in
Worked-Out Problems 253

Subtracting Eq. (5.53) from Eq. (5.52) and solving Solution:


we get (A) Let the three numbers be a - d, a, a + d. By hypothesis
2a(m - n) + d(m2 - n2 ) - d(m - n) = 2(n - m) (a - d) + a + (a + d) = 12
2a + (m + n - 1)d = - 2 3a = 12
Therefore sum of the first (m + n) terms is a=4
m+n Also it is given that the sum of their cubes is 288;
[2a + (m + n - 1)d] = -(m + n)
2 therefore
Answer: (B) Æ (r) (a - d)3 + a3 + (a + d)3 = 288
(C) Sum of the n AM’s between x and y is
3a3 + 6ad2 = 288
æx+ yö a3 + 2ad2 = 96
nç ÷ø
è 2
64 + 8d2 = 96
Therefore sum of the five AM’s between 2 and 4 is
d = ±2
æ 2 + 4ö
5ç = 15 Therefore the numbers are 2, 4, 6.
è 2 ÷ø
Answer: (A) Æ (r)
Answer: (C) Æ (p) (B) We have
(D) By hypothesis we have
2n æ nö
n [2a + (2 n - 1)d] = 3 ç ÷ [2a + (n - 1)d]
[2a + (n - 1)d] = 3n2 2 è 2ø
2 (5.54)
2a = (n + 1)d
2a + (n - 1)d = 6 n
Therefore
Similarly,
S3 n 3[2a + (3n - 1)d]
2a + (m - 1) d = 6m (5.55) =
sn 2a + (n - 1)d
Solving Eqs. (5.54) and (5.55) we get
3[(n + 1)d + (3n - 1)d]
d = 6, a = 3 =
(n + 1)d + (n - 1)d
d
Therefore, the sum of the first 3 terms is
æ 4n ö
3 = 3ç ÷ = 6
(6 + 2 ´ 6) = 27 è 2n ø
2
Answer: (B) Æ (r)
Answer: (D) Æ (q)
(C) The nth term is
3. Match the items in Column I with those in Column II.
4[n2 - (n - 1)2 ] = 4(2 n - 1)

Column I Column II Substituting n = 1, 2, 3, …, we get the series 4, 12, 20, … .


(A) The sum of three numbers which (p) 25 Answer: (C) Æ (t)
are in AP is 12 and the sum of their (D) By hypothesis
cubes is 288. The greater of the three
numbers is (q) 26 n
[6 + (n - 1)3] > 1000
(B) Let sn denote the sum of the first n 2
terms of an AP. If s2n = 3sn, then s3n/sn n(3n + 3) > 2000
equals (r) 6
2
(C) 4n2 is the sum of the first n terms of æ 1ö 2000 1
an AP whose common difference is çè n + ÷ø > +
(s) 7 2 3 4
(D) The least value of n for which the
sum 3 + 6 + 9 + + n is greater than
1000 is (t) 8
www.jeeneetbooks.in
254 Chapter 5 Progressions, Sequences and Series

1 Now
n+ > 25.8
2
æ 1ö
n = 26 x = log 5 ç ÷
è 2ø
Answer: (D) Æ (q)
= -2 log5 2
4. Match the items in Column I with those in Column II. Therefore
y2 = (0.2)x
Column I Column II
= (5-1 )-2log5 2 = 4
(A) If four GMs are inserted between (p) 7 Hence y = 2.
160 and 5, then the second mean is
Answer: (D) Æ (t)
(B) Sum to infinity of the series
(q) 20
1 1 5. Match the items in Column I with those in Column II.
1+ + 2 +
2 2
(C) If 52 × 54 × 56 52 n = (0.04)-28 , then the (r) 40 Column I Column II
value of the n is (A) A GP contains even number of (p) 1
(D) If y > 0 and y2 = (0.2)x where (s) 10 terms. The sum of all terms is
æ1 1 1 ö equal to five times the sum of all
x = log 5 ç + + + + ¥÷
è 4 8 16 ø odd terms. Then the common
then the value of y is (t) 2 ratio is
(B) In a GP, the terms are alternately (q) -2
Solution: positive and negative, beginning
(A) Let g1 , g2 , g3 , g4 be the four GMs between 160 and 5. with a positive term. Any term
Then gi = 160ri , 1 £ i £ 4 , and 5 = 160r5. Therefore is the AM of the next immediate
two terms. Then the common (r) 2
5
5 1 æ 1ö ratio is
r5 = = =ç ÷
160 32 è 2 ø 2 4 6
(C) If y = c(sin x + sin x + sin x + + ¥ )loge 2
(0 < x < p / 2) satisfies the equation
and hence r = 1/2. Hence
x2 - 17x + 16 = 0, then the value of
2 sin 2 x /(1 + cos2 x) is (s) 4
æ 1ö
g2 = 160 ç ÷ = 40
è 2ø (D) If the same y in (C) satisfies the same
Answer: (A) Æ (r) equation x2 - 17x + 16 = 0, then the
value of 6 sin x /(sin x + cos x) is
1 1 1 1 equal to (t) 2/3
(B) 1 + + + + + ¥ = =2
2 22 23 1 - (1/ 2)
Solution:
Answer: (B) Æ (t)
(A) Let a, ar, ar2 , …, ar2 n-1 be the 2n terms of the GP. It
(C) We have
is given that
52 + 4 + 6 + + 2 n = (0.04)-28
a + ar + ar2 + + ar2 n - 1 = 5(a + ar2 + ar4 + + ar2 n - 2 )
-28
æ 1ö
52 n[( n + 1)/ 2 ] = ç ÷ = 556 Therefore
è 25 ø
a(r2 n - 1) a[(r2 )n - 1] a(r2n - 1)
n(n + 1) = 56 =5 = 5
r-1 r2 - 1 (r - 1)(r + 1)
n=7
Solving this we get r + 1 = 5 or r = 4.
Answer: (C) Æ (p)
Answer: (A) Æ (s)
(D) We have
(B) Let the numbers be a, a( - r ), a( - r )2
, a(- r )3 , … where
y2 = (0.2)x a > 0 and r > 0. Since any term is the AM of the imme-
where diate next two terms, therefore

æ1 1 1 ö
x = log 5 ç + + + + ¥÷
è 4 8 16 ø
www.jeeneetbooks.in
Worked-Out Problems 255

a(- r ) + ar2 = 2a (B) Length of the diagonal of

r2 - r - 2 = 0 1
S4 = 2 a4 = 2 × a3 = 5
2
(r - 2)(r + 1) = 0
Answer: (B) Æ (q)
which gives r = 2.
(C) a1, a2, a3, … are in GP with a1 = 10 and common ratio
Answer: (B) Æ (r)
For parts (C) and (D) r = 1/ 2 . Therefore
n-1
p æ 1 ö
0 < x < Þ 0 < sin x < 1 an = 10 ç
2 è 2 ÷ø

Therefore, The area of Sn is


n-1
sin2 x æ 1ö
sin2 x + sin4 x + sin6 x + ¥ = = tan2 x an2 = 100 ç ÷
1 - sin2 x è 2ø
2
x + sin4 x + ¥ )loge 2 2
x×loge 2 2
e(sin = etan = 2tan x
100
and an2 < 1 Û < 1 Û 100 < 2 n-1 Û 7 < n
2n-1
The roots of x - 17x + 16 = 0 are 1 and 16. Then
2

2 Answer: (C) Æ (p)


(i) 2tan 2 x = 1 Þ tan x = 0, which is false since 0 < x < p / 2
(ii) 2tan x = 16 Þ tan2 x = 4 or tan x = 2 (D) Sum of the areas of the squares is
sin 2 x 2 sin x cos x 2 tan x 2 tan x 4 2 ¥ ¥
100
(C)
1 + cos x
2
=
1 + cos x
2
=
1 + sec x
2
=
2+4
= =
6 3 å an2 = å
n=1 n=1 2n-1

Answer: (C) Æ (t)


æ 1 1 ö
= 100 ç 1 + + 2 + ÷
6 sin x 6 tan x 6(2) è 2 2 ø
(D) = = =4
sin x + cos x tan x + 1 2 + 1 = 100(2)
Answer: (D) Æ (s) = 200 sq. unitts
Answer: (D) Æ (s)
6. Let S1, S2, S3, … be squares such that the length of the
side of Sn is equal to the length of the diagonal of Sn+1. 7. Match the items in Column I with those in Column II.
Match the items in Column I with those in Column II,
if the length of the side of S1 is equal to 10 units.
Column I Column II
Column I Column II (A) If n = 3, then the numbers 2n, (p) GP
n(n - 1) and n(n - 1)(n - 2) are in
(A) Length of the side of S3 is (p) 7
(B) If a, b, c are in AP, then
(B ) Length of the diagonal of S4 (q) 5
a + 1/bc, b + 1/ac and c + 1/ab are in (q) HP
(C ) The area of Sn is less than 1 if (r) 6
(C) If x > 1, y > 1 and z > 1are three
n is greater than (s) 200
numbers in GP, then the numbers
(D) Sum of the areas of the squares is (t) 10 2 /( 2 - 1) 1 1 1
, , are in (r) AGP
1 + log x 1 + log y 1 + log z
Solution: Let an be the length of the side of Sn. It is
(D) If a, b, c are in HP, then the
given that
1 b b b
an = 2an + 1 or an + 1 = an numbers a - , , c - are in (s) AP
2 2 2 2

Solution:
(A) Let a1 = 10. Then
(A) If n = 3, the three numbers are 6, 6, 6, which are in
1 1 1 AP, GP, HP and AGP.
a2 = 10 and a3 = a2 = 10 = 5
2 2 2 Answer: (A) Æ (p), (q), (r), (s)
Answer: (A) Æ (q)
www.jeeneetbooks.in
256 Chapter 5 Progressions, Sequences and Series

(B) Let a, b, c be in AP. Then b - a = c - b. Therefore Solution:


(A) We have
æ 1ö æ 1ö b-a
çè b + ÷ø - çè a + ÷ø = (b - a) + a + bK b + cK c + dK
ca bc abc = =
a - bK b - cK c - dK
c-b
= (c - b) +
abc a b c
= =
a - bK b - cK c - dK
æ 1ö æ 1ö
= çc + ÷ - çb + ÷
è ab ø è ca ø a - bK b - cK c - dK
= =
a b c
Answer: (B) Æ (s)
b c d
(C) Let x, y, z be in GP; x > 1, y > 1 and z > 1. Then = =
a b c
1 + log x, 1 + log y, 1 + log z are in AP
This implies that a, b, c, d are in GP.
and so Answer: (A) Æ (s)
1 1 1 (B) We have
, , are in HP
1 + log x 1 + log y 1 + log z a1 + a4 a2 + a3
=
a1 a4 a2 a3
Answer: (C) Æ (q)
1 1 1 1
(D) Let a, b, c be in HP. Then + = +
a1 a4 a2 a3
2ac
b= 1 1 1 1
a+c - = - (5.56)
a2 a1 a4 a3
Now
Also,
æ bö æ bö b b2
çè a - ÷ø çè c - ÷ = ac - ( a + c ) + æa -a ö a a
2 2ø 2 4 3ç 2 3 ÷ = 2 3
è a1 - a4 ø a1 a4
ac b2
= ac - (a + c ) + æ1 1ö 1 1
a+c 4 3ç - ÷ = - (5.57)
2 è a3 a2 ø a4 a1
b
=
4 From Eqs. (5.56) and (5.57), we get that
Answer: (D) Æ (p)
1 1 1 1
, , and
8. Match the items in Column I with those in Column II.
a1 a2 a3 a4
are in AP. Therefore a1, a2, a3, a4 are in HP.
Column I Column II Answer: (B) Æ (q)
(C) We have
(A) a, b, c and d are positive, each (p) AP
is not equal to 1 and K ¹ 1. If 2a2 = a1 + a3
a + bK b + cK c + dK
= = then a, b, a32 = a2 a4
a - bK b - cK c - dK
c, and d are in 2a3 a5
(q) HP a4 =
(B) If a1, a2, a3, and a4 are four numbers a3 + a5
a a a + a3 æa -a ö
such that 2 3 = 2 = 3ç 2 3 ÷ Therefore
a1 a4 a1 + a4 è a1 - a4 ø
then a1, a2, a3, and a4 are in æ a + a ö æ 2a a ö
a32 = ç 1 3 ÷ ç 3 5 ÷
(C) If a1, a2, a3 are in AP; a2, a3, a4 are in GP (r) AGP è 2 ø è a3 + a5 ø
and a3, a4, a5 are in HP, then a1, a3, a5
a3 (a3 + a5 ) = a5 (a1 + a3 )
are in
(D) If the sum to n terms of a series is
pn2, then the series is in (s) GP
www.jeeneetbooks.in
Worked-Out Problems 257

a32 = a1 a5 (D) The nth term is given by

Therefore a1, a2, a3 are in GP. un = sn - sn -1 = p(n2 ) - p(n - 1)2 = p(2 n - 1)


Answer: (C) Æ (s)
The series is p, 3p, 5p, 7p, … which is an AP.
Answer: (D) Æ (p)

Comprehension-Type Questions
1. Passage: Let a be the first term and d the common So, the sum of the first (m + n) terms is
difference of an AP. Then sum of the first n terms is
m+n
n [2a + (m + n - 1)d] = 0
[2a + (n - 1)d] 2
2
Answer: (D)
If n AMs are inserted between a and b, then the kth (ii) We have
AM is
n(n + 1)
(b - a) S1 = 1 + 2 + 3 + + n =
a+k (k = 1, 2, 3, …, n) 2
n+1
n
S2 = [2 + (n - 1)2] = n2
Now, answer the following questions. 2
(i) If the sum of the first m terms of an AP is same as n n(3n - 1)
the sum of the first n terms, then sum of the first S3 = [2 + (n - 1)3] =
2 2
(m + n) terms is equal to
(A) mn (m + n) (B) (mn + 1)(m + n) Now
(C) (mn - 1)(m + n) (D) 0 n
S1 + S3 = [n + 1 + 3n - 1] = 2 n2 = 2S2
(ii) S1, S2, S3 are sums of first n terms of three APs 2
whose first terms are unity and the common
difference are respectively 1, 2, 3. Then S1 + S3 is Answer: (C)
equal to (iii) We have f (n) = 3n - 1 which implies that f (1), f (2),
(A) S2 (B) 3S2 (C) 2S2 (D) S22 f (3), … are in AP with first term 2 and common
difference 3. Therefore
(iii) Let N be the natural number set and f : N ® 
be a function defined by f (n) = 3n - 1. If n n
155 = [4 + (n - 1)3] = [3n + 1]
2 2
n

å f (k) = 155
k =1
3n2 + n - 310 = 0

then (n - 10)(3n + 31) = 0


(A) n = 8 (B) n = 10 (C) n = 11 (D) n = 9 n = 10
Solution: Answer: (B)
(i) We have
2. Passage: Let p < q < r < s and p, q, r, s be in AP. Further,
m n
[2a + (m - 1)d] = sm = sn = [2a + (n - 1)d] let p and q be the roots of the equation x2 - 2x + A = 0
2 2 while r and s be the roots of x2 - 18x + B = 0. Answer
Therefore the following questions.
(i) | A + B | is equal to
2a(m - n) = d[n2 - m2 + m - n]
(A) 80 (B) 74 (C) 84 (D) 77
2a = d[-(m + n) + 1] = - d(m + n - 1)
2a + d(m + n - 1) = 0
www.jeeneetbooks.in
258 Chapter 5 Progressions, Sequences and Series

(ii) If an AP is formed with A as first term and 8 as (iii) Sum of the nine means is
common difference then B appears at
9
(A) 11th place (B) 12th place (-3 + 77) = 37 ´ 9 = 333
2
(C) 10th place (D) no place
Answer: (C)
(iii) In the above question, the sum of the nine AMs
inserted between A and B is
3. Passage: Let a, b, c be in GP. Answer the following
(A) 233 (B) 323 three questions.
(C) 333 (D) 222
a2 + ab + b2
Solution: (i) is equal to
ab + bc + ca
(i) Let p = a - 3d, q = a - d, r = a + d and s = a + 3d. Now
(A) (a + b)/(b + c) (B) (b + c)/(c + a)
p < q < r < s Þ d > 0 Since p and q are the roots of the
equation x2 - 2x + A = 0 we have (C) (c + a)/(a + b) (D) (a + b)/ 2(b + c)
(ii) If ab + bc + ca = 156 and abc = 216 and the num-
p+q=2 bers are in the descending order, then the
a - 3d + a - d = 2 common ratio is
(5.58)
(A) 1/2 (B) 2 (C) 3 (D) 1/3
2a - 4d = 2
(iii) If a + b + c = 14 and a + 1, b + 1 and c - 1 are in AP,
a - 2d = 1 then the sum to infinity of the GP whose first three
terms are a, b and c (in the descending order) is
Since r and s are the roots of x2 - 18x + B = 0 we have
(A) 8 (B) 16 (C) 4 (D) 2
r + s = 18
Solution:
a + d + a + 3d = 18 (i) Let b = ar and c = ar2. Then
(5.59)
2a + 4d = 18
a2 + ab + b2 a2 (1 + r + r2 )
a + 2d = 9 = 2
ab + bc + ca a r + a2 r3 + a2 r2
Solving Eqs. (5.58) and (5.59) we get a = 5. Substitu- a2 (1 + r + r2 ) 1
ting the value of a in Eq. (5.58) we get = =
a2 r(1 + r + r2 ) r
a - 2d = 1 a(r + 1) ar + a a+b
= = 2 =
5 - 2d = 1 r(r + 1)a ar + ar b + c
Answer: (A)
4 = 2d
(ii) Let the numbers a, b and c be x/r, x and xr, respec-
2=d tively. Then
Now x
× x × xr = 216
r
A = pq = (5 - 6)(5 - 2) = - 3
x3 = 216
B = rs = (5 + 2)(5 + 6) = 77
x=6
Therefore
Also,
| A + B | = 74
æ xö æ xö
Answer: (B) çè ÷ø x + x( xr ) + ( xr ) çè ÷ø = 156
r r
(ii) We have
æ1 ö
x2 ç + r + 1÷ = 156
77 = - 3 + 8(n - 1) èr ø
n = 11 1 + r + r2 156 156 13
= 2 = =
Answer: (A) r x 36 3
3r2 - 10r + 3 = 0
www.jeeneetbooks.in
Worked-Out Problems 259

(3r - 1)(r - 3) = 0 (ii) The common ratio of the GP is


1 (A) 2 (B) 1/2 (C) 3 (D) 1/3
r= or 3
3 (iii) The sum of four AM’s between x and y and
product of four GMs between x and y is
The numbers are in descending order. Therefore
(A) 4 + 4 3
r = 1/3.
Answer: (D) (B) 4(2 + 3 )
(iii) Let a = x/r, b = x and c = xr. Then (C) 8(1 + 3 )
(D) 17
æ1 ö
x ç + 1 + r ÷ = 14 (5.60) Solution:
èr ø
(i) Since x, x + 2y and 2x + y are in AP, we have
Also, given that (x/r) + 1, x + 1 and xr - 1 are in AP. x + (2x + y) = 2 ( x + 2y)
Therefore
x = 3y
æx ö Again, since ( y + 1)2, xy + 5 and (x + 1)2 are in GP,
çè + 1÷ø + ( xr - 1) = 2( x + 1)
r we have
æ r2 + 1 ö ( x + 1)2 ( y + 1)2 = ( xy + 5)2
xç - 2÷ = 2 (5.61)
è r ø
By substituting x = 3y in this, we get that
From Eqs. (5.60) and (5.61),
(3 y2 + 5)2 = (3 y + 1)2 ( y + 1)2 = (3 y2 + 4 y + 1)2
r 2 + r + 1 14
= =7 (3 y2 + 5 + 3 y2 + 4 y + 1)[3 y2 + 5 - (3 y2 + 4 y + 1)] = 0
r 2 - 2r + 1 2
(6 y2 + 4 y + 6)(- 4 y + 4) = 0
6r 2 - 15r + 6 = 0
(3 y2 + 2 y + 3)( y - 1) = 0
2r 2 - 5r + 2 = 0
(2r - 1)(r - 2) = 0 Now 3y2 + 2y + 3 = 0 has no real roots and hence
y = 1 and x = 3y = 3. Therefore, the AP is 3, 5
1
r= orr 2 and 7. The common difference is 2.
2
Answer: (A)
Then (ii) We have

r=
1
and x=4 ( y + 1)2 = 22
2
xy + 5 = 3 + 5 = 8
a = 8, b = 4, c = 2
( x + 1)2 = 42 = 16
The sum to infinity is
Therefore the common ratio is 2.
8
s¥ = = 16 Answer: (A)
1 - (1/ 2)
(iii) We have x = 3 and y = 1. The sum of four AM’s
Answer: (B) between 3 and 1 is
(3 + 1)
4. Passage: Let x and y be real numbers such that x, 4 =8
2
x + 2y and 2x + y are in AP and ( y + 1)2, xy + 5 and
( x + 1)2 are in GP. With this information, answer the The product of four GMs between 3 and 1 is
following three questions. ( 3 )4 = 9. Therefore the required sum is 8 + 9 = 17.
(i) The common difference of the AP is Answer: (D)
(A) 2 (B) 3 (C) 4 (D) 3/2
www.jeeneetbooks.in
260 Chapter 5 Progressions, Sequences and Series

Assertion–Reasoning Type Questions


In each of the following, two statements, I and II, are Solution: Let d be the common difference. Then
given and one of the following four alternatives has to
be chosen. a5 + a20 = (a + 4d) + (a + 19d) = 2a + 23d = a1 + a24
(A) Both I and II are correct and II is a correct reasoning a10 + a15 = (a + 9d) + (a + 14d) = 2a + 23d = a1 + a24
for I.
Therefore Statement II is true. Also
(B) Both I and II are correct but II is not a correct
reasoning for I. 225 = a1 + a5 + a10 + a15 + a20 + a24 = 3(a1 + a24 )
(C) I is true, but II is not true. a1 + a24 = 75
(D) I is not true, but II is true.
Hence
1. Statement I: If log a, log b, log c are in AP and log a - 24
log 2b, log 2b - log 3c, log 3c - log a are also in AP, then 24
a, b, c form the sides of a triangle.
åa
k =1
k =
2
(a1 + a24 ) = 12 ´ 75 = 900

Statement II: Three positive real numbers form the Therefore Statement I is false.
sides of a triangle, if the sum of any two of them is Answer: (D)
greater than the third.
Solution: Clearly a, b, c are positive and 3. Statement I: If a, b, c, are real numbers satisfying the
relation 25(9a2 + b2) + 9c2 - 15(5ab + bc + 3ca) = 0, then
log a + log c = 2 log b Þ b = ac
2
(5.62) a, b, c are in AP.

Also Statement II: If x, y, z are any real numbers such that


x2 + y2 + z2 - xy - yz - zx = 0 , then x = y = z.
(log a - log 2b) + (log 3c - log a) = 2(log 2b - log 3c)
Solution: We have
log 3c - log 2b = 2(log 2b
b - log 3c)
x2 + y2 + z2 - xy - yz - zx = 0
3(log 3c - log 2b) = 0
1
Û [( x - y)2 + ( y - z)2 + (z - x)2 ] = 0
2b = 3c (5.63) 2
From Eqs. (5.62) and (5.63) we get Ûx= y=z

9c 3c Therefore, Statement II is true. For Statement I, take


a= ,b= x = 15a, y = 5b and z = 3c so that
4 2
Now x2 + y2 + z2 - xy - yz - zx = 0
15c Þx= y=z
a+b= >c
4 Þ 15a = 5b = 3c
3c 5c 5 æ 4a ö Þ b = 3a and c = 5a
b+c= +c= = ç ÷ >a
2 2 2è 9 ø
Therefore, a, b, c are in AP.
9c 13c 3c Answer: (A)
c+a=c+ = > =b
4 4 2
4. Statement I: A ball is dropped from a height of 8 feet.
Therefore a, b, c form the sides of a triangle. In other words,
Each time the ball hits the ground, it rebounds half
Statements I and II are true and II is a correct reason for I.
the height. The total distance travelled by the ball
Answer: (A) when it comes to rest is 16 feet.

2. Statement I: Let a1 , a2 , a3 , … , a24 be in AP. If a1 + a5 + Statement II: Sum to infinity of GP with common
a10 + a15 + a20 + a24 = 225, then a1 + a2 + a3 + + a24 is ratio r (| r | < 1) and first term a is a /(1 – r).
equal to 800. Solution: Statement II is clearly true. But the distance
travelled by the ball follows an infinite GP after hitting the
Statement II: a5 + a20 = a10 + a15 = a1 + a24 .
ground. Therefore, distance travelled by the ball equals
www.jeeneetbooks.in
Worked-Out Problems 261

é 1 1 ù æ 4 ö which is true because 0 < r < 1/2. Therefore both


8 + 2 ê 4 + 2 + 1 + + + ú = 8 + 2 ç
è 1 - (1/ 2) ÷ø
Statements I and II are true and II is a correct explana-
ë 2 4 û
tion of I.
= 8 + 16 = 24 feet Answer: (A)
I is false and II is correct.
7. Statement I: In a GP, the sum of the first n terms is
Answer: (D) 255, the nth term is 128 and the common ratio is 2.
Then the value of n is 8.
5. Statement I: The sum to infinity of the series 1(0.1) +
3(0.01) + 5(0.001) + 29/81. Statement II: The sum of the first n terms of a GP
whose first term is a and common ratio r is
Statement II: Sum to infinity of AGP a + (a + d)r +
(a + 2d)r2 + is a(1 - rn )
1- r
a dr
+ when | r | < 1
1 - r (1 - r )2 Solution: The nth term is arn- 1 = 128 and r = 2. Therefore
Solution: The series given in Statement I is an AGP a × 2n - 1 = 128
with a = 1/ 10, d = 2 and r = 1/ 10. Therefore the sum to
infinity is given by Now
a dr a(1 - rn ) a - arn a - 256
+ 255 = = =
1 - r (1 - r )2 1- r 1- r 1- 2
Substituting the values we get a=1

1/ 10 2(1/ 10) 1 20 29 Also,


+ = + =
1 - (1/ 10) [1 - (1/ 10)]2 9 81 81 arn - 1 = 128
Statements I and II are both correct and II is the correct 2n - 1 = 128
explanation for I.
n - 1 = 7 or n = 8
Answer: (A)
Answer: (A)
6. Statement I: In a GP, if the common ratio is positive
and less than 1/2 and the first term is positive, then 8. Statement I: ABC is an equilateral triangle with side 24.
each term of the GP is greater than sum to infinity of D A1B1C1 is formed from D ABC joining the midpoints
all the terms of the GP that follow it. of its sides. Again D A2B2C2 is formed by joining the
midpoints of the sides of D A1B1C1. The process is con-
Statement II: If the common ratio r of a GP is such
tinued infinitely. Then the sum of the perimeters of all
that -1 < r < 1 and the first term is a, then the sum to
the triangles including D ABC is 144.
infinity of the GP is
a Statement II: The area of an equilateral triangle of
side ‘a’ units is ( 3 / 4) a2 square units.
1- r
Solution: Sum to infinity of the perimeters is given by
Solution: Statement II is clearly correct. Let the GP be
a, ar, ar2 , ar3 , … and 0 < r < 1/ 2. The nth term is arn-1 (= tn, æ aö æ aö
3a + 3 ç ÷ + 3 ç 2 ÷ +
say). Now, è 2ø è2 ø
arn Substituting a = 24 we get
arn + arn + 1 + arn + 2 + = arn (1 + r + r2 + ) =
1- r
æ 1 1 ö 1
Therefore 3a ç 1 + + 2 + ÷ = 3 ´ 24 ´ = 144
è 2 2 ø 1 - (1/ 2)
ar n + 1
tn > tn + 1 + tn + 2 + ¥ Û arn - 1 > Therefore Statements I and II are correct. However,
1- r
Statement II is not the correct explanation for I.
Û arn - 1 (1 - r ) > arn + 1
Answer: (B)
Û1-r > r 2
www.jeeneetbooks.in
262 Chapter 5 Progressions, Sequences and Series

9. Statement I: If a, b, c and x are real and (a2 + b2) x2 - Hence


2b(a + c) x + (b2 + c2 ) = 0 , then a, b, c are in GP with x
- d å bc(q - r ) = å(b - c) = 0
as common ratio.
å bc(q - r ) = 0
Statement II: For any real numbers p and q, p2 + q2 = 0
Û p = 0 = q. Statement II is correct and I is false.
Solution: The given equation can be written as Answer: (D)

(a2 x2 - 2abx + b2 ) + (b2 x2 - 2bcx + c2 ) = 0 11. Statement I: If

(ax - b)2 + (bx - c)2 = 0 Hn = 1 +


1 1
+ + +
1
2 3 n
ax - b = 0 and bx - c = 0
then
Solving for x we get
æ1 2 3 n - 1ö
b c Hn = n - ç + + + + ÷
x= = è2 3 4 n ø
a b
Therefore a, b, c are in GP with common ratio x. Statement II: If K > 1 is an integer, then
Answer: (A) 1 K -1
+ =1
K K
10. Statement I: If a, b, c are, respectively, the pth, qth, rth
Solution: Hn can be written as
terms of an HP, then (q - r)bc + (r - p)ca + (p - q) ab =
( p + q + r)abc. æ 1ö æ 2ö æ 3ö æ n - 1ö
Hn = 1 + ç 1 - ÷ + ç 1 - ÷ + ç 1 - ÷ + + ç 1 - ÷
Statement II: The nth term of an HP is of the form è 2 ø è 3 ø è 4 ø è n ø
1 æ1 2 3 n - 1ö
= n -ç + + + + ÷
a + (n - 1)d è2 3 4 n ø

Solution: Since a is the pth term of an HP, let Both Statements I and II are correct and II is a correct
explanation of I.
1
a= Answer: (A)
a1 + ( p - 1)d
12. Statement I: If a, b, c are in AP and b, c, d are in HP,
Therefore then a : b = c : d.
1 Statement II: AM of x and y is ( x + y)/ 2 and HM of
= a1 + ( p - 1)d
a x and y is 2xy/(x + y).
Similarly, Solution: We have

1 2bd
= a1 + (q - 1)d 2b = a + c and c =
b b+d

1 Therefore
= a1 + (r - 1)d
c (a + c)d
c=
Now b+d

1 1 bc + cd = ad + cd
- = (r - q)d
c b bc = ad or a : b = c : d
Therefore Both Statements I and II are correct and II is a correct
explanation of I.
b - c = - (q - r )bcd
Answer: (A)
Similarly,
13. Statement I: If a, b and c are positive real numbers, then
c - a = - (r - p)cad
æ 1 1 1ö
a - b = - ( p - q)abd (a + b + c ) ç + + ÷ ³ 9
è a b cø
www.jeeneetbooks.in
Worked-Out Problems 263

Statement II: AM of three positive real numbers ≥ and hence the sum equals
their GM.
(a2 K - a1 ) K
- × K × (a1 + a2 K ) = (a12 - a22K )
Solution: We have 2K - 1 2K - 1
a+b+c Statements I and II are both correct and II is a correct
³ (abc)1/ 3 explanation of I.
3
1 1 1 Answer: (A)
+ + 1/ 3
a b c ³ æ 1 × 1 × 1ö
çè ÷ 15. Statement I: Sum to infinity of the series
3 a b cø
3 5 9 15 23
æ 1 1 1ö æ 1 ö
1/ 3
+ + + + +
(a + b + c) ç + + ÷ ³ (abc)1/ 3 ç ×9 = 9 1! 2 ! 3! 4 ! 5!
è a b cø è abc ÷ø
is 4e.
Statements I and II are both correct and II is a correct
Statement II: The nth term of the series in Statement I is
explanation for I.
Answer: (A) n2 - n + 3 1 1 1
and e = 1 + + + + ¥
n! 1! 2 ! 3!
14. Statement I: If
Solution: The nth term is
1 1 1
, , …, , … n2 - n + 3 n(n - 1) + 3 1 3
a1 a2 an un = = = + for all n ³ 2
n! n! (n - 2)! n !
are in HP, then
Therefore
K ¥
a12 - a22 + a32 - a42 + + a22K - 1 - a22K = (a12 - a22K ) æ 1 1 1 ö æ1 1 1 ö
2K - 1 åu
n= 2
n = ç 1 + + + + ÷ + 3 ç + + + ÷
è 1! 2 ! 3! ø è 2 ! 3! 4 ! ø
Statement II: If x1 , x2 , … , xn , … are in AP, then x2 - = e + 3(e - 2) = 4e - 6
x1 = x3 - x2 = x4 - x3 =
The required sum is 3 + (4e - 6) = 4e - 3. Statement II is
Solution: Let correct and I is not correct.
1 1 Answer: (D)
, ,…
a1 a2 ¥
K 1
16. Statement I: åK 4
=
+K +1 2
2
be in HP. Then a1 , a2 , … are in AP, with common differ- K =1

ence, say d. Now, K


Statement II: K 4 + K 2 + 1
d = a2 - a1 = a3 - a2 = a4 - a3 =
1æ 1 1 ö
= - 2
2 è K - K + 1 K + K + 1÷ø
ç
Therefore 2

a12 - a22 = (a1 - a2 )(a1 + a2 ) = - d(a1 + a2 )


Solution: Statement II is correct and
a32 - a42 = - d(a3 + a4 ) ¥
K ¥
1é 1 1 ù
a2
2 K -1 -a 2
2K = - d(a2 K - 1 + a2 K ) åK
K =1
4
=å ê 2 - ú = lim sn
+ K + 1 K = 1 2 ë K - K + 1 K 2 + K + 1 û n®¥
2

Adding we get where


n
K
a12 - a22 + a32 - a42 + + a22K - 1 - a22K = - d(a1 + a2 + + a2 K ) sn = åK
K =1
4
+ K2 + 1
2K
= -d × [a1 + a2 K ] 1æ 1 ö
2 = ç1- 2 ÷
= - dK (a1 + a2 K ) 2è n + n + 1ø

Now, a2 K = a1 + (2 K - 1)d . Therefore Therefore


1
lim sn =
a2 K - a1 n ®¥ 2
d= Answer: (A)
2K - 1
www.jeeneetbooks.in
264 Chapter 5 Progressions, Sequences and Series

SUMMARY
+
5.1 Sequence: Let  be the set of all positive integers QUICK LOOK
and X any set. Then a mapping a : + ® X is called
a sequence in x. For any n Î+, we prefer to write 1. If {an} is an AP and K is any real number, then
an for the imag a(n) and the sequence is denoted by {an + K} is also an AP with same common difference.
{an}. 2. {Kan} is also an AP.
3. If {an} and {bn} are arithmetic progressions, then
5.2 Finite and infinite sequences: A sequence is said
{an + bn} is also an AP.
to be finite if its range is finite. A sequence which is
not finite is said to be infinite sequence.

5.3 Constant and ultimately constant sequences: A sequ- 5.11 Product of two AP’s: Product of two arithmetic
ence {an} is called a constant sequence if an = am progressions is also an AP if and only if one of
for all positive integers n and m. Sequence {an} them must be a constant sequence.
in called ultimately constant, if there is a positive
integer m such that an is constant for n > m, that is 5.12 Arithmetic mean (AM): If three real numbers
am+1 = am+2 = am+3 = a, b, c are in AP, then b is called AM between a
and c and in this case 2b = a + c.
5.4 Series: If {an} is a sequence of real or complex numbers,
then an expression of the form a1 + a2 + a3 + is 5.13 Arithmetic means (AM’s): If a, A1, A2, … An and b
called series. If sn is the sum of the first n terms of the are in AP, the A1, A2, … An are called n AM’s
sequence {an}, then again {sn} is a sequence called nth between a and b.
partial sum of the series or simply the sequence of
5.14 Formula for n AM’s between a and b: If A1,
partial sums of the series.
A2, … An are n AM’s between a and b, then the Kth
mean AK is given by
5.5 Limit of a sequence: Let {an} be a sequence of real
numbers and l a real number. Then l is said to be (b - a)
AK = a + K for K = 1, 2, …, n
limit of the sequence {an} if, for each positive real n+1
number Î (epsilon) there exists a positive integer n0
(depending on Î) such that | an - l | < Î for all n ³ n0. 5.15 Sum to first n terms of an AP: Let sn be the sum
to first n terms of an AP with first term ‘a’ and
5.6 Uniqueness of a limit: If a sequence has a limit, common difference ‘d’. Then
then the limit is unique.
n n
sn = [2a + (n - 1)d] or sn = [first term + nth term]
5.7 Notation: If l is the limit of a sequence {an}, then 2 2
we write lim an = l (or lt an = l ) and some times we
n®¥ n®¥
write an ® l. QUICK LOOK

If A1, A2, …, An are n AM’s between a and b then


5.8 Sum of an infinite series: Let {an} be a sequence of real
n(a + b)
numbers and sn = a1 + a2 + + an. If the sequence {sn} A1 + A2 + + An =
¥ 2
of partial sums has limit s, then we write å n = 1 an = s.
If {sn} has no finite limit, then the series is said to be
divergent.
5.16 Ratio of nth terms of two AP’s: Let tn be the nth term
of an AP whose first term is a and common differ-
5.9 Arithmetic progression (AP): A sequence {an}
ence d and sn is its sum to first n terms. Let tn¢ be the
of real numbers is called an arithmetic progres-
nth term of another AP with first term b and common
sion (AP) if an+1 - an is constant for all positive inte-
difference e whose sum to first n terms is sn¢ . Then
gers n ³ 1, and this constant number is called the
common difference of the AP. tn s2 n - 1
=
tn¢ s2¢n - 1
5.10 General form of AP: The terms of an AP with first
term ‘a’ and common difference d are a, a + d, a + 2d, 5.17 Characterization of an AP: A sequence of real
a + 3d, …, and the nth term being a + (n - 1)d. numbers is an arithmetic progression if and only if
www.jeeneetbooks.in
Summary 265

its sum of the first n terms is a quadratic expression 5.26 Arithmetic geometric progression (AGP): Sequence
in n with constant term zero. of numbers of the form a, (a + d)r, (a + 2d)r2, + is
called AGP and sum to n terms of an AGP is
5.18 Helping points:
a dr(1 - rn-1 ) (a + (n - 1)d)rn
(1) Three numbers in AP can be taken as a - d, a, + -
1- r (1 - r )2 1- r
a + d.
(2) Four numbers in AP can be taken as a - 3d, a dr
a - d, a + d, a + 3d. and +
1 - r (1 - r )2
(3) Five numbers in AP can be taken as a - 2d,
a - d, a, a + d, a + 2d. is the sum to infinity.

5.19 Geometric progression (GP): A sequence {an} of 5.27 AM–GM inequality: Let a1, a2, …, an be positive
non-zero real numbers is called GP if an/an-1 = an+1/ reals. Then
an for n ³ 2. That is the ratio an+1/an is constant for a1 + a2 + + an
n ³ 1 and this constant ratio is called the common n
ratio of the GP and is generally denoted by r.
is called AM of a1, a2, …, an and (a1 a2 an)1/n is
5.20 General form: GP with first term a and common called their GM. Further
ratio r can be expressed as a, ar, ar2, … whose nth
a1 + a2 + + an
term is arn-1. ³ (a1 a2 an )1/n
n
QUICK LOOK and equality holds if and only if a1 = a2 = a3 = = an.
1. If three numbers are in GP, then they can be taken
as a/r, a, ar. 5.28 Harmonic progression (HP): A sequence of non-zero
reals is said to be in HP, if their reciprocals are in AP.
2. If four numbers are in GP, then they can be taken
as a/r2, a/r, ar, ar2.
5.29 General form of an HP: Sequence of real numbers

1 1 1 1
5.21 Sum to first n-terms of a GP: The sum of the first , , , ,
a a + d a + 2d a + (n - 1)d
n-terms of a GP with first term ‘a’ and common
ratio r ¹ 1 is can be taken as general form of an HP.
a(1 - rn )
5.30 Harmonic mean and Harmonic means:
1- r
(1) If a, b, c are in HP, then b is called the Harmonic
5.22 Sum to infinity of a GP: If -1 < r < l is the common mean (HM) between a and c and in this case
ratio of a GP whose first term is a, then a/1 - r is b = 2ac/a + c.
called sum to infinity of the GP. (2) If a, h1, h2, …, hn, b are in HP then h1, h2, …, hn
are called n HM’s between a and b and further
5.23 Geometric mean and geometric means: If three
numbers a, b and c are in GP, then b is called ab(n + 1)
hK = for K = 1, 2, …, n
the Geometric mean (GM) between a and c and b(n + 1) + K (a - b)
b2 = ac. If x and y are positive real numbers, then x,
xy , y are in GP. 5.31 Theorem: Let a1, a2, …, an be positive reals and A, G
If a, g1, g2, … gn, b are in GP, then g1, g2, …, gn are be AM and GM of the given numbers. Let
called n geometric means between a and b.
n
H=
5.24 Formula for GM’s: If g1, g2, g3, …, gn are n GM’s 1/a1 + 1/a2 + + 1/an
between a and b, then kth GM gk is given by
gk = a(b/a)k/n+1 for k = 1, 2, …, n. which is called harmonic mean of a1, a2, …, an.
Then A ³ G ³ H and equality holds if and only if
5.25 Product of n GM’s: The product of n GM’s between a1 = a2 = a3 = = an.
a and b is ( ab )n.
www.jeeneetbooks.in
266 Chapter 5 Progressions, Sequences and Series

EXERCISES
Single Correct Choice Type Questions
1. If a, b and c are in AP, then (a - c)2 is equal to 8. Let a and b be positive real numbers such that a > b
(A) 2(b - ac)
2
(B) 4b - ac
2 and a + b = 4 ab . Then a : b is equal to
(C) b2 - 4ac (D) 4(b - ac)
2 (A) 2 + 3 : 2 - 3 (B) 3 + 2 : 3 - 2
(C) 4 + 3 : 4 - 3 (D) 3 : 2
2. Let a1 , a2 , a3 , … be an AP. If a3 = 7 and a7 = 3a3 + 2 ,
then the common difference is 9. The sum of the first n terms of two sequences in AP
(A) 1 (B) 4 (C) –1 (D) – 4 are in the ratio (3n – 13) : (5n + 21). Then the ratio of
their 24th terms is
3. Let a and b be positive real numbers. Then the sum of (A) 2 : 3 (B) 3 : 2 (C) 1 : 2 (D) 3 : 4
the first 10 terms of the series
10. In an AP , the first term, the (n - 1)th term and the
æ a2 ö æ a3 ö æ a4 ö nth term are a, b and c, respectively. Then the sum of
log a + log ç ÷ + log ç 2 ÷ + log ç 3 ÷ +
è bø èb ø èb ø the first n terms is
is
(a + b + 2c)(a + b) (a + b - 2c)(a + b)
(A) 5 (11 log a - 9 log b) (B) 5 (10 log a - 9 log b) (A) (B)
2(b - c) 2(b - c)
æ aö
(C) 10 (11 log a - 9 log b) (D) 50 log ç ÷ (a + b + 2c)(a + c) (2c - a - b)(a + c)
è bø (C) (D)
4. The fourth power of the common difference of an 2(c - b) 2(c - b)
arithmetic progression with integer entries is added to 11. In an AP , if the mth term is 1/n and the nth term is
the product of any four consecutive terms of it. Then 1/m, then the (mn)th term is
the resultant is
(A) (perfect square of an integer) + 1 mn (mn + 1) mn (mn - 1)
(A) mn - 1 (B) mn + 1
(B) cube of an integer
(C) perfect square of an integer mn (m + n)
(C) m-n (D) independent of m
(D) (cube of an integer) + 1 and n
5. If 12. The sums of the first n, 2n and 3n terms of an AP are
s1, s2 and s3, respectively. Then s3 is equal to
æ 7ö
log3 2, log3 (2x - 5), log3 ç 2x - ÷ (A) 2(s2 - s1) (B) 3(s2 - s1)
è 2ø
(C) 4(s2 - s1) (D) 2s1s2
are AP , then the value of x is
(A) 2 (B) 3 (C) 4 (D) 2 or 3 13. The ages of boys in a certain class of a school follow
an AP with the common difference 4 months. If the
6. Let a and b be two positive integers and youngest boy is of 8 years and the sum of the ages of
all the boys in the class is 168 years, then the number
a+b of boys in the class is
= x, ab = y and 2 x + y2 = 27
2 (A) 16 (B) 17 (C) 18 (D) 19
Then the numbers a and b are
14. Let sn be the first n terms of an AP with first term a
(A) 6, 3 (B) 5, 10 (C) 6, 10 (D) 6, 12
and common difference d. If skn/sn is independent of n,
7. Let a1, a2, a3, … be an arithmetic progression of then
positive real numbers. Then (A) Kn = 6 (B) d = 2a
1 1 1 (C) a = 2d (D) Kn = 8
+ + + =
a1 + a2 a2 + a3 an - 1 + an
15. If the sum of the first m terms of an AP is equal to
n+1 n-1 the sum of either the next n terms or to the next p
(A) (B) terms, then
a1 + an a1 + an
æ 1 1ö æ 1 1ö
n n (A) (m + n) ç - ÷ = (m + p) ç - ÷
(C) (D) è m pø è m nø
a1 + an an - a1
www.jeeneetbooks.in
Exercises 267

æ 1 1ö æ 1 1ö 23. Let a and b be distinct positive real numbers. If a,


(B) (m + n) ç + ÷ = (m + p) ç + ÷ A1, A2, … A2 n - 1 , b are in AP; a, G1 , G2 , … , G2 n - 1 , b are
è m pø è m nø
in GP and a, H1 , H2 , … , H2 n - 1 , b are in HP, then the
æ 1 1ö æ 1 1ö roots of the equation An x2 - Gn x + Hn = 0 are
(C) (m - n) ç - ÷ = (m - p) ç - ÷
è m pø è m nø (A) real and unequal (B) real and equal
æ 1 1ö æ 1 1ö (C) imaginary (D) rational
(D) (m - n) ç + ÷ = (m - p) ç + ÷
è m pø è m nø
24. If Sr denotes the sum of the first r terms of a GP, then
16. Suppose a, b, c are in AP and a2, b2, c2 are in GP. If Sn, S2n - Sn and S3n - S2n are in
a < b < c and a + b + c = 3/2, then the value of a is (A) AP (B) GP (C) HP (D) AGP
(A) 1/ 2 2 (B) 1/ 2 3
(C) (1/ 2) - (1/ 3 ) (D) (1/ 2) - (1/ 2 ) 25. Let a be the first term and r the common ratio of a
GP. If A and H are the AM and HM of the first n
17. The sum of an infinite geometric series is 162 and terms of the GP, then the product A × H is equal to
the sum of its first n terms is 160. The inverse of the (A) a2r n-1 (B) ar n (C) a2r n (D) a2r 2n
common ratio is a positive integer. Then a possible
value of the common ratio is 26. In a GP of alternately positive and negative terms,
(A) -1/3 (B) 1/3 (C) 1/2 (D) -1/2 any term is the AM of the next two terms. Then the
common ratio (¹ -1) is
18. Suppose that a, b, c are in GP and a x = b y = c z. Then (A) -1/3 (B) -3 (C) -2 (D) -1/2
(A) x, y, z are in GP (B) x, y, z are in AP
(C) x, y, z are in HP (D) xy, yz, zx are in HP 27. If x, y and z are positive real numbers, then
x y z
19. The distances passed over by a pendulum bob in + +
y z x
successive swings are 16, 12, 9, 6.75, … . Then the
total distance traversed by the bob before it comes belongs to the interval
to rest is
(A) [2, +¥) (B) [3, +¥)
(A) 60 (B) 64 (C) 65 (D) 67
(C) (3, +¥) (D) (-¥, 3)
20. x1, x2, x3, … is an infinite sequence of positive inte-
gers in the ascending order are in GP such that 28. a, b, c be positive numbers in AP. Let A1 and G1 be
x1×x2×x3×x4 = 64. Then x5 is equal to AM and GM, respectively, between a and b, while
A2 and G2 are AM and GM, respectively, between b
(A) 4 (B) 64 (C) 128 (D) 16 and c. Then
21. If sn = 1 + 2 + 3 + + n and Sn = 13 + 23 + 33 + + n3, (A) A12 + A22 = G12 + G22 (B) A1 A2 = G1 G2
then (C) A12 - A22 = G12 - G22 (D) A1 G2 = A2 G1
(A) Sn = 2sn (B) Sn = s2n
29. Let a, b, c be positive and
(C) 2Sn = 3s2n (D) 2Sn = s2n
P = a2 b + ab2 - ac2 - a2 c
22. If x = 1 + 3a + 6a2 + 10a3 + ¥ and y = 1 + 4b +
10b2 + 20b3 + ¥ where -1 < a, b < 1, then 1 + 3ab + Q = b2 c + bc2 - a2 b - ab2
5(ab)2 + + ¥ is (1 + ab)/(1 - ab)2 , where and R = c2 a + ca2 - b2 c - bc2
If the quadratic equation Px2 + Qx + R = 0 has equal
x1/ 3 - 1 y1/ 4 - 1 roots, then a, b and c are in
(A) a = , b =
x1/ 3 y1/ 4 (A) AP (B) GP (C) HP (D) AGP
x1/ 3 + 1 y1/ 3 + 1
(B) a = 1/ 3
, b = 1/ 3 30. If a1 , a2 , … , an are in HP and
x y
x1/ 3
+1 1/ 4
y +1 a1a2 + a2a3 + a3a4 + + an–1 an = Ka1an
(C) a = 1/ 3
,b= 1/ 4
x y then K is equal to
( xy) - 1
1 / 12
(A) n (B) n – 1 (C) n + 1 (D) n + 2
(D) a =
( xy)1/ 12
www.jeeneetbooks.in
268 Chapter 5 Progressions, Sequences and Series

31. If H1 , H2 , … , Hn are n HMs between a and b, then n2


(C) [3n2 - 14 n + 110]
H1 + a Hn + b 12
+ is equal to
H1 - a Hn - b n
(D) [3n3 + 14 n2 - 3n + 100]
(A) n/2 (B) n (C) 3n (D) 2n 12
39. Sum to n terms of the series 6 + 3 + 2 + 3 + 6 + 11 +
32. If S1, S2, S3 are sums of the first n terms of three APs is
whose first terms are unity and their common differ- n n
ences are in HP, then (A) (2 n2 - 15n + 49) (B) (n2 + 15n + 49)
6 6
2S3 S1 - S1 S2 - S2 S3 n n
= (C) (2 n2 - 10 n + 49) (D) (2 n2 + 10 n + 49)
(S1 - 2S2 + S3 ) 6 6
(A) n (B) 2n (C) 2(n - 1) (D) 3n 40. Sum to n terms of the series 7 + 10 + 14 + 20 + 30 +
48 + 82 + is
33. If H1, H2, ¼, Hn are n HMs between a and b and n is a
(A) 2n - 1 + n2 + 5n (B) 2n + n2 + 5n - 1
root of the equation (1 - ab) x2 - (a2 + b2) x - (1 + ab) =
0, then H1 - Hn is equal to (C) 2n - 1 + n2 + 5n - 1 (D) 2n + n2 + 5n + 1
(A) ab(a - b) (B) ab(a + b) 41. Given that a , b , a, b are in AP; a , b , c, d are in GP
(C) ab(a + b)2 (D) a2b2(a + b) and a , b , e, f are in HP. If b, d, f are in GP, then

34. Sum to first n terms of the series b6 - a6


=
2 3 ab (b 4 - a 4 )
æ 1ö æ 1ö æ 1ö
1 + 2ç1 + ÷ø + 3 çè 1 + ÷ø + 4 çè 1 + ÷ +
è n n nø (A) 2/3 (B) 3/2 (C) 4/3 (D) 3/4
is
(A) n2 + 1 (B) (n - 1)2 (C) n2 (D) (n + 1)2 42. The sum of first 10 terms of an AP is 155, and the
sum of the first 2 terms of a GP is 9. If the first term
35. If a, b, c and d are positive such that a + b + c + d = 2, of the AP is equal to the common ratio of GP and
then M = (a + b)(c + d)satisfies the first term of the GP is equal to the common dif-
ference of AP, then the sum of the common differ-
(A) 0 < M £ 1 (B) 1 £ M £ 2
ence of AP and the common ratio of GP maybe
(C) 2 £ M £ 3 (D) 3 £ M £ 4
(A) 8 (B) 5 (C) 4 (D) 16
36. Sum to first n terms of the series 1(1!) + 2(2!) + 3(3!) + 43. The sum of an infinite GP is 2 and the sum of their
4(4!) + is cubes is 24. Then, values of the first term and the
(A) n! + 1 (B) (n + 1)! + 1 common ratio are, respectively,
(C) (n + 1)! (D) (n + 1)! - 1 (A) 3, -1/2 (B) -3, -1/2
(C) 2, -1/3 (D) -2, 1/3
37. Sum to infinity of the series
44. Let sn represent the sum of the first n terms of a
1 1
+ GP with first term a and common ratio r. Then
(1 + x)(1 + 2 x) (1 + 2 x)(1 + 3 x) s1 + s2 + s3 + + sn is equal to
1 na ar(1 - r n ) na ar(1 - r n )
+ + ¥ (A) - (B) -
(1 + 3 x)(1 + 4 x) 1- r (1 - r )2 1- r 1 - r2
where x ¹ 0 is na ar (1 - r n ) na ar (1 - r n )
(C) + (D) +
(A) 1/x (B) 1/x + 1 1- r (1 - r )2 1- r 1 - r2
(C) 1/x(1 + x) (D) 1/(x + 1)2 45. In a GP, the (m + n)th term is p and (m - n)th term
(m > n) is q. Then the mth term is
38. Sum to n terms of the series 8 + 4 + 2 + 8 + 28 + 68 + (A) pq (B) ( pq)
n/m

154 + is ( m + n )/( m - n )
n (C) pq (D) ( pq)
(A) [3n3 - 14 n2 - 3n + 100]
12 46. The sides of a right-angled triangle are in GP. If A
n and C are acute angles of the given triangle, then the
(B) [3n3 - 14 n2 - 3n + 110] values of tan A and tan C are
12
www.jeeneetbooks.in
Exercises 269

A third square is formed by joining the midpoints of


5+1 5-1 5+1 5-1
(A) , (B) , the sides of the second square and this process is con-
2 2 2 2 tinued so on. Then the sum of the areas of all these
squares is
1 3+1 3-1
(C) 5, (D) , (A) a2 2 (B) 2a2 (C) 3/2a2 (D) 4a2
5 2 2

47. The length of the side of a square is ‘a’ units. A second


square is formed by joining the midpoints of the sides.

Multiple Correct Choice Type Questions


1. Let sn and sn¢ be sums of first n terms of two AP’s with 6. If a, b are the roots of the equation x2 - 4x + g = 0
first terms a and b and common differences d and e and g, d are the roots of the equation x2 - 64x + m = 0
respectively. If and a < b < g < d are n GPs, then
sn a + (n - 1)d b + (n - 1)e (A) l = 64/25 (B) m = 47/25
= 2 and = =4 (C) l = 8/5 (D) m = 64/25
sn¢ b a
then 7. The sum of three numbers in GP is 70; if the two
d a + (n - 1)d 7 extreme terms be multiplied each with 4 and the
(A) = 26 (B) = middle by 5, the products are in AP. Then possible
e b + (n - 1)e 2
values of the common ratio are
d 2 a + (n - 1)d
(C) = (D) =2 (A) 2 (B) 3 (C) 1/2 (D) 1/3
e 7 b + (n - 1)e
8. The first term of an infinite GP is unity and any term
2. The ratio of sums to first n terms of two AP’s is (7n + 1) : is equal to the sum of all the succeeding terms. If the
(4n + 27). Then common ratio is r, then
(A) the ratio of their nth terms is (14n - 6) : (8n + 23) (A) r = 1/2
(B) the ratio of their mth terms is (14m + 6) : (8m + 23) (B) r = 1/3
(C) the ratio of their first terms is 8 : 31 (C) 1 + r + r2 + + ¥ is 2
(D) the ratio of the first terms is 20 : 31 (D) 1 + 3r + 5r2 + 7r3 + + ¥ is 6
3. If the mth term of an AP is 1/n and the nth term is 1/m, 9. If 1 + (x - 1) + (x - 1)2 + (x - 1)3 + + ¥ exists, thus
then x may lie in the interval
(A) the first term is 1/mn (A) 0 < x < 2 (B) 0 < x < 1
(B) common differences 1/mn (C) -1 < x < 0 (D) -1 < x < -1/2
(C) (mn)th term is 1
å k, 10 / 3 å k =1 k 2
n n
(D) sum to mn terms is (mn + 1)/2 10. Let n be a positive integer. If k =1
and å k =1 k 3 are in GP, then
n

4. Three numbers form an AP. The sum of the three (A) n = 4


terms is 3 and the sum of their cubes is 4. Then
(B) n = 5
(A) common difference is ± 1/ 6
(C) The sum of the given terms is 10
(B) common difference is 1/6
(D) The common ratio of the GP is 10
(C) product of the numbers is 5/6
(D) product of the numbers is 6 11. If a1, a2, a3, … are in GP such that
a4 : a6 = 1 : 4 and a2 + a5 = 216
5. Let 1, a1, a2, a3, a4, a5 and 0.3 be in AP. Then
then
(A) the common difference is 7/60 (A) common ratio is ± 2
(B) a3 = 13/20 (B) a1 = 12 or 108/7
(C) a1 + a2 + a3 + a4 + a5 = 0.65 (C) sum of the first five terms is 200
(D) a1 = 53/60 (D) common ratio is ± 1/2 and a1 = 6 or 7/108
www.jeeneetbooks.in
270 Chapter 5 Progressions, Sequences and Series

12. For 0 < x < p /2, if sin x, 2 (sin x + 1) and 6(sin x + 1) 1 1 1 5


+ + =
are in GP, then x y z 3
(A) common ratio is 1/2 (B) first term is 1/2 In such case
(C) common ratio is 3 2 (D) fifth term is 162 (A) a = 1, b = 9 (B) a = 2, b = 3
(C) a = 9, b = 1 (D) a = 3, b = 2
13. For 0 < q < p/2, let
¥ ¥ ¥
19. If a, b, c are in AP and a2, b2, c2 are in HP, then which
x = å cos2 nq , y = å sin2 nq and z = å cos2 nq sin2 nq
n=0 n=0 n=0
of the following is true?
(A) a = b = c (B) -a/2, b, c are in GP
then
(C) a, b, -c/2 are in GP (D) a/2, b, c are in HP
(A) xyz = xz + y (B) xyz = xy + y
(C) xyz = x + y + z (D) xyz = yz + x 20. Assume d is the GM between ca and ab, e is the GM
between ab and bc and f is the GM between bc and
14. x, y, z are greater than 1 and are in GP. Let
ca. If a, b, c are in AP, then
1 1 1 (A) d2, e2, f 2 are in AP
a= ,b= and c =
1 + log x 1 + log y 1 + log z (B) d2, e2, f 2 are in GP
Then (C) e + f, f + d, d + e are in GP
(A) (1 - a)/a, (1 - b)/b, (1 - c)/c are in AP (D) e + f, f + d, d + e are in HP
(B) a, b, c are in GP
(C) 1/a, 1/b, 1/c are in AP 21. If a, b, c are in HP; b, c, d are in GP and c, d, e are in
AP, then
(D) b = (2ac)/(a + c)
(A) a, c, e are in GP (B) a, d, e are in GP
15. Let a, x, b be in AP; a, y, b in GP and a, z, b in HP (C) b, c, e are in GP (D) e = (ab2)/(2a - b)2
where a and b are distinct positive real numbers. If
x = y + 2 and a = 5z, then 22. If a, b, c and d are distinct positive real numbers and
(A) y2 = zx (B) x > y > z are in HP, then
(C) a = 9, b = 1 (D) a = 1/4, b - 9/4 (A) ad < bc (B) ad > bd
(C) (a + d) > (b + c) (D) (a + d) < (b + c)
16. Let a, b, c, be three real numbers. Then
(A) a, b, c are in AP if (a - b)/(b - c) = 1 23. Let sn and s¥ be, respectively, sum to n terms and sum
(B) a, b, c are in GP if (a - b)/(b - c) = a/b to infinity of the series
(C) a, b, c are in HP if (a - b)/(b - c) = a/c 1× 2 2 × 22 3 × 23 4 × 24
(D) a, b, c are in HP if (a - b)/(b - c) = c/a + + + +
3! 4! 5! 6!
17. Let a1, a2, a3 and a4 be four positive real numbers. Then
Then (A) the nth term is n × 2n/(n + 2)!
(A) a2a3 - a1a4 > 0 if a1, a2, a3, a4 are in AP (B) sn = 1 - 2n+1/(n + 2)!
(B) a2a3 - a1a4 = 0 if a1, a2, a3, a4 are in GP (C) sn = 1/2 - 2n/(n + 1)!
(C) a2a3 - a1a4 < 0 if a1, a2, a3, a4 are in HP (D) s¥ = 1
(D) a1, a2, a3 and a4 are positive numbers not all equal,
then 24. Let sn be the sum to n terms of the series
2 3
1 3 æ 1ö 4 æ 1ö 5 æ 1ö
(a1 + a2 + a3 + a4 ) > (a1 a2 a3 a4 )1/ 4 çè ÷ø + çè ÷ø + ç ÷ +
4 1× 2 2 2×3 2 3× 4 è 2 ø
4
> then
(1/ a1 ) + (1/ a2 ) + (1/ a3 ) + (1/ a4 )
(A) sn = 1 - 1/(n + 1)2n (B) sn = 1/2 - 1/n· 2n
18. If a, x, y, z, b are in AP, then the value of x + y + z is 1
(C) lim sn = (D) lim sn = 1
15, when a, x, y, z, b are in HP, then n®¥ 2 n®¥
www.jeeneetbooks.in
Exercises 271

Matrix-Match Type Questions


In each of the following questions, statements are given in 3. Match the items of Column I with those of Column II.
two columns, which have to be matched. The statements in
Column I are labeled as (A), (B), (C) and (D), while those Column I Column II
in Column II are labeled as (p), (q), (r), (s) and (t). Any
given statement in Column I can have correct matching (A) If 1/a(b + c), 1/b(c + a), 1/c(a + b) are
in HP, then a, b and c are in (p) AP
with one or more statements in Column II. The appropriate
bubbles corresponding to the answers to these questions (B) If b + c, c + a, a + b are in HP, then (q) GP
have to be darkened as illustrated in the following example. a / (b + c), b/(c + a), c/(a + b) are in
Example: If the correct matches are (A) ® (p), (s); (C) If a, b, c are in HP, then (1/a) + (1/bc), (r) HP
(B) ® (q), (s), (t); (C) ® (r); (D) ® (r), (t); that is if the (1/b) + (1/ca), (1/c) + (1/ab) are in
matches are (A) ® (p) and (s); (B) ® (q), (s) and (t); (D) If a, b, c are in AP, then (bc)/a(b + c), (s) Not in
(C) ® (r); and (D) ® (r), (t); then the correct darkening (ca)/b(c + a), (ab)/c(a + b) AP/GP/
of bubbles will look as follows: HP

p q r s t
4. In Column I some series are given and in Column II
A
their nth terms are given. Match them.
B
C Column I Column II
D
(A) 3/4 + 5/36 + 7/144 + 9/400 (p) 3n-1 + n
+
1. Match the items in Column I with those in Column II. (B) 2 + 5 + 12 + 31 + 86 + (q) 1/6(n3 + 6n2 +
249 + 11n + 6)
Column I Column II
(C) 4 + 10 + 20 + 35 + 56 + 84 + (r) 1/2(n2 - n + 2)
(A) The sum of all integers between 250 (p) 3050 120 +
and 1000 which are divisible by 3 is
(B) The sum of all odd numbers (D) 1 + 2 + 4 + 7 + 11 + (s) (2n + 1)/n2(n + 1)2
(q) 156375
between 1 and 1000 that are
divisible by 3 is 5. Match the items of Column I to those of Column II.
(r) 3550
(C) The sum of all integers
from 1 to 100 which are divisible by Column I Column II
exactly one of 2 and 5 is (s) 83667
(D) If 7100 AMs are inserted between (A) If a, b, c are positive real numbers (p) 8
sin2 q and cos2 q, then their sum is such that sum of any two is greater
(t) 83666
than the third, then
a
2. Match the items of Column I with those of Column II.
å b+ c- a
Column I Column II is greater than or equal to
(B) If a, b, c are positive and a + b + c = 1, (q) 2
(A) æ 1 1 1 1 ö (p) 7
100 ç + + + + ÷ = then the minimum value of (1 + a)
è 1× 2 2 × 3 3 × 4 99 × 100 ø
(1 + b) (1 + c)/(1 - a) (1 - b) (1 - c) =
(B) If x is the AM between two real (q) 9 (C) If a, b, c are positive reals then
numbers a and b, y = a2 / 3 × b1/ 3 and the minimum value of (a + b + c)
z = a1/ 3 × b2 / 3 , then (1/a) + (1/b) + (1/c) is K2 where K is (r) 3
(r) 99
y3 + z3/xyz = (D) P is a point interior to the DABC.
(C) If 198 AMs are inserted between 1/4 The lines AP, BP and CP meet
and 3/4, then the sum of these AM’s is (s) 100 the opposite sides in D, E and F,
(D) If n is a positive integer such that respectively. Then, the minimum
n, [n(n - 1)]/ 2 and [n(n - 1)(n - 2)]/ 6 value of
are in AP, then the value of n is
(t) 2 AP/PD + BP/PE + CP/PF = (s) 6
www.jeeneetbooks.in
272 Chapter 5 Progressions, Sequences and Series

Comprehension-Type Questions
1. Passage: The terms 1, log xy , logzy and - 15 logzx are in AP. (iii) Which of the following is a correct statement?
Based on this information, answer the following three (A) Q1, Q2, Q3, … are in AP with common
questions. difference 5
(i) The common difference of the AP is (B) Q1, Q2, Q3, … are in AP with common
(A) 2 (B) -2 (C) 1/2 (D) -1/2 difference 6
(ii) The value of xy is (C) Q1, Q2, Q3, … are in AP with common
difference 11
(A) 1 (B) -1 (C) z2 (D) z3
(D) Q1 = Q2 = Q3 =
(iii) yz is equal to
(A) x (B) x2 (C) z-2 (D) z-3 4. Passage: Let A1, G1, H1 denote the AM, GM, and HM,
respectively, of two distinct positive reals. For n ³ 2, let
2. Passage: a1, a2, a3, …, an, … are in AP with common dif- An-1 and Hn-1 have AM, GM and HM as An, Gn and Hn,
ference d. Further sin (A - B) = sin A cos B - cos A sin B. respectively. Answer the following questions:
Based on its information, answer the following questions.
(i) Which one of the following statement is correct?
(i) sec a1 sec a2 + sec a2 sec a3 + + sec an-1 sec an is equal
to (A) G1 > G2 > G3 >
(A) tan (an+1) - tan a1/sin d (B) G1 < G2 < G3 <
(B) cot (an+1) - cot a1/sin d (C) G1 = G2 = G3 =
(C) tan (an+1) + tan a1/sin d (D) G1 < G3 < G5 < and G2 > G4 > G6 >
(D) cot (an+1) + cot a1/sin d (ii) Which of the following statements is correct?
(ii) coseca1 coseca2 + coseca2 coseca3 + + cosecan-1 (A) A1 < A2 < A3 <
cosecan is (B) A1 > A2 > A3 >
(A) cot (an+1) + cot a1/sin d (C) A1 > A3 > A5 >
(B) cot (an+1) - cot a1/sin d (D) A1 < A3 < A5 < and A2 > A4 > A6 >
(C) cot a1 - cot (an+1)/sin d (iii) Which of the following statements is correct?
(D) tan (an+1) - cot an/sin d (A) H1 > H2 > H3 >
(iii) If a1 = 0, then (B) H1 > H3 > H5 > and H2 < H4 < H6 <
æ a3 a4 a5 an ö æ 1 1 1 ö (C) H1 < H2 < H3 <
çè a + a + a + + a ÷ø - a2 çè a + a + + a ÷ø = (D) H1 < H3 < H5 < and H2 > H4 > H6 >
2 3 4 n-1 2 3 n- 2

(A) an-1/a2 + a2/an-1 (B) an/a2 + a2/an 5. Passage: Let a and b distinct positive real numbers and
(C) a2/an-1 - an-1/a2 (D) a2/an - an/ a2 a+b 2ab
A= , G = ab and H =
2 a+b
3. Passage: Let vr denote the sum of the first r terms of
an AP whose first term is r and the common differ- Answer the following questions:
ence (2r - 1). Let Tr = vr+1 - vr - 2 and Qr = Tr+1 - Tr for (i) If a, b are roots of the equation x2 - lx + m = 0,
r = 1, 2, 3, … . Then answer the following questions: then
(i) The sum v1 + v2 + + vn is equal to (A) l = 2A, m = G2 (B) l = A, m = G
(A) 1/12n(n + 1)(3n2 - n + 1) (C) l = -2A, m = G 2
(D) l = -A, m = G
(B) 1/12n(n + 1)(3n2 + n + 2)
(ii) If a(b - c)x2 + b(c - a)x + c(a - b) = 0 has equal
(C) n/2(2n2 - n + 1) roots, then a, b, c are in
(D) 1/3(2n3 - 2n + 3) (A) AP (B) GP
(ii) Tr is always (C) HP (D) Not in AP/GP/HP
(A) an odd number (iii) A relation between A, G, H is
(B) an even number (A) 2H = A + G (B) 2G = A + H
(C) a prime number (C) G2 = A + H (D) G2 = AH
(D) a composite number
www.jeeneetbooks.in
Exercises 273

Assertion–Reasoning Type Questions


In each of the following, two statements, I and II, are a1 a2 a3 a
, , , …, n , …
given and one of the following four alternatives has to K K K K
be chosen.
are also in AP.
(A) Both I and II are correct and II is a correct reasoning
for I. 6. Statement I: In DABC, if
(B) Both I and II are correct but II is not a correct tan A tan B + tan B tan C+ tan C tan A = 9
reasoning for I.
then the triangle is equilateral.
(C) I is true, but II is not true.
(D) I is not true, but II is true. Statement II: The arithmetic mean of finite set of
positive real numbers is greater than or equal to
1. Statement I: x, y, z are positive and each is different their geometric (equality occurs if and only if all the
from 1. If 2x4 = y4 + z4, xyz = 8 and logyx, logzy and numbers are equal).
logxz and log y x, log z y and log x z are in GP, then x =
7. Statement I: One can eliminate some terms of an
y = z = 2.
AP of positive integers in such a way that the remain-
Statement II: If a, b are positive and each is different ing terms form a GP.
from 1, then
Statement II: If “a” is the first term and d be the
log a common difference where both a and d are posi-
logab = tive integer of an AP, then a + ad, a + (2a + ad)d, …,
log b
a(1 + d)n, n ³ 1 belong to the AP.
2. Statement I:
8. Statement I: The sum to n terms of the series
1 1 1 1
+ + + + is equal
1× 2 × 3 2 × 3 × 4 3 × 4 × 5 (n - 2)(n - 1)n 3 5 7 9 6n
+ 2 + 2 + 2 + is
1 é1 1 ù 12 1 + 2 2
1 +2 +3
2 2
1 +2 +3 +4
2 2 2
n+1
to ê - ú for n ³ 3 .
2 ë 2 (n - 1)n û Statement II: The nth term of the series
Statement II: If K is a positive integer, then 1 1 1
+ + +
1× 2 2 × 3 3 × 4
1 1é 1 1 ù
= ê - ú
K (K + 1)(K + 2) 2 ë K (K + 1) (K + 1)(K + 2) û is

3. Statement I: æ1 1 ö
çè n - n + 1÷ø
æ 1ö æ 1ö æ 1 öæ 1ö æ 1 ö
çè 1 + ÷ ç1 + ÷ø çè 1 + ÷ø çè 1 + 8 ÷ø çè 1 + 2n ÷ø is equal
3ø è 9 81 3 3 9. Statement I: If d, e, f are in GP and the quadratic
3æ 1 ö equations ax2 + 2bx + c = 0 and dx2 + 2ex + f = 0 and
to ç 1 - 2n+1 ÷ dx 2 + ex + f = 0 have a common root, then d/a, e/b, f/c
2è 3 ø
are in HP.
Statement II: (a - b)(a + b) = a - b
2 2

Statement II: If a is a common root of the quadratic


4. Statement I: If the third term of a GP is 4, then the equations a1 x2 + b1 x + c1 = 0 and a2 x2 + b2 x + c2 = 0 ,
product of its first 5 terms is 44. then
Statement II: In a GP with first term a and common c1 a2 - c2 a1
a=
ratio r, the product of the first five terms is the fifth a1 b2 - a2 b1
power of the third term. -x
10. Statement I: If log2 (5.2 + 1), log4 (21 + 1) and 1 are
x

5. Statement I: If non-zero numbers a, b, c, d are in AP, in AP, then the value of x is log2 (0.4).
then the numbers abc, abd, acd, bcd are in HP. Statement II: If a, b are positive and equal to 1, then
Statement II: If a1 , a2 , a3 , …, an … are in AP and k ¹ 0, loga
then logab =
logb
www.jeeneetbooks.in
274 Chapter 5 Progressions, Sequences and Series

Integer Answer Type Questions


The answer to each of the questions in this section is a of the above equation and x1 < x2 < x3< are in AP
non-negative integer. The appropriate bubbles below the whose common difference is .
respective question numbers have to be darkened. For
5. x, y, z are real such that x + y + z = 3, x + y + z = 5
2 2 2
example, as shown in the figure, if the correct answer to
the question number Y is 246, then the bubbles under and x3 + y3 + z3 = 7. If xy + yz + zx, x3 + y3 + z3 -
Y labeled as 2, 4, 6 are to be darkened. 3xyz and x4 + y4 + z4 + K are in AP (K > 0), then the
values of K is .
X Y Z W
0 0 0 0 6. If a, b, c are positive reals, then the maximum value of
1 1 1 1 K such that (1 + a)(1 + b)(1 + c) > K (abc)4 / 7 is .
2 2 2
3 3 3 3 7. If x, y are positive real numbers and 3 x + 4 y = 5 ,
then the greatest value of 16 x2 y3 is .
4 4 4
5 5 5 5 8. If a, b, c are positive and a + b + c = 1. Then the mini-
6 6 6 mum value of
7 7 7 7
æ1 öæ1 öæ1 ö
8 8 8 8 çè - 1÷ø çè - 1÷ø çè - 1÷ø
a b c
9 9 9 9
is .
1. Sum of one hundred AM’s inserted between the
9. If x, y, z are positive real numbers such that x y z = 7,
3 2 4
numbers log10 2 and log10 5 is .
then
2. f :  ®  is a function satisfying the relation f (x + y) = 1/ 9
æ 525 ö
f (x) + f( y) for all rational numbers x and y and f (1) = 1. 2 x + 5 y + 3z ³ 9 ç K ÷
è 2 ø
If å K = 1 f (K ) = 45, then n value is
n
.
where K is equal to .
3. For positive integer n, if f (x) = (2 - x ) and g (x) =
n 1/n

f ( f (x)), then g(1), g(2), g(3), … form and AP with 10. Three HMs are inserted between 1 and 3. Then
common differences . 5[(first mean)/(third mean)] is equal to .
4. Consider the equation
11. a > b are positive real numbers and A, G are, respec-
é x ù é 2 x ù é y ù é 4 y ù 7 x 21y tively, their AM and GM. If A = 2G, then the ratio
êë 2 úû + êë 3 úû + êë 4 úû + êë 5 úû = 6 + 20 a / b = K + 4 3 where K is .

where 0 < x, y < 30 and [×] denotes the integer part of 12. The second term of an infinite GP is 2 and its sum to
a real number. (x1, y1), (x2, y2), (x3, y3) are solutions infinity is 8. The first term is .

ANSWERS
Single Correct Choice Type Questions
1. (D) 9. (C)
2. (B) 10. (D)
3. (A) 11. (D)
4. (C) 12. (B)
5. (B) 13. (A)
6. (A) 14. (B)
7. (B) 15. (A)
8. (A) 16. (D)
www.jeeneetbooks.in
Answers 275

17. (B) 33. (A)


18. (C) 34. (C)
19. (B) 35. (A)
20. (D) 36. (D)
21. (B) 37. (C)
22. (A) 38. (B)
23. (C) 39. (A)
24. (B) 40. (B)
25. (A) 41. (B)
26. (C) 42. (B)
27. (B) 43. (A)
28. (C) 44. (A)
29. (C) 45. (C)
30. (B) 46. (B)
31. (D) 47. (B)
32. (A)

Multiple Correct Choice Type Questions


1. (A), (B) 14. (A), (C), (D)
2. (A), (C) 15. (A), (B)
3. (A), (B), (C), (D) 16. (A), (B), (C)
4. (A), (C) 17. (A), (B), (C), (D)
5. (B), (D) 18. (A), (C)
6. (A), (B) 19. (A), (B), (C)
7. (A), (C) 20. (A), (D)
8. (A), (C), (D) 21. (A), (D)
9. (A), (B) 22. (B), (C)
10. (A), (D) 23. (A), (B), (D)
11. (A), (B) 24. (A), (D)
12. (B), (C), (D)
13. (B), (C)

Matrix-Match Type Questions


1. (A) ® (q), (B) ® (s), (C) ® (p), (D) ® (r) 4. (A) ® (s), (B) ® (p),
2. (A) ® (r), (B) ® (t), (C) ® (r), (D) ® (p), (t) (C) ® (q), (D) ® (r)
3. (A) ® (r), (B) ® (p), (C) ® (r), (D) ® (r) 5. (A) ® (q), (r) (B) ® (p),
(C) ® (r), (D) ® (s)

Comprehension-Type Questions
1. (i) (B) (ii) (A) (iii) (C) 4. (i) (C) (ii) (B) (iii) (C)
2. (i) (A) (ii) (C) (iii) (A) 5. (i) (A) (ii) (C) (iii) (D)
3. (i) (B) (ii) (D) (iii) (B)

Assertion–Reasoning Type Questions


1. (A) 6. (A)
2. (A) 7. (A)
3. (A) 8. (A)
4. (D) 9. (B)
5. (A) 10. (A)
www.jeeneetbooks.in
276 Chapter 5 Progressions, Sequences and Series

Integer Answer Type Questions


1. 50 7. 3
2. 9 8. 8
3. 1 9. 7
4. 6 10. 3
5. 7 11. 7
6. 7 12. 4
www.jeeneetbooks.in

Permutations and
Combinations 6
Contents
6.1 Factorial Notation
6.2 Permutations
Permutations and Combinations

6.3 Combinations

Worked-Out Problems
Summary
Exercises
Answers

Permutation: A permutation
of a set of values is an arrange-
Johann Peter Gustav Lejeune Dirichlet was a German
mathematician credited with the modern formal definition ment of those values into a
of a function. Dirichlet’s brain is preserved in the anatomical particular order. The arrange-
collection of the University of Göttingen ment can be linear or circular.
Combination: A combination
1 is selection of objects from
a set.
2

2 4

1
2 4
3
7 6 5
www.jeeneetbooks.in
278 Chapter 6 Permutations and Combinations

The concepts of permutations and combinations are important in view of several applications in day-to-day life and
in the theory of probability (Probability is covered in Vol. II). A combination is only a selection while permutation is
selection as well as arrangement.

Example

Forming a four-letter word using the letters of the word In simpler terms, whenever there is importance to
CHAPTER is a permutation, since it involves two steps, the arrangement or order in which the objects are placed,
namely selection of four letters from among C, H, A, P, T, it is a permutation and, if there is no importance to the
E and R and arrangement of these four letters. Suppose arrangement or order and only selection is required, it
we select C, H, A and T. We can arrange them to form a is a combination. One should be in a position to clearly
four-letter word such as CHAT, CAHT, TCHA, so on. see whether the concept of permutation or the concept
Forming a set with four letters is a combination which of combination is applicable in a given situation. These
involves only one step, namely selection of four letters, say concepts and methods we are going to develop in this
A, C, H, T. Then the four-element set formed is {A, C, H, T} chapter help us to determine the number of permutations
which is same as {C, H, A, T}, {C, A, H, T}, {T, C, H, A}, etc. or combinations without actually counting them.

6.1 | Factorial Notation


First, we introduce the factorial notation which is crucial in determining the number of permutations or combinations.
For any positive integer n, we define n! or n (read as n factorial or factorial n) recursively as follows:

ì1 if n = 1
n! = í
î(n - 1)!× n if n > 1

Examples

(1) 1! = 1, 2 ! = 1!× 2 = 2 (4) 5! = 4 !× 5 = 24 × 5 = 120


(2) 3! = 2 !× 3 = 2 × 3 = 6 (5) Also, for convenience, we define 0 ! = 1.
(3) 4 ! = 3!× 4 = 6 × 4 = 24

6.2 | Permutations
Before going to formal definitions and derivations we introduce the “Fundamental Principle” which plays a major role
in the theory of permutations and combinations.

FUN D A MEN TAL P R I N CI P L E If a work W1 can be performed in m different ways and another work W2 in
n different ways, then the two works can be performed simultaneously in mn
different ways.

Example 6.1

A person has to travel from Chennai to Mumbai via Solution: By the fundamental principle, there are
Hyderabad. There are four different modes of travel from 4 ´ 3 = 12 different ways of travel from Chennai to
Chennai to Hyderabad, namely, car, bus, train and aeroplane Mumbai via Hyderabad. These are
(we denote these by A1, A2, A3 and A4, respectively)
and that there are three different modes of travel from A1 B1 A1 B2 A1 B3
Hyderabad to Mumbai, namely bus, train and aeroplane A2 B1 A2 B2 A2 B3
(we denote these by B1, B2 and B3, respectively). Then how
many different modes of travel are available to that person A3 B1 A3 B2 A3 B3
to travel from Chennai to Mumbai via Hyderabad? A4 B1 A4 B2 A4 B3
www.jeeneetbooks.in
6.2 Permutations 279

Here, A1 B1 means travelling from Chennai to A4 B2 means travelling from Chennai to Hyderabad by
Hyderabad by car and from Hyderabad to Mumbai by bus; aeroplane and from Hyderabad to Mumbai by train, etc.

The following is an abstraction of the fundamental principle.

T H E O R E M 6 .1 If A is a set with m elements and B is a set with n elements, then A ´ B is a set with mn elements.

PROOF Let A = {a1, a2 , …, am } and B = {b1, b2 , …, bn }. Then


A ´ B = {(ai , bj ) | 1 £ i £ m and 1 £ j £ n}
For each ai Î A, there are n number of pairs whose first coordinate is ai. These are
(ai , b1 ), (ai , b2 ), …, (ai , bn )
The number of ai s is m. Therefore, the total number of elements in A ´ B is
n + n + + n (m times) = mn ■

DEF IN IT ION 6 . 1 For any finite set X, a bijection of X onto itself is called a permutation of X.

In other words, suppose X has n elements, say


X = {x1, x2 , … , xn }
and suppose we have to keep elements of X in n different boxes, one in each box, as shown.

Xn Xn – 1 Xn – 2 X1 (n boxes)

An arrangement of this type is called a permutation. In the following, all permutations of a three-element set {a1, a2 , a3 }
are given.

a1 a2 a3

a1 a3 a2

a2 a3 a1

a2 a1 a3

a3 a1 a2

a3 a2 a1

There are six permutations of a three-element set. If the number of elements of a given set X is large, it is not easy,
as above, to enumerate the permutations of X. In the following, we develop a formula to find the number of such
permutations.

Linear Permutations
In this section we would discuss linear permutations (i.e., arrangements of given objects in a line) with or without
repetitions.

T H E O R E M 6 .2 The number of permutations of an n-element set, taken all at a time, is n!

PROOF We will use induction on n.


If n = 1, then clearly there is only one permutation of a one-element set and 1! = 1.
www.jeeneetbooks.in
280 Chapter 6 Permutations and Combinations

Let n > 1 and suppose that the number of permutations of any (n - 1) -element set is (n - 1)!.
Let X be an n-element set, say X = {x1, x2 , …, xn }. Note that a permutation is a way of filling n
blanks using the n elements of X with one element in each blank. Consider n blanks as given below:

1 2 3 n –1 n

To fill the first blank, we can use any of the n elements x1, x2 , … , xn in X. After filling the first
blank, we are left with n – 1 elements of X and these are to be used to fill up the remaining n – 1
blanks. By induction hypothesis, the number of such permutations (filling the n – 1 blanks with
n – 1 elements) is (n - 1)!.

Now, we have two works W1 and W2. Work W1 is filling up the first blank and work W2 is fill-
ing up the remaining (n - 1) blanks. There are n ways of doing work W1 and (n - 1)! ways of doing
work W2. Therefore, by the fundamental principle, the number of ways of doing works W1 and W2
simultaneously is n × (n - 1)! = n !. Thus there are n! number of permutations of X. ■

Try it out Consider a five-element set. Choose any three elements and arrange them in three blanks. How
many such arrangements can be made?

In the following we desire a formula for such a situation which generalizes the above theory.

T H E O R E M 6 .3 Let n and r be positive integers and r £ n. Then the number of permutations of n (dissimilar)
objects taken r at a time is equal to
r -1
n(n - 1)(n - 2) (n - r + 1) = Õ (n - s)
s=0

PROOF Note that the number of required permutations is equal to the number of ways of filling r blanks
using the given n objects with one object in each blank. Again, we will use induction on n.
If n = 1, then r = 1 and the theorem is trivial.
Let n > 1 and assume the theorem is true for n – 1; that is, for any k, 1 £ k £ n - 1, the number
of permutations of n – 1 dissimilar objects taken k at a time is
k -1

Õ [(n - 1) - s] = (n - 1)(n - 2) [(n - 1) - (k - 1)]


s=0

= (n - 1)(n - 2) (n - k )
Diagrammatically it can be represented as follows:

1 2 3 r –1 r

To fill the first blank, we can use any one of the given n objects. Therefore, the first blank can
be filled in n different ways. After filling up the first blank, we are left with (n - 1) objects and the
left over (r - 1) blanks can be filled up with these (n - 1) objects. The number of different ways of
filling the (r - 1) blanks using (n - 1) objects is
( r - 1) - 1

Õ
s=0
[(n - 1) - s] = (n - 1)(n - 2) (n - r + 1)

Thus, by the fundamental principle, the number of ways of filling up the r places using n dissimilar
objects is
n × (n - 1)(n - 2) (n - r + 1) ■
www.jeeneetbooks.in
6.2 Permutations 281

DEF IN IT ION 6 . 2 The number of permutations of n dissimilar things taken r at a time is denoted by n Pr or
P(n, r). However, n Pr is more familiar and so we use this notation only. Therefore
n
Pr = n(n - 1)(n - 2) (n - r + 1)

Note: As a convention, we define n P0 = 1.

T H E O R E M 6 .4 The following hold good for any positive integers n and r such that r £ n.
n!
1. n Pr =
(n - r )!
2. n Pr = n × ( n - 1) P( r - 1)
3. n Pr = ( n - 1) Pr + r × ( n - 1) P( r - 1)

PROOF 1. We have
n
Pr = n(n - 1)(n - 2) (n - r + 1)
n(n - 1) (n - r + 1)(n - r )(n - r - 1) 3 × 2 × 1
=
(n - r )(n - r - 1) 3 × 2 × 1
n!
=
(n - r )!
2. We have
n! n × (n - 1)!
n
Pr = = = n × ( n - 1) P( r - 1)
(n - r )! [(n - 1) - (r - 1)]!
3. We have
( n - 1) (n - 1)! (n - 1)! r
Pr + r × ( n - 1) Pr - 1 = +
(n - 1 - r )! [(n - 1) - (r - 1)]!
(n - 1)! (n - 1)!
= + r
(n - r - 1)! (n - r )!
(n - 1)! æ r ö
= çè 1 + ÷
(n - r - 1)! n - rø
(n - 1)! n - r + r
=
(n - r - 1)! (n - r )
n!
= = n Pr
(n - r )! ■

Example 6.2

Find the number of permutations of four dissimilar things taken three at a time.

Solution: The number of permutations is


4! 4!
4
P3 = = = 24
(4 - 3)! 1

Example 6.3

Find the number of four-letter words that can be (i) each word begins with letter T
formed using the letters of the word CHEMISTRY (ii) each word ends with letter Y
such that (iii) each word begins with letter T and ends with letter Y
www.jeeneetbooks.in
282 Chapter 6 Permutations and Combinations

Solution: There are nine letters in the word (ii) A four-letter word is to end with Y means, we
CHEMISTRY. The total number of four-letter words can choose the first three letters from among
using these nine letters is the remaining 9 - 1 = 8 letters. Therefore, the
number of four-letter words each ending with Y is
9! 8
P3 = 336.
9
P4 = = 9 × 8 × 7 × 6 = 3024
(9 - 4)! (iii) If a four-letter word is to begin with T and end with
(i) If a four-letter word is to begin with T, the next three Y then the middle two letters can be chosen from
letters in the word can be chosen from the remaining among the remaining 9 - 2 = 7 s. Therefore, the
9 - 1 = 8 letters. Therefore, the number of four-letter number of such words of is
words each beginning with T is 7!
7
P2 = = 42
8! (7 - 2)!
8
P3 = = 8 × 7 × 6 = 336
(8 - 3)!

Example 6.4

Find the number of ways of arranging 5 boys and 4 girls B B


in a line so that there will be a boy in the beginning and
at the ending.
All the 7 places in the middle are to be filled up by
7 (9 – 2) persons (3 boys and 4 girls). The number of ways
Solution: There are 9 (5 boys + 4 girls) persons alto- of doing this is 7!. Therefore, the total number of the
gether. The first place and the last place are to be filled required arrangements is 20 ´ 7 ! = 100800.
up by two boys from among 5 boys. The number of ways
of doing is 5 P2 = 20.

Example 6.5

Find the number of four-letter words that can be formed to be filled using the remaining 6 letters. This can be
using the letters of the word FRIENDS which contain done in 6P3 ways. Similarly, the number of 4-letter
the letter S and those which do not contain S. words with S in the second place is 6P3 and so are the
numbers of words with S in each of third and fourth
Solution: There are 7 letters in the word FRIENDS. places. Therefore, the total number of 4-letter words
The total number of 4-letter words using these 7 letters containing S that can be formed with the letters in the
word FRIENDS is
is 7 P4 = 840.
4 ´ 6 P3 = 4 ´ 120 = 480
The number of words not containing S is 840 - 480 =
360. [Note that this is same as the number of 4-letter
Consider the four blanks given above. If the first words using the 6 letters (other than S); that is 6 P4 .]
blank is filled with S, then the remaining 3 blanks are

Example 6.6

Find the number of ways of arranging the letters of the Then we have 5 consonants +1 unit of vowels = 6
word KRISHNA such that all the vowels come together. objects. These can be arranged in 6! ways.
The vowels can be permuted among themselves in
Solution: The number of letters in KRISHNA is 7 and 2! ways.
among them there are two vowels, I and A. The vowels Therefore, the total number of arrangements in
coming together means we have to treat the two vowels which the two vowels come together is 6 ! ´ 2 ! = 1440.
as one single unit.

D EFINIT ION 6 . 3 If the words in a given set of words are arranged in the alphabetical order (as in a dictionary) and
if a particular word is in the nth place in the list, then n is called the rank of that word.
www.jeeneetbooks.in
6.2 Permutations 283

Example 6.7

If the letters of the word PRISON are permuted in all O is 5! = 120


possible ways and the words thus formed are arranged in P is 5! = 120
dictionary order, then find the rank of the word SIPRON. R is 5! = 120
SIN is 3! = 6
SIO is 3! = 6
Solution: The sequence of the letters of the word
SIPN is 2! = 2
PRISON in alphabetical order is INOPRS. In dictionary SIPO is 2! = 2
the words starting with I come first and the number of SIPRN is 1! = 1
these is 5! = 120. Next come the words starting with N SIPRON is 0! = 1
and so on. The number of words starting with
Therefore, the rank of SIPRON is
I is 5! = 120
N is 5! = 120 (5 ´ 120) + 6 + 6 + 2 + 2 + 1 + 1 = 618

Now, we derive a formula for the number of permutations of n dissimilar things taken r at a time when each thing
can be repeated any number of times. First, we have below a natural generalization of the fundamental principle.

T H E O R E M 6 .5 Let A1, A2 , … , Ar be finite sets with n1, n2 , … , nr elements, respectively. Then the number of
elements in A1 ´ A2 ´ ´ Ar is the product n1 n2 nr.
PROOF We will use the fundamental principle and apply induction on r. If r = 1, the theorem is clean.
Suppose that r > 1 and assume the theorem for r – 1. That is, the number of elements in A1 ´ A2 ´ ´
Ar-1 is the product n1 n2 nr - 1 . Since A1 ´ A2 ´ ´ Ar-1 ´ Ar and (A1 ´ A2 ´ ´ Ar-1) ´ Ar are
bijective and are finite sets, they have the same number of elements. By the fundamental principle
and the induction hypothesis, the number of elements in ( A1 ´ A2 ´ ´ Ar - 1 ) ´ Ar is (n1 n2 nr - 1 )nr
and hence the number of elements in A1 ´ A2 ´ ´ Ar - 1 ´ Ar is n1 n2 nr - 1 nr. ■

C O R O L L A R Y 6 .1 Let W1, W2 , … , Wr be certain works. Suppose that Wi can be performed in ni number of ways. Then
the number of ways in which W1, W2 , … , Wr can simultaneously be performed is n1 n2 nr.

C O R O L L A R Y 6 .2 Let n and r be positive integers such that r £ n. Then the number of permutations of n dissimilar
things taken r at a time, when repetition of things is allowed any number of times, is nr.
PROOF The number of required permutations is equal to the number of ways of filling up r blanks using
the given n dissimilar things. If Wi is the work of filling the i th blank using the n things, then Wi
can be performed in n number of ways. Therefore, W1, W2 , … , Wr can be performed simultane-
ously in n × n n(r times) = nr.

1 2 3 r –1 r ■

C O R O L L A R Y 6 .3 The number of permutations of n dissimilar things taken r at a time, with atleast one repetition, is nr - nPr.
PROOF The total number of permutations of n dissimilar things taken r at a time, without repetitions, is n Pr.
Therefore, the number of required permutations, with atleast one repetition, is nr - n Pr. ■

Example 6.8

A number lock has four rings and each ring has 10 digits, Solution: Each ring can be rotated in 10 different ways
0, 1, 2, … , 9. Find the maximum number of unsuccessful to get a digit on the top. Therefore, the total number of
attempts that can be made by a thief who tries to open ways in which the four rings can be rotated is 104. Out of
the lock without knowing the key code. these, only one is a successful attempt and all the others
are unsuccessful. Therefore, the maximum number of
unsuccessful attempts is 10 4 - 1 = 9999.
www.jeeneetbooks.in
284 Chapter 6 Permutations and Combinations

Circular Permutations
We now turn our attention on the arrangements of the given objects around a circle, that is, circular permutations.
In this context, we come across two types of circular permutations. One is clockwise arrangement and the other is
anticlockwise arrangement as shown in Figure 6.1. These two are same, but for the direction. In general, the direction
is also important in circular permutations and hence we regard the two permutations shown in the figures below as
two different circular permutations.

e a b a

d b d e
c

(a) (b)
FIGURE 6.1 (a) Clockwise arrangement and (b) anticlockwise arrangement.

T H E O R E M 6 .6 The number of circular permutations of n dissimilar things taken all at a time is (n - 1)!.
PROOF Let N be the number of circular permutation of n things taken all at a time. If we take one such
permutation it looks like as in Figure 6.2.
Starting at some point and reading in either clockwise or anticlockwise direction, but not both,
we get n linear permutations from each circular permutation as shown for the one given in Figure 6.2.
a2 a3 a4 an - 1 an a1
a3 a4 a5 an a1 a2
a4 a5 a6 an a1 a2 a3
        
an a1 a2 an - 2 an - 1
a1 a2 a3 an - 1 an
Thus, each circular permutation gives rise to n linear permutations. Therefore, N circular
permutations give rise to N ´ n linear permutations. But, we know that the number of linear
permutations of n things, taken all at a time, is n!. Therefore

N ´ n = n ! = n ´ (n - 1)!
N = (n - 1)!
an a1

an –1 a2

an –2 a3

a4

FIGURE 6.2 Theorem 6.6. ■


www.jeeneetbooks.in
6.2 Permutations 285

QUICK LOOK 1

Suppose we are to prepare a garland using n given should treat them as identical. Therefore, in such cases,
flowers or a chain using n beeds. Any hanging type the number of circular permutations of n things is half
circular permutation looks like clockwise arrangement of the actual number of circular permutations; that is,
from one side and anticlockwise arrangement from the 1
opposite side (in the same order of things). Hence, we (n - 1)!
2

Example 6.9

Find the number of ways of arranging 5 boys and 8 girls Solution: Total number of persons = 5 + 8 = 13. The
around a circular table. total number of circular permutations is
(13 - 1)! = (12)!

Example 6.10

Suppose we are given 7 red roses and 4 yellow roses (no arranged in these 7 gaps in 7P4 ways. Therefore, the total
two of these are identical). Find the number of different number of circular permutations is
ways of preparing a garland using all the given roses such
6 ! ´ 7P4
that no two yellow roses come together.
But, in the case of garlands, clockwise and anti-
Solution: First arrange the 7 red roses in a circular clockwise arrangements look alike. Thus, the number of
possible distinct garlands is
form in (7 - 1)! = 6 ! ways.
Now, imagine a gap between two successive red 1
(6 ! ´ 7 P4 )
roses. There are 7 such gaps and 4 yellow roses can be 2

Next we consider permutations of things in which some are alike and the rest are different. For example, we may
want to find the number of ways of permuting the letters of the word MATHEMATICS, in which there are 2 Ms, 2 As,
2 Ts and the rest are different. We derive formulae that can be used in such cases.

T H E O R E M 6 .7 The number of linear permutations of n things, in which p things are alike and the rest are different,
is n !/ p!.
PROOF Suppose that we are given n things in which p are alike and the remaining are different. Let N be
the number of permutations of these n things.
When we take one such permutation, it contains p like things. If we replace these p like things
by p dissimilar things, then we can arrange these p things among themselves (without disturbing the
relative positions of other things) in p! ways. Therefore, each permutation when p things are alike
gives rise to p! permutations when all are different. Therefore, from the N such permutations we get
N ´ p! permutations. But we know that the number of permutations of n different things is n!. Thus
N ´ p! = n !
n!
N=
p! ■
We can extend, using induction, the above theorem for the case of having more than one set of like things in the
given n things, by using Theorem 6.7 repeatedly.

C O R O L L A R Y 6.4 The number of linear permutations of n things, in which there are p alike things of one kind, q
alike things of second kind and r alike things of third kind and the rest are different, is
n!
p! q ! r !
www.jeeneetbooks.in
286 Chapter 6 Permutations and Combinations

Example 6.11

Find the number of ways of arranging the letters of the (C, T, N) are different. Therefore, the required number of
word ASSOCIATIONS. permutations is
(12)! (12)!
Solution: There are 12 letters in the given word, among =
which there are 2 As, 3 Ss, 2 Os, 2 Is and the others 2 ! ´ 3! ´ 2 ! ´ 2 ! (2 !)3 ´ (3!)

6.3 | Combinations
A combination is only a selection. There is no importance, as in the case of a permutation, to the order or arrangement
of things in a combination. Thus, a combination of n things taken r at a time can be regarded as a subset with r elements
of a set containing n elements.

QUICK LOOK 2

The number of combination of n dissimilar things taken r at a time is denoted by

n æ nö
Cr or çè r ÷ø or C (n, r)
n
Note: C r is precisely the number of r-element subsets of an n-element set.

In the following theorem, we derive the formula for n C r.

T H E O R E M 6 .8 The combination of n dissimilar things taken r at a time is given by


n
Pr n! n(n - 1)(n - 2) (n - r + 1)
n
Cr = = =
r ! (n - r )! r ! 1 × 2 × 3 (r - 1)r

That is, the number of combinations of n dissimilar things taken r at a time is


n!
(n - r )! r !

PROOF Any combination of r elements from among n dissimilar things can be treated as an r-element
subset of an n-element set. Let us select one such combination of r elements and these r elements
can be arranged in a line in r! ways. Therefore, each combination of r elements gives rise to r!
number of permutations of r elements. So, the total number of permutations of n dissimilar things,
taken r at a time, is equal to n C r ´ r !. Therefore
n
Pr = n C r ´ r !

Thus
n
Pr n!
n
Cr = =
r ! (n - r )! r !
n(n - 1) (n - r + 1)(n - r ) 3 × 2 × 1
=
(n - r )(n - r - 1) 3 × 2 × 1 × r(r - 1) 2
n(n - 1) (n - r + 1)
=
1 × 2 × 3 (r - 1)r ■
www.jeeneetbooks.in
6.3 Combinations 287

C O R O L L A R Y 6.5 The number of r-element subsets of an n-element set is


n!
n
Cr =
(n - r )! r !
PROOF The r-element subsets of an n-element set are precisely combinations of n dissimilar things taken
r at a time. ■

Examples

(1) The number of subsets with exactly 4 elements in a (2) The number of ways of constituting a committee of
set of 6 elements is 5 members from a group of 20 persons is
6! 6×5 20 ! 20 × 19 × 18 × 17 × 16
= = 15 = = 15504
(6 - 4)! 4 ! 1 × 2 (20 - 5)! 5! 1× 2 × 3 × 4 × 5

When we select r elements from n elements, we will be left with n – r elements. Therefore, the number of ways of
selecting r elements from the given n elements is equal to the number of ways of leaving n – r elements. This is formally
proved in the following theorem.

T H E O R E M 6 .9 For any positive integers n and r with r £ n,


n
Cr = nC n-r
PROOF We have
n!
n
Cr =
(n - r )! r !
n!
=
r ! (n - r )!
n!
= = nC n-r
[n - (n - r )]! (n - r )! ■

C O R O L L A R Y 6.6 For any positive integer n, n C n = n C 0 = 1.

T H E O R E M 6 .10 Let m and n be distinct positive integers. Then the number of ways of dividing m + n things into
two groups containing m things and n things is
(m + n)!
m! n !
PROOF When we select m things out of the given (m + n) things, then n things will be left out. Therefore,
the required number is precisely the number of ways of selecting m things from the m + n things,
( m + n)
Cm, equals
(m + n)! (m + n)!
=
[(m + n) - m]! m! n ! m! ■

C O R O L L A R Y 6.7 Let m, n and k be distinct positive integers. Then the number of ways of dividing m + n + k things
into three groups containing m things, n things and k things is

(m + n + k )!
m! n ! k !
www.jeeneetbooks.in
288 Chapter 6 Permutations and Combinations

PROOF First we select m things from m + n + k things and then select n things from the remaining n + k
things and finally k things will be left out and they form the third group. The number of ways of
selecting m things from the given m + n + k is
(m + n + k )! (m + n + k )!
=
[(m + n + k ) - m]! m! (n + k )! n

The number of ways of selecting n things from n + k things is

(n + k )!
n! k !

By the fundamental principle, the number of ways of dividing m + n + k things into three
groups of m things, n things and k things is

(m + n + k )! (n + k )! (m + n + k )!
´ =
(n + k )! m! n! k ! m! n ! k ! ■

C O R O L L A R Y 6 .8 The number of ways of dividing 2n things into two equal groups of n things each is

(2 n)!
2! n! n!

PROOF By Theorem 6.10, we can divide 2n things into two groups in (2n !)/ n ! n ! ways. Since the groups
have equal number of elements, we can interchange them in 2! ways. They give rise to the same
division. Therefore, 2n things can be divided into two equal groups of n things each in

(2 n)!
ways
2! n! n! ■

The above can be generalized for the division of mn things into equal m groups, as given in the following.

C O R O L L A R Y 6 .9 Let m and n be positive integers. Then the number of ways of dividing mn things into m groups,
each containing n things, is
(mn)!
m! (n !)m

C O R O L L A R Y 6 .10 Let m and n be positive integers. The number of ways of distributing mn things equally among
m persons is
(mn)!
(n !)m

PROOF By Corollary 6.9, we can divide mn things into m groups, each containing n things, in

(mn)!
ways
m! (n !)m

In each such division, there are m groups, which are not identical but only contain equal
number of things. We have to distribute m groups to m persons in m! ways. Therefore, the total
number of required distributions is
(mn)! (mn)!
´ m! =
m! (n !) m
(n !)m ■
www.jeeneetbooks.in
6.3 Combinations 289

Example 6.12

Find the number of ways of selecting 11 member cricket Table 6.1 Example 6.12
team from a group of players consisting 7 batsmen,
Wicket Number of ways of
5 bowlers and 3 wicket keepers such that there must be
Bowlers keepers Batsman selecting the team
atleast 3 bowlers and 2 wicket keepers in the team.
5 3 3 5
C 5 ´ 3 C 3 ´ 7C 3 = 35
Solution: The teams can be selected with the composi-
tions given in Table 6.1. The last column of the table gives 5 2 4 5
C 5 ´ 3C 2 ´ 7C 4 = 105
the number of ways of selecting the team. 4 3 4 5
C 4 ´ 3C 3 ´ 7C 4 = 175
Therefore, the total number of ways of selecting the
4 2 5 5
C 4 ´ 3C 2 ´ 7C 5 = 315
11-member cricket team is 35 + 105 + 175 + 315 + 210 +
210 which is equal to 1050. 3 3 5 5
C 3 ´ 3C 3 ´ 7C 5 = 210
3 2 6 5
C 3 ´ 3C 2 ´ 7C 6 = 210

Example 6.13

Suppose that a set of m parallel lines intersect another Solution: To form a parallelogram, we have to select
set of n parallel lines. Then, find the number of parallelo- two lines from the first set and two lines from the second
grams formed by these lines. set. The number of such selections is m C 2 ´ n C 2.

We have proved earlier in Theorem 6.9 that, for any positive integers n and r such that r £ n, n C r = nC n - r. Converse of
this is proved in the following.

T H E O R E M 6 .11 Let r, s and n be positive integers such that r £ n and s £ n. Then n C r = n C s if and only if r = s or
r = n – s.

PROOF Suppose that n C r = n C s and r ¹ s. We can assume that r < s. Then n – s < n – r. Consider
n
Cr = nC s
n! n!
=
(n - r )! r ! (n - s)! s !
(n - r )! r ! = (n - s)! s !
(n - r )(n - r - 1) ((n - s + 1)(n - s)! r ! = (n - s)! s( s - 1)( s - 2) (r + 1)r ! (since n - s < n - r and r < s)
(n - r )(n - r - 1) (n - s + 1) = s( s - 1) (r + 1)
(a + 1)(a + 2) (a + K ) = (r + 1)(r + 2) (r + K )
where a = n – s and K = s – r. This gives a = r and hence n - s = r. [Otherwise, if a < r, then
a + i < r + i for all 1 £ i £ K and hence (a + 1)(a + 2) (a + K ) < (r + 1)(r + 2) (r + K ), since all
these are positive integers.] Thus, if n C r = n C s , then r = s or r = n – s (i.e., r + s = n). ■

T H E O R E M 6 .12 Let r and n be positive integers such that r £ n. Then,


n
C r - 1 + n C r = n + 1C r
PROOF We have
n! n!
n
Cr -1 + nCr = +
[n - (r - 1)]! (r - 1)! (n - r )! r !
n! é 1 1ù
= ê + ú
(n - r )! (r - 1)! ë n - r + 1 r û
www.jeeneetbooks.in
290 Chapter 6 Permutations and Combinations

n! é n+1 ù
=
(n - r )! (r - 1)! êë (n - r + 1)r úû
(n + 1)! n+1
= = Cr
[(n + 1) - r ]! r ! ■

C O R O L L A R Y 6 .11 For 2 £ r £ n,
n
Cr - 2 + 2 × nCr -1 + nCr = n+ 2Cr
PROOF We have
n
C r - 2 + 2 × n C r - 1 + n C r = (n C r - 2 + n C r - 1 ) + (n C r + n C r - 1 )
= n + 1C r - 1 + n + 1C r = n + 2 C r (from Theorem 6.12) ■

Example 6.14

Find the value of 25 C 4 + å r = 0 ( 29 - r ) C 3.


4
= 26 C 4 + 26 C 3 + 27 C 3 + 28 C 3 + 29 C 3

= 27 C 4 + 27 C 3 + 28 C 3 + 29 C 3
Solution: We have
4 = 28 C 4 + 28 C 3 + 29 C 3
25
C4 + å ( 29 - r )
C3 = C4 + C3 + C3 + C3 + C3 + C3
25 25 26 27 28 29

r =0 = 29 C 4 + 29 C 3 = 30 C 4

Example 6.15

If 12 C s + 1 = 12 C 2 s - 5 , then find the value of s. Þ s = 6 or 3s = 16

Solution: We take s = 6 (since s is an integer).


12
C s + 1 = 12 C 2 s - 5 Þ s + 1 = 2 s - 5 or s + 1 = 12 - (2 s - 5)

T H E O R E M 6 .13 If p alike things are of one kind, q alike things are of second kind and r alike things are of third
kind, then the number of ways of selecting any number of things (one or more) out of them is
( p + 1)(q + 1)(r + 1) - 1

PROOF From the first p things, we can select 0 or 1 or 2 or … or p things. Since all the p things are alike,
we have to decide only the number of things to be selected. This can be done in p + 1 ways.
Similarly, we can select any number of things from the second kind in q + 1 ways and from the
third kind in r + 1 ways. Hence by the fundamental principle, we can select any number of things
from the three groups in ( p + 1)(q + 1)(r + 1) ways. But this includes the selection of 0 from each
group. Since, we have to select one or more things, the number of required ways is
( p + 1)(q + 1)(r + 1) - 1 ■

C O R O L L A R Y 6 .12 If p1, p2 , … , pr are distinct primes and a1, a2 , … , ar are positive integers, then the number of posi-
tive integers that divide p1a1 p2a2 prar is (a1 + 1)(a2 + 1) (ar + 1).

PROOF Let n = p1a1 p2a2 prar. Then any positive integer that divides n must be of the form p1b1 p2b2 prbr,
where 0 £ bi £ ai for all 1 £ i £ r and bis are integers. Now, each bi can take ai + 1 values that is
0, 1, 2, … , ai . Therefore, as in Theorem 6.13, the number of positive integral divisors of n is
(a1 + 1)(a2 + 1) (ar + 1) ■
www.jeeneetbooks.in
6.3 Combinations 291

Examples

(1) If there are 6 red beads of same type, 8 blue beads (2) Any divisor of n other than 1 and n is called a proper
of same type and 10 yellow beads of same type, then divisor of n. The number of proper positive integral
the number of ways of selecting any number of beads divisors of 10800 (= 2 4 ´ 33 ´ 52 ) is
(one or more) is
(4 + 1)(3 + 1)(2 + 1) - 2 = 58
(6 + 1)(8 + 1)(10 + 1) - 1 = 693

T H E O R E M 6 .14 Let n be a positive integer. Then the number of ways in which n can be written as a sum of
(atleast two) positive integers, considering the same set of integers in a different order as being
different, is 2 n- 1 - 1.

PROOF Write n number of 1s on a line and put the symbol “(” on the left of the first 1 and the symbol “)”
on the right of the last 1, as shown below:
(1 - 1 - 1 - - 1 - 1)
Consider the n – 1 spaces between the two consecutive 1s. By filling each of these n – 1 spaces
with one of the two symbols “+” and “) + (”, we get an expression of n as a sum of positive inte-
gers and vice versa. For example,
(1 + 1) + (1 + 1 + 1) + (1 + 1) + (1 + 1 + 1)
corresponds 2 + 3 + 2 + 3 = 10 and
(1 + 1) + (1) + (1 + 1) + (1 + 1) + (1 + 1 + 1)
corresponds to 2 + 1 + 2 + 2 + 3.
The number of ways of filling n – 1 spaces each with one of the two symbols is 2 n- 1. Among
these we have to exclude one expression, namely,
(1 + 1 + 1 + + 1) = n
Since, we are interested only in sums with atleast two summands, thus, the number of required
ways in which n can be written as a sum of (atleast two) positive integers is
2 n- 1 - 1 ■

The proofs of the following two results are similar to that of Theorem 6.14.

T H E O R E M 6 .15 Let m and n be positive integers such that m £ n. Then the number of m-tuples ( x1, x2 , … , xm )
of positive integers satisfying the equation x1 + x2 + + xm = n is n - 1C m - 1.
PROOF As in the proof of Theorem 6.14, write n number of 1s on a line and put the symbol “(” on the
left of the first 1 and the symbol “)” on the right of the last 1 as shown below:
(1 - 1 - 1 - 1 - - 1 - 1)
Consider the n – 1 gaps between the two consecutive 1s. Choose any m – 1 of these gaps and fill
them with the symbol “) + (” and the remaining gaps be filled with the symbol +. Then, we get
an m-tuple ( x1, x2 , … , xm ) such that
x1 + x2 + + xm = n
and vice-versa. For example, for n = 10 and m = 4
(1 + 1 + 1) + (1) + (1 + 1) + (1 + 1 + 1 + 1)
gives a 4-tuple (3, 1, 2, 4) with 3 + 1 + 2 + 4 = 10 and conversely the tuple (2, 2, 3, 3) is obtained
by the expression
(1 + 1) + (1 + 1) + (1 + 1 + 1) + (1 + 1 + 1)
www.jeeneetbooks.in
292 Chapter 6 Permutations and Combinations

Therefore, each choice of m – 1 gaps from n – 1 gaps gives rise to an m-tuple ( x1, x2 , … , xm ) of
positive integers satisfying the equation
x1 + x2 + + xm = n
and vice-versa. Thus the number of required m-tuples is n - 1C m - 1 . ■

T H E O R E M 6 .16 Let m and n be positive integers. Then the number of m-tuples ( x1, x2 , … , xm ) of non-negative
integers satisfying the equation x1 + x2 + + xm = n is ( n + m - 1) C( m - 1).

PROOF We will slightly modify the proof of Theorem 6.15. Here xi s can be 0 also. Consider n + m – 1
boxes in a row as shown:

Let us choose any m – 1 of these and label these chosen boxes as b1, b2 , …, bm - 1 from left to right.
For 1 £ i £ m - 1, let xi +1 be the number of boxes that are not chosen between bi and bi +1. Let x1
be the number of boxes to the left of b1 and let xm be the number of boxes to the right of bm-1. It
can be easily seen that this is a one-to-one correspondence between the (m – 1)-element subsets
of the (n + m – 1)-element set of boxes onto the m-tuples ( x1, x2 , … , xm ) of non-negative inte-
gers satisfying the equation x1 + x2 + + xm = n. For example, for n = 4 and m = 6, the 6-tuple
(1, 0, 0, 1, 0, 2) corresponds to the choice of b1, b2, b3, b4, b5 given below.

b1 b2 b3 b4 b5

( m + n - 1)
Thus, the number of required m-tuples is C( m - 1). ■

Examples

(1) The number of 6-tuples ( x1 , x2 , x3 , x4 , x5 , x6 ) (2) The number of 6-tuples (x1, x2, x3, x4, x5, x6) of non-
of positive integers satisfying the equation negative integers satisfying the equation x1 + x2 +
x1 + x2 + x3 + x4 + x5 + x6 = 12 is x3 + x4 + x5 + x6 = 12 is
( 12 + 6 - 1)
( 12 - 1)
C( 6 - 1) = 11C 5 = 462 C( 6 - 1) = 17 C 5 = 6188

T H E O R E M 6 .17 The maximum number of parts into which a plane is cut by n lines is
n2 + n + 2
2
PROOF Let y (n) denote the maximum number of parts into which a plane is cut by n lines. We shall
prove that

n2 + n + 2
y (n) =
2
by using induction on n. Clearly

12 + 1 + 2
y (1) = 2 =
2
Note that the number of parts cut by n lines is maximum only when any two of these lines
intersect. We can see from the adjoining figure that
22 + 2 + 2
y ( 2) = 4 =
2
www.jeeneetbooks.in
6.3 Combinations 293

When we draw another line, intersecting these two, we get three more parts, as shown in Figure 6.3.
In general, we can get n more parts by considering the nth line in addition to y (n - 1). That is,
y (n) = y (n - 1) + n
Figure 6.3 shows the same for n = 4. By induction, we have
y (n) = y (n - 1) + n

(n - 1)2 + (n - 1) + 2
= +n
2
n2 + n + 2
=
2
8

1
1 9
1 4 2
4 2
4 2 3
3
5 6 7 10
3 5 6 7

11
FIGURE 6.3 Theorem 6.17. ■

Example
82 + 8 + 2
The maximum number of parts into which a plane is cut by 8 lines is = 37.
2

Note: The minimum number of parts into which a plane is cut by n lines is n + 1, since n parallel lines give us n + 1 parts.
Figure 6.4 shows the same for 4 parallel lines. Any pair of intersecting lines gives us more number of parts.
1
2
3
4
5
FIGURE 6.4 A plane cut by 4 lines into 5 parts.

Now, we will turn our attention to the number of various types of functions from a finite set into another finite set.
We first prove following simple theorem.

T H E O R E M 6 .18 Let X and Y be non-empty finite sets, | X | = m and |Y | = n. Then


1. The number of functions from Y into X is mn.
2. The number of injections (one-one functions) from Y into X is zero if m < n, and m C n × n !
(= m Pn ) if m ³ n.
3. The number of bijections of Y onto X is zero if m ¹ n, and m! if m = n.
PROOF 1. With each function f : Y ® X, each element of Y is to be mapped onto one element in
the m-element set X. Since Y has n elements, by the fundamental principle, the number of
functions of Y into X is mn.
2. If there is an injection of Y into X, then | Y | £ | X |. Therefore, if n > m, then there are no injec-
tions of Y into X. Suppose that n £ m. If f : Y ® X is an injection, then | Y | = | f (Y ) |, that is, f(Y )
is an n-element subset of X. On the other hand, with each n-element subset Z of X, we can get
www.jeeneetbooks.in
294 Chapter 6 Permutations and Combinations

n! number of bijections from Y onto Z, each of which can be treated as an injection of Y


into X. Since | X | = m, the number of n-element subsets of X is mC n. Thus, the number of
injections of Y into X is
m!
C n × n! =
m
n ! = m Pn
(m - n)! n !
3. We already have this (from part 2). ■

T H E O R E M 6 .19 For any positive integers m and r such that m ³ r, let am(r) be the number of surjections of an
m-element set onto an r-element set. Then
r

å
s =1
r
C s a m ( s) = r m

PROOF Let m ³ r > 0, A be an m-element set and B be an r-element set. The total number of mappings
of A into B is rm. Each mapping f : A ® B can be regarded as a surjection of A onto f(A); also
1 £ | f ( A) | £ r. On the other hand, with each s-element subset (1 £ s £ r ) C of B, any surjection
of A onto C can be regarded as a mapping of A into B. Therefore, the total number of mappings
of A into B is equal to the total number of surjections of A onto non-empty subsets of B. For
each 1 £ s £ r, there are r C s number of subsets of B and hence the number of mappings f : A ® B
such that | f ( A) | = s is r C sa m ( s). Therefore,
r

å r
C s a m ( s) = r m
s =1 ■

C O R O L L A R Y 6 .13 For any integers m ³ r > 0, the number a m (r ) of surjections of an m-element set onto an
r-element set is given by a recursive formula
r -1
a m (r ) = r m - å r C s a m ( s)
s =1

and a m (1) = 1

Try it out Prove that am (r ) is also equal to


r -1

å (-1)
s=0
s r
C( r - s ) (r - s)m

Example 6.16

Let A be a 4-element set and B a 3-element set. Then a 4 (3) = 34 - 3C 1a 4 (1) - 3C 2a 4 (2)
evaluate the number a 4 (3) of surjection of A onto B.
= 81 - 3 × 1 - 3 × 14
Solution: We have = 36
a 4 (1) = 1 Thus, there are 36 surjections of a 4-element set onto a
a 4 (2) = 2 4 - 2 C 1 × a 4 (1) = 14 3-element set.

D E FINIT ION 6. 4 Let X be a non-empty set. A bijection of X onto itself is called a permutation on X. A permutation
f on X is called a derangement of X if f ( x) ¹ x for all x Î X.

We will derive a recursive formula for the number of derangements of a finite set.
www.jeeneetbooks.in
6.3 Combinations 295

T H E O R E M 6 .20 For any positive integer r, let dr be the number of derangements of an r-element set. Then
n
1 + å n C r dr = n !
r =1

for any integer n > 0 or


n

å
r =0
n
C r dr = n ! (where d0 = 1)

n-1
and dn = n ! - å n C r dr
r =0

PROOF Let X be an n-element set, n > 0, and P(X ) be the set of all permutations on X. It is well-known
that P(X ) has n! elements. For any subset A of X, let
D( A) = { f Î P( X ) f (a) ¹ a for all a Î A; f ( x) = x for all x Î X - A}
That is,
D( A) = { f Î P( X ) | f ( x) ¹ x Û x Î A}
Then, clearly D(0) has only one element, namely the identity map and D(X ) is precisely the set
of all derangements of X. For any f ÎP( X ), we set that f Î D( A), where A = {x Î X | f ( x) ¹ x}.
Therefore, we get that

P( X ) =
AÍ X
∪ D( A)
It can be easily verified that
D( A) Ç D( B) = 0 whenever A ¹ B
and hence P(X ) is the disjoint union of D(A)s, A Í X. Since there are nCr number of subsets
of X, each with r elements, it follows that
n
n ! = | P( X ) | = å | D( A) | = 1 + å
AÍ X r =1
n
Cr dr (since | D(0) | = 1)

where dr is the number of derangements of an r-element set [since the members of D(A) are in
one-to-one correspondence with the derangements of A; f  f /A is that one-to-one correspon-
dence]. Thus
n
n ! = 1 + å n C r dr
r =1

This also can be expressed as


n
n ! = å n C r dr
r =0

where d0 = 1. Note that d1 = 0 and hence


n-1
dn = n ! - 1 - å n C r dr
r=2 ■

Try it out Prove that the number of derangements of an n-element set is


n
(-1)k
n! å
k =0 k !
www.jeeneetbooks.in
296 Chapter 6 Permutations and Combinations

Examples

We have (4) d3 = 3! - 1 - 3C 2 d2 = 6 - 1 - 3 × 1 = 2
(1) d0 = 1, by definition. (5) d4 = 4 ! - 1 - 4 C 2 d2 - 4 C 3 d3 = 24 - 1 - 6 × 1 - 4 × 2 = 9
(2) d1 = 0, since a singleton set cannot have derangement. (6) d5 = 5! - 1 - 5C 2 d2 - 5C 3 d3 - 5C 4 d4
(3) d2 = 2! - 1 = 1 = 120 - 1 - 10 × 1 - 10 × 2 - 5 × 9 = 44

Example 6.17

List all the derangements of the 4-element set {1, 2, 3, 4}. 1® 3 1® 3 1® 3


2®1 2®4 2®4
Solution: The derangements are as follows:
3® 4 3®1 3® 2
1® 2 1® 2 1® 2 4®2 4®2 4®1
2®1 2®3 2®4
3® 4 3® 4 3®1 1® 4 1® 4 1® 4
4®3 4®1 4®3 2®1 2®3 2®3
3® 2 3®1 3® 2
4®3 4®2 4®1

Try it out Show that there are 44 derangements of a 5-element set and there are 265 of a 6-element set.

T H E O R E M 6 .21 Let n be a positive integer and n = p1a1 p2a 2 pka k be a prime decomposition of n. Then the
number of distinct ordered pairs of positive integers ( p, q), such that the least common multiple
of p and q is n, is

(2a 1 + 1)(2a 2 + 1) (2a k + 1)

PROOF Since both p and q are factors n, we can suppose


p = p1x1 p2x2 pkxk and q = p1y1 p2y2 pkyk

where xi and yi (i = 1, 2, …, k) are non-negative integers. As n is the least common multiple of


p and q, we have
max{xi, yi} = ai

Hence, (xi, yi) can be equal to (0, ai), (1, ai), (2, ai), …, (aI, aI) and (aI, 0), (aI, 1), (aI, 2), …, (aI, aI−1)
whose number is 2aI + 1. By multiplication principle, there are (2a 1 + 1)(2a 2 + 1) (2a k + 1)
ordered pairs of positive integers ( p, q) whose least common multiple is

n = p1a1 p2a 2 pka k ■

Example

Consider n = 23 ´ 52 ´ 75. Then the number of distinct (6 + 1)(4 + 1)(10 + 1) = 7 ´ 5 ´ 11


ordered pairs of positive integers ( p, q), whose least
= 385
common multiple is n = 23 ´ 32 ´ 75, is
www.jeeneetbooks.in
Worked-Out Problems 297

WORKED-OUT PROBLEMS
Single Correct Choice Type Questions
m+n
1. If m and n are positive integers such thatP2 = 90 4 × n! 5 × (n - 1)!
m- n =
and P2 = 30, then the number of ordered pairs (n - 3)! (n - 1 - 3)!
(m, n) of such integers is
4n
(A) 4 (B) 3 (C) 2 (D) 1 =5
n-3
Solution: Given that
4 n = 5(n - 3)
90 = m + n P2 = (m + n)(m + n - 1)
n = 15
and 30 = m - n P2 = (m - n)(m - n - 1) Answer: (B)
Therefore
4. If 56
Pr + 6 : 54
Pr + 3 = 30800 : 1, then r is equal to
(m + n)2 - (m + n) - 90 = 0
(A) 41 (B) 31 (C) 21 (D) 39
(m + n - 10)(m + n + 9) = 0
Solution: By hypothesis
We take m + n = 10 (since m and n are positive). Similarly,
(56)! (54)!
m - n = 6 (we have to consider m > n only). Therefore = 30800 ×
m = 8 and n - 2 or (56 - (r + 6))! [54 - (r + 3)]!

(m, n) = (8, 2) (56)! 30800 × (54)!


=
Answer: (D) (50 - r )! (51 - r )!
30800
2. If
2n+1
Pn - 1 : 2 n - 1 Pn = 3 : 5, then the value of n is 56 × 55 =
(51 - r )
(A) 5 (B) 4 (C) 6 (D) 7
30800
Solution: Given that 51 - r = = 10
56 × 55
5 × 2 n + 1 Pn - 1 = 3 × 2 n - 1 Pn r = 41
5 × (2 n + 1)! 3 × (2 n - 1)! Answer: (A)
=
[(2 n + 1) - (n - 1)]! (2 n - 1 - n)!
5. If 9P5 + 5 × 9P4 = 10Pr, then r is equal to
5(2 n + 1)! 3 × (2 n - 1)!
= (A) 6 (B) 5 (C) 4 (D) 3
(n + 2)! (n - 1)!
Solution: By hypothesis,
5(2 n + 1) × 2 n
=3 9! 5 × 9! (10)!
(n + 2)(n + 1)n + =
(9 - 5)! (9 - 4)! (10 - r )!
10(2n + 1) = 3(n + 2)(n + 1)
æ 1 5ö (10)!
3n 2 - 11n - 4 = 0 9 !ç + ÷ =
è 4 ! 5!ø (10 - r))!
(n - 4)(3n + 1) = 0 (10)! 9 ! 10 !
= 2× =
n=4 (10 - r )! 4 ! 5!
Answer: (B) 10 - r = 5

3. If four times the number of permutations of n distinct r=5


objects taken three at a time is equal to five times the Answer: (B)
number of permutations of n - 1 distinct objects taken
three at a time, then n is equal to 6. There are finite number of distinct objects. If the
(A) 20 (B) 15 (C) 10 (D) 25 arrangements of 4 objects (in a row) is 12 times the
number of arrangements of 2 objects, then the number
Solution: By hypothesis, of objects is
4 nP3 = 5 n-1 P3 (A) 10 (B) 8 (C) 4 (D) 6
www.jeeneetbooks.in
298 Chapter 6 Permutations and Combinations

Solution: Let the number of objects be n. Then, by Solution: Consider A, B, C as one single object so that
hypothesis, including this, there are 5 objects which can be arranged in
5! ways. In each of these A, B, C can be arranged among
n
P4 = 12 ´ n P2
themselves in 3! ways. Therefore, the total number of
n! n! required arrangements is 3! ´ 5!
= 12 ´
(n - 4)! (n - 2)! Answer: (D)
12 10. A total of 6 boys and 5 girls are to be arranged in a
1=
(n - 2)(n - 3) row. The number of arrangements such that no two
girls stand together is
n - 5n - 6 = 0
2

(A) 11 P5 (B) 11 P6 + 11 P5
(n - 6)(n + 1) = 0 11! 7!
(C) (D) ´ 6 !
Now n ¹ -1, therefore n = 6. 6! 2!
Answer: (D) Solution: Given that each girl should stand in between
two boys (there are 5 such places since the number of
å
20
7. The value of K =1
K× K PK is boys is 6) or before the boys or after the boys. Therefore,
there are 7 eligible places for the 5 girls and hence
(A) 20 ! + 1 (B) 21! - 1
they can be arranged in 7P5 ways. But the 6 boys can be
(C) 20 ! - 1 (D) 21! - 20
arranged among themselves in 6! ways. Therefore the
Solution: We have required number of arrangements
20 20
K!
7
P5 ´ 6 !
å K × K PK =
K =1
åK×
K =1 1 Answer: (D)
20
= å [(K + 1) - 1]K !
K =1
11. A total of 5 mathematics, 3 physics and 4 chemistry
books are to be arranged in a shelf such that the
20
books on the same subject are never separated. If one
= å [(K + 1)! - K !] particular mathematics book is to be in the middle
K =1 of all the mathematics books, then the number of
arrangements is
= (2 ! - 1!) + (3! - 2 !) + + [(21)! - (20)!]
(A) 3!(5! + 4 ! + 3!) (B) 3!(5! ´ 4 ! ´ 3!)
= (21)! - 1! (C) 3!(4 ! ´ 4 ! ´ 3!) (D) 3!(4 ! ´ 3! ´ 3!)
Answer: (B)
Solution: Consider the books on the same subject as
a single bundle, so that 3 bundles can be arranged in
8. The number of 6-digit numbers that can be formed
3! ways. But mathematics books can be arranged among
by using the numerals 0, 1, 2, 3, 4 and 5 (without
themselves in 4! ways (since one book is fixed in the
repetition of the digits) such that even numbers
middle) the physics books in 3! ways and chemistry
occupy odd places is
books in 4! ways. Therefore, the required number is
(A) 48 (B) 24 (C) 36 (D) 72
3!(4 ! ´ 4 ! ´ 3!)
Solution: We can arrange 0, 2, 4 in the odd places
Answer: (C)
in 3! ways. After filling the odd places, the remaining
3 places can be filled by the remaining numbers (1, 3 and 5)
12. The number of arrangement of the letters of the
in 3! ways. But among these numbers, there are 2 ! ´ 3!
word BANANA in which two Ns do not appear
numbers in which 0 occupies the first place from the left.
adjacently is
Therefore, the required number is
(A) 40 (B) 60 (C) 80 (D) 100
3! × 3! - 2 ! 3! = 36 - 12 = 24
Solution: Letters other than Ns are BAAA. There are
Answer: (B)
4 ways of arranging these (B is the first place, B in the
second place, etc.). Two Ns are to be arranged in between
9. The first 7 letters of the English alphabet are arranged
B, A, A, A. There are five places:
in a row. The number of arrangements in which A, B
and C are never separated is B A A A
(A) 5! (B) 3 ´ 5! (C) 4 ! ´ 5! (D) 3! ´ 5! 1st 2nd 3rd 4th 5th
www.jeeneetbooks.in
Worked-Out Problems 299

The insertion of Ns can be made in 5 P2 ´ (1/ 2 !) ways. Solution: Each of the places can be filled in 3 ways
Therefore the total required number is (with 3, 5 or 7). The total number of ways

5! 3 ´ 3 ´ ´ 3 (n times) = 3n
´ 4 = 40
3! 2 ! Now,
Answer: (A) 3n ³ 900 Û 3n - 2 ³ 100 Û n - 2 ³ 5 Û n ³ 7
13. A five-letter word is to be formed by using the letters Therefore the smallest value of such n = 7.
of the word MATHEMATICS such that Answer: (B)
(i) odd places of the word are to be filled with
unrepeated letters and 16. The letters of the word MOTHER are arranged
in all possible ways and the resulting words are
(ii) even places are to be filled with repeated letters.
written as in a dictionary. Then the rank of the word
Then the number of words thus formed is “Mother” is
(A) 300 (B) 360 (C) 180 (D) 540 (A) 301 (B) 304 (C) 307 (D) 309
Solution: We have five places: Solution:
1 2 3 4 5 No. of words beginning with E = 5! = 120
No. of words beginning with H = 5! = 120
There are three odd places and two even places in a five- No. of words beginning with ME = 4 ! = 24
letter word. No. of words beginning with MH = 4 ! = 24
Unrepeated letters: H, E, I, C, S No. of words beginning with MOE = 3! = 6
Repeated letters: AA, MM, TT No. of words beginning with MOH = 3! = 6
No. of words beginning with MOR = 3! = 6
The 3 odd places can be filled with the 5 unrepeated
No. of words beginning with MOTE = 2 ! = 2
letters in
No. of words beginning with MOTHER = 1
5
P3 = 60 ways Therefore, the rank of the word “Mother” is
The 2 even places can be filled with 2 different or 2 alike
120 + 120 + 24 + 24 + 6 + 6 + 6 + 2 + 1 = 309
letters from the repeated letters in
Answer: (D)
3
P2 + 3 = 9 ways
Therefore, the number of words thus formed is 17. There are 2 professors each of mathematics, physics
60 ´ 9 = 540 and chemistry. The number of ways these 6 profes-
sors can be seated in a row so that professors of the
Answer: (D) same subject be seated together is
14. A five-digit number divisible by 3 is to be formed (A) 48 (B) 36 (C) 24 (D) 120
using the numerals 0, 1, 2, 3, 4 and 5 without repeti- Solution: Treat professors of the same subject as a
tion. The total number of ways this can be done is single object. Now 3 objects can be arranged in 3! ways.
(A) 216 (B) 240 (C) 600 (D) 3125 But, professors of the same subject can be interchanged
among themselves in 2 ! ´ 2 ! ´ 2 ! ways. Therefore, the
Solution: A number is divisible by 3 if and only if the required number is
sum of its digits is divisible by 3. Therefore the numerals
to be used are 0, 1, 2, 4, 5 or 1, 2, 3, 4, 5. 3! ´ 2 ! ´ 2 ! ´ 2 ! = 48
In the first case, the number is 5! - 4 ! = 96. Answer: (A)
In the second case this is 5! = 120.
Therefore, the required number is 96 + 120 = 216. 18. Suppose one has to form 7-digit numbers using the
numerals 1, 2, 3, 4, 5, 6 and 7. If the extreme places are
Answer: (A)
occupied by even numerals, then the number of such
numbers is
15. An n-digit number means a positive integer having
n digits. A total of 900 distinct n-digit numbers are (A) 720 (B) 360 (C) 5040 (D) 120
to be formed using only the numerals 2, 5 and 7. Solution: There are 3 even numbers (2, 4 and 6) and
The smallest value of n for which this is possible is 2 places (first and last) are to be filled by these. This can
(A) 6 (B) 7 (C) 8 (D) 9 be done in 3 P2 ways. The remaining five places are to be
www.jeeneetbooks.in
300 Chapter 6 Permutations and Combinations

filled by the remaining five numerals. This can be done in Solution: The required number is 4 ! = 24.
5! ways. Therefore the required number is Answer: (A)
3
P2 ´ 5! = 720
23. The total number of numbers that can be formed by
Answer: (A)
using all the digits 1, 2, 3, 4, 3, 2, 1 so that odd digits
always occupy odd places is
19. The number of ways of arranging the digits 1, 1, 1, 1,
1, 2, 2, 2, 3, 3, 3, 4, 5, 5, and 6 taken all at a time so that (A) 24 (B) 20 (C) 18 (D) 28
the 3s are separated is Solution: Among the given digits, there are four odd
(A) 95315040 (B) 95135040 digits (1, 3, 3, 1) and there are four odd places (the first,
(C) 95135400 (D) 95153040 third, fifth, seventh). These four odd places can be filled
by 1, 3, 3, 1 in
Solution: Each 3 can be placed in between the other
digits or before them or at the end in 4!
=6 ways
13
P3 2! 2!
ways
3! The remaining 3 places (second, fourth, sixth) can be
filled with 3 digits (2, 4, 2) in
Remaining 12 can be arranged in
3!
12 ! = 3 ways
ways 2!
5! 3! 2 !
The total number of required arrangements is 6 ´ 3 = 18.
Therefore the required number is Answer: (C)
13
P3 12 !
´ = 95135040 24. Let there be four lines and four circles in a plane. Let
3! 5! 3! 2 !
A be the set of all points of intersections of the lines,
Answer: (B) B the set of all points of intersections of the circles
and C the set of all points of intersections of the lines
20. Five letters are to be inserted into five addressed and circles. If n(S) denotes the number of elements
envelopes. The number of ways of inserting the in S, then the maximum value of n( A È B È C ) is
letters so that no letter goes to its corresponding
(A) 40 (B) 50 (C) 66 (D) 70
envelope is
(A) 120 (B) 44 (C) 31 (D) 41 Solution: The maximum value of n(A) is 4C 2 = 6.
The maximum value of n( B) is 2 ´ 4C 2 = 12. Since any
Solution: The number of required ways is line cuts a circle at the most in 2 points, the maximum
number of points in which the lines intersect the circles
æ 1 1 1 1 1ö æ1 1 1 1 ö
5!ç 1 - + - + - ÷ = 120 ç - + - is 8 ´ 4 = 32. Therefore the maximum value of
è 1! 2 ! 3! 4 ! 5!ø è 2 6 24 120 ÷ø
n( A È B È C ) = 6 + 12 + 32 = 50
= 60 - 20 + 5 - 1 = 44
Answer: (B)
(Note: We have made use of the result in “Try it out”
after Theorem 6.20.) 25. The total number of mappings from a four element
Answer: (B) set to a three element set is
(A) 54 (B) 12 (C) 27 (D) 81
21. The number of 5-digit numbers by using 1, 1, 1, 2, 2 is
Solution: If A = {x1, x2, x3, x4} and B = {a1, a2, a3}, then any
(A) 7 (B) 8 (C) 9 (D) 10 mapping of A into B should take x1 to any of a1, a2, a3 and
Solution: The required number is likewise x2, x3 and x4. Therefore the number of mapping
from A into B is 3 ´ 3 ´ 3 ´ 3 = 81.
5!
= 10 Answer: (D)
3! 2 !
26. Let f ( x) º x - 3 x + 3 and x1, x2 , x3 and x4 be solu-
2
Answer: (D)
tions of the equation f ( f ( x)) = x. Then the number
22. The number of cyclic permutations of 5 distinct of arrangements of x1, x2, x3 and x4, taken all at a
objects is time, is
(A) 24 (B) 120 (C) 60 (D) 42 (A) 24 (B) 4 (C) 6 (D) 1
www.jeeneetbooks.in
Worked-Out Problems 301

Solution: Note that every solution of f ( x) = x is also a The no. of words beginning with CI is 4!
solution of f ( f ( x)) = x. The no. of words beginning with CN is 4!
The next word is COCHIN
f ( x) = x Þ x 2 - 4 x + 3 = 0 Þ x = 3 or 1
Therefore, the number of words before COCHIN is
Therefore, 3 and 1 are roots of f ( x) = x. Also, 4 ´ 4 ! = 96.
f ( f ( x)) = x Þ ( x 2 - 3 x + 3)2 - 3( x 2 - 3 x + 3) + 3 = x Answer: (C)

Þ x 4 - 6 x 3 + 12 x 2 - 10 x + 3 = 0 30. The number of 7-digit numbers whose sum of the


Since 3 and 1 are roots of f ( x) = x, they are roots of digits equals 10 and which is formed by using the
f ( f ( x)) = x also and therefore digits 1, 2 and 3 only is
f ( f ( x)) - x = ( x - 3)( x - 1)( x 2 - 2 x + 1) = ( x - 3)( x - 1)3 (A) 55 (B) 66 (C) 77 (D) 88

Therefore, 3, 1, 1, 1 are solutions of f ( f ( x)) = x. Hence Solution: In a 7-digit number formed by using 1, 2
the number of arrangements of the solutions is and 3, suppose that 1 appears x times, 2 appears y times
and 3 appears z times. Then by hypothesis
4!
=4 x + 2 y + 3z = 10 and x+ y+z = 7
3!
Answer: (B) Solving these equations we get
y + 2z = 3
27. Professors a, b, c and d are conducting an oral exami-
nation for a Ph.D. student x on combinatorics. The from which we get either
professors are to sit in chairs in a row. Professors
y = 1, z = 1 and x=5
a and b are to sit together. Professor c is the guide
of x and he has to sit by the side of Professors a and or y = 3, z = 0 and x=4
b. The number of arrangements is
Therefore, the total number is
(A) 8 (B) 7 (C) 6 (D) 5
7! 7!
Solution: The arrangements of a, b, c must be abc, bac, + = 42 + 35 = 77
cab, cba. In each of these arrangements, d can take his seat 5! 4 ! 3!
at either end. Therefore the number of arrangements is Answer: (C)
4´2=8
31. Eight chairs are numbered 1 to 8. Two women and
Answer: (A)
three men wish to occupy one chair each. First the
women choose the chairs from among the chairs
28. There are 2 copies of each of 3 different books. The
numbered 1 to 4 and then men select chairs from
number of ways they can be arranged in a shelf is
among the remaining. The number of possible
(A) 12 (B) 60 (C) 120 (D) 90 arrangements is
Solution: Totally there are 3 sets of 2 alike books. The (A) 120 (B) 1440 (C) 16 (D) 240
total number of books is 6. Therefore the number of
Solution: Two women can sit in 4 chairs in 4 P2 = 12
arrangements is
ways. After the women, the 3 men can sit in the remaining
6! 720 6 chairs in 6 P3 = 120 ways. Therefore the total number of
= = 90 arrangements
2! 2! 2! 8
Answer: (D) 12 ´ 120 = 1440
Answer: (B)
29. The letters of the word COCHIN are permuted and
all permutations are arranged in alphabetical order 32. Total number of ways in which six “+” signs and
as in a dictionary. The number of words that appear four “–” signs can be arranged in a row so that no
before the word COCHIN is two “–” signs occur together is
(A) 360 (B) 192 (C) 96 (D) 48 (A) 55 (B) 25 (C) 45 (D) 35
Solution: The given word is COCHIN. Solution: First arrange the six “+” signs. This can be
The no. of words beginning with CC is 4! done in only one way. In between the “+” signs, there
The no. of words beginning with CH is 4! are 7 gaps (including the left most and right most places)
www.jeeneetbooks.in
302 Chapter 6 Permutations and Combinations

37. If C r - C 3 = C 2 , then r is equal to


8 7 7
The four signs can be arranged in these 7 gaps in
P4 7 × 6 × 5 × 4
7 (A) 3 or 5 (B) 5 or 4 (C) 4 or 6 (D) 6 or 5
= = 35
4! 24 Solution: We have
Answer: (D)
Cr - 7C3 = 7C2
8

33. Six different coloured hats of the same size are 8


Cr = 7C2 + 7C3 = 8C3
to be arranged circularly. The number of arrange-
This gives
ments is
(A) 60 (B) 50 (C) 40 (D) 45 r = 3 or r + 3 = 8 Þ r = 3 or 5
Answer: (A)
Solution: The required number of arrangements is
n
1
1 38. If an = å
(6 - 1)! = 60 r =0
n
Cr
2
n
Answer: (A) r
then å
r =0
n
Cr
34. If C 3r = 15C r + 3 , then the value of r is
15

is equal to
(A) 5 (B) 4 (C) 6 (D) 3
(A) (n - 1)an (B) n an
Solution: If n C r = n C s , then either r = s or r + s = n. n an
Therefore (C) (D) (n + 1)an
2
15
C 3r = 15C r + 3 Þ 3r + r + 3 = 15 (since r Î  + , 3r ¹ r + 3) Solution: Let
Þr=3 n
r 1 2 n
Answer: (D) s=å n
= 0+ n +n + + n
r =0 Cr C1 C2 Cn
35. If C 7 = C 4 , then C 8 is equal to
n n n
Also
(A) 156 (B) 165 (C) 265 (D) 256 n n-1
s= n
+n + +0 (∵ n C r = n C n - r )
Solution: We have Cn C n-1
n
C 7 = n C 4 Þ 7 + 4 = n (since 7 ¹ 4) Therefore
Therefore n n n n
2s = n
+ + + + n
11 × 10 × 9 C n n C n-1 n C n- 2 C0
n
C 8 = 11C 8 = 11C 3 = = 165
1× 2 × 3 n
1
Answer: (B) = nå n
= n an
r =0 Cr

å
5 ( 52 - r )
36. The value of C 4 +
47
C 3 is n an
r =1 s=
2
(A) 51C 4 (B) 53 C 3 (C) 52 C 4 (D) 53 C 4
Answer: (C)
Solution: It is known that nCr + n Cr + 1 = (n + 1)Cr + 1. There-
39. If C 2 r : C 2 r - 4 = 225 : 11, then r is equal to
28 24
fore
5 (A) 24 (B) 14 (C) 7 (D) 12
47
C 4 + å ( 52 - r ) C 3 = (51C 3 + 50 C 3 + 49 C 3 + 48 C 3 + 47 C 3 ) + 47 C 4
r =1 Solution: We have
= C3 + C3 + C3 + C3 + ( C3 + C4 )
51 50 49 48 47 47 28
C2r 225
24
=
= C3 + C3 + C3 + C3 + C4
51 50 49 48 48 C2r -4 11
Therefore
= 51C 3 + 50 C 3 + 49 C 3 + 49 C 4
28 ! (2r - 4)!(28 - 2r )! 225
= 51C 3 + 50 C 3 + 50 C 4 ´ =
(2r )!(28 - 2r )! (24)! 11
= 51C 3 + 51C 4 = 52 C 4
28 × 27 × 26 × 25 225
Answer: (C) =
(2r )(2r - 1)(2r - 2)(2r - 3) 11
www.jeeneetbooks.in
Worked-Out Problems 303

11 × 28 × 27 × 26 × 25 Solution: By hypothesis,
(2r )(2r - 1)(2r - 2)(2r - 3) =
225 n-1
Cr
= K2 - 3
= 11 × (14 × 2) × (13 × 2) × 3 n
Cr +1
= 11 × 12 × 13 × 14 Therefore
From this we have r+1
2r - 3 = 11 = K2 - 3
n
r=7 Since n ³ r + 1, we have
Answer: (C)
r+1
0< £1
40. The inequality ( n + 1)
C 6 + n C 4 > ( n + 2 ) C 5 - n C 5 holds for n
all n greater than Hence
(A) 8 (B) 9 (C) 7 (D) 1 0 < K2 - 3 £ 1
Solution: Consider Þ 3 < K2 £ 4
n+1
C 6 + n C 4 + n C 5 = n + 1C 6 + n + 1C 5 = n + 2 C 6 Þ 3<K£2
n+ 2 n+ 2
The given inequality holds Û C6 > C5 Answer: (B)
(n + 2)! (n + 2)!
Û > 43. Let Tn denote the number of triangles which can be
6 !(n + 2 - 6)! 5!(n + 2 - 5)! formed using the vertices of a regular polygon of n
1 1 sides. If Tn + 1 - Tn = 21, then n is equal to
Û >
6 !(n - 4)! 5!(n - 3)! (A) 6 (B) 7 (C) 5 (D) 4

1 1 Solution: To form a triangle, we need three non-collinear


Û > points. Therefore, Tn = nC 3 . Now
6 n-3
Û n-3 > 6 Tn + 1 - Tn = 21
n+1
Û n>9 C 3 - n C 3 = 21
Answer: (B) (n C 3 + n C 2 ) - n C 3 = 21
4 n(n - 1)
41. If CK denotes C K , then the value of = 21
2
2
æ K × CK
4
ö n 2 - n - 42 = 0
2å ç ÷ø
èC +C
K =1 4-K
K
(n - 7)(n + 6) = 0
is
n = 7 (since n > 0)
(A) 12 (B) 13 (C) 14 (D) 15
Answer: (B)
Solution: We have
44. The number of selections of 5 distinct letters from
K × CK K × CK K
= = the letters of the word INTERNATIONAL is
CK + C4-K 2 × CK 2
(A) 140 (B) 56 (C) 21 (D) 66
Therefore Solution: The distinct letters are I, N, A, T, E, O, L, R.
2 2 Therefore the number of selections of 5 letters is
4
æ K × CK ö æ 4 Kö 1
2å ç ÷ = 2 ç å ÷ = (12 + 2 2 + 32 + 4 2 ) = 15 8
C 5 = 56
K =1 è C K + C 4-K ø è K =1 2 ø 2
Answer: (A)
Answer: (D)
45. The sides AB, BC and CA of DABC have 3, 4 and 5
n-1
42. If C r = (K 2 - 3) × n C r + 1 and K is positive, then K interior points respectively on them (Figure 6.5). The
belongs to the interval number of triangles that can be formed using these
(A) (- 3 , 3 ) (B) ( 3 , 2] interior points is
(C) [0, 3 ] (D) ( 3 , 2) (A) 180 (B) 185 (C) 210 (D) 205
www.jeeneetbooks.in
304 Chapter 6 Permutations and Combinations

C 49. In a polygon (Figure 6.6), no three diagonals are


concurrent. If the total number of points of intersec-
tions of the diagonals interior to the polygon is 70,
then the number of diagonals of the polygon is
(A) 30 (B) 20 (C) 28 (D) 8

A B
FIGURE 6.5 Single correct choice type question 45.

Solution: The number of ways of selecting 3 points


from among 3 + 4 + 5 (= 12) points is 12 C 3 . But from
among these, we have to discount collinear sets of points.
Therefore the number of triangles is FIGURE 6.6 Single correct choice type question 49.

12 × 11 × 10 Solution: To get a point of intersection of two


12
C 3 - 3C 3 - 4 C 3 - 5C 3 = - 1 - 4 - 10 = 205
3! diagonals interior to the polygon, we need 4 vertices of
Answer: (D) the polygon. It is given that n C 4 = 70. Therefore
n(n - 1)(n - 2)(n - 3) = 70 ´ 4 !
46. A box contains two white, three black and four red
balls. The number of ways of selecting 3 balls from = 8´7´6´5
the box with atleast one black is n=8
(A) 64 (B) 74 (C) 54 (D) 84 The polygon has 8 vertices and hence 8 sides. Therefore
the number of diagonals is
Solution: The number of ways is 8
C 2 - 8 = 20
9
C 3 - 6 C 3 = 84 - 20 = 64
Answer: (B)
Answer: (A)
50. A total of 930 greeting cards are exchanged among the
47. Five balls of different colours are to be placed in
residents of flats. If every resident sends a card to every
three boxes of different sizes. Each box can hold
other resident of the same flats, then the number of resi-
all the five balls. The number of ways of placing the
dents is
balls so that no box is empty is
(A) 30 (B) 29 (C) 32 (D) 31
(A) 140 (B) 150 (C) 240 (D) 250
Solution: Let n be the number of residents in the flats.
Solution: The number of placings of five different balls
Then 2 ´ n C 2 = 930. Therefore
in three boxes of different sizes is equal to the number of
surjections of a five-element set onto a three-element set n(n - 1) = 930
which is equal to (n - 31)(n + 30) = 0
2

å (-1)
K =0
K 3
C 3 - K (3 - K ) = C 3 × 3 - C 2 × 2 + C 1 × 1
5 3 5 3 5 3 5
This gives n = 31 or –30. The second value is not possible
and hence, n = 31.
= 243 - 96 + 3 = 150
Answer: (D)
(See Corollary 6.13 or “Try it out” following it.)
Answer: (B) 51. From 5 vowels and 5 consonants, the number of
four-letter words (without repetition) having 2
48. There are 10 points in a plane of which no three are vowels and 2 consonants that can be formed is
collinear and some four points are concyclic. The (A) 100 (B) 2400 (C) 1600 (D) 24
maximum number of circles that can be drawn using
these is Solution: Now 2 vowels and 2 consonants can be
(A) 116 (B) 120 (C) 117 (D) 110 selected in
5
C 2 ´ 5C 2 = 10 ´ 10 = 100 ways
Solution: We can draw a circle passing through any
three given non-collinear points. Therefore, maximum After selection, the four letters can be permuted in
number of circles is 4! ways. Therefore, the number of words is
(10 C 3 - 4 C 3 ) + 1 = 117 100 ´ 4 ! = 2400
Answer: (C) Answer: (B)
www.jeeneetbooks.in
Worked-Out Problems 305

52. A total of 6 boys and 6 girls are to sit in a row 54. The number of proper divisors of 240 is
alternatively and in a circle alternatively. Let m be (A) 18 (B) 20 (C) 19 (D) 24
the number of arrangements in a row and n the
number of arrangements in a circle. If m = Kn, then Solution: Proper divisors of a number are divisors
the value of K is other than unity and itself. We have
(A) 10 (B) 11 (C) 12 (D) 13 240 = 2 4 ´ 31 ´ 51
Solution: Linear permutations with boy in the first Any divisor of 240 is of the form 2 a ´ 3b ´ 5c, 1 where
place are of the form B G B G B G B G B G B G and the 0 £ a £ 4, 0 £ b £ 1 and 0 £ c £ 1. Therefore the number
number of such is 6 ! ´ 6 !. The number of linear permu- of proper divisors is
tations with girl in the first place is 6 ! ´ 6 !. Therefore the
number of row arrangements is 5 ´ 2 ´ 2 - 2 = 20 - 2 = 18
Answer: (A)
2 ´ 6! ´ 6!
Regarding circular permutation, start with a place which 55. There are 7 distinguishable rings. The number of
can filled by a boy or a girl and after that the arrange- possible five-ring arrangements on the four fingers
ment becomes linear. Placing the boys first and then (except the thumb) of one hand (the order of the
arranging the girls in 6 gaps, the number of such circu- rings on each finger is to be counted and it is not
lar arrangements is 5! ´ 6 !. Now, m = 2 ´ 6 ! ´ 6 ! and required that each finger has a ring) is
n = 5! ´ 6 !. Therefore (A) 214110 (B) 211410
m = Kn (C) 124110 (D) 141120

2 ´ 6 ! ´ 6 ! = K ´ 5! ´ 6 ! Solution: There are 7 C 5 ways of selecting the rings to be


worn. If a, b, c, d are the numbers of rings on the fingers,
K = 12 we need to find the number of quadruples (a, b, c, d) of
Answer: (C) non-negative integers such that a + b + c + d = 5. The
number of such quadruples is
53. From the vertices of a regular polygon of 10 sides,
( 5 + 4 - 1)
the number of ways of selecting three vertices such C( 4 - 1) (= 8 C 3 )
that no two vertices are consecutive is
For each set of 5 rings, there are 5! assignments. Therefore,
(A) 10 (B) 30 (C) 50 (D) 40 the total number of required arrangements is
Solution: Let A1, A2, …, A10 be vertices of a regular 7
C 5 ´ 8 C 3 ´ 5! = 141120
polygon of 10 sides.
Answer: (D)
The number of ways of selecting 3 vertices is 10 C 3 .
The number of ways of selecting 3 consecutive vertices is 56. The number of ordered pairs of positive integers (a, b),
(i.e., A1 A2 A3 , A2 A3 A4 , … , A10 A1 A2 ) = 10 . such that their least common multiple is the given
The number of ways of selecting three vertices such that positive integer 72 ´ 113 ´ 194, is
two vertices are consecutive = (First select 2 consecutive
(A) 215 (B) 315 (C) 415 (D) 195
vertices, leave their neighboring two vertices and select
one more from the remaining 6 vertices) is Solution: By Theorem 6.21, the required number
of ordered pairs of positive integers (a, b), such that
10 ´ 6 C 1 = 60 the least common multiple of a and b is the number
The total number of required selections is 72 ´ 113 ´ 194, is equal to
10
C 3 - 10 - 60 = 120 - 70 = 50 (2 ´ 2 + 1)(2 ´ 3 + 1)(2 ´ 4 + 1) = 5 ´ 7 ´ 9 = 315
Answer: (C) Answer: (B)

Multiple Correct Choice Type Questions


1. Let n and r be positive integers such that 1 £ r £ n. Solution: We have
Which of the following is/are true?
n! n! n!
(A) n Pn = n Pn -1 (B) n Pn = 2 ´ n Pn - 2
n
Pn = = = = n Pn - 1
(n - n)! 1 [n - (n - 1)]!
(C) n Pr = n ´ n - 1 Pr - 1 (D) n Pr = r ´ n - 1 Pr - 1
www.jeeneetbooks.in
306 Chapter 6 Permutations and Combinations

n! n! (A) The total number of words that can be formed


n
Pn = = 2 = 2 ´ n Pn - 2
1 2! using all the letters of the word is 7560.
(B) The number of words which begin with A and end
n! (n - 1)!
n
Pr = = n× = n × n - 1 Pr - 1 with A is 1260.
(n - r )! [(n - 1) - (r - 1)]! (C) The number of words in which vowels occupy the
n-1 (n - 1)! n even places is 60.
r´ Pr - 1 = r ´ ¹ Pr (D) The number of words in which all the four vowels
(n - r )!
occupy adjacent places is 360.
Answers: (A), (B), (C)
Solution: The word ALLAHABAD consists of 4 As,
2. Given that Pr = n(n - 1)(n - 2) … (n - r + 1). Then
n 2 Ls, 1 B, 1 D and 1 H.
which of the following are true? (A) Out of the total 9 objects, 4 are alike of one kind and
2 are alike of another kind. Therefore, the number
(A) n P4 = 1680 Þ n = 8 (B) 12 Pr = 1320 Þ r = 3
of words is
(C) 13Pr = 1220 Þ r = 4 (D) n P3 = 1220 Þ n = 9
9! 9×8×7×6×5
Solution: (A) We have = = 7560
4! 2! 2
n(n - 1)(n - 2)(n - 3) = 1680 = 8 ´ 7 ´ 6 ´ 5 Þ n = 8 (B) Put one A in the first place and another in the last
(B) We have place. In the remaining there are 2 As and 2 Ls. The
number of such words is
12(12 - 1) (12 - r + 1) = 1320
7!
11 × 10 (13 - r ) = 110 = 1260
2! 2!
r=3 (C) There are four even places and 4 vowels A, A, A, A.
(C) It is not true, since 13 is not a factor of 1220. These can be put in even places in only one way. The
(D) For the similar reason as (C), (D) is also not true. remaining 5 letters can be arranged in
Answers: (A), (B) 5!
= 60 ways
2!
3. x is one among n distinct objects and 1 £ r £ n an (D) Consider all the four as a single letter, so that among
integer. Then which of the following are true? six objects, 2 are alike. Therefore, the number of such
(A) The number of permutations of r objects that arrangements is
involve the object x is r ! ´ n - 1 Pr - 1 .
6!
(B) The number of permutations of r-objects that do = 360
not involve the object x is n -1 Pr . 2!
(C) n Pr = (n - 1)Pr + r ´ n - 1 Pr. Answers: (A), (B), (C), (D)
(D) The number of permutations of r-objects that
involve x is r ´ n - 1 Pr - 1. 5. Consider the letters of the word INTERMEDIATE.
Which of the following is (are) true?
Solution: A permutation involving x implies that x is
(A) The number of words formed by using all the
in one of the r places. The remaining r – 1 places are to be
letters of the given word is (12 !)/(3! 2 !).
filled with n – 1 objects. This can be done in n - 1 Pr - 1 ways.
(B) The number of words which begin with I and end
Therefore, the total number of permutations of r objects
with E is (10 !)/(2 ! 2 !).
involving x is r ´ n - 1 Pr - 1. Therefore (D) is correct and
(C) The number of words in which all the vowels
(A) is not correct.
come together is (7 ! × 6 !)/(3! 2 ! 2 !).
The permutations of r-objects not involving x is n -1 Pr ,
(D) The number of words in which no two vowels
since r places have to filled with objects of (n – 1)-element
come together is 360 ´ 420.
set. Therefore (B) is correct. Also,
n-1 n-1
Solution: The given word consists of 12 letters in which
n
Pr = Pr + (r ´ Pr - 1 ) there are 3 Es, 2 Is and 2 Ts and the remaining 5 are distinct.
and hence (C) is correct. (A) The number of words using all the letters is
Answers: (B), (C), (D)
(12)!
4. Consider the word ALLAHABAD. Which of the
3! 2 ! 2 !
following statements are true? Therefore (A) is false.
www.jeeneetbooks.in
Worked-Out Problems 307

(B) Put I in the first place and E in the last place. In after that the remaining 4 letters can be arranged in 4!
the remaining 10 letters, there are 2 Es and 2 Ts. ways. Therefore, the required number is 3! ´ 4 ! = 144.
Therefore the number of such words is (C) As in (B), the required number is 4P3 ´ 4! = 4! ´
(10)! 4! = 576.
2! 2! (D) With I in the middle, the remaining 6 letters can be
arranged in 6 ! = 720 ways.
Therefore (B) is true. Answers: (A), (B), (C) and (D)
(C) Treat all the vowels as a single object (letter). In
the remaining six letters, there are 2 Ts. Now, the 7. If C r - 1 = 36, C r = 84 and C r + 1 = 126, then
n n n

7 let ters can be arranged in 7 !/ 2 ! ways. But the


(A) n = 8 (B) r = 3 (C) n = 9 (D) r = 4
vowels (3 Es, 2 Is and 1 A) can be arranged among
themselves in Solution: We have
6! n
Cr n - r + 1 84
ways = =
3! 2 ! n
Cr -1 r 36
Therefore the number of such words is
n-r+1 7
7! 6! = (6.1)
´ r 3
2 ! 3! 2 !
3n - 10r = -3
Therefore (C) is true.
Again
(D) Among the six consonants, there are seven gaps in
which the vowels can be arranged in n
C r + 1 n - r 126
= = (6.2)
7
P6
n
Cr r + 1 84
ways
3! 2 ! 3 n-r
=
The consonants can be arranged in 2 r+1
6! 2 n - 5r = 3
ways
2!
Solving Eqs. (6.1) and (6.2), we get n = 9 and r = 3.
Therefore the number of words in which no two Answers: (B) and (C)
vowels come together is
8. If Pr = Pr +1 and C r = C r -1, then
n n n n
7
P6 6!
´ = 420 ´ 360 (A) n = 3 (B) r = 1 (C) r = 2 (D) n = 4
3! 2 ! 2 !
Answers: (B), (C), (D) Solution: We have
n
Pr = n Pr +1
6. The letters of the word ARTICLE are arranged in
all possible ways. Then which of the following is n! n! (6.3)
(are) true? =
(n - r )! (n - r - 1)!
(A) Number of words formed by using all the letters
is 5040. n-r = 1
(B) The number of words with vowels in even places n
C r = n C r -1
is 144.
(C) The number of words with vowels in odd places n! n!
= (6.4)
is 576. r ! ´ (n - r )! (r - 1)!(n - r + 1)!
(D) The number of words with I in the middle is 720.
1 1
Solution: =
r n-r+1
(A) Seven different objects can be arranged in 7! =
5040 ways. 2r - 1 = n
(B) There are 3 even places and 3 vowels. Therefore the Solving Eqs. (6.3) and (6.4), we get r = 2 and n = 3.
vowels can be arranged in even places in 3!, ways and Answers: (A) and (C)
www.jeeneetbooks.in
308 Chapter 6 Permutations and Combinations

Matrix-Match Type Questions


1. A total of 6 boys and 6 girls are to be arranged in a row. (B) Total number of even numbers that (q) 10
Certain stipulations on their arrangements are given can be formed is
(r) 13442
in Column I and the number of such arrangements is (C) Total number of odd numbers that
given in Column II. Match the items in Column I with can be formed is (s) 18
those in Column II. (D) The sum of all the four-digit numbers is (t) 13440

Column I Column II Solution:


(A) The number of arrangements in (p) 7 ! ´ 6 ! (A) The total number of four-digit numbers that can be
which all the girls are together is formed is given by
(B) The number of arrangements in (q) 6 ! ´ 6 ! ´ 7 (No. of arrangements of 0, 3, 4 and 5) – (No. of
which no two girls are together is arrangements with 0 in the left end)
(C) The number of arrangements in which (r) 2 ´ 6 ! ´ 6 ! 4 ! - 3! = 18
boys and girls come alternately is
Answer: (A) Æ (s)
(D) The number of arrangements (s) (10)!
(B) A number among these is even if 0 or 4 is in the units
in which the first place is to be
place. The number of even numbers
occupied by a specified girl and
the last place by a specified boy is (i) with 0 in the units place = 3! = 6
(ii) with 4 in the units place = 3! - 2 ! = 4
Solution: Therefore the total number of even numbers
(A) Consider all the 6 girls as a single block so that there are = 6 + 4 = 10.
6 + 1 = 7 objects which can be arranged in 7! ways. In
Answer: (B) Æ (q)
the block of girls, 6 can be arranged among themselves
in 6! ways. Therefore the required arrangements are (C) No. of odd numbers = Total No. – No. of even numbers

7 ! ´ 6 ! or 6 ! ´ 6 ! ´ 7 = 18 - 10 = 8
Answer: (A) Æ (p) Answer: (C) Æ (p)
(B) Since no two girls should come together, arrange the 6 (D) Contribution of 0 to the sum = 100 + 10 + 0 = 110
girls in 7 gaps (including before the boys and after the Contribution of 3 to the sum = 3000 + 300 + 30 + 3 = 3333
boys) which can be done in 7 P6 ways. After arranging Contribution of 4 to the sum = 4000 + 400 + 40 + 4 = 4444
the girls, the 6 boys can be arranged among themselves Contribution of 5 to the sum = 5000 + 500 + 50 + 5 = 5555
in6! ways. Therefore, the required arrangements are The sum of all the numbers =
P6 ´ 6 ! = 7 ! ´ 6 ! = 6 ! ´ 6 ! ´ 7
7
110 + 3333 + 4444 + 5555 = 110 + 1111(3 + 4 + 5)
Answer: (B) Æ (p), (q) = 110 + 13332 = 13442
(C) Since the boys and girls come alternately, the
Answer: (D) Æ (r)
arrangement may begin with a boy or a girl as BG
BG BG BG BG BG or GB GB GB GB GB GB. 3. In Column I the types of distributions of playing cards
Number of such arrangements is 2 ´ 6 ! ´ 6 !. and, in Column II, their corresponding number of
Answer: (C) Æ (r) distributions is given. Match the items in Column I
(D) Put the specified girl in the first place and the with those in Column II.
specified boy in the last place. The remaining
10(5 + 5) can be arranged in (10)! ways. Column I Column II
Answer: (D) Æ (s) (A) 52 playing cards are to be equally (52)!
distributed among four players. The (p)
[(13)!]4
2. Four-digit numbers, without repetition of digits, are number of possible distributions is
formed using the digits 0, 3, 4, 5. Certain stipulations (52)!
(B) 52 cards are to be divided into (q)
on arrangements are given in Column I and their 4 ![(13)!)]4
four equal groups
numbers are given in Column II. Match these.
(C) 52 cards are to be divided into 4 (52)!
sets, three of them having 17 cards (r)
Column I Column II 3![(17)!]
each and the 4th has just one card
(A) Total number of four-digit numbers (p) 8 (D) 52 cards are to be divided equally (52)!
(s)
that can be formed is into two sets 2 !((26)!)2
www.jeeneetbooks.in
Worked-Out Problems 309

Solution: (B) The number of ways of forming the committee with


(A) The required number of distributions is atleast 5 women is

(52)! (39)! (9C 5 ´ 8 C 7 ) + (9 C 6 ´ 8 C 6 ) + (9 C 7 ´ 8 C 5 )


52
C 13 ´ C 13 ´ C 13 ´ C 13 =
39 26 13
´
(13)!(52 - 13)! (13)!(39 - 13)! + (9 C 8 ´ 8 C 4 ) + (9 C 9 ´ 8 C 3 )

´
(26)!
´1 æ 9 × 8 × 7 × 6 ö æ 9 × 8 × 7 8 × 7ö æ 9 × 8 8 × 7 × 6ö
=ç ´ 8÷ + ç ´ ÷ +ç ´ ÷
(13)!(26 - 13)! è 4! ø è 3! 2! ø è 2! 3! ø
(52)! æ 8 × 7 × 6 × 5ö æ 8 × 7 × 6ö
= + ç9 ´ ÷ + ç1´ ÷
[(13)!]4 è 4! ø è 3! ø
Answer: (A) Æ (p) = 1008 + 2352 + 2016 + 630 + 56 = 6062
(B) To be divided into 4 equal groups. Elements of one Answer: (B) Æ (r)
group can be exchanged with another. This is possible (C) The number of ways of forming the committee with
in 4! ways. Therefore, the number of divisions is women in majority
(52)! (9C 7 ´ 8 C 5 ) + (9 C 8 ´ 8 C 4 ) + (9 C 9 ´ 8 C 3 )
4 ![(13)!]4
æ 9 × 8 8 × 7 × 6ö æ 8 × 7 × 6 × 5ö æ 8 × 7 × 6ö
Answer: (B) Æ (q) =ç ´ ÷ + ç9 ´ ÷ + ç1´ ÷
è 2! 3! ø è 4! ø è 3! ø
(C) Three of them get 17 each. Again cards can be
= 2016 + 630 + 56 = 2702
exchanged among these 3 in 3! ways. Therefore, the
number of divisions is Answer: (C) Æ (q)
(52)! (D) The number of ways of forming the committee with
3![(17)!]3 atleast 5 women and with men in majority

Answer: (C) Æ (r) 9×8×7×6


C5 ´ 8C7 =
9
´ 8 = 1008
4!
(D) The number of distributions is
Answer: (D) Æ (p)
52
C 26 × 26 C 26 (52)!
=
2! 2 ![(26)!]2 5. A total of 11 players are to be selected for a cricket
Answer: (D) Æ (s) match from a cricket squad consisting of 6 specialist
batsmen, 3 all rounders, 6 specialist bowlers and
4. A committee of 12 members is to be formed from 2 wicketkeepers (who can also bat well). Match the
9 women and 8 men. Match the statements in Column I items in Column I with those in Column II.
with the numbers in Column II.
Column I Column II
Column I Column II
(A) The number of selections which (p) 600
(A) The number of ways of forming the (p) 1008 contain 4 specialist batsmen, 3 all
committee with 6 men and 6 women rounders, 3 specialist bowlers and a
(B) The number of ways of forming the (q) 2702 wicketkeeper
committee with atleast 5 women (B) The number of selections which contain (q) 720
(C) The number of ways of forming the (r) 6062 5 specialist batsman, 2 all rounders, 3
committee with women in majority specialist bowlers and a wicketkeeper
(D) The number of ways of forming the (s) 2352 (C) The number of selections which (r) 675
committee with atleast 5 women and contain 4 specialist batsman, 1 all
with men in majority rounder, 4 specialist bowlers and
2 wicketkeepers
Solution: (D) The number of selections which (s) 900
(A) The number of ways of forming the committee with contain 4 specialist batsmen, 2 all
6 men and 6 women from 9 women and 8 men is rounders, 3 specialist bowlers and
2 wicketkeepers
9×8×7 8×7
C6 ´ 8C6 =
9
´ = 84 ´ 28 = 2352
3! 2! Solution: Consider the table on the next page.
Answer: (A) Æ (s) Answer: (A) Æ (p); (B) Æ (q); (C) Æ (r); (D) Æ (s)
www.jeeneetbooks.in
310 Chapter 6 Permutations and Combinations

Item in Specialist Specialist


column I batsmen (6) All rounders (3) bowlers (6) Wicketkeepers (2) Number of selections
(A) 4 3 3 1
6
C 4 ´ 3C 3 ´ 6 C 3 ´ 2 C1 = 600 (p)
(B) 5 2 3 1
6
C 5 ´ 3C 2 ´ 6 C 3 ´ 2 C 1 = 720 (q)
(C) 4 1 4 2
6
C 4 ´ 3C 1 ´ 6 C 4 ´ 2 C 2 = 675 (r)
(D) 4 2 3 2
6
C 4 ´ 3C 2 ´ 6 C 3 ´ 2 C 2 = 900 (s)

6. A 17 member hockey squad contains 4 peculiar


Column I Column II
players A, B, C and D. Players A and B wish to play
together or be out of the team together. Players C (A) There are 12 points in a plane out (p) 1296
and D are such that if one plays the other does not of which 5 are collinear and no 3 of
want to play. A team of 11 players is to be selected the remaining are collinear. Then
from the squad. Match the items in Column I with the number of lines that can be
those in Column II. formed by joining pairs of these
points is
Column I Column II (B) The number of triangles that can (q) 57
be formed by using the points
(A) No. of selections including A and B (p) 13 C 9 mentioned above is
and one of C, D is (C) The number of rectangles that can (r) 420
(B) No. of selections including A and B (q) 13 C 11 be formed by using the squares in a
and excluding both C and D is chess board is
(C) No. of selections excluding A and (r) 13 C 8 ´ 2 (D) A set of 8 parallel lines are (s) 210
B and including one of C and D is intersected by another set of 6
(D) No. of selections excluding all of (s) 2 ´ 13C 10 parallel lines. Then the number of
A, B, C and D is parallelograms thus formed is

Solution: Solution:
(A) In addition to A, B and C, 8 more are to be selected (A) The five collinear points give us one straight line.
from out of 13 (other than A, B, C and D) in Therefore the required number is
13
C 8 ways. Similarly, in addition to A, B and D,
12 × 11 5 × 4
another 13 C 8 ways. Therefore, the required number 12
C 2 - 5C 2 + 1 = - + 1 = 57
is 2 ´ 13C 8 . 2 2
Answer: (A) Æ (r) Answer: (A) Æ (q)
(B) In addition to A and B, 9 more are to be selected (B) A triangle is formed with three non-collinear points.
from among of 13 (other than A, B, C and D) in Therefore the number of triangle that can be formed is
13
C 9 ways. 12
C 3 - 5C 3 = 210
Answer: (B) Æ (p)
Answer: (B) Æ (s)
(C) 10 players are to be selected, in addition to C, from
(C) A chess board consists of 9 horizontal and 9 vertical
among 13 (other than A, B, C and D). This can be
lines. To form a rectangle (it may be a square) we need
done in 13 C 10 ways. Similarly, selections including D
2 horizontal and 2 vertical lines. Therefore the
can be made in 13 C 10 ways. Therefore the required no. number of rectangles is
is 2 ´ 13C 10 .
Answer: (C) Æ (s) C 2 ´ 9 C 2 = 36 ´ 36 = 1296
9

(D) The number of selections of 11 persons from among Answer: (C) Æ (p)
13 persons (other than A, B, C and D) is 13 C 11. (D) We select 2 from 8-lines set and 2 from 6-lines set.
Answer: (D) Æ (q) Therefore the number of parallelograms is

7. Match the statements in Column I with the numbers


8
C 2 ´ 6 C 2 = 28 ´ 15 = 420
given in Column II. Answer: (D) Æ (r)
www.jeeneetbooks.in
Worked-Out Problems 311

Comprehension-Type Questions
1. Passage: 4 Indians, 3 Americans and 2 Britishers are (ii) The number of ways in which all the four prizes can
to be arranged around a round table. Answer the be given to any one of the 6 students = 6. Therefore
following questions. the required number of ways is 6 4 - 6 = 1290.
(i) The number of ways of arranging them is Answer: (B)
1 1 (iii) Give a set of two prizes to the particular student.
(A) 9! (B) 9! (C) 8! (D) 8!
2 2 Then the remaining 2 can be distributed among
5 students in 52 ways. There are 4 C 2 sets, each
(ii) The number of ways arranging them so that the
containing 2 prizes. Therefore the required number
two Britishers should never come together is
of ways of distributing the prizes is
(A) 7 ! ´ 2 ! (B) 6 ! ´ 2 ! (C) 7! (D) 6 ! 6 P2
(iii) The number of ways of arranging them so that 52 ´ 4C 2 = 25 ´ 6 = 150
the three Americans should sit together is Answer: (C)
(A) 7 ! ´ 3! (B) 6 ! ´ 3! (C) 6 ! 6 P3 (D) 6 ! 7 P3
3. Passage: A security of 12 persons is to form from a
Solution: group of 20 persons. Answer the following questions.
(i) n distinct objects can be arranged around a circular (i) The number of times that two particular persons
table in (n - 1)! ways. Therefore the number of ways are together on duty is
of arranging 4 + 3 + 2 people = 8!.
20 ! 18 ! 20 ! 20 !
Answer: (C) (A) (B) (C) (D)
12 ! 8 ! 10 ! 8 ! 10 ! 8 ! 10 ! 10 !
(ii) First arrange 4 Indians and 3 Americans around a
round table in 6! ways. Among the six gaps, arrange (ii) The number of times that three particular
the two Britishers in 6 P2 ways. Therefore the total persons are together on duty is
number of arrangements in which Britishers are 17 ! 17 ! 20 ! 20 !
(A) (B) (C) (D)
separated is 6 ! ´ 6 P2 . 8! 9! 8! 8! 17 ! 3! 9! 8!
Answer: (D) (iii) The number of ways of selecting 12 guards such
(iii) Treating the 3 Americans as a single object, 7 (= 4 + that two particular guards are out of duty and
1 + 2) objects can be arranged cyclically in 6! ways. three particular guards are together on duty is
In each of these, Americans can be arranged among (20)! (18)! (15)! (15)!
themselves in 3! ways. Therefore, the number of (A) (B) (C) (D)
(15)! 5! 9 ! 3! 9! 6! 5! (10)!
required arrangements is 6 ! ´ 3!.
Answer: (B) Solution:
(i) Let A and B two particular guards who want to be
2. Passage: 4 prizes are to be distributed among in duty together. We can select 10 more from the
6 students. Answer the following three questions. remaining 18 persons in 18 C 10 ways. Therefore the
required number is
(i) The number of ways of distributing the prizes, if
a student can receive any number of prizes, is (18)!
(A) 1296 (B) 163 (C) 15 (D) 30 (10)! 8 !
(ii) The number of ways of distributing the prizes, if Answer: (B)
a student cannot receive all the prizes, is
(ii) In addition to the three particular persons who want
(A) 163 – 16 (B) 1290 (C) 11 (D) 26 to be in duty together, we can select 9 more from
(iii) If a particular student is to receive exactly 2 prizes, the remaining 17 persons in 17 C 9 ways. Therefore,
then the number of ways of distributing the prizes is the required number is
(A) 25 (B) 32 (C) 150 (D) 36 17 !
Solution: 8! 9!
(i) Let the prizes be P1, P2, P3 and P4. P1 can be given Answer: (A)
to any one of the 6 students and so are P2, P3 and P4. (iii) Let A and B be out of duty and P, Q, R be
Therefore the number of distributions is 6 4 = 1296. particular persons who want to be in duty together.
Answer: (A) Then, we can choose 9 more from among the
www.jeeneetbooks.in
312 Chapter 6 Permutations and Combinations

remaining 15 persons (excluding A and B) in 15 C 9 8!


ways. Therefore, the required number is (since there are two l)
2!
(15)! In each of these arrangements, vowels may occur in
9! 6! 3! ways. Therefore the number of arrangements in
Answer: (C) which u, i, e occur in this order is

8! 1
4. Passage: The letters of the word MULTIPLE are ´ = 3360
2 ! 3!
arranged in all possible ways. Answer the following
three questions. Answer: (D)
(i) The number of arrangements in which the order (ii) Keeping u, i, e in their respective places, the number
of the vowels does not change is of arrangements is
(A) 3330 (B) 3320 (C) 3340 (D) 3360
5!
(ii) The number of arrangements in which the = 60
2!
vowels’ positions are not disturbed is
(A) 60 (B) 260 (C) 160 (D) 320 Answer: (A)
(iii) The number of arrangements in which the relative (iii) Keeping the relative positions of vowels and
order of vowels and consonants is not disturbed is consonants means, the vowels can be interchanged
among themselves and so can the consonants.
(A) 460 (B) 420 (C) 360 (D) 440
Therefore the number of required arrangements is
Solution:
(i) The order of vowels does not change means, first u 5!
´ 3! = 60 ´ 6 = 360
occurs, then i and then e must occur. The total number 2!
of arrangements is Answer: (C)

SUMMARY
6.1 The symbol n! (Factorial n): 0! = 1. If n is a positive 6.7 Symbol nPr : If n is a positive integer and 0 ≤ r ≤ n is
integer, the n! means the number n(n - 1) (n - 2) 2·1. an integer, then n Pr denotes the number of permuta-
One can note that n! = n(n - 1)! tions of n distinct objects taken r at a time without
repetitions and this
Permutations n!
n
Pr = n(n - 1) (n - 2) (n - r + 1) =
(n - r )!
6.2 Permutation: Arrangement of objects on a line is
called linear permutation. 6.8 Useful formulae:

n!
(1) Pr =
n
6.3 Circular permutation: Arrangement of objects in a
circular form.
( n - r )!
(2) n Pr = n × ( n- 1) P( r - 1)
6.4 Permutation as a bijection: If X is a finite set,
(3) n Pr = ( n- 1) Pr + r × ( n- 1) Pr - 1
then any bijection from X onto X is a permuta-
tion. That is arrangement of n distinct objects
6.9 Permutations with repetitions:
taken all at a time.
(1) The number of permutations of n dissimilar
6.5 Theorem: The number of arrangements of n distinct things taken r at time, when repetition of objects
objects taken all at a time is n!. is allowed any number of times is nr.
(2) Total number of permutations of n dissimilar objects
6.6 Theorem: The number of permutations of n objects taken r at a time with atleast one repetition is nr - nPr .
taken r at a time (0 ≤ r ≤ n) is
6.10 Circular permutations: The number of circular
n! permutations of n dissimilar things is (n - 1)!. This
n(n - 1) (n - 2) (n - r + 1) =
(n - r )! number includes both anticlockwise and clockwise
www.jeeneetbooks.in
Summary 313

senses. If the sense is not considered, then the 6.17 Number of divisons: If 1 < n = p1a 1 × p2a2 pKaK is
number is a positive integer where p1, p2, …, pK are distinct
(n - 1)! prime numbers, then the number of positive divi-
sions of n is (a1 + 1) (a2 + 1) (aK + 1).
2
Note that this number includes both 1 and n.
QUICK LOOK 6.18 Writing a positive integer as a sum of (atleast
(1) For live objects the sense will be considered. two) positive integers considering the same set of
(2) For non-live objects the sense will not be considered. integers in a different order being different is 2 n- 1 - 1.

6.19 Useful results (on integer solutions):


6.11 Permutations with alike objects: (1) Let m and n be positive integers such that
(1) Out of n objects, suppose p objects are alike and m ≤ n. Then the number of m-tuples (x1, x2, …,
the rest are distinct. Then, the number of permu- xm) of positive integers satisfying the equation
tations of the n objects taken all at a time is n !/p!. x1 + x2 + + xm = n is (n -1)C(m-1).
(2) Suppose n1, n2, …, nK are number of alike objects (2) The number of m-tuples (x1, x2, …, xm) of
of different kinds. Then the number of permuta- non-negative integers satisfying the equation
tions of all these objects is x1 + x2 + + xm = n is ( n + m- 1) C ( m- 1) .
(n1 + n2 + + nK )!
6.20 Plane divided by lines: The maximum number of
n1 ! n2 ! nK !
parts into which a plane is divided by n lines is
Combinations n2 + n + 2
2
6.12 Combination: Selection of objects.

æ nö 6.21 Number of injections and bijections: Let X and Y


6.13 Symbol n Cr or ç ÷ : The number of combinations
èrø be non-empty finite sets having m elements and n
of n distinct objects taken r(0 ≤ r ≤ n) at a time is elements, respectively. Then
denoted by n C r . (1) The number of functions (mappings) from Y to
n
X is mn.
6.14 Value of Cr :
(2) The number of injections from Y to X is zero if
n
n
P n ( n - 1) ( n - 2 ) ( n - r + 1) m < n and m Pn if m ≥ n.
Cr = r =
r! r! (3) The number of bijections from Y to X is zero if
n! m < n and m! if m = n.
=
r ! ( n - r )!
6.22 Number of surjections:
n
C0 = nC n = 1
(1) Recursive formula: For any positive integers
m ≥ r > 0, the number a m (r ) of surjections from
QUICK LOOK an m-element set onto an r-element set is given
by a recursive formula
n
Cr = nC n-r
r -1
a m (r ) = r m - å r C s a m ( s)
6.15 Useful tips: s =1

(1) C r = C s Þ Either r = s or r + s = n.
n n
(2) Direct formula:
( n + 1)
(2) C r + C r - 1 =
n n
C r. r -1

n ´ ( n- 1)
a m (r ) = å (-1)s r C( r - s ) (r - s)m
(3) r ´ n C r = C r- 1. s=0

6.16 Combinations with alike objects: If p1, p2, …, pK are 6.23 Derangement: Let X be a non-empty set and
number of alike objects of different kinds, then f : X ® X is a bijection (also called permutation),
the number of all selections (with one or more) is such that f(x) ≠ x for all x Î x. Then f is called
( p1 + 1) ( p2 + 1) … ( pK + 1) - 1. derangement of X.
www.jeeneetbooks.in
314 Chapter 6 Permutations and Combinations

6.24 Number of derangements: 6.25 Set divided into groups:


(1) Let n be a positive integer and 0 ≤ r ≤ n. Let (1) If a set contains m + n (m ≠ n) elements, then the
dr denote the number of Derangements of an number of ways the set can be divided into two
r-element set with d0 = 1. Then groups containing m elements and n elements
n n
respectably is
å
r =0
n
C r dr = n ! and dn = n !- å n C r dr
r =0
( m + n )!
m! n !
(2) Direct formula: The number dn of the number of
(2) If m = n, then the number of divisions into two
Derangements of an n-element set is given by
equal groups is
n
(-1) K
( 2 n )!
dn = (n !) å
K =0 K! (n! n!) 2 !

EXERCISES
Single Correct Choice Type Questions
1. The number of different 6 letters words that can be 7. The number of pairs of words (x, y), x containing
formed by using 5 distinct consonants, 4 vowels and 3 4 letters and y containing 3 letters, from the letters of
capital letters (distinct) which begin with a capital letter, the word STATICS is
but consisting of 1 capital, 3 consonants and 2 vowels is (A) 1260 (B) 396 (C) 829 (D) 796
(A) 21500 (B) 21600 (C) 20600 (D) 20500
8. Let n ³ 3 and A be a set of n elements. Let P = (x, y,
2. The number of ways of arranging the letters of the z) where x, y, z Î A. Then the number of points P such
word SINGLETON excluding the given word is that atleast two of x, y, z are equal is
(A) 9! (B) 9! – 1 (A) 3n 2 - 2 n + 1 (B) n 3
1 1 n
(C) P3 (D) n 3 - (n P3 )
(C) (9 !) - 1 (D) (9 ! - 1)
2 2
9. Let A = {1, 2, 3, … , n}. The number of bijections f
3. Let A = {a | a is a prime number and a < 31}. The from A onto A for which f (1) ¹ 1 is
number of rational numbers of the form a / b, where a (A) n! - n (B) n! - (n-1)!
and b Î A and a ¹ b, is (C) n2 - n (D) (n-1)!
(A) 180 (B) 92 (C) 91 (D) 90
10. The number of triangles whose vertices are the
4. Let A be the set of all four-digit numbers of the form vertices of a polygon of n sides but whose sides are
x1 x2 x3 x4 where x1 > x2 > x3 > x4 and each xi may not the sides of the polygon is
take the values from 1 to 9. Then n(A) is n n
(A) (n - 4)(n - 5) (B) (n - 3)(n - 4)
(A) 126 (B) 84 (C) 210 (D) 64 6 6
n n
5. The numerals 1, 2, 3, … , 9 are arranged in all possible (C) (n - 1)(n - 2) (D) (n - 2)(n - 3)
6 6
ways such that the digit in the middle is greater than
all its preceding digits and less than all its succeeding 11. Ten lines are given in a plane such that no two are
digits. The number of such arrangements is parallel and no three are concurrent. Then the number
(A) 24 (B) 120 (C) 360 (D) 576 of regions into which the plane is divided by these lines is
(A) 56 (B) 66 (C) 46 (D) 99
6. There are three coplanar lines. On each of these lines,
p number of points are taken. The maximum number 12. The sum of all four-digit numbers (without repeti-
of triangles that can be formed with vertices at these tion) that can be formed by using the numerals 2, 3,
points is 4 are 5 is
(A) 3 p2 ( p - 1) (B) 3 p2 ( p - 1) + 1 (A) 93234 (B) 49332
(C) p2 (4 p - 3) (D) 3 p2 (4 p - 3) (C) 93324 (D) 94332
www.jeeneetbooks.in
Exercises 315

13. The number of different nine-digit numbers that can 15. If r, s, t are prime numbers and p, q are two positive
be formed from the number 223355888 such that integers such that the LCM of p, q is r 2 s 4 t 2 , then the
even digits occupy odd places is number of ordered pairs ( p, q) is
(A) 16 (B) 36 (C) 60 (D) 180 (A) 225 (B) 224 (C) 248 (D) 255

14. A, B are two speakers along with three more to 16. The number of derangements of a four element set is
address a public meeting. If B addresses immedi- (A) 8 (B) 9 (C) 10 (D) 12
ately after A, the number of ways of arranging the
list is 17. The number of surjections from a five-element set
(A) 24 (B) 36 (C) 48 (D) 30 on to a four-element set is
(A) 340 (B) 220 (C) 320 (D) 240

Multiple Correct Choice Type Questions


1. Consider n points in a plane of which only p points are 6. A total of 5 mathematics books, 4 physics books and
collinear. Then the number of straight lines that can be 2 chemistry books are to be arranged in a row in a
drawn by joining these points is book shelf. Which of the following is/are true?
(A) ( n - p)C 2 + p(n - p) + 1 (B) n C 2 (A) The number of arrangements that two chemistry
(C) n C 2 - p C 2 + 1 (D) n C 2 - p C 2 books are separated is 9 ´ 10!
(B) The number of arrangements in which four
2. Let f ( x) = ( 7 - x ) P( x - 3) . Then physics books are together is 8! 4!
(A) The domain of f is {3, 4, 5} (C) The number of arrangements in which no two
(B) Range of f is {2, 3, 24} mathematics books are together is (7·6) (6!)
(C) The domain of f is {3, 4, 5, 6} (D) The number of arrangements in which the books
(D) f ( x) is one-one on the same subject are all together is 12 (4! 5!)

3. Consider the letters of the word TATANAGAR. 7. If C r + 1 = 13 C 3r - 5 , then


13

Which of the following is/are true? (A) r = 4 (B) r = 3 (C) r = 9 (D) rC2 = 3
(A) The number of arrangements of all the letters is 7560
8. Let x = 2 × 3 . Which of the following is/are true?
4 4
(B) The number of words that begin with N is 840
(C) The number of five letter words in which no letter (A) The number of proper divisors of x is 23
is repeated is 120 (B) The sum of all positive divisors of x is 31 ´ 112
(D) The number of words that can be formed using all (C) The sum of all divisors of x is 112
the letters without changing the position of N is 840 (D) 6 4 - 1 is divisible by 5

4. Let n be a positive integer and r an integer such that 9. Consider the word VARANASI. Which of the
0 £ r £ n. Then following is/are true?
(A) n Pr = n ´ ( n - 1) P( r - 1) (A) The number of words that can be formed using
(B) n Pr = r ! ´ n C r all the letters is 6720
(C) n Pr = ( n - 1) Pr + r ´ ( n - 1) P( r - 1) (B) The number of words without disturbing the three
(D) The number of permutations of n distinct objects A’s is 120
taken r at a time with atleast one repetition is nr - n Pr (C) The number of words such that all the three A’s
together is 720
5. Certain 5-digit numbers are formed by using the numerals (D) The number of words which begin with A and
0, 1, 2, 3, … , 9. Which of the following is/are true? end with A is 720
(A) The total number of numbers without using 0 in
the first place from left and using any numeral any 10. Which of the following is (are) correct?
number of times is 9 ´ (10)4 (A) The number of diagonals of a 10-gon is 35
(B) Total number of numbers without repetitions is 10P5 - 9P4 (B) The number of points of intersection of the diago-
(C) Total number of numbers with atleast one repeated nals of an octagon which lie inside the octagon is 70
digit is 62784 (C) If n Pr = n C r , then r = 1 or 0
(D) If repetitions are allowed, the number of 5-digit (D) The maximum number of points in which 8 lines
numbers not containing 0 is 9 5 intersect 4 circles in the same plane is 64
www.jeeneetbooks.in
316 Chapter 6 Permutations and Combinations

Matrix-Match Type Questions


In each of the following questions, statements are given (D) The number of words that can (t) 120660
in two columns, which have to be matched. The state- be formed from the letters of the
ments in Column I are labeled as (A), (B), (C) and word MANESHPURI with vowels
(D), while those in Column II are labeled as (p), (q), together is
(r), (s) and (t). Any given statement in Column I can
have correct matching with one or more statements in
3. Certain requirements of arranging the letters of
Column II. The appropriate bubbles corresponding to
word ARRANGE are given in Column I and their
the answers to these questions have to be darkened as
respective number of arrangements are given in
illustrated in the following example.
Column II. Match the items in Column I with those in
Example: If the correct matches are (A) ® (p),(s); Column II.
(B) ® (q),(s),(t); (C) ® (r); (D) ®(r),(t); that is if
the matches are (A) ® (p) and (s); (B) ® (q),(s) and (t);
(C) ® (r); and (D) ® (r),(t), then the correct darkening Column I Column II
of bubbles will look as follows: (A) Two Rs are never together (p) 900
p q r s t (B) Two As are together, but Rs are (q) 240
A separated
B (C) Neither two Rs nor two As are (r) 660
together
C
(D) Rs in the first and last places, but A (s) 24
D
is in the middle place

1. In Column I, certain types of arrangements of the


letters of the word ORDINATE are given. Column II 4. Match the items in Column I with those in Column II.
contains number of arrangements. Match the items in
Column I with those in Column II. Column I Column II
(A) Out of 8 sailors on a boat, 3 can (11)!
Column I Column II (p) (9 !)2
work at row side only and 2 can 5! 6 !
(A) The number of words with I in the (p) 576 work at bow side only. The number
fourth place of arrangements of the sailors, if
(q) 676 each side accommodates 4 sailors
(B) The number of words with vowels
only, is
occupying odd places
(r) 5040 (B) 18 guests have to be seated, half (q) 4356
(C) The number of words with
on each side of a long table. Four
consonants in the odd places (s) 720 particular guests desire to sit on
(D) The number of words beginning with one particular side and three on
O and ending with E (t) 5050 the other side. The number of
seating arrangements is
2. Match the items in Column I with those in Column II.
(C) ABCD is a parallelogram. Ten (r) 15
lines each are drawn parallel to
Column I Column II AB and BC intersecting the sides.
(A) Total number of arrangements of the (p) 1120 The number of parallelograms
letters a 2 b3 c 4 written in full length is that are formed is
(B) Six-digit numbers are to form using (D) In a chess tournament, each (s) 1728
(q) 120960 player should play one game
the numerals 1, 2, 3, 4. If all the
numerals appear atleast once in the with each of the others. Two
same number, then the number of (r) 1260 players left the tournament
such number is on personal reasons having
played 3 games each. If the total
(C) The number of words that can be (s) 1560
number of games played is 84,
formed using all the letters of the
the number of participants in the
word MISSISSIPI which begin with
beginning of the tournament is
I and end with S is
www.jeeneetbooks.in
Exercises 317

Comprehension-Type Questions
1. Passage: Consider the digits 1, 2, 3, 4, 5 and 6. Answer 2. Passage: The letters of the word EAMCET are
the following three questions. arranged in all possible ways. Answer the following
(i) The number of four-digit numbers, allowing three questions.
repetition of digits any number of times, is (i) The number of words that can be formed, without
(A) 1296 (B) 4096 (C) 3096 (D) 2096 disturbing the places of E, is
(ii) When repetitions are allowed, the number of (A) 120 (B) 24 (C) 48 (D) 720
four-digit even numbers is (ii) The number of words that can be formed without
(A) 448 (B) 216 (C) 1296 (D) 648 separating the two Es is
(iii) When repetitions are allowed, the number of (A) 120 (B) 240 (C) 24 (D) 360
four-digit numbers, that are divisible by 3, is (iii) If all possible words are written as in the
(A) 632 (B) 532 (C) 432 (D) 332 dictionary, the rank of the word EAMCET is
(A) 134 (B) 135 (C) 132 (D) 133

Assertion–Reasoning Type Questions


Statement I and Statement II are given in each of the 4. Statement I: Let x1, x2 , … , xn be a permutation of
questions in this section. Your answers should be as per the natural numbers 1, 2, … , n. If n is odd, then the
the following pattern: product ( x1 - 1) ( x2 - 2) … ( xn - n) is even.
(A) If both Statements I and II are correct and II is a
Statement II: å i = 1 ( xi - i) = 0
n
correct reason for I
(B) If both Statements I and II are correct and II is not a
correct reason for I 5. Statement I: If n ³ 1 is an integer, then (n 2 )!/(n 2 )!n is
(C) If Statement I is correct and Statement II is false an integer.
(D) If Statement I is false and Statement II is correct.
Statement II: mn objects can be divided among n
persons in (mn)!/(m!)n ways.
1. Statement I: The number of words that can be formed
using all the letters of the word ASSASSINATION is
69300. 6. Statement I: Consider n straight lines in a plane
of which no two are parallel and no three are
Statement II: There are m1 similar objects of one kind, concurrent. Then the number of new lines that can
m2 similar objects of another kind, , mk similar objects be formed by joining the points of intersection of
of different kind. The total number of arrangements of these n lines is
all these objects is
1
(m1 + m2 + + mK )! (n - 3)(n - 2)(n - 1)n
8
m1 ! m2 ! mK !
Statement II: Two coplanar non-parallel lines inter-
2. Statement I: A and B are two speakers to address a sect in a point.
public meeting with four more speakers. The number
of ways they can address such that A always speaks 7. Statement I: Out of 2 n + 1 consecutive positive inte-
before B is 360. gers 3 are to be selected such that they are in AP. The
number of ways of selecting them is n 2.
Statement II: The number of ways that A can speak
before B is equal to the number of ways that B speak Statement II: Positive integers a, b, c are in AP if and
before A. only if either both a and c are even or both a and c are
odd.
3. Statement I: A number lock has four rings and
each ring has 9 digits 1, 2, 3, … , 9. The number of 8. Statement I: In a lake, there are crocodiles each
unsuccessful attempts by a thief who does not know having teeth varying from 1 to 32. The number of croc-
the key code to open the lock is 6560. odiles in the lake is 2 32 .
Statement II: If repetitions are allowed, the number of Statement II: The number of elements in the power
permutations of n dissimilar objects taken r at a time is nr. set Ã( s) of a set S containing n elements is 2 n.
www.jeeneetbooks.in
318 Chapter 6 Permutations and Combinations

Integer Answer Type Questions


The answer to each of the questions in this section is a 6. There 5 ladies and 10 gentlemen. A committee
non-negative integer. The appropriate bubbles below the of 5 members is to be formed with two ladies and
respective question numbers have to be darkened. For three gentlemen. The number of ways of forming
example, as shown in the figure, if the correct answer to the committee, excluding two particular ladies and
the question number Y is 246, then the bubbles under Y including two particular gentlemen, is .
labeled as 2, 4, 6 are to be darkened.
7. The number of arrangements of n distinct object
X Y Z W taken all at a time is equal to K times the number
0 0 0 0 of arrangements of n objects which contain two
1 1 1 1 similar objects of one kind and three similar objects
of another kind. In such case K is equal to .
2 2 2
3 3 3 3 8. If A = {1, 2, 3, 4} and B = {a, b}, then the number of
4 4 4 surjections from A onto B is .
5 5 5 5
6 6 6
9. The number of ordered triplets ( x, y, z) of positive
integers such that their product is 24 is .
7 7 7 7
8 8 8 8 10. The least positive integer n such that (n-1)C5 + (n-1)C6 < nC7
9 9 9 9 is .

11. Six xs are to be placed in the squares of the given


1. Out of 10 points in a plane, p points are collinear
figure (containing 8 squares) with not more than
(0 < p < 10). The number of triangles formed with
one x in each square and such that each row contains
vertices at these points is 110. Then the value of p is
atleast one x. The number of ways that this can be
.
done is .
2. In a panchayat election, the number of candidates
contesting for a ward is one more than the maximum
number of candidates a voter can vote. If the total
number of ways of which a voter can vote is 62, then
the number of candidates is .

3. From four couples (wife and husband) a four-member


team is to be constituted. The number of teams that
can be formed which contain no couple is .
12. Five points on positive x-axis and 10 points on
4. In a test there are n students. 2 n - k students gave wrong positive y-axis are marked and line segments
answers to k questions (1 £ k £ n). If the total number connecting these points are drawn. Then the
of wrong answers given by the students is 2047, then n maximum number of points of intersection of these
is equal to . 50 line segments in the interior of the first quadrant
is .
5. If the number of permutations of n different objects
taken n – 1 at a time is K times the number of permu- 13. The number of triangles whose vertices are at the
tations of n objects (of which two are identical) taken vertices of an octagon but sides are not the sides of
n - 1 at a time, then K is equal to . the octagon is .
www.jeeneetbooks.in
Answers 319

ANSWERS
Single Correct Choice Type Questions
1. (B) 10. (A)
2. (C) 11. (A)
3. (D) 12. (C)
4. (A) 13. (C)
5. (D) 14. (A)
6. (C) 15. (A)
7. (A) 16. (B)
8. (D) 17. (D)
9. (B)

Multiple Correct Choice Type Questions


1. (A), (C) 6. (A), (B), (D)
2. (A), (D) 7. (B), (D)
3. (A), (B), (C), (D) 8. (A), (B), (D)
4. (A), (B), (C), (D) 9. (A), (B), (C), (D)
5. (A), (B), (C), (D) 10. (A), (B), (C), (D)

Matrix-Match Type Questions


1. (A) ® (r), (B) ® (p), (C) ® (p), (D) ® (s) 3. (A) ® (p), (B) ® (q), (C) ® (r), (D) ® (s)
2. (A) ® (r), (B) ® (s), (C) ® (p), (D) ® (q) 4. (A) ® (s), (B) ® (p), (C) ® (q), (D) ® (r)

Comprehension-Type Questions
1. (i) (A); (ii) (D); (iii) (C) 2. (i) (B); (ii) (A); (iii) (D)

Assertion–Reasoning Type Questions


1. (D) 5. (A)
2. (A) 6. (A)
3. (A) 7. (C)
4. (A) 8. (D)

Integer Answer Type Questions


1. 5 8. 14
2. 6 9. 30
3. 16 10. 14
4. 11 11. 26
5. 2 12. 450
6. 24 13. 16
7. 12
www.jeeneetbooks.in
www.jeeneetbooks.in

Binomial Theorem
7
Contents
7.1 Binomial Theorem for
Positive Integral Index
7.2 Binomial Theorem for
Rational Index

Worked-Out Problems
Summary
Exercises
Answers

1
The binomial theorem desc-
1 1 ribes the algebraic expan-
sion of powers of a binomial.
Binomial Theorem

1 2 1 According to the theorem,


it is possible to expand the
1 3 3 1 power (x + y)n into a sum
involving terms of the form
axbyc. The binomial coeffi-
1 4 6 4 1 cients appear as the entries of
Pascal’s triangle.
www.jeeneetbooks.in
322 Chapter 7 Binomial Theorem

The theorem which gives expansion of (a + b)n into the sum of n + 1 terms, where n is a positive integer, is called the
binomial theorem. We have come across formulas like
(a + b)2 = a2 + 2ab + b2
(a + b)3 = a3 + 3a2 b + 3ab2 + b3
(a + b)4 = a4 + 4a3b + 6a2 b2 + 4ab3 + b4
The coefficients involved in these expansions are called binominal coefficients. In this chapter, we derive expan-
sion of (a + b)n for a positive integer n and study the properties of the binominal coefficients in these expansions. This
will be further extended to a negative integer n or a rational number.

7.1 | Binomial Theorem for Positive Integral Index


In the expansions of (a + b)2, (a + b)3 and (a + b)4 given above, observe that as we proceed from left to right, the index
of a decreases by 1 while the index of b increases by 1. Also, observe that
(a + b)2 = 2 C0 a2 + 2 C1ab + 2 C2 b2
(a + b)3 = 3 C0 a3 + 3 C1a2 b + 3 C2 ab2 + 3 C3 b3
(a + b)4 = 4 C0 a4 + 4 C1a3 b + 4 C2 a2 b2 + 4 C3 ab3 + 4 C4 b4
Keeping these in mind, we derive a formula for (x + a)n in the following. The idea behind writing x as one of the
summands in (x + a)n is just to look at it as a polynomial of degree n, so that we can apply various results of addition
and multiplication of polynomials in the study of the binominal coefficients,

T H E O R E M 7 .1 For any positive integer n and any real or complex number a,


(BINOMIAL
( x + a) n = nC0 xn + n C1 xn - 1a + nC2 x n - 2 a2 + + n Cn an
THEOREM
FOR POSITIVE n

INTEGRAL = å n Cr x n - r ar
r =0
INDEX)
PROOF We use induction on n. For n = 1, this is trivial, since 1C0 = 1 = 1C1.
Let n > 1 and assume the theorem for n - 1; that is
n-1
( x + a)n - 1 = å n - 1 Cr x n - 1- r ar
r =0

Then, we have
( x + a)n = ( x + a)n - 1 ( x + a)
æ n-1 ö
= ç å n - 1 Cr xn - 1- r ar ÷ ( x + a)
è r =0 ø
n-1 n-1
= å n - 1 Cr xn - r ar + å n - 1 Cr xn - 1- r ar + 1
r =0 r =0
n-1 n-2
= n - 1 C0 xn - 0 a0 + å n - 1 Cr xn - r ar + å n - 1 Cr xn - 1- r ar + 1 + n - 1 Cn - 1 x0 a( n - 1) + 1
r =1 r =0
n-1 n-1
= xn + å n - 1 Cr xn - r ar + å n - 1 Cr - 1 xn - r ar + an
r =1 r =1
n-1
= n C0 xn a0 + å ( n - 1 Cr + n - 1 Cr - 1 ) xn - r ar + n Cn x0 an
r =1
www.jeeneetbooks.in
7.1 Binomial Theorem for Positive Integral Index 323

n-1
= n C0 xn a0 + å nCr xn - r ar + n Cn x0 an
r =1

n
= å n Cr xn - r ar
r =0

Thus, the theorem is valid for all positive integers n. ■

DEF IN IT ION 7 . 1 Note that there are n + 1 terms in the above expansion of (x + a)n. The (r + 1)th term is called
the general term and is denoted by Tr+1. It is given by
Tr +1 = nCr xn-r ar (0 £ r £ n)

C O R O L L A RY 7.1 For any positive integer n and for any real number a,
n
( x - a)n = å n Cr xn - r (-a)r
r =0

= xn - n C1 xn - 1a + n C2 xn - 2 a2 − n C3 xn - 3 a3 +
+ (-1)r n Cr xn− r ar + + (-1) n - 1 n Cn - 1 xan - 1 + (-1)n an
The general term in the above expansion is (-1)r nCr xn - r ar .

Examples

(1) The fourth term in the expansion of (2 x + 5a)8 is (2) The ninth term in the expansion of (2 x - 3a)17 is
8
C4 -1 (2 x)8 -( 4 -1) (5a)4 -1 = 8C3 (2)5 x5 (5)3 a3 17
C8 (2 x)17 - 8 (-3a)8 = 17C8 (2)9 (3)8 x9 a8
= 224000 × x5a3

C O R O L L A RY 7.2 For any positive integer n and any real numbers a, b and c,
n! r s t
(a + b + c)n = å
0 £ r, s, t £ n r ! s !t !
abc
r + s+t = n

PROOF First, treat b + c as single real number and use Theorem 7.1 to get
n
(a + b + c)n = å nCr an - r (b + c)r
r =0

and hence expand (b + c)r to get (r + 1) terms. Therefore, the expansion of (a + b + c)n contains
n+1
n
(n + 1)(n + 2)
å (r + 1) = å s =
r =0 s =1 2
= (n+ 2)
C2

number of terms and we can see that


n! r s t
(a + b + c)n = å
0 £ r, s, t £ n r ! s !t !
abc
r + s+t =n

Here the summation is taken over all ordered triples (r, s, t) of non-negative integers such that
r + s + t = n.
www.jeeneetbooks.in
324 Chapter 7 Binomial Theorem

Note: In general, for any positive integers n and m,


n!
( x1 + x2 + + xm )n = å
r1 + r2 + + rm = n r1 ! r2 ! rm !
x1r1 x2r2 xmrm

The summation is taken over all ordered m-tuples (r1, r2, …, rm) of non-negative integers such that
r1 + r2 + + rm = n.
By Theorem 6.16, the number of terms in the above expansion is ( n + m - 1)C( m - 1) . In particular,
the number of terms in (a + b + c)n is

(n+ 2) (n + 2)(n + 1)
C2 =
2
Also, the number of terms in the expansion of (a + b + c + d)n is

( n + 4 - 1) (n + 1)(n + 2)(n + 3)
C4 - 1 = ( n + 3)C3 =
6 ■
DEF IN IT ION 7 . 2 The middle term(s) in the expansion of ( x + a) is defined to be the term T( n / 2 ) + 1 if n is even and
n

the terms T( n +1)/ 2 and T( n + 3)/ 2 if n is odd.

Note that, if n is even then the expansion of ( x + a)n contains n + 1 terms and there are equal number of terms before
and after the term T( n / 2 ) + 1. If n is odd, then the expansion of ( x + a)n has even (n + 1) number of terms and there are
exactly (n – 1)/2 terms each before T( n +1)/ 2 and after T( n + 3)/ 2. The total number of terms is

n-1 n-1
+ 1+ 1+ = n+1
2 2

Examples

1. The middle term in the expansion of (3 x + 4a)16 is T(11+ 1)/ 2 = T6 = 11 C5 (2 x)11- 5 (-3a)5 = - 11 C5 26 × 35 × a5 × x6
T(16 / 2 ) + 1 = T9 which is given by
and T(11+ 3)/ 2 = T7 = 11 C6 (2 x)11- 6 (-3a)6 = 11 C6 25 × 36 × a6 × x5
16 - 8
16
C8 (3 x) (4a) = C8 × 3 × 4 × a × x
8 16 8 8 8 8

2. There are two middle terms in the expansions of


(2 x - 3a)11. These are

DEF IN IT ION 7 . 3 The binomial expansion of (1 + x)n is


n
C0 + nC1 x + nC2 x2 + + nCn - 1 xn - 1 + nCn xn
This is called the standard binomial expansion and the coefficients in this are called the
binomial coefficients. That is, nC0 , nC1 , nC2 , …, nCr , …, nCn-1 , nCn are called the binomial coef-
ficients. These are simply denoted by C0 , C1 , … , Cr , … , Cn - 1 , Cn.

Note that Cr alone has no meaning, unless we specify n also. If we say that C0 , C1 , … , Cn are the binomial coefficients
means that these are n C0 , n C1 , … , n Cn, respectively. In the following we prove certain important properties of the bino-
mial coefficients.

T H E O R E M 7 .2 Let C0 , C1 , C2 , … , Cn be the binomial efficients. Then the following hold good.


1. Cr = Cn - r for all 0 £ r £ n
2. C0 + C1 + C2 + + Cn = 2n
3. C0 + C2 + C4 + = C1 + C3 + C5 + = 2n - 1. That is,

åC
r even
r = åC
r odd
r = 2n - 1

4. C 0 + 2 × C 1 + 3 × C 2 + + (n + 1) × C n = (n + 2)2 n - 1
www.jeeneetbooks.in
7.1 Binomial Theorem for Positive Integral Index 325

PROOF Note that


n!
Cr = nCr = for each 0 ≤ r ≤ n
(n - r )! r !
1. It is trivial.
2. We have the standard binomial expansion
(1 + x) n = nC0 + nC1 x + nC2 x2 + + nCn - 1 x n - 1 + nCn x n
By substituting 1 for x, we get that
2 n = nC0 + nC1 + n C2 + + nCn - 1 + nCn
= C0 + C1 + C2 + + Cn - 1 + Cn
3. By substituting –1 for x in the standard binomial expansion, we get that
0 = C0 - C1 + C2 - + (-1)r × Cr + + (-1)n × C1
Therefore
C0 - C1 + C2 - C3 + C4 - = 0
C0 + C2 + C4 + = C1 + C3 + C5 +

and by part (1), each of these is 2n/ 2 = 2n - 1.


4. Consider
S = C0 + 2 × C1 + 3 × C2 + + (n + 1) × Cn (7 .1)
Writing the terms in the reverse order, we get that
S = (n + 1) × Cn + n × Cn - 1 + + 3 × C2 + 2 × C1 + C2
Since C r = C n - r for each 0 £ r £ n, we get
S = (n + 1) × C0 + n × C1 + + 3 × Cn - 2 + 2 × Cn - 1 + Cn
Adding this to Eq. (7.1), we get that
2S = (n + 2) × C0 + (n + 2) × C1 + + (n + 2) × Cn
= (n + 2)(C0 + C1 + + Cn )
= (n + 2)2n
Therefore S = (n + 2)2 n - 1 . That is,
C0 + 2C1 + 3C2 + + (n + 1)Cn = (n + 2)2n - 1 ■

T H E O R E M 7 .3 Let C 0 , C 1 , C 2 , … , C n be the binomial coefficients. Then the following hold good.


1. For any real numbers a and d,
a × C 0 + (a + d) × C 1 + (a + 2d) × C 2 + + (a + nd) × C n = (2a + nd) × 2 n - 1
n
2. å r ×C
r =1
r = n × 2n - 1

n
3. å r(r - 1)C
r =1
r = n(n - 1) × 2n - 2

n
4. år
r =1
2
× Cr = n(n + 1) × 2n - 2
www.jeeneetbooks.in
326 Chapter 7 Binomial Theorem

PROOF 1. Put S = a × C0 + (a + d) × C1 + (a + 2d) × C2 + + (a + nd) × Cn. Writing the terms in the reverse
order and using C r = C n - r, we have
S = (a + nd) × C0 + [a + (n - 1)d] × C1 + + (a + d) × Cn - 1 + a × Cn
Adding two equations, we get
2S = (2a + nd) × C0 + (2a + nd) × C1 + + (2a + nd) × Cn

Therefore
1 1
S= (2a + nd)(C0 + C1 + + Cn ) = (2a + nd)2n
2 2
Thus S = (2a + nd)2 n - 1.
2. Substituting 0 for a and 1 for d in part (1), we get
0 × C 0 + 1 × C 1 + 2 × C 2 + + n × C n = (0 + n × 1)2 n - 1
Therefore
n

å r ×C
r =1
r = n × 2n - 1

In the following solution, differentiation is used which we discuss in Vol. III.


3. Consider (1 + x)n = C0 + C1 × x + C2 × x2 + + Cn - 1 xn - 1 + Cn xn. On differentiating both sides with
respect to x, we get

n(1 + x)n - 1 = C1 + C2 × 2 x + C3 × 3 x2 + + Cn × nxn - 1


Again on differentiating we get that
n(n - 1)(1 + x)n - 1 = 2 × C2 + 3 × 2 × C3 x + 4 × 3 × C4 + + n(n - 1) × Cn xn - 1
Substituting 1 for x in the above, we get that

n(n - 1)2n - 2 = 2 × 1 × C2 + 3 × 2 × C3 + + n(n - 1)Cn


Thus
n

å r(r - 1)C
r =1
r = n(n - 1)2 n - 2

4. We have
n n

år
r =1
2
× Cr = å (r(r - 1) + r ) × Cr
r =1

n n
= å r(r - 1) × Cr + å r × Cr
r =1 r =1

n n
= å r(r - 1) × Cr + å r × Cr
r=2 r =1

= n(n - 1) × 2n - 2 + n × 2n - 1

= (n(n - 1) + 2 n)2n - 2

= n(n + 1)2n - 2 ■
www.jeeneetbooks.in
7.1 Binomial Theorem for Positive Integral Index 327

Next, we will discuss about numerically greatest term among the (n + 1) terms in the expansion of (1 + x)n. Before
this, let us recall that, for any real number x, [x] denotes the largest integer less than or equal to x and that [x] is called
the integral part of x. Also, x - [x] is called the fractional part of x and is denoted by {x}. Note that x = [ x] + {x}, [ x] Î 
and 0 £ {x} < 1.

DEF IN IT ION 7 . 4 In the binomial expansion of (1 + x)n, a term Tr is called numerically greatest if |Ti | £ |Tr | for all
1 £ i £ n + 1.

T H E O R E M 7 .4 Let x be a non-zero real number, n a positive integer and m the integral part of (n + 1)| x |/(1 + | x |).

1. If m < (n + 1) | x |/(1 + | x |) < m + 1, then Tm+1 is the numerically greatest term in the binomial
expansion of (1 + x)n.
2. If m = [(n + 1) | x |]/(1+ | x |), then Tm and Tm+1 are the numerically greatest terms in the binomial
expansion of (1 + x)n.

PROOF Let T1 , T2 , …, Tn+1 be all the terms in the binomial expansion of (1 + x)n. Then

Tr + 1 = nCr xr for all 0 £ r £ n

Since x ¹ 0, Tr + 1 is a non-zero real number for each r. Now, consider

Tr + 1 n
C xr n ! xr (n - r + 1)!(r - 1)!
= n r r -1 = ´
Tr Cr - 1 x (n - r )! r ! n ! xr - 1

(n - r + 1)
= ×x
r
Therefore

Tr + 1 (n - r + 1)
= | x| (7.2)
Tr r

Now
(n - r + 1)
|Tr + 1 | ³ |Tr | Û | x| ³ 1
r
n+1 1
Û -1³
r | x|

n + 1 1 + | x|
Û ³
r | x|

1 1 + | x|
Û ³
r (n + 1) | x |
(n + 1) | x |
Ûr£
1 + | x|

Therefore, for any integer r with 1 £ r £ n + 1 , we have

é (n + 1) | x | ù
|Tr + 1 | ³ |Tr | Û r £ ê ú=m (7.3)
ë 1 + | x| û
www.jeeneetbooks.in
328 Chapter 7 Binomial Theorem

Also, by Eq. (7.1) again


(n + 1)| x |
|Tr + 1 | = |Tr | Û r = (7.4)
1 + | x|
(n + 1)| x |
and |Tr + 1 | £ |Tr | Û r ³ (7.5)
1 +| x|

Now, we shall distinguish two cases. First note that since m is the integral part of (n + 1)| x |/(1 + | x |),
we have
(n + 1) | x |
m£ <m+1
1 + | x|

1. Suppose that m < [(n + 1) | x |]/(1 + | x |) < m + 1. From Eqs. (7.3)–(7.5), we have

|T1 | < |T2 | < < |Tm | < |Tm + 1 | and |Tm+1 | > |Tm+ 2 | > > |Tn+1 |

Thus
|Tm + 1 | > |Ti | for all i ¹ m + 1

and therefore Tm+1 is the numerically greatest term.


2. Suppose that m = [(n + 1) | x |]/(1 + | x |). From Eq. (7.4), we have
|Tm + 1 | = |Tm |

Again, by Eqs. (7.3) and (7.5), we get

|T1 | < |T2 | < < |Tm | = |Tm + 1 | > |Tm + 2 | > > |Tn + 1 |

Thus, Tm and Tm+1 are the numerically greatest terms in the binomial expansion of (1 + x)n. ■

C OROLLARY 7.3 For any non-zero real numbers a and x, if Tm is numerically greatest term in [1 + ( x / a)]n, then a nTm
is numerically greatest term in (a + x)n.

QUICK LOOK 1

1. If [(n + 1) | x |]/(1 + | x |) is not an integer, and m is the in (1 + x)n. In this case note that |Tm | = |Tm +1 | and that
integral part of [(n + 1) | x |]/(1 + | x |), then Tm+1 is the Tm may not be equal to Tm+1 and we can only infer
numerically greatest term in (1 + x)n. that Tm = ±Tm +1.
2. If [(n + 1) | x |]/(1 + | x |) is an integer and is equal to m,
then Tm and Tm+1 are both numerically greatest terms

Example 7.1

Find the numerically greatest term(s) in the binomial and the integral part of this is 3. Therefore, by part (1)
expansion of (1 - 2 x)12 for x = 1/5. of Theorem 7.4, T4 is the numerically greatest term in
(1 + X )12 and
Solution: Put X = –2/5 and consider (1 + X )12. We have 9 9
12 - 3 12 × 11× 10 æ -2 ö æ 2ö
(12 + 1) | X | 13 × (2 /5) 26 T4 = C3 X
12
= ç ÷ = - 220 çè ÷ø
= = 1× 2 × 3 è 5 ø 5
1 + |X | 1 + (2 / 5) 7
www.jeeneetbooks.in
7.2 Binomial Theorem for Rational Index 329

Example 7.2

Find the numerically greatest term in the binomial expansion of (1 + X )19 and of (2 x - 3 y)19. These terms are
expansion of (2 x - 3 y)19 when x = 1/ 4 and y = 1/ 6. given by
10 9
Solution: We have æ 1ö æ 1ö 1
T10 = 19C9 (2 x)10 (-3 y)9 = - 19C9 ç 2 × ÷ ç 3 × ÷ = - 19C 9 × 19
19
è 4ø è 6ø 2
æ 3y ö
(2 x - 3 y)19 = (2 x)19 ç 1 - ÷
è 2x ø æ 1ö æ 1ö
9 10

and T11 = 19C10 (2 x)9 (-3 y)10 = 19C10 ç 2 × ÷ ç 3 × ÷


è 4ø è 6ø
Put X = -3 y/2 x and consider
1
(19 + 1) | X | 20 | -1| = 19C10 ×
= = 10 219
1 + |X | 1 + | -1|
Note that, in this case T10 = - T11 .
which is an integer. By part (2) of Theorem 7.4, T10 and
T11 are the numerically greatest terms in the binomial

T H E O R E M 7 .5 1. If n is even, then nCn /2 is the greatest among the binomial coefficients n C0 , nC1 , n C2 , … , n Cn.
2. If n is odd, then nC( n-1)/ 2 = nC( n+1)/ 2 are the greatest among nC0 , nC1 , nC2 , … , nCn .

PROOF This follows from Theorem 7.4 by taking x = 1 and from the part that Tm = nCm - 1 . ■

7.2 | Binomial Theorem for Rational Index


In the earlier section, we have proved that, for any positive integer n and for any real number x,

(1 + x)n = nC0 + nC1 x1 + nC2 x2 + + nCn xn


This also can be expressed as
¥
n(n - 1)(n - 2) (n - r + 1) r
(1 + x)n = 1 + å x
r =1 1 × 2 × 3 r

For r > n , the coefficient of xr becomes zero and the above is an expression of n + 1 terms only. However, for a
negative integer n or for a fraction (rational number) n, we have a similar formula consisting of infinitely many terms,
provided | x | < 1. The proof of this is beyond the scope of this book and we state the following without proof and derive
certain useful consequences.

T H E O R E M 7 .6 Let x be a real number such that -1 < x < 1. Then for any rational number m,
¥
m(m - 1) (m - r + 1) r
(1 + x)m = 1 + å x
r =1 1× 2 × 3 r

C OROLLARY 7.4 Let n be a positive integer and x a real number such that -1 < x < 1. Then
¥
1. (1 + x)- n = å (-1)r ( n + r - 1)
Cr xr
r =0

¥
2. (1 - x)- n = å n+ r -1
Cr xr
r =0
www.jeeneetbooks.in
330 Chapter 7 Binomial Theorem

PROOF From Theorem 7.6, we get that


¥
(- n)(- n - 1)(- n - 2) (- n - r + 1) r
(1 + x)- n = 1 + å
r=1 1× 2 × 3 r
x

n n(n + 1) 2 n(n + 1)(n + 2) 3 n(n + 1) (n + r - 1)


=1- x+ x - x + + (-1)r +
1 1× 2 1× 2 × 3 1× 2 × 3 r
¥
(n + r - 1)(n + r - 2) (n + 1)n r
= 1 + å (-1)r x
r=1 1× 2 × 3 r
¥
= å (-1)
r=0
r ( n + r - 1)
Cr xr

Also, by replacing x with –x in the above, we have


¥
(1 - x)- n = å (-1) ×
r=0
r ( n + r - 1)
C r ( - x )r

¥
= å ( n + r - 1)
Cr xr
r=0 ■

C O R O L L A RY 7.5 Let m and n be positive integers and x a number such that -1 < x < 1. Then we have the following.
2 r
m x m(m - n) æ x ö m(m - n) [m - (r - 1)n] æ x ö
1. (1 + x) = 1 + + çè ÷ø + + çè ÷ø +
m/ n

1 n 1× 2 n 1× 2 rr n
2 r
m x m(m - n) æ x ö r m( m - n) [ m - (r - 1)n] æ x ö
2. (1 - x)m / n = 1 - × + çè ÷ø + + (-1) çè ÷ø +
1 n 1× 2 n 1 × 2 r n
2 r
m x m(m + n) æ x ö r m( m + n) [ m + (r - 1)n] æ x ö
3. (1 + x)- m / n = 1 - × + çè ÷ø + + (-1) çè ÷ø +
1 n 1× 2 n 1× 2 r n
2 r
m x m(m + n) æ x ö m(m + n) [m + (r - 1)n] æ x ö
4. (1 - x)- m / n = 1 + × + çè ÷ø + + çè ÷ø +
1 n 1× 2 n 1× 2 r n
PROOF Here we will prove part (1) only. Parts (2), (3) and (4) can be similarly proved. From Theorem 7.6,
¥
(m / n)[(m / n) - 1] [(m / n) - r + 1] r
(1 + x)m / n = 1 + å x
r =1 1× 2 × 3 r
(m / n) (m / n)[(m / n) - 1] 2 (m / n)[(m / n) - 1][(m / n) - 2] 3
=1+ x+ x + x +
1 1× 2 1× 2 × 3
(m / n)[(m / n) - 1] [(m / n) - (r - 1)] r
+ x +
1× 2 × 3 r
2 3
m x m(m - n) æ x ö m(m - n)(m - 2 n) æ x ö
=1+ × + çè ÷ø + çè ÷ø +
1 n 1× 2 n 1× 2 × 3 n
r
m(m - n) (m - (r - 1)n) æ x ö
+ çè ÷ø +
1× 2 n n ■

Try it out Solve parts (2)–(4) of Corollary 7.4.


www.jeeneetbooks.in
7.2 Binomial Theorem for Rational Index 331

QUICK LOOK 2

For any real number x with - 1 < x < 1 , we have the following:
1. (1 + x)-1 = 1 - x + x2 - x3 + + (- 1)r xr +
2. (1 - x)-1 = 1 + x + x2 + x3 + + xr +
3. (1 + x)-2 = 1 - 2x + 3x2 - 4x3 + + (-1)r · (r + 1)xr +
4. (1 - x)-2 = 1 + 2 x + 3 x2 + 4 x3 + + (r + 1) xr +
3× 4 2 3× 4× 5 3 3 × 4 × 5 r(r + 1)(r + 2) r
5. (1 + x)-3 = 1 - 3 x + x - x + + (- 1)r x +
1× 2 1× 2 × 3 1 × 2 × 3 r
3×4 2 4×5 3 (r + 1)(r + 2) r
= 1 - 3x + x - x + + (- 1)r × x +
1× 2 1× 2 1× 2
3×4 2 4×5 3 (r + 1)(r + 2) r
6. (1 - x)-3 = 1 + 3 x + x + x + + x +
1× 2 1× 2 1× 2

In the following examples we will use the fact that the general term in the expansion of (1 + x)- m is given by
m(m + 1) (m + r - 1) r
Tr + 1 = (-1)r x
1 × 2 × 3 r
This is the (r + 1)th term in the expansion of (1 + x)- m for r > 0 and first term is always 1.

Example 7.3

Obtain the fifth term of [1 + ( x / 3)]-8. 8(8 + 1) [8 + (4 - 1)] æ x ö


4

T5 = T4 + 1 = (-1)4 çè ÷ø
1× 2 × 3× 4 3
Solution: The fifth term is given by
8 × 9 × 10 × 11 x4 110 4
= = x
1 × 2 × 3 × 4 34 27

Example 7.4

Obtain the sixth term in the expansion of [1 - ( x2 / 4)]- 4 . Solution: The sixth term is given by
5
4 × 5 × 6 × 7 [4 + (5 - 1)] æ x2 ö
T6 = T5+1 = çè 4 ÷ø
1× 2 × 3× 4 × 5
4 × 5 × 6 × 7 x10 7
= = 5 x10
1 × 2 × 3 × 4 × 5 45 4

Example 7.5

Obtain the fifth term in the expansion of [6 + (5 y / 11)]6/5. The fifth term in the expansion is given by
é 6(6 - 5)(6 - 2 × 5)(6 - 3 × 5) æ 5 y ö 4 ù
Solution: The given expression can be written as T5 = T4 +1 = 66 / 5 ê çè ÷ø ú
ë 1× 2 × 3× 4 66 û
6/5
æ 5y ö
6/5
æ 5y ö
çè 6 + ÷ø = 66 / 5 ç 1 +
4
6 × 1 × (- 4)(- 9) æ 5 ö 4
11 è 6 × 11÷ø = 66 / 5 ç ÷ y
1 × 2 × 3 × 4 è 66 ø
www.jeeneetbooks.in
332 Chapter 7 Binomial Theorem

Example 7.6

Obtain the 10th term in the expansion of (3 - 4 x)2 / 3. The 10th term in the expansion is given by
9
2(2 - 3)(2 - 2 × 3) (2 - 8 × 3) æ 4 x ö 2 / 3
Solution: We can write the given expression as T10 = T9 + 1 = (-1)9 çè ÷ø × 3
1× 2 9 3
2/3
æ 4 ö
(3 - 4 x)2 / 3 = 32 / 3 ç 1 - x÷ 9
2 × 1 × 4 × 7 × 10 × 13 × 16 × 19 × 22 æ 4 ö 2 / 3 9
è 3 ø =- çè ÷ø × 3 × x
1 × 2 × 3 9 3

Example 7.7

Obtain the values of x for which the binomial expansion Therefore, the binomial expansion of (3 - 4x)-7 is valid
of (3 - 4 x)-7 is valid. if and only if that of [1 - (4 x / 3)] -7 is valid and this is
valid if -1 < 4 x/ 3 < 1. That is
Solution: The given expression can be written as -3 3 3
-7
<x< or | x| <
æ 4x ö 4 4 4
(3 - 4 x)-7 = 3-7 ç 1 - ÷
è 3ø

Example 7.8

¥ é
Find the sum of the infinite series m(m + n) (m + (r - 1)n) æ x ö ù
r

= 1+ åê ×ç ÷ ú
1 1× 3 1× 3 × 5 r =1 ê
ë 1× 2 r è n ø úû
1+ + + +
3 3× 6 3× 6 × 9
where m = 1, n = 2, x = 2 / 3. Substituting these values we
get
Solution: This sum can be written as
-1 / 2
æ 2ö
2
1 1 1× 3 æ 1 ö 1× 3 × 5 æ 1 ö
3
S = ç1 - ÷ [by part (4) of Corollary 7.4]
S =1+ × + ç ÷ + ç ÷ + è 3ø
1 3 1× 2 è 3 ø 1× 2 × 3 è 3 ø
= 3

Example 7.9

Find the coefficient of x4 in (1 - 4 x)-3 / 5. Therefore x4 appears in T5 only and hence


4
3 × 8 × 13 × 18 æ 4 x ö 13 × 18 × 44 4
Solution: The general term in the expansion of (1 - T5 = ç ÷ = x
4x)-3/5 is 1× 2 × 3× 4 è 5 ø 54
r The coefficient of x4 in the expansion of (1 - 4x)-3 is
3(3 + 5)(3 + 2 × 5) + + (3 + (r - 1)5) æ 4 x ö
Tr + 1 = çè ÷ø (13 × 18 × 256)/ 625.
1× 2 × 3 r 5

Example 7.10

Using the binomial theorem for rational index, find the é 1 1 (1/ 5)[(1/ 5) - 1] æ 1 ö ù
2

approximate value of (242)1 / 5 correct to 4 decimals. = 3 ê1 - × + çè ÷ ú


êë 5 243 1× 2 243 ø úû
Solution: Consider é 1 2 ù
= 3 ê1 - (0.00243) - (0.00243)2 ú
(242)1 / 5 = (243 - 1)1 / 5 ë 5 25 û
1/ 5 é 1 æ 1ö
5
ù
æ 1 ö ê since = çè ÷ø  (0.3) ú
5
= (243) 1/ 5
çè 1 - ÷ êë 243 3 úû
243 ø
www.jeeneetbooks.in
Worked-Out Problems 333

3 6 = 2.9998541
= 3 - (0.00243) - (0.00243)2
5 25
 2.9985 (corrected to 4 decimals)
= 3 - 0.001458 - 0.000001417176
(by neglecting other terms)

Note that when we are required to find an approximation of an expression correct to K decimal places, we choose r,
the number of terms to be taken in consideration, such that the magnitude of the rth term is less than 1/ 10K + 2 so that
its decimal representation has atleast K + 2 zeros immediately after the decimal.

Example 7.11

Find the approximate value of 4 + 3 x /(3 - 2 x)2, when 2æ 3 öæ 4 ö


| x | is so small that x2 and higher powers of x can be = çè 1 + x÷ø çè 1 + x÷ø
9 8 3
neglected.
2æ 3 4 ö
= çè 1 + x + x÷ø (by neglecting x too)
2

Solution: The given expression can be written as 9 8 3


4 + 3x 2[1 + (3 x / 4)]1 / 2 2æ 41 ö
= = çè 1 + x÷
(3 - 2 x) 2
9[1 - (2 x / 3)]2 9 24 ø

2æ 3x ö
1/ 2
æ 2x ö
-2 2 41 24 + 41x
= = + x=
çè 1 + ÷ø × ç1 -
è ÷ 9 108 108
9 4 3ø
2æ 1 3 öé æ -2x ö ù
= çè 1 + × x÷ø ê1 + (- 2) çè ÷
9 2 4 ë 3 ø úû
(by neglecting x2 and the higher powers of x)

WORKED-OUT PROBLEMS
Single Correct Choice Type Questions
1. If the sum of the fifth and sixth terms is zero in the (A) a = 2b (B) b = 2a
binomial expansion of (a - b)n, n £ 5, then the value (C) a = 3b (D) b = 3a
of a/b is
Solution: xn occurs in the (n + 1)th term of (1 + x)2n or
(A) (n - 4)/5 (B) 5/(n - 4) (C) 5 (D) 1/5
(1 + x)2n-1. Therefore
Solution: If n £ 5, then the fifth and sixth terms exist in
(2 n)! (2 n - 1)!
the binomial expansion of (a - b)n only when n = 5, then a = 2 nCn = and b = ( 2 n - 1)Cn =
in (a - b)5, n! n! (n - 1)! n !

Fifth term = 5C4 a5 - 4 (-b)4 = 5ab4 Hence


Sixth term = 5C5 a5 - 5 (- b)5 = - b5 2n
b = a or a = 2b
If the sum of fifth and sixth terms is zero, then n
Answer: (A)
5ab4 - b5 = 0
3. If in the expansion of (1 + x) (1 - x) , the coefficients
m n
5ab4 = b5 2
of x and x are 3 and –6, respectively, then m and n are
a 1 respectively
=
b 5 (A) 12, 9 (B) 13, 9 (C) 9, 13 (D) 9, 12
Answer: (D) Solution: We have

2. If a and b are the coefficients of xn in the expansions (1 + x)m (1 - x)n = (1 + mC1 x + mC2 x2 + )
2 n-1
of (1 + x) and (1 + x)
2n
, respectively, then
´ (1 - nC1 x + nC2 x2 + )
www.jeeneetbooks.in
334 Chapter 7 Binomial Theorem

Therefore coefficient of x is given by x-1 1


and =1+
m
C1 - C1 = m - n = 3
n
(7.6) x- x z

Now coefficient of x2 is The given expansion is


10 10
mC2 + nC2 - mC1 × nC1 = - 6 æ 1ö æ3 1 ö
çè y + 1 - 1 - ÷ø = çè x - ÷
m(m - 1) + n(n - 1) - 2 mn = - 12 z xø

(m - n)2 - (m + n) = - 12 (7.7) Therefore

9 - (m
m + n) = - 12 Tr + 1 = 10Cr × x(10 - r ) / 3× (-1)r × x- r / 2
m + n = 21 Tr +1 is independent of x implies
Solving Eqs. (7.6) and (7.7), we get m = 12, n = 9.
10 - r r
Answer: (A) - =0
3 2
4. In the expansion of (1 + x)15, if the coefficients of Hence
(r - 1)th and (2r + 3)rd terms are equal, then r is equal to
20 - 5r = 0 or r = 4
(A) 4 (B) 6 (C) 5 (D) 7
Therefore independent term value is
Solution: The coefficients of (r - 1)th and (2r + 3)rd
terms are
10
C4 (-1)4 = 210
Answer: (C)
Tr - 1 = 15Cr - 2 xr - 2 and T2 r + 3 = 15C2 r + 2 x2 r + 2
Since it is given that they are equal, we have 7. The coefficient of x50 in the expression (1 + x)1000 +
2 x(1 + x)999 + 3 x2 (1 + x)998 + + (1001) x1000 is
15
Cr - 2 = 15C2 r + 2
(A) (1000 )C53 (B) (1002 )C52 (C) (1002 )C50 (D) ( 1002 )
C51
r - 2 ¹ 2r + 2
Solution: Let
Þ (r - 2) + (2r + 2) = 15
s = (1 + x)1000 + 2 x(1 + x)999 + 3 x2 (1 + x)998 + + (1001) x1000
Þr=5 The terms of the given sum follow arithmetic geometric
Answer: (C) progression with first factors of the terms as 1, 2, 3, ¼,
and second factors are in GP with common ratio
10
5. Let a be the coefficient of x in the expansion of r = x /(1 + x). Therefore
(1 - x ) and b the term independent of x in the
2 10
(1 + x)1001 {1 - [ x /(1 + x)]1001 }
expansion [ x - (2 / x)]10 . Then a : b is equal to s= - (1001) x1001
(A) 1 : 12 (B) 1 : 22 (C) 6 : 1 (D) 1 : 32 1 - [ x /(1 + x)]

Solution: We have x10 = ( x2 )5. The coefficient of x10 is 10C5 . é æ x ö 1001


ù
Independent term in the expansion [x − (2/x)]10 is −25∙10C5. = (1 + x)10002 ê1 - ç ÷ ú - (1001) x1001
êë è 1 + x ø úû
Therefore
a : b = 1 : 25 = 1 : 32 = (1 + x)1002 - (1 + x) x1001 - (1001) x1001
Answer: (D)
= (1 + x)1002 - (1002) x1001 - x1002
6. The value of the term independent of x in the expan- Therefore the coefficient of x50 in the expression = coef-
sion of ficient of x50 in the expansion of (1 + x)1002 which is equal
10
to (1002 )C50 .
æ x+1 x-1 ö Answer: (C)
ç3 2 3 - ÷
è x - x + 1 x - xø
8. Let n be a positive integer and
is n+ 4

(A) 110 (B) 90 (C) 210 (D) 200 (1 + x2 )2 (1 + x)n = åa


K=0
K xK

Solution: Put 3
x = y and x = z so that
If a1, a2 and a3 are in AP, then the number of values of n is
x+1
= y+1 (A) 2 (B) 3 (C) 4 (D) 5
3
x2 - 3 x + 1
www.jeeneetbooks.in
Worked-Out Problems 335

Solution: We have 2 y2 - 3 y - 2 = 0
n+ 4 ( y - 2)(2 y + 1) = 0
åa K xK = (1 + 2 x2 + x4 )[1 + nC1 x + nC2 x2 + + xn ]
K =0 y = 2 or - 1
On the LHS, a1, a2 and a3 are respectively the coefficients For these values of y we have
of x, x2 and x3. Therefore equating the coefficients of x, x2
and x3 on both the sides we get x = 102 or x = 10-1

a1 = nC1 ; a2 = 2 + nC2 ; a3 = 2 × nC1 + nC3 Since it is given that x > 1, x = 102 .


From these we have Answer: (B)

a1 + a3 = 2a2 11. The coefficient of x in the expansion of [(x/2) -


4

2 10
(3/x )] is
n
C1 + 2 × nC1 + nC3 = 2(2 + nC2 )
405 504
n(n - 1)(n - 2) (A) (B)
3n + = 4 + n(n - 1) 256 259
6
450 400
n3 - 9 n2 + 26 n - 24 = 0 (C) (D)
263 263
(n - 2)(n - 3)(n - 4) = 0
Solution: The (r + 1)th term is given by
n = 2, 3, 4
10 - r r
Answer: (B) æ xö æ -3 ö
Tr + 1 = 10Cr ç ÷ çè 2 ÷ø
è 2ø x
9. Let n be a positive integer. If the coefficients of 1
second, third and fourth terms in the expansion of = 10Cr ´ 10 - r
(-3)r x10 - 3r
2
(1 + x)n are in AP, then the value of n is
(A) 2 (B) 5 (C) 6 (D) 7 Therefore
n
Solution: By hypothesis C1, C2 and C3 are in AP. n n
10 - 3r = 4 Þ r = 2
Therefore
Hence coefficient of x4 is
n
C1 + C3 = 2 ´ C2
n n

32 9 × 10 9 405
n(n - 1)(n - 2)
10
C2 ´ 8
= ´ =
n+ = n(n - 1) 2 2 256 256
6 Answer: (A)
6 + n2 - 3n + 2 = 6 n - 6
12. The expression (x + x3 - 1)5 + (x - x3 - 1)5 is a poly-
n2 - 9 n + 14 = 0
nomial of degree:
(n - 2)(n - 7) = 0 (A) 5 (B) 6
Since, there are more than three terms in the expansion, (C) 7 (D) 8
the value of n must be 7. Solution: We know that
Answer: (D)
(a + b)5 + (a - b)5 = 2[ 5C0 a5 + 5C2 a3 b2 + 5C4 ab4 ]
10. For x > 1, if the third term in the expansion of [(1/x) +
= 2(a5 + 10a3 b2 + 5ab4 )
xlog10 x]5 is 1000, then the value of x is
(A) 10 (B) 100 (C) 5 2 (D) 50 = 2[ x5 + 10 x3 ( x3 - 1) + 5 x( x3 - 1)2 ]
Solution: Put log10 x = y. Therefore where x = a and b = x3 - 1. Therefore the given expres-
5 sion is a polynomial of degree 7.
æ1 log10 x ö -y y2 5
çè + x ÷ø = (10 + 10 ) Answer: (C)
x
The third term is given by 13. The coefficient of t
24
in (1 + t2 )12 (1 + t12 )(1 + t24 ) is
2
5
C2 10-3 y× 102 y = 1000 (A) 12C6 + 3 (B) 12C6 + 1
2
10-3 y× 102 y = 100 = 102 (C) 12C6 (D) 12C6 + 2
www.jeeneetbooks.in
336 Chapter 7 Binomial Theorem

Solution: We have By mathematical induction, we can show that the


coefficient of xn-1 in the expansion of (x + a1)(x + a2)
(1 + t2 )12 (1 + t12 )(1 + t24 ) = (1 + t2 )12 [1 + t12 + t24 + t36 ]
(x + a3) (x + an) is a1 + a2 + + an. Therefore the
= [1 + 12
C1t2 + 12
C2 (t2 )2 + + t24 ] coefficient of x99 in the given expansion is

´ (1 + t12 + t24 + t36 ) 100 ´ 101


1 + 2 + 3 + + 100 = = 5050
2
Therefore the coefficient of t24 is 1 + 12C6 + 1 = 12C6 + 2
Answer: (A)
Answer: (D)
17. Let Tr denote the rth term in the expansion of
14. The sum of the rational terms in the expansion of [2x + (1/ 4x )]n. If the ratio T3 : T2 = 7 : 1 and sum of the
( 2 + 31 / 5 )10 is coefficients of second and third terms is 36, then x
(A) 31 (B) 41 (C) 51 (D) 61 value is
Solution: The general term is given by 1 -1 1 -1
(A) (B) (C) (D)
2 2 3 3
Tr + 1 = 10Cr 2(10 - r ) / 2× 3r / 5
Solution: It is given that
This is rational, if 10 - r is even and r is a multiple of 5.
T3
10 - r =7
r = 5Þ is not an integer T2
2
This implies
r
r = 10 Þ 10 - r = 0 and =2 -2 x
5 æ n - 2 + 1ö æ 2 ö
çè ÷ø ç x ÷ = 7
10 - r 2 è 2 ø
r=0Þ =5
2 n - 1 -3 x
×2 = 7 (7.8)
Therefore the sum of the rational terms is 2
Also
10
C0× 25 + 10
C10× 32 = 32 + 9 = 41 n
C1 + nC2 = 36
Answer: (B)
n2 + n - 72 = 0
15. The sum of the coefficients of the polynomial
(n + 9)(n - 8) = 0
(1 + x - 3 x2 )2163 is
(A) 0 (B) 22163 (C) 1 (D) –1 n=8

Solution: If Putting the value n = 8 in Eq. (7. 8), we have

f ( x) º a0 xn + a1 xn - 1 + a2 xn - 2 + + an 7 -3 x
×2 = 7
2
then the sum of the coefficients is
2-3 x = 2
a0 + a1 + a2 + + an = f (1)
Therefore
Now let f ( x) º (1 + x - 3 x2 )2163. Therefore the sum of the
coefficients is -1
x=
3
f (1) = (-1)2163 = - 1
Answer: (D)
Answer: (D)
18. If the sixth term in the expansion of
16. The coefficient of x99 in the expansion of (x + 1)
(x + 2)(x + 3) (x + 99) (x + 100) is
7
é log2 9x -1 + 7 1 ù
ê2 + x -1 ú
(A) 5050 (B) 5500 (C) 5005 (D) 5000 ë 2(1/ 5) log2 ( 3 + 1) û
Solution: We have is 84, then the sum of the values of x is
( x + a )( x + b ) = x + (a + b ) x + ab
2
(A) 3 (B) 4 (C) 9 (D) 16
( x + a )( x + b )( x + g ) = x3 + (a + b + g ) x2 Solution: By hypothesis
+ (ab + bg + ga )x + abg æ 1 ö
7
C5 ( 9x - 1 + 7 )2 ç x - 1 ÷ = 84
è 3 + 1ø
www.jeeneetbooks.in
Worked-Out Problems 337

Therefore Hence
2 ( x - 1)
3 +7 1
=4 an = (n + 1)th term = a0 + nd =
3x - 1 + 1 2n + 1
Answer: (B)
Substituting y = 3x - 1 we get
y2 + 7 = 4 y + 4 22. If a1, a2, a3, and a4 are the coefficients of any four
consecutive terms in the expansion of (1 + x)n, then
y - 4y + 3 = 0
2

This gives y = 1, 3 which implies that 3x-1 = 1 or 3x-1 = 3. a1 a2 a3


, ,
Therefore x = 1, 2. The sum of the values = 1 + 2 = 3. a1 + a2 a2 + a3 a3 + a4
Answer: (A) are in
(A) AP (B) GP (C) HP (D) AGP
19. The integral part of ( 2 + 1)6 is
(A) 298 (B) 297 (C) 198 (D) 197 Solution: Let a1, a2, a3, a4 be the coefficients of rth,
(r + 1)th, (r + 2)th and (r + 3)th terms, respectively.
Solution: We have Then
( 2 + 1) + ( 2 - 1)6 = 2[( 2 )6 + 15( 2 )4 + 15( 2 )2 + 1]
6
a1 = n Cr - 1, a2 = n Cr, a3 = n Cr + 1, a4 = n Cr + 3
= 198
We know that
Now 0 < 2 - 1 < 1 implies 197 < ( 2 + 1)6 < 198. There-
fore the integral part of ( 2 + 1)6 = 197.
n
CK n-K +1
n
=
Answer: (D) CK - 1 K
Therefore
20. The last term in (2 + 2-1 / 2 )n is (3-5 / 3 )log3 8 . Then, the
1/ 3

value of the fifth term is a2 n - r + 1 a n+1


= Þ1+ 2 =
(A) 110 (B) 210 (C) 310 (D) 220 a1 r a1 r
a3 n - r a n+r
Solution: We are given that = Þ1+ 3 =
a2 r + 1 a2 r + 1
2- n / 2 = 3-5 / 3×( 3×log3 2 ) = 2-5 a4 n - r - 1 a n+1
= Þ1+ 4 =
Therefore, n = 10. The fifth term is a3 r+2 a3 r + 2
7 × 8 × 9 × 10 and hence
10
C4× (21 / 3 )6× (2-1 / 2 )4 = = 210
24 a1 a3 r r+2 æ r + 1ö æ a ö
Answer: (B) + = + = 2ç ÷ = 2ç 2 ÷
a1 + a2 a3 + a4 n + 1 n + 1 è n + 1ø è a2 + a3 ø
21. Let (1 - x + x2 )n = a0 + a1 x + a2 x2 + + a2 n x2 n. If a0, a1, This gives that the following are in AP:
a2, ¼, a2n are in AP, then an equals a1 a2 a3
1 1 , ,
(A) 2 n + 1 (B) (C) 2 n - 1 (D) a1 + a2 a2 + a3 a3 + a4
2n + 1 2n - 1 Answer: (A)
Solution: We have
23. If the middle term in the expansion of (1 + x)2 n is
2n + 1
1 = a0 + a1 + a2 + + a2 n = (a0 + a2 n ) K (2n/ n !) xn, then K is equal to
2
(A) (2n)! (B) 1× 3 × 5 (2 n - 1)
Therefore 1
(C) (2 n - 1)! (D) (2 n - 1)!
2 2
a0 + a2 n =
2n + 1 Solution: Since there are 2 n + 1 terms in the expan-
2 sion, the (n + 1)th term will be the middle term. Therefore
a0 + (a0 + 2 nd) = the middle term is given by
2n + 1
(where d is the common difference) (2 n)! n
( 2 n)
Cn xn = x
1 n! n!
a0 + nd =
2n + 1 1 × 2 × 3 2n n
= x
n! n!
www.jeeneetbooks.in
338 Chapter 7 Binomial Theorem

[1 × 3 × 5 (2 n - 1)][2 × 4 × 6 (2 n)]xn Solution: In the given identity, substituting x = 1 and


= x = -1 both sides and adding
n! n!
[1 × 3 × 5 (2 n - 1)]2n (n !) xn 2(a0 + a2 + a4 + + a40 ) = 240 + 220
=
n! n!
Therefore
Therefore K = 1× 3 × 5 (2n - 1).
a0 + a2 + a4 + + a40 = 239 + 219 (7 .9)
Answer: (B)
But a40 is the coefficient of x40 which is 220. Therefore
53
24. The coefficient of x in the expansion of from Eq. (7.9), we get
å
100 10 - K
K= 0
100
CK ( x - 3) ×2 . K
a0 + a2 + a4 + + a38 = 239 + 219 - 220 = 239 - 219
(A) - 100C53 (B) 100
C53 (C) - 100C52 (D) 100
C52 Answer: (C)
Solution: We have
28. The sum
100

å é1 ù
n
100
CK ( x - 3)100 - K × 2K = ( x - 3 + 2)100 = (1 - x)100 3 5 7
K =0
å (-1)
r=0
r n
Cr ê r + 2 r + 3r + 4 r + upto m terms ú
ë2 2 2 2 û
Therefore coefficient of x = - 53 100
C53 . is equal to
Answer: (A) 2mn + 1 2mn - 1
(A) (B)
25. The coefficient of xr in the expansion of (x + 3)n - 1 +
2mn (2n - 1) 2m (2n - 1)
(x + 3)n - 2(x + 2) + (x + 3)n - 3(x + 2)2 + + (x + 2)n - 1 is 22mn - 1 2mn - 1
(C) (D)
(A) n Cr (3n - r - 2n - r ) (B) n Cr - 1 (3n - r + 1 - 3n - r + 1 ) 2 (2m - 1)
mn
2 (2n - 1)
mn

(C) n Cr + 1 (3r - 2r ) (D) n Cr (3n - r + 2n - r ) Solution: We have


Solution: The terms of the given sum follow GP with r n n
æ 1ö æ 1ö æ 1ö
n
first term (x + 3)n-1 and common ratio (x + 2)/(x + 3). å (-1)r n Cr ç ÷ = ç 1 -
è 2ø è ÷ =ç ÷
2ø è 2ø
Therefore the given sum is r=0

r n n
{1 - [( x + 2)/( x + 3)]n } n
æ 3 ö
n
æ 3ö æ 3ö æ 1ö
( x + 3)n - 1
1 - [( x + 2)/( x + 3)]
= ( x + 3)n - ( x + 2)n å
r=0
( -1)rn
Cr ç 2r ÷ =
è 2 ø r=0å (-1)r n Cr ç ÷ = ç 1 -
è 4ø è ÷ =ç ÷
4ø è 4ø

Therefore the coefficient of xr of the given sum is and so on. Therefore the given sum is
n-r n-r n-r n-r
n
Cn - r 3 - Cn - r 2
n
= Cr (3
n
-2 ) (∵ Cr = Cn - r )
n n n
æ 1ö æ 1ö æ 1ö æ 1 ö
n n n

çè ÷ø + çè ÷ø + çè ÷ø + çè ÷ø + upto m terms
Answer: (A) 2 4 8 16

26. The coefficient of x in the expansion of (1 + x + x + x ) is


8 2 3 4 æ 1ö é æ 1ö æ 1ö
n n 2n
æ 1ö
3n
ù
= ç ÷ ê1 + ç ÷ + ç ÷ + ç ÷ + upto m terms ú
è 2ø ë è 2ø è 2ø è 2ø û
(A) 30 (B) 31 (C) 32 (D) 36
Solution: We have 1 1 - (1/ 2n )m 2mn - 1
= =
2n 1 - (1/ 2n ) 2mn (2n - 1)
(1 + x + x2 + x3 )4 = (1 + x)4 (1 + x2 )4
Answer: (D)
= (1 + 4 x + 6 x2 + 4 x3 + x4 )
(1 + 4 x2 + 6 x4 + 4 x6 + x8 ) 29. Let p and q be positive integers. Let

Therefore the coefficient of x8 = 6 ´ 4 + 1 ´ 6 + 1 = 31. ì p!


ï p!×( p - q)! when p ³ q
Answer: (B)
p
Cq = í
ï0 when p < q
î
27. If (1 + x + 2x2)20 º a0 + a1x + a2 x2 + + a40 x40, then the
value of a0 + a2 + a4 + + a38 is Then the sum å r = 0 ( 10Cr ´ 20 Cm - r ) is maximum when
m

(A) 220 (220 - 1) (B) 220 (220 + 1) m is


(C) 239 - 219 (D) 239 + 219 (A) 5 (B) 10 (C) 15 (D) 20
www.jeeneetbooks.in
Worked-Out Problems 339

Solution: We have It is known that nCr is maximum if r = n/2 when n is even.


Therefore the given sum is maximum, if
(1 + x)10 = C0 + C1 x + C2 x2 + + C10 x10 (where Cr = 10Cr )
30
(1 + x)20 = C0 + C1 x + C2 x2 + + C20 x20 (where Cr = 20Cr ) m= = 15
2
Therefore
Answer: (C)
m

å(
r=0
10
Cr ´ 20Cm - r ) = coefficient of xm in the expansion of

(1 + x)10 (1 + x)20 = coefficient of xm in (1 + x)30 = 30Cm

Multiple Correct Choice Type Questions


log3 ( 25)x + 7
1. If the third term in the expansion of ( x + x
log10 x 5
) is 3. If the ninth term in the expansion of [3 +
x -1
-( 1/ 8 )log3 ( 5 + 1) 10
10,00,000, then the value(s) of x may be 3 ] is equal to 180 where x > 1, then x
(A) 10 (B) 102 (C) 105/2 (D) 10-5/2 value is
Solution: Put log10 x = y . Therefore (A) log5 3 + 1 (B) log5 15 (C) log5 3 + 2 (D) log10 15
5
C2 x3 ( xy )2 = 106 Solution: Put a = 25x - 1 + 7 = 5 2( x - 1) + 7 and b = (5x–1 +
1)-1/8. Therefore the ninth term is
x3 + 2 y = 105
(3 + 2 y) log10 x = 5 10
C8 a2 b8 = 45(52( x - 1) + 7)(5x - 1 + 1)-1 = 180
(3 + 2 y) y = 5 Substituting y = 5x - 1 in this we get
2 y + 3y - 5 = 0
2
y2 + 7 = 4( y + 1)
( y - 1)(2 y + 5) = 0 y2 - 4 y + 3 = 0
This gives y = 1 or 3
-5 Now
y = 1 or y=
2 y = 1 Þ 5x - 1 = 1 Þ x = 1 (reject as x > 1)
Therefore x = 10 or 10-5 / 2 . y = 3 Þ 5x - 1 = 3 Þ x = log5 15
Answers: (A), (D) Answers: (A) and (B)

2. If (1 + ax)n = 1 + 8 x + 24 x2 + , then 4. Which of the following statements are true?


(A) a = 3 (B) n = 4 (C) a = 2 (D) n = 5 (A) The digit at unit place in the number 171995 + 111995 -
Solution: We have 71995 is 1.
(B) (106)85 - (85)106 is divisible by 7.
(1 + ax)n = 1 + n C1 (ax) + n C2 (ax)2 +
(C) The positive integer which is just greater than
Therefore (1 + 0.0001)1000 is 2.
(D) If (1 + 2 x - 3 x2 )2010 = a0 + a1 x + a2 x2 + + a4020 x4020,
(nC1 )a = 8 Þ an = 8 then a0 + a2 + a4 + a6 + + a4020 is an even integer.
(nC2 )a2 = 24 Þ n(n - 1)a2 = 48 Solution:
Now, 48 = n(n - 1)a = (an)(an - a) = 8(8 - a).
2 (A) (17)1995 + 111995 - 71995 = (10 + 7)1995 + (10 + 7)1995 - 71995 =
Therefore 1 + (a multiple of 10) as 71995 and -71995 cancelled with
each other. Therefore the digit at the unit place is 1.
8-a=6
Therefore (A) is true.
a=2 (B) (106)85 - (85)106 = (1 + 105)85 - (1 + 84)106. When bino-
a=2 Þn=4 mially expanded 1, –1 will be cancelled and in the
remaining terms, 105 and 84 occur and are divisible
Hence the answer is a = 2, n = 4. by 7. Therefore (B) is true.
Answers: (B) and (C)
www.jeeneetbooks.in
340 Chapter 7 Binomial Theorem

(C) It is easy to see that Therefore I + F - G is an integer and 0 < F < 1, 0 < G < 1
1000 implies that F = G.
æ 1 ö Also [P] = I is an even integer and
1 < (1 + 0.0001)1000 = ç 1 + 4 ÷ <2
è 10 ø
PF = ( I + F )F = ( I + F )G
Therefore the integer just greater than [1 + (1/ 104 )]1000
is 2. Hence (C) is true. = (6 6 + 14)2 n + 1 (6 6 - 14)2 n + 1 = (20)2 n + 1
(D) Put x = 1 and –1 and add. Answers: (A) and (C)
0 + (- 4)2010 = a0 + a2 + a4 + a6 + + a4020
7. If (1 + x)n = C0 + C1x + C2x2 + + Cnxn, where Cr
Therefore (D) is true. denotes nCr, then
Answers: (A), (B), (C), (D) C0 C1 C C
(A) - + 2 - + (-1)n n
x x+1 x+2 x+n
5. If (7 + 4 3 )n = I + f , where I is an integer and 0 < f < 1,
n!
then =
x( x + 1)( x + 2) ( x + n)
(A) I is an even integer (B) I is an odd integer
for all x ¹ - 1, - 2, - 3, ¼
(C) ( I + f )(1 - f ) = 1 (D) f 2 - f + 1 = 0
C0 C1 C2 C3 C 1
Solution: By hypothesis (7 + 4 3)n = I + f. Let (7 - (B) - + - + + (-1)n n =
1 2 3 4 n+1 n+1
4 3)n = G, where 0 < G < 1. Therefore
C0 C1 C2 C3 Cn
( I + f ) + G = 2[7n + nC2 7n - 2 ( 3 )2 + nC4 7n - 4 ( 3 )4 + ] (C) - + - + + (-1)n
2 3 4 5 n+2
When 0 < f + G < 2 and I + f + G is an integer Þ f + G = 1. 1
Therefore =
(n + 1)(n + 2)
I = 2 (some integer) – 1 is an odd number (D) C0 - C1 + C2 - C3 + + (-1)n Cn = 0
Hence (B) is true. Again Solution: We prove (A) by mathematical induction.
( I + f )(1 - f ) = ( I + f )G = (7 + 4 3 ) (7 - 4 3 )
n n For n = 1,

= (49 - 48)n = 1 C0 C1 1 1 1 1!
- = - = =
x x + 1 x x + 1 x( x + 1) x( x + 1)
Therefore (C) is true.
Answers: (B) and (C) Assume for n, that is

6. If (6 6 + 14)
2n+1
= P, then C0 C1 C2 C3 Cn
- + - + + (-1)n
x x+1 x+2 x+3 x+n
(A) [P] is an even integer (B) PF = 22 n + 1
n!
(C) PF = 202 n + 1 (D) [P] is an odd integer =
x( x + 1)( x + 2) ( x + n)
Note: [·] denotes integral part and F is the fractional
part of P. Change x to x + 1 on both sides
Solution: Let
C0 C1 C2 Cn
2n+1 - + - + (-1)n
(6 6 + 14) =P=I+F x+1 x+2 x+3 x+n+1
where I = [ P ] and F = P - [ P ]. Let n!
=
( x + 1)( x + 2) ( x + n + 1)
(6 6 - 14)2 n + 1 = G
so that 0 < G < 1. Now On subtraction and using nCr + nCr -1 = (n + 1)Cr , we have

P - G = 2[(2 n + 1)C1 (6 6 )2 n (14) (n + 1)C0 (n + 1)C1 (n + 1)C2 (n + 1)C3


- + - +
+ (2n + 1)C3 (6 6 ) 2 n- 2
´ (14) + ]
2 x x+1 x+2 x+3
= 2 (some positive integer) (n + 1)Cn + 1
+ (-1)n
x+n+1
www.jeeneetbooks.in
Worked-Out Problems 341

n ![( x + n + 1) - x] 9. Let Cr be the binomial coefficient in the expansion of


= (1 + x)n. Then
x( x + 1)( x + 2) ( x + n + 1)
C0 C2 C4 C6 2n
(n + 1)! (A) + + + + =
= 1 3 5 7 n+1
x( x + 1)( x + 2) ( x + n + 1)
The result is also true for n + 1. Hence (A) is true. C1 C3 C5 2n - 1
(B) + + + =
In (A), by substituting x = 1, 2 we see that (B) and (C) 2 4 6 n+1
are true.
In (1 + x)n expansion, putting x = –1 we get (D) is true. (C) C0C1 + C1C2 + C2C3 + + Cn-1Cn = 2 nCn+1
Answers: (A), (B), (C), (D) (D) If sn = C0C1 + C1C2 + C2C3 + + Cn-1Cn and if
sn + 1/sn = 15/4, then n = 2 or 4.
8. If (1 + x)n = C0 + C1x + C2x2 + + Cnxn, where Cr =
n
Cr, then Solution:
(A) a × C0 + (a + d)× C1 + (a + 2d) × C2 + + (a + nd)× Cn (A) We have

= 2n-1 (2a + nd) C2 r


=
n!
= ( n + 1) C( 2 r + 1)×
1
(B) 1× C0 + 2 × C1 + 3 × C2 + + (n + 1) × Cn = 2 (n + 1) n 2r + 1 (n - 2r )!(2r )!(2r + 1) (n + 1)
(C) 1× C0 + 2 × C1 + 3 × C2 + + (n + 1) × Cn = 2n - 1 (2n + 1) Therefore
C1 C C C n(n + 1) C2 r 1
+ 2 × 2 + 3× 3 + + n n =
(D)
C0 C1 C2 Cn - 1 2 å 2r + 1 = å ( n + 1)
C( 2 r + 1)×
(n + 1)
Solution: 1 ( n+1)
= [ C1 + ( n+1)C3 + ( n+1)C5 + ]
(A) Let n+1
1 ( n+1)
s = a · C0 + (a + d) · C1 + (a + 2d) · C2 + + (a + nd) · Cn = 2
n+1
Since Cr = Cn - r we have
2n
= [see Theorem 7.2 part (3)]
s = (a + nd) × C0 + [a + (n - 1)d]× C1 + + a × Cn n+1
By adding both the above equations we get Therefore (A) is true.
2 s = (2a + nd)[C0 + C1 + + Cn ] = (2a + nd)2n C2 r - 1 1 ( n + 1)
(B) = × C2 r
2r n+1
Therefore
Therefore
s = 2n - 1 (2a + nd)
C2 r - 1 1 ( n + 1) 1
Therefore (A) is true.
Putting a = 1 = d, we get
å 2r
=
n+1
[ C2 + ( n + 1) C4 + ] =
n+1
(2n - 1)

s = 2n - 1 (n + 2) Therefore (B) is true.

Therefore (B) and (C) are not true. (C) (1 + x)2 n = (1 + x)n (1 + x)n
(D) Let = (C0 + C1 x + + Cn xn )
Cr n! (n - r + 1)!(r - 1)! ´ (C0 xn + C1 xn - 1 + C2 xn - 2 + + Cn )
ur = r × = r× ×
Cr - 1 (n - r )! r ! n!
Equating the coefficients of xn + 1 (or xn - 1) both sides,
Therefore we get

n-r+1 C0 C1 + C1C2 + C2 C3 + + Cn - 1Cn = 2 n Cn + 1


ur = r × =n-r+1
r Therefore (C) is true.
n
Cr n n
n(n + 1)
å
r =1
r × = å
Cr - 1 r = 1
ur = å
r =1
(n - r + 1) =
2
(D) In (C) above, replace n with n + 1. Then
15 sn + 1 (2 n + 2)(2 n + 1)
Therefore (D) is true. = =
4 sn n(n + 2)
Answers: (A) and (D)
www.jeeneetbooks.in
342 Chapter 7 Binomial Theorem

Solving we get Therefore


n - 6n + 8 = 0
2
éæ n ö æ n - 1ö æ n - 2ö æ mö ù
s = êç ÷ + ç
è ø è ÷ø + çè m ÷ø + + çè m÷ø ú
(n - 2)(n - 4) = 0 ë m m û
n = 2 or 4 éæ n - 1ö æ n - 2ö æ mö ù
Hence (D) is true.
+ êç
è m ÷ø + çè m ÷ø + + çè m÷ø ú
ë û
Answers: (A), (B), (C), (D)
éæ n - 2ö æ n - 3ö æ mö ù
+ êç ÷ø + çè m ÷ø + + çè m÷ø ú
10. For any positive integers m, n (with m £ n). Let ë è m û
æ nö n n!
çè m÷ø = Cm = m!(n - m)!

Then æ n + 1ö
First row sum = ç
æ n ö æ n - 1ö æ n - 2ö æ mö æ n + 1 ö è m + 1÷ø
(A) ç ÷ + ç ÷ +ç ÷ + + ç ÷ = ç
è mø è m ø è m ø è mø è m + 1÷ø æ n ö
Second row sum = ç
è m + 1÷ø
æ nö æ n - 1ö æ n - 2ö
(B) ç ÷ + 2 ç ÷ + 3× ç +
è mø è m ø è m ÷ø æ n - 1ö
Third row sum = ç , etc.
è m + 1÷ø
æ mö æ n + 2 ö
+ (n - m + 1) × ç ÷ = ç
è mø è m + 2÷ø Hence

æ nö æ n - 1ö æ n - 2ö æ n + 1ö æ n ö æ n - 1ö æ m + 1ö æ n + 2 ö
(C) ç ÷ + 2 ç + 3× ç + s=ç + + + + ç =
è mø è m ø ÷ è m ÷ø è m + 1÷ø çè m + 1÷ø çè m + 1÷ø è m + 1÷ø çè m + 2÷ø

æ mö æ n + 1 ö Therefore (B) is true.


+ (n - m + 1) × ç ÷ = ç
è mø è m + 2÷ø (D) In (A), take n = 10 and m = 2. So, (D) is true.
Answers: (A), (B), (D)
æ 10ö æ 9ö æ 8ö æ 2ö æ 11ö
(D) ç ÷ + ç ÷ + ç ÷ + + ç ÷ = ç ÷
è 2 ø è 2ø è 2ø è 2ø è 3 ø 11. Let (1 + x) = C0 + C1x + C2x + + Cnx where Cr
n 2 n

n
means Cr. Then
Solution:
(2 n)!
(A) We have (A) C02 + C12 + C22 + + C2n =
(n !)2
æ n ö æ n - 1ö æ n - 2ö æ mö
çè m÷ø + çè m ÷ø + çè m ÷ø + + çè m÷ø (B) a × C02 + (a + d) × C12 + (a + 2d) × C22 +
(2n - 1)!
= Coefficient of xm in the expression + (a + nd) × C2n =
n !(n - 1)!
(1 + x)n + (1 + x)n-1 + (1 + x)n- 2 + + (1 + x)m
(C) Sum of the products of C0, C1, C2,¼, Cn taken two
= nCm + ( n-1)Cm + ( n- 2 )Cm + + mCm at a time is equal to 22n - 1 - (2n - 1)!/[n!(n - 1)!]
= [ mCm + ( m+1)Cm ] + [ ( m+ 2 )Cm + ( m+ 3)Cm + + nCm ] (D) C2 + 2 × C3 + 3 × C4 + + (n - 1) × Cn = 1 + (n - 2)2n - 1
= [ ( m+1)Cm+1 + ( m+1)Cm ] + [ ( m+ 2 )Cm + ( m+ 3)Cm + + nCm ] Solution:

= [ ( m+ 2 )Cm+1 + ( m+ 2 )Cm ] + [ ( m+ 3)Cm + ( m+ 4 )Cm + + nCm ] (A) (1 + x)2 n = (1 + x)n (1 + x)n

= [ ( m+ 3)Cm+1 + ( m+ 3)Cm ] + [ ( m+ 4)Cm + + nCm ] = (C0 + C1 x + C2 x2 + + Cn xn )

Finally nCm + 1 + nCm = ( n + 1)Cm + 1 . ´ (C0 xn + C1 xn - 1 + + Cn ) (∵Cr = Cn - r )

Therefore (A) is true. Equating coefficient of xn on both sides


(B) Let 2n
Cn = C02 + C12 + C22 + + Cn2
æ nö æ n - 1ö æ n - 2ö æ mö
s = ç ÷ + 2ç ÷ + 3× ç ÷ + + (n - m + 1) × ç ÷ Therefore (A) is true.
è mø è m ø è m ø è mø
www.jeeneetbooks.in
Worked-Out Problems 343

(B) Let (2 n)!


= 22n -
s = a × C02 + (a + d) × C12 + (a + 2d) × C22 + + (a + nd) × C2n (n !)(n !)

and s = (a + nd) × C02 + [a + (n - 1)d]× C12 So (C) is true.


(D) We have
+ [a + (n - 2)d]× C22 + + a × C2n
s = C2 + 2 × C3 + 3× C4 + + (n - 1)× Cn
Therefore
= [(-1)× C0 + 0 × C1 + 1× C2 + 2 × C3 +
2 s = (2a + nd)(C02 + C12 + C22 + + C2n )
+ (n - 1)× Cn ] + 1 (∵ C0 = 1)
(2 n)!
= (2a + nd)
(n !)(n !) = [-1 + (n - 1)]× 2n-1 + 1 [see Q38 part (A)]
(2 n - 1)!
s = (2a + nd) = 1 + (n - 2)× 2n-1
(n !)(n - 1)!
Therefore (D) is true.
Hence (B) is true.
Answers: (A), (B), (C), (D)
(C) We have
2å Ci C j = (C0 + C1 + C2 + + Cn )2
0£i< j £ n

- (C02 + C12 + C22 + + Cn2)

Matrix-Match Type Questions


1. Match the items of Column I with those of Column II. (B) The coefficient of a6 is given by
(1 + a)6 + (1 + a)7 + + (1 + a)15 = (1 + a)6 [1 + (1 + a)
Column I Column II
+ (1 + a)2 + + (1 + a)9 ]
(A) The term independent of x in the (p) 17
C 8 + 17 C 7
expansion of (1 - x)2 [ x + (1/ x)]10 [(1 + a)10 - 1]
= (1 + a)6
is (1 + a) - 1
(B) The coefficient of a6 in the (q) 16
C7 (1 + a)16 - (1 + a)6
expansion of (1 + a)6 + =
a
(1 + a)7 + + (1 + a)15 is
Therefore coefficient of a6 = 16 C7 .
(C) The coefficient of x8 y10 in the (r) 11 C5
expansion of (x + y)18 is Answer: (B) Æ (q)
18 - r
(C) We have Tr + 1 = Cr x y . Therefore the coefficient
18 r
1 1 1 1 210
(D) + + + + equals (s) of x8 y10 will be obtained when r = 10 and the coef-
1!10 ! 3! 8 ! 5! 6 ! 11! 11! ficient is 18 C10 = 18 C8 = 17 C8 + 17 C7 .
Answer: (C) Æ (p)
Solution:
(D) The given sum is
(A) We have
1 é 11! 11! 11! 11! ù
é 10 10 ù + + + +
9 æ 1ö 11! êë 1!10 ! 3! 8 ! 5! 6 ! 11! úû
10 ê x + C1 x çè x ÷ø ú
æ 1ö
(1 - x)2 ç x + ÷ = (1 - 2 x + x2 ) ê ú
=
1 11
é C1 + 11C3 + 11C5 + + 11C11 ùû
è xø ê ú
11! ë
2
ê + 10 C2 x8 æç ö÷ + +
1 1 ú
ëê è xø x10 úû 210
=
Therefore the term independent of x is 11!
Answer: (D) Æ (s)
10
C5 + (-2)(0) + 10 C6 = 11C6 = 11C5
Answer: (A) Æ (r)
www.jeeneetbooks.in
344 Chapter 7 Binomial Theorem

2. Match the items of Column I with the items of Column II. n


C p = nC p - 1
2p - 1 = n
Column I Column II
æ 3n - 5 ö
2ç =n+1
(A) If the sum of the coefficients in (p) 1 è 5 ÷ø
the expansion of (a2 x2 - 2ax + 1)50
is zero, then the value of n = 15
a + 2a + 3a + + 10a is Answer: (C) Æ (q)
( 1024 - r ) / 2
(B) If x = (3 + 2 2 ) = [ x] + f where
6
(q) 15 (D) We have Tr + 1 = (1024)Cr 5 × 7r / 8 is an integer.
[x] is the integral part of x and f = Therefore r is a multiple of 8. Since 0 £ r £ 1024 , the
x – [x], then x (1 – f ) is number of multiples of 8 which lie between 0 and
1024 (both inclusive) is 129.
(C) If the sum of the coefficients of (r) 129
x5 and x10 in the expansion of Answer: (D) Æ (r)
[ x3 - (1/ x2 )]n is zero, then n equals 3. Match the items of Column I with the items of Column II.
(D) Number of integral terms in the (s) 55
expansion of ( 5 + 8 7 )1024 is Column I Column II

Solution: (A) Coefficient of xn in the (p) (2n)Cn


expansion of (1 + x)2n is
(A) Sum of the coefficients in the expansion of (ax2 -
2ax + 1)50 = (a - 1)50 = 0 Þ a = 1. Therefore (B) Coefficient of xn in the (q) 2n
expansion of ( x2 + 2 x)n is
10 × 11
a + 2a + 3a + + 10a = 1 + 2 + 3 + + 10 = = 55 (C) Coefficient of xn in the (r) n C1 ´ ( n - 1)C1 ´ 2n - 2
2
expansion of n( x2 + 2 x)n - 1
Answer: (A) Æ (s) is
(B) Let x = (3 + 2 2 ) = [ x] + f and g = (3 - 2 2 )6 . Then
6
(D) Coefficient of xn in (s) nC2 ´ ( n - 2 )C2 ´ 2n - 4
[ x] + f + g = x + g = 2 (some integer) the expansion of
n
C2 ( x2 + 2 x)n - 2 is
Now 0 < f, g < 1 and [x] + f + g is an integer. This
implies f + g = 1. Therefore Solution:
x(1 - f ) = xg = (3 + 2 2 ) (3 - 2 6 ) = 1
6 6
(A) (1 + x)2 n = 2 nC0 + 2 nC1 x + 2 nC2 x2 + + 2 nC2 n x2 n
Answer: (B) Æ (p) Therefore coefficient of xn = 2 nCn .
(C) We have Answer: (A) Æ (p)
r (B) Tr + 1 = (r + 1)th term in the expansion of ( x2 + 2 x)n
æ -1ö
Tr + 1 = Cr ( x ) × ç 2 ÷ = (-1)r ×nCr x3 n - 5r
n 3 n-r
èx ø = nCr ( x2 )n - r × (2 x)r = nCr x2 n - r × 2r

Now 2n - r = n Þ r = n

3n - 5 Therefore coefficient of xn = nCn× 2n = 2n.


3n - 5r = 5 Þ r = =p (say) (7.10)
5 Answer: (B) Æ (q)

3n - 10 (C) Tr + 1 = [(n - 1)Cr ( x2 )n - 1- r × (2 x)r ]n


3n - 5r = 10 Þ =q (say) (7.11)
5 = (n - 1)Cr x2n - 2 - r × 2r × n
Therefore from Eqs. (7.10) and (7.11), 2n - 2 - r = n Þ r = n - 2
p–q=1 (7.12) Therefore coefficient of xn in n( x2 + 2 x)n - 1 is
( n - 1)
Hence Cn - 2× n × 2n - 2 = nC1 ´ ( n - 1)C1× 2n - 2
Answer: (C) Æ (r)
n
Cp (-1)p + nCq (-1)q = 0
p-1
(D) Similarly, the coefficient of xn in nC2 ( x2 + 2 x)n - 2 is
n
Cp (-1) + Cp - 1 (-1)
p n
=0
n
C2 ´ ( n - 2 )C2 ´ 2n - 4
Answer: (D) Æ (s)
www.jeeneetbooks.in
Worked-Out Problems 345

4. Match the items of Column I to the items of Column II, Solution: It is known that (from Q7 in Multiple Correct
if (1 + x)n = C0 + C1 x + C2 x2 + + Cn xn where Cr is nCr . Choice Type Questions)
C0 C1 C2 C3
Column I Column II - + - +
x x+1 x+2 x+3

(A) C0 - C1 + C2 - C3 + 2n Cn n!
(p) + (-1)n× =
1 2 3 4 1× 3 × 5 × (2 n + 1) x + n x( x + 1)( x + 2) ( x + n)
C
+ (-1)n× n is for all x ¹ 0, - 1, - 2, - 3, …
n+1
2n Put x = 1, 2, 1/2 and -1/2 to get the result. This is a simple
(B) C0 - C1 + C2 - C3 + (q) exercise left to the students.
2 3 4 5 1× 3 × 5 × (2 n - 1)
C Answer: (A) Æ (r), B Æ (s), (C) Æ (p), (D) Æ (q)
+ (-1)n× n is
n+2

æ C0 C1 C2 C3 ö 1
- + - (r)
1ç 1 3 5 7 ÷ n+1
(C) ç ÷ is
n! ç Cn ÷
çè + + (-1) × 2 n + 1÷ø
n

(D) 1
(s)
(n + 1)(n + 2)
é C1 C2 C3 C4 ù
ê - C0 - 1 + 3 - 5 + 7 ú
-1 ê ú is
n! ê Cn ú
ê + + (-1) ×
n
ú
ë 2n - 1 û

Comprehension-Type Questions
1. Passage: In the expansion of (x + a)n, the general term Therefore
is nCrxn - r·ar and the number of terms in the expansion
n(n - 1)(n - 2) n - 3 n - 6 5
is n + 1. Answer the following three questions: × p ×x =
6 2
(i) If the fourth term in the expansion of [px + (1/x)]n
is 5/2, then np is equal to Now n = 6 (since the term is independent of x),
(A) 4 (B) 3 (C) 9/2 (D) 10 hence
(ii) The number of terms in the expansion of 6 × 5× 4 3 5
×p =
( x + y + z)n is 6 2
n(n + 1) 3
(A) (B) (n + 1)(n + 2) æ 1ö 1
2 p3 = ç ÷ Þ p =
è 2ø 2
(n + 1)(n + 2)
(C) (D) (n + 2)(n + 3) Therefore np = 3.
2
Answer: (B)
(iii) The coefficient of x5 in the expansion of [3x2 -
n-1
(1/3x3)]10 is (ii) ( x + y + z) = x + C1 x
n n n
( y + z)
(A) –9520 (B) 9520 (C) 9720 (D) –9720 + nC2 xn - 2 ( y + z)2 + + ( y + z)2
Solution: It can be observed that second, third, fourth, ¼,
(i) The fourth term in the expansion of [px + (1/x)]n is (n + 1)th terms contain 2, 3, 4, ¼, (n + 1) terms
5/2, that is in their respective expansions. Therefore the
number of terms in the given expansion is
æ 1ö 5
n
C3 ( px)n - 3× ç 3 ÷ =
èx ø 2 (n + 1)(n + 2)
1 + 2 + 3 + + (n + 1) =
2
Answer: (C)
www.jeeneetbooks.in
346 Chapter 7 Binomial Theorem

(iii) We have So
æ 1 ö
r
(15 + 1)(1/ 3) 16
Tr + 1 = 10Cr (3 x2 )10 - r ç - 3 ÷ p= = =4
è 3x ø 1 + (1/ 3) 4
1 is an integer. Therefore numerically greatest terms
= 10Cr 310 - r (-1)r x20 - 5r × are fourth and fifth terms.
3r
x5 = x20 - 5r Þ r = 3 Answer: (A)
(iii) Consider (1 + 1)20. Then
Therefore the coefficient of x5 is
(20 + 1)1
æ 8 × 9 × 10 ö p=
-( C3× 3 ) = - ç
10 4
× 81 = -(120 ´ 81) = - 9720 1+ 1
è 6 ÷ø
Answer: (D) Therefore [ p] = 10 and hence 11th term is greatest
and its value is 20C10.
2. Passage: Let (1 + x) = C0 + C1x + C2x + + Cnx
n 2 n
Answer: (C)
where Cr = Cr . For a given value of x, let p = (n + 1)| x |/
n

(| x | + 1). If p is an integer, then the numerical values 3. Passage: If n is a positive integer, x and a are real
of pth and (p + 1)th terms are equal and they are the (complex), then (r + 1)th term in the expansion of
numerically greatest terms in the expansion of (1 + x)n. (x + a)n is nCr xn - r × ar. Answer the following questions:
If p is not an integer and [ p] denotes the integral part (i) If the coefficient of x7 in [ax2 + (1/ bx)]11 is equal
of p, then ([ p] + 1)th term is numerically greatest term.
to the coefficient of x–7 in [ax - (1/ bx2 )]11 , then
Answer the following questions:
(i) The value of numerically greatest term in the expa- (A) ab = 1 (B) ab = –1 (C) ab = 2 (D) ab = –2
nsion of 2 [1 + (1/ 2 )]16 is (ii) If the coefficients (2r + 4)th and (r – 2)th terms in the
expansion of (1 + x)18 are equal, thus r is equal to
1 16 1 16
(A) × C7 (B) × C8 (A) 5 (B) 4 (C) 6 (D) 7
16 16 50
(iii) Coefficient of x in the expansion of
1 16 1 16 (1 + x)41(1 - x + x2 )40 is
(C) × C8 (D) × C7
18 8 (A) –1 (B) 1 (C) 40 (D) 0
(ii) Numerically greatest term in the expansion of Solution:
(3 - 5 x)15 when x = 1/ 5 is (are)
(i) We have that the (r + 1)th term is
(A) fourth and fifth term (B) sixth term
r
(C) seventh term (D) eighth term æ 1ö 1
11
Cr (ax2 )11- r ç ÷ = 11Cr a11- r × r × x22 - 3r
è bx ø b
(iii) The greatest value of 20 Cr (0 £ r £ 20) is
22 - 3r = 7
(A) 20C8 (B) 20C9 (C) 20C10 (D) 20C7
Solution: r=5

(i) We have Therefore the coefficient of x7 in the first expansion


is 11 C5 (a6 / b5 ) .
(16 + 1)(1/ 2 ) Similarly the coefficient of x–7 in the second
p= = 17( 2 - 1)
(1/ 2 ) + 1 expansion is 11 C6 (a5 / b6 ) . Therefore

Therefore [ p] = 7. The eighth term is numerically æ a6 ö æ a5 ö


11
C5 ç 5 ÷ = 11C6 ç 6 ÷ Þ ab = 1
greatest and its value is èb ø èb ø
7
æ 1 ö 1 16 Answer: (A)
2 ×16C7× ç = × C7
è 2 ÷ø 8 (ii) By hypothesis,
Answer: (D) 18
C2 r + 3 = 18Cr - 3
(ii) We have
15 15
2r + 3 ¹ r - 3
æ 5x ö æ 1ö æ 1ö
(3 - 5 x) = 3 ç 1 - ÷ = 315 ç 1 -
15 15
÷ çè∵ x = ÷ (2r + 3) + (r - 3) = 18
è 3 ø è 3ø 5ø
r=6
Answer: (C)
www.jeeneetbooks.in
Worked-Out Problems 347

(iii) We have (ii) Let


(1 + x)41 (1 - x + x2 )40 = (1 + x)(1 - x3 )40 Tr +1 = (- 1)r×(r + 1) × Cr
The power of x in this expansion is of the form either = (- 1)r× r × Cr + (- 1)r× Cr
3r or 3r + 1 but 50 is of the form 3k + 2. Hence, coef-
ficient of x50 is zero. = (- 1)r× n ×(n - 1)Cr -1 + (- 1)r× Cr for r = 1, 2, 3, …, n
Answer: (D) Therefore the given sum is
n n
4. Passage: If n is a positive integer, then C0 + nå (-1)r × (n - 1)Cr - 1 + å (-1)r × Cr
r =1 r =1
(1 + x)n = C0 + C1 x + C2 x2 + + Cn xn
n-1
= C0 - (1 - 1) + [(1 - 1) - C0 ] = 0
n

= C0 xn + C1 xn - 1 + C2 xn - 2 + + Cn
Answer: (A)
where Cr means nCr. Answer the following questions: Aliter: (1 + x)n = C0 + C1 x + C2 x2 + + Cn xn
(i) C0× C2 + C1× C3 + C2× C4 + + Cn - 2× Cn =
Therefore
(A) 2nCn (B) 2nCn - 2 (C) 2nCn + 1 (D) 22n
(ii) C0 - 2 × C1 + 3 × C2 - 4 × C3 + + (-1)n× (n + 1) × Cn = x(1 + x)n = C0 x + C1 x2 + C2 x3 + + Cn xn + 1
(A) 0 (B) 22n - 1 Differentiating both sides with respect to x and then
(C) (2 n)Cn - 1 (D) 2n - 1 (n + 1) substituting x = –1 on both sides we get
(iii) (C1 )2 + 2 × (C2 )2 + 3 × (C3 )2 + + n × (Cn )2 = 0 = C0 - 2 × C1 + 3 × C2 - 4 × C3 + + (-1)n× (n + 1) × Cn
(A) (2n)!/n!n! (iii) Let s = 0 × C02 + 1× C12 + 2 × C22 + + n × C2n
(B) (2n)!/n!(n - 1)!
Therefore
(C) coefficient of xn–1 in n(1 + x)2n–1
(D) (n + 1) × 2n s = n × C02 + (n - 1) × C12 + (n - 2) × C22 + + 0 × C2n

Solution: (∵ Cr = Cn - r )
(i) (1 + x)2 n = (1 + x)n ( x + 1)n This gives

= (C0 + C1 x + C2 x2 + + Cn xn ) 2 s = n[C02 + C12 + C22 + + Cn2 ] = n( 2 n Cn )


Hence
(C0 xn + C1 xn - 1 + + Cn )
n 2n n é (2 n)! ù é (2 n - 1)! ù
Equating the coefficients xn – 2(or xn + 2) on both sides s= × Cn = ê = nê ú
we get 2 2 ë n ! n ! úû ë (n - 1)! n ! û
2n
Cn - 2 = C0× C2 + C1× C3 + C2× C4 + + Cn - 2× Cn = Coefficient of xn - 1 in n(1 + x)2 n - 1
Answer: (B) Answer: (C)

Assertion–Reasoning Type Questions


In the following set of questions, a Statement I is given Statement II: n Cr - 1 + n Cr = ( n + 1) Cr
and a corresponding Statement II is given just below it.
Solution: We have
Mark the correct answer as:
n! n!
(A) Both I and II are true and II is a correct reason for I n
Cr - 1 + n Cr = +
(n - r + 1)!(r - 1)! (n - r )!(r )!
(B) Both I and II are true and II is not a correct reason
for I n !(r + n - r + 1)
=
(C) I is true, but II is false r !(n - r + 1)!
(D) I is false, but II is true (n + 1)!
=
1. Statement I: If Cr is the binomial coefficient in the r !(n + 1 - r )!
expansion of (1 + x)n, then = ( n + 1) Cr
Cr + 2 × Cr - 1 + Cr - 2 = ( n + 2 )Cr for 2 £ r £ n Therefore Statement II is true.
www.jeeneetbooks.in
348 Chapter 7 Binomial Theorem

Cr + 2 × Cr - 1 + Cr - 2 = (Cr + Cr - 1 ) + (Cr - 1 + Cr - 2 ) Again multiplying both sides with x and differentiating


w.r.t. x we get
= (n Cr + n Cr - 1 ) + (n Cr - 1 + n Cr - 2 )
(1 + x)n + 3nx(1 + x)n - 1 + n(n - 1) x2 (1 + x)n - 2
= ( n + 1)Cr + ( n + 1) Cr - 1
= C0 + 22× C1 x + 32× C2 x2 + + (n + 1)2× Cn xn
= ( n + 2 )Cr
Substituting x = –1 on both sides, we have
Hence Statement I is also true and Statement II is a correct
explanation of Statement I. 0 = C0 - 22 · C1 + 32 · C2 - 42 · C3 + + (-1)n · (n + 1)2 · Cn
Answer: (A)
Hence Statement I is true and Statement II is true. Also
Statement II is a correct explanation of Statement I.
2. Statement I: If
Answer: (A)
2n 2n

å a ( x - 2) = å b ( x - 3)
r=0
r
r

r=0
r
r
4. Statement I: If (1 + x)n = C0 + C1 x + C2 x2 + + Cn xn ,
then
and ak = 1 for all k ³ n, then bn = ( 2 n + 1)Cn + 1.
12× C1 + 22× C2 + 32× C3 + + n2× Cn = n(n + 1)2n - 2 for n ³ 1
Statement II: Two polynomials of same degree are
equal, if the corresponding coefficients are equal. Statement II: Any polynomial function in x is differ-
entiable for all real values of x.
Solution: Statement II is true according to the defini-
tion of equality of polynomials. Put x – 3 = y. Therefore Solution: Clearly Statement II is true:
2n 2n
(1 + x)n = C0 + C1 x + C2 x2 + + Cn xn
å a ( y + 1) = å b y
r=0
r
r

r=0
r
r

Differentiating both sides w.r.t. x we get


n
Equating coefficient of y on both sides, we get
n(1 + x)n - 1 = C1 + 2 × C2 x + 3 × C3 x2 + + n × Cn xn - 1
( n + 1) (n+ 2)
an + C1× an + 1 + C2× an + 2 + + ( 2 n)
Cn× a2 n = bn
Now, multiplying both sides with x, differentiating both
( n + 1) ( n + 1) (n+ 2)
C0 + C1 + C2 + + ( 2 n)
Cn = bn (∵ ak = 1 for k ³ n) sides w.r.t. x, and then substituting x = 1 on both sides
we get
Using nCr + nCr - 1 = ( n + 1)Cr, we have ( 2 n + 1)Cn = bn
Hence Statement I is also true and Statement II is a 12× C1 + 22× C2 + 32× C3 + + n2× Cn = n(n + 1)2n - 2
correct explanation of Statement I.
Hence both statements are true and Statement II is a
Answer: (A) correct explanation of Statement I.
Answer: (A)
3. Statement I: If (1 + x) = C0 + C1 x + C2 x2 + + Cn x ,
n n

then
5. Statement I: If n is an even positive integer and
C0 - 22× C1 + 32× C2 - 42× C3 + K = 3n / 2 then

+ (-1)n× (n + 1)2× Cn = 0 for n ³ 2 K

å (-3) r - 1 ( 3n)
× C2 r - 1 = 0
Statement II: Any polynomial function in x is differ- r =1

entiable for all real values of x.


Statement II: If m is a positive integer, and q is real
Solution: Statement II is true is clear. then (cos q + i sin q )m = cos mq + i sin mq .
(1 + x)n = C0 + C1 x + C2 x2 + + Cn xn Solution: We have
Therefore æ p pö
1 + i 3 = 2 ç cos + i sin ÷ (where i = -1)
è 3 3ø
x(1 + x)n = C0 x + C1 x2 + C2 x3 + + Cn xn + 1
Differentiating both w.r.t. x we get By De Moivre’s Theorem we have

(1 + x)n + nx(1 + x)n - 1 = C0 + 2 × C1 x + 3 × C2 x2 + é np np ù


(1 + i 3 )n = 2n êcos + i sin (7 .13)
ë 3 3 úû
+ (n + 1) × Cn
www.jeeneetbooks.in
Worked-Out Problems 349

Let n = 2m where m is a positive integer. Replacing n n(n + 1)2 (n + 2)


with 6m in Eq. (7.13), we have =
12
26 m [cos(2 mp ) + i sin(2 mp )] = (1 + i 3 )6 m 6 × 72× 8
= 142 = 22× 72 =
Therefore 12

26 m = (1 + i 3 )6 m = 1 + 6 mC1 (i 3 ) + 6 mC2 (i 3 )2 This gives n = 6. Therefore sum of the coefficients in


the expansion of ( x - 3 x2 + x3 )6 = (1 - 3 + 1)6 = 1. Hence
+ 6 mC3 (i 3 )3 + + (i 3 )6 m Statement I is true and Statement II is not true.
Equating imaginary parts we get Answer: (C)

3[ 6 mC1 - 6 mC3 (3) + 6 mC5 32 - 6 mC7 33 +


7. Statement I: If (2n+1)
C0 + (2n+1)
C3 + (2n+1)
C6 + = 170,
(-1)3m-1 6 m 3m-1
C6m - 1 3 ]=i then n = 4.
K
3n Statement II: If w in non-real cube root of unity, w3 = 1
å (-3)r -1 (3n)C2r -1 = 0
r =1
where K =
2 and 1 + w + w2 = 0.

Hence both statements are true and Statement II is a Solution: We have


correct explanation of Statement I. (1 + x)2 n + 1 = ( 2 n + 1)C0 + ( 2 n + 1)C1 x + ( 2 n + 1)C2 x2 +
Answer: (A)
+ ( 2 n + 1)C2 n + 1 x2 n + 1
6. Statement I: If Put x = 1, w and w2 and add. We now have
2
n
æ C ö 22 n + 1 + (1 + w)2 n + 1 + (1 + w2 )2 n + 1
å K ç K ÷ = 196 where Cr = nCr
3

è CK - 1 ø
K =1
= 3[ ( 2 n + 1)C0 + ( 2 n + 1)C3 + ( 2 n + 1)C6 + ] (7.14)
then sum of the coefficients in the expansion of LHS of Eq. (7.14)
( x - 3 x2 + x3 )n is 1.
= 22 n+1 - w4 n+ 2 - w2 n+ 2 = 22 n+1 - w4 n+ 2 - w2 n+1
Cr n-r+1
Statement II: = From Eq. (7.14), we have
Cr - 1 r+1
22 n + 1 - wn + 2 - w2 n + 1 = 3 ´ 170 = 510
Solution: We have
Therefore
Cr n
C n! (r - 1)!(n - r + 1)! n - r + 1
= n r = ´ =
Cr - 1 Cr - 1 r !(n - r )! n! r 22 n + 1 - w3 (wn - 1 + w2( n - 1) ) = 510

Therefore Statement II is not true. Now 22 n + 1 - (wn - 1 + w2( n - 1) ) = 510 (7 .15)


2
n
æ C ö n
It is known that 1 + wn + w2 n = 3 or 0 according as n is
å
K =1
K3 ç K ÷ =
è CK - 1 ø
å K (n - K + 1)2
K =1
a multiple of 3 or not. Therefore wn - 1 + w2( n - 1) = 2 or - 1
according as n – 1 is a multiple of 3 or not.
n
If wn - 1 + w2( n - 1) = - 1, then from Eq. (7 .15)
= å K[(n + 1) - 2K (n + 1) + K
K =1
2 2
]
22 n+ 1 = 509
n n n
= (n + 1)2 å K - 2(n + 1) å K 2 + åK 3
which is not possible. Therefore
K =1 K =1 K =1
wn - 1 + w2( n - 1) = 2
(n + 1) n 2(n + 1)n(n + 1)(2 n + 1) n (n + 1)
3 2 2
= - + From Eq. (7.15),
2 6 4
n(n + 1)2 22 n+ 1 = 512 = 29
= [6(n + 1) - 4(2n + 1) + 3n]
12 which implies n = 4.
Answer: (A)
www.jeeneetbooks.in
350 Chapter 7 Binomial Theorem

Integer Answer Type Questions


1. Let Cr denote Cr . If
n
Solution: We have
20 20
Cr + 4 × Cr + 1 + 6 × Cr + 2 + 4 × Cr + 3 + Cr + 4 n + k æ 3 1 ö é æ 3 1 öù æ 3 1ö
çè x + 3 + 1÷ø = ê1 + çè x + 3 ÷ø ú = 1 + C1 çè x + 3 ÷ø
20
=
Cr + 3 × Cr + 1 + 3 × Cr + 2 + Cr + 3 r+k x ë x û x
2 20
then the value of K is . æ 1ö æ 1ö
+ 20C2 ç x3 + 3 ÷ + + 20C20 ç x3 + 3 ÷
Solution: The numerator is è x ø è x ø

(Cr + Cr + 1 ) + 3(Cr + 1 + Cr + 2 ) + 3(Cr + 2 + Cr + 3 ) + (Cr + 3 + Cr + 4 ) Therefore


= ( n+1)Cr +1 + 3×( n+1)Cr + 2 + 3×( n+1)Cr + 3 + ( n+1)Cr + 4 1 æ 1ö æ 1ö
2
æ 1ö
3 20

1, x3, ( x3 )2, ( x3 )3, …, ( x3 )20, , ,


3 ç 3÷ ç 3÷
, …, ç 3 ÷
x èx ø èx ø èx ø
= [ ( n+1)Cr +1 + ( n+1)Cr + 2 ] + 2 ×[ ( n+1)Cr + 2 + ( n+1)Cr + 3 ]
are all distinct whose number is 1 + 20 + 20 = 41.
+ [ ( n+1)Cr + 3 + ( n+1)Cr + 4 ]
Answer: 41
= ( n++ 2 )Cr + 2 + 2 ×( n+ 2 )Cr + 3 + ( n+ 2 )Cr + 4
4. The greatest value of the term independent of x in
=[ ( n+ 2 )
Cr + 2 + ( n+ 2 )
Cr + 3 ] + [ ( n+ 2)
Cr + 3 + ( n+ 2 )
Cr + 4 ] the expansion of [ x sin a + (cos a / x)]20 as a is real is
20
C10 2- b . Then b value is .
= ( n+ 3)Cr + 3 + ( n+ 3)Cr + 4 Solution: We have
r
= ( n+ 4 )Cr + 4 æ cos a ö
Tr + 1 = 20Cr ( x sin a )20 - r ç
è x ÷ø
Similarly, the denominator = ( n + 3)Cr + 3 . Therefore
(n+ 4) = 20Cr (sin a )20 - r (cos a )r x20 - 2 r
Cr + 4 n + K n+4 n+K
( n + 3)
= Þ = ÞK=4 This is independent of x. Since 20 - 2r = 0 so r = 10.
Cr + 3 r + K r+4 r+K
Therefore
Answer: 4
1
T11 = 20C10 (sin a cos a )10 = 20C10 (sin 2a )10 £ 20C10 2-10
2. If (1 + 2x + 3x ) = a0 + a1x + a2x + + a20 x20, then
2 10 2
210
a1 + a2 is equal to . and equality holds when 2a = (4 n ± 1)(p /2). Therefore
Solution: (1 + 2 x + 3 x ) = [1 + x(2 + 3 x)]10
2 10
b = 10.
= 1 + 10C1 x(2 + 3 x) + 10C2 x2 (2 + 3 x)2 + Answer: 10

Therefore 5. The number of rational terms in the expansion of


a1 = C1 ´ 2 = 20
10 (52 / 3 + 10-1 / 4 )20 is .
Solution: We have
and a2 = 10C2 ´ 22 + 3 ´ 10C1 = 180 + 30 = 210
Tr + 1 = 20Cr × (52 / 3 )20 - r (10-1 / 4 )r = 20Cr 5(160 - 11r )/ 12× 2- r / 4
Adding the two we get
which is rational when r = 8 and 20.
a1 + a2 = 230 Answer: 2
Answer: 230
6. The number of non-zero terms in the expansion of
3. The number of distinct terms in the expansion of ( 11 + 1)75 - ( 11 - 1)75 is .
20
æ 3 1 ö Solution: We have
çè x + 3 + 1÷ø
x
( 11 + 1)75 - ( 11 - 1)75 = 2[C1 ( 11)74 + C3 ( 11)72
when x is real and x ¹ ±1 is .
+ C5 ( 11)70 + ]
www.jeeneetbooks.in
Worked-Out Problems 351

Therefore number of non-zero terms is Therefore


( n + 1)
75 + 1 C2 + 2[ nC2 + ( n - 1)C2 + ( n - 2 )C2 + + 2C2 ]
= 38
2
= 12 + 22 + 32 + + n2
Answer: 38
Now substituting n = 10, we have
1 1 1 1 2K
7. If + + + + = then K value 11
C2 + 2[ 10C2 + 9C2 + 8C2 + + 2C2 ] = 12 + 22 + 32 + + 102
1!10 ! 3! 8 ! 5! 6 ! 11!1! 11!
is . 10 × 11× 21
= = 385
6
Solution: We have
Answer: 385
é 1 1 1 1 ù
11! ê + + + +
ë 1!10 ! 3! 8 ! 5! 6 ! 11!1! úû C15 2K - 1
15 15 15 15
C1 C3 C5
9. If + + + + = then K is
2 4 6 16 16
11 11 11 11
= + + + + equal to .
1!10 ! 3! 8 ! 5! 6 ! 11!
= 11C1 + 11C3 + 11C5 + 11C7 + 11C9 + 11C11 Solution: Since
( n + 1)
= sum of the even coefficient in the expansion Cr Cr + 1 r + 1
× xr = ×x
of (1 + x)11 r+1 (n + 1) x

= 210 for r = 0, 1, 2, … , n, we have

Therefore K = 10. C1 C C (1 + x)n + 1 - 1


C0 + x + 2 x2 + + n xn =
Answer: 10 2 3 n+1 x(n + 1)

Substituting x = 1, - 1 we have
8. C2 + 2[ 10C2 + 9C2 + 8C2 + + 2C2 ] is equal to
11
.
C1 C2 C 2n + 1 - 1
Solution: In general we prove that for any positive C0 + + + + n =
2 3 n+1 n+1
integer n ³ 2,
( n + 1) C1 C2 C3 (-1)n× Cn 1
C2 + 2[ nC2 + ( n - 1)C2 + ( n - 2 )C2 + + 2C2 ] C0 - + - + + =
2 3 4 n+1 n+1
= 12 + 22 + 32 + + n2
On subtraction we get
We know that n+1
éC C C ù 2 -2
K (K - 1) 2 ê 1 + 3 + 5 + ú =
K
C2 = ë2 4 6 û n+1
2
Therefore
Therefore
C1 C3 C5 2n - 1
2 ×K C2 = K 2 - K + + + =
2 4 6 n+1
Put K = n, n - 1, n - 2, … , 2 and add. We get
Putting n = 15, we get
2[ nC2 + ( n-1)C2 + ( n- 2 )C2 + + 2C2 ]
C1 C3 C5 C 215 - 1
+ + + + 15 =
= (1 + 2 + 3 + + n ) - (1 + 2 + 3 + + n)
2 2 2 2
2 4 6 16 16
n(n + 1) Hence K = 15.
= (12 + 22 + 32 + + n2 ) -
2 Answer: 15
= (12 + 22 + 32 + + n2 ) - ( n+1)C2
www.jeeneetbooks.in
352 Chapter 7 Binomial Theorem

SUMMARY
7.1 Binomial theorem: If n is a positive integer and a is 7.8 Most useful result: If Cr is the binomial coefficient in
any real or complex number, then the expansion of (1 + x)n, and x ¹ 0, -1, -2, -3,¼, then
( x + a)n = n C0 xn + n C1 xn - 1a + n C2 xn - 2 a2 + + n Cn an C0 C1 C2 C3 Cn
- + - + + (-1)n
x x+1 x+2 x+3 x+n
7.2 General term: The (r + 1)th term nCr xn - r ar is called
the general term in the expansion of (x + a)n. n!
=
x( x + 1)( x + 2) ( x + 3)
7.3 Number of terms:
For example,
(1) The number of terms in the expansion of (x + a)n
is n + 1. (1) When x = 1, then
n
(2) The number of terms in the expansion of (x + y + z)
is (n + 1)(n + 2)/2 which is ( n+ 2 ) C0 C1 C2 1
C2 . - + - =
1 2 3 n+1
(3) The number of terms in the expansion of (x1 +
x2 + + xK )n is ( n + K - 1) C( K - 1) . (2) When x = 2, then

7.4 Middle term(s): C0 C1 C2 1


- + - =
2 3 4 (n + 1)(n + 2)
(1) If n is even, then the [(n / 2) + 1]th term is the
middle term in the expansion of (x + a)n. 7.9 Greatest term: Consider the expansion of (1 + x)n.
(2) If n is odd, then [(n + 1)/ 2]th and [(n + 3)/2]th are
the middle terms. Let
(n + 1) x
7.5 Binomial coefficients: If n is a positive integer, then p=
(1 + x)n = nC0 + nC1x + nC2x2 + + nCnxn. The coef- 1+ x
ficients xr(r = 0, 1, 2, … n) viz. Then
n
C0, C1, C2, …, Cn are called binomial coefficients
n n n
(1) pth and ( p + 1)th terms are numerically equal
and they will be denoted by C0, C1, C2, … , Cn.
and they are the numerically greatest terms in
the expansion of (1 + x)n, if p is an integer.
7.6 Properties of binomial coefficients: Let Cr be the
binomial coefficient in the expansion of (1 + x)n for (2) If p is not an integer and [ p] denotes the inte-
r = 0, 1, 2, …, n. Then gral part of p, then ([ p] + 1)th term is numerically
(1) Cr = Cn−r greatest term.
(2) C0 + C1 + C2 + + Cn = 2 .
n
7.10 Greatest value of nCr:
(3) C0 + C2 + C4 + = C1 + C3 + C5 + = 2n-1
Greatest value of
(4) 1 · C0 + 2 · C1 + 3 · C2 + + (n + 1) · Cn = (n + 2)2
n-1

ìï n Cn / 2 if n is even
7.7 Useful formulae: Let C0, C1, C2, … , Cn be binomial
n
Cr = í
ïî C( n -1)/ 2 = C( n + 1)/ 2
n n
if n is odd
coefficients in the expansion of (1 + x)n. Then
(1) 1 · C1 + 2 · C2 + 3 · C3 + + n · Cn = n · 2
n−1
Binomial Theorem for Rational Index
(2) a · C0 + (a + d)·C1 + (a + 2d)·C2 + + (a + (n − 1)d)· Cn
= (2a + (n − 1)d) 2n-1 7.11 Theorem: If n is a rational number and -1 < x < 1, then
n
n n(n - 1) 2
(3) å r(r - 1) × C = n(n - 1)2
r =1
r
n- 2
(1 + x)n = 1 +
1!
x+
2!
x

n n(n - 1)(n - 2) 3
(4) å r × C = n(n + 1)2
r =1
2
r
n- 2 +
3!
x + + ¥

the general term is


(2 n )!
(5) C02 + C12 + C22 + + Cn2 = 2 nCn =
(n ! )2 n(n - 1)(n - 2) (n - r + 1) r
x
(2n)! r!
(6) C0Cr + C1Cr + 1 + C2Cr + 2 + + Cn - rCn =
(n - r )!× (n + r )!
www.jeeneetbooks.in
Exercises 353

7.12 Useful expansions: n n(n + 1) 2


(3) (1 - x)- n = 1 + x+ x + +
n n(n - 1) 2 1! 2!
(1) (1 - x)n = 1 - x+ x
1! 2! n(n + 1)(n + 2) 3
x + +¥
n(n - 1)(n - 2) 3 3!
- x ¥
3!
(4) (1 - x)-1 = 1 + x + x2 + x3 + + ¥
-n n n(n + 1) 2 (5) (1 + x)-1 = 1 - x + x2 - x3 + + ¥
(2) (1 + x) = 1 - x+ x
1! 2!
(6) (1 - x)-2 = 1 + 2x + 3x2 + 4x3 + + ¥
n(n + 1)(n + 2) 3
- x ¥ (7) (1 + x)-2 = 1 - 2x + 3x2 - 4x3 + + ¥
3!

EXERCISES
Single Correct Choice Type Questions
1. The coefficient of x4 in the expansion of (x2 - x - 2)5 is (A) 2n + 1 (B) 2n + 1 - 1 (C) 22n (D) 2n - 1
(A) 490 (B) – 490 (C) 390 (D) 30
10. The first integral term other than the first term
2. The number of non-zero terms in the expansion of beginning from the left in the expansion of
( 5 + 1) + ( 5 - 1) is
6 6 ( 3 + 3 2 )9 is
(A) 3 (B) 4 (C) 5 (D) 2 (A) second term (B) third term
(C) fourth term (D) fifth term
3. 2 × 10 C2 + 3 × 2 × 10 C3 + 4 × 3 × 10 C4 + + 10 × 9 × 10 C10 =
11. The largest term in the expansion of (2 + 3x) when
25
(A) 35 · 29 (B) 45 · 28 (C) 45 · 210 (D) 45 · 29
x = 2 is its
n-1
4. C1 - 2 × C2 + 3 × C3 - 4 × C4 + + (-1) × n × Cn = (A) thirteenth term (B) twentieth term
(A) 1 (B) 0 (C) –1 (D) n (C) twenty-sixth term (D) nineteenth term

5. The numerically greatest term in the expansion of 12. If n is even, then the last term in the expansion of
(1 - 3 x)10 when x = 1/ 2 is cosn q in terms of cosines of multiples of q is
6 7
æ 2ö æ 2ö 1 n 1 1
(A) 10
C6 ç ÷ (B) 10
C7 ç ÷ (A) nC( n / 2 ) (B) C( n / 2 ) (C) n nC( n / 2 ) (D) n
è 3ø è 3ø 2n - 1 2 2
æ 3ö
7
æ 3ö
6
13. The last term in the expansion of sin 9 q as sines of
10 10
(C) C6 ç ÷ (D) C6 ç ÷ multiples of q is
è 2ø è 2ø
63 -63 63 -63
(A) sinq (B) sinq (C) (D)
6. If (1 + x + x ) = a0 + a1x + a2x + + a2nx
2 n 2
and n is 2n
128 128 128 128
odd, then the value of a0 - a2 + a4 - a6 + + a2 n is
(A) 1 (B) –1 (C) 0 (D) 22n 14. Given positive integers n > 2, r > 1 and the coeffi-
cients of (3r)th and (r + 2)th terms in the binomial
7. If Cr is the binomial coefficient in the expansion of expansion of (1 + x)2 n are equal, then
(1 + x)10 and a = å r = 0 1/ Cr , then å
10 10
r =0
r / Cr is equal to (A) n = 2r (B) n = 3r
(A) 9a (B) 10a (C) 5a (D) 11a (C) n = 2r + 1 (D) n = 3r + 1

15. In the expansion of [2a - (a / 4)] , the sum of the


2 9
8. Which one of the expansions of the following will
contain x ? 2 middle terms is
(A) ( x-1 / 5 + 2 x3 / 5 )25 (B) ( x - 2 x )
3/ 5 -1 / 5 23
æ 63 ö æ 63 ö
(A) ç ÷ a14 (a + 8) (B) ç ÷ a14 (a - 8)
(C) ( x3 / 5 + 2 x-1 / 5 )22 (D) ( x3 / 5 + 2 x-1 / 5 )24 è 32 ø è 32 ø

æ 63 ö æ 63 ö
9. The coefficient of x in the expansion of (x + C0)
n
(C) ç ÷ a13 (a - 8) (D) ç ÷ a13 (8 - a)
è 32 ø è 32 ø
(x + C1) ( x + C2 ) ( x + Cn ) where Cr = 2 n + 1Cr is
www.jeeneetbooks.in
354 Chapter 7 Binomial Theorem

16. The coefficient of x4 in the expansion of (1 + x + x3 + (A) 8 (B) 9 (C) 7 (D) 22


x4)10. is
40 10 21. If the sum of the coefficients in the expansions of
(A) C4 (B) C4 (C) 210 (D) 310
(1 - 3 x + 10 x2 )n and (1 + x2 )n are, respectively, a and
b, then
17. If (1 + x + x ) = a0 + a1x + a2x + + a2nx , then
2 n 2 2n

a0a1 - a1a2 + a2a3 - a3a4 + is equal to (A) a = 2b (B) a = 3b (C) a = b2 (D) a = b3


(A) 0 (B) 22n (C) 22n – 1 (D) –1 n æ r -1 ö
7 8
22. å n
Cr ç å r Cp 2p ÷ =
è p= 0 ø
18. If the coefficients of x and x in the expansion of r =1

[2 + ( x / 3)] are equal, then n is equal to


n
(A) 43 - 3n + 1 (B) 4n - 3n - 1
(A) 56 (B) 55 (C) 45 (D) 15 (C) 4n - 3n (D) (B) 4n - 3n + 2

19. If the coefficients of rth, (r + 1)th and (r + 2)th terms 23. If the fourth term in the expansion of ( x
[ 1 /( 1+ log10 x )]
+
in (1 + x) are in HP, then
n
12 6
x ) is equal to 200 and x > 1, then x is equal to
(A) n + (n - 2r )2 = 0 (B) n - (n + 2r )2 = 0 (A) 10 (B) 100 (C) 10 2 (D) 104
(C) n - (n - 2r )2 = 0 (D) n + (n - r )2 = 0
6
24. The coefficient of (ab) in the expansion of
20. If the total number of terms in the expansion of [a - (b / a)] is
2 12

( x + y + 2z)n is 45, then n is equal to (A) – 824 (B) 824 (C) 924 (D) –924

Multiple Choice Type Questions


1. In the expansion of [ x + (a / x2 )]n, a ¹ 0, if no term is (C) If n = 4 K + 1, where K is a positive integer, then
independent of x, then n may be a1 - a3 + a5 - a7 + = 1.
(A) 10 (B) 12 (C) 16 (D) 20 (D) If n is a multiple of 4, then a0 - a2 + a4 - a6 + = 1.

2. If a and b are non-zero and only one term in each 6. Which of the following statements is (are) true?
of the expansions of [ x - (a / x)]n and [ x + (b / x2 )]n is (A) There are two consecutive terms in the expansion
independent of x, then n is divisible by of (3 + 2 x)74 whose coefficients are equal.
(A) 2 (B) 3 (C) 4 (D) 6 (B) For a positive integer n, the coefficients of
second, third and fourth terms in the expansion
3. If the third, fourth and fifth terms in the expansion of of (1 + x)2 n are not in AP.
( x + a)n are respectively 84, 280 and 560, then
(C) Larger of 9950 + 10050 and 10150 is 10150.
(A) x = 1 (B) a = 2
(D) The sum of the coefficients in the binomial expan-
(C) n = 7 (D) x = 2, a = 3, n = 8 sion of (5 x - 4 y)21 is 1.

4. Which of the following is (are) true? 7. Which of the following are true?
(A) The coefficient of x-1 in the expansion of [x +
(A) (10C0 )2 - (10C1 )2 + (10C2 )2 - (10C3 )2 + + (10 C10 )2
(1/x2)]8 is 56.
-(10)!
(B) The coefficient of x in the expansion of [x + =
5! 5!
(1/x2)]8 is 0.
(B) (11 C0 )2 - (11 C1 )2 + (11 C2 )2 - (11 C3 )2 + - (11 C11 )2
(C) The coefficient of x9 in the expansion of [2x2 -
(1/x)]20 is 0. (11)!
=
(D) The coefficient of x30 in the expansion of (x3 + 6 ! 5!
3x2 + 3 x + 1)15 is 45C15 . C1 C C C
(C) 2 × C0 + 22× + 23× 2 + 24× 3 + + 211× 10
2 3 4 11
5. It is given that (1 + x + x2)n = a0 + a1x + a2x2 + + a2nx2n.
311 - 1
Which of the following is (are) correct? = (where Cr = 10 Cr )
11
(A) If n is odd, then a0 - a2 + a4 - a6 + = 0.
(B) If n is even, then a1 - a3 + a5 - a7 + = 0. (D) The coefficient of x3 in the expansion of 1 + (1 + x) +
(1 + x)2 + (1 + x)3 + + (1 + x)10 is 11C4 .
www.jeeneetbooks.in
Exercises 355

Matrix-Match Type Questions


In each of the following questions, statements are given in 2. Match the items in Column I with those in Column II
two columns, which have to be matched. The statements in
Column I are labeled as (A), (B), (C) and (D), while those Column I Column II
in Column II are labeled as (p), (q), (r), (s) and (t). Any
given statement in Column I can have correct matching (A) The coefficient of x (p) 378
with one or more statements in Column II. The appropriate in the expansion of
bubbles corresponding to the answers to these questions (1 - 2 x3 + 3 x5 )[1 + (1/ x)]8 is
have to be darkened as illustrated in the following example.
(B) The coefficient of x3 (q) 154
Example: If the correct matches are (A) ® (p), (s); (B) ® in the expansion of
(q), (s), (t); (C) ® (r); (D) ® (r), (t) ; that is if the matches (1 + x + 2 x2 )[2 x2 - (1/ 3 x)]9 is
are (A) ® (p) and (s); (B) ® (q), (s) and (t); (C) ® (r);
and (D) ® (r), (t); then the correct darkening of bubbles (C) The coefficient of x5 in the (r) 31
will look as follows: expansion of (1 + x + x3 )9 is
(D) If (1 + x - 2 x2 )6 (s) -224/24
p q r s t = 1 + a1x + a2 x2 + + a12 x12, then
A a2 + a4 + a6 + a8 + a10 + a12 =
B
C
D

1. Match the items in Column I with those in Column II.

Column I Column II

(A) If the middle term 10


C2
in the expansion of (p)
108
[ x / 3 + ( 3 / 2 x2 )]10 is axK,
then a =
(B) The term independent
of x in the expansion of (q) 10C4 ´ 5184
[ x / 3 + ( 3 / 2 x2 )]10 is
(C) The term independent 10
of x in the expansion of C5
(r)
[2 x2 - (3 / x3 )]10 is 32
(D) The coefficient of the middle (s) -65 ´ 10C5
term in the expansion of
[2 x2 - (3 / x3 )]10 is b, then b is

Comprehension-Type Questions
1. Let C0 , C1, C2 , … , Cn be binomial coefficients in the C0 C1 C2 Cn
(ii) - + - + (-1)n =
expansion of (1 + x) . n
4 5 6 n+4
Answer the following questions: 6
(A)
(n + 1)(n + 2)(n + 3)(n + 4)
(i) (C0 + C1 + C2 + + Cn )2 =
(n + 2)(n + 3)(n + 4)
(A) 22n + 1 (B)
6
(B) 1 + 2 nC1 + 2 nC2 + + 2 nC2 n
(C) 0
(C) 22n - 1 (n + 1)(n + 2)(n + 3)
(D)
(D) 2n
C1 + C2 + C3 + + C2 n
2n 2n 2n
6
www.jeeneetbooks.in
356 Chapter 7 Binomial Theorem

(iii) -C0 + C1 + 3 × C2 + 5 × C3 + + (2 n - 1) × Cn = C1 C C
(ii) 2 × C0 + 22× + 23× 2 + + 2n + 1× n =
(A) n × 2 n-1
(B) (n - 1)2n-1 2 3 n+1
(C) n ×2n
(D) (n - 1)2n 3n - 1 3n + 1
(A) (B)
n+1 n+1
2. (Note: This question may be attempted after studying
3n + 1 + 1 3n + 1 - 1
integration): (C) (D)
b
n+1 n+1
Let (1 + x)n = C0 + C1 x + C2 x2 + + Cn xn and ò xr dx =
a C1 C2 C3 C
[1/(r + 1)](br + 1 - ar + 1 ) . Using this information, answer (iii) C0 - + - + + (-1)n n =
2 3 4 n+1
the following questions:
2n + 1 - 1
C C C C (A) 0 (B)
(i) 0 + 1 + 2 + + n = n
1 2 3 n+1
1 1
2n - 1 2n + 1 - 1 2n + 1 + 1 2n + 1 (C) (D)
(A) (B) (C) (D) n+1 (n + 1)(n + 2)
n+1 n+1 n+1 n+1

Assertion−Reasoning Type Questions


Statement I and Statement II are given in each of the 3. Statement I: If
questions in this section.Your answers should be as per the (1 + x)n = C0 + C1 x + C2 x2 + + Cn xn
following pattern:
(A) If both Statements I and II are correct and II is a = C0 xn + C1 xn - 1 + C2 xn - 2 + + Cn
correct reason for I then
(B) If both Statements I and II are correct and II is n
not a correct reason for I
å r(n - r)C
r =0
2
r = n2 ( 2 n - 2 ) Cn
(C) If Statement I is correct and Statement II is false.
(D) If Statement I is false and Statement II is correct. Statement II: nCK = nCn - K and the derivative of (x +
n-1 n
Cr n a)n = n(x + a)n - 1.
1. Statement I: å
r =0
n
=
Cr + Cr + 1 2
n

4. Statement I: If n is a positive integer, then in the expa-


Statement II: nCK + nCK - 1 = ( n + 1)CK nsion of (1 + x)n, the coefficients of (r + 1)th, (r + 2)th,
and (r + 3)th terms are in G.P.
2. Statement I: If sin7 q is expressed as a series of sines
of multiples of q, then the coefficient of sin 5q is 7 / 64. Statement II: Three non-zero numbers a, b and c are
in GP if and only if ac = b2 .
Statement II: If x = cos q + i sin q , then
5. Statement I: No three consecutive coefficients in the
1
xK + = 2 cos Kq expansion of (1 + x)n are in HP.
xK
Statement II: Non-zero numbers a, b and c are in HP
1 if 1/ a, 1/ b and 1/ c are in AP.
and x - K = 2 i sin K q
K

x
where K is a positive integer.

Integer Answer Type Questions


The answer to each of the questions in this section is a the question number Y is 246, then the bubbles under Y
non-negative integer. The appropriate bubbles below the labeled as 2, 4, 6 are to be darkened.
respective question numbers have to be darkened. For
example, as shown in the figure, if the correct answer to
www.jeeneetbooks.in
Answers 357

X Y Z W 3. If 32 cos6 q = a1 cos 6q + a2 cos 4q + a3 cos 2q + a4, then


0 0 0 0 a4 is equal to .
1 1 1 1
4. If 256 sin7q · cos2 q = a1 sin 9q + a2 sin 7q + a3 sin 5q +
2 2 2
a4 sinq, then a4 is equal to .
3 3 3 3
4 4 4 5. The digit at the unit’s place in the number 172010 +
5 5 5 5 112010 - 72010 is .
6 6 6
7 7 7 7
8 8 8 8
9 9 9 9

1. If the second term in the expansion of (13 x + x x ) is


n

14 × x , then C3 / C2 =
5/ 2 n n
.

2. If P and Q are, respectively, the sum of even and odd


terms in the expansion of (x + a)10, then (x + a)20 -
(x - a)20 = k PQ where k is .

ANSWERS
Single Correct Choice Type Questions
1. (D) 13. (A)
2. (B) 14. (A)
3. (D) 15. (D)
4. (B) 16. (D)
5. (D) 17. (A)
6. (C) 18. (B)
7. (C) 19. (A)
8. (C) 20. (A)
9. (C) 21. (D)
10. (C) 22. (C)
11. (B) 23. (A)
12. (C) 24. (C)

Multiple Correct Choice Type Questions


1. (A), (C), (D) 5. (A), (B), (C), (D)
2. (A), (B), (D) 6. (A), (B), (C), (D)
3. (A), (B), (C) 7. (A), (C), (D)
4. (A), (B), (C), (D)

Matrix-Match Type Questions


1. (A) ® (r), (B) ® (p), (C) ® (q), (D) ® (s) 2. (A) ® (q), (B) ® (s), (C) ® (p), (D) ® (r)

Comprehension-Type Questions
1. (i) (B), (ii) (A), (iii) (D) 2. (i) (B), (ii) (D), (iii) (C)
www.jeeneetbooks.in
358 Chapter 7 Binomial Theorem

Assertion–Reasoning Type Questions


1. (A) 4. (D)
2. (A) 5. (A)
3. (A)

Integer Answer Type Questions


1. 4 4. 14
2. 4 5. 1
3. 10
www.jeeneetbooks.in

Matrices, Determinants and


System of Equations 8
Contents
8.1 Matrices
8.2 Determinants
8.3 System of Equations

Worked-Out Problems
Summary
Exercises
Matrices, Determinants and

Answers

m-by-n matrix
ai, j n columns j changes Matrices: A matrix (plural
m
matrices) is a rectangular array
System of Equations

rows of numbers. Matrices are a key


a11 a12 a13 tool in linear algebra. One
i
c use of matrices is to represent
h a21 a22 a23 linear transformations.
a
n
g a31 a32 a33 Determinants: The determin-
e ant is a special number associ-
s
ated with any square matrix.
The fundamental geometric
meaning of a determinant is a
scale factor for measure when
the matrix is regarded as a
linear transformation.
www.jeeneetbooks.in
360 Chapter 8 Matrices, Determinants and System of Equations

The theory of matrices plays an important role in almost all branches of Mathematics and other subjects. A very important
application of matrices is to find solutions of system of linear equations. Let us consider a simple situation where three stu-
dents Ram, Rahim and Robert have appeared for class tests in four subjects English, Mathematics, Physics and Chemistry
and the marks obtained by each of them in these subjects are given in a tabular form given below.

English Mathematics Physics Chemistry


Ram 80 86 78 75
Rahim 75 84 72 68
Robert 78 68 74 78

This also can be represented by an array of numbers without drawing lines and not writing the names of the
subjects on the top row and the names of the students on the left most column, as given below.

é 80 86 78 75 ù
ê 75 84 72 68 ú
ê ú
êë 78 68 74 78 úû

The brackets given on the left end and right end do not convey any meaning but just improve the presentation style.
The first horizontal line of numbers shows the marks obtained by Ram in English, Mathematics, Physics and Chemistry,
respectively. Similarly the second and third horizontal lines show the same for Rahim and Robert, respectively. The
first vertical line of numbers shows the marks obtained in English by Ram, Rahim and Robert. The second, third and
fourth vertical lines show the same for Mathematics, Physics and Chemistry, respectively.
The horizontal lines are called rows and the vertical lines are called columns. The rows are numbered from top to
bottom. The top row is called the first row and the subsequent rows are called second row, third row, etc. The columns
are numbered from left to right. The left most column is called the first column and the subsequent columns are called
second column, third column, etc.
In this chapter, we make a detailed study of matrices whose entries are real or complex numbers.

8.1 | Matrices

In this section we shall give a formal definition of a matrix and discuss various types of matrices and their properties.

D E F I N I T I O N 8 . 1 Matrix, Rows, Columns, Order An ordered rectangular array of real or complex numbers
or functions or of any kind of expressions is called a matrix. The horizontal lines in the
array are called rows and the vertical lines are called columns. If there are m rows and n
columns in a matrix A, then A is called an m ´ n matrix or an “m by n” matrix or a matrix
of order m ´ n.

DE F IN IT ION 8 . 2 Elements or Entries The numbers or functions or expressions in a matrix A are called
“elements” or “entries” of A. If A is m ´ n matrix, then there are m rows of elements and
n columns of elements. In each row of an m ´ n matrix there are exactly n elements and in each
column there are exactly m elements.

Examples

(1) Consider the matrix Then A is a 3 ´ 4 matrix, since there are 3 rows and
4 columns in A. Here
é 2 3 4 1ù
2 3 4 1 is the first row
A = êê - 1 2 1 3úú
–1 2 1 3 is the second row
êë 5 - 3 2 - 4 úû
5 –3 2 –4 is the third row
www.jeeneetbooks.in
8.1 Matrices 361

é 2ù é 1 3 2ù
ê - 1ú is the first column ê 1 ú
ê ú ê- 3 0ú
êë 5úû (2) êê ú is a 4 ´ 3 matrix, since there are 4 rows
2
2 - 3 - 1ú and 3 columns.
ê ú
é 3ù ê 1 2 ú
ê 2 ú is the second column êë 3 4 úû
ê ú 3
êë - 3úû
(3) é 2 - 1 3ù is a 2 ´ 3 matrix, since there are 2 rows
ê - 4 0 2 ú and 3 columns.
é4ù ë û
ê 1 ú is the third column
ê ú é 2 - 1ù
êë 2 úû (4) ê is a 2 ´ 2 matrix.
ë0 3úû

é 1ù (5) [2] is a 1 ´ 1 matrix.


ê 3ú is the fourth column
ê ú
êë - 4 úû

In general, an m ´ n matrix is of the form

é a11 a12 a1 n ù
êa a22 a2 n ú
A=ê ú
21

ê  ú
ê ú
êëam1 am 2 amn úû

where each aij is a number or a function or an expression. The entries in the ith row are
ai 1 , ai 2 , … , ain
and the entries in the jth column are
a1 j , a2 j , … , amj

for 1 £ i £ m and 1 £ j £ n. The variable aij stands for the entry which is common for the ith row and jth column. For
simplicity, we write
A = (aij )m´ n or A = (aij ) or A = [aij ] m´ n

to denote an m ´ n matrix whose entry in the ith row and jth column is aij. For convenience, aij is called the ijth entry
of the matrix A = (aij ). Further m ´ n is called the order of A.

DEFIN IT ION 8 . 3 Equality of Matrices Two matrices are said to be equal if they are of the same order and for
any i and j, the ijth entries of the two matrices are same. In other words, if A = (aij ) is an m ´ n
matrix and B = (bij ) is a p ´ q matrix, then we say that A and B are equal and write A = B if
m = p, n = q and aij = bij for all 1 £ i £ m = p and 1 £ j £ n = q.

Note: A 2 ´ 3 matrix can never be equal to a 3 ´ 2 matrix, since their orders are different. For example

é 2 1ù
ê -3 é 1ù
4ú ê2 -3
ê ú¹ 2ú
ê 1 ú ê ú
êë 2 0ú ë1 4 0û
û
DEF IN IT ION 8 . 4 Square Matrix An m ´ n matrix is said to be a square matrix if m = n, that is, the number of
rows is equal to the number of columns.
www.jeeneetbooks.in
362 Chapter 8 Matrices, Determinants and System of Equations

Example

The matrices are square matrices. If A is an n ´ n matrix, then we say


that A is a square matrix of order n.
é 1 2 -2ù
é 1 - 1ù ê ú
[2], ê ú and ê 3 4 - 1ú
ë0 2 û êë 2 0 1úû

DEF IN IT ION 8 . 5 Let A = (aij ) be an m ´ n matrix. A is called a vertical matrix if m > n and a horizontal matrix
if m < n.

Examples

é2 3 1ù é2 1ù
(1) ê ú is a horizontal matrix, since the columns
ë 1 0 - 1û (2) ê 3 0 úú is a vertical matrix, since the rows are more
ê
are more in number than rows. êë 1 - 1úû in number than the columns.

Recall that, if the rows and columns are equal in number, then the matrix is called the square matrix. Note that any
matrix must be either a square matrix or a vertical matrix or a horizontal matrix.

DEF IN IT ION 8 . 6 Row Matrix and Column Matrix A matrix is called a row matrix if it has only one row and is
called a column matrix if it has only one column.

Example

é 2ù
The matrix [ 2 3 1] is a row matrix and the matrix êê 1úú is a column matrix.
êë - 2 úû

DEFINITION 8.7 Zero Matrix A matrix is called a zero matrix or null matrix if all its entries are zero. A zero
matrix is usually denoted by O, without mentioning its order and is to be understood as per
the context.

Example

é0 0 ù é0 0 0 ù
The matrices [0], ê
0 0 ú and ê0 0 0 ú are all zero matrices.
ë û ë û

DEF IN IT ION 8 . 8 Diagonal of a Matrix Let A = (aij ) be a square matrix of order n, that is A is a n ´ n matrix.
Then the elements

a11 , a22 , a33 , … , anm

are called the diagonal elements and the line along which these elements lie is called the
principal diagonal or main diagonal or simply the diagonal of the matrix.
www.jeeneetbooks.in
8.1 Matrices 363

é a11 a12 a13 a1n ù


êa a22 a23 a2n ú
ê 21 ú
A = êa31 a32 a33 a3n ú
ê ú
ê ú
ê an1 an2 an3 ann úû
ë

Example

é2 3 0ù
é 1 2ù ê 1 1 -2ú .
1, 0 are the diagonal elements in ê ú and 2, 1, –1 are the diagonal elements in ê ú
ë-1 0 û êë 4 0 - 1úû

DEF IN IT ION 8 . 9 Diagonal Matrix A square matrix A = (aij ) is said to be a diagonal matrix if aij = 0 for all
i ¹ j, that is, except those in the diagonal of A, all the entries in A are zeros. Note that the
diagonal elements need not be zeros.

Examples

é0 0 ù é3 0 0 0ù
(1) ê ú is a diagonal matrix. ê0
ë0 2 û é2 0 0ù 1 0 0ú
ê ú
(2) êê 0 1 0 úú and ê 1 ú are diagonal matrices.
ê0 0 0ú
êë 0 0 - 1úû ê 2 ú
êë0 0 0 - 2 úû

Note that the zero matrix of order n ´ n is also a diagonal matrix for any n.

DEF IN IT ION 8 . 10 Scalar Matrix A diagonal matrix A = (aij ) is called a scalar matrix if aii = ajj for all i and j,
that is, a matrix A = (aij ) is a scalar matrix if all the diagonal elements are equal and the other
elements are zeros.

If a is any real or complex number and n is any positive integer, then define

ïìa if i = j
aij = í
ïî0 if i ¹ j

for any 1 £ i, j £ n. Then (aij ) is a scalar matrix

éa 0 0 ù
ê0 a 0 ú
ê ú
ê ú
ê ú
ë0 0 a û

and any scalar matrix is of this form.

DEF IN IT ION 8 . 11 Identity Matrix A scalar matrix is called the identity matrix or a unit matrix if each of the
diagonal element is the number 1. That is, a square matrix A = (aij ) of order n is called the
identity matrix of order n if
www.jeeneetbooks.in
364 Chapter 8 Matrices, Determinants and System of Equations

ïì1 if i = j
aij = í
ïî0 if i ¹ j

for all 1 £ i, j £ n. The identity matrix of order n is denoted by In. The identity or unit matrix
In of order n will be simply called the identity and denoted by I when there is no ambiguity
about n.

Example

é1 0 0 0ù
é1 0 0ù ê
é1 0ù 0 1 0 0ú
I1 = [1] , I2 = ê ú , I3 = êê0 1 0 úú , I4 = ê ú
ë0 1û ê 0 0 1 0ú
êë0 0 1úû ê ú
ë 0 0 0 1û

DEF IN IT ION 8 . 12 Triangular Matrices A square matrix A = (aij ) is called an upper triangular matrix if
aij = 0 for all i > j A = (aij ) is called a lower triangular matrix if
aij = 0 for all i < j

Example

Upper Triangular Matrices Lower Triangular Matrices

é1 2 3 -2ù é 4 0 0 0ù
é2 3 - 2ù é 3 0 0ù ê
é 1 2ù ê
ê0 - 2 0
ú and ê 4ú é2 0ù ê ú 2 0 0 0ú
- ú ê ú
,
ê 0 - 1ú ê 0 1 1 ú ê0 3ú ê 1 - 1ú , ê 2 1 0 ú and ê 3 - 2 5 0ú
ë û ê 0 5 ë û ê
ë0 0 4 úû ê ú ë - 1 0 2 úû ê ú
ë0 0 0 - 1û ë-1 0 3 -9û

Note that a square matrix is both upper and lower triangular matrix if and only it is a diagonal matrix.

DEF IN IT ION 8 . 13 Addition of Matrices Let A = (aij ) and B = (bij ) be matrices of order m ´ n. Then, we define

A + B = (aij + bij ) for all 1 £ i £ m and 1 £ j £ n


That is, the ijth entry in A + B is the sum of the ijth entries in A and B. A + B is called the sum
of A and B and the operation + is called the addition of matrices.

Note that the addition is defined among matrices of the same order. For any m ´ n matrix A = (aij ), we defined an
m ´ n matrix -A by
- A = (- aij )
and for any m ´ n matrices A and B, we write, as usual, A - B for
A + (- B) = (aij - bij )
where A = (aij ) and B = (bij ).
Recall that two matrices A = (aij ) and B = (bij ) are said to equal if A and B are of the same order, say m ´ n and
aij = bij for all 1 £ i £ m and 1 £ j £ n.
www.jeeneetbooks.in
8.1 Matrices 365

Example 8.1

If Solution: We have
é2 0 1 3ù é 1 -1 2 4ù é 2+1 0 + (- 1) 1+ 2 3+4 ù
ê ú ê 1 2 úú , ê
A = ê 1 - 1 2 1ú and B = ê - 2 3 A + B = ê1 + (- 2) - 1 + 3 2+1 1 + 2 úú
êë 3 - 2 4 2 úû êë 4 0 - 4 - 3úû
ëê 3 + 4 (- 2) + 0 4 + (- 4) 2 + (- 3)úû
then find out A + B.
é 3 -1 3 7 ù
= êê - 1 2 3 3úú
êë 7 - 2 0 - 1úû

We shall use this technique in proving the following, in which all matrices are considered to be over real or complex
numbers.

T H E O R E M 8 .1 Let A = (aij ), B = (bij ) and C = (cij ) be matrices of order m ´ n. Then the following are true.
1. Associative law for addition: A + ( B + C ) = ( A + B) + C.
2. Commutative law for addition: A + B = B + A.
3. A + O = A, where O is the m ´ n zero matrix and is called the additive identity.
4. A + (- A) = O. Here -A is called the additive inverse of A.
5. Cancellation laws for addition: A + B = A + C Þ B = C and B + A = C + A Þ B = C.
6. There exists unique matrix D such that A + D = B.
PROOF 1. For any 1 £ i £ m and 1 £ j £ n,
ijth entry in A + ( B + C ) = aij + (bij + cij )
= (aij + bij ) + cij (since + is associative for numbers)
= ijth entry in ( A + B) + C
Therefore A + ( B + C ) = ( A + B) + C.
2. For any 1 £ i £ m and 1 £ j £ n,
ijth entry in A + B = aij + bij
= bij + aij (since + is commutative for numbers)
= ijth entry in B + A
Therefore A + B = B + A.
3. A + O = (aij ) + (0)
= (aij + 0)
= (aij ) = A
4. A + (- A) = (aij ) + (- aij )
= (aij + (- aij ))

= ( 0) = O
www.jeeneetbooks.in
366 Chapter 8 Matrices, Determinants and System of Equations

5. Suppose that A + B = A + C. Then, for any 1 £ i £ m and 1 £ j £ n,


aij + bij = ijth entry in A + B
= ijth entry in A + C
= aij + cij
Hence bij = cij . Therefore B = (bij ) = (cij ) = C. Also
B+ A=C + AÞ A+ B= A+C
Þ B = C [by (2)]

6. Put D = B - A. Then
A + D = A + ( B - A) = ( B - A) + A = B + (- A + A) = B + O = B ■

DEF IN IT ION 8 . 14 A real or complex number is called a scalar. For any matrix A = (aij ) of numbers and for any
scalar k, we define the matrix kA as the one whose ijth entry is obtained by multiplying the
ijth entry of A by k, that is

kA = (kaij )

Example

If then

é 1 2 - 3ù é 4 8 - 12 ù
ê0 9 4ú ê 0 36 16 ú
A=ê ú, 4A = ê ú
ê5 -6 7ú ê 20 - 24 28 ú
ê ú ê ú
ë8 - 5 2û ë 32 - 20 8û

Hereon, all matrices are assumed to be with entries as real or complex numbers and we would not specify this any further.

T H E O R E M 8 .2 Let A = (aij ) and B = (bij ) be m ´ n matrices and s and t be scalars. Then the following properties
are satisfied:
1. s( A + B) = sA + sB
2. ( s + t ) A = sA + tA
3. s(tA) = ( st ) A = t( sA)
4. (- s) A = -( sA) = s(- A)
5. 0A = O (0 on the left side is the scalar zero and O on the right is the zero matrix)
6. sO = O
PROOF Let 1 £ i £ m and 1 £ j £ n.
1. ijth entry in s( A + B) = s(aij + bij )
= saij + sbij
= ijth entry in sA + sB
Therefore, s( A + B) = sA + sB.
2. ijth entry in ( s + t ) A = ( s + t )aij
= saij + taij
= ijth entry in sA + tA
Therefore, ( s + t ) A = sA + tA.
www.jeeneetbooks.in
8.1 Matrices 367

3. s(taij ) = ( st )aij = (ts)aij = t( saij ). Therefore, s(tA) = ( st ) A = t( sA).


4. (- s)aij = -( saij ) = s(- aij ). Therefore, (- s) A = -( sA) = s(- A).
5. and 6. Since 0 × aij = 0 = s × 0, therefore OA = O = sO. ■

Example 8.2

Let A and B be 2 ´ 2 matrices. Find A and B such that Similarly, by using

é 9 4ù é17 - 1ù
3A + 2B = ê ú 2 A + 5B = ê ú
ë- 2 -6û ë 6 - 15û
we get that
é17 - 1ù
and 2 A + 5B = ê ú 17 = 2a11 + 5b11 (8.5)
ë 6 - 15û
-1 = 2a12 + 5b12 (8.6)
Solution: Let
6 = 2a21 + 5b21 (8.7)
é a11 a12 ù é b11 b12 ù
A=ê ú and B = ê -15 = 2a22 + 5b22
b22 úû
(8.8)
ëa21 a22 û ëb21
By solving Eqs. (8.1) and (8.5), we can find a11 and b11
Then, from the hypothesis, as a11 = 1 and b11 = 3. Similarly, by solving Eqs. (8.2)
and (8.6), Eqs. (8.3) and (8.7), and Eqs. (8.4) and (8.8),
é 9 4ù é a11 a12 ù é b11 b12 ù we get
ê - 2 - 6 ú = 3 A + 2 B = 3 êa + 2ê
ë û ë 21
ú
a22 û ëb21 b22 úû
a12 = 2 and b12 = - 1
é 3a11 + 2 b11 3a12 + 2 b12 ù a21 = - 2 and b21 = 2

ë 3a21 + 2 b21 3a22 + 2 b22 úû and a22 = 0 and b22 = - 3

and therefore, by equating the corresponding ijth entries Therefore


on both sides, we get
é 1 2ù é 3 - 1ù
A=ê ú and B = ê ú
9 = 3a11 + 2b11 (8.1) ë- 2 0û ë 2 - 3û
4 = 3a12 + 2b12 (8.2)
-2 = 3a21 + 2b21 (8.3)
-6 = 3a22 + 2b22 (8.4)

Example 8.3

Evaluate the following: é cos2 q cos q sin q ù


=ê ú
é cos q sin q ù é sin q - cos q ù ë - cos q sin q cos2 q û
cos q ê ú + sin q ê
ë - sin q cos q û ë cos q sin q úû é sin2 q - sin q cos q ù
+ê ú
Solution: We have ësin q cos q sin2 q û

é cos q sin q ù é sin q - cos q ù é cos2 q + sin2 q 0 ù


cos q ê + sin q ê =ê 2 ú
ë - sin q cos q úû ëcos q sin q úû ë 0 cos q + sin q û
2

é1 0ù
=ê ú = I2
ë0 1û
www.jeeneetbooks.in
368 Chapter 8 Matrices, Determinants and System of Equations

Example 8.4

Compute the matrix X, if it is given that 2 X + 3 A = 3B, 3 æ é 3 5ù é 1 - 1ùö


= -
2 çè êë 4 6 úû êë0 2 úû÷ø
where

é 1 -1ù é 3 5ù
A=ê ú and B = ê ú 3 é 3 - 1 5 - (- 1)ù
=
ë0 2 û ë4 6û 2 êë 4 - 0 6 - 2 úû
Solution: Suppose that 2 X + 3 A = 3B. Then 2X = 3B- 3 é2 6 ù
3 A = 3( B - A). Therefore =
2 êë 4 4 úû
3
X = [ B - A] é3 9ù
2 =ê ú
ë6 6 û

In the following, we define the product AB of two matrices A and B only when the number of columns in A is equal
to the number of rows in B.

DEF IN IT ION 8 . 15 Multiplication of Matrices Let A be an m ´ n matrix and B an n ´ p matrix. If A = (aij) and
B = (bij), then the product AB is defined as the m ´ p matrix (cij), where
n
cij = å air brj = ai 1b1 j + ai 2 b2 j + + ain bnj
r =1

for all 1 £ i £ m and 1 £ j £ p.

Example 8.5

Let A be a 4 ´ 3 matrix and B be a 3 ´ 2 matrix given by c22 = a21b12 + a22 b22 + a23b32

é 2 3 - 1ù = 1 ´ (- 3) + 0 ´ 4 + 4 ´ 1
ê 1 é 2 - 3ù
0 4ú = -3 + 0 + 4 = 1
A=ê ú and B = ê - 1 4 ú
ê 0 2 -2ú ê ú c31 = a31b11 + a32 b21 + a33b31
ê ú êë 3 1úû
ë- 3 1 2û = 0 ´ 2 + 2 ´ (- 1) + (- 2)(3)
= 0 - 2 - 6 = -8
Find the product AB.
c32 = a31 b12 + a32 b22 + a33 b32
Solution: We get the product AB as a 4 ´ 2 matrix
given by = 0 ´ (- 3) + 2 ´ 4 + (- 2) ´ 1

AB = (cij ) =0+8-2=6
c41 = a41b11 + a42 b21 + a43b31
år=1air brj for all 1 £ i £ 4 and 1 £ j £ 2 and
3
where cij = = (- 3)2 + 1(- 1) + 2 ´ 3
A = (aij) and B = (bij). Now = - 6 - 1 + 6 = -1
c11 = a11b11 + a12 b21 + a13b31 c42 = a41b12 + a42 b22 + a43b32
= 2 ´ 2 + 3 ´ (- 1) + (- 1) ´ 3 = (- 3) (- 3) + 1 ´ 4 + 2 ´ 1
= 4 - 3 - 3 = -2 = 9 + 4 + 2 = 15
c12 = a11b12 + a12 b22 + a13b32
Therefore
= 2 ´ (- 3) + 3 ´ 4 + (- 1)1
= - 6 + 12 - 1 = 5 é - 2 5ù
ê 14 1ú
c21 = a21b11 + a22 b21 + a23b31 AB = (cij ) = ê ú
ê-8 6ú
= 1 ´ 2 + 0 ´ (- 1) + 4 + 3 ê ú
= 2 + 0 + 12 = 14 ë - 1 15û
www.jeeneetbooks.in
8.1 Matrices 369

Note: The ijth entry in the product AB is simply obtained by multiplying the ith row of A and jth column of B. Note
that the ith row of A and jth column of B are both n-tuples

æ b1 j ö
çb ÷
(ai 1 , ai 2 , ai 3 , …, ain ) and ç ÷
2j

ç  ÷
çb ÷
è nj ø

respectively and the ijth entry in AB is the sum of the products ai 1 b1 j , ai 2 b2 j , … , ain bnj .

T H E O R E M 8 .3 Let A = (aij ) be an m ´ n matrix, B = (bij ) an n ´ p matrix and C = (cij ) a p ´ q matrix. Then

A( BC ) = ( AB)C
PROOF Note that BC is an n ´ q matrix and AB is an m ´ p matrix. Let BC = (dij ), for 1 £ i £ n, 1 £ j £ q,
and AB = (uij ), for 1 £ i £ m, 1 £ j £ p. Then

p
dij = å bir crj (8.9)
r =1

n
and uij = å ais csj (8.10)
s =1

Both A( BC ) and ( AB)C are m ´ q matrices. For any 1 £ i £ m and 1 £ j £ q,


n
ijth entry of A( BC ) = å ait dtj
t =1

n
æ p ö
= å ait ç å btr crj ÷ [by Eq. (8.9)]
t =1 è r =1 ø
n p
= å å (ait btr crj )
t =1 r =1

p
æ n ö
= å ç å ait btr ÷ crj
r =1 è t =1 ø
p
= å uir crj
r =1

= ijth entry in ( AB)C [by Eq. (8.10)]

Thus ( AB)C = ( AB)C. ■

QUICK LOOK 1

We have proved earlier that A + B = B + A for any is not commutative. In fact, if A × B is defined, then B × A
matrices A and B of the same order; that is the addition may not be defined.
is commutative. However, the multiplication of matrices
www.jeeneetbooks.in
370 Chapter 8 Matrices, Determinants and System of Equations

Consider the following examples.

Examples

(1) Let A be a 3 ´ 2 matrix and B a 2 ´ 4 matrix. Then (2) Even if A × B and B × A are defined, they may not be
A × B is defined, since the number of columns in A is of same order. For example, let A be a 2 ´ 3 matrix
2 which is same as the number of rows in B. However and B be a 3 ´ 2 matrix. Then A × B and B × A are both
B × A is not defined, since the number of columns in defined. However, A × B is a 2 ´ 2 matrix and B × A is
B(= 4) is not equal to the number of rows in A(= 3). 3 ´ 3 matrix.

Even if A × B and B × A are defined and of same order, A × B and B × A may not be equal matrices.

Example 8.6

Let Solution: We have

é1 0ù é 2 1ù é 1× 2 + 0(- 1) 1× 1 + 0 × 0 ù é 2 1 ù
A=ê ú and B = ê - 1 0 ú A× B = ê ú=ê ú
ë 2 - 1û ë û ë 2 × 2 + (- 1)(- 1) 2 × 1 + (- 1)0 û ë 5 2 û
Show that A × B and B × A may not be equal matrices. é 2 × 1 + 1× 2 2 × 0 + 1(- 1) ù é 4 - 1ù
and B × A = ê =
ë( - 1)1 + 0 × 2 ( - 1)0 + 0(- 1)úû êë - 1 0 ûú

Therefore A × B ¹ A × B.

T H E O R E M 8 .4 Let A and B be any matrices. Then AB and BA are both defined and are of same order if and only
if both A and B are square matrices of same order.

PROOF Let A be an m ´ n matrix and B be a p ´ q matrix. Suppose that A × B and B × A are defined and of
same order. Then n = p and q = m. Also, A × B is of order m ´ q and B × A is of order p ´ n. Since
A × B and B × A are of same order, we have m = p and q = n. Thus
m=q=n and p=n=q
Therefore, A and B are square matrices of same order m ´ m. Converse is clear. ■

Examples

(1) If A is a 3 ´ 4 matrix and B is 4 ´ 3 matrix, then A × B (2) If A and B are square matrices each of order 3 ´ 3,
is defined and is a matrix of order 3 ´ 3. Also, B × A is then AB and BA defined and each of them is of
defined and is a matrix of order 4 ´ 4. order 3 ´ 3.

T H E O R E M 8 .5 The multiplication of matrices is distributive over addition in the following sense.


1. If A is an m ´ n matrix and B and C are n ´ p matrices, then
A( B + C ) = A × B + A × C
2. If A and B are m ´ n matrices and C is an n ´ p matrix, then
( A + B)C = AC + BC
PROOF Let A = (aij ), B = (bij ) and C = (cij ).
1. Suppose that A is of order m ´ n and B and C are of order n ´ p. Then A(B + C) and A × B +
A × C are both of order m ´ p. For any 1 £ i £ m and 1 £ j £ p,
n
ijth entry in A( B + C ) = å air (brj + crj )
r =1
www.jeeneetbooks.in
8.1 Matrices 371

n n
= å air brj + å air crj
r =1 r =1

= ijth entry in A × B + A × C

Therefore, A( B + C ) = AB + AC. Note that we have used the fact that the multiplication of
numbers is distributive over addition of numbers.
2. This can be proved on similar lines. ■

In Definition 8.15, we have defined the multiplication of a matrix by a scalar. A scalar (i.e., a real or complex number)
can be identified with a scalar matrix (see Definition 8.10) and the scalar multiplication of a matrix A is actually a
multiplication of A with a scalar matrix, as we see below.

T H E O R E M 8 .6 Let A be an m ´ n matrix and k a scalar. Then BA = kA = AC, where B is the m ´ m scalar matrix
and C is the n ´ n scalar matrix with k as diagonal entries.
PROOF Let B be the m ´ m diagonal matrix

ék 0 0 0 ù
ê0 k 0 0 ú
ê ú
ê ú
ê ú
ë0 0 0 kû

and C be the n ´ n diagonal matrix

ék 0 0 0 ù
ê0 k 0 0 ú
ê ú
ê ú
ê ú
ë0 0 0 kû

Then BA, kA and AC are all m ´ n matrices. For any 1 £ i £ m and 1 £ j £ n,


m
ijth entry in BA = å bir arj = kaij
r =1

where B = (bij ) and bij = k if i ¹ j and bij = 0 if i ¹ j. Therefore BA = kA. Similarly AC = kA. ■

C O R O L L A R Y 8 .1 If A is any n ´ n square matrix and k is an n ´ n scalar matrix, then A × k = k × A.

The converse of this is proved in the following, that is, we prove next that the scalar matrices are the only matrices
which commute with all similar matrices (i.e., matrices of same order).

T H E O R E M 8 .7 Let A be an n ´ n square matrix such that AB = BA for all n ´ n matrices B. Then A is a scalar
matrix.
PROOF Let A = (aij). We shall prove that aii = ajj for all 1 £ i, j £ n and aij = 0 for all 1 £ i ¹ j £ n. Let 1 £ i,
j £ n be fixed and define B = (bst) by

ì1 if s = i and t = j
bst = í
î0 otherwise

Then, since AB = BA, by taking ijth entries both in AB and BA, we get that
n n

åa b = åb a
r =1
ir rj
r =1
ir rj
www.jeeneetbooks.in
372 Chapter 8 Matrices, Determinants and System of Equations

By substituting for brj and bir, we get


aii = ajj

Therefore, all the diagonal elements in A are same. Also, let 1 £ i ¹ j £ n and define C = (cst ) as

ì1 if s = j = t
cst = í
î0 if s ¹ j or t ¹ j
Again, from AC = CA, we get that
n n

åa c = åc a
r =1
ir rj
r =1
ir rj

Substituting for both sides, we have aij = 0 (cir = 0 for all r, since i ¹ j ). Thus, aij = 0 for all i ¹ j and
hence A is a scalar matrix of order n ´ n. ■

T H E O R E M 8 .8 Let A = (aij) be an m ´ n matrix and Im and In be unit matrices of order m ´ m and n ´ n, respec-
tively. Then
Im A = A = A In

PROOF Note that Im is the square matrix of order m ´ m in which each of the diagonal entries is 1 and all
the non-diagonal entries are 0; that is,
Im = (eij )

where

ïì1 if i = j
eij = í
îï0 if i ¹ j

Now, for any 1 £ i £ m and 1 £ j £ n, the ijth entry of Im A is


m

åe a
r =1
ir rj = aij (since eir = 0 for all r ¹ i )

Therefore I m × A = A. Similarly A × In = A. ■

C O R O L L A R Y 8 .2 If A is square matrix of order n ´ n, then In × A = A = A × In.

DEF IN IT ION 8 . 16 Recall from Definition 8.11 that the matrix In is called the identity matrix or unit matrix order n.
In view of Corollary 8.2, In is also called the multiplicative identity of order n. When there is
no ambiguity about n, In is simply denoted by I and one has to take the order of I depending
on the context where it is used.

When we multiply a matrix A with I, from right or left, A is duplicated. If we multiply A with the zero matrix, we get
the zero matrix as in the case of number systems.

T H E O R E M 8 .9 Let A be an m ´ n matrix and Om and On be zero matrices of order m ´ m and n ´ n respectively.


Then
Om A = Om ´ n = A × On

where Om ´ n is the zero matrix of order m ´ n.


www.jeeneetbooks.in
8.1 Matrices 373

PROOF Recall that Om is a square matrix of order m ´ m in which all the entries are zero. Now, consider
Om × A = (Om + Om ) A
= Om A + Om A [by part (2) of Theorem 8.5]
Therefore, we have
Om A + Om A = Om A = Om A + Om´ n [by part (3) of Theorem 8.1]
By the cancellation law [part (5) of Theorem 8.1], we get
Om A = Om ´ n
Similar arguments yield
AOn = Om ´ n ■

T H E O R E M 8 .10 Let A be m ´ n matrix and B and C be n ´ p matrices. Then the following hold:
1. A(- B) = - ( AB) = (- A)B
2. A( B - C ) = AB - AC
3. ( A - D)B = AB - DB for any m ´ n matrix D.
PROOF 1. Consider the zero matrix On ´ p. Then, we have
AB + A(- B) = A[ B + (- B)] [by part (1), Theorem 8.5]
= A ×On´ p [by part (4), Theorem 8.1]
= Om´ p [by Theorem 8.9]
= AB + [- ( AB)] [by part (4), Theorem 8.1]
By the cancellation law [part (5), Theorem 8.1], we get that
A(- B) = - ( AB)
Similar argument gives us that (- A)B = -( AB).
2. We have
A( B - C ) = A[ B + (-C )]
= AB + A(-C )
= AB + (- AC ) = AB - AC

3. We have
( A - D)B = [ A + (- D)]B
= AB + (- D)B
= AB + (- DB) = AB - DB ■

Unlike in the number system, product of two non-zero matrices can be zero. Consider the following example.

Example 8.7

Let A and B be non-zero matrices given by Solution: We have


é2 2ù é 2 -2 ù é 2 ´ 2 + 2(- 2) 2(- 2) + 2 × 2 ù é0 0 ù
A=ê ú and B = ê AB = ê ú=ê
ë2 2û ë -2 2 úû ú
ë 2 ´ 2 + 2(- 2) 2(- 2) + 2 × 2 û ë0 0 û
and
Show that AB and BA are zero matrices.
é 2 ´ 2 + (- 2)2 2 × 2 + (- 2)2 ù é0 0 ù
BA = ê ú=ê ú
ë(- 2)2 + 2 ´ 2 (- 2)2 + 2 ´ 2 û ë0 0 û
www.jeeneetbooks.in
374 Chapter 8 Matrices, Determinants and System of Equations

Further, for two matrices A and B, it is quite possible that AB = 0 without BA being zero. In the following we have
such an instant.

Example 8.8

Let A and B be 2 ´ 2 matrices given by and

é0 1ù é 1 1ù é 1 1ù é0 1ù é 1× 0 + 1× 0 1× 1 + 1× 1 ù é0 2 ù
A=ê ú and B = ê ú B× A = ê úê ú=ê ú=ê ú
ë0 1û ë0 0 û ë0 0 û ë0 1û ë0 × 0 + 0 × 0 0 × 1 + 0 × 1û ë0 0 û
Find out AB and BA. Are these zero matrices? Therefore A × B = O and B × A ¹ O.

Solution: We have

é0 1ù é 1 1ù é0 × 1 + 1× 0 0 × 1 + 1× 0 ù é0 0 ù
A× B = ê úê ú=ê ú=ê ú
ë0 1û ë0 0 û ë0 × 1 + 1× 0 0 × 1 + 1× 0 û ë0 0 û

DEF IN IT ION 8 . 17 Let A be a square matrix of order m ´ m. For any non-negative integer n, define An recur-
sively as follows:

ìI m if n = 0
An = í n - 1
î A × A if n > 0
Note that
A1 = A0 × A = Im × A = A; A2 = A1 × A = A × A; A3 = A2 × A = ( A × A) × A; etc.
Also A2, A3, … are defined only when A is a square matrix.

Example 8.9

Let A be a scalar matrix éa2 0 0ù


ê ú
éa 0 0 ù =ê0 a 2

ê0 a 0 ú ê0 0 a2 úû
ê ú ë
êë0 0 a úû
Infact, for any n ³ 0,
where a is a given scalar. Then, find A2 and An.
éan 0 0ù
Solution: For given A we have ê ú
An = (a I 3 )n = an I3 = ê 0 n
a 0ú
A = (a I3 ) × (a I3 ) = a( I3 × a)I3
2 ê0 0 an úû
ë
= a(a I3 ) I3 = a2 I3

Example

For a non-zero square matrix A and a positive integer n, é 2 4ù é 2 4ù


An may be zero, but A may not be zero. For example, Then A2 = ê ú ê ú
consider ë-1 - 2 û ë-1 - 2 û

é 2 4ù
A=ê ú
ë -1 - 2 û
www.jeeneetbooks.in
8.1 Matrices 375

é 2 × 2 + 4(- 1) 2 × 4 + 4(- 2) ù é0 0 ù
=ê ú=ê ú
ë(- 1)2 + (- 2)(- 1) (- 1) × 4 + (- 2)(- 2)û ë0 0 û

T H E O R E M 8 .11 Let A and B be square matrices of order m ´ m and s a scalar. Then the following hold good:
1. For any integer n ³ 0, ( sA)n = sn An
2. ( A + B)2 = A2 + AB + BA + B2
3. ( A - B)2 = A2 - AB - BA + B2
4. ( A + B)( A - B) = A2 - AB + BA - B2
5. If AB = BA, then ( A + B)( A - B) = A2 - B2
6. An × Ar = An + r
7. ( An )r = Anr
8. ( A + B)3 = A3 + A2 B + ABA + BAB + B2 A + AB2 + BA2 + B3
9. If AB = BA, then ( A + B)3 = A3 + 3 A2 B + 3 AB2 + B3
PROOF The proofs of (1), (6) and (7) are by induction. The others are straightforward verifications and
are left to the reader. ■

DEF IN IT ION 8 . 18 Let f ( x) = a0 + a1 x + a2 x2 + an xn be a polynomial in the indeterminate x and the coefficients


ai’s be scalars. Then, for any m ´ m matrix A, we define
f ( A) = a0 + a1 A + a2 A2 + + an An
where each ai is treated as the scalar matrix of order m ´ m in which each diagonal entry is ai
and the other entries are 0. Note that f(A) is again an m ´ m matrix. f(A) is said to be a matrix
polynomial.

Example

For any square matrix A and a scalar s This is a matrix polynomial and is equal to f(A) where
n n
( s + A)n = å n Cr An - r sr f ( x) = ( s + x)n = å n Cr An - r sr
r =0 r =0

Example

The product of any two diagonal (scalar) matrices of the Therefore, except the diagonal entries, all the other
same order is again a diagonal (scalar) matrix. entries in AB are zero. Therefore AB is a diagonal matrix.
If A = (aij) and B = (bij) are diagonal matrices of If A and B are scalar matrices, then
order n ´ n, then
aii = bjj and bii = bjj for all 1 £ i, j £ n
aij = 0 = bij for all 1 £ i ¹ j £ n
and hence aii bii = ajj × bjj and this shows that AB is also a
The ijth entry in the product AB is scalar matrix.
n
ìaii bii if i = j
åa b ir rj = aii bij = í
if i ¹ j
r =1 î 0
www.jeeneetbooks.in
376 Chapter 8 Matrices, Determinants and System of Equations

Example 8.10

Solve the matrix equation éa + 2b 2a + 3bù é 2 1ù


êc + 2d 2c + 3d ú = ê 1 3ú
XA = B ë û ë û
where Equating the corresponding entries on both sides, we
have
é1 2ù é 2 1ù
A=ê ú and B = ê ú
ë2 3û ë 1 3û a + 2b = 2; c + 2d = 1; 2a + 3b = 1; 2c + 3d = 3
Solving these, we get that a = - 4, b = 3, c = 3 and d = - 1.
Solution: Since A and B are 2 ´ 2 matrices, to satisfy
Therefore
the equation XA = B, X also must be a 2 ´ 2 matrix. Let
é- 4 3ù
éa b ù X=ê ú
X=ê ú ë 3 - 1û
ëc d û

From XA = B, we have

Example 8.11

Let A be the matrix éa a a a ù é1 1 1 1ù


é1 1ù
êa a a a ú ê1 1 1 1ú
1 1 Am = Am -1 × A = ê úê ú
ê1 1 1 1ú êa a a a ú ê1 1 1 1ú
ê ú ê úê ú
ê1 1 1 1ú ëa a a a û ë1 1 1 1û
ê ú
ë1 1 1 1û é 4a 4a 4a 4a ù
ê 4a 4a 4a 4a ú
Prove that =ê ú (wherre a = 4m - 2 )
ê 4a 4a 4a 4a ú
ê ú
é 4n - 1 4n - 1 4n - 1 4n - 1 ù ë 4a 4a 4a 4a û
ê n-1 ú
4 4n - 1 4n - 1 4n - 1 ú é 4m - 1 4m - 1 4m - 1 4m - 1 ù
An = ê n - 1 ê m-1 ú
ê4 4n - 1 4n - 1 4n - 1 ú 4 4m - 1 4m - 1 4m - 1 ú
ê n-1 ú = ê m-1
êë 4 4n - 1 4n - 1 4n - 1 úû ê4 4m - 1 4m - 1 4m - 1 ú
ê m-1 ú
for any positive integer n. êë 4 4m - 1 4m - 1 4m - 1 úû

Solution: We shall use induction on n. If n = 1, it is clear.


Let m > 1 and assume that result is true for n = m - 1. Then

Example 8.12

Two persons X and Y wanted to purchase onions, Market I Market II


tomatoes and potatoes for each of their families. The Onions 10 9
quantities in kilograms of each of the items required by
X and Y are given in the table below: Tomatoes 6 7
Potatoes 8 9
Onions Tomatoes Potatoes
X 12 6 6 Prepare a comparison table, showing the probable expen-
ditures of the persons X and Y in the two markets I,
Y 10 4 5
and II, and their preferences of I and II. It is given that
There are two markets in the town, market I and market II. they can go to only one market each.
The costs per kilogram of each item in the two markets are
given in rupees in the table in the right column: Solution: Let A be the person–item matrix, that is

é12 6 6ù
A=ê
ë10 4 5 úû
www.jeeneetbooks.in
8.1 Matrices 377

and B be the item–market matrix, that is é 204 204 ù


=ê ú
é10 9ù ë164 163 û
B = êê 6 7 úú Therefore the required table is
êë 8 9 úû
Market I Market II
We have to compute person–market matrix, that is, we X Rs. 204 Rs. 204
have to calculate the product A·B. Now A is a 2 ´ 3 matrix Y 164 163
(2 person – 3 items) and B is a 3 ´ 2 matrix (3 items – 2
markets) and therefore A·B is a 2 ´ 2 matrix (2 person – 2 Then X understands that both markets are equally prefe-
markets) given by rable, while Y decides to go to market II, when it is given
that the qualities are same in markets I and II.
é12 × 10 + 6 × 6 + 6 × 8 12 × 9 + 6 × 7 + 6 × 9 ù
A× B = ê ú
ë10 × 10 + 4 × 6 + 5 × 8 10 × 9 + 4 × 7 + 5 × 9 û

DEF IN IT ION 8 . 19 For any matrix A, the transpose of A is defined to be the matrix obtained by interchanging
the rows and columns in A. The transpose of A is denoted by AT or A¢. If A = (aij ) is an m ´ n
matrix, then the transpose AT is an n ´ m matrix given by
AT = (aij¢ )
where aij¢ = aji . That is, the ijth entry in A becomes the jith entry in AT.

Examples
é 1 3ù é 2 3 4ù
é1 2 - 1ù ê- 4 - 2 é2 -4 1 2ù
(1) If A = ê A = ê 2 - 1úú
Tê 3ú
- 1 2 úû
, then
(2) If A = ê ú , then AT = ê 3 -2 6 - 3 úú
ë3 êë - 1 2 úû ê 1 6 5ú ê
ê ú ê4
ë 3 5 - 1 úû
ë 2 - 3 -1û

T H E O R E M 8 .12 Let A and B be m ´ n matrices and s a scalar. Then


1. ( A + B)T = AT + BT
2. ( sA)T = s × AT
3. (- A)T = - AT
PROOF Let A = (aij) and B = (bij). Then, A, B and A + B are matrices of order m ´ n and therefore
AT , BT , AT + BT and (A + B)T are all n ´ m matrices. For any 1 £ i £ n and 1 £ j £ m, ijth entry in

( A + B)T = jith entry in A + B


= aji + bji
= ijth entry in AT + ij th entry in BT
= ijth entry in AT + BT

Therefore

( A + B)T = AT + BT
Similarly, we can prove that ( sA)T = s × AT and deduce, by taking s = - 1, that (- A)T = - AT. ■

Example 8.13

Consider the following matrices: é 2 3 1ù é -1 2 - 3ù


A=ê ú and B = ê
ë- 4 -1 5û ë- 2 1 4 úû
www.jeeneetbooks.in
378 Chapter 8 Matrices, Determinants and System of Equations

Compute AT , BT , AT + BT , A + B and ( A + B)T . Therefore

Solution: We have é 1 -6ù


( A + B) = êê 5
T
0 úú
é2 -4ù é -1 - 2 ù êë - 2 9 úû
A = êê 3
T
- 1 ú and B = êê 2
ú T
1 úú
êë 1 5 úû êë - 3 4 úû Also

Now é 2 + (- 1) - 4 + (- 2)ù
A + B = êê 3 + 2
T T
- 1 + 1 úú
é 2 + (- 1) 3+ 2 1 + (- 3) ù êë 1 + (- 3) 5 + 4 úû
A+ B=ê
ë - 4 + (- 2) -1 + 1 5 + 4 úû
é 1 -6ù
é 1

5 -2ù = êê 5 0 úú = ( A + B)T
ë-6 0 9 úû êë - 2 9 úû

T H E O R E M 8 .13 Let A be an m ´ n matrix and B an n ´ p matrix. Then


( AB)T = BT × AT
PROOF First note that AB is defined, since A and B are of order m ´ n and n ´ p, respectively, and that
AB is of order m ´ p and hence ( AB)T is of order p ´ m. Also, since BT and AT are of order
p ´ n and n ´ m, respectively; BT × AT is defined and is of order p ´ m. Therefore ( AB)T and BT AT
are both of order p ´ m. For any 1 £ i £ p and 1 £ j £ m, we have

ijth entry in ( AB)T = jith entry in AB


n
= å ajr bri
r =1
n
= å bri × ajr
r =1
n
= å bi¢r × arj¢ [where AT = (ars¢ ) and BT = (brs¢ )]
r =1

= ijth entry in BT × AT
Thus ( AB)T = BT × AT. ■

Example 8.14

Consider the following matrices: é 5 -5 7ù


AB = ê
é4 - 3 1ù ë- 2 3 - 7 úû
é 2 -3 1ù
A=ê and B = ê 2 - 1 - 2 úú
ê
é 5 -2ù
ë -1 4 - 2 úû
êë 3 - 2 - 1 úû ( AB) = êê - 5
T
3úú
êë 7 7 úû
Compute AT , BT , AT × BT , A × B and ( AB)T .
Also
Solution: Since A and B are of order 2 ´ 3 and 3 ´ 3,
respectively, AB is defined and is of order 2 ´ 3 . We have é 8 - 6 + 3 - 4 + 8 - 6ù é 5 -2ù
é 2 -1ù é 4 2 3ù B A = êê - 6 + 3 - 2
T T
3 - 4 + 4 úú = êê - 5 3úú
ê
A = ê-3
T ú T ê
4 ú and B = ê - 3 - 1 - 2 úú êë 2 + 6 - 1 - 1 - 8 + 2 úû êë 7 - 7 úû
êë 1 - 2 úû êë 1 - 2 - 1 úû
www.jeeneetbooks.in
8.1 Matrices 379

Example 8.15

Recall Example 8.12 where two persons X and Y wanted é 12 10 ù


é10 6 8 ùê
4 úú
to purchase the items onions, tomatoes and potatoes
B ×A = ê
T T
6
in markets I or II. The matrix A is given as the person– ë 9 7 9 úû ê
item matrix and B as the item–market matrix. Instead, êë 6 5 úû
suppose we are given the item–person matrix and the é10 × 12 + 6 × 6 + 8 × 6 10 × 10 + 6×× 4 + 8 × 5ù
market–item matrix. Then we have the following tables. =ê ú
ë 9 × 12 + 7 × 6 + 9 × 6 9 × 10 + 7 × 4 + 9 × 5 û
X Y é 204 164 ù
=ê ú = (A
A × B)T
Onions 12 10 ë 204 163 û
Tomatoes 6 4
The required table is
Potatoes 6 5
X Y
Onions Tomatoes Potatoes Market I 204 164
Market I 10 6 8 Market II 204 163
Market II 9 7 9

These are simply AT and BT, respectively. BT is a 2 ´ 3


matrix and AT is the 3 ´ 2 matrix. If we take product
BT × AT , then we get the market–person matrix. Now

DEF IN IT ION 8 . 20 A matrix A is said to be symmetric if it is equal to its transpose, that is, A = AT . If A = (aij ),
then A is called symmetric if aij = aji for all i and j. Note that if A is an m ´ n matrix, then AT
is an n ´ m matrix and therefore A can be symmetric only if A is a square matrix.

Examples

é1 2 0ù é 2 1ù
(2) ê ú is not a symmetric matrix
(1) ê 2 - 1 4 ú is a symmetric matrix, because when we ë 4 3û
ê ú
êë 0 4 3 úû é2 3 1ù
interchange the rows and columns, we get the same (3) ê ú is not symmetric, since a symmetric
matrix. ë 4 - 2 - 3û
matrix is necessarily a square matrix.

QUICK LOOK 2

1. The zero matrix of order n ´ n and the identity 2. Any diagonal matrix is always symmetric.
matrix are both symmetric matrices.

Note that a square matrix is symmetric if and only if the entries on the lower side of the diagonal are precisely the
reflections of those on the upper side of in the diagonal as shown in Figure 8.1.

4 2 0 1

2 -1 3 -2

0 3 -4 -1

1 -2 -1 5 Diagonal

FIGURE 8.1 Symmetric square matrix.


www.jeeneetbooks.in
380 Chapter 8 Matrices, Determinants and System of Equations

DEF IN IT ION 8 . 21 A square matrix A = (aij ) is said to skew-symmetric if aij = - aji for all i and j. In other words,
A is skew-symmetric if and only if A = - A .
T

Examples

é 0 2 3 4ù é 1 2 3 4ù
ê- 2 0 1 - 3ú ê- 2 0 1 3ú
(1) The matrix ê ú is skew-symmetric. (2) The matrix ê ú is not skew-symmetric
ê - 3 -1 0 - 1ú ê - 3 -1 0 2 ú since a ¹ - a .
ê ú ê ú 11 11
ë- 4 3 1 0û ë- 4 - 3 - 2 0û

In the following we derive certain important properties of symmetric matrices and skew-symmetric matrices.

T H E O R E M 8 .14 Let A and B be any n ´ n square matrices.


1. If A and B are symmetric, then so is A ± B.
2. If A and B are skew-symmetric, then so is A ± B.
3. If AB = BA and A and B are symmetric (skew-symmetric), then AB is symmetric.
4. If A is symmetric (skew-symmetric), then so is sA for any scalar s.
PROOF Recall that A is symmetric if and only if A = AT and that A is skew-symmetric if and only if A = - AT .
1. Suppose that A and B are symmetric. Then, by Theorem 8.12, we have
( A ± B)T = AT ± BT = A ± B
and therefore A ± B is also symmetric.
2. If A and B are skew-symmetric matrices, then
( A ± B)T = AT ± BT = - A ∓ B = - ( A ± B)
and therefore A ± B is skew-symmetric.
3. Case I: Suppose A and B are symmetric and AB = BA. Then
( AB)T = BT AT = B × A = AB
and therefore AB is symmetric.
Case II: If A and B are skew-symmetric and AB = BA, then
( AB)T = BT AT = (- B)(- A) = BA = AB
and therefore AB is symmetric.
4. Case I: If A is symmetric and s is a scalar, then
( sA)T = sAT = sA
and hence sA is symmetric.
Case II: If A is skew-symmetric, then
( sA)T = sAT = s(- A) = -( sA)
and hence sA is skew-symmetric. ■

Note that, if A and B are skew-symmetric and AB = BA, then AB is not a skew-symmetric; however, AB is symmetric.
In this context, we have the following.

T H E O R E M 8 .15 Let A and B be square matrices of same order such that AB = BA. If one of A and B is symmetric
and the other is skew-symmetric, then AB is skew-symmetric.
www.jeeneetbooks.in
8.1 Matrices 381

PROOF Suppose that A is symmetric and B is skew-symmetric (there is no loss of generality, since
AB = BA). Then
( AB)T = BT AT = (- B) A = -( BA) = -( AB)

and therefore AB is skew-symmetric. ■

T H E O R E M 8 .16 Let A be a square matrix. Then A is symmetric (skew-symmetric) if and only if AT is symmetric
(skew-symmetric).
PROOF This follows from the fact that ( AT )T = A. Also since A is symmetric
A = AT Þ AT = A = ( AT )T
and A = -AT Þ AT = -A = -( AT )T ■

T H E O R E M 8 .17 If A is a skew-symmetric matrix, then all the diagonal entries in A are zero.

PROOF Let A = (aij) be a skew-symmetric matrix. Then aij = - aji for all i and j. In particular, aii = -aii and
hence 2aii = 0 or aii = 0 for all i. Therefore all the diagonal entries aii are zero. ■

Note: The converse of Theorem 8.17 is not true. For example the matrix

é0 1 2ù
ê3 0 4ú
ê ú
êë 1 - 1 0 úû

is not skew-symmetric.

T H E O R E M 8 .18 For any square matrix A, A + AT is symmetric and A – AT is skew-symmetric.

PROOF Let A be a square matrix. Then


( A + AT )T = AT + ( AT )T = AT + A = A + AT
and hence A + AT is symmetric. Also,

( A - AT ) = [ A + (- AT )]T
= AT + (- AT )T = AT + [-( AT )T ]
= AT + (- A)
= AT - A = -( A - AT )

Therefore A - AT is skew-symmetric. ■

T H E O R E M 8 .19 Any square matrix can be uniquely expressed as a sum of a symmetric matrix and a skew-
symmetric matrix.
PROOF Let A be any square matrix. Then, by Theorem 8.18, A + AT is symmetric and A - AT is skew-
symmetric. Also, by part (4) of Theorem 8.14 we have
1
( A + AT ) is symmetric
2

1
and ( A - AT ) is skew-symmetric
2
www.jeeneetbooks.in
382 Chapter 8 Matrices, Determinants and System of Equations

Now, we have
1 1
A= ( A + AT ) + ( A - AT ) (8.11)
2 2
To prove the uniqueness of this expression, let A = B + C, where B is a symmetric matrix and C is
a skew-symmetric matrix. Then

A + AT = ( B + C ) + ( B + C )T
= B + C + BT + C T
= B+C + B-C
= 2B
and therefore
1
B= ( A + AT )
2
Also,
A - AT = ( B + C ) - ( B + C )T
= B + C - ( BT + C T )
= B + C - (B - C )
= 2C
and therefore
1
C= ( A - AT )
2
Thus, Eq. (8.11) is an unique expression of A as a sum of a symmetric matrix and a skew-symmetric
matrix. ■

T H E O R E M 8 .20 Let A and B be symmetric matrices of the same order. Then the following hold:
1. An is symmetric for all positive integers n.
2. AB is symmetric if and only if AB = BA.
3. AB + BA is symmetric.
4. AB - BA is skew-symmetric.
PROOF 1. For any positive integer n,
( An )T = ( A A)T = AT AT = A A = An
and therefore An is symmetric.
2. AB is symmetric Û ( AB)T = AB
Û BT AT = AB
Û BA = AB
3. ( AB + BA)T = ( AB)T + ( BA)T
= BT AT + AT BT
= BA + AB = AB + BA
Therefore AB + BA is symmetric.
4. ( AB - BA)T = ( AB)T - ( BA)T
= BT AT - AT BT
= BA - AB = - ( AB - BA)
Therefore AB – BA is skew-symmetric. ■
www.jeeneetbooks.in
8.1 Matrices 383

T H E O R E M 8 .21 For any square matrix A, AAT and ATA are both symmetric.
PROOF We have
( AAT )T = ( AT )T AT = AAT

and ( AT A)T = AT ( AT )T = AT A
Therefore AAT and ATA are both symmetric. ■

Example 8.16

Express the matrix A as a sum of a symmetric matrix and Again


a skew-symmetric matrix.
é 2 - 2 3 - (- 1) 1- 5 ù é 0 4 -4ù
é 2 A - AT = êê - 1 - 3 - 2 - (- 2) 4 - (- 3) úú = êê - 4 7 úú
3 1ù
0
ê
A = ê-1 - 2 4 úú êë 5 - 1 -3 - 4 - 5 - (- 5)úû êë 4 - 7 0 úû
êë 5 - 3 - 5úû
é ù
Solution: To do this, we should compute ê 0 2 -2 ú
ê ú
1 7ú
1
( A + AT ) and
1
( A - AT ) ( A - AT ) = ê - 2 0 (8.13)
2 2 2 ê 2ú
ê -7 ú
Now transpose of A is given by ê 2 0ú
ë 2 û
é2 -1 5 ù By Theorem 8.19 and using Eqs. (8.12) and (8.13) we
A = êê 3 - 2 - 3úú
T
have
êë 1 4 - 5úû 1 1
A = ( A + AT ) + ( A - AT )
2 2
For AT, first row of A becomes the first column of AT, the
ith row of A becomes the ith column of AT. Now é ù é ù
ê2 1 3ú ê 0 2 -2 ú
é 2 + 2 3 + (- 1) 1 + 5 ù é4 2 6ù ê ú ê ú
ê ú ê 1ú ê 7ú
A + A = ê - 1 + 3 - 2 + (- 2) 4 + (- 3) ú = ê 2 - 4
T
1 úú = ê1 -2 + -2 0
ê 2ú ê 2ú
êë 5 + 1 -3 + 4 - 5 + (- 5)úû êë 6 1 - 10 úû ê ú ê ú
1 -7
ê3 -5 ú ê 2 0ú
ë 2 û êë 2 úû
é ù
ê2 1 3ú symmetric skew-symmetric
ê ú
1 1ú
( A + AT ) = ê 1 - 2 (8.12)
2 ê 2ú
ê 1 ú
ê3 - 5ú
ë 2 û

Example

Let A and B be square matrices of same order. If B is = ABT AT


symmetric (skew-symmetric), then so is ABAT. That is
ì ABAT if B is symmetric
( ABA ) = ( A ) B A
T T T T T T =í
î A(- B) A if B is skew-symmetric = - ABA
T T

DEF IN IT ION 8 . 22 A square matrix A is said to be an orthogonal matrix if AT × A = I, where I is the identity
matrix of order same as that of A.
www.jeeneetbooks.in
384 Chapter 8 Matrices, Determinants and System of Equations

Example 8.17

Prove that the following matrices are orthogonal: é1 0ù


=ê ú=I
é cos a - sin a ù ë0 1û
(1) A = ê
ë - sin a cos a úû Therefore, A is an orthogonal matrix.
é sin a cos a ù (2) Let
(2) A = ê
ë - cos a sin a úû é sin a cos a ù
A=ê
ë - cos a sin a úû
Solution:
(1) Consider the matrix Then
é cos a sin a ù é sin a - cos a ù
A=ê
ë - sin a cos a úû AT = ê
sin a úû
ëcos a
Then and
écos a - sin a ù ésin2 a + cos2 a sin a cos a - cos a sin a ù
AT = ê
cos a úû A A=ê ú
T
ë sin a êëcos a sin a - sin a cos a cos2 a + sin2 a úû
and é1 0ù
=ê ú=I
écos2 a + sin2 a cos a sin a - sin a cos a ù ë0 1û
AT× A = ê ú
êësin a cos a - cos a sin a sin2 a + cos2 a úû Therefore, A is an orthogonal matrix.

Example 8.18

Let é 2a2 0 0 ù
ê ú
é 0 2b cù =ê 0 6b2 0 ú
ê
A = êa b - c úú ê 0 0 3c2 úû
ë
êëa - b c úû
A is orthogonal if and only if
Find the values of a, b and c such that A is an orthogonal
é1 0 0ù
matrix.
AT A = I = êê0 1 0 úú
Solution: For the given matrix we have êë0 0 1úû

é 0 a aù that is,
AT = êê 2b b - búú 1 1 1
êë c - c c úû a=± , b=± and c = ±
2 6 3

Multiplying AT with A we get


AT A

é(0 + a2 (0 × 2b + ab (0 × c - ca ù
ê + a2 ) - ab) + ca) ú
ê ú
ê ú
(2b × 0 + ba [2b × 2b + b × b [2b × c + b(- c) ú
=ê - ba) + (- b)(- b)] + (- b)c] ú
ê
ê ú
ê[c × 0 + (- c)a [2b × c + (- c)b [c2 + (- c)(- c)ú
ê + c × a] + c(- b)] + c2 ] ú
êë úû
www.jeeneetbooks.in
8.1 Matrices 385

DEF IN IT ION 8 . 23 The following operations on matrices are called elementary transformations or elementary
operations:
1. The interchange of any two rows (or columns).
2. The multiplication of any row (or column) by a non-zero scalar.
3. The addition to the entries of a row (or column) the corresponding entries of any other
row (or column) multiplied by a non-zero scalar.

There are totally six types of elementary transformations on a matrix, three types are due to rows and three types due
to columns. An elementary transformation is called a row transformation or a column transformation according as it
applies to rows or columns, respectively. We follow a fixed notation to describe these six elementary transformations
as detailed below.
1. We denote the elementary transformations of interchanging the ith row and jth row by Ri « Rj . For example, for
a matrix

é 2 3 1 4ù
A = êê - 1 2 - 3 2 úú
êë 3 4 2 1 úû

applying the elementary transformation R2 « R3, that is, interchanging the second row and third row, we get the
matrix

é 2 3 1 4ù
ê 3 4 2 1ú
ê ú
êë - 1 2 - 3 2 úû

2. The elementary transformation of interchanging the ith column and jth column is denoted by Ci « Cj . For example,
by applying the transformation C1 « C3 to the matrix A given above we get the matrix

é 1 3 2 4ù
ê- 3 2 -1 2 ú
ê ú
êë 2 4 3 1 úû

3. The elementary transformation of multiplying the entries in the ith row by a non-zero scalar s is denoted by Ri ® sRi .
For example, application of the transformation R3 ® 2 R3 to the matrix

é 2 3 1 4ù
A = êê - 1 2 - 3 2 úú
êë 3 4 2 1 úû

gives us the matrix

é 2 3 1 4ù
ê-1 2 - 3 2 ú
ê ú
êë 6 8 4 2 úû

4. The elementary transformation of multiplying the entries of the ith column by a non-zero scalar s is denoted by
Ci « sCi . For example, the application at C3 « 2C3 to the matrix A given above gives us the matrix

é 2 3 2 ×1 4ù é 2 3 2 4ù
ê - 1 2 2(- 3) 2 ú = ê - 1 2 - 6 2 ú
ê ú ê ú
êë 3 4 2 × 2 1 úû êë 3 4 4 1 úû
www.jeeneetbooks.in
386 Chapter 8 Matrices, Determinants and System of Equations

5. The transformation of adding to the entries in ith row, the corresponding entries of the jth row multiplied by a non-
zero scalar s is denoted by Ri ® Ri + sRj . For example, when we apply the transformation R2 ® R2 + 3R1 to the matrix

é 2 3 1 4ù
A = ê - 1 2 - 3 2 úú
ê
êë 3 4 2 1 úû

we get the matrix

é 2 3 1 4 ù é2 3 1 4 ù
ê - 1 + 3 × 2 2 + 3 × 3 - 3 + 3 × 1 2 + 3 × 4 ú = ê 5 11 0 14 ú
ê ú ê ú
êë 3 4 2 1 úû êë 3 4 2 1 úû

6. Lastly, the transformation of adding to the entries in ith column, the corresponding entries in the jth column
multiplied by a non-zero scalar s is denoted by Ci ® Ci + sCj . For example, the application of C2 ® C2 + 3C1 to the
matrix A in (5) gives us the matrix

é2 3 + 3× 2 1 4ù é 2 9 1 4ù
ê - 1 2 + 3(- 1) - 3 2 ú = ê - 1 - 1 - 3 2 ú
ê ú ê ú
êë 3 4 + 3× 3 2 1 úû êë 2 13 2 1 úû

DEF IN IT ION 8 . 24 Let A and B be two matrices of the same order and et be an elementary transformation
et
(i.e., et is any six transformations described above). Then we write A ¾¾ ® B to denote
that B is obtained by applying the elementary transformation et to A. For example
R «R
A ¾¾¾
1 2
®B

denotes that B is obtained by interchanging the ith row and jth row in A. It can be easily seen that
et f
A ¾¾
® B implies B ¾¾
®A

where f is another elementary transformation, which may be called the inverse transformation of et.

For example, Ri « Rj (or Ci « Cj ) is inverse of itself. Ri ® (1/ s)Ri [Ci ® (1/ s)Ci ] is the inverse of Ri ® sRi (Ci ® sCi ).
Also, Ri ® Ri + sRj is the inverse of Ri ® Ri + (- s)Rj and Ci ® Ci + sCj is the inverse of Ci ® Ci + (- s)Cj . In other words,
the inverse of an elementary transformation is again an elementary transformation.

DEF IN IT ION 8 . 25 A square matrix A is said to be an elementary row (column) matrix if it is obtained by apply-
et
ing an elementary row (column) transformation to the identity matrix I, that is, I ¾¾ ® A.

Examples

(1) The matrix (2) Let

é1 0 0ù é0 0 1ù
ê0 0 1ú E = êê0 1 0 úú
ê ú
êë0 1 0 úû êë 1 0 0 úû

is an elementary matrix, since it is obtained by inter- Then E is an elementary column matrix, since
C1 «C3
changing the second and third rows in the identity I ¾¾¾ ® E.
matrix (3) Let
é1 0 0ù
ê0 1 0 ú é 1 0 3ù
ê ú E = êê0 1 0 úú
êë0 0 1úû
êë0 0 1úû
www.jeeneetbooks.in
8.1 Matrices 387

C ®C + 3C
Then I ¾¾¾¾¾
3 3 1
® E and hence E is an elementary é1 0 0ù é1 0 0ù
column matrix. ê ú R2 ® 4 R2
(4) ê0 1 0 ú ¾¾¾¾ ® êê0 4 0 úú
êë0 0 1úû êë0 0 1úû
and therefore this is an elementary row matrix.

The following theorem is a straightforward verification and is left for the reader.

Try it out

T H E O R E M 8.2 2 Let A and B square matrices of same order.


ert
1. For any elementary row transformation ert, A ¾¾® B if and only if EA = B, where E is
ert
the elementary matrix for which I ¾¾® E.
ect
2. For any elementary column transformation ect, A ¾¾® B if and only if B = AE, where E
ect
is the elementary matrix for which I ¾¾® E.

Examples

(1) Let (2) Let

é3 1 4ù é3 1 4 ù é 1 2 3ù é 1 11 3ù
ê
A=ê 2 3 - 2 ú and B = êê 2 3 - 2 úú
ú ê ú
A = ê 4 5 6 ú and B = êê 4 23 6 úú
êë - 1 - 2 - 3 úû êë 3 4 - 7 úû êë 7 8 9 úû êë 7 35 9 úû
R ®R + 2 R C ®C + 3C
Then A ¾¾¾¾¾
3 3 2
® B and Then A ¾¾¾¾¾
2 2 3
® B and

é3 1 4 ù é1 0 0ù é 3 1 4ù é 1 11 3ù é 1 2 3ù é 1 0 0 ù
ê ú ê ú ê
B = ê 2 3 - 2 ú = ê0 1 0 ú ê 2 3 - 2 úú = EA B = êê 4 23 6 úú = êê 4 5 6 úú êê0 1 0 úú = A × E
êë 3 4 - 7 úû êë0 2 1úû êë - 1 - 2 - 3 úû êë 7 35 9 úû êë 7 8 9 úû êë0 3 1úû

where where

é1 0 0ù é1 0 0ù é1 0 0ù
E = ê0 1 0 ú and I = êê0 1 0 úú ¾¾¾
ê ú
E = êê0 1 0 úú and I ¾¾¾¾¾
R3 ® R3 + 2 R1 C2 ®C2 + 3C3
¾¾ ®E ®E
êë0 2 1úû êë0 0 1úû êë0 3 1úû

DEF IN IT ION 8 . 26 Two matrices A and B of same order are said to be similar or equivalent if one can be obtained
from the other by applying a finite number of elementary transformations. If A and B are
similar or equivalent, we denote this by A ∼ B.

In other words A ∼ B if there exist finite number of matrices B1 , B2 , … , Bn = B such that


f f f f f
A ¾¾
1
® B1 ¾¾
2
® B2 ¾¾
3
® B3 ¾¾
4
® ¾¾
® Bn-1 ¾¾
n
® Bn = B

where f1 , f2 , … , fn are some elementary transformations. We assume the validity of the following theorem without
going to the intracacies of the proof.

T H E O R E M 8 .23 Being similar is an equivalence relation on the class of all matrices, that is, for any matrices A, B
and C, the following hold:
1. A ∼ A
2. A ∼ B and B ∼ C Þ A ∼ C
3. A ∼ B Þ B ∼ A
www.jeeneetbooks.in
388 Chapter 8 Matrices, Determinants and System of Equations

Examples

é3 5 9ù é1 0 4 ù
(1) Let A = êê 2 6 4 úú . Then ® êê 3 1 12 úú
C3 ®C3 + 4C1
¾¾¾¾¾
êë 1 2 3 úû êë0 0 1 úû

é3 5 0 ù é1 0 4 ù
® êê 2 6 - 2 úú ® êê 3 1 12 úú
C3 ®C3 - 3C1 R3 ® R3 + 2 R2
A ¾¾¾¾¾ ¾¾¾¾¾
êë 1 4 0 úû êë6 2 25úû

é3 5 0ù é 7 2 29 ù
¾¾¾¾® êê - 1 1 - 2 úú ¾¾¾¾® êê 3 1 12 úú = A, say.
R2 ® R2 - R1 R1 ® R1 + R3

êë 1 4 0 úû êë 6 2 25 úû

é3 5 0 ù Therefore I3 ∼ A.
¾¾¾¾® êê0 5 - 2 úú
R2 ® R2 + R3
(3) Consider
êë 1 4 0 úû
é 7 2 29 ù é1 0 4 ù
A = êê 3 1 12 úú ¾¾¾¾ ê 3 1 12 ú
R1 ® R1 - R3
é2 1 0 ù ® ê ú
® êê 0 5 - 2 úú êë 6 2 25 úû êë6 2 25úû
R1 ® R1 - R3
¾¾¾¾
êë 1 4 0 úû
é1 0 4 ù
® êê 3 1 12 úú
R3 ® R3 - 2 R2
é2 1 0ù ¾¾¾¾¾
® êê 0 5 - 2 úú êë0 0 1 úû
R3 ® R3 - 2 R1
¾¾¾¾¾
êë - 3 2 0 úû
é1 0 0ù
® êê 3 1 0 úú
C3 ®C3 - 4C1
é2 1 0ù
5
¾¾¾¾¾
C2 ®C2 + C3
® êê 0 0 - 2 úú = B(say) êë0 0 1úû
2
¾¾¾¾¾
êë - 3 2 0 úû
é1 0 0ù
® êê0 1 0 úú = I3
R2 ® R2 - 3 R1
Therefore A ∼ B. ¾¾¾¾¾
êë0 0 1úû
é1 0 0ù é1 0 0ù
ê ú R2 ® R2 + 3 R1
(2) I3 = ê0 1 0 ú ¾¾¾¾¾ ® êê 3 1 0 úú Therefore A ∼ I3.
êë0 0 1úû êë0 0 1 úû

DEF IN IT ION 8 . 27 A square matrix A of order n is said to be invertible or non-singular if there exists a square
matrix B of order n such that
AB = In = BA
where In is the identity matrix of order n, B is called inverse of A and is denoted by A–1.

T H E O R E M 8 .24 For any n ´ n matrix A, there is atmost one inverse of A.

PROOF Suppose that B and C are inverses of A. Then AB = In = BA and AC = In = CA. Now,
B = In B = (CA)B = C ( AB) = CIn = C

Therefore B = C or there is atmost one inverse of A. ■


www.jeeneetbooks.in
8.1 Matrices 389

Example

Let
é1 0 0ù
é 2 - 1 3ù é - 7 - 9 10 ù = êê0 1 0 úú = I3
A = ê - 5 3 1ú and B = êê - 12 - 15 17 úú
ê ú
êë0 0 1úû
êë - 3 2 3úû êë 1 1 - 1úû

Then Similarly, BA = I3 = AB. Therefore A is invertible and


A–1 = B.
AB

é[2(- 7) + (- 1)(- 12) [2(- 9) + (- 1)(- 15) [2 × 10 + (- 1)17 ù


ê + 3 × 1] + 3 × 1] + 3(- 1)] ú
ê ú
ê[(- 5)(- 7)) + 3(- 12) [(- 5)(- 9) + 3(- 15) [(- 5) × 10 + 3 × 17 ú
=ê + 1× 1] + 1× 1] + 1(- 1)] ú
ú
ê
ê ú
ê[(- 3)(- 7) + 2(- 12) [(- 3)(- 9) + 2(- 15) [(- 3) × 10 + 2 × 17 ú
ê + 3 ×11] + 1× 1] + 3(- 1)]ú
ë û

Before going to find an algorithm to find the inverse, if it exists, of a square matrix, we have the following.

T H E O R E M 8 .25 Let A and B be square matrices of the same order.


1. For any elementary row transformation f,
f ( AB) = f ( A) B
2. For any elementary column transformation g
g( AB) = A × g( B)
PROOF 1. By part (1) of Theorem 8.22, we have
f ( AB) = E( AB) = (EA)B = f ( A)B
where E = f ( I ).
2. Again by part (2) of Theorem 8.22, we have
g( AB) = ( AB)E = A( BE) = Ag( B)
where E = g( I ) ■

T H E O R E M 8 .26 Let A and B be invertible square matrices of the same order. Then AB is invertible and
( AB)-1 = B-1 × A-1

PROOF We have A × A-1 = I = A-1 × A and BB-1 = I = B-1 B. Now

( AB)( B-1 A-1 ) = A( BB-1 ) A-1 = AIA-1 = AA-1 = I


and ( B-1 A-1 )( AB) = B-1 ( A-1 A)B = B-1 IB = B-1 B = I
and hence AB is invertible and ( AB)-1 = B-1 A-1. ■

In the following, we prove that, for any invertible matrix, the operations of taking transpose and inverse commute with
each other.
www.jeeneetbooks.in
390 Chapter 8 Matrices, Determinants and System of Equations

T H E O R E M 8 .27 A square matrix A is invertible if and only if its transpose is invertible and, in this case
( AT )-1 = ( A-1 )T
That is, the transpose of the inverse of A is the inverse of transpose of A.

PROOF Suppose that A is invertible. Then


A × A-1 = I = A-1 × A
Taking transposes, we get
( A-1 )T AT = ( AA-1 )T = I T = I = ( A-1 A)T = AT × ( A-1 )T
Therefore, AT is invertible and its inverse is ( A-1 )T and therefore ( AT )-1 = ( A-1 )T . The converse
follows from the facts that ( AT )T = A and ( A-1 )-1 = A. ■

T H E O R E M 8 .28 Every elementary matrix is invertible.


PROOF First recall, from the discussion made after Definition 8.24, that any elementary row (or
column) transformation has an inverse which is again an elementary row (respectively column)
transformation. If E is an elementary matrix of order n ´ n, then
E = f (I )
for a suitable elementary row (or column) transformation f, where I is the identity matrix of order n.
Then f -1 is also an elementary row (or column) transformation. Let
F = f -1 ( I )
Suppose that f is an elementary row transformation. Then, by part (1) of Theorem 8.25, we have
E × F = f ( I )F = f ( IF ) = f (F ) = f ( f -1 ( I )) = I
and FE = f -1 ( I )E = f -1 ( IE) = f -1 (E) = f -1 ( f ( I )) = I
If f is a column transformation, again by part (2) of Theorem 8.25, we have
E × F = E × f -1 ( I ) = f -1 (EI ) = f -1 (E) = f -1 ( f ( I )) = I
and F ) = f ( f -1 ( I )) = I
F × E = F × f ( I ) = f (FI ) = f (F
Thus EF = I = FE and hence E is invertible and F is the inverse of E. ■

T H E O R E M 8 .29 Let A be an invertible matrix. Then in each row (and in each column) there is atleast one non-zero entry.

PROOF Let A = (aij ) and A-1 = (bij ). Then, for each i, the ith diagonal entry (i.e., iith entry) in A × A-1 (= I )
is 1 and hence
n
1 = å air ari
r =1

Therefore air ¹ 0 for some r (otherwise, the above sum becomes 0). Similarly, for each j,
arj ¹ 0 for some r ■

T H E O R E M 8 .30 Let A be a square matrix of order n. Then A is invertible if and only if A and In are similar.

PROOF Suppose that A and In are similar. Then there exist finite number of matrices B1 , B2 , … , Bm = In
such that
f f f f f
A ¾¾
1
® B1 ¾¾
2
® B2 ¾¾
3
® ¾¾¾
m- 1
® Bm-1 ¾¾
m
® Bm = In
www.jeeneetbooks.in
8.1 Matrices 391

where each fi is either an elementary row transformation or an elementary column transformation.


Then, by Theorem 8.22,

Bi = R Bi - 1 or Bi = Bi - 1C

for each i, where R is an elementary row matrix or C is an elementary column matrix, according
as whether fi is a row transformation or column transformation, respectively. Therefore, there
exists elementary row matrices R1 , R2 , … , Rs and elementary column matrices C1 , C2 , … , Ct such
that

In = (R1 R2 Rs ) A (C1 C2 Ct ), s + t = m

Put R = R1 R2 Rs and C = C1 C2 Ct . Then, by Theorems 8.28 and 8.26, R and C are invertible
matrices and In = RAC. Hence
A = R-1 InC -1 = R-1C -1 = (CR)-1
Thus, A is invertible.
Conversely, suppose that A is invertible and let A = (aij ). If a11 = 0, then some entry, say ai1
in the first column of A, is non-zero and we interchange the first row and ith row, by apply-
ing the row transformation R1 « Ri , to get a matrix whose (1-1)th entry is not zero. Then, by
applying the transformation R1 ® (1/ a11 )R1, we get a matrix whose (1-1)th entry is 1. Then apply
R2 ® R2 - a21R1 , R3 ® R3 - a31R1 , …, Rn ® Rn - an1R1 successively to get a matrix whose (1-1)th
entry is 1 and the other entries in the first column are zeroes. Next, in the resulting matrix, consider
second diagonal element (i.e., (2-2)th entry). For some r ³ 2, ar 2 ¹ 0 (see the following remark)
and then exchange the rth row with the second row. (2-2)th entry is not zero. Make it 1 by applying
a row transformation and then make ar 2 = 0 for all r ¹ 2. Now take up (3-3)th element and make
it 1 and other r3th elements (r ¹ 3) zeroes. We can continue the process until we get In. We thus
have a sequence
f f f
A ¾¾
1
® A1 ¾¾
2
® A2 ¾¾
3
® A3 ¾¾
® ¾¾
® Am = In

of elementary row operations. Therefore A and In are similar. ■

The reader is advised to go through Examples 8.19 and 8.20 to get a better understanding of the above proof.

Remark: Let A = (aij ) be an n ´ n invertible matrix and 1 £ r £ n satisfying the following:


aii = 1 for all i < r
aij = 0 for all 1 £ i £ n, and i ¹ j < r

Then there exists i ³ r such that air ¹ 0. For, suppose that air = 0 for all i ³ r. Consider
C ®C - a C C ®C - a C Cr ®Cr - ar -1, rCr -1
A ¾¾¾¾¾
r r 1r 1
® A1 ¾¾¾¾¾
r r 2r 2
® A2 ¾¾
® ¾¾
® Ar -1 ¾¾¾¾¾¾
® Ar

Now A and Ar are similar and hence Ar is similar to In so that Ar is also invertible. But the rth column of Ar contains
only zeroes, which is a contradiction to Theorem 8.29. Therefore air ¹ 0 for some i ³ r.

C O R O L L A R Y 8 .3 Let A and B be similar square matrices of the same order. Then A is invertible if and only B is
invertible.
PROOF Since A ∼ B, we have A ∼ In Û B ∼ In. Now, we can use the above theorem to get the required
result. ■

Note that, in the second part of the proof of Theorem 8.30, we have reduced a given invertible matrix to In by using
certain row transformations only. A similar procedure can be followed using the column transformations only.
Therefore, we have the following.
www.jeeneetbooks.in
392 Chapter 8 Matrices, Determinants and System of Equations

C O R O L L A R Y 8 .4 Any invertible matrix can be expressed as a product of finite number of elementary row matrices
as well as a product of finite number of elementary column matrices.

Example 8.19
Consider the matrix é - 22 ù
é 2 3 1ù ê1 0 34 ú
ê- 3 ê ú
4 5úú
11
ê
R3 ® R3 - R2
2 ê 13 ú
¾¾¾¾¾ ® ê0 1
êë - 5 - 2 1úû
ê 17 úú
Reduce it to I3 using elementary row transformations. ê0 0 - 24 ú
ê 34 úû
ë
Solution: We have
é 3 1ù
- 11 ù
1 é
é 2 3 1ù R1 ® 1 R1 ê 2ú ê1 0 17 ú
2
ê ú
A = êê - 3 4 5úú ¾¾¾¾
2
® ê- 3 4 5ú ê ú
ê ê 13 ú
êë - 5 - 2 1úû ë- 5 - 2 1 ûú = ê0 1
ê 17 úú
é 3 1ù ê0 0 - 12 ú
ê 1 ê 17 úû
2 2ú ë
ê ú
17 13 ú
®ê 0
R2 ® R2 + 3 R1
¾¾¾¾¾ é -11 ù
ê 2 2ú
ê- 5 ê1 0 17 ú
ë -2 1 úû 17 ê ú
R3 ®- R3
2 ê 13 ú
¾¾¾¾® ê0 1
é 3 1ù 17 ú
ê1 2 ê ú
2ú êë0 0 1 úû
ê ú
17 13 ú
® ê0
R3 ® R3 + 5 R1
¾¾¾¾¾ -11 ù
ê 2 2ú é
ê 11 7 úú ê1 0 17 ú
ê0 ê ú
13
R2 ® R2 - R3
17
êë 2 2 úû ¾¾¾¾¾® ê0 1 0ú
ê ú
ë0 0 1û
é 3 1ù
ê1 2 2ú
2 ê ú 11
R1 ® R1 + R3
é1 0 0 ù
® êê0 1 0 úú = I3
R2 ® R2
13 ú
¾¾¾¾® ê0 1
17
17
¾¾¾¾¾
ê 17 ú
ê ú êë0 0 1 úû
ê0 11 7ú
êë 2 2 úû

é -22 ù
ê1 0 34 ú
3 ê ú
R1 ® R1 - R2
2 ê 13 ú
¾¾¾¾¾ ® ê0 1
17 ú
ê ú
ê0 11 7ú
ê 2 2 úû
ë

Example 8.20

Reduce the matrix given in Example 8.19 to I3 using é 3 1ù


é 2 3 1ù C ® 1 C ê 1
column transformations only. 2ú
A = êê - 3 4 5úú ¾¾¾¾
1 1 2
®ê ú
2

ê -3 4 5ú
Solution: We have êë - 5 - 2 1úû ê -5 -2
ë 1 úû
www.jeeneetbooks.in
8.1 Matrices 393

é 1ù é ù
ê 1 0
2ú ê 1 13 0 0 ú
3
C2 ®C2 - C1 ê ú ê
C3 ®C3 - C2
2 ú
17 ¾¾¾¾¾® ê 0 1 0 ú
® ê- 3 5ú
2
¾¾¾¾¾
ê 2 ú ê 52 11 - 24 ú
ê ú ê- ú
ê- 5 11 ë 17 34 û
1ú 17
êë 2 úû
é ù
ê 134 0 0ú
é ù ê
C3 ®- C3
24 ú
ê 1 0 0ú ¾¾¾¾® ê 0 1 0ú
1
C3 ®C3 - C1 ê ú ê 52 11 ú
17 13 ú ê- 1ú
® ê- 3
2
¾¾¾¾¾ ë 17 17 û
ê 2 2ú
ê 11 7ú
ê- 5 ú é ù
ë 2 2û ê 1 11 0 0ú
ê
C2 ®C2 - C3
17 ú
é ù ¾¾¾¾¾® ê 0 1 0ú
ê 1 0 0ú ê - 52 ú
2
C2 ® C2 ê ú ê 0 1ú
13 ú
¾¾¾¾® ê - 3 1 ë 17 û
17

ê 2ú
ê 11 7ú é1 0 0ù 52
ê- 5 ú C1 ®C1 + C3
® êê0 1 0 úú = I3
17
ë 17 2û ¾¾¾¾¾
êë0 0 1úû
é ù
ê 1 0 0ú
ê ú
C1 ®C1 + 3C2 13 ú
¾¾¾¾¾ ®ê 0 1
ê 2ú
ê 52 11 7ú
ê- ú
ë 17 17 2û

In the above discussion, we have given a procedure to reduce an invertible matrix into the identity matrix using only
the row transformations (only the column transformations). This can be used as an algorithm to find the inverse of a
given invertible matrix.

8.1.1 Algorithm to Find Inverse Using Only the Row Transformations (Gauss–Jordan Method)
Let A be an invertible matrix of order n ´ n. Then we get matrices A1 , A2 , … , As = In and elementary row transforma-
tions f1 , f2 , … , fs such that
f f f f f
A ¾¾
1
® A1 ¾¾
2
® A2 ¾¾
3
® ¾¾
s -1
® As -1 ¾¾
s
® As = In

Then In = As = fs ( As - 1 ) = ( fs × fs - 1 )( As - 2 ) = ( fs fs - 1 f1 )( A). If R1 , R2 , … , Rs are elementary row matrices correspond-


ing to f1 , f2 , … , fs , respectively, then
In = Rs × Rs -1 R1 × A
and therefore Rs Rs-1 R1 is the inverse of A. Note that this is same as fs ( I ) fs -1 ( I ) f1 ( I ).
This procedure can be easily remembered by the following method. Consider the equation
A = In A
Apply successively f1 , f2 , … , fs to get
f1 ( A) = f1 ( IA) = f1 ( I ) A
f2 ( f1 ( A)) = f2 ( f1 ( I ) A) = f2 ( f1 ( I )) × A

I = ( fs fs - 1 f1 )( A) = ( fs fs - 1 f2 f1 )( I ) × A
Therefore ( fs fs-1 f2 f1 )( I ) is the inverse of A.
www.jeeneetbooks.in
394 Chapter 8 Matrices, Determinants and System of Equations

Example 8.21

Find the inverse of the matrix é 1 0 - 1ù é - 2 1 0ù


ê0 1 æ 1 ö
é0 1 2 ù ê 2 úú = ê 1 0 0 ú A ç by R3 ® R3 ÷
ê ú è 2 ø
A = êê 1 2 3 úú êë0 0 1 úû ê 5
-
3 1ú
ê ú
êë 3 1 1 úû ë 2 2 2û

by using elementary row transformations. é 1 0 - 1ù é - 2 1 0ù


ê0 1 0 ú = ê - 4 3 - 1 ú A (byy R2 ® R2 - 2 R3 )
ê ú ê ú
Solution: We have A = IA. That is êë0 0 1úû ê 5 3 1ú
ê - ú
ë 2 2 2û
é0 1 2 ù é 1 0 0 ù
ê 1 2 3 ú = ê0 1 0 ú A é 1 1 1ù
ê ú ê ú -
êë 3 1 1 úû êë0 0 1úû é1 0 0ù ê 2 2 2ú
ê0 1 0 ú = ê - 4 3
ú
- 1 ú A (by R1 ® R1 - R3 )
ê ú ê
é 1 2 3 ù é0 1 0 ù êë0 0 1úû ê 5 3 1ú
ê0 1 2 ú = ê 1 0 0 ú A (by R « R ) ê - ú
ê ú ê ú 1 2 ë 2 2 2û
êë 3 1 1 ûú êë0 0 1úû
Therefore
é1 2 3 ù é0 1 0ù
ú = ê1 é 1 1 1ù
ê0
ê 1 2 ú ê 0 0 úú A (by R3 ® R3 - 3R1 ) ê 2 -
2 2ú
êë0 - 5 - 8 úû êë0 - 3 1úû -1 ê ú
A = ê -4 3 -1 ú
ê 5 3 1ú
é1 0 -1 ù é - 2 1 0ù ê - ú
ê0 ú ê ë 2 2 2û
ê 1 2ú = ê 1 0 0 úú A (by R1 ® R1 - 2 R2 )
êë0 - 5 - 8 úû êë 0 - 3 1úû

é 1 0 - 1ù é - 2 1 0ù
ê0 1 ú ê
2ú = ê 1 0 0 úú A (by R3 ® R3 + 5R2 )
ê
êë0 0 2 úû êë 5 - 3 1úû

8.1.2 Algorithm to Find Inverse Using Only the Column Transformations


Let A be an invertible matrix of order n ´ n. Then by Corollary 8.4, we get matrices B1 , B2 , … , Br = In and elementary
column transformations g1 , g2 , … , gr such that
g1 g2 g3 g4 gr -1 gr
A ¾¾® B1 ¾¾® B2 ¾¾® B3 ¾¾® ¾¾¾
® Br - 1 ¾¾® Br = In

Then In = Br = gr ( Br - 1 ) = gr ( gr - 1 ( Br - 2 )) = = ( gr gr - 1 g1 )( A). If C1 , C2 , … , Cr are elementary column matrices corre-


sponding to g1 , g2 , … , gr respectively, then
Bi = gi ( Bi - 1 ) = gi ( Bi - 1 × I ) = Bi - 1 gi ( I ) = Bi - 1Ci

for all 1 £ i £ r (where I = In and B0 = A). Therefore


I = Br = Br - 1Cr = Br - 2Cr - 1Cr = = AC1C2 C

and hence C1 C2 Cr is the inverse of A. This procedure can be easily remembered as follows. Let us consider the
equation
A = AI
www.jeeneetbooks.in
8.2 Determinants 395

Apply g1 , g2 , … , gr successively to get


g1 ( A) = Ag1 ( I ) = AC1 = AC1 I
g2 ( g1 ( A)) = AC1 g2 ( I ) = AC1C2 = AC1C2 I

I = gr gr - 1 g1 ( A) = AC1C2 Cr
and hence C1 C2 Cr is the inverse of A.

Example 8.22

Find the inverse of the matrix given in Example 8.21 by é 1ù


using elementary column transformations. é 1 0 0ù ê- 2 1 2ú
ê 0 1 0ú = A ê 1 0 ú æ 1 ö
- 1 ú ç by C3 ® C3 ÷
ê ú ê è
Solution: We have A = AI where
ê 2 ø
êë - 5 3 1úû 1ú
ê 0 0 ú
é0 1 2 ù ë 2û
A = êê 1 2 3 úú
é 1 1ù
êë 3 1 1 úû é 1 0 0ù ê- 2 - 2 2ú
ê 0 1 0ú = A ê 1 3
ú
- 1 ú (by C2 ® C2 - 3C3 )
So we have ê ú ê
êë - 5 0 1úû ê 3 1ú
é0 1 2 ù é 1 0 0ù ê 0 - ú
ë 2 2û
ê 1 2 3ú = A ê0 1 0 ú
ê ú ê ú
êë 3 1 1úû êë0 0 1úû é 1 1 1ù
ê 2 -
é1 0 0ù 2 2ú
é1 0 2ù é0 1 0 ù ê0 1 0 ú = A ê - 4 3
ú
- 1 ú (by C1 ® C1 + 5C3 )
ê 2 1 3 ú = A ê 1 0 0 ú (by C « C ) ê ú ê
ê ú ê ú 1 2 êë0 0 1úû ê 5 3 1ú
êë 1 3 1 úû êë0 0 1 úû ê - ú
ë 2 2 2û
é1 0 0ù é0 1 0ù Therefore
ê 2 1 - 1ú = A ê 1 0 - 2 ú (by C ® C - 2C )
ê ú ê ú 3 3 1 é 1 1 1ù
êë 1 3 - 1úû êë0 0 1 úû ê 2 -
2 2ú
-1 ê ú
é 1 0 0ù é- 2 1 0ù A = ê -4 3 -1 ú
ê 0 1 - 1ú = A ê 1 0 - 2 ú (by C ® C - 2C ) ê 5 3 1ú
ê ú ê ú 1 1 2 ê - ú
êë - 5 3 - 1úû êë 0 0 1 úû ë 2 2 2û

é 1 0 0ù é- 2 1 1ù
ê 0 1 0 ú = A ê 1 0 - 2 ú (by C ® C + C )
ê ú ê ú 3 3 2

êë - 5 3 2 úû êë 0 0 1 úû

The process of finding the inverse of A by the elementary column transformations, as demonstrated in Example 8.22,
is abstracted in the following.

8.2 | Determinants
Let us recall that a system of two equations in two unknowns, for example,
a11 x1 + a12 x2 = b1
a21 x1 + a22 x2 = b2
has a unique solution if a11a22 - a21a12 ¹ 0. This system of equations can be expressed in matrix form as
é a11 a12 ù é x1 ù é b1 ù
êa =
ë 21 a22 úû êë x2 úû êë b2 úû
www.jeeneetbooks.in
396 Chapter 8 Matrices, Determinants and System of Equations

If the 2 ´ 2 matrix on the left side is invertible, then we can multiply the above matrix equation by the inverse of this
matrix on the left side to get that
-1
é x1 ù é a11 a12 ù é b1 ù
ê x ú = êa a22 úû êëb2 úû
ë 2 û ë 21
The existence of the inverse of this 2 ´ 2 matrix depends on whether the number a11a22 - a21a12 is non-zero. This number
is known as the determinant of the matrix. In this section, we shall define the concept of the determinant of any square
matrix and study its properties. As usual we take matrices over real or complex numbers only.

DEF IN IT ION 8 . 28 Let A be a square matrix of order n ´ n. Then the determinant of A, which is denoted by | A |
or det A, is defined inductively as follows.

1. If A is a 1 ´ 1 matrix, say A = [a11 ], then

det A = a11
2. If A is a 2 ´ 2 matrix, say
é a11 a12 ù
A=ê
ëa21 a22 úû

then we define
det A = a11a22 - a21a12
3. If A is a 3 ´ 3 matrix, say

é a11 a12 a13 ù


A = (aij ) = êêa21 a22 a23 úú
êëa31 a32 a33 úû

then det A is defined as

é a22 a23 ù éa21 a23 ù éa21 a22 ù


a11 det ê ú - a12 det ê ú + a13 det ê
ëa32 a33 û ëa31 a33 û ëa31 a32 úû

4. In general, let A = (aij ) be an n ´ n matrix, say

é a11 a12 a13 a1n ù


êa a22 a23 a2 n ú
A=ê ú
21

ê     ú
ê ú
ëan1 an 2 an 3 ann û

For any 1 £ i, j £ n, let Bij be the matrix of order (n - 1) ´ (n - 1) obtained from A by


deleting the ith row and jth column. Then determinant of A is given by
n
det A = å (- 1)1+ j a1 j det B1 j = a11 det B11 - a12 det B12 + + (- 1)n + 1 a1n det B1n
j =1

Example 8.23

Find the determinant of the following matrices: é 2 1 3ù


é 1 2ù
ê
(2) A = ê - 3 4 - 2 úú
(1) A = ê ú êë - 1 2 1úû
ë3 4û
www.jeeneetbooks.in
8.2 Determinants 397

Solution: second column from A and B13 is obtained by delet-


(1) For the given matrix, the determinant is ing first row and third column from A.) Now

det A = 1× 4 - 3 × 2 = 4 - 6 = - 2 det A = (- 1)1+ 1 a11 det B11 + (- 1)1+ 2 a12 det B12
(2) For the given matrix, the determinant can be calcu- + (- 1)1+ 3 a13 det B13
lated as = 2(4 × 1 - 2(- 2)) - 1× (- 3 × 1 - (- 2)(- 1))
+ 3(- 3 × 2 - (- 1)4)
é4 - 2ù é- 3 - 2ù é- 3 4ù
B11 = ê ú , B12 = ê ú and B13 = ê ú = 2(4 + 4) - (- 3 - 2) + 3(- 6 + 4)
ë2 1û ë -1 1û ë -1 2 û
= 16 + 5 - 6 = 15
(B11 is obtained from A by deleting first row and
first column, B12 is obtained by deleting first row and

N O TAT I O N 8 .1 For a square matrix A = (aij ), we write |A| or |aij| also for the det A. For example, if
é 2 3 1ù
A = êê 1 2 3úú
êë - 4 - 1 3úû

then

2 3 1
det A = 1 2 3 = | A|
- 4 -1 3

QUICK LOOK

1. The determinant is defined only for a square matrix. That is, the determinant will remain the same on
2. For any square matrix A = (aij ) and 1 £ i, j £ n, let Bij expanding it along any row or any column.
be the matrix obtained from A by deleting the ith row 3. In particular, | A | = | AT |; that is, the determinant of a
and jth column in A. Then, it can be proved that square matrix A is same as that of its transpose.
n n
det A = å (- 1)i + j aij det Bij = å (- 1)i + j aij det Bij
j =1 i =1

Example

Let = 2(4 - ( - 4)) + 3(1 - 6) - ( - 2 - 12)


é2 1 3ù = 16 - 15 + 14
A = êê - 3 4 - 2 úú
= 15 = | A | [see part ( 2 ) , Examplle 8.23]
êë - 1 2 1 úû
Also, expanding det A along the second column of A we get
Then (- 1)1+ 2 × 1×(- 3 - 2) + (- 1)2 + 2 × 4 ×(2 + 3) + (- 1)2 + 3 × 2 ×(- 4 + 9)
é 2 - 3 - 1ù = - (- 5) + 20 - 10 = 15 = | A|
A = êê 1 4
T
2 úú Again, expanding det A along the third row we get
êë 3 - 2 1 úû (- 1)3 + 1 (- 1)(- 2 - 12) + (- 1)3 + 2 × 2 × (- 4 + 9)
and its determinant is given by + (- 1)3 + 3 × 1× (8 + 3)
= 14 - 10 + 11 = 15 = | A |
4 2 1 2 1 4
| AT | = 2 - ( - 3) + ( - 1) The reader can check that det AT = - 40 = det A [part (3),
-2 1 3 1 3 -2
Example 8.23].
www.jeeneetbooks.in
398 Chapter 8 Matrices, Determinants and System of Equations

8.2.1 Evaluation of the Determinant of a 3 ¥ 3 Matrix (Sarrus Diagram)


Let

é a11 a12 a13 ù


A = (aij ) = êêa21 a22 a23 úú
êëa31 a32 a33 ûú

Then
| A | = a11 (a22 a33 - a32 a23 ) - a12 (a21a33 - a31a23 ) + a13 (a21a32 - a31a222 )
= a11a22 a23 + a12 a31a23 + a13 a21a32 - (a11a32 a23 + a12 a21a33 + a13 a31a22 )

There is an easy of way of remembering this to evaluate the determinant of A. Write down the columns of the matrix A.
Write down the first and the second columns on the right side and draw broken lines as shown in Figure 8.2. Put + sign
before the products of the triplets on the downward arrows and – sign before the products of the triplets on the upward
arrows. The diagram in Figure 8.2 is called the Sarrus diagram.
a11 a12 a13 a11 a12

a21 a21 a23 a21 a22

a31 a32 a33 a31 a32

+, -
FIGURE 8.2 Sarrus diagram.

Example 8.24

Let Solution:
4 2 3 4 2
é4 2 3ù
A = ê 1 - 3 0 úú
ê
1 -3 0 1 -3
êë - 2 5 2 úû
-2 5 2 -2 5
Find the determinant of A by using the Sarrus diagram.
FIGURE 8.3 Sarrus diagram for Example 8.24.

From Figure 8.3 we have


| A | = 4(- 3)2 + 2 × 0(- 2) + 3 × 1× 5
- (- 2)(- 3)3 - 5 × 0 × 4 - 2 × 1× 2
= - 24 + 0 + 15 - 18 - 0 - 4 = - 31

In the following theorems, we state certain properties of determinants of matrices. The reader is advised to assume
these for the present and verify these in simpler cases of 2 ´ 2 and 3 ´ 3 matrices. Let us begin with the following
definition.

DE FIN IT ION 8 . 29 Let A = (aij ) be a square matrix of order n ´ n. For any 1 £ i, j £ n, let Mij be the determinant
of the (n - 1) ´ (n - 1) matrix obtained from A by deleting the ith row and jth column in A.
Note that the ith row is one in which aij occurs and the jth column is one in which aij occurs.
Then Mij is called the minor of aij. There will be n2 number of minors corresponding to an
n ´ n matrix, for each entry in the matrix. If A is 3 ´ 3 matrix then there will be 9 minors
associated with A.
www.jeeneetbooks.in
8.2 Determinants 399

Example 8.25

Find the minors M11, M12 and M23 of the following matrix: 1 3
M11 = = -4
é4 3 2ù 0 -4
ê
A = ê5 1 3 úú
5 3
êë 2 0 - 4 úû M12 = = 5(- 4) - 2 × 3 = - 26
2 -4
Solution: The minors associated with the given matrix 4 3
are: M23 = = -6
2 0

Try it out Find all the minors of the matrix A given in Example 8.25.

DEF IN IT ION 8 . 30 Let A = (aij ) be an n ´ n matrix. For any 1 £ i, j £ n the cofactor of aij is defined by (- 1)i + j Mij ,
where Mij is the minor of aij; that is, the determinant of the (n - 1) ´ (n - 1) matrix obtained
from A by deleting the ith row and jth column in A, multiplied by (- 1)i + j. The cofactor of aij
is denoted by Aij and is given by
Cofactor of aij = Aij = (- 1)i + j Mij

8.2.2 Formula for Determinant in Cofactors


Let A = (aij ) be an n ´ n matrix. Then
n
det A = | A | = å aij Aij for any 1 £ i £ n
j =1

n
= å aij Aij for any 1 £ j £ n
i =1

That is,
| A | = a11 A11 + a12 A12 + + a1n A1n = a11 A11 + a21 A21 + + a2 n A2 n

Example 8.26

Find the cofactors of the following matrices: (2) For the given matrix, the cofactors are
é3 2ù 2 6 4 6
(1) A = ê ú A11 = (- 1)1+ 1 = - 6; A12 = (- 1)1+ 2 =6
ë 1 4û 0 -3 -1 - 3
4 2 3 1
é 2 3 1ù A13 = (- 1)1+ 3 = 2; A21 = (- 1)2 + 1 =9
-1 0 0 -3
(2) A = êê 4 2 6 úú
êë - 1 0 - 3úû 2 1 2 3
A22 = (- 1)2 + 2 = - 5; A23 = (- 1)2 + 3 = -3
-1 - 3 -1 0
Solution: 3 1 2 1
A31 = (- 1)3 + 1 = 16; A32 = (- 1)3 + 2 = -8
(1) For the given matrix, the cofactors are 2 6 4 6

A11 = (- 1)1+ 1 4 = 4; A12 = (- 1)1+ 2 × 1 = - 1 2 3


A33 = (- 1)3 + 3 = -8
A21 = (- 1) × 2 = - 2; A22 = (- 1)2 + 2 × 3 = 3
2+1 4 2
www.jeeneetbooks.in
400 Chapter 8 Matrices, Determinants and System of Equations

T H E O R E M 8 .31 Let A = (aij ) be an n ´ n matrix. Let B be the matrix obtained by interchanging two rows (or column)
in A. Then | B | = - | A |.
PROOF We shall verify the theorem in special case when n = 3. Let

é a 11 a 12 a 13 ù éa 31 a 32 a 33 ù
A = êêa 21 a 22 a 23 úú and B = êêa 21 a 22 a 23 úú
êëa 31 a 32 a 33 úû êë a 11 a 12 a 13 úû

Now B is obtained from A by applying the fundamental row transformation R1 « R3. Then

| B | = a 31(a 22 a 13 - a 12 a 23) - a 32(a 21a 13 - a 11a 23) + a 33(a 21a 12 - a 11a 222 )
= a 31a 22 a 13 - a 31a 12 a23 - a 32 a 21a 13 + a 32 a 11a 23 + a 33 a 21a 12 - a 33 a 11a 22
= - a 11(a 22 a 33 - a 32 a 23) + a 12 (a 21a 33 - a 31a 23 ) - a 13(a 21a 32 - a 31a 22 )
= - | A|

Similar proof works for interchanging two columns. ■

C O R O L L A R Y 8 .5 Let A be an n ´ n matrix in which any two rows (or two columns) are identical. Then |A| = 0.
PROOF Let ith and kth rows in A be identical and B be the matrix obtained from A by interchanging the
Ri « Rk
ith row and kth row, that is A ¾¾¾® B. Then, by Theorem 8.31
| A| = - | B|
But, since the ith and kth rows are identical, A = B, and hence |A| = |B|. Therefore |A| = 0. Similarly
when two columns are identical, | A | = 0. ■

Examples
C1 « C3
(1) Let Then A ¾¾¾® B. We have
é1 2 3ù | A | = 1(1 × 2 - 4 × 3) - 3(2 × 2 - 3 × 3) + 4(2 × 4 - 3 × 1)
A = êê 3 1 4 úú = - 10 + 15 + 20 = 25
êë 1 2 3 úû | B | = 4(1 × 3 - 4 × 2) - 3(3 × 3 - 2 × 2) + 1(3 × 4 - 2 × 1)
Then |A| = 0, since first and third rows in A are iden- = - 20 - 15 + 10 = - 25
tical. We can check this, by actual evaluation of | A |: Therefore |A| = -|B|.
| A | = 1(1× 3 - 2 × 4) - 2(3 × 3 - 1× 4) + 3(3 × 2 - 1× 1)
= - 5 - 10 + 15 = 0
(2) Let
é1 3 4ù é4 3 1ù
A = ê 2 1 3 ú and B = êê 3 1 2 úú
ê ú
êë 3 4 2 úû êë 2 4 3 úû

T H E O R E M 8 .32 Let A = (aij) be a square matrix of order n ´ n and s a scalar (i.e., s is a real or complex number).
Let B be the matrix obtained from A by multiplying all the entries in a row (or a column) by s.
That is, for some 1 £ k £ n, if
Rk ® sRk Ck ® sCk
A ¾¾¾¾
® B or A ¾¾¾¾
®B

then | B | = s | A |.
www.jeeneetbooks.in
8.2 Determinants 401

Rk ® sRk
PROOF Suppose that A ¾¾¾¾ ® B. Let us evaluate the determinant of B along the kth row of B. Note
that, for any 1 £ j £ n, the kjth cofactor of B (i.e., the cofactor of the kjth entry in B) is same as that
of A. Now,
n
| B | = å s akj × Bkj
j =1
n
= å s akj × Akj
j =1

æ n ö
= s ç å akj × Akj ÷ = s | A |
è j =1 ø ■

Example 8.27

Let = 4(- 2 + 3) - 3(- 1 - 6) + 2(1 + 4)


é 4 3 2ù = 4 + 21 + 10 = 35
A = ê 1 2 - 3úú and
ê R2 ® 4R2
A ¾¾¾¾
®B | B| = 4 [ 8(- 1) - 1(- 12)] - 3[ 4(- 1)
êë - 2 1 - 1úû - (- 2)(- 12)] + 2 [4 × 1 - (- 2)8]
= 4(- 8 + 12) - 3(- 4 - 24) + 2(4 + 16)
Find B, |A|, |B| and s such that |B| = s|A|.
= 16 + 84 + 40 = 140
Solution: By hypothesis we get This gives

é4 3 2 ù |B | = 140 = 4 × 35 = 4 | A |
B = êê 4 8 - 12 úú
Therefore s = 4.
êë - 2 1 - 1 úû

Now
| A| = 4 [ 2(- 1) - 1(- 3)] - 3 [ 1(- 1)
- (- 2)(- 3)] + 2 [1× 1 - (- 2)2]

C O R O L L A R Y 8 .6 For any square matrix A of order n ´ n and for any scalar s,


| sA | = sn | A |

T H E O R E M 8 .33 Let A = (aij) be a square matrix of order n ´ n. For a fixed k, let each entry in the kth row of A be
a sum of two terms bkj and ckj, that is,
akj = bkj + ckj for each 1 £ j £ n
Let B = (bij) and C = (cij), where
bij = aij = cij for all i ¹ k
Then |A| = |B| + |C|.
PROOF Without loss of generality we can assume that k = 1 we are given that

éb 11 + c 11 b 12 + c 12 b 1n + c 1n ù
ê a a 22 a 2n ú
A=ê ú
21

ê    ú
ê ú
ë a n1 a n2 a nn û
www.jeeneetbooks.in
402 Chapter 8 Matrices, Determinants and System of Equations

é b 11 b 12 b 1n ù
êa a 22 a 2 n ú
B=ê ú
21

ê    ú
ê ú
ëa n1 a n 2 a nn û
é c 11 c 12 c 1n ù
êa a 22 a 2 n ú
C=ê ú
21

ê    ú
ê ú
ëa n1 a n 2 a nn û

Let us evaluate the determinants along the first rows. Note that, for any 1 £ j £ n, A1j = B1j = C1j ,
that is,
(1j)th cofactor of A = (1j)th cofactor of B = (1j)th cofactor of C
Therefore

n
| A | = å (b1 j + c1 j ) A1 j
j =1

n n
= å b1 j A1 j + å c1 j A1 j
j =1 j =1

n n
= å b1 j B1 j + å c1 j C1 j
j =1 j =1

= | B | + |C | ■

Try it out
Let

é2 + 3 4 + 5 1 + 3ù
A = êê 2 6 - 4 úú
êë - 3 -4 - 2 úû

Show that

2 4 1 3 5 3
| A| = 2 6 -4 + 2 6 -4
-3 - 4 -2 -3 - 4 - 2

Also, verify that

2+3 4 5 2 4 5 3 4 5
1+ 2 -3 6 = 1 -3 6 + 2 -3 6
4+5 -2 1 4 -2 1 5 -2 1

C O R O L L A RY 8.7 The value of the determinant of a matrix does not change when any row (or column) is multi-
R ® R + sR
plied by a scalar s and then added to any other row (or column); that is, if A ¾¾¾¾¾
i i k
® B or
C ®C + sC
A ¾¾¾¾¾ ® B, then | A | = | B |.
i i k

R1 ® R1 + sR3
PROOF Let A ¾¾¾¾® B and A = (aij ). Then
www.jeeneetbooks.in
8.2 Determinants 403

éa11 + sa31 a12 + sa32 a1n + sa3 n ù


ê a a22 a2 n ú
ê 21 ú
B = ê a31 a32 a3 n ú
ê ú
ê   ú
êë an1 an 2 ann úû

By Theorem 8.33,

sa31 sa32 sa3 n


a11 a12 a1n
a21 a22 a2 n
a21 a22 a2 n
| B| = + a31 a32 a3 n
  
   
an1 an2 ann
an1 an2  ann
a31 a32 a3 n
a21 a22 a2 n
= | A| + s a31 a32 a3 n (by Theorem 8.32)
   
an1 an 2 ann
= | A| + s × 0 = | A | (∵ the first and third rows are identical) ■

Example 8.28

Let Determinant of B is

é4 3 2ù | B | = 4(10 × 5 - 2 . 3) - 3 [14 × 5 - (- 1)3] + 2 [14 × 2 - (- 1)10]


A = êê 2 1 - 3úú and = 4 ´ 44 - 3 ´ 73 + 2 ´ 388 = 33
R2 ® R2 + 3 R1
A ¾¾¾¾¾
®B
êë - 1 2 5 úû Now
Find out B and show that |B| = |A|. | A | = 4 [1× 5 - 2(- 3)] - 3[2 × 5 - (- 1)(- 3)] + 2 [2 × 2 - (- 1)1]
= 4 ´ 11 - 3 ´ 7 + 2 ´ 5 = 33
Solution: By hypothesis we get B as
Therefore
é 4 3 2 ù é 4 3 2ù
B = ê 2 + 3 × 4 1 + 3 × 3 - 3 + 3 × 2 úú = êê 14 10 3 úú
ê |B| = |A|
êë - 1 2 5 úû êë - 1 2 5 úû

Example 8.29

Let A be a 3 ´ 3 matrix, in which each row is in geometric Solution: We have


progression. That is,
1 r r2 1 r r2 1 r r2
é a ar ar ù 2
| A | = a b bs bs2 = ab 1 s s2 = abc 1 s s2
ê ú
A = êb bs bs 2 ú c ct ct2 c ct ct2 1 t t2
ê c ct ct 2 ú
ë û
1 r r2 (applying R2 ® R2 − R1
Then show that | A | = abc(r - s)( s - t )(t - r ).
= abc 0 s - r s2 - r2 and R3 ® R3 − R1 and
0 t-r t2 - r2 using Theorem 8.32)
www.jeeneetbooks.in
404 Chapter 8 Matrices, Determinants and System of Equations

1 r r2 1 s+r
= abc( s - r )(t - r )
= abc( s - r ) 0 1 s+r (by Theorem 8.32) 1 t+r
0 t - r t - r2
2
= abc( s - r )(t - r )[t + r - ( s + r )]
1 r r2 = abc( s - r )(t - r )(t - s)
= abc( s - r )(t - r ) 0 1 s + r = abc(r - s)( s - t )(t - r )
0 1 t+r

Example 8.30

Let a, b, c be in AP. Evaluate the determinant of x+1 x+2 x+a


é x + 1 x + 2 x + aù | A| = 1 1 b-a
A = êê x + 2 x + 3 x + búú 2 2 c-a
êë x + 3 x + 4 x + c úû
é x + 1 x + 2 x + aù
Solution: Since a, b, c are in AP, b – a = c – b or 2b = = 2 êê 1 1 d úú = 0 (since R2 = R3 )
a + c. Let d be the common difference b – a = c – b. By first êë 1 1 d úû
applying R2 ® R2 - R1 and then R3 ® R3 - R1, we get

Let us recall that, for any square matrix A = (aij), the ijth minor is defined as the determinant of the matrix obtained by
deleting the ith row and the jth column from A. It is denoted by Mij. Also, the (ij)th cofactor is defined as (–1)i+jMij and
is denoted by Aij. Further recall that the determinant of A is defined by
n n
| A | = å aij Aij = å (- 1)i + j aij Mij
j =1 j =1

for any 1 £ i £ n and 1 £ j £ n.

DEF IN IT ION 8 . 31 Let A = (aij) be a square matrix. Then adjoint of A is defined as the transpose of the matrix
(Aij), where Aij is the ijth cofactor in A. The adjoint of A is denoted by adjoint A or adj A. The
ijth entry in adj A is the jith cofactor in A.

Note that adj A is also a square matrix whose order is same as that of A.

Example 8.31

Find adj A where A is given by 3 2 4 2


A31 = (- 1)3+1 = - 8; A32 = (- 1)3+ 2 = 14
é 4 3 2ù 1 -2 3 -2
A = ê 3 1 - 2 úú
ê 4 3
A33 = (- 1)3+ 3 = -5
êë - 1 2 - 4 úû 3 1

Solution: The cofactors of A are Therefore


T
1 -2 3 -2 é 0 14 7 ù é 0 16 - 8 ù
A11 = (- 1)1+1 = 0; A12 = (- 1)1+ 2 = 14
2 -4 -1 - 4 adj A = ê 16 - 14 - 11úú = êê14 - 14 14 úú
ê

3 1 3 2 êë - 8 14 - 5úû êë 7 - 11 - 5 úû
A13 = (- 1)1+ 3 = 7; A21 = (- 1)2 +1 = 16
-1 2 2 -4
4 2 4 3
A22 = (- 1)2 + 2 = - 14; A23 = (- 1)2 + 3 = - 11
-1 - 4 -1 2
www.jeeneetbooks.in
8.2 Determinants 405

T H E O R E M 8 .34 Let A be a square matrix of order n. Then


A × (adj A) = (adj A) × A = | A | In
PROOF Let A = (aij) and Aij be the cofactor of aij. Then, it can be verified that, for any 1 £ i, k £ n,

n
ì| A | if i = k
åa A ij kj =í
if i ¹ k
j =1 î0
We have

T éA A21 A31 … An1 ù


é A11 A12 … A1n ù ê 11
êA A A22 A32 … An 2 ú
A22 … A2 n ú ê 12 ú
adj A = ê ú =ê… … … … …ú
21

ê   …  ú ê ú
ê ú … … … … …ú
ë An1 An 2 … Ann û ê
êë A1n A2 n A3 n … Ann úû

Therefore

é| A | 0 0 … ù
0
ê 0 | A| 0 … ú
0
ê ú
A × adj A = ê 0 0 | A| … ú = | A |× In
0
ê ú
ê  … … … …ú
êë 0 0 0 … | A |úû

Similarly, (adj A) × A = |A| × In. ■

Try it out Verify Theorem 8.34 by considering a 3 ´ 3 matrix.

C O R O L L A R Y 8 .8 A square matrix A is invertible if and only if the determinant |A| is non-zero and, in this case,

1
A-1 = (adj A)
| A|

PROOF If |A| = 0, then by Theorem 8.34, we have (adj A) × A = O, the zero matrix. Therefore, we cannot
find a matrix B such that AB = In (otherwise adj A = O and A = O) and so A is not invertible.
Conversely, if |A| ¹ 0, then

æ 1 ö æ 1 ö
çè | A | adj A÷ø × A = In = A × çè | A | adj A÷ø

Hence A is invertible and


1
A-1 = (adj A)
| A| ■

Let us recall that an invertible matrix is also called a non-singular matrix. A matrix which is not invertible is called a
singular matrix. From the above result, a matrix is non-singular if and only if its determinant is non-zero. The following
theorem is stated without proof, as it is beyond the scope of this book. However, the reader can assume this and use
it freely in any instance.
www.jeeneetbooks.in
406 Chapter 8 Matrices, Determinants and System of Equations

T H E O R E M 8 .35 For any square matrices A and B of the same order

| AB | = | A | | B |
That is, the determinant of the product is equal to the product of the determinants. ■

C O R O L L A R Y 8 .9 Let A and B be square matrices of the same order. Then the product AB is non-singular if and
only if both A and B are non-singular.
PROOF Since |A| and |B| are real numbers, we have
| AB | = | A | | B | ¹ 0 Û | A | ¹ 0 and | B | ¹ 0
and the result follows from Corollary 8.8. ■

C O R O L L A R Y 8 .10 A square matrix is non-singular if and only if its transpose is non-singular.


PROOF Let A be a square matrix and AT be its transpose. Then, we know that |A| = | AT| [see part (4),
Quick Look 3]. Therefore A is non-singular if and only if AT is non-singular. ■

T H E O R E M 8 .36 Let A be a non-singular matrix. Then A is symmetric if and only if A-1 is symmetric.
PROOF Recall that a square matrix is called symmetric if it is equal to its transpose. By Theorem 8.27, we
have (AT )-1 = (A-1 )T. Therefore
A is symmetric Û A = AT
Û A-1 = ( AT )-1 = ( A-1 )T
Û A-1 is symmetric ■

T H E O R E M 8 .37 Let A be a skew-symmetric matrix of order n, where n is an odd integer. Then A is singular, that
is, its determinant is zero and hence A is not invertible.
PROOF Since A is skew-symmetric, we have
AT = -A
Therefore,
-| A | = (- 1)n | A | (since n is odd)
= | - A|

= | AT| = | A|
and hence | A | + | A | = 0 so that | A | = 0. Therefore, A is singular and hence not invertible. ■

T H E O R E M 8 .38 Let A be a non-singular matrix of order n. Then


| adj A| = | A|n-1
and hence adj A is non-singular.
PROOF In Theorem 8.34, we have proved that
A ×(adj A) = | A| In
Note that | A|In is the scalar matrix, in which each of the diagonal entries is |A| and each of the
non-diagonal entries is zero. Now, by Theorem 8.35,
www.jeeneetbooks.in
8.2 Determinants 407

| A| | adj A| = | A(adj A)|


= || A| In|
= | A|n

and, since | A| ¹ 0, it follows that


| adj A| = | A|n-1 ¹ 0 (since | A | ¹ 0)
and therefore adj A is non-singular. ■

Try it out
Prove that | adj A| = | A |n-1 is also true, even if | A | = 0.
Hint: When | A | = 0, then | adj A| = 0.

T H E O R E M 8 .39 Let A be a non-singular matrix of order n, where n ³ 2. Then


adj (adj A) = | A |n -2 × A

PROOF Put B = adj A. By Theorem 8.34,


B (adj B) = | B | In = | A |n -1 In

Therefore,
A × B(adj B) = A(| A |n - 1 In ) = | A |n - 1 ( AIn ) = | A |n - 1 A
Also
| A | In (adj B) = A × (adj A) × (adj B)
= AB (adj B)
= | A |n -1 A

Therefore, adj (adj A) = adj B = | A |n -2 × A. ■

T H E O R E M 8 .40 Let A be a non-singular matrix of order n and B and C any square matrices of order n. Then
AB = AC Þ B = C (left cancellation law )
BA = CA Þ B = C (right cancellation la
aw)
PROOF We have
AB = AC Þ A-1 ( AB) = A-1 ( AC )
Þ ( A-1 A)B = ( A-1 A)C
Þ In B = InC
Þ B=C
-1
Similarly, by multiplying with A on the right side, we get
BA = CA Þ B = C ■

T H E O R E M 8 .41 Let A and B be non-singular matrices of same order n. Then


adj ( AB) = (adj B)(adj A)
www.jeeneetbooks.in
408 Chapter 8 Matrices, Determinants and System of Equations

PROOF Recall that AB is non-singular and ( AB)-1 = B-1 A-1. Now, consider
( AB)(adj B)(adj A) = A( B × adj B)(adj A)
= A(| B |× In )(adj A) (by Theorem 8.34)
= | B | ( A In )(adj A)
= | B | ( A × adj A)
= | B || A | In
= | A || B | In
= | AB | In
= ( AB)[adj ( AB)] (by Theorem 8.34)
Since AB is non-singular, by Theorem 8.40, we get that
(adj B)(adj A) = adj ( AB) ■

T H E O R E M 8 .42 Let A be non-singular matrix of order n. Then


(adj A)T = adj (AT)
That is, the transpose of the adjoint is the adjoint of the transpose.

PROOF Since | AT | = |A| ¹ 0, AT and A are both invertible. Consider

(adj A)T × AT = ( A × adj A)T


= (| A | In )T
= | A | InT
= | A | In
= | AT | In
= (adj AT ) × AT
Therefore, by the right cancellation law,
(adj A)T = adj (AT) ■

T H E O R E M 8 .43 Let A be a non-singular matrix of order n. Then A is symmetric if and only if adj A is symmetric.
PROOF We have
A is symmetric Þ AT = A
Þ adj(AT ) = adj A

Þ (adj A)T = adj A


Þ adj A is symmetric
Conversely,
adj A is symmetric Þ adj(adj A) is symmetric
Þ | A |n - 2 × A is symmetric (by Theorem 8.39)
Þ A is symmetric ■

If B is symmetric, then s B is also symmetric for any non-zero scalar s. Recall that, for any real number s, the scalar
matrix, in which each diagonal entry is s and each of the other entries is zero, is also denoted by s. By writing s A, we
mean the product of the scalar matrix s with the matrix A. For example, if
www.jeeneetbooks.in
8.2 Determinants 409

é 2 3 1ù
A = êê - 2 1 - 3úú
êë - 1 2 4 úû

then
é s 0 0ù é 2 3 1ù
s A = ê0 s 0 ú ê - 2 1 - 3úú
ê ú ê
êë0 0 s úû êë - 1 2 4 úû

Also, s A is the matrix in which each entry is obtained by multiplying the corresponding entry in A with s. If A is a
non-singular matrix of order n, then sA is also non-singular for any non-zero scalar s and | s A | = sn | A |, since the deter-
minant of a scalar matrix s is equal to sn. Infact, the determinant of a diagonal matrix or a triangular matrix is equal to
the product of the diagonal entries.

T H E O R E M 8 .44 Let f (x) = a0 + a1 x + a2 x2 + + amxm be a polynomial in x, where a0, a1, a2, …, am are real numbers,
such that a0 ¹ 0. If A is a non-singular matrix such that f (A) = 0, then
-1
A-1 = (a1 + a2 A + a3 A2 + + am Am - 1 )
a0

PROOF Let A be a non-singular matrix and f (A) = 0. That is,

a0 + a1 A + a2 A2 + + am Am - 1 = 0
By multiplying both sides with A-1, we get that

a0 A-1 + a1 AA-1 + a2 A2 A-1 + + am Am × A-1 = 0


Therefore
-1
A-1 = (a1 + a2 A + a3 A2 + + am Am - 1 )
a0 ■

Example 8.32

Find real numbers a and b such that a + bA + A2 = O é a + 3b + 11 2b + 8 ù



a + b + 3úû
where
ë b+4
é3 2ù
A=ê ú Therefore,
ë1 1û
Also find A–1. a + 3b + 11 = 0; 2b + 8 = 0
b + 4 = 0; a + b + 3 = 0
Solution: First
Solving these we get b = - 4 and a = 1. Therefore
é3 2ù é3 2ù
A2 = A × A = ê úê 1 - 4 A + A2 = 0
ë1 1û ë1 1 úû
A-1 - 4 AA-1 + A2 A-1 = 0
é3× 3 + 2 × 1 3 × 2 + 2 × 1ù é11 8 ù
=ê =
ë 1× 3 + 1× 1 1× 2 + 1× 1 úû êë 4 3úû This gives the inverse as

Now, suppose that a and b are real numbers such that é4 0 ù é3 2ù é 1 - 2ù


A-1 = 4 - A = ê ú-ê ú=ê ú
0 = a + bA + A 2 ë0 4 û ë1 1û ë-1 3 û
éa 0 ù é 3b 2bù é11 8 ù
=ê ú+ê ú+ê ú
ë0 a û ë b b û ë 4 3û
www.jeeneetbooks.in
410 Chapter 8 Matrices, Determinants and System of Equations

Example 8.33

Find adj A, | A | and A-1 where Therefore,


T T
é3 2ù é A11 A12 ù é 1 - 1ù é 1 - 2 ù
A=ê ú adj A = ê = =
ë1 1û ë A21 A22 úû êë - 2 3úû êë - 1 3 úû

Solution: For the given matrix, we have 1 é 1 -2ù


A-1 = (adj A) = ê ú
| A| = 3 × 1 - 2 × 1 = 1 ¹ 0 | A| ë -1 3 û

Therefore, A is non-singular. The cofactors for A are


A11 = (- 1)1+ 1 × 1 = 1; A12 = (- 1)1+ 2 × 1 = - 1
A21 = (- 1)2 + 1 × 2 = - 2; A22 = (- 1)2 + 2 × 3 = 3

To find the inverse of a square matrix A, or to express A-1 in terms of A, the concept of a characteristic polynomial of
a square matrix and the much known Cayley–Hamilton Theorem are useful, especially for 2 ´ 2 and 3 ´ 3 matrices. Let
us begin with a definition.

DEF IN IT ION 8 . 32 If A is a square matrix and I is the corresponding unit matrix, then the polynomial |A - xI| in
x is called characteristic polynomial of A and the equation |A - xI | = 0 is called the character-
istic equation of the matrix A.

Example 8.34

Find the characteristic polynomial and characteristic so that


equation of
2-x 1
é2 1ù | A - xI | = = ( x - 2)( x + 1) - 1 = x2 - x - 3
1 -1 - x
ê 1 - 1ú
ë û
is the characteristic polynomial of A and its characteristic
Solution: equation is
Let
é2 1ù x2 - x - 3 = 0
A=ê ú
ë 1 - 1û

Example 8.35

Find the characteristic polynomial and characteristic = (1 - x)[(2 - x)(3 - x) - 0] + 1(0 + 1) + 2(2 - x)
equation for A given by
= (1 - x)(2 - x)(3 - x) + (5 - 2 x)
é 1 -1 2 ù = (1 - x)(6 - 5 x + x2 ) + 5 - 2 x
A = êê 0 2 1 úú = - x3 + 6 x2 - 13 x + 11
êë - 1 0 3 úû
This is the characteristic polynomial of A and its charac-
Solution: We have teristic equation is x3 - 6 x2 + 13 x - 11 = 0.

1- x -1 2
| A - xI | = 0 2-x 1
-1 0 3- x

The following theorem is stated without proof as it is not necessary for a student of plus two class. However, the
student can assume this and use it freely whenever it is needed.
www.jeeneetbooks.in
8.2 Determinants 411

T H E O R E M 8 .45 Every square matrix satisfies its characteristic equation; that is, if A is a square matrix of order
( C AY L E Y – n and
H A M I LT O N )
f ( x) = | A - xI | = a0 + a1 x + a2 x2 + + an xn = 0
is its characteristic equation, then
f ( A) = a0 In + a1 A + a2 A2 + + an An = 0
Also if a0 ¹ 0, then
-1
A-1 = (a1 I + a2 A + a3 A2 + + an An - 1 )
a0

Note that A-1 exists if and only if the constant term of the characteristic of A is non-zero. ■

Example 8.36

Show that the matrix A satisfies its characteristic equa- Also


tion and hence find A–1, where
é5 1ù é2 1ù é - 3 0 ù
é2 1ù f ( A) = A2 - A - 3 = ê ú -ê ú+ê ú
A=ê ú ë 1 2 û ë 1 - 1û ë 0 - 3û
ë 1 - 1û
é 5 - 2 - 3 1 - 1 + 0 ù é0 0 ù
Solution: The characteristic equation of the given =ê ú=ê ú
ë 1 - 1 + 0 2 + 1 - 3û ë 0 0 û
matrix is x2 - x - 3 = 0 (see Example 8.34). That is
Therefore, A satisfies its characteristic equation and
f ( x) = x2 - x - 3
1
Now A2 - A = 3I Þ A-1 = ( A - I )
3
é2 1ù é 2 1ù é 5 1 ù
A2 = ê ú ê ú=ê ú
ë 1 - 1û ë 1 - 1û ë 1 2 û

A few more examples of this type are discussed in worked-out problems. It is better for the reader to know few more
kinds of matrices as discussed next.

DEF IN IT ION 8 . 33 A square matrix A is called idempotent matrix if A2 = A.

Example 8.37

Show that the matrix A is idempotent é 4 + 2 - 4 - 4 - 6 + 8 - 8 - 8 + 12 ù


é 2 -2 -4ù = êê - 2 - 3 + 4 2 + 9 - 8 4 + 12 - 12 úú
A = êê - 1 3 4 úú êë 2 + 2 - 3 - 2 - 6 + 6 - 4 - 8 + 9 úû
êë 1 - 2 - 3 úû
é 2 - 2 - 4ù
Solution: By definition, a matrix is idempotent if A = A. 2
= êê - 1 3 4 úú = A
Now êë 1 - 2 - 3úû
é 2 -2 -4ù é 2 -2 -4ù
Therefore, A is an idempotent matrix.
A = êê - 1 3
2
4 úú êê - 1 3 4 úú
êë 1 - 2 - 3 úû êë 1 - 2 - 3 úû

DEFIN IT ION 8 . 34 A square matrix A is called nilpotent matrix if there exists a positive integer m such that Am
is a zero matrix. Among such positive integers m, the least positive one is called the index of
the nilpotent matrix.
www.jeeneetbooks.in
412 Chapter 8 Matrices, Determinants and System of Equations

Examples

é0 1ù (2) Every zero square matrix is a nilpotent matrix of


(1) ê ú is a nilpotent matrix of index 2. index 1.
ë0 0 û

DEF IN IT ION 8 . 35 A square matrix A is called involutary if A2 is equal to unit matrix of same order. Note that
a square matrix A is involutary if and only if A-1 = A.

Examples

é0 1ù (2) Every unit matrix is involutary.


(1) ê ú is an involutary matrix.
ë1 0û

DEF IN IT ION 8 . 36 A square matrix A is said to be periodic matrix, if Ak + 1 = A for some positive integer k. If k
is the least positive integer such that Ak + 1 = A, then k is called the period of A.

Example 8.38

Show that A is periodic matrix and find its period. éi 0ù é i 0 ù é -1 0 ù


A2 = ê = = - I2
éi 0ù ë0 - i úû êë0 - i úû êë 0 - 1úû
A=ê (where i = - 1)
ë0 - i úû
Therefore A4 = I2 and A5 = A. Hence A is a periodic matrix
Solution: For the given matrix we have of period 4.

8.3 | Solutions of Linear Equations


In this section, we shall apply the results on matrices and determinants in solving systems of linear equations. In
particular, we derive certain conditions on the coefficients for the system of equations to have a unique solution.

DEF IN IT ION 8 . 37 An equation of the form

a1x 1 + a 2 x 2 + + a n x n = b

where a1 , a2 , … , an and b are real numbers and x1 , x2 , … , xn are unknowns, is called linear
equation in n unknowns. Also n linear equations in n unknowns of the form
a 11 x 1 + a 12 x 2 + + a 1n x n = b1
a 21 x 1 + a 22 x 2 + + a 2 n x n = b2

a n1 x 1 + a n 2 x 2 + + a nn xn = bn

is a called a system of linear equations in n unknowns. The above system of linear equations
can be expressed in the form of a matrix equation as
AX = B
where A the n ´ n matrix and X and B are the n ´ 1 matrices given by
www.jeeneetbooks.in
8.3 Solutions of Linear Equations 413

é a11 a12 a1n ù é x1 ù é b1 ù


êa a22 a2n ú êx ú êb ú
ê 21 ú ê 2ú ê 2ú
A = ê ú , X = ê  ú and B = ê  ú
ê ú ê ú ê ú
ê ú êú ê ú
êëan1 an 2 ú
ann û ê ú
ë xn û êëb n úû

Recall that AX is the product of the square matrix A of order n ´ n and the matrix X of order n ´ 1. Therefore both
AX and B are n ´ 1 matrices. The equation AX = B means that the corresponding entries in the matrices AX and B
are equal; that is, for each 1 £ i £ n,
ai 1 x1 + ai 2 x 2 + + ain xn = bi
A and B are given matrices and we have to find X satisfying AX = B. The matrix A is called the coefficient matrix.

DEF IN IT ION 8 . 38 Let AX = B be a system of n linear equations in n unknowns given by


a11x1 + a12 x2 + + a1n xn = b1
a21x1 + a22 x2 + + a2n xn = b2



an1 x1 + an 2 x2 + + ann xn = bn

If b1 = b2 = = bn = 0, then the system is called a system of homogenous linear equations. If


atleast one bi ¹ 0, then the system is called a non-homogenous system of linear equations.
A solution of the system AX = B is defined to be an n-tuple (a1 , a2 , … , an ) of real numbers
which satisfy each of the above equations; that is,
ai 1a 1 + ai 2a 2 + + ainan = bi for all 1 £ i £ n

DEF IN IT ION 8 . 39 A system AX = B of linear equations is said to be consistent if there exists a solution for the
system; otherwise the system is called inconsistent.

8.3.1 Crammer’s Rule


T H E O R E M 8 .46 Let AX = B be a system of n linear equations in n unknowns. If A is a non-singular matrix, then
(CRAMMER’S the system is consistent and has a unique solution.
RULE)
PROOF Let
a11x1 + a12 x2 + + a1n xn = b1
a21x2 + a22 x2 + + a2 nxn = b2



a0 x1 + an2 x2 + + ann xn = bn

be a system of linear equations. Let


www.jeeneetbooks.in
414 Chapter 8 Matrices, Determinants and System of Equations

é a11 a12 … a1n ù é x1 ù é b1 ù


êa a22 … a2 n ú ê x ú êb ú
A=ê ú , X = ê 2 ú and B = ê 2 ú
21

ê… … … …ú êú êú
ê ú ê ú ê ú
ëan1 an 2 … ann û ë xn û ëbn û

Then AX = B represents the given system of linear equations in n unknowns x1, x2, ¼, xn. Suppose
that A is non-singular; that is, A has a multiplicative inverse A-1. Now,

X = In X = ( A-1 A) X = A-1 ( AX ) = A-1 B

and hence (x1, x2, ¼, xn) is a solution of AX = B. Further,

é A11 A21 A31 … An1 ù é b1 ù


ê A32 … An 2 ú ê b2 ú
æ adj A ö 1 ê A12 A22
úê ú
X = A-1 B = ç B =
è | A| ÷ø | A| ê    …  úê  ú
ê úê ú
ë A1n A2 n A3 n … Ann û ëbn û

where Aij is the ijth cofactor of the matrix A. This implies


n n n
bi Ai 1 bA bA
x1 = å , x2 = å i i 2 , … , xn = å i in
i =1 | A| i =1 | A| i =1 | A|

One can observe that å k = 1 bk Aik is the determinant of the matrix obtained from the matrix A
n

by replacing its kth column with B. If we denote this determinant obtained by replacing the kth
column of A by B with Dk, then
Dk
xk =
| A|

Thus
D1 D D
x1 = , x2 = 2 , … , xn = n
| A| | A| | A|

which shows that the solution X is unique because X = A-1 B always satisfies the equation
AX = B. ■

Example 8.39

Find the solution of the system of equations x + y + 1 6 1


z = 6, x - y + z = 2, 2 x + y - z = 1 using Crammer’s rule.
D2 = 1 2 1 = 1(- 2 - 1) - 6(- 1 - 2) + 1(1 - 4) = 12
Solution: Here the coefficient matrix is given by 2 1 -1

é 1 1 1ù é6 ù éxù 1 1 6
A = êê 1 - 1 1úú and B = êê 2 úú , X = êê y úú D3 = 1 - 1 2 = 1(- 1 - 2) - 1(1 - 4) + 6(1 + 2) = 18
êë 2 1 - 1úû êë 1 úû êë z úû 2 1 1
Now
Therefore by Crammer’s rule
| A | = 1(1 - 1) - 1(- 1 - 2) + 1(1 + 2) = 6 ¹ 0
Hence A is non-singular and the system will have unique D1 6 D 12 D 18
x= = = 1, y = 2 = = 2 and z = 3 = =3
solution. Also | A| 6 | A| 6 | A| 6

6 1 1 is the solution.
D1 = 2 - 1 1 = 6(1 - 1) - 1(- 2 - 1) + 1(2 + 1) = 6
1 1 -1
www.jeeneetbooks.in
8.3 Solutions of Linear Equations 415

8.3.2 Gauss–Jordan Method


Now we are going to discuss another method of solving the equation AX = B by applying elementary row operations on A,
where A is a non-singular matrix and B is a column matrix, and same operations on B. The equation will be reduced to the
form IX = D, where I is the unit matrix of the same order as A and D is a column matrix whose elements are d1 , d2 , … , dn. So
x1 = d1 , x2 = d2 , … , xn = dn is the solution. This method of finding the solution of AX = B (| A | ¹ 0) is called Gauss–Jordan
method. In this method, we use the following theorem which is stated and whose proof is beyond the scope of this book.

T H E O R E M 8 .47 Solution of the equation AX = B will not be altered by applying elementary row operations on
the equation.

Example 8.40

Using Gauss–Jordan method, solve the equations é ù


x + 2z = 2, y + z = 3 and 2 x + y = 1 é1 0 2ù ê2ú
ê ú ê ú æ -1 ö
∼ ê0 1 1 ú X = ê 3 ú ç R3 ∼ R3 ÷
è 5 ø
Solution: The matrix equation is êë0 0 1 úû ê6ú
ê ú
é1 0 2ù é2ù éxù ë5û
ê 0 1 1 ú X = ê 3 ú , where X = ê y ú é 12 ù
ê ú ê ú ê ú ê2 - 5 ú
êë 2 1 0 úû êë 1 úû êë z úû é1 0 0ù ê ú (R ∼ R - 2 R and
∼ êê0 1 0 úú X = ê 3 - ú
6 1 1 3

é1 0 2 ù é2ù ê 5ú R2 ∼ R2 - R3 )
êë0 0 1úû ê ú
∼ ê0 1 1 ú X = êê 3 úú (R3 ∼ R3 - 2R1 )
ê ú
ê 6 ú
êë0 1 - 4 úû êë - 3úû ëê 5 ûú

é1 0 2 ù é2ù Therefore, the solution is


∼ êê0 1 1 úú X = êê 3 úú (R3 ∼ R3 - R2 ) 12 - 2 9 6
êë0 0 - 5úû êë - 6 úû x=2- = , y= and z=
5 5 5 5

8.3.3 Consistent and Inconsistent Systems


Illustration
The discussion till now provides the reader the technique of solving the equation AX = B when A is non-singular
matrix (i.e., | A| ¹ 0).
If | A| = 0, then the system may have or may not have solution.
1. If the system has no solution, then it is called inconsistent system.
2. If it has solution, the system is called consistent system.
Let us consider the system
AX = B (8.14)
where | A| = 0.
Applying series of elementary row operations simultaneously on both sides of Eq. (8.14), suppose at a stage, we
obtain zero row (i.e., all elements of the row are zeros) in the transformed matrix of A and the corresponding element
in the transformed form of B is non-zero, then the system is inconsistent otherwise it is consistent. When the system is
consistent, then we rewrite the equivalent system and express x, y, z in terms of a parameter(s) which shows that the
system has infinite number of solutions. This process will be explained by the following examples.

Example 8.41

Check if the following system of equations is consistent x + y + z = 1, x + 2 y + 4z = 3 and x + 4 y + 10z = 9


or inconsistent.
www.jeeneetbooks.in
416 Chapter 8 Matrices, Determinants and System of Equations

Solution: The matrix equation representing the system is The system is inconsistent because 0x + 0y + 0z = 2. Note
that, here
é1 1 1 ù é 1ù
ê1 2 4 ú X = ê 3 ú 1 1 1
ê ú ê ú = 1(20 - 16) - 1(10 - 4) + 1(4 - 2)
êë1 4 10 úû êë 9 úû | A| = 1 2 4
=4-6+2=0
1 4 10
é 1 1 1ù é 1ù
ê0 1 3ú X = ê 2 ú (R ∼ R - R and R ∼ R - R )
ê ú ê ú 2 2 1 3 3 1 That is, in this case | A| = 0 and the system is inconsistent.
êë0 3 9 úû êë 8 úû

é 1 1 1ù é 1ù
ê0 1 3ú X = ê 2 ú (R ∼ R - 3R )
ê ú ê ú 3 3 2

êë0 0 0 úû êë 2 úû

Example 8.42

Consider the following system of equations. Check for é 1 1 1ù é 1ù


consistency of this system x + y + z = 1, x + 2y + 4z = 2, ê0 1 3ú X = ê 1ú
x + 4 y + 10z = 4. ê ú ê ú
êë0 0 0 úû êë0 úû
Solution: The matrix equation representing the system is
é1 0 -2ù é0 ù
ê0 1 ú X = ê 1 ú (R ∼ R - R )
é1 1 1 ù é 1ù ê 3 ú ê ú 1 1 2
ê1 2 4 ú X = ê 2 ú êë0 0 0 úû êë0 úû
ê ú ê ú
êë1 4 10 úû êë 4 úû Therefore we get
Proceeding in similar fashion as in Example 8. 41 we get x - 2z = 0, y + 3z = 1
This is a system of two equations in three variables. Let
é 1 1 1ù é 1ù
ê 0 1 3ú X = ê 1ú z = k, so that x = 2k, y = 1 - 3k. Therefore x = 2k, y = 1 – 3k
ê ú ê ú and z = k is a solution for all real values of k. Hence the
êë0 3 9 úû êë 3úû system has infinite number of solutions. Here also | A| = 0.

Examples 8. 41 and 8. 42 revealed that AX = B is inconsistent in one case whereas it is consistent in other case. In both
cases, the coefficient matrix is singular.

8.3.4 Homogenous System of Equations


Now let us turn our attention to homogenous system of equations which are given below. If aij (1 £ i, j £ n) are real,
then the system of equations
a11x1 + a12 x2 + + a1n xn = 0
a21x1 + a22 x2 + + a2n xn = 0
   
an1 x1 + an 2 x2 + + ann xn = 0

is called a homogenous system of n equations in n variables x1 , x2 , … , xn. If

é a11 a12 … a1n ù


êa a22 … a2n ú
A=ê ú
21

ê   …  ú
ê ú
a a
ë n1 n 2 … ann û
www.jeeneetbooks.in
8.3 Solutions of Linear Equations 417

then the matrix equation representing the above system of equations is AX = O, where

é x1 ù
êx ú
X=ê ú
2

êú
ê ú
ë xn û

and O is On ´ 1 zero matrix. Note that x1 = 0, x2 = 0, … , xn = 0 is always a solution of the system and this solution is
called trivial solution of the system. Any solution in which at least one xi ¹ 0 is called non-trivial solution or non-zero
solution.

T H E O R E M 8 .48 If A is a non-singular matrix with real entries, then X = O is the only solution of AX = O. If X is a
non-zero solution, then A is a singular matrix.
PROOF Suppose A is non-singular matrix so that A-1 exists. Therefore

AX = O Þ A-1 ( AX ) = O
Þ (A-1 A) X = O
Þ X =O

Now suppose X = X1 is a non-zero solution. If A is non-singular, then A-1(AX1) = O and hence


X1 = O which is a contradiction. Hence A must be singular matrix. ■

In solving AX = O, we employ the same technique as in the case of AX = B which is explained in illustration in
Section 8.3.3.

Example 8.43

Find the solution of the system of equations é 10 ù


x + 3 y - 2z = 0, 2 x - y + 4z = 0 and x - 11y + 14z = 0 ê1 0 7 ú
ê ú
ê0 1 -8 ú
X = O (R1 ∼ R1 - 3R2 )
Solution: The matrix equation representing the given ê 7 ú
system is ê ú
ë0 0 0 û
é1 3 -2ù
ê2 -1 Therefore
ê 4 úú X = O
êë 1 - 11 14 úû x+
10
z = 0 and
8
y- z=0
7 7
é1 3 -2ù
ê0 - 7 If we put z = k, then the solution is
ê 8 úú X = O (R2 ∼ R2 - 2 R1 and R3 ∼ R3 - R1 )
êë0 - 14 16 úû - 10 8
x= k, y = k, z = k
7 7
é1 3 -2ù
ê0 - 7 8 úú X = O (R3 ∼ R3 - 2R2 )
ê
êë0 0 0 úû

é1 3 -2 ù
ê ú
ê0 1 - 8 ú X = O æ R ∼ -1 R ö
çè 2 2÷
ê 7ú 7 ø
ê ú
ë0 0 0û
www.jeeneetbooks.in
418 Chapter 8 Matrices, Determinants and System of Equations

WORKED-OUT PROBLEMS
Single Correct Choice Type Questions
1. Let é1 -aù
=ê ú=I+A
é ab b ù2
ëa 1 û
A=ê 2 ú
ë- a - abû Answer: (A)
where ab ¹ 0. Then 3. If
(A) A2 = A (B) A2 = O (C) A2 = I (D) A3 = A
é3 - 4ù é x ù é 3 ù
Solution: We have ê 1 2 ú ê y ú = ê11ú
ë ûë û ë û
é ab b2 ù é ab b2 ù
A2 = ê 2 úê ú then
ë-a - abû ë - a2 - abû
(A) x = 3, y = 5 (B) x = 4, y = 3
é a2 b2 - a2 b2 ab3 - ab3 ù
=ê 3 2 2ú
(C) x = 4, y = 5 (D) x = 5, y = 3
ë-a b + a b -a b + a b û
3 2 2

Solution: We have
é0 0 ù
=ê ú=O
ë0 0 û é 3 ù é 3 - 4 ù é x ù é 3x - 4 yù
ê11ú = ê 1 2 ú ê y ú = ê x + 2 y ú
Answer: (B) ë û ë ûë û ë û

2. Let
Therefore we get
3 x - 4 y = 3 and x + 2 y = 11
é 0 - tan q / 2 ù
A=ê ú (q ¹ np ) Solving these equations, we have x = 5 and y = 3.
ëtan q / 2 0 û
écos q - sin q ù é1 0ù Answer: (D)
B=ê ú , I = ê0 1ú
ë sin q cos q û ë û 4. If
Then the matrix I + A is equal to
é- 4 1ù
(A) (I - A)B (B) (I - A)2B A=ê ú and f ( x) = x2 - 2 x + 3
ë 3 2û
(C) (I + A)2B (D) (I - A)2
then f (A) is
Solution: Put tan(q / 2) = a so that
é 1 - a2 - 2a ù é 30 4 ù é 30 - 4 ù é 30 - 4 ù
(A) O (B) ê ú (C) ê (D) ê
ê
1 + a2 1 + a2 ú
ú ë - 12 6 û ë - 12 6 úû ë12 6 úû
B=ê
ê 2a 1 - a2 ú
ê ú Solution: We have f ( A) = A2 - 2 A + 3I where
êë 1 + a 1 + a2 úû
2

é1 0ù
Therefore I=ê ú
ë0 1û
é 1 - a2 - 2a ù
ê ú Now
é 1 aù ê 1 + a 1 + a2 ú
2

( I - A)B = ê ú
ë - a 1 û ê 2a 1 - a2 ú é - 4 1 ù é - 4 1 ù é 19 - 2 ù
ê ú A2 = ê úê ú=ê ú
êë 1 + a 1 + a2 úû ë 3 2û ë 3 2û ë-6 7 û
2

é 1 - a2 2a2 - 2a a(1 - a2 ) ù é 8 -2ù


ê + + ú -2A = ê ú
1 + a2 1 + a2 1 + a2 1 + a2 ú ë-6 - 4û

ê - a(1 - a2 ) 2a 2a2 1 - a2 ú
ê + + ú é3 0ù
êë 1 + a2
1 + a2 1+ a 2
1 + a2 úû 3I = ê ú
ë0 3û
www.jeeneetbooks.in
Worked-Out Problems 419

Substituting these in the equation for f(A) we get é 3m 3m 3m ù


ê ú
é 30 - 4 ù = ê 3m 3m 3m ú
f ( A) = ê ê 3m 3m úû
ë - 12 6 úû ë 3m

Answer: (C) Hence by induction,


é 3n - 1 3n - 1 3n - 1 ù
ê ú
5. Let An = ê 3n - 1 3n - 1 3n - 1 ú
ê 3n - 1 3n - 1 3n - 1 úû
é1 1 1ù ë
A = êê1 1 1úú Answer: (B)
êë1 1 1úû
6. If
and n be a positive integer. Then
é 1 3 2ù é 1ù
él n-1
l n-1
l ùn-1
[1 x 1] êê 2 5 1 úú êê 2 úú = O
ê ú êë15 3 2 úû êë x úû
A = êln-1
n
ln-1 ln-1 ú
êln-1 ln-1 l n - 1 úû
ë then value of x is
where l equals (A) -2 or -14 (B) 2 or - 4
(A) 2 (B) 3 (C) 9 (D) 6 (C) 2 or -14 (D) 2 or 14

Solution: For the given matrix A we have Solution: We have

é1 1 1ù é1 1 1ù é 3 3 3ù é 3
2 -1
32 - 1 32 - 1 ù é 1 3 2ù é 1ù é 1 + 6 + 2x ù
ê
A2 = êê1 1 1úú êê1 1 1úú = êê 3 3 3úú = ê 32 - 1 32 - 1 32 - 1 ú
ú [1 x 1] ê 2 5 1 ú ê 2 ú = [1 x 1] êê 2 + 10 + x úú
ê ú ê ú

êë1 1 1úû êë1 1 1úû êë 3 3 3úû êë 32 - 1 32 - 1 32 - 1 úû êë15 3 2 úû êë x úû êë15 + 6 + 2 x úû


é 2x + 7 ù
= [1 x 1] êê x + 12 úú
Also
é 31- 1 31- 1 31- 1 ù êë 2 x + 21úû
ê ú
A = ê 31- 1 31- 1 31- 1 ú
By hypothesis (2 x + 7) + x( x + 12) + (2 x + 21) = 0
ê 31- 1 31- 1 31- 1 úû
ë
(2 x + 7) + ( x2 + 12 x) + 2 x + 21 = 0
Therefore for n = 1, 2 x2 + 16 x + 28 = 0
é 3n - 1 3n - 1 3n - 1 ù ( x + 2)( x + 14) = 0
ê ú x = - 2 or -14
A = ê 3n - 1
n
3n - 1 3n - 1 ú
ê 3n - 1 3n - 1 3n - 1 úû Answer: (A)
ë

is true. Assume that 7. Let

é 3m - 1 3m - 1 3m - 1 ù é1 2 2ù
A = ê 2 1 - 2 úú

ê ú
Am = ê 3m - 1 3m - 1 3m - 1 ú 3
ê 3m - 1 êë x 2 y úû
ë 3m - 1 3m - 1 úû
If AAT = I3, then
Then
(A) x = 2, y = -1 (B) x = 2, y = 1
é 3m - 1 3m - 1 3m - 1 ù é1 1 1ù (C) x = -2, y = -1 (D) x = -2, y = 1
ê ú
A = A × A = ê 3m - 1
m+1 m
3m - 1 3m - 1 ú êê1 1 1úú Solution: For the given matrix A we have
ê 3m - 1 3m - 1 3m - 1 úû êë1 1 1úû
ë
é 1 2 2 ù é1 2 xù

é 3 × 3m - 1 3 × 3m - 1 3 × 3m - 1 ù AAT = 2 1 - 2 úú êê 2 1 2 úú
ê ú 9ê
= ê 3 × 3m - 1 3 × 3m - 1 3 × 3m - 1 ú êë x 2 y úû êë 2 - 2 y úû
ê 3 × 3m - 1 3 × 3m - 1 3 × 3m - 1 úû
ë
www.jeeneetbooks.in
420 Chapter 8 Matrices, Determinants and System of Equations

é 1+ 4 + 4 2+2-4 x + 4 + 2y ù Therefore
= êê 2 + 2 - 4 2 x + 2 - 2 y úú
1
4 + 1+ 4 é0 + 4 y2 + z2 0 + 2 y2 - z2 0 - 2 y2 + z2 ù
9
ëê x + 4 + 2 y 2 x + 2 - 2 y x + 4 + y úû
2 2 ê ú
= ê 0 + 2 y2 - z2 x2 + y2 + z2 x2 - y2 - z2 ú
ê 0 - 2 y2 + z2 x2 - y2 - z2 x2 + y2 + z2 ú
é 9 0 x + 2y + 4 ù ë û

2 x - 2 y + 2 úú é 4y + z 2y - z -2y + z ù
2 2 2 2 2 2
= ê 0 9
9 ê ú
êë x + 2 y + 4 2 x - 2 y + 2 x2 + y2 + 4 úû = ê 2 y2 - z2 x2 + y2 + z2 x2 - y2 - z2 ú
ê -2 y2 + z2 x2 - y2 - z2 x2 + y2 + z2 ú
ë û
Now But it is given that AAT = I, therefore

é 1 0 ( x + 2 y + 4)/ 9 ù é1 0 0ù
ê
AA = ê
T
0 1
ú
(2 x - 2 y + 2)/ 9 ú AA = êê0 1 0 úú
T

ê(9 x + 2 y + 4)/ 9 (2 x - 2 y + 2)/ 9 ( x2 + y2 + 4)/ 9 ú êë0 0 1úû


ë û

é 1 0 0ù Solving we get [taking (2-2)th entry]


I = êê0 1 0 úú
3
x2 + y2 + z2 = 1
êë0 0 1úû Answer: (B)

Solving we get 9. If

x + 2y + 4 = 0 (8.15) éa b ù
A=ê
ë c 1 - a úû
2x - 2y + 2 = 0 (8.16)
is idempotent matrix and bc = 1/4 then the value of a is
x2 + y2 + 4 = 9 (8.17) (A) 1 (B) -1 (C) 1/2 (D) -1/2
From Eqs. (8.15) and (8.16), we get x = -2, y = -1. These Solution: If A is idempotent matrix then A2 = A. This
values also satisfy Eq. (8.17). implies
Answer: (C) éa2 + bc b ù éa b ù
ê 2ú
=ê ú
ë c bc + (1 - a) û ë c 1 - a û
8. If x, y, z are real and
Solving we get
é0 2y z ù
A = êê x y - zúú a2 + bc = a and bc + (1 - a)2 = 1 - a
êë x - y z úû Using bc = 1/4 we get

is such that AAT = I, then 1


a2 + =a
4
(A) x + y + z = 1
(2a - 1)2 = 0
(B) x2 + y2 + z2 = 1
1
(C) x + y + z = xyz a=
2
(D) x2 + y2 + z2 = 2xyz
Answer: (C)
Solution: For the given matrix A we have
10. If
é0 x xù
ê
A = ê2 y
T
y - y úú é1 0ù é1 0ù
ê 2 1ú = ê0 1ú + B
êë z - z zúû ë û ë û
then
é0 2 y z ù é 0 x xù
(A) B2 = B (B) B2 = I
AAT = êê x y - zúú êê 2 y y - y úú
êë x - y z úû êë z - z z úû é1 0ù
(C) B2 = O (D) B2 = ê ú
ë2 0û
www.jeeneetbooks.in
Worked-Out Problems 421

Solution: Let 12. If a non-zero square matrix of order 3 ´ 3 commutes


with every square matrix of order 3 ´ 3, then the
éa b ù matrix is necessarily
B=ê ú
ëc d û (A) a scalar matrix (B) a unit matrix
so that (C) an idempotent matrix (D) a nilpotent matrix

é 1 0 ù é 1 0 ù éa bù éa + 1 b ù Solution: Let
ê 2 1ú = ê0 1ú + ê c d ú = ê c d + 1úû
ë û ë û ë û ë é a11 a12 a13 ù
Solving we get A = êêa21 a22 a23 úú
êëa31 a32 a33 ûú
a+1=1
b=0 and suppose A commutes with every matrix of 3 ´ 3
c=2 order. Choose
d+1=1 éb1 0 0ù
So B = êê 0 b2 0 úú
a = 0, b = 0, c = 2, d = 0 êë 0 0 b3 úû
Therefore where b1, b2 and b3 are distinct. The (i, j)th element of
AB = bi aij, whereas (i, j)th element of BA = bi aij. Now
é0 0 ù
B=ê ú and B = O
2

ë2 0û AB = BA Þ bj aij = bi aij
Answer: (C) Þ aij = 0 when i ¹ j (∵ bi ¹ bj for i ¹ j )
Note: One can observe that Therefore
é 1 0ù é 1 0ù é0 0ù éa11 0 0ù
B=ê ú-ê ú=ê ú
ë 2 1û ë0 1û ë 2 0 û A = êê 0 a22 0 úú
êë 0 0 a33 ûú
11. If
Again choose
é3 - 2ù é3 3ù
ê3 ú éx xù ê ú
0ú ê ú = ê 3x 3x ú é d11 d12 d13 ù
ê
D = êêd21 d23 úú
ë y y û ê
êë 2 4 úû ë 10 10 úû d22
êëd31 d32 d33 úû
then the integral part of x + y is
(A) 3 (B) 2 (C) 4 (D) 1 where dij ¹ 0 for 1 £ i, j £ 3. Again
Solution: We have AD = DA Þ aii dij = ajj dij Þ aii = ajj (∵ dij ¹ 0)
é 3x - 2 y 3x - 2 y ù é 3 3 ù Therefore
ê 3x 3x úú = êê 3 x 3 x úú
ê a11 = a22 = a33
êë 2 x + 4 y 2 x + 4 y úû êë 10 10 úû
and hence A is a scalar matrix.
Solving we get Answer: (A)
3x - 2y = 3 (8.18) Note: For more general case, the reader is advised to see
2x + 4y = 10 (8.19) Theorem 8.7.

Solving Eqs. (8.18) and (8.19), we get x = 2, y = 3/2. 13. If


Therefore
éa 0ù é1 0ù
7 1 A=ê ú and B = ê ú
x+ y= =3 ë1 1û ë 5 1û
2 2
Hence, the integral part of x + y = 3. then the value of a for which A2 = B is
Answer: (A) (A) 1 (B) -1 (C) no real value (D) 4
www.jeeneetbooks.in
422 Chapter 8 Matrices, Determinants and System of Equations

Solution: We have from the hypothesis that 16. If P is a 2 ´ 2 matrix satisfying the relation

é1 0ù éa 0 ù éa 0ù é a2 0ù é2 1ù é3 2 ù é2 4ù
ê 5 1ú = B = A = ê 1
2
=ê ú ê 3 2 ú P ê 5 - 3 ú = ê 3 - 1ú
ë û ë 1úû êë 1 ú
1û ëa + 1 1û ë û ë û ë û
Therefore a 2 = 1 and a + 1 = 5 which are inconsistent. then P is equal to
Hence there is no real value of a.
Answer: (C) 1 é - 48 25 ù 1 é 48 - 25ù
(A) (B)
19 ë 70 - 42 úû
ê 19 êë - 70 42 úû
14. If A is a square matrix such that A3 = O, then I + A + A2
(I is the corresponding unit matrix) is - 1 é 38 - 25ù - 1 é 48 25 ù
(C) (D)
19 êë - 75 42 úû 19 êë 70 42 úû
(A) I + A (B) (I + A)-1
(C) I - A (D) (I - A)-1 Solution: Let
Solution: We have
é2 1ù é3 2 ù
A=ê ú and B = ê ú
(I - A) (I + A + A ) = I - A = I
2 3
ë3 2û ë 5 - 3û
Therefore
so that
I + A + A2 = (I - A)-1
Answer: (D) é 2 - 1ù 1 é3 2 ù
A-1 = ê ú and B-1 =
ë- 3 2û 19 êë 5 - 3úû
15. If the product of the matrices
Now by hypothesis
é 1 1ù é 1 2 ù é 1 3ù é 1 n ù é 1 378 ù
ê0 1ú ê0 1 ú ê0 1ú ê0 1 ú = ê0 1 ú é2 4ù -1 é
2 4 ù -1
ë ûë ûë û ë û ë û APB = ê ú Þ P = A ê 3 - 1ú B
ë 3 - 1 û ë û
then n is equal to -
1 é 2 1 ùé 2 4 ùé 3 2ù
= ê ú
(A) 27 (B) 26 (C) 376 (D) 378 19 ë - 3 2 û ë 3 - 1û ë 5 - 3úû
ê ú ê

Solution: We have 1 é 2 - 1ù é 26 - 8 ù
=
19 êë - 3 2 úû êë 4 9 úû
é 1 1ù é 1 2 ù é 1 3ù é 1 1 + 2 ù
ê0 1ú ê0 1 ú = ê0 1ú = ê0 1 úû 1 é 48 - 25ù
ë ûë û ë û ë =
19 êë - 70 42 úû
Again
Answer: (B)
é 1 3ù é 1 3ù é 1 6 ù é 1 1 + 2 + 3ù
ê0 1ú ê0 1ú = ê 0 1ú = ê 0 1 ú 17. If
ë ûë û ë û ë û
é2 x 0 ù é 1 0ù
By induction, A=ê ú and A-1 = ê ú
ë x xû ë-1 2 û
é n
ù
LHS = ê
1 å k ú = é1 378 ù then x is equal to
ê k =1 ú ê0 1 úû
êë0 1 úû ë (A) 2 (B) 1/2 (C) 1 (D) 3
-1
Solution: We know that AA = I. Hence
Therefore
é2 x 0 ù é 1 0 ù é1 0ù
n(n + 1)
= 378 or n(n + 1) = 27 ´ 28 ê x x ú ê - 1 2 ú = ê0 1ú
2 ë ûë û ë û

Hence n = 27. é2 x 0 ù é1 0ù
ê 0 2 x ú = ê0 1ú
Answer: (A) ë û ë û

This gives x = 1/2.


Answer: (B)
www.jeeneetbooks.in
Worked-Out Problems 423

18. Let w ¹ 1 be a cube root of unity and Now

éw 0 ù ( A + B)2 = A2 + AB + BA + B2 = A2 + O + B2 = A2 + B2
A=ê ú
ë 0 wû Answer: (B)

Then A2010 is equal to 21. If A and B are square matrices of same order such
(A) A (B) A2 (C) A3 (D) 3A that A + B = AB, then
Solution: For the given matrix, we have (A) A + B = -BA (B) A - B = BA
(C) AB = BA (D) A - B = 0
éw 0 ù éw 0 ù éw2 0ù
A2 = ê úê ú=ê ú Solution: We have
ë 0 wû ë 0 wû ë 0 w2 û
A + B = AB Þ AB - A - B + I = I
éw2 0 ù éw 0 ù
A =ê
3
2úê ú Þ ( I - A)( I - B) = I
ë 0 w û ë 0 wû
éw3 0 ù Þ I - A is invertible and its inverse is I - B
=ê 3ú
ë0 w û Therefore
é1 0ù ( I - B)( I - A) = I
=ê ú
ë0 1û I - B - A + BA = I
A2010 = ( A3 )670 = I 670 = I = A3 A + B = BA
Answer: (C) AB = A + B = BA
Answer: (C)
19. The number of idempotent diagonal matrices of
3 ´ 3 order is 22. Let
(A) 8 (B) 2 (C) 6 (D) infinite
é 2 3ù é -1 2 ù é 2 - 1ù
Solution: Let A = ê - 1 4 ú , B = ê 3 1 ú and C = êê 2
ê ú ê ú 3úú
éd1 0 0ù êë 1 0 úû êë 5 4 úû êë - 4 1úû
D = êê 0 d2 0 úú and D2 = D If
êë 0 0 d3 ûú
é 7 -1 ù
Now ê
A - B + xC = ê 0 9 úú
êë - 12 - 2 úû
éd12 0 0 ù éd1 0 0ù
ê ú
ê0 d2
2 0 ú = êê 0 d2 0 úú then the value of x is
ê0 0 d32 úû ëê 0 0 d3 ûú (A) -2 (C) -3
ë (B) 3 (D) 2
Solution: By hypothesis
Solving we get
d12 = d1 ; d22 = d2 ; d32 = d3 é 7 -1ù é 3 + 2x 1- x ù
ê 0 9 ú = A - B + xC = ê - 4 + 2 x 3 + 3x úú
ú ê
d1 = 0, 1; d2 = 0, 1; d3 = 0, 1 ê
êë - 12 - 2 úû êë - 4 - 4 x - 4 + x úû
Answer: (A)
Therefore
20. Let A be a non-singular square matrix. If B is a
square matrix such that B = -A-1BA, then the matrix 3 + 2 x = 7; 1 - x = - 1; - 4 + 2 x = 0
(A + B)2 is equal to 3 + 3 x = 9; - 4 - 4 x = - 12; - 4 + x = - 2
(A) A + B (B) A2 + B2 (C) O (D) I
Solving any one gives x = 2.
Solution: We have Answer: (D)
-1
AB = -AA BA = -BA
AB + BA = O
www.jeeneetbooks.in
424 Chapter 8 Matrices, Determinants and System of Equations

23. If l, m and n are positive real numbers and the 24. Let A, B and C be square matrices of order 3 ´ 3. If
matrix A is invertible and (A - B)C = BA-1, then
(A) C(A - B) = A-1B (B) C(A - B) = BA-1
é0 2m nù -1
ê (C) (A - B)C = A B
m - n úú
(D) all the above
A = êl
êë l - m n úû Solution: We have

is such that AAT = I (unit matrix), then the ordered ( A - B)C = BA-1 Þ AC - BC = BA-1
triple (l, m, n) may be Þ AC - BC - BA-1 + AA-1 = I (unit
matrix)
æ 1 1 1 ö æ 1 1 1 ö
(A) ç , , (B) ç , , Þ ( A - B)C + ( A - B) A-1 = I
è 3 6 2 ÷ø è 6 3 2 ÷ø
Þ ( A - B)(C + A-1 ) = I
æ 1 1 1 ö æ 1 1 1 ö
(C) ç , , (D) ç , ,
è 2 3 6 ÷ø è 2 6 3 ÷ø Therefore, C + A-1 is the inverse of A - B. This implies

Solution: We have for the given matrix that (C + A-1 )( A - B) = I


C ( A - B) = I - A-1 A + A-1 B
é0 2m n ù é 0 l l ù
C ( A - B) = A-1 B
AAT = êê l m - n úú êê 2 m m - m úú
êë l - m n úû êë n - n n úû Answer: (A)

é 4 m2 + n2 2 m2 - n2 - 2 m2 + n2 ù 25. If
ê ú
= ê 2 m2 - n2 l +m +n
2 2 2
l2 - m2 - n2 ú éa 2 ù
ê A=ê ú and | A | = 125
3

ë -2 m + n
2 2
l2 - m2 - n2 l2 + m2 + n2 úû ë2 aû

But AAT = I, that is then a is equal to


(A) ±3 (B) ±2 (C) ±5 (D) 0
é 4 m2 + n2 2 m2 - n2 - 2 n2 + n2 ù é 1 0 0 ù
ê ú
ê 2m - n
2 2
l + m2 + n2
2
l - m2 - n2 ú = êê0 1 0 úú
2 Solution: By hypothesis
ê - 2 m2 + n2 l2 - m2 - n2 l2 + m2 + n2 úû êë0 0 1úû 125 = | A3 | = | A|3 = (a 2 - 4)3
ë

Solving this we get Hence a 2 - 4 = 5 Þ a = ±3.


Answer: (A)
4m2 + n2 = 1 (8.18)
2m2 - n2 = 0 (8.19) 26. If

-2m + n = 0
2 2
(8.20) é3 - 4ù
A=ê ú
l +m +n =1
2 2 2
(8.21) ë1 -1û

l2 - m2 - n2 = 0 (8.22) then | A2003 | - 4 | A2002 | =


Adding Eqs. (8.18) and (8.19) we get (A) -3 (B) 0 (C) 9 (D) -9

±1 Solution: We have that | A| = - 3 + 4 = 1. Therefore


6 m2 = 1 Þ m =
6 | A2003 | - 4 | A2002 | = | A|2003 - 4 | A|2002 = 1 - 4 = - 3
Adding Eqs. (8.21) and (8.22) we get Answer: (A)
±1 27. Let A be 3 ´ 3 matrix such that A = aA, where a ¹ 1.
3
2 l2 = 1 Þ l =
2 Then, the matrix A = I is
Equation (8.18) - Eq. (8.19) + Eq. (8.20) gives (A) non-singular
(B) idempotent
±1
3n = 1 Þ n =
2
(C) nilpotent matrix
3
(D) symmetric matrix
Answer: (D)
www.jeeneetbooks.in
Worked-Out Problems 425

Solution: Let B = A + I. Then Solution: It can be seen that P = P


T -1

A3 = ( B - I )3 Q = PAPT = PAP-1 Þ P-1Q = AP-1


a A = ( B - I ) = B - 3B + 3B - I
3 3 2
Now
B - 3B + 3B - I = a A = a ( B - I )
3 2

X = PTQ2010P = P -1Q2010P
B3 - 3B2 + 3B - aB = (1 - a )I
= ( P -1Q)Q 2009P = A( P -1Q 2009P )
(B2 - 3B + (3 - a )I )B = (1 - a )I
= A( P -1Q)Q2008 × P = A2 P -1Q2008P
This gives
Finally
det [ B2 - 3B + (3 - a )I ]det B = (1 - a )3 ¹ 0
X = A2010 P-1 P = A2010 (8.23)
Hence det B ¹ 0. This implies B = A + I is non-singular.
Now
Answer: (A) 1ù é 1 1ù é 1 2ù
é1
A2 = ê =
28. A and B are different square matrices of same order ë0 1úû êë0 1úû êë0 1 úû
such that A3 = B3 and A2B = B2A. Then é1 1ù é 1 2 ù é 1 3ù
A3 = ê =
(A) A2 + B2 is singular matrix ë0 1úû êë0 1 úû êë0 1úû
(B) A2 + B2 is non-singular
By induction we can see that
(C) A2 + B2 is idempotent
(D) A2 - B2 is symmetric é1 nù
An = ê ú
Solution: We know that ë0 1 û
Therefore
(A + B )(A - B) = A - A B + B A - B = O
2 2 3 2 2 3
é 1 2010 ù
X = A2010 = ê
If A + B is non-singular, then 1 úû
2 2
ë0
(A2 + B2)-1(A2 + B2)(A - B) = 0 Answer: (D)
Therefore, A - B = O or A = B, which is a contradiction.
Hence A2 + B2 must be singular matrix. 30. The number of real roots of the equation
Answer: (A) a a x
29. If
b x b =0
x x x
é 3 1 ù
ê ú é 1 1ù where a and b are distinct non-zero real numbers, is
2 2 ú
P=ê ,A=ê ú and Q = PAP
T

ê -1 3ú ë0 1û (A) 2 (B) 3 (C) 1 (D) 0


ê ú
ë 2 2 û Solution: Clearly x = 0 is a root. When x = a, first and
third rows are identical and when x = b, the second and
and X = P Q T 2010
P, then X is equal to
third rows are identical. Therefore x = 0, a, b are the
é 4 + 2010 3 8015 ù roots.
(A) ê ú Answer: (B)
êë 2010 4 - 2010 5 úû
31. If n ³ 3 is even and
é 2010 2 - 3ù
(B) ê ú (n- 2) (n- 2) (n- 2)
êë 2 + 3 2010 úû Cr - 2 Cr - 1 Cr
Dr = -3 1 1
é 2 + 3 1ù
(C) ê ú 2 -1 0
êë 2 - 3 1úû
n
then å (- 2)r Dr is
é 1 2010 ù r=2
(D) ê
ë0 1 úû (A) 2n - 1 (B) 2n + 1 (C) 2n (D) 3n
www.jeeneetbooks.in
426 Chapter 8 Matrices, Determinants and System of Equations

Solution: We have then m + n value is


Dr = ( n- 2 )Cr - 2 + 2 ( n- 2 )Cr -1 + (n- 2)Cr (A) 4 (B) 6 (C) 8 (D) 7

= [( n- 2 ) Cr - 2 + ( n- 2 )Cr -1 ] + [( n- 2 ) Cr -1 + ( n- 2 )Cr ] Solution: Let D be the given determinant. Taking a, b, c


common from C1, C2 and C3, respectively, we get
= ( n-1)Cr -1 + ( n-1)Cr
= nCr a c a+c
D = abc a + b b a
Therefore we get
b b+c c
n n

å (- 2) D = å (- 2)
r=2
r
r
r=2
r n
Cr
Now the column operation C3 - (C1 + C2 ) gives
= C2 × 2 - C3 × 2 + C4 × 2 - (- 2) Cn
n 2 n 3 n 4 n
a c 0
= (1 - 2)n - 1 + n C1 × 2 D= a+b b - 2b (abc)
= (- 1)n - 1 + 2 n b b + c - 2b
= 2n (∵ n is even) a c 0
Answer: (C) = (abc)(- 2b) a + b b 1
b b+c 1
32. Let
= - 2b (abc)[[a(b - b - c) - c(a + b - b)]
2 xy x 2
y 2
= - 2b (abc)(- 2ac)
D= x 2
y 2
2 xy = 4 a2 b2 c2
y2 2 xy x2 Answer: (B)

Then D is equal to 34. If


(A) (x2 + y2)3 (B) (x3 + y3)2
a-b-c 2a 2a
(C) -(x2 + y2)3 (D) -(x3 + y3)2
D= 2b b-c-a 2b
Solution: Adding R2 and R3 to R1 and taking (x + y)2 2c 2c c-a-b
common from R1, we get
then D is equal to
1 1 1
(A) (a + b + c)2
D = ( x + y) x2
2
y 2
2 xy
(B) (a + b + c)(a - b)(b - c)(c - a)
y2 2 xy x2
(C) (a2 + b2 + c2 )(ab + bc + ca)
1 0 0 (D) (a + b + c)3
(by C2 - C1
= ( x + y) x2 2
y - x2 2
2 xy - x2
and C3 - C1 ) Solution: By the operations C1 - C2 and C2 - C3 and
y2 2 xy - y2 x2 - y2 taking a + b + c common from C1 and C2 we get
= ( x + y)2 [-( x2 - y2 )2 - xy(2 y - x)(2 x - y)] -1 0 2a
= - ( x + y) [( x - y ) + 4 x y - 2 xy( x + y ) + x y ]
2 2 2 2 2 2 2 2 2 2
D = (a + b + c ) 2
1 -1 2b = (a + b + c)3
= - ( x + y) [( x + y ) - 2 xy( x + y ) + x y ]
2 2 2 2 2 2 2 2 0 1 c-a-b

= - ( x + y)2 [( x2 + y2 ) - xy]2 Answer: (D)

= - ( x3 + y3 )2 é1 0 0ù
Answer: (D) 35. Let
ê
A = ê0 1 1 úú
êë0 - 2 4 úû
33. If
If
a2 bc ac + c2
a2 + ab b2 ac = m an bn cn A-1 =
1 2
( A + xA + yI )
ab b + bc
2
c2 6
www.jeeneetbooks.in
Worked-Out Problems 427

where x, y are scalars and I is 3 ´ 3 unit matrix, then = (1 - x)[ x2 - 5 x + 6]


x, y are, respectively,
= - x3 + 6 x2 - 11x + 6
(A) -11, 6 (B) -6, 11
(C) 6, 11 (D) -6, -11 By Caley–Hamilton theorem f (A) = O. Hence
Solution: We have det A = 4 + 2 = 6. Identify A with - A3 + 6 A2 - 11A + 6 = 0
1 2
é a1 b1 c1 ù ( A - 6 A + 11I ) A = I
êa b2 c2 úú
6
ê 2 1
êë a3 b3 c3 ûú A-1 = ( A2 - 6 A + 11I )
6
and represent the cofactors of ai, bi, ci, respectively, with x = - 6, y = 11
Ai, Bi and Ci. Therefore
36. The parameter on which the determinant of the foll-
A1 = 4 + 2 = 6, B1 = - (0 - 0) = 0, C1 = 0 owing matrix does not depend is
A2 = - (0 - 0) = 0, B2 = 4, C2 = - (- 2 - 0) = 2
é 1 a a2 ù
A3 = 0, B3 = - 1, C3 = 1 ê ú
A = êcos( p - d) x cos px cos( p + d) x ú
Therefore ê sin( p - d) x sin px sin( p + d)x ú
ë û
é6 0 0 ù (A) a (B) p (C) d (D) x
(adj A) = ê0 4 - 1úú
-1 1 1ê
A =
det A 6 Solution: Add C3 - (2 cos dx)C2 to C1. Using
êë0 2 1úû
cos( A + B) + cos( A - B) = 2 cos A cos B
Also
and sin( A + B) + sin( A - B) = 2 sin A cos B
é1 0 0ù é1 0 0ù é1 0 0ù we have
ê
A = ê0
2 ú
1 1ú ê0 ê ú ê
1 1ú = ê0 - 1 5 úú
1 + a2 - 2a cos dx a a2
êë0 - 2 4 úû êë0 - 2 4 úû êë0 - 10 14 úû
det A = 0 cos px cos( p + d) x
So by hypothesis 0 sin px sin( p + d) x

é6 0 0 ù = (1 + a2 - 2a cos dx) [sin( p + d) x cos px



0 4 - 1úú = A-1 = ( A2 + xA + yI )
1
6 ê 6 - cos( p + d) x sin px]
êë0 2 1úû
= (1 + a2 - 2a cos dx)sin( p + d - p) x
é1 + x + y 0 0 ù
= êê ú
1 = (1 + a2 - 2a cos dx)sin dx
0 -1 + x + y 5+ x ú
6
êë 0 - 10 - 2 x 14 + 4 x + y úû which does not contain the parameter p.
Comparing the two sides we get Answer: (B)

x + y = 5; 5 + x = - 1; - 10 - 2 x = 2; 14 + 4 x + y = 1 37. Let

From the first two equations, we get x = -6, y = 11, which 2ax 2ax - 1 2ax + b + 1
also satisfy the other two equations.
f ( x) = b b+1 -1
Answer: (B)
2(ax + b) 2(ax + b) + 1 2ax + b
Second Method (Easy Method): Consider the character-
istic polynomial of the matrix A, which is where a and b are real constants and a ¹ 0, then f -1(x)
equals
1- x 0 0 x+b x-b x-b 2x + b
(A) (B) (C) (D)
f ( x) = 0 1- x 1 2a a 2a a
0 -2 4 - x
Solution: Substracting R1 + 2R2 from R3 (i.e., R3 ® R3 -
= (1 - x)[(1 - x)(4 - x) + 2] R1 - 2R2) we have
www.jeeneetbooks.in
428 Chapter 8 Matrices, Determinants and System of Equations

2ax 2ax - 1 2ax + b + 1 has infinitely many solutions, is (are)


f ( x) = b b+1 -1 (A) 0 (B) 1 (C) 2 (D) ¥
0 0 1 Solution: For a system of non-homogenous equations
to have infinitely many solution, the determinant of the
Simplifying we get coefficient matrix is necessarily be zero. That is
f ( x) = 2ax(b + 1) - b(2ax - 1) = 2ax + b (k + 1)(k + 3) - 8k = 0
Therefore k2 - 4k + 3 = 0
x-b (k - 1)(k - 3) = 0
f -1 ( x) = (∵ a ¹ 0)
2a
(1) When k = 1, the system reduces to one equation
Answer: (C) which is x + 4y = 2 and has infinitely many solutions.
(2) When x = 3, the system will be x + 2y = 3 and x +
38. If 2y = 3/8 and hence is inconsistent.
1 x x+1 Answer: (B)
f ( x) = 2x x( x - 1) x( x + 1)
41. Consider
3 x( x - 1) x( x - 1)( x - 2) ( x - 1) x( x + 1)
2 x - y + 2z = 2, x - 2 y + z = - 4, x + y + lz = 4
Then f (2010) is equal to
Then the value of l such that the given system has
(A) 1 (B) 2010 (C) 2009 (D) 0 no solution is:
Solution: Using the column operation C3 - (C1 + C2 ) (A) 3 (B) 1 (C) 0 (D) -3
we get
Solution: The given system of equations is
1 x 0
é2 -1 2 ù é x ù é 2 ù
f ( x) = 2x x( x - 1) 0 ê 1 - 2 1 ú ê yú = ê- 4ú
3 x( x - 1) x( x - 1)( x - 2) 0 ê úê ú ê ú
êë 1 1 l úû êë z úû êë 4 úû
Therefore f (x) = 0 for all real x. Hence
Using row transformations R1 - 2 R2 and R3 - R2 we get
f(2010) = 0
Answer: (D) é0 3 0 ù é x ù é 10 ù
ê1 -2 1 úú êê y úú = êê - 4 úú
ê
39. The system of equations x - ky - z = 0, kx - y - z = 0, êë0 3 l - 1úû êë z úû êë 8 úû
x + y - z = 0 has a non-zero solution. Then possible
values of k are é0 3 0 ù é x ù é 10 ù
(A) -1, 2 (B) 1, 2 (C) 0, 1 (D) -1, 1 ê
Again using R3 - R1 we get ê 1 - 2 1 úú êê y úú = êê - 4 úú
Solution: If A is a square matrix and X is a column êë0 0 l - 1úû êë z úû êë - 2 úû
matrix, then the matrix equation AX = 0 has non-zero
solution if det A is equal to zero. Therefore If l = 1, then the system is equivalent to the system

1 - k -1 3 x = 10, x - 2 y + z = - 4 and 0 x + 0 y + 0z = - 2
k -1 -1 = 0 which is impossible. Therefore when l = 1, the system
1 1 -1 has no solution.
1(1 + 1) + k(- k + 1) - 1(k + 1) = 0 Answer: (B)

- k2 + 1 = 0 Þ k = ± 1 42. The number of values of l for which the system of


Answer: (D) equation
3 x - y + 3z = 3, x + 2 y - 3z = - 2, 6 x + 5 y + lz = - 3
40. The number of values of k for which the system of
equations has unique solution is
(k + 1) x + 8 y = 4k (A) 2 (B) 4 (C) 8 (D) infinite
kx + (k + 3) y = 3k - 1
www.jeeneetbooks.in
Worked-Out Problems 429

Solution: System of non-homogenous equations AX = n n n


B (A is a square matrix) has unique solution if and only
= n(n + 1) n2 + n + 1 n2 + n
if A is non-singular. Here
n2 n2 n2 + n + 1
é 3 -1 3 ù
n 0 0
A = êê 1 2 - 3úú
= n(n + 1) 1 0 = n(n + 1) = 56 = 7 ´ 8
êë6 5 l úû
n 2
0 n+1
Therefore the determinant is Answer: (D)
det A = 3(2 l + 15) + 1(l + 18) + 3(5 - 12) = 7 l + 42
44. If x, y, z are positive and none of them is 1, then the
Now A is non-singular if det A ¹ 0, that is l ¹ -6. value of the following determinant is
Answer: (D)
1 logx y logx z
43. If logy x 1 logy z is
1 n n logz x logz y 1
Dk = 2k n +n+1
2
n +n
2
(A) 1 (B) 0 (C) 2 (D) -2
2k - 1 n2
n +n+1
2
Solution: Let D be the given determinant. Then
and å k = 1 Dk = 56, then n is equal to
n
log y log z
1
(A) 4 (B) 6 (C) 8 (D) 7 log x log x
log x log z
Solution: We have D= 1
log y log y
å
1
n n log x log y
1
n log z log z
å 2k
n
n +n+1
2
n +n
2
åD k =
k =1 log x log y log z
k =1 1
n = log x log y log z = 0
å (2k - 1)
k =1
n2 n2 + n + 1 (log x)(log y)(log z)
log x log y log z
(since all the three rows are the same)
Answer: (B)

Multiple Correct Choice Type Questions


1. If é 0 - i ù é 0 - i ù é - i2 0 ù é1 0ù
B2 = ê úê ú=ê ú=ê ú
é0 1ù é0 - i ù éi 0 ù ëi 0 ûëi 0 û ë 0 - i2 û ë0 1û
A=ê ú , B=ê ú and C = ê ú
ë1 0û ëi 0 û ë0 - i û
é i 0 ù é i 0 ù éi2 0 ù é-1 0 ù
C2 = ê úê ú=ê ú=ê ú
ë0 -ii û ë0 - i û ë 0 i2 û ë 0 - 1û
where i2 = -1, then
é1 0ù é1 0ù é0 1ù é0 - i ù é i 0 ù
(A) A2 = ê ú (B) B2 = ê ú AB = ê úê ú=ê ú
ë0 1û ë0 1û ë 1 0 û ë i 0 û ë0 - i û
é-1 0 ù é0 - i ù é0 1ù é - i 0 ù
(C) C 2 = ê ú (D) AB + BA = O BA = ê
ë 0 - 1û úê ú=ê ú = -AB
ë i 0 û ë1 0û ë 0 i û
Solution: We have Therefore, AB + BA = 0.
é0 1ù é0 1ù é 1 0 ù Answers: (A), (B), (C), (D)
A =ê
2
úê ú=ê ú
ë 1 0 û ë 1 0 û ë0 1û
www.jeeneetbooks.in
430 Chapter 8 Matrices, Determinants and System of Equations

2. Let This implies A - I, B - I, C - I are invertible matrices and


x 3 7 the inverse of C - I is -(A - I) (B -I). This gives
f ( x) = 2 x 2 (C - I) (A - I )(B - I) = -I
7 6 x CAB - (CA + AB + CB) + A + B + C = 0
If x = -9 is a root of f (x) = 0, then the other roots are CAB - (CA + AB + CB) + AB + BC + CA= 0
(A) 2 (B) 3 (C) 7 (D) 6 CAB = CB - BC
Solution: Adding R2 and R3 to R1 we get Therefore (i) Þ (iii).
Similarly, by permuting the letters A, B, C, we can
x+9 x+9 x+9
show that (i), (ii) and (iii) are equivalent statements.
f ( x) = 2 x 2
Answers: (A), (B), (C), (D)
7 6 x
1 1 1 4. If x is real and
= ( x + 9) 2 x 2 x2 + x 2 x - 1 x + 3
7 6 x D( x) = 3 x + 1 2 + x2 x3 - 3
1 0 0 x - 3 x2 + 4 2x
= ( x + 9) 2 x - 2 0
= a0 x7 + a1 x6 + a2 x5 + + a6 x + a7
7 -1 x - 7
then
= ( x + 9)( x - 2)( x - 7)
6

Therefore the other roots are 2 and 7. (A) a7 = 21 (B) åa


k =0
k = 111
Answers: (A), (C)
(C) D (- 1) = - 32 (D) D (1) = 121
3. Let A, B, C be square matrices of same order and I
Solution: We have
the unit matrix of the same order such that A + B +
C = AB + BC + CA. Consider the following three 0 -1 3
statements.
a7 = D(0) = 1 2 - 3
(i) ABC = AC - CA
-3 4 0
(ii) BCA = BA - AB
= 1(0 - 9) + 3(4 + 6)
(iii) CAB = CB - BC
= - 9 + 30 = 21
Then
(A) (i) and (ii) equivalent Therefore (A) is true. Again
(B) (ii) and (iii) are equivalent 2 1 4
7
(C) (iii) and (i) are equivalent å
k =0
ak = D(1) = 4 3 - 2
(D) all the three statements are equivalent -2 5 2
Solution: Assume (i). That is = 2(6 + 10) - 1(8 - 4) + 4(20 + 6) = 32 - 4 + 104 = 132
ABC = AC - CA (8.24) Therefore
Now 6

åa k = 132 - a7 = 132 - 21 = 111


ABC + A + B + C = ( AC - CA) + AB + BC + CA k =0

= AC + AB + BC Therefore (B) is true. Now


Therefore 0 -3 2
(A - I) (B - I)(C - I) = ABC - (AC + AB + BC) D(- 1) = - 2 3 - 4 = 3(4 - 16) + 2(- 10 + 12)
+ A + B+ C - I - 4 5 -2
= ABC - (AC + AB + BC) = - 36 + 4 = - 32
+ AB + BC + CA - I
Therefore (C) is true.
= ABC + CA - AC - I
Answers: (A), (B), (C)
= -I [by Eq. (8.24)]
www.jeeneetbooks.in
Worked-Out Problems 431

5. Let a, b, c be real numbers and Solution: It is known that D1 = (a - b)(b - c) (c - a).


Now
1 1 1 1 1 1
D1 = a b c , D2 = b + c c+a a+b a + b + c b + c a2
a2 b2 c2 b2 + c2 c2 + a2 a2 + b2 D2 = a + b + c c + a b2 (by adding C2 to C1 )
a + b + c a + b c2
Then
a + b + c - a a2
(A) D1 + D2 = 0
= a + b + c - b b2 (by C2 - C1 )
(B) D1 = D2
a + b + c - c c2
(C) D2 = (ab + bc + ca)D1
(D) D1 = D2 = (b - c)(c - a)(a - b) 1 a a2
Solution: We have = - (a + b + c) 1 b b2 = - (a + b + c)D1
1 c c2
1 0 0
1 0 0
D1 = a b-a c-a
D3 = a b-a c-a (C2 - C1 and C3 - C1 )
a2 b2 - a2 c2 - a2
a3
b3 - a3 c3 - a3
= (b - a)(c2 - a2 ) - (c - a)(b2 - a2 )
= (b - a)(c3 - a3 ) - (c - a)(b3 - a3 )
= (a - b)(cc - a)[-(c + a) + (b + a)] = (b - a)(c - a)[(c2 + ca + a2 ) - (b2 + ab + a2 )]
= (a - b)(b - c)(c - a) = (b - a)(c - a)[(c - b)(c + b) + a(c - b)]
= (b - a)(c - a)(c - b)(a + b + c)
Also
= (a - b) (b - c)(c - a) (a + b + c)
1 1 1 = - D2
D2 = b + c c+a a+b
b2 + c2 c2 + a2 a2 + b2 This gives D3 = -D2 = (a + b + c)D1. Therefore (B) and
(C) are true.
1 0 0 Answers: (B), (C)
= b+c a-b a-c
b2 + c2 a2 - b2 a2 - c2 7. If P is any square matrix, then the sum of its prin-
cipal diagonal elements is called trace of P and is
= (a - b)(a2 - c2 ) - (a - c)(a2 - b2 ) denoted by tr(P). Let A and B be two square matri-
= (a - b)(c - a)[-(c + a) + (a + b)] ces of same order and l a scalar. Which of the fol-
= (a - b) (b - c)(c - a) lowing are true?
(A) tr ( A + B) = tr ( A) + tr ( B) (B) tr (lA) = ltr ( A)
Therefore D1 = D2 = (a - b)(b - c)(c - a). So (B) and (D)
are true. (C) tr ( AB) = tr ( A)tr ( B) (D) tr ( AB) = tr ( BA)
Answers: (B), (D) Solution: Let A = [aij ]n´ n, B = [bij ]n´ n. Therefore
n n n
6. Let a, b, c be real numbers and tr ( A + B) = å (aii + bii ) = å aii + å bii
i =1 i =1 i =1
1 a a2 a b + c a2 1 1 1 = tr ( A) + tr ( B)
D1 = 1 b b , D2 = b c + a b , D3 = a
2 2
b c
This implies (A) is true. Again
1 c c2 c a + b c2 a3 b3 c3
n
tr (lA) = å laii
Then i =1

(A) D2 = D1 (a + b + c) n
= l å aii = ltr ( A)
(B) D3 = - D2 i =1

(C) - D2 = (a + b + c)D1 = D3
This implies (B) is true. Let AB = [cik]n ´ n where
(D) D3 = D2 = (ab + bc + ca)D1 n
cik = åa j =1
ij bjk
www.jeeneetbooks.in
432 Chapter 8 Matrices, Determinants and System of Equations

Therefore Therefore
x + (m + 5) y = m - 20
n n

åc = ååa b
ii
i =1 j =1
ij ji and - (2 m + 15) y = 60 - 2 m
n n Therefore
= å å bji aij
j =1 i =1 2 m - 60 25m
y= and x=
= tr ( BA) 2 m + 15 2 m + 15
Clearly tr ( AB) ¹ (tr A)(tr B). Therefore (A), (B) and (D) This gives
are true.
Answers: (A), (B), (D) æ - 15 ö
x > 0 and y > 0 Û m Î ç - ¥, ÷ È (30, ¥)
è 2 ø
8. Consider the system of equations 3x + my = m and 2x -
5y = 20. Then Therefore (D) is true.
(A) the system is inconsistent (i.e., has no solution) if Answers: (B), (C), (D)
m = 15/2
9. It is given that
(B) the system has no solution, if 2m = -15
(C) has unique solution, if m ¹ -15/2 x2 + x x+1 x-2
(D) has solutions with x > 0, y > 0 if and only if 2 x + 3x - 1
2
3x 3 x - 3 = xA + B
æ - 15 ö x + 2x + 3 2x - 1 2x - 1
2
m Î ç - ¥, ÷ È (30, ¥)
è 2 ø
where A and B are determinants of order 3 not involv-
Solution: The given system is equivalent to the matrix ing x. Then
equation
1 1 1 0 1 -2
é3 m ù émù (A) A = - 4 0 0 (B) B = - 4 0 0
ê 2 - 5ú X = ê 20 ú
ë û ë û 3 -3 3 3 -3 3
where 1 1 1 0 1 -2
éxù (C) A = 4 0 0 (D) B = - 4 0 0
X=ê ú
ë yû 3 -3 3 3 3 3

If Solution: Let

3 m x2 + x x+1 x-2
=0
2 -5 D = 2 x2 + 3 x - 1 3x 3x - 3
x + 2x + 3 2x - 1 2x - 1
2

then m = -15/2 and hence the equations are


x2 + x x+1 x-2
6x - 15y = - 15 and 2x - 5y = 20
D= -4 0 0 [by R2 ® R2 - (R1 + R3)]
which are inconsistent. Therefore (B) is true. x + 2x + 3 2x - 1 2x - 1
2

If m ¹ -15/2, by Crammer’s rule, the system has unique


solution. Therefore (C) is true. é æ x2 ö
Now using row operations R1 ® R1 - R2 and R2 ® R2 - x x+1 x-2 ê byy R1 ® R1 + ç ÷ R2
ë è 4ø
2R1, respectively, we get D = -4 0 0
2x + 3 2x - 1 2x - 1 x2 ù
é 1 m + 5ù é m - 20 ù and R3 ® R3 + ú
X=ê 4 R2 û
ê2 -5 ûú ú
ë ë 20 û
x x+1 x-2
é1 +5 ù é m - 20 ù D = -4 0 0 (by R3 ® R3 - 2 R1)
and ê0 - 2 m - 15ú X = ê - 2 m + 60 ú 3 -3 3
ë û ë û
www.jeeneetbooks.in
Worked-Out Problems 433

1 1 1 0 1 -2 1 3 -1
= x -4 0 0 + -4 0 0 f ( x ) = ( x - 2) 2 - 3 x x - 3
3 -3 3 3 -3 3 -3 2x x + 2
1 3 -1
= xA + B
f ( x) = ( x - 2) 0 - 3 x - 6 x - 1
Answers: (A), (B) 0 2x + 9 x - 1
(by R2 ® R2 - 2R1, R3 ® R3 + 3R1 )
10. Let
1 0 0
x -6 -1 f ( x) = ( x - 2) 0 - 3 x - 6 x - 1
f ( x) = 2 - 3 x x-3 0 2x + 9 x - 1
- 3 2x x+2
(by C2 ® C2 - 3C1, C3 ® C3 + C1 )
and a < b < g be the roots of f(x) = 0. Then
Therefore
(A) g = 2
(B) f ( x) < 0 for a < x < b f ( x) = ( x - 2)( x - 1)(- 5 x - 15)
f ( x) = - 3( x + 3)( x - 1)( x - 2)
(C) f ( x) > 0 for b < x < g
(D) f ( x) > 0 for a < x < b Hence we get

Solution: By the row operation R1 ® R1 - R2 and a = - 3, b = 1, g = 2


taking x - 2 common from R1, we get Answers: (A), (B), (C)

Matrix-Match Type Questions


1. Match the items of Column I with those of Column II. é 1 2 ù é -1 1ù
Let (B) A + B = ê ú+ê ú
ë3 4û ë- 2 0û
é1 2ù é -1 1ù
A=ê ú and B = ê ú é0 3 ù
ë3 4û ë- 2 0û =ê ú
ë1 4û
Column I Column II Therefore
é0 1ù
(A) A + AT é - 5 - 11ù ( A + B)T = ê ú
(p) ê
ë1 3 úû ë3 4û

é 2 5ù Answer: (B) Æ (r)


(B) (A + B)T (q) ê ú
ë5 8û é 1 2 ù é -1 1ù é -1 - 4 1 + 0 ù é -5 1ù
(C) AB = ê úê ú=ê ú=ê ú
(C) (AB)T
é0 1 ù ë 3 4 û ë -2 0 û ë -3 - 8 3 + 0 û ë -11 3û
(r) ê ú
ë3 4û
Therefore
é 4 1ù é - 5 - 11ù
(D) BTAT (s) ê ú ( AB)T = ê
ë3 0û ë1 3 úû

Solution: Answer: (C) Æ (p)

é 1 2 ù é 1 3 ù é 2 5ù (D) ( AB)T = BTAT


(A) A + AT = ê ú+ê ú=ê ú Answer: (D) Æ (p)
ë3 4û ë2 4û ë 5 8û

Answer: (A) Æ (q)


www.jeeneetbooks.in
434 Chapter 8 Matrices, Determinants and System of Equations

2. Match the items of Column I with those of Column II. (D) The system has infinitely many solutions

1 1 -3
Column I Column II
Þ 1+ l 2 + l -8 = 0
(A) (p) 1 1 -(1 + l ) 2 + l

cos( A - P ) cos( A - Q) cos( A - R) 1 0 0


cos( B - P ) cos( B - Q) cos( B - R) Þ 1+ l 1 3l - 5 = 0
cos(C - P ) cos(C - Q) cos(C - R) 1 -2 - l 5 + l
equals
Therefore 3l2 + 2l - 5 = 0 and hence
(B) If a, b, g are roots of x3 + bx + c = 0, (q) -1
then the value of the determinant -5
l = 1,
a b g 3
b g a is Answer: (D) Æ (p), (s)
g a b
3. Match the items of Column I with those of Column II.
b+c a 1 Let
(C) c + a b 1 is equal to (r) 0 é0 1 1ù é2 2 0 ù
a+b c 1 S = êê 1 0 1úú and A = êê 0 2 0 úú
êë 1 1 0 úû êë 0 0 2 úû
(D) If the system of equations (s) -5/3
x + y = 3z, (1 + l ) x + (2 + l ) y = 8z
Column I Column II
x - (1 + l ) y = -(l + 2)
has infinitely many solutions, then é-1 1 1 ù
value of l is (A) S2 = (p) ê 1 - 1 1 ú
ê ú
êë 1 1 - 1úû
Solution:
(A) Given determinant is é0 1 1ù
(B) 2S =-1
(q) êê 1 1 1úú
cos A sin A 0 cos P sin P 0
êë 1 2 0 úû
cos B sin B 0 ´ cos Q sin Q 0 = 0 ´ 0 = 0
cos C sin C 0 cos R sin R 0 é2 0 0ù
(C) 1/2(SA) = ê
(r) ê 1 1 1úú
Answer: (A) Æ (r)
êë 1 - 1 3úû
(B) a b g a +b +g a +b +g a +b +g
é2 1 1ù
(s) êê 1 2 1 úú
b g a = b g a -1
(D) SAS =
g a b g a b êë 1 1 2 úû
0 0 0
= b g a ( ∵ a + b + g = 0) Solution:
g a b é0 1 1ù é0 1 1ù é 2 1 1 ù
=0 (A) S2 = êê 1 0 1úú êê 1 0 1úú = êê 1 2 1 úú
êë 1 1 0 úû êë 1 1 0 úû êë 1 1 2 úû
Answer: (B) Æ (r)
(C) b + c a 1 a+b+c a 1 Answer: (A) Æ (s)
c+a b 1 = a+b+c b 1 =0
é-1 1 1 ù
a+b c 1 a+b+c c 1
(B) adj S = êê 1 - 1 1 úú
Answer: (C) Æ (r) êë 1 1 - 1úû
www.jeeneetbooks.in
Worked-Out Problems 435

Therefore Solution:
adj S 1 1 1 w2
S-1 = = adj S
det S 2 (A) D = Given determinant = 1 1 w
Answer: (B) Æ (p) w2 w 1
é0 1 1ù é 2 2 0 ù
1 w2
(C) SA = ê 1 0 1 úú êê 0 2 0 úú
1 1ê 0
2 2 D= 0 1 w = (w2 - w)(w - w2 ) (by C1 - C2 )
êë 1 1 0 úû êë 0 0 2 úû
w2 - w w 1
é0 1 1ù é 1 1 0 ù é0 1 1ù
= - (w4 - 2w3 + w2 )
= êê 1 0 1úú êê0 1 0 úú = êê 1 1 1úú
= - [w - 2 + w2 ]
êë 1 1 0 úû êë0 0 1úû êë 1 2 0 úû
= - [-1 - 2] = 3
Answer: (C) Æ (q)
Answer: (A) Æ (r)
é0 2 2 ù é-1 1 1 ù (B) Given
(D) SAS = ê 2 2 2 ú ´ ê 1 - 1 1 úú
ê ú
-1 1ê
2 0 0 0
êë 2 4 0 úû êë 1 1 - 1úû
determinant = b - c c - a a - b (on adding R2 + R3
é0 1 1ù é -1 1 1 ù to R1) = 0
c-a a-b b-c
= êê 1 1 1úú ´ êê 1 - 1 1 úú
êë 1 Answer: (B) Æ (p)
2 0 úû êë 1 1 - 1úû
é2 0 0ù
= êê 1 1 1úú (C) Given
êë 1 - 1 3úû
1 a a2 1 a bc
Answer: (D) Æ (r) determinant = 1 b b - 1 b ca
2

1 c c2 1 c ab
4. Match the items of Column I with those of Column II.
1 a a2 a a2 abc
1
Column I Column II = 1 b b2 - b b2 abc
abc
1 c c2 c c2 abc
(A) If w ¹ 1 is a cube root of unity, (p) 0
then the value of the determinant 1 a a2 a a2 1
1 w 3 2
w = 1 b b2 - b b2 1 =0
w3 1 w 1 c c2 c c2 1
w2 w 1 Answer: (C) Æ (p)

a-b b-c c-a (D) Given determinant equals


(B) b - c c - a a - b (q) 2 a+b+c b+c c+a
c-a a-b b-c 2 a + b + c c + a a + b (by C1 + C2 + C3)
a+b+c a+b b+c
1 a a2 - bc
(C) 1 b b - ca
2
(r) 3 a + b + c -a -b 1 a b
1 c c2 - ab = 2 a + b + c - b - c = 2(a + b + c) 1 b c
a + b + c -c -a 1 c a
a+b b+c c+a
1 a b
b+c c+a a+b
(D) If (s) -3 = 2(a + b + c) 0 b - a c - b
c+a a+b b+c
0 c-a a-b
= k(3abc - a3 - b3 - c3 ), then k is
www.jeeneetbooks.in
436 Chapter 8 Matrices, Determinants and System of Equations

= 2(a + b + c)[- (a - b)2 - (c - a)(c - b)] é 2 -2 - 4ù


= - 2(a + b + c)[a + b + c - ab - bc - ca]
2 2 2
(D) Let Q = êê - 1 3 4 úú
= 2[3abc - a3 - b3 - c3 ] êë 1 - 2 - 3 úû
Answer: (D) Æ (q) Now
5. Match the items of Column I with those of Column II. é 2 -2 - 4ù é 2 -2 - 4ù
Q = êê - 1
2
3 4 úú êê - 1 3 4 úú
Column I Column II
êë 1 - 2 - 3 úû êë 1 - 2 - 3 úû
éa b ù é 4 + 2 - 4 - 4 - 6 + 8 - 8 - 8 + 12 ù
(A) Let A = ê ú . If A = O, then (p) orthogonal
3

ëc d û matrix = êê - 2 - 3 + 4 2+9-8 4 + 12 - 12 úú = Q
A2 is êë 2 + 2 - 3 - 2 - 6 + 6 - 4 - 8 + 9 úû
é cos q sin q ù Answer: (D) Æ (r)
(B) The matrix ê ú is (q) zero matrix
ë - sin q cos q û
(C) If A and B are symmetric (r) idempotent 6. Match the items of Column I with those of Column II.
matrices, then AB + BA is matrix
Column I Column II
é 2 -2 - 4ù
ê
(D) The matrix ê - 1 3 4 úú is
(s) symmetric (A) The system of equations (p) 1
matrix lx + y + z = 0, - x + ly + z = 0,
êë 1 - 2 - 3 úû
- x - y + lz = 0 will have non-zero
solution, if real value of l is
Solution: (B) In the system of equations given (q) -1
in (A) if l = 1, then the number of
(A) A3 = O Þ det A = 0 Þ ad - bc = 0 Þ ad = bc
solutions is
Now A satisfies its characteristic equation | A = xI | = 0
where I is a 2 ´ 2 unit matrix. This implies (C) If the system of equations (r) ±1
x = cy + bz, y = az + cx, z = bx + ay
A2 - (a + d) A = 0 (∵ ad = bc) has non-zero solution in x, y and z,
A = (a + d ) A
2
then a2 + b2 + c2 + 2abc is equal to
(1) If a + d = 0, then A2 = 0. (D) If P is a matrix of order 3 ´ 3 such that (s) 0
(2) If a + d ¹ 0, then PT P = I (unit matrix of order 3 ´ 3)
and det P = 1 then det (P – I) equals
0 = A3 = (a + d) A2 and hence A2 = O
Answer: (A) Æ (q), (r), (s) Solution:
(A) The system has non-zero solution, if
(B) Let
é cos q sin q ù l 1 1
P=ê
ë - sin q cos q úû -1 l 1 = 0
-1 -1 l
Therefore
Solving this we get
é cos q sin q ù écos q - sin q ù é 1 0 ù
PP = ê
T
=
ë - sin q cos q úû êë sin q cos q úû êë0 1úû l (l 2 + 3) = 0 Þ l = 0
Answer: (A) Æ (s)
Therefore P is orthogonal.
(B) In the above system, if l = 1, then the matrix
Answer: (B) Æ (p)
é l 1 1ù
ê-1 l 1 ú
(C) By hypothesis A = A and B = B. Then
T T
ê ú
( AB + BA)T = ( AB)T + ( BA)T êë - 1 - 1 l úû

= BT AT + AT BT = BA + AB = AB + BA is non-singular and hence x = 0, y = 0, z = 0 is the only


Hence AB + BA is symmetric. solution. This is called trivial solution.
Answer: (C) Æ (s) Answer: (B) Æ (p)
www.jeeneetbooks.in
Worked-Out Problems 437

(C) The system has non-zero solution. This implies (D) | P - I | = | PT ( P - I )| = | PT P - PT | = | I - PT |


-1 c b = |( I - P )T | + | I - P | = - | P - I |
c -1 a = c Therefore | P - I | = 0.
b a -1 Answer: (D) Æ (s)

- 1(1 - a2 ) - c(- c - ab) + b(ca + b) = 0


a2 + b2 + c2 + 2abc = 1
Answer: (C) Æ (p)

Comprehension-Type Questions
1. Passage: If A is a square matrix, then the polynomial - x3 + 3 x2 + x + 8 + x + 6 x - 10 = 0
equation f(x) º |A - xI| = 0 is called characteristic equa-
x3 - 3 x2 - 8 x + 2 = 0
tion of the matrix A. It is given that every square matrix
satisfies its characteristic equation, that is f(A) = O. If Answer: (A)
(ii) From the given information
é0 1 2 ù
A3 - 3 A2 - 8 A + 2 I = O
A = êê 1 2 3 úú
1
êë 3 1 1 úû - ( A2 - 3 A - 8 I ) A = I
2
then answer the following questions: 1
A-1 = - ( A2 - 3 A - 8 I )
(i) The characteristic equation of the matrix A is 2
(A) x3 - 3 x2 - 8 x + 2 = 0 (B) x3 + 3 x2 - 8 x + 2 = 0 Answer: (B)
(C) x3 + 3 x2 - 8 x - 2 = 0 (D) x3 + 3 x2 + 8 x + 2 = 0
1
(iii) ( A-1 )2 = A-1 A-1 = - ( A2 - 3 A - 8 I ) A-1
(ii) A-1 is equal to 2
1
é -1 ù = - ( A - 3I - 8 A-1 )
ê8 1 - 1ú
é - 1 1 - 1ù 2
Answer: (B)
1ê ú
(B) - ê 8 - 6 2 úú

(A) ê 8 - 6 2 ú
2ê ú 2
ë - 5 3 - 1û êë - 5 3 - 1úû 2. Passage: Let A be a 3 ´ 3 matrix,

é - 1 - 1 - 1ù é - 1 1 - 1ù éxù é b1 ù
(C) êê 8 - 6 2 úú ê
(D) - 8 6 2 úú X = ê y ú and b = êêb2 úú
ê ú
ê
êë - 3 3 - 1úû êë 5 - 3 1 úû êë z úû êë b3 úû

Using elementary row operations on the matrix equa-


(iii) (A-1)2 equals
tion AX = B, we obtain an equation of the form A¢X =
1 -1 3 1 B¢ which is equivalent to the system AX = B. That is
(A) 4 I + A + I (B) - ( A - 8 A-1 - 3I )
2 2 2 either both systems are inconsistent or both have the
same set of solutions in x, y and z. Consider the fol-
1 1
(C) ( A + 8 A-1 + 3I ) (D) ( A + 8 A-1 - 3I ) lowing system of equations
2 2
x + y + z = 6, x + 2 y + 2z = 10 and x + 2 y + lz = m
Solution:
Answer the following questions
(i) Characteristic equation of A is
(i) The number of values of l for which the system
-x 1 2 has unique solution is
1 2-x 3 =0 (A) only one value
3 1 1- x (B) all real values except two values
(C) only two real values
Solving we get
(D) all real values except one value
- x[(2 - x)(1 - x) - 3] - 1(1 - x - 9) + 2(1 - 6 + 3x) = 0
- x[2 - 3 x + x2 - 3] + 8 + x + 6 x - 10 = 0
www.jeeneetbooks.in
438 Chapter 8 Matrices, Determinants and System of Equations

(ii) The system has no solution if given that, applying elementary row operations on the
(A) l = 2, m ¹ 10 equation AX = B, we get a system of the form A¢X = B¢
such that both systems are equivalent. Based on this
(B) l = 3, m = 10
information, answer the following questions for the
(C) l = - 3, m = 10
equations
(D) l = - 3, m = - 10
x+ y+z=1
(iii) The system has infinitely many solutions, if
x + 2 y + 4z = l
(A) l ¹ 3, m ¹ 10
and x + 4 y + 10z = l 2
(B) l = 2, m = 10
(C) l ¹ 3, m = 10 (i) The system is consistent for
(D) l = 0, m = 10 (A) only one value of l
(B) only two values of l
Solution:
(C) all real values except two values
(i) The given system is (D) infinite number of values
é1 1 1ù é6ù (ii) The system is inconsistent for
ê1 (A) only one value of l
ê 2 2 úú X = êê10 úú
(B) only two values of l
êë1 2 l úû êë m úû (C) only three values of l
é1 1 1 ù é 6 ù (D) infinite number of values of l
ê0 1 1 ú X = êê 4 úú (by R2 ® R2 - R1 ,
ú (iii) When l = 1, solution of the given system is given by
ê
êë0 1 l - 1úû êë m - 6 úû R3 ® R3 - R1 ) (A) x = 2k + 1, y = - 3k, z = k
(B) x = 2k, y = - 3k + 1, z = k
é1 0 0 ù é 2 ù
ê0 1 (by R1 ® R1 - R2 , (C) x = k, y = - 3k, z = k
ê 1 ú X = êê 4 úú
ú
R3 ® R3 - R2 ) (D) x = 2k - 1, y = - 3k, z = k
êë0 0 l - 2 úû êë m - 10 úû
where k is any real number.
Let
Solution: Given system is
é1 1 1 ù
A = êê1 2 2 úú é1 1 1 ù é1ù
ê1 2 4 ú X = ê l ú
êë1 2 l úû ê ú ê ú
êë1 4 10 úû êë l 2 úû
then det A = l - 2. Therefore l ¹ 2 Þ A is non-
singular matrix and hence é 1 1 1ù é 1 ù
ê0 1 3ú X = ê l - 1 ú (by R2 ® R2 - R1 ,
AX = B Þ X = A-1 B ê ú ê ú R3 ® R3 - R1 )
is the unique solution. êë0 3 9 úû êë l 2 - 1úû
Answer: (D)
(ii) If l = 2 and m ¹ 10, then the system reduces to é1 0 - 2ù é 2-l ù
ê0 1 3 ú X = ê l - 1 ú (by R1 ® R1 - R2 ,
x = 6, y + z = 4, 0 x + 0 y + 0z = m - 10 ¹ 0 ê ú ê ú R3 ® R3 - 3R2 )
êë0 0 0 úû êë l 2 - 3l + 2 úû
Hence, no solution, if l = 2 and m ¹ 10.
Answer: (A) (i) The system is inconsistent if l ¹ 1 and 2.
Answer: (B)
(iii) For l = 2, m = 10 ; the system has infinite number of
(ii) The system is consistent, if l = 1, 2.
solutions
Answer: (B) Answer: (D)
(iii) If l = 1, the given system is equivalent to the system
3. Passage: Suppose A is a square matrix of order 3 ´ 3, x - 2z = 1, y + 3z = 0 whose solution is x = 2k + 1,
éxù y = -3k and z = k.
X = êê y úú and B is column matrix of 3 ´ 1 order. It is Answer: (A)
êë z úû
www.jeeneetbooks.in
Worked-Out Problems 439

Assertion–Reasoning Type Questions


In each of the following, two statements, I and II, are Statement II: If A and B are square matrices of the
given and one of the following four alternatives has to same order and P, Q are non-singular matrices compat-
be chosen. ible for multiplication with A such that PAQ = B, then
A = P -1BQ-1.
(A) Both I and II are correct and II is a correct reason-
ing for I. Solution: Clearly Statement II is true. Now let
(B) Both I and II are correct but II is not a correct rea-
é2 1ù é- 3 2 ù
soning for I. P=ê ú , and Q = ê ú
(C) I is true, but II is not true. ë3 2û ë 5 - 3û
(D) I is not true, but II is true. é 2 - 1ù é3 2ù
P -1 = ê ú and Q-1 = ê ú
1. Statement I: There exit matrices B and C of order
ë- 3 2û ë5 3û
2 ´ 2 with integer elements such that é 1 0 ù -1 é 2 - 1ù é 3 2 ù
A = P-1 ê -1 -1
ú Q = P Q = ê- 3 2 ú ê5 3ú
ë 0 1 û ë ûë û
é -1 1 ù
B3 + C 3 = ê ú é1 1ù é1 - 1ù
ë 0 -2û =ê ú¹ê ú
ë1 0 û ë1 0 û
Statement II: Every square matrix satisfies its charac-
teristic equation. That is, if A is a square matrix, then Therefore statement I is false.
A satisfies the polynomial equation det (A - xI) = 0
where I is a unit matrix of same order as that of A. 3. Statement I: If A and B are square matrices of order
3 ´ 3 then adj (AB) = (adj B)(adj A).
Solution: Let
Statement II: For a square matrix of order 3 ´ 3,
é-1 1 ù
A=ê ú P(adj P ) = (adj P )P = (det P )I
ë 0 - 2û
where I is the third order unit matrix.
Therefore
-1 - x 1 Solution: P(adj P ) = (adj P )P = (det P )I
| A - xI | = = (1 + x)(2 + x)
0 -2 - x Therefore Statement II is true. Now

The characteristic equation is AB(adj B) × (adj A) = A( B adj B )(adj A)


f ( x) º (1 + x)(2 + x) = 0 = A(det B)I (adj A)
that is = (det B) A(adj A)
= (det B)(det A)I
f ( x) º x2 + 3 x + 2 = 0
= [det( AB)]I
Statement II is true by Cayley–Hamilton theorem. Similarly (adj B)(adj A)AB = [det (AB)]I
f ( A) = 0 Þ A2 + 3 A + 2 I = 0 Therefore adj (AB) = (adj B)(adjA).
Þ A3 + 3 A2 + 2 A = 0 Answer: (A)
Þ (A + 1) - I = A
3

4. Statement I: If A is a nonsingular matrix, then adj


Take (A-1) = (adj A)-1.
é0 1ù
B= A+ I =ê ú and C = -I
ë0 - 1û Statement II: If P and Q are square matrices then
We get A = B3 + C 3. adj (PQ) = (adj Q)(adj P).
Solution: Statement II is clear from Q3 above.
2. Statement I: If
Now (adj A)(adj A-1 ) = adj ( A-1 A)
é2 1ù é- 3 2 ù é 1 0 ù = adj ( I ) = I
ê 3 2 ú A ê 5 - 3ú = ê 0 1ú Also
ë û ë û ë û
(adj A-1 )(adj A) = adj( AA-1 ) = adj I = I
then A is equal to Therefore (adj A)-1 = adj ( A-1 ).
é1 - 1ù Answer: (A)
ê1 0 ú
ë û
www.jeeneetbooks.in
440 Chapter 8 Matrices, Determinants and System of Equations

5. Statement I: Let B be a matrix of 3 ´ 3 order and Statement II:


adj B = A. If P and Q are matrices of 3 ´ 3 order such
that | P | = 1 = |Q | then adj (Q-1 BP-1 ) = PAQ. (b + c)2 a2 a2
b2 (c + a)2 b2 = 2abc (a + b + c)3
Statement II: If M is non-singular square matrix of
order 3 ´ 3, then adj (M -1 ) = (adj M )-1. c2 c2 (a + b)2

Solution: Statement II is true is clear from Q4 above. Solution: Let


-1 -1 -1 -1
adj (Q BP ) = (adj P )(adj B)(adj Q )
(b + c)2 a2 a2
-1 -1
= (adj P )(adj B)(adj Q) D = b2 (c + a)2 b2
= P(adj B) Q c2 c2 (a + b)2
= PAQ
(b + c)2 a2 - (b + c)2 a2 - (b + c)2
since | P | = 1 = | Q | Þ (adj P ) -1 = P and (adj Q) -1 = Q.
D = b2 (c + a)2 - b2 0
Answer: (A) c2 0 (a + b)2 - c2
(by C2 ® C2 - C1 , C3 ® C3 - C1 )
6. Statement I: If
é1 2 0 ù Taking a + b + c common from C2 and C3 we get
A = êê 2 -1 0 úú (b + c)2 a-b-c a-b-c
êë 0 0 -1úû D = (a + b + c)2 b2 c+a-b 0
c2 0 a+b-c
then
1 2bc - 2c - 2b
( A-1 )2 = [ A + I - 5A-1 ]
5 D = (a + b + c)2 b2 c + a - b 0
c2
0 a+b-c
Statement II: If P is a square matrix of order 3 ´ 3,
then P satisfies the polynomial equation | P - xI | = 0. [by R1 ® R1 - (R2 + R3 )]
Solution: Statement II is Cayley–Hamilton theorem.
2bc 0 0
1- x 2 0
b2
| A - xI | = 2 -1 - x 0 = - x3 - x2 + 5 x + 5 D = b2 c+a (a + b + c)2
c
0 0 -1 - x
c2
c2 a+b
Since A satisfies its characteristic equation we get b
- A3 - A2 + 5 A + 5I = O æ 1 1 ö
çè by C2 ® C2 + C1 , C3 ® C3 + C1 ÷ø
A3 + A2 - 5 A - 5I = O b c
1 = 2bc [(c + a)(a + b) - bc](a + b + c)2
A( A2 + A - 5I ) = I
5 = 2abc (a + b + c)3
1
A-1 = ( A2 + A - 5I ) Therefore Statement II is true. Now put s – a = x, s – b = y,
5
s – c = z so that
1
( A-1 )2 = ( A + I - 5 A-1 ) x + y + z = s, y + z = a, z + x = b, x + y = c
5
Answer: (A) Therefore

7. Statement I: If 2s = a + b + c, then a2 ( s - a)2 ( s - a)2 ( y + z)2 x2 x2


( s - b)2
b2
( s - b) =
2
y2
(z + x )2
y2
a2 ( s - a)2 ( s - a)2 ( s - c)2 ( s - c)2 c2 z2 z2 ( x + y)2
( s - b)2 b2 ( s - b)2 = 2 s3 ( s - a)( s - b)( s - c)
( s - c)2 ( s - c)2 c2 Use Statement II. Hence, Statement I is correct.
Answer: (A)
www.jeeneetbooks.in
Worked-Out Problems 441

Integer Answer Type Questions


1. If A is a square matrix, then the number of ordered 4. The number of pairs (A, B) where A and B are 3 ´ 3
pairs of matrices (P, Q) where P is a symmetric matrix matrices such that AB - BA = I (I is the unit matrix
and Q is a skew-symmetric matrix such that A = P + Q of 3 ´ 3 order) is .
is .
Solution: Let
Solution: If A is any square matrix, then 1/ 2( A + AT )
is symmetric and 1/ 2( A - AT ) is skew-symmetric and é a1 b1 c1 ù é a11 b11 c11 ù
ê ú ê ú
A = êa2 b2 c2 ú and B = êa12 b12 c12 ú
1 1
A= ( A + AT ) + ( A - AT ) êë a3 b3 c3 úû ê a13 b13 c13 úû
2 2 ë
Suppose A = P + Q where P is symmetric matrix and Q is such that AB - BA = I . Principal diagonal elements of
skew-symmetric matrix. Then AB - BA are equal to 1.
AT = PT + QT = P - Q (a12b1 - a2b11 ) + (c1a13 - c11a3) = 1 (8.25)
But (a2b11 - a12b1) + (c2b13 - c12b3) = 1 (8.26)
1 1 (a c - a c ) + (b c - b c ) = 1
1 1 1 1
(8.27)
P= ( A + AT ) Q = ( A - AT ) 3 1 3 1 3 2 3 2

2 2 Adding Eqs. (8.26) and (8.27) we get


Therefore (P, Q) is a unique pair. (a2b11 - a12b1) + (a3c11 - a13c1) = 2
Answer: 1
which is impossible according to Eq. (8.25). Hence, there
2. If exist no such matrices.
Answer: 0
x2 + 3 x x-1 x+3
x+1 - 2 x x - 4 = a0 x4 + a1x3 + a2 x2 + a3x + a4 5. Let S be the set of all symmetric matrices of order
x-3 x + 4 3x 3 ´ 3, all of whose elements are either 0 or 1. If five of
these elements are 1 and four of them are 0, then the
then the value of a4 is . number of matrices in S is .

Solution: For x = 0 we get Solution: Let A Î S. In a symmetric matrix the (i, j)


th element is same as (j, i)th element for i ¹ j . That is,
0 -1 3 upper and lower parts of the principal diagonal are
a4 = 1 0 - 4 reflections of each other through the principal diago-
nal. Hence the principal diagonal of A must have three
-3 4 0
1’s or two 0’s and a single because A has five 1’s and
= 1(0 - 12) + 3(4 - 0) = 0 four 0’s. If all the three diagonal elements are 1, the
Answer: 0 number of such matrices is 3C1. If two diagonal elements
are zeros and one is 1, then the number of such matri-
3. If ces is 3C1 ´ 3C1. Therefore, total number of matrices in
S = 3C1 + 3C1 ´ 3C1 = 3 + 3 ´ 3 = 12.
6i - 3i 1 Answer: 12
4 3i - 1 = x + iy
6. Let a, b, c be real positive numbers such that abc = 1
20 3 i
and
where i = - 1, given x + y is equal to
2 2
. éa b c ù
Solution: The given determinant is equal to A = êêb c a úú
6i(- 3 + 3) + 3i(4i + 20) + 1(12 - 60i) = êë c a búû

- 12 + 60i + 12 - 60i = 0 = 0 + i 0 Let S be the set of all such matrices A such that
AT A = I. Then, the number of matrices in S is .
Therefore x = 0, y = 0.
Answer: 0
www.jeeneetbooks.in
442 Chapter 8 Matrices, Determinants and System of Equations

Solution: Solution: We have


éa2 + b2 + c2 å ab å ab ù é cos a1 cos a2 cos a1 sin a2 ù
ê
ATA = I Þ ê å ab a + b2 + c2
2
å ab ú
ú A1 A2 = ê ú cos(a1 - a2 )
ë sin a1 cos a2 sin a1 sin a2 û
ê å ab å ab a2 + b2 + c2 úû
ë é0 0 ù é∵ a1 - a2 is an odd mutiple ù
=ê ú ê ú
é1 0 0ù ë0 0 û êë of p / 2, cos(a1 - a2 ) = 0 úû
= êê0 1 0 úú
Sum of the elements of A1 A2 is 0.
êë0 0 1úû
Answer: 0
Now å ab = 0 which is not possible as a, b, c are positive. 9. If x, y, z are non-zero real numbers and
Hence S is an empty set.
1+ x 1 1
Note: In 2003 (JEE), under the same hypothesis, it was
asked to find the value of a3 + b3 + c3 for which many 1 + y 1 + 2y 1 =0
authors gave a3 + b3 + c3 value as 4, without verifying the 1 + z 1 + z 1 + 3z
fact whether such matrices exist or not.
Answer: 0 æ 1 1 1ö
then - ç + + ÷ is equal to .
è x y zø
7. Consider the 8 × 8 square matrix filled with the natural
Solution: Let Δ be the given determinant. Then apply
numbers from 1 to 64 as is given below.
row transformation R1 ® R1 - R3 , R2 ® R2 - R3 we get
é1 2 3 4 8ù
x 0 1
ê 9 10 11 12 16 ú
ê ú D= y 2y 0 =0
ê 17 18 19 13 24 ú
ê ú - 2z - 2z 1 + 3z
ê      ú
êë 57 58 59 60 64 úû x 0 0
A number is selected from the board and the cor- y æ 1 ö
D= y 2y 1- = 0 ç by C3 ® C3 - C1 ÷
responding row and column are deleted. Again x è x ø
another number is selected and the row and column 2z
- 2z - 2z 1 + 3z +
are deleted. The process is continued upto 8 times so x
that no row and no column is left. Then the sum of the
numbers so selected is . Solving we get

Solution: Observe that the element in the ith row and jth é æ 2z ö æ yö ù
x ê 2 y ç 1 + 3z + ÷ + 2z ç 1 - ÷ ú = 0
column is (i - 1)8 + j. Let a1j1 , a2j2 , … , a8j8 be the numbers ë è xø è xø û
so selected where j1 , j2 , … , j8 are different. Therefore [2 xy + 6zxy + 4 yz + 2zx - 2 yz] = 0
é1 1 1 ù
8 8

åa iji = å [(i - 1)8 + ji ] 2( xyz) ê + + + 3ú = 0


x y z
i =1 i =1 ë û
= 8(0 + 1 + 2 + 3 + + 7) + (1 + 2 + 3 + + 8) æ 1 1 1ö
= 8 ´ 28 + 36 -ç + + ÷ = 3
è x y zø
= 224 + 36
Answer: 3
= 260
Answer: 260 10. If

x 2 x
8. Let
f ( x) = x2 x 6 = ax4 + bx3 + cx2 + dx + e
é cos2 ak cos ak sin ak ù x x 6
Ak = ê ú (k = 1, 2)
ëcos ak sin ak sin2 ak û
then the absolute value of 5a + 4b + 3c + 2d + e is
If the difference between a1 and a2 is an odd multiple .
of p /2, then A1 A2 is a matrix whose sum of all its ele-
ments is equal to
www.jeeneetbooks.in
Summary 443

Solution: We have Therefore a = 1, b = - 1, c = - 12, d = 12, e = 0. Also


f ( x) = x(6 x - 6 x) - 2(6 x2 - 6 x) + x( x3 - x2 ) 5a + 4b + 3c + 2d + e = 5 - 4 - 36 + 24 + 0 = - 11
= x - x - 12 x + 12 x
4 3 2
So that absolute value is 11.
Answer: 11

SUMMARY
8.1 Matrix: Let aij (l £ i £ m and l £ j £ n; m and n are 8.6 Upper and lower triangular matrices: A square
positive integers) be real numbers or complex num- matrix A = [aij ]n ´ n is called upper triangular matrix,
bers or functions or any kind of expressions. Then if aij = 0 for i > j (i.e., the elements below the princi-
the arrangement of these aij in the shape of a rectan- pal diagonal are zeros).
gle enclosed by two brackets is called a rectangular It is called lower triangular, if aij = 0 for i < j
matrix of order m ´ n. (i.e., the elements above the principal diagonal are
zeros).
é a11 a12 a13 a1n ù
ê a21 a22 a23 a2 n ú
ê ú 8.7 Diagonal matrix: A matrix which is both upper and
ê × × ú lower triangular is a diagonal matrix or a square
ê × × ú matrix A = [aij ]n ´ n is called diagonal matrix, if aij = 0
êa ú
ë m1 am 2 am 3 amn û for i ≠ j.
is an m ´ n matrix
8.8 Scalar matrix: In a diagonal matrix, if all the principal
An m ´ n matrix in which the (i, j)th element is
diagonal elements are equal, it is called scalar matrix.
aij will be written as [aij ]m ´ n or [aij ]m ´ n.
That is in a square matrix A = [aij ]n ´ n, if
Horizontal lines are called rows and vertical
lines are called columns aij is the element in the ith ì0 for i ¹ j
row and jth column position. aij í
îl (real or complex) for i = j
8.2 Vertical and horizontal matrices: If the number then A is called scalar matrix.
of rows is greater than the number of columns, it is
called vertical matrix. 8.9 Transpose of a matrix: The matrix obtained from a
If the number of rows is less than the number of given matrix by changing its rows in to columns is
columns, it is called horizontal matrix. called transpose of the given matrix. If A is a matrix,
its transpose is denoted by AT or A1.
QUICK LOOK
QUICK LOOK
Rectangular matrix means either vertical or horizontal
matrix. About transpose:
1. If A is of order m ´ n, then AT is of order n ´ m.
8.3 Square matrix: If the number of rows is same as 2. The (i-j)th element of A is equal to ( j-i)th element
the number of columns, then the matrix is called a of AT.
square matrix. 3. (AT)T = A.

8.4 Principal diagonal and trace: In a square matrix


8.10 Addition of matrices: If A = [aij ]m ´ n and B = [bij ]m ´ n
[aij ]n ´ n, the elements a11, a22, a33, ¼, ann are called
are two matrices of same order m ´ n, then the
principal diagonal elements and their sum is called
matrix whose (i-j)th element is aij + bij called sum of
Trace of the matrix and is denoted by Trace A where
A and B and is denoted by A + B = [aij + bij ]m ´ n.
A is a given square matrix.
8.11 Scalar multiplication: If A = [aij ]m ´ n is a matrix and
8.5 Zero (null ) matrix and unit matrix: In a matrix, if all
k is a scalar (i.e., real or complex) then kA is the
the elements are zeros, then it is called zero matrix.
matrix [kaij]m ´ n.
In a square matrix, if the principal diagonal ele-
In particular, if k = −1, then (−1) A is denoted
ments are equal to 1 and the rest are zeros, it is called
by −A.
unit matrix.
www.jeeneetbooks.in
444 Chapter 8 Matrices, Determinants and System of Equations

8.12 Difference of matrices: If A and B are two matrices (3) 0A = A0 = 0 where 0 is the zero matrices of
of same order, then A - B is defined as A + (−B). appropriate orders.
That is, if A = [aij ]m ´ n and B = [bij ]m ´ n, then A − B = (4) If Im and In are unit matrices of orders m and n
[aij − bij]m´n. respectively, then Im A = A = AIn.

8.13 Theorem: The following hold for any matrices of 8.16 Distributive laws:
same order.
(1) Let A, B be matrices of same order m ´ n and C
(1) If A and B are matrices of same order, then be any matrix of order n ´ p. Then
A + B = B + A. (commutative law)
(A + B)C = AC + BC
(2) If A, B and C are matrices of same order, then
(2) Let A be of order m ´ n and B, C be of order
(A + B) + C = A + (B + C) (Associative law).
n ´ p. Then
(3) A + O = O + A = A for any matrix of order
m ´ n where O is a zero matrix of m ´ n order. A(B + C) = AB + AC
(4) A + (-A) = (-A) + A = O.
8.17 Important feature of a scalar matrix: Square matrix
(5) If A and B are of same order, and l is a scalar, A is a scalar matrix if and only if A commutes with
then l(A + B) = lA + lB. every matrix of the same order.
(6) If l and μ are any two scalars, then (l + μ)A =
lA + μA for any matrix A. 8.18 Transpose of a product: Let A and B be two matrices
(7) If A is an m ´ n matrix and l = 0 is the usual zero of m ´ n and n ´ p orders , respectively. Then
scalar and 0 is the m ´ n zero matrix, then 0A = 0. (AB)T = BTAT
(8) (AT)T = A.
8.19 Inverse of a matrix: Let A be a square matrix of
(9) (A ± B)T = AT ± BT.
order n ´ n. If B is a square matrix of the same order
(10) (lA) = lA where l is a scalar.
T T
n × n such that AB = BA = In (unit matrix of order n),
then B is called inverse of A and is denoted by A−1.
8.14 Matrix multiplication: Let A = [aij]m´n and B = [bjk]n´p If a matrix has inverse, then it is called invertible
be two matrices of orders m ´ n and n ´ p respectively. matrix.
Let
n 8.20 Inverse
Cik = å (aij bjk ) = ai 1b1k + ai 2 b2 k + ai 3 b3k + + ain br
j =1
(1) (A−1)−1 = A.
(2) If A and B are square matrices of same order
Then the matrix [cik]m´p (order m ´ p) is the prod-
having inverses, then AB has also inverse and
uct AB.
(AB)−1 = B−1A−1.
(3) If A is an invertible matrix, then
QUICK LOOK
(A−1)T = (AT)−1
1. For convenience and easy to remember, the
general element of A is taken as (i-j)th element 8.21 Some kinds of matrices:
aij and that of B as (j-k)th element bjk and writ-
ten (i-k)th element as the general element of the (1) Symetric matrix: A square matrix A is called
product AB. In fact to write some rth row and symmetric matrix, if AT = A.
sth column element of the product AB, take the (2) Skew-symmetric matrix: A square matrix A is
rth row of A and sth column of B, multiply the called skew-symmetric if AT = −A.
corresponding elements and add. (3) Orthogonal matrix: Square matrix A is called
2. The product AB is defined only when the number orthogonal matrix, if ATA = I.
of columns of A is same as the number of rows of B.
(4) Idempotent matrix: Square matrix A is called
3. In general AB and BA are not equal, even though idempotent matrix if A2 = A.
when both products are defined.

QUICK LOOK
8.15 Theorem: Let A, B, C be m ´ n, n ´ p and p ´ q
matrices and l, any scalar. Then, the following hold. In a skew-symmetric matrix, all the principal diagonal
elements must be zeros. The converse is not true.
(1) (AB)C = A(BC) (Associative law).
(2) (lA)B = l(AB) = A(lB).
www.jeeneetbooks.in
Summary 445

(5) Nilpotent matrix: Square matrix A is called For example, if


nilpotent matrix, if Am = O for some positive
integer m. The least positive integer m such é1 + i -i 2 ù
that Am = O is called the index of the nilpotent A = ê1 ú
ê -i 2i a + ibú
matrix A. ë2 û
(6) Periodic matrix: Square matrix A is called
periodic matrix, if Ap+1 = A for some positive then
integer p. The least such positive integer p is
é1- i i 2 ù
called the period of A. ê ú
A= 1
ê + i - 2i a - ib ú
QUICK LOOK ë2 û

An idempotent matrix is a periodic matrix of period 1. 8.26 Some properties of conjugate:

(1) ( A) = A.
(7) Involutary matrix: Square matrix A is called (2) (lA) = l A for any scalar l.
involuntary matrix, if A2 = I. (3) ( A ± B) = A ± B.
(4) AB = A B.
QUICK LOOK
8.27 Theorem: If A is any matrix, then
Every involuntary matrix is a periodic matrix of
period 2. ( A)T = ( AT )
8.28 Notation: ( A)T is denoted by A*.

8.22 Theorem: Let A and B be square matrices of order


8.29 Hermitian and skew-Hermitian matrices: Square
n ´ n. Then the following hold. matrix A is called Hermitian or skew-Hermitian
(1) If A and B are symmetric matrices then so is according as A* = A [i.e., ( A)T = A ] or
A ± B.
(2) If A and B are skew-symmetric matrices, then so is A* = - A [i.e., (A)T = - A]
A ± B.
8.30 Theorem: Let A and B be matrices. Then the
(3) If AB = BA and A and B are symmetric (skew-
following hold.
symmetric) then AB is symmetric.
(4) If A is symmetric, then for any scalar l, lA is (1) (A*)* = A.
also symmetric. If A is skew-symmetric, then (2) (l A)* = l A* for any scalar l where l is the
lA is also skew-symmetric. complex conjugate of l.
(5) If AB = BA, then AB is skew-symmetric prov- (3) ( A ± B)* = A* ± B*.
ided one of A and B is symmetric and the other (4) (AB)* = B*A* when A and B are compatible
is skew-symmetric. for multiplication.

8.23 Theorem: If A is any square matrix, then A + A is


T
8.31 On Hermitian and skew-Hermitian matrices:
T
symmetric and A − A is skew-symmetric.
(1) If A is any square matrix, then A + A* is
8.24 Representing square matrix in terms of symmetric Hermitian and A − A* is skew-Hermitian.
and skew-symmetric matrices: Every square matrix A (2) If l is real and A is Hermitian (skew-Hermi-
can be expressed as a sum of symmetric and skew- tian) then lA is Hermitian (skew-Hermitian).
symmetric matrices uniquely and the representation is (3) If A is Hermitian, then iA is skew-Hermitian
1 1 and iA is Hermitian, if A is skew-Hermitian.
A= ( A + AT ) + ( A - AT ) (4) If A and B are Hermitian, then so is A ± B.
2 2
(5) If A and B are skew-Hermitian, then so is A ± B.
8.25 Conjugate of a matrix: It A is a matrix whose (6) If AB = BA and A, B are Hermitian, then AB is
elements (i.e., entries) are complex numbers, then also Hermitian.
the matrix obtained from A by replacing its elements
(7) If A and B are skew-Hermitian and AB = BA,
with their corresponding complex conjugates is called
then AB is Hermitian.
conjugate of A and is denoted by A.
www.jeeneetbooks.in
446 Chapter 8 Matrices, Determinants and System of Equations

(8) If AB = BA and one of A and B is Hermitian 8.36 Determinant of 3 ´ 3 matrix:


while the other is skew-Hermitian, then AB is Let
skew-Hermitian.
é a1 b1 c1 ù
8.32 Decomposition of a square matrix in terms of A = êêa2 b2 c2 úú
Hermitian and skew-Hermitian matrices: Let A êë a3 b3 c3 ûú
be a square matrix. Then A can be expressed as sum
of Hermitian and skew-Hermitian matrices in one Then det A (or | A | ) = a1A1 + b1B1 + c1C1, where
and only one way and the representation is A1, B1 and C1 are the cofactors of a1, b1 and c1,
respectively.
1 1
A = ( A + A*) + ( A - A*)
2 2 Expansion:
det A = a1 (b2 c3 - b3c2 ) - b1 (a2 c3 - a3c2 )
QUICK LOOK
b2 c2 a2 c2 a2 c2
In the above, if we take + c1 (a2 b3 - a3b2 ) = a1 - b1 + c1
b3 c3 a3 c3 a3 c3
1 i ( A* - A)
P = ( A + A*) and Q=
2 2 8.37 Properties of determinants:
where i = -1, then A = P + iQ, where both P and Q Let
are Hermitian matrices.
é a1 b1 c1 ù
ê
A = êa2 b2 c2 úú
Determinants êë a3 b3 c3 ûú
Even though the determinant of any square matrix whose and capital letters Ai, Bi and Ci respectively denote
elements are real (complex) numbers may be defined, cofactors of ai, bi and ci for i = 1, 2, 3. Then
our main focus is on determinants of 2 ´ 2 matrix or 3 ´ 3
matrix. We begin with minor and cofactors of the elements (1) a2A2 + b2B2 + c2C2 = a3A3 + b3B3 + c3C3 = a1A1 +
of a matrix. a2A2 + a3A3 = b1B1 + b2B2 + b3B3 = c1C1 + c2C2 +
Throughout this summary on determinants our matri- c3C3 = det A
ces are 3 ´ 3 matrices and in some cases 2 ´ 2 matrices. That is, we can expand the determinant in
any row or any column.
8.33 Let (2) In a matrix A, if any two rows (columns), are
interchanged, then the sign of the determinant
éa b ù will change.
A=ê ú
ëc d û (3) In a matrix, if two rows (columns) are identical,
then the value of the determinant is zero.
Then the number ad − bc is called the determinant
of A and is denoted by det A or | A |. (4) det A = det (AT).
(5) The elements of a row (column) are multiplied by
8.34 Minor: Let A be a 3 ´ 3 matrix. Then, the determi- some non-zero constant l amounts that, the deter-
nant of the 2 ´ 2 matrix obtained from A, by delet- minant is multiplied with the same constant l.
ing the ith row and jth column of A is called minor In other words, if l is a common factor of
of A with respect to (i−j)th element (i = 1, 2, 3 and all the elements of a row (column), then the
j = 1, 2, 3). (i−j)th minor is denoted by Mij. determinant of the matrix is equal to l times the
determinant of the matrix obtained after taking
8.35 Cofactor: (−1)i+j Mij is called the cofactor of the ele- away l from the elements of that row (column).
ment aij with respect to the matrix A. The cofactor For example
of the element aij is denoted by Aij (capital letter).
la1 lb1 lc1 a1 b1 c1
QUICK LOOK a2 b2 c2 = l a2 b2 c2
a3 b3 c3 a3 b3 c3
In the formation of Aij, the ith row and jth column will
not participate. (6) det (lA) = l3 det A. In general, if A is a square
matrix of order n, then det (lA) = ln det A.
www.jeeneetbooks.in
Summary 447

(7) aiAj + biBj + ciCj = 0 for i ≠ j.


QUICK LOOK
That is, the sum of the products of the
elements of a row with cofactors of the cor- A−1 exists if and only if A is a non-singular matrix. That
responding elements of another row is always is, non-singular matrices only, will have inverses.
zero. The same is true for columns also.
(8) The determinant of a matrix is unaltered by
adding constant times the elements of a row Elementary Row (Column) Operations
to the corresponding elements of another row.
The same is true for columns also. 8.42 The following are called elementary row (column)
(9) If each element of a row (column) is sum of two operations on a matrix.
elements, then the determinant of the matrix
(1) Interchanging of two rows (columns) denoted
can be expressed as sum of two determinants.
by Rij(Cij). That is, interchanging of ith and jth
(10) If A and B are two square matrices of same rows (ith and jth columns).
order, then det (AB) = (det A) (det B).
(2) Multiplication of the elements of a row
8.38 Adjoint of a matrix: Let A be a square matrix and
(column) by a non-zero constant k denoted by
B is the matrix obtained from A, by replacing its Ri(k)(Ci(k)).
elements with their corresponding cofactors. Then (3) Multiplying the elements of a row (column)
BT is called adjoint of A. Adjoint of A is written as with a non-zero constant k and adding to
adj A. the corresponding elements of another row
For example, let (column) denoted by Rs + Rr(k) or Rs: Rs + Rr(k).
That is multiplying the elements of rth row
é a1 b1 c1 ù by k and adding to the corresponding elements
A = êêa2 b2 c2 úú of sth row. Same is Cs : Cs + Cr(k).
êë a3 b3 c3 ûú Elementary transformation means either row
or column transformation.
and A1, B1, C1, etc. denote the cofactors of a1, b1, c1
etc. Then 8.43 Elementary matrix: Matrix obtained from unit
matrix by applying elementary transformations.
é A1 A2 A3 ù Every elementary matrix is invertible.
adj A = êê B1 B2 B3 úú
êëC1 C2 C3 úû 8.44 Theorem: Every non-singular matrix can be expre-
ssed as a product of elementary matrices.
8.39 Theorem: If A is a square matrix, then A(adj
A) = (det A) I = (adj A) A, where I is the corresponding Systems of Linear Equations
unit matrix.
8.45 Homogeneous system: Let aij (l £ i £ m, l £ j £ n) be
8.40 Existence of inverse and formula for inverse: If mn real numbers. Then the system of equations
A is a square matrix, then A−1 exists if and only if
det A ≠ 0 and in such a case a11 x1 + a12 x2 + + a1n xn = 0
a21 x1 + a22 x2 + + a2 n xn = 0
adj A
A-1 =    
det A
   
8.41 Non-singular and singular matrices: Square matrix am1 x1 + am 2 x2 + + amn xn = 0
A is called non-singular or singular according as is called homogeneous system of m equations in n
det A ≠ 0 or det A = 0. unknowns.
Matrix equation: If

é x1 ù
êx ú
A = [aij ]m´ n , X = ê ú
2

êú
ê ú
ë xn ûn´1
www.jeeneetbooks.in
448 Chapter 8 Matrices, Determinants and System of Equations

and O is m ´ 1 zero matrix, then the above homoge- Then


neous system of equations can be represented by the
D1 D D
matrix equation AX = O. x1 = , x2 = 2 , x3 = 3
D D D
Non-homogeneous system: AX = B where
is the solution for the given simultaneous system
é b1 ù of equations.
êb ú
ê 2ú 8.51 Consistency and inconsistency system: If AX = B
B=ê  ú
ê ú has a solution, then it is called consistent system,
ê ú otherwise inconsistent system.
êëbm úû
8.52 Geometrical interpretation: Consider the system
is an m ´ 1 matrix and atleast one bi ≠ 0.
of three simultaneous equations a1x + b1y + c1z = d1,
In both systems A is called coefficient matrix.
a2x + b2y + c2z = d2, a3x + b3y + c3z = d3 which rep-
resent planes in the three dimensional space. Then
8.46 Zero solution or trivial solution: For AX = O, t1 = x2 =
x3 = = xn = 0 is always solution and this solution is (1) Unique solution means, all the three planes are
called zero solution or trivial solution. concurrent at a single point.

8.47 Non-zero solution (non-trivial solution)

é x1 ù I II
êx ú
ê 2ú
X=ê ú III
ê ú
êú
êë xn úû
(2) Infinite number of solutions means, all the three
is called a non-zero solution, if atleast one xi ≠ 0 and planes pass through a single straight line.
satisfies the equation AX = O.

8.48 Existence of non-zero solution: Suppose A is a non-


zero square matrix. A is non-singular if and only if
III
X = O is the only solution of AX = O and hence
AX = O has non-zero solution if and only if A is a
singular matrix (i.e., det A = O) and in such a case, I
the system has infinitely many solutions.
II
8.49 About AX = B (unique solution): If A is non-
singular matrix and B is non-zero column matrix,
then AX = B has unique solution, viz., X = A−1B. (3) Inconsistent means either all the three planes are
parallel to each other or form a triangular prism.
8.50 Crammer’s rule: Consider the system of simultane-
ous equations a1x + b1y + c1z = d1, a2x + b2y + c2z = d2
and a3x + b3y + c3z = d2 where atleast one di ≠ 0. Let I

a1 b1 c1
D = a2 b2 c2
a3 b3 c3 I II
II
and Δk is the determinant obtained from Δ by
replacing its kth column with III

é d1 ù III
êd ú
ê 2ú
êë d3 úû
www.jeeneetbooks.in
Exercises 449

8.53 Let f(x) º a0xm + a1xm–1 + + am where a0, a1, 8.55 Cayley–Hamilton theorem: Every square matrix
a2, ¼, am are real (complex) numbers. If A is a statisfies its characteristic equation. That is, if A is
square matrix, then f(A) means, the matrix a0Am + a square matrix and f ( x) = | A - xI |, then f (A) = O
a1Am–1 + a2Am–2 + + am–1A + amI where I is the (zero matrix).
unit matrix of order same as A.
8.56 Condition for a non-singular matrix: Let A be
8.54 Characteristic polynomial (equation of a matrix): square matrix of order n f(x) = |A - xI| = a0xn +
If A is a square matrix, then | A - xI | which is a poly- a1xn-1 + a1xn-2 + + an. Then A is non-singular if
nomial with real or complex coefficients is called and only if an ≠ 0.
characteristic polynomial of the matrix A and the
equation | A - xI | = 0 is called characteristic equa-
tion of the matrix A.

EXERCISES
Single Correct Choice Type Questions
1. The number of 2 ´ 2 matrices with real entries which 4. If
commute with the matrix éê
1 2ù
ú is écos x - sin x 0 ù
ë 1 - 1û
-
F ( x) = êê sin x cos x 0 úú
(A) 1 (B) 2 (C) 4 (D) infinite êë 0 0 1úû

éa b ù then F ( x)F ( y) is equal to


2. If A = ê where abcd ¹ 0, then AAT - ATA is
ëc d úû æ xö
equal to (A) F ( xy) (B) F ç ÷
è yø
éb + c d - a ù
(A) (c - b) ê ú (C) F ( x + y) (D) F ( x - y)
ëd - a - b - c û
5. Let a ¹ -1, b ¹ -1, c ¹ -1, be real numbers. If the equa-
éb + c d - a ù
(B) (b - c) ê ú tions a(y + z) = x, b(z + x) = y, and c(x + y) = z has
ëd - a - b - c û non-zero solution, then the value of
éd - a - b - c ù 1
+
1
+
1
(C) (c - d) ê ú is
ëb + c d - a û 1+ a 1+ b 1+ c
(A) 2 (B) 1 (C) 1/2 (D) -2
éa - d b + c ù
(D) (a + b - c - d) ê ú
ëb + c a - dû 6. The value of x Î[0, p / 2] such that the matrix

é 2 sin x - 1 sin x cos x ù


é 1 1ù ê ú
, then å k = 1 Ak is equal to
n
3. If A = ê ú
ë0 1û ê - sin x 2 cos - 3 tan x ú
ê - cos x - tan x 0 úû
n n + 1ù ë
(A) éê
ë0 n úû is skew-symmetric is
(A) p/2 (B) p/3 (C) p/4 (D) p/6
n-1
(B) éê
n ù
ë 0 n - 1úû 7. If w is a non-real cube root of unity and i = -1, then
the value of the determinant
é n(n + 1) ù
n
(C) ê 2 ú 1 w2 1 + i + w2
ê ú
ë0 n û -i -1 -1 - i + w
1-i w -1
2
-1
é (n + 1)(n + 2) ù
ê n+1 ú
(D) 2 is
ê ú
ë 0 n+1 û (A) 1 (B) i (C) w (D) 0
www.jeeneetbooks.in
450 Chapter 8 Matrices, Determinants and System of Equations

8. Let a, b, c be positive and x any real number. Then, bc a2 a2 1 a a2


the value of the determinant
(A) b2 ca b2 (B) 1 b b2
1 1 1 c2 c2 ab 1 c c2
-x 2 -x 2
(a + a )
x
(b + b )
x
(c + c- x )2
x
a b c a a2 a3
(ax - a- x )2 (bx - b- x )2 (cx - c- x )2
(C) b c a (D) b b2 b3
is c a b c c2 c3
(A) 0 (B) (a + b + c)2
(C) (a + b + c)x (D) (a + b + c)- x 29 26 22
14. 25 31 27 equals
a a+d a + 2d 63 54 46
9. If a 2
(a + d) (a + 2d)2 = 0,
2

(A) 122 (B) 132 (C) 0 (D) 1


2a + 3d 2a + 2d 2a + d
then a-b-c 2a 2a
(A) d = 0 (B) d = - a 15. 2b b-c-a 2b is equal to
(C) a = 0 or d = - a (D) d = 0 or d = - a 2c 2c c-a-b

10. Let 1 £ x, y, z £ 9 be integers which are in AP. If x51, (A) abc (a + b + c)3 (B) (ab + bc + ca)(a + b + c)2
y41 and z 31 are three digit numbers, then the value (C) (a + b + c)3 (D) (ab + bc + ca)(a + b + c)
of the determinant
16. If the system of equations
5 4 3
x 51 y 41 z 31 ax + 4 y + z = 0, bx + 3 y + z = 0, cx + 2 y + z = 0
x y z has non-zero solution, then a, b, c are in
is equal to (A) AP (B) GP (C) HP (D) AGP
(A) x + y + z (B) x - y + z (C) xyz (D) 0 17. For a fixed positive integer n, let

11. Which one of the following systems of equations has n! (n + 1)! (n + 2)!
unique solution? D = (n + 1)! (n + 2)! (n + 3)!
(A) 3 x - y + 4z = 3 (B) x + y - 2z = 0 (n + 2)! (n + 3)! (n + 4)!
x + 2 y - 3z = - 2 2 x - 3y + z = 0
Then (D /(n!)3 - 4) is divisible by
6 x + 5 y - 5z = - 3 x - 5 y + 4z = 1
(A) n (B) (n !)2 + 4 (C) n! + 4 (D) n + 4
(C) x+ y+z=9 (D) y + z = 1
2 x + 5 y + 7z = 52 z- x=1 18. If A and B are symmetric matrices of same order,
2x + y - z = 0 x+ y=1 then the matrix AB - BA is
(A) symmetric matrix (B) skew-symmetric matrix
12. The value of the determinant (C) diagonal matrix (D) null matrix
11 11 12
C4 C5 Cm
12 12 13 0 ab2 ac2
C6 C7 Cm + 2 2
13 13 14 19. a b 0 bc2 is equal to
C8 C9 Cm + 4
a c b2 c
2
0
is equal to zero when the value of m is
(A) a3b3c3 (B) 2 a3b3c3
(A) 6 (B) 5 (C) 4 (D) 1
(C) 2(a2 + b2 + c2 )(a + b + c) (D) 4 a2 b2 c2
bc ab ca
20. Which one of the following matrices is non-singular?
13. The determinant ab ca bc is equal to
ca bc ab é1 1 1ù é4 - 5 - 2ù
(A) ê1 - 1 - 5úú
ê (B) êê 5 - 4 2 úú
êë1 2 4 úû êë 2 2 8 úû
www.jeeneetbooks.in
Exercises 451

é2 - 7 -6 ù é - 2 - 10 8 ù é 2 - 10 8 ù
(C) éê
2 5ù
ú (D) êê 3 5 - 2 úú ê - 5 19 - 13ú
(A) ê ú
ê
(B) ê - 5 19 - 13ú
ú
ë 6 15û êë 4 - 2 - 7 úû êë 3 - 7 - 1 úû êë 3 - 7 - 1 úû

é 2 0 - 1ù é 2 - 10 8 ù é - 2 - 10 8 ù
ê ú ê ú ê ú
21. If A = 5 1 0 , then A-1 equals (C) ê - 5 - 19 13 ú (D) ê 5 - 19 13 ú
ê ú
êë 0 1 3 úû êë 3 - 7 - 1úû êë 3 - 7 - 1úû

(A) A2 + 6 A - 11I (B) A2 + 6 A + 11I


24. Value of the determinant
(C) A2 - 6 A + 11I (D) A2 - 6 A - 6 I
1 + a2 - b2 2ab -2b
22. Let w ¹ 1 be a cube root of unity and 2ab 1 - a2 + b2 2a
é1 w w2 ù 2b - 2a 1 - a2 - b2
ê ú
A= êw w2 1ú
is
êw2 1 w úû
ë (A) (1 + a2 + b2 )3 (B) (1 + a3 + b3 )2
Then A-1 = (C) (a2 + b2 )(1 + a2 + b2 )2 (D) 2(1 + a2 + b2 )3

é1 w2 wù é1 1 0ù 25. Let
ê ú
(A) ê w 1 w2 ú (B) êê0 1 1úú
é13 b1 c1 ù
êw2 1 úû êë0 0 1úû
A = êê 5 b2 15úú
ë w

é 1 0 1ù êë x b3 c3 úû
ê ú
(C) ê0 1 0 ú (D) does not exist If the sum of the elements of each row, each column
êë0 0 1úû and each of the diagonals of A are equal, then value
of x is
(A) 9 (B) 10
é 5 3 1ù
ê ú
23. If A = 2 1 3 , then adj A is equal to
(C) 12 (D) cannot be determined
ê ú
êë 1 2 4 úû

Multiple Correct Answer Type Questions


1. Consider the following system of equations: (C) a, b, c are in HP
x - 2 y + z = - 4, x + y + lz = 4, 2 x - y + 2z = 2 (D) x - a is a factor of ax2 + 2bx + c

Which of the following statements are true?


é4 - 5 - 2ù
(A) System has infinitely many solutions when l = 2 ê ú
3. Let A = 5 - 4 2 , then
(B) Unique solution when l ¹ 1 ê ú
êë 2 2 8 úû
(C) Has no solution when l = 1
(A) A is non-singular (B) A-1 does not exist
(D) Unique solution when l = 1
(C) det (adj A) = 0 (D) A is idempotent
2. If the determinant
4. Consider the matrix
a b aa + b
é0 1 2 ù
b c ba + c = 0
A = êê 1 2 3 úú
aa + b ba + c 0
êë 3 1 1 úû
then which of the following may be true?
and the system of equations
(A) a, b, c are in AP
(B) a, b, c are in GP y + 2z + 8 = 0, x + 2 y + 3z + 14 = 0
and 3x + y + z + 8 = 0
www.jeeneetbooks.in
452 Chapter 8 Matrices, Determinants and System of Equations

Then éx 2ù
10. If A = ê ú and | A | = 125, then x may be
3

(A) A is non-singular ë 2 x û
(B) The system has unique solution (A) 5 (B) 3 (C) –5 (D) –3
(C) A is singular
(D) The system has infinitely many solutions éd1 0 0ù
ê
11. Let D = 0 d2 0 úú which of the following are true?
ê
é 1 -1 1ù êë 0 0 d3 úû
ê - 1 0 ú. Then
5. Let A = 2
ê ú (A) D is a symmetric matrix
êë 1 0 0 úû
(B) If d1, d2, d3 ¹ 0, then
(A) A2 = A (B) A3 = I
é1 ù
(C) A2 = I (D) A2 = A-1 ê 0 0ú
ê d1 ú
ê 1 ú
é3 8ù D-1 = ê 0 0ú
6. Let A = ê . Then d2
ë2 1úû ê ú
ê 1ú
ê0 0 ú
é 1 - 8ù ë d3 û
(A) 13 A-1 = A - 4 I (B) adj A = ê ú
ë- 2 3 û (C) Trace of D = d1 + d2 + d3
(C) |adj A| = 13 (D) A3 = A (D) D commutes with every 3 ´ 3 order matrix

7. Let A be a matrix whose elements are real or complex. éi 0ù é0 - 1ù é0 i ù


A matrix is obtained from A whose elements are the 12. Let A = ê ú ,B=ê ú and C = ê ú, where
ë0 -i û ë1 0 û ë i 0û
complex conjugates of the corresponding elements
of A is denoted by A. That is, if A = [aij]m´n, then i = - 1. Then
A = [aij ]m ´ n. In such case (A) A2 = B2 = C 2 = - I (B) - B = A-1 BA
(A) ( A) = A (C) A = - CB-1 (D) C -1 = - B-1 A-1
(B) If l is a scalar, then (lA) = lA
13. Let a > b > c. If the system of equations ax + by + cz = 0,
(C) AB = AB bx + cy + az = 0 and cx + ay + bz = 0 has non-zero solu-
(D) ( AB)T = ( BT )( AT ) tion, then the quadratic equation at2 + bt + c = 0 has
(A) real roots
8. If A is any matrix, then ( A) = ( A ) and we denote
T T
(B) one positive root
T
( A) by A*. Which of the following are true? (C) one positive and one negative root
(A) (A*)* = A (D) non-real roots
(B) (A + B)* = A* + B*
(C) (AB)* = B*A* é 1 -1 1ù
ê ú
14. Let A = 2 - 1 0 . Then
(D) If l is a scalar, then (lA)* = lA* ê ú
êë 1 0 0 úû
9. A square matrix A is called Hermitian or skew-
(A) A3 n = I for all positive integers n
Hermitian according as A* = A or A* = - A where
A* is ( A)T. Which of the following are true? (B) A-1 = A2
(A) In a skew-Hermitian matrix, each principal diag- (C) A is a periodic matrix with least period 3
onal element is either zero or pure imaginary. (D) |adj A| = 1
(B) If A and B are Hermitian matrices and AB = BA,
15. If A and B are square matrices of same order such
then AB is also Hermitian matrix.
that AB = A and BA = B, then
(C) If A and B are Hermitian matrices, then AB - BA
(A) A2 = A and B2 = B
is skew-Hermitian.
(B) A2 = B and B2 = A
(D) If A is Hermitian and i = - 1, then iA is skew- (C) AB = BA
Hermitian. (D) A and B are periodic matrices
www.jeeneetbooks.in
Exercises 453

Matrix-Match Type Questions


In each of the following questions, statements are given 2. Let w ¹ 1 be a cube root of unity and
in two columns, which have to be matched. The state-
ments in Column I are labeled as (A), (B), (C) and (D), éw 0 ù
A=ê ú
while those in Column II are labeled as (p), (q), (r), (s) ë 0 wû
and (t). Any given statement in Column I can have cor-
rect matching with one or more statements in Column II. Match the items of Column I with the items of Column II.
The appropriate bubbles corresponding to the answers
to these questions have to be darkened as illustrated in
the following example. Column I Column II
Example: If the correct matches are (A) ® (p), (s); (B) ® (A) A2 éw2 0ù
(q), (s), (t); (C) ® (r); (D) ® (r), (t); that is if the matches (p) ê ú
ë0 w2 û
are (A) ® (p) and (s); (B) ® (q), (s) and (t); (C) ® (r);
and (D) ® (r), (t); then the correct darkening of bubbles
(q) éê
(B) A3 1 0ù
will look as follows: ú
ë 1û
0

(C) A-1
(r) êé
p q r s t 0 1ù
ú
A ë1 0û
B
(s) êé
(D) A2010 w 0ù
C
ú
D ë 0 wû

1. Column I contains some matrices while Column II 3. w ¹ 1 is a cube root of unity. Column I consists of
contains their corresponding determinants values. some matrices and Column II consists of their corre-
Match them. sponding determinant values. Match them.

Column I Column II Column I Column II

é a b c ù é1 w w2 ù
ê ú
(A) êa - b b - c c - a úú
ê (p) a3 + b3 + c3 - 3abc (A) ê w w2 1ú (p) -3w2
êë b + c c + a a + búû êw2 1 w úû
ë

éb c a ù é1 1 1ù
ê w2 úú
(B) êê a b c úú (q) 3abc - a - b - c
3 3 3
(B) ê1 - 1 - w2 (q) 0
êë c a búû êë1 w2 w4 ûú

é a2 bc ac + c2 ù é 1 + w w2 -w ù
ê 2 ú ê ú
(C) êa + ab b2 ac ú (r) 4 a2 b2 c2 (C) ê 1 + w2 w - w2 ú (r) –3
ê ab b2 + bc c2 úû êw2 + w w - w2 úû
ë ë

éb2 + c2 a2 a2 ù é1 1 wù
ê ú (D) êê 1 1 w2 úú (s) 3w (w - 1)
(D) ê b2 c2 + a2 b2 ú (s) (a + b + c) 3

ê c2 c2 a2 + b2 úû êëw2 w 1 úû
ë
www.jeeneetbooks.in
454 Chapter 8 Matrices, Determinants and System of Equations

4. Match the items of Column II with those of Column II. 5. Match the items of Column I with those in Column II.

Column I Column II Column I Column II

1 yz y + z é0 0 0 ù
(A) 1 zx z + x = (p) (x – y)(y – z)(z – x) (A) The matrix êê0 1 0 úú is (p) Nilpotent matrix
1 xy x + y (xy + yz + zx) êë0 0 1úû

1 x x2 éa h gù
ê f úú is
(B) 1 y y2 = (q) –(x + y + z)(x – y)(y – z) (B) The matrix ê h b (q) Diagonal matrix
1 z z2 (z – x) êë g f c úû

x y + z x2 é 1 - 3 -4ù
(C) y z + x y2 = (r) (x - y)(y - z)(z - x) (C) Matrix êê - 1 3 4 úú is (r) Idempotent matrix
z x + y z2 êë 1 - 3 - 4 úû
x y z (D) If
(D) x2 y2 z2 = (s) (x + y + z)(x - y)(y - z)(z - x) écos a - sin a 0ù
yz zx xy ê
F(a ) = ê sin a cos a 0 úú ,
(s) Symmetric matrix
êë 0 0 1úû
2
æ æ pöö
then the matrix ç F ç ÷ ÷ is
è è 2øø

Comprehension-Type Questions
1. Passage: Let A be a square matrix. Then (A) idempotent matrix
(A) A is called idempotent matrix, if A2 = A. (B) involutory
(B) A is called nilpotent matrix of index k, if Ak = O (C) nilpotent matrix of index 2
and Ak-1 ¹ O. (D) AAT = I.
(C) A is called involutory matrix if A2 = I.
2. Passage: Let A be 3 ´ 3 matrix and B is adj A. Answer
(D) A is called periodic matrix with least periodic k, if the following questions:
Ak + 1 = A and Ak ¹ A.
é0 1 1ù
(i) If A = êê 1 2 0 úú , then A-1 is equal to
Answer the following questions:

0 - 1ù êë 3 - 1 4 úû
(i) The matrix éê ú is
-
ë 1 0û
é 8 -5 -2ù é- 8 5 2 ù
1 ê
- 4 - 3 1 úú
1 ê
(A) idempotent (B) involutory (A) (B) 4 3 - 1úú
11 ê 11 ê
(C) nilpotent (D) skew-symmetric êë - 7 3 - 1 úû êë 7 - 3 1 úû
(ii) If A is an idempotent matrix, then I - A is
é - 7 3 -1ù é- 8 5 2 ù
-1 ê
- 4 - 3 1 úú
1 ê
7 - 3 1 úú
(A) idempotent
(C) ê (D) ê
(B) nilpotent 11 11
êë 8 - 5 - 2 úû êë 4 3 - 1úû
(C) involutory
(D) periodic matrix with least period 4 é4 - 5 - 2ù
(ii) If A = ê 5 - 4 2 úú , then adj B is equal to
ê
é 1 -3 - 4ù
êë 2 2 8 úû
(iii) The matrix A = êê - 1 3 4 úú is
êë 1 - 3 - 4 úû
www.jeeneetbooks.in
Exercises 455

(A) 0 (B) I é5ù


(A) [5]1´ 1 (B) ê ú
é - 36 36 - 18 ù é - 36 - 36 18 ù ë 2 û1´ 1
(C) êê - 36 36 18 úú (D) êê 36 36 - 18 úú é3ù
êë 18 - 18 0 úû êë 18 - 18 (C) [4]1´ 1 (D) ê ú
0 úû ë 2 û1´ 1
(iii) If det A ¹ 0, then B-1 is
(A) A (B) | A | A 4. Passage: Let X1, X2 and X3 be column matrices such that
-1
(C) A (D) A é 1 2 3ù é6 ù é 1 2 3ù
| A| | A| ê 2 4 1ú X = ê 7 ú , ê 2 4
ê ú 1 ê ú ê 1úú
é1 0 0ù êë 3 2 1úû êë 6 úû êë 3 2 9 úû
ê ú
3. Passage: Let A = 2 1 0 and X1 , X2 , X3 be column
ê ú é6ù é1 1 1 ù é6 ù
êë 2 2 1úû X2 = êê17 úú and ê 1 -1 1 ú X = ê2 ú
ê ú 3 ê ú
matrices such that
êë 2 úû êë 2 1 - 1úû êë 1 úû
é 1ù é2ù é2ù A is the 3 ´ 3 matrix whose first, second and third
AX1 = êê0 úú , AX2 = êê 3 úú and AX3 = êê 3 úú rows are respectively X1T, X2T and X3T . Answer the
êë0 úû êë 0 úû êë 1 úû following questions:

Let X be the 3 ´ 3 matrix whose first, second and third (i) det A is equal to
columns are, respectively, X1 , X2 and X3. Answer the (A) 2 (B) 0 (C) –8 (D) 8
following questions: (ii) Sum of all the elements of A is
(i) The value of det(X ) is (A) an even number
(A) 3 (B) –3 (C) 3/2 (D) 2 (B) a number of the form 4k + 3
-1 (C) a prime number
(ii) The sum of all the elements of X is
(A) –1 (B) 0 (C) 1 (D) 3 (D) a perfect square of an integer
(iii) tr (adj A) is
é3ù
ê ú (A) even number
(iii) The matrix [3 2 0]1´ 3 X ê 2 ú is
ê ú (B) number of the form 3k + 2
ë0 û (C) perfect cube of an integer
(D) a prime number

Assertion–Reasoning Type Questions


In each of the following, two statements, I and II, are Statement II: The determinant of a skew-symmetric
given and one of the following four alternatives has to matrix of odd order is zero.
be chosen.
(A) Both I and II are correct and II is a correct reason- 2. Statement I: The system of equations x + y + z = 4,
ing for I. 2x – y + 2x = 5, x – 2y – z = –3 has unique solution.
(B) Both I and II are correct but II is not a correct reason- Statement II: If A is a 3 ´ 3 matrix and B is a 3 ´ 1
ing for I. non-zero column matrix, then the equation AX = B
(C) I is true, but II is not true. has unique solution if A is non-singular.
(D) I is not true, but II is true.
é1 0 0ù
é0 -1 - 2 ù ê ú
3. Statement I: The matrix ê 0 0 1 ú is an idempotent
ê 3 úú
1. Statement I: The determinant of the matrix 1 0
ê êë0 1 0 úû
êë 2 - 3 0 úû matrix.
is zero.
Statement II: If A is an idempotent matrix, then A4 = A.
www.jeeneetbooks.in
456 Chapter 8 Matrices, Determinants and System of Equations

4. Statement I: The inverse of the matrix in the above 5. Statement I: If A and B are symmetric matrices
problem 3 is itself. of same order, then AB + BA is symmetric and
AB - BA is skew-symmetric.
Statement II: The inverse of any idempotent matrix
is itself. Statement II: If P and Q are matrices of same order,
then ( P ± Q)T = PT ± QT and if P, Q are compatible
for multiplication, then ( PQ)T = QT PT .

Integer Answer Type Questions


The answer to each of the questions in this section is a 2. Let Z1, Z2, Z3, be non-zero complex numbers and
non-negative integer. The appropriate bubbles below the éa b c ù
|Z1| = a, |Z2| = b, |Z3| = c. If the matrix êêb c a úú is
respective question numbers have to be darkened. For
example, as shown in the figure, if the correct answer to
the question number Y is 246, then the bubbles under Y êë c a búû
labeled as 2, 4, 6 are to be darkened. singular and D is the area of the triangle whose verti-
ces are at Z1 , Z2 and Z3 and R is its circumradius, then
X Y Z W 4 D is equal to .
0 0 0 0 R2 3
1 1 1 1
2 2 2 3. Let S be the set of all 2 ´ 2 matrices whose elements
are 0 or 1. Then the number of non-singular matrices
3 3 3 3
belonging to S is .
4 4 4
5 5 5 5 4. If A is 3 ´ 3 matrix and |A| = 2, then |adj(adj A)| is
6 6 6 .
7 7 7 7
8 8 8 8 é0 1ù écos a - sin a ù
5. If A = ê ú and B = ê sin a , then det (A + B)
9 9 9 9 ë1 0û ë cos a úû
is .
é1 2 3ù
ê
1. If A = 1 2 3 úú , then the least positive integer k
ê
êë - 1 - 2 - 3úû
such that Ak = 0 is .

ANSWERS
Single Correct Choice Type Questions
1. (D) 14. (B)
2. (B) 15. (C)
3. (C) 16. (A)
4. (C) 17. (A)
5. (A) 18. (B)
6. (D) 19. (B)
7. (D) 20. (D)
8. (A) 21. (C)
9. (D) 22. (D)
10. (D) 23. (A)
11. (C) 24. (A)
12. (B) 25. (C)
13. (A)
www.jeeneetbooks.in
Answers 457

Multiple Correct Choice Type Questions


1. (B), (C) 9. (A), (B), (C), (D)
2. (B), (D) 10. (B), (D)
3. (B), (C) 11. (A), (B), (C)
4. (A), (B) 12. (A), (B), (C), (D)
5. (B), (D) 13. (A), (B)
6. (A), (B), (C) 14. (A), (B), (C), (D)
7. (A), (B), (C), (D) 15. (A), (D)
8. (A), (B), (C), (D)

Matrix-Match Type Questions


1. (A) ® (p), (B) ® (p), (C) ® (r), (D) ® (r) 4. (A) ® (r), (B) ® (r), (C) ® (q), (D) ® (p)
2. (A) ® (p), (B) ® (q), (C) ® (p), (D) ® (q) 5. (A) ® (q), (r) , (s) (B) ® (s), (C) ® (p), (D) ® (q),(s)
3. (A) ® (q), (B) ® (s), (C) ® (p), (D) ® (r)

Comprehension-Type Question
1. (i) (B); (ii) (A); (iii) (C) 3. (i) (A); (ii) (B); (iii) (A)
2. (i) (B); (ii) (A); (iii) (C) 4. (i) (D); (ii) (C); (iii) (D)

Assertion–Reasoning Type Questions


1. (A) 4. (C)
2. (A) 5. (A)
3. (D)

Integer Answer Type Questions


1. 2 4. 16
2. 3 5. 0
3. 6
www.jeeneetbooks.in
www.jeeneetbooks.in

Partial Fractions
9
Contents
9.1 Rational Fractions
9.2 Partial Fractions

Worked-Out Problems
Summary
Exercises
Answers

The partial fraction decom-


position or partial fraction
–2 3 1 – 2x expansion is used to reduce
x+1 (x + 1)2 x 2+2x +1
the degree of either the
numerator or the denomi-
nator of a rational function.
The partial fraction decom-
partial fractions position may be seen as the
Partial Fractions

inverse procedure of the


more elementary operation
of addition of fractions, that
produces a single rational
fraction with a numerator
and denominator usually of
high degree.
www.jeeneetbooks.in
460 Chapter 9 Partial Fractions

It is well known that a polynomial in x is an expression of the form a0 + a1 x + + an xn, where a0, a1, ¼, an are real
numbers or complex numbers. Polynomials are usually denoted by the symbols f (x), g(x), etc. The degree of a
polynomial f (x) = a0 + a1 x + + an xn is defined to be n if an ¹ 0. The ai are called the coefficients of xi in f (x). A poly-
nomial f (x) is said to be the zero polynomial if each ai = 0. A zero polynomial can have any degree and every non-zero
real (complex) number is considered to be a polynomial of degree zero. Zero polynomial is denoted by usual 0 (zero).
If two polynomials f (x) and g(x) are equal, then we write f (x) = g(x). Two polynomials are said to be equal if their
corresponding coefficients are equal. We are familiar with adding and multiplying two polynomials. In this chapter, we
confine to polynomials whose coefficients are all real numbers.

9.1 | Rational Fractions


DEF IN IT ION 9 . 1 An expression of the form f (x) /g(x), where f (x) and g(x) are polynomials and g(x) ¹ 0, is
called a rational fraction.

DEF IN IT ION 9 . 2 A rational fraction f (x) /g(x) is called a proper fraction if either f (x) = 0 or degree of f (x) is less
than degree of g(x). If deg f (x) ³ deg g(x), then f (x) /g(x) is called an improper fraction.

Examples

1+ x 2 + 3x + 4 2 1 Note that 3/2 is an improper fraction, since 3 and 2 are


(1) , , , 0 and are both polynomials of degree 0.
1 + x + x 2 3 + 2 x + 3 x2 1 + x 1 + x2
all proper fractions.

1 + x + x2 1 + 2 x + x2 3 1 + x8
(2) , , , and are all
3 + 2 x + 4 x2 2+ x 2 1 + x7
improper fractions.

T H E O R E M 9 .1 Let f (x) /g(x) be a rational fraction. Then there exist unique polynomials q (x) and r(x) such that
f ( x) r ( x)
= q( x) +
g( x) g( x)

and r(x) /g(x) is a proper fraction.

PROOF Since f (x) /g(x) is a rational fraction, f (x) and g(x) are polynomials and g(x) ¹ 0. If f (x) /g(x) is a
proper fraction, then we can take q(x) = 0 and r(x) = f(x). Suppose that f (x) /g(x) is an improper
fraction. Then deg f (x) ³ deg g(x).
Let
f ( x) = a0 + a1 x + + am xm , am ¹ 0
and g( x) = b0 + b1 x + + bn xn , bn ¹ 0
Then deg f (x) = m ³ n = deg g(x). We shall apply induction on m.
If m = 0, then n = 0 and f (x)/g(x) = a0 /b0, which is a real number. In this case, we can take
q(x) = a0 /b0 and r(x) = 0. Let m > 0 and suppose that the theorem is true for all rational fractions
h (x)/g(x) with deg (h(x)) < m. Then put
am m- n
h( x) = f ( x) - x g( x)
bn

It can be easily seen that deg h (x) £ m - 1 < m and therefore, by the induction hypothesis, we can
write
h( x) r( x)
= q1 ( x) +
g( x) g( x)
www.jeeneetbooks.in
9.1 Rational Fractions 461

for some polynomials q1(x) and r(x) such that r(x)/g(x) is a proper fraction. Now, consider

f ( x) h( x) + am bn- 1 xm- n g( x)
=
g( x) g( x)
- 1 m- n h( x)
= am bn x +
g( x)
- 1 m- n r( x)
= am bn x + q1 ( x) +
g( x)
r( x)
= q( x) +
g( x)

where q(x) = ambn-1xm-n + q1(x) and r(x)/g(x) is a proper fraction. Therefore, we have proved that
there exist polynomials q(x) and r(x) such that

f ( x) r( x)
= q( x) +
g( x) g( x)

and r(x)/g(x) is a proper fraction.


To prove uniqueness of q(x) and r(x), suppose q¢(x) and r¢(x) are also polynomials such that

f ( x) r ¢( x )
= q ¢( x ) +
g( x) g( x)

where r¢(x)/g(x) is a proper fraction. Then

q(x) g(x) + r(x) = f(x) = q¢(x) g(x) + r¢(x)

and hence

[q(x) - q¢(x)] g(x) = r¢(x) - r(x)

If r(x) - r¢(x) ¹ 0, then

deg [r¢(x) - r(x)] = deg [{q(x) - q¢(x)} g(x)] ³ deg g(x)

which is a contradiction, since deg r(x) < deg g(x) and deg r¢(x) < deg g(x). Therefore r¢(x) - r(x) = 0
and hence [q(x) - q¢(x)] g(x) = 0, so that q(x) - q¢(x) = 0 (since g(x) ¹ 0). Thus

q(x) = q¢(x) and r(x) = r¢(x)

The unique polynomials q(x) and r(x) found above are called the quotient and the remainder,
respectively, and the algorithm to find q(x) and r(x) is called the division algorithm for polynomials.
Note that q(x) and r(x) are unique polynomials satisfying the property

f (x) = q(x) g(x) + r(x)

such that r(x) = 0 or deg r(x) < deg g(x).


The reader might be familiar with the algorithm to find the quotient and remainder. If r(x) = 0
in the above, then we say that g(x) divides f (x). Further, any polynomial f (x) of degree greater
than one can be uniquely expressed as

f (x) = q(x)(x - a) + r

for some real number r, where a is a given real number, and therefore, we have r = f (a) so that

f (x) = q(x)(x - a) + f (a)

Also, x - a divides f (x) if and only if f (a) = 0; this is popularly known as the factorization
theorem. ■
www.jeeneetbooks.in
462 Chapter 9 Partial Fractions

Examples

1 + x2 2 1 + x + x2 1
(1) = (-1 + x) + (2) =x+
1+ x 1+ x 1+ x 1+ x

C O R O L L A RY 9.1 Let q(x), q¢(x), r(x), r¢(x), g(x) and g¢(x) be polynomials in x such that r(x)/g(x) and r¢(x)/g¢(x) are
proper fractions and
r( x) r ¢( x )
q( x) + = q ¢( x ) +
g( x) g ¢( x )

Then
r ( x ) r ¢( x )
q( x) = q ¢( x) and =
g ( x ) g ¢( x )

DEF IN IT ION 9 . 3 Two polynomials f (x) and g(x) are said to be relatively prime (or prime to each other) if there
is no polynomial of positive degree dividing both f (x) and g(x).

Examples

(1) The polynomials 1 + x and 1 - x are relatively prime, (2) 1 + 2x + x2 and 1 - x2 are not relatively prime since
since any common divisor of these must divide their 1 + x is a common divisor of these.
sum, which is a polynomial of degree 0.

We assume the following theorem whose proof is beyond the scope of this book.

T H E O R E M 9 .2 Two polynomials f (x) and g (x) are relatively prime if and only if there exist polynomials p(x) and
q(x) such that
f (x) p(x) + g(x) q(x) = 1

9.2 | Partial Fractions


In this section we discuss several methods of expressing a rational fraction as a sum of similar fractions; such a rational
fraction is known as partial fraction. First, we have the following main result.

T H E O R E M 9 .3 Let f (x), g(x) and h(x) be polynomials such that h(x) /f (x) g(x) is a proper fraction. Suppose
(F U N D A M E N TA L that f (x) and g(x) are relatively prime. Then there exist proper fractions a(x)/f (x) and b(x)/g(x)
THEOREM) such that
h( x) a ( x) b ( x)
= +
f ( x) g( x) f ( x) g( x)

PROOF Since f (x) and g(x) are relatively prime, there exist polynomials p(x) and q(x) such that
f (x) q(x) + g(x) p(x) = 1
Now,
h( x) h( x)[ f ( x) q( x) + g( x) p( x)] h( x) p( x) h( x) q( x)
= = +
f ( x) g( x) f ( x) g( x) f ( x) g( x)
www.jeeneetbooks.in
9.2 Partial Fractions 463

If h(x)p(x)/f (x) and h(x)q(x)/g(x) are not proper fractions, then by Theorem 9.1, we can write
h( x) p( x) a ( x) h( x)q( x) b ( x)
= t( x) + and = s( x) +
f ( x) f ( x) g( x) g( x)

for some polynomials t(x), s(x), a(x) and b(x) such that a(x)/f (x) and b(x)/g(x) are proper fractions
and, therefore, we have
h( x) h( x) p( x) h( x)q( x) a ( x) b ( x) a ( x)g( x) + b ( x) f ( x)
= + = s( x) + t( x) + + = s( x) + t( x) +
f ( x) g( x) f ( x) g( x) f ( x) g( x) f ( x)g( x)

Since deg a (x) < deg f (x) and deg b(x) < deg g(x), we get that deg [a(x)g(x) + b(x)f (x)] < deg [ f(x)
g(x)] which implies that [a (x)g(x) + b(x)f (x)]/ f(x) g(x)] is a proper fraction. Hence h(x)/f(x)g(x)
and therefore
h( x) a ( x) b ( x)
s( x) + t( x) = 0 and = +
f ( x)g( x) f ( x) g( x) ■

In the following theorems, we will prove that a proper fraction can be resolved into sum of its simplest partial
fractions, in various cases. First, we have the following.

DEF IN IT ION 9 . 4 If a proper fraction is expressed as the sum of two or more proper fractions, then each of these
is called a partial fraction of the given proper fraction. The process of finding partial fractions
of a given proper fraction is known as resolving the proper fraction into partial fractions.

Example 9.1

Find the partial fractions of (b) The given fraction can be simplified as
1 2 1 -1
(a) 6 - 5 x + x2 = +
x2 - 1 x - 1 x + 1
2 Therefore
(b) x2 - 1
1 -1
and
Solution: x-1 x+1
(a) The given fraction can be simplified as
are partial fractions of 2/(x2 - 1).
1 1 1 -1
= = +
6 - 5x + x2
( x - 3)( x - 2) x - 3 x - 2
Therefore
1 -1
and
x-3 x-2

are called the partial fractions of the given fraction.

DEF IN IT ION 9 . 5 A polynomial of positive degree is said to be irreducible if it cannot be expressed as a product
of two or more polynomials of positive degree.

Examples

(1) Any polynomial of degree 1 is irreducible (such (2) 1 + x + x2 is an irreducible polynomial.


polynomials are called linear polynomials).
www.jeeneetbooks.in
464 Chapter 9 Partial Fractions

Example 9.2

Show that the following polynomials are not irreducible. Solution:


(a) 1 + x + x2 + x3 (a) 1 + x + x2 + x3 is not an irreducible polynomial, since
(b) x3 - 6x2 + 11x - 6 (1 + x)(1 + x2) = 1 + x + x2 + x3.
(b) x3 - 6x2 + 11x - 6 is not an irreducible polynomial,
since x3 - 6x2 + 11x - 6 = (x - 3)(x - 2)(x - 1).

T H E O R E M 9 .4 Let f (x)/g(x) be a proper fraction and ax + b a non-repeated factor of g(x). Then f (x)/g(x) has a
partial fraction of the form
A
ax + b
where A is a constant.
PROOF Since ax + b is a non-repeated factor of g(x), we can write
g(x) = (ax + b)h(x)

where ax + b and h(x) are relatively prime. Then by the fundamental theorem (Theorem 9.3), we
can write
f ( x) a ( x) b ( x)
= +
g( x) ax + b h( x)

where a(x)/(ax + b) and b(x)/h(x) are proper fractions.


In particular, deg a(x) < deg (ax + b) = 1 and hence a(x) is a constant, say A. Therefore
A f ( x)
is a partial fraction of
ax + b g( x) ■

T H E O R E M 9 .5 If f (x)/g(x) is a proper fraction and ax2 + bx + c (a ¹ 0) is a non-repeated irreducible factor of g(x),


then f (x)/g(x) has a partial fraction of the form
Ax + B
ax2 + bx + c
PROOF Same as in the above theorem. ■

In the following example, we demonstrate a method of finding partial fractions.

Example 9.3

Resolve the following fraction into partial fractions: Then


3x + 2 3x + 2 A (2 x + 1) + B( x - 3)
=
( x - 3)(2 x + 1) ( x - 3)(2 x + 1) ( x - 3)(2 x + 1)

Solution: Write
Therefore
3x + 2 A B
= + 3x + 2 = A(2x + 1) + B(x - 3)
( x - 3)(2 x + 1) x - 3 2 x + 1
www.jeeneetbooks.in
9.2 Partial Fractions 465

By taking x = 3, we get Similarly, by taking x = -1/2, we get B = -1/7. Therefore


3 × 3 + 2 = A(2 × 3 + 1) + B(3 - 3) 3x + 2 11 / 7 -1/ 7 11 1
= + = -
11 ( x - 3)(2 x + 1) x - 3 2 x + 1 7( x - 3) 7(2 x + 1)
A=
7

T H E O R E M 9 .6 Let f (x)/(ax + b)n be a proper fraction. Then, there exist constants A1, A2, ¼, An such that
f ( x) A1 A2 An
= + + +
(ax + b)n ax + b (ax + b)2 (ax + b)n

PROOF This is a repeated application of Theorem 9.1. Since f (x)/(ax + b)n is a proper fraction, deg f (x) < n.
First use Theorem 9.1 to write
f ( x) r ( x)
= q1 ( x) + 1 (9.1)
ax + b ax + b
where q1(x) and r1(x) are some polynomials and deg r1(x) < deg (ax + b) = 1. Therefore r1(x) is a
constant. Put B1 = r1(x). Again, we can write (using Theorem 9.1),
q 1( x) B2
= q 2 ( x) + (9.2)
ax + b ax + b
where q2(x) is a polynomial and B2 is a constant. From Eqs. (9.1) and (9.2), we get that

f ( x) q ( x) r1 ( x) B2 B1
= 1 + = q 2 ( x) + +
(ax + b) 2
ax + b (ax + b) 2
ax + b (ax + b)2

Again,

q 2 ( x) B3
= q3 ( x) +
(ax + b) ax + b

for some polynomial q3(x) and a constant B3. Substituting this in the above, we get

f ( x) B3 B2 B1
= q 3( x) + + +
(ax + b) 3
ax + b (ax + b)2
(ax + b)3

The process can be performed n times to get


f ( x) Bn Bn - 1 B1
= qn ( x) + + + +
(ax + b) n
ax + b (ax + b) 2
(ax + b)n

where B1, B2, ¼, Bn are constants. Since deg f (x) < n, we get qn(x) = 0. Now, put Ai = Bn-i+1 to get
f ( x) A1 A2 An
= + + +
(ax + b)n ax + b (ax + b)2 (ax + b)n ■

Example 9.4

Resolve the following fraction into partial fractions 1 + x + x2 A1 A2 A3


= + + (9.3)
1 + x + x2 ( x + 2)3 x + 2 ( x + 2)2 ( x + 2)3
( x + 2)3
We have to find the values of the constants A1, A2, and
Solution: By Theorem 9.6, we can write A3. From Eq. (9.3), we have
www.jeeneetbooks.in
466 Chapter 9 Partial Fractions

1 + x + x2 = A1(x + 2)2 + A2(x + 2) + A3 (9.4) and 0 = 9A1 + 3A2 or 3A1 + A2 = 0


By substituting –2 for x, we get By solving these two, we get that A1 = 1 and A2 = -3 from
Eq. (9.3), we have
1 + (-2) + (-2)2 = 0 + 0 + A3
1 + x + x2 1 3 3
A3 = 3 = - +
( x + 2) 3
( x + 2) ( x + 2)2
( x + 2) 3
Taking x = 0 and x = 1 in Eq. (9.4), we get
-2 = 4A1 + 2A2 or 2A1 + A2 = -1

Example 9.5

Resolve the following into partial fractions: Then

1 + x + x2 + x3 1 + x2 = A1(1 - x)2 + A2(1 + x)(1 - x) + A3(1 + x)


(1 + 2 x + x2 )(1 - 2 x + x2 ) (a) Taking x = 0, A1 + A2 + A3 = 1
Solution: First, note that (1 + x)(1 + x ) = 1 + x + x + x .
2 2 3 (b) Taking x = 1, 2A3 = 2 or A3 = 1
Therefore, the given fraction is (c ) Taking x = -1, 4A1 = 2 or A1 = 1/2
Therefore
(1 + x)(1 + x2 ) 1 + x2
= 1 1
(1 + x)2 (1 - x)2 (1 + x)(1 - x)2 A2 = 1 - A1 - A3 = 1 - -1= -
2 2
Let
and so
1 + x2 A1 A2 A3
= + + 1 + x + x2 + x3 1 1 1
(1 + x)(1 - x) 2
1 + x 1 - x (1 - x) 2 = - +
(1 + 2 x + x )(1 - 2 x + x ) 2(1 + x) 2(1 - x) (1 - x) 2
2 2

Example 9.6

Resolve the following into partial fractions: Put x = 4. Then C = -2. Equating coefficient of x2 on both
sides of Eq. (9.5), we get
42 - 19 x
( x - 4)( x 2 + 1) 0=A+CÞA=2
Equating the coefficient of x on both sides of Eq. (9.5),
Solution: Let
we get
42 - 19 x Ax + B C
= 2 + -4A + B = -19 so that B = -11
( x - 4)( x 2 + 1) x +1 x-4
Therefore
Therefore
42 - 19 x 2 x - 11 2
42 - 19x = (Ax + B)(x - 4) + C(x2 + 1) (9.5) = 2 -
( x - 4)( x 2 + 1) x +1 x-4

WORKED-OUT PROBLEMS
Single Correct Choice Type Questions
1. If then A + B is equal to
mx + n A B (A) m (B) n (C) m + n (D) mn
= +
( x - a)( x + b) x - a x + b Solution: We have
mx + n = A(x + b) + B(x - a)
www.jeeneetbooks.in
Worked-Out Problems 467

Now Now
ma + n x = 1 Þ A = -3
x=aÞ A=
a+b
1
x= ÞB=4
- mb + n mb - n 2
x = -bÞ B = =
-(a + b) a+b Answer: (A)
Adding we get
4. Let
m(a + b)
A+ B= =m 9 A B C
a+b = + +
( x + 1)( x - 2) 2 x + 1 x - 2 ( x - 2) 2
Answer: (A)
then the ordered triple (A, B, C) is
2. If (A) (1, 3, -1) (B) (1, -1, 3)
7x - 1 A B (C) (3, -1, 1) (D) (1, 1, -3)
= +
6 x - 5x + 1 3x - 1 2 x - 1
2
Solution: We have
then B – A equals 9 = A(x - 2)2 + B(x + 1)(x - 2) + C(x + 1)
(A) 1 (B) 7 (C) 8 (D) 9
Now for
Solution: We have
x = -1 Þ A = 1
7x - 1 = A(2x - 1) + B (3x - 1) x=2ÞC=3
Now Now A + B = Coefficient of x2 = 0. Therefore B = -1 and so
1 7 æ2 ö A = 1, B = -1, C = 3
x= Þ - 1 = A ç - 1÷ Þ A = - 4
3 3 è3 ø
Answer: (B)
1 7 æ3 ö
x= Þ - 1 = B ç - 1÷ Þ B = 5 5. Suppose
2 2 è2 ø

Therefore 3 x3 - 8 x 2 + 10 a b c d
= + + +
( x - 1) 4
x - 1 ( x - 1) 2
( x - 1) 3
( x - 1) 4
B−A=5+4=9
Answer: (D) Then a + b + c + d is equal to
(A) 1 (B) 2 (C) 4 (D) –5
3. If
Solution: Put x - 1 = y. Therefore we have
2 x2 + 3 x + 1 A B
= + 3( y + 1)3 - 8( y + 1)2 + 10
(1 - 2 x)(1 - x ) 1 - x 1 - 2 x
2

y4
then A, B are, respectively,
3( y3 + 3 y2 + 3 y + 1) - 8( y2 + 2 y + 1) + 10
(A) -3, 4 (B) -3, -4 (C) 3, 4 (D) 3, -4 =
y4
Solution: Clearly 3 y3 + y2 - 7 y + 5 5 7 1 3
4
= 4 - 3 + 2 +
2 x2 + 3 x + 1 (2 x + 1)( x + 1) y y y y y
=
(1 - 2 x)(1 - x)(1 + x) (1 - 2 x)(1 - x)(1 + x) 5 7 1 3
= - + +
( x - 1)4
( x - 1)3
( x - 1)2
x-1
2x + 1
= Answer: (B)
(1 - 2 x)(1 - x)

Therefore 6. If

2x + 1 = A(1 - 2x) + B(1 - x) x2 + 4 A B


= 2 + 2
( x + 1)(2 x + 3) x + 1 2 x + 3
2 2
www.jeeneetbooks.in
468 Chapter 9 Partial Fractions

then A - B is a3
(A) 6 (B) 8 (C) 9 (D) 11 å (a - b)(a - c)(a - d)
Solution: Put y = x in the given expression. We get the
2
where d is any real number not equal to a, b and c is
fraction
(A) 0 (B) 1
y+4 A B 1 1 1 1
= + (C) a + b + c + d (D) + + +
( y + 1)(2 y + 3) y + 1 2 y + 3 a b c d
y + 4 = A(2 y + 3) + B( y + 1) Solution: We have

Solving we get x3 A B C
=1+ + +
( x - a)( x - b)( x - c) x-a x-b x-c
y = -1Þ A = 3
x3 = ( x - a)( x - b)( x - c) + A( x - b)( x - c)
-3
y= Þ B = -5 + B( x - a)( x - c) + C ( x - a)( x - b)
2
Therefore A – B = 3 – (–5) = 8. Therefore
Note that the substitutions y = -1 and -3/2 are for
a3 b3 c3
the fraction in y, but not for the fraction in x. A= ,B= ,C =
(a - b)(a - c) (b - a)(b - c) (c - a)(c - b)
Answer: (B)
which gives
7. Consider the series
x3 A
1 x x2 = 1+
+ + + ( x - a)( x - b)( x - c) ( x - a)(a - b)(a - c)
(1 + x)(1 + x2 ) (1 + x2 )(1 + x3 ) (1 + x3 )(1 + x4 )
B C
+ +
If x > 1, then sum to infinity of the series is ( x - b)(b - a)(b - c) ( x - c)(c - a)(c - b)
1 1
(A) 1 - x2 (B) x2 - 1 Put x = d on both sides. We get

a3
(C)
1
(D)
1 å (a - b)(a - c)(a - d) = 1
x (1 - x2 ) x ( x2 - 1)
Solution: Let Answer: (B)

xk - 1 9. When x4/[(x - a)(x - b)(x - c)] is resolved into partial


uk =
(1 + x )(1 + xk + 1 )
k
fractions and d is any real number different from a, b
and c, then
1 é 1 1 ù
= ê - ú
x( x - 1) ë 1 + xk
1 + xk + 1 û a4
å (a - b)(a - c)(a - d) =
Let sn be the sum to n terms of the series. Then
(A) 1 (B) 0
n
1 é 1 1 ù
sn = å uk = ê - ú (C) a + b + c + d (D) abc + abd + acd + bcd
k =1 x ( x - 1) ë 1 + x 1 + xn + 1 û
Solution: We have
Therefore
x4 A B
1 = ( x + a + b + c) + +
Lt sn = ( x - a)( x - b)( x - c) x-a x-b
n®¥ x( x - 1)
2

C
Answer: (D) +
x-c
8. When Therefore as in the above, we have
x3 a4
( x - a)( x - b)( x - c) å (a - b)(a - c)(a - d) = a + b + c + d
is resolved into partial fractions, then Answer: (C)
www.jeeneetbooks.in
Worked-Out Problems 469

Multiple Correct Choice Type Questions


1. If Solution: Simplifying the given expression we get
13 x + 46 A B 3 - 2 x2 = A(1 - x)(2 - x)2 + B(2 - x)2
= +
12 x2 - 11x - 15 3 x - 5 4 x + 3
+ C (2 - x)(1 - x)2 + D(1 - x)2
then
(A) A = -5 ( B) B = 7 Now for
(C ) A = 7 (D) B = -5 x=1ÞB=1
Solution: We have x = 2 Þ D = -5
13x + 46 = A(4x + 3) + B(3x - 5)
0 = coefficient of x3 = A + C (9.6)
Now
x = 0 Þ 4A + 4B + 2C + D = 3
5 æ 29 ö 203 Þ 4A + 4 + 2C -5 = 3
x = Þ Aç ÷ = Þ A=7
3 è 3ø 3
Therefore
-3 æ -29 ö 145
x= Þ Bç = Þ B= -5
4 è 4 ÷ø 4 2A + C = 2 (9.7)
Answers: (C), (D)
From Eqs. (9.6) and (9.7) we get A = 2, C = -2. Hence
2. If 3 - 2 x2 2 1 2 5
= + - -
2x + 1 A Bx + C (1 - x) (2 - x)
2 2
1 - x (1 - x)2
2 - x (2 - x)2
= + 2
( x - 1)( x + 1) x - 1
2
x +1 Answers: (A), (B), (D)
then
3 3 1 4. If
(A) A = (B) B = , C =
2 2 2 ( x + 1)2 A Bx + C
-3 -3 1 = + 2
(C) A = (D) B = ,C = x( x2 + 1) x x +1
2 2 2
Solution: From the given expression we have then

2x + 1 = A(x2 + 1) + (Bx + C)(x - 1) (A) A = 1 (B) B = 1 (C) C = 2 (D) B = 0


Solution: Simplifying the given expression we get
For x = 1 we get A = 3/2.
Also 0 = coefficienct of x2 = A + B. Therefore (x + 1)2 = A(x2 + 1) + (Bx + C) x

-3 Now
B=
2
x=0ÞA=1
For x = 0
2 = coefficient of x = C
3 1
A -C = 1ÞC = A - 1= - 1= 1 = coefficient of x2 = A + B = 1 + B Þ B = 0
2 2
Answers: (A), (D) Hence

3. If ( x + 1)2 1 2
= + 2
x( x + 1) x x + 1
2
3 - 2 x2 A B C D
= + + +
( x - 3 x + 2)2 1 - x (1 - x)2 2 - x (2 - x)2
2
Answers: (A), (C), (D)
then
5. If
(A) A + B = 3 (B) A + D = -3
(C) B + C = 3 (D) A + B + C + D = -4 3x + 4 A B C
= + +
( x + 1)( x2 - 1) x - 1 x + 1 ( x + 1)2
www.jeeneetbooks.in
470 Chapter 9 Partial Fractions

then 6. If
7
(A) A = ( B) A + B = 0 4x
=
A
+
B
4 x4 + x2 + 1 x2 - x + 1 x2 + x + 1
1 -1
(C ) C = (D) C = then
2 2
Solution: We have (A) A + B = 0 (B) AB = -4
3x + 4 3x + 4 A
= =
A
+
B
+
C (C) A = 2, B = -2 (D) = -1
B
( x + 1)( x2 - 1) ( x + 1)2 ( x - 1) x - 1 x + 1 ( x + 1)2
Solution: We have
3 x + 4 = A( x + 1)2 + B( x2 - 1) + C( x - 1)
4x 4x
Now = 2
x + x + 1 ( x - x + 1)( x2 + x + 1)
4 2

7
x = 1 Þ 7 = 4 A or A= é 1 1 ù
4 = 2ê 2 - 2 ú
-1 ë x - x + 1 x + x + 1 û
x = -1 Þ -2C = 1 or C =
2 Therefore, we get A = 2, B = -2
-7
0 = coefficient of x2 = A + B Þ B = Answers: (A), (B), (C), (D)
4
Answers: (A), (B), (D)

SUMMARY
9.1 Polynomial: If a0, a1, a2, ¼, an are real or complex 9.6 Proper and improper fractions: Let f (x) and g(x)
numbers, a0 ≠ 0 and n is a positive integer, thus an be two polynomials and g(x) ⬅ 0. Then f (x)/g(x) is
expression of the form a0 xn + a1 xn−1 + a2 x n−2 + + an called proper or improper fraction according as the
is called polynomial of degree n. degree of f(x) is less than or greater than the degree
a0, a1, a2, ¼, an are called coefficients of the of g(x).
polynomial. If a0, a1, a2, ¼, an are real numbers, then
the polynomial is called polynomial with real coef- 9.7 Relatively prime polynomials: Two polynomials are
ficients and if a0, a1, a2, ¼, an are complex numbers, said to be relatively prime to each other (or coprime
then it is called polynomial with complex coef- to each other) if they do not have a common factor
ficients. Generally polynomials are denoted by of positive degree.
f(x), g(x), h(x), etc. Two polynomials f(x) and g(x) are relatively
prime to each other if and only if there exist polyno-
9.2 Constant polynomial: Every non-zero number is mials p(x) and q(x) such that
considered as a polynomial of zero degree and it is
f(x) p(x) + g(x) q(x) = 1
called constant polynomial.
9.8 Fundamental theorem: Let f (x), g(x) and h(x) be
9.3 Zero (null) polynomial: A polynomial is called zero polynomials such that h(x)/f (x) g(x) is a proper frac-
polynomial if all of its coefficients are zeros. tion. If f (x) and g(x) are relatively prime to each
other, then there exist proper fractions p (x)/f (x)
9.4 Degree of a zero polynomial: Any positive integer can and q (x)/g(x) such that
be considered to be the degree of zero polynomial.
h (x) p (x) q (x)
= +
9.5 Division algorithm (or Euclid’s algorithm): If f (x) f (x) g (x) f (x) g (x)
and g(x) are two polynomials and g(x) ⬅ 0, then there
exist unique polynomials q(x) and r(x) such that 9.9 Partial fractions: If a proper fraction of two poly-
f (x) = q(x) g(x) + r(x) where either r(x) º 0 (i.e., zero nomials is expressed as sum of two or more proper
polynomial) or degree of r(x) is less than the degree fractions, then each of these proper fractions
of g(x). If r(x) º 0, then g(x) is called factor of f(x). is called partial fraction of the given proper
fraction.
www.jeeneetbooks.in
Exercises 471

9.10 Irreducible polynomial: A polynomial of positive is a partial fraction of f (x)/g(x) for some real
degree is said to be irreducible if it cannot be constant A and B.
expressed as a product of two or more polynomials (3) If g(x) has a repeated linear factor of the form
of positive degrees. (ax + b)n, then

9.11 Cases of partial fractions: Let f (x)/g(x) be a A1 A2 An


, , ¼,
proper fraction [i.e., degree of f(x) is less than the ax + b (ax + b)2 (ax + b)n
degree of g(x)]. Then
(1) If g(x) has a non-repeated linear factor ax + b, where A1, A2, ¼, An are constants occur as partial
then A/ax + b is a partial fraction of f (x)/g(x) fractions in f (x)/g(x).
where A is a constant which can be determined. (4) If (ax2 + bx + c)k, where ax2 + bx + c is irreduc-
(2) If g(x) has a non-repeated irreducible quadratic ible, is a repeated factor of g(x), then the frac-
factor ax2 + bx + c. Then tions of the form

Ax + B A1 x + B1 A2 x + B2 Ak x + Bk
, , ¼,
ax 2 + bx + c ax 2 + bx + c (ax 2 + bx + c)2 (ax 2 + bx + c)k

occur as partial fraction for f (x)/g(x).

EXERCISES
Single Correct Choice Type Questions
1. If then the ordered triple (A, B, C) is
5x + 6 A B (A) (2, 3, 1) (B) (1, 2, 3)
= + (C) (0, 2, 3) (D) (2, 3, 0)
( x + 2)( x - 1) x + 2 x - 1

then A + B is equal to 5. If
(A) 4 (B) 3 (C) 5 (D) –5 x2 + 5 1 A
= 2 + 2
( x + 2)
2 2
x + 2 ( x + 2)2
2. If
then
x3 A B
= ( x - 1) + + (A) A = 3 (B) 2A = 5
( x - 1)( x + 2) x-1 x+2
(C) 3A = 1 (D) A = -1
then A + B is
(A) –3 (B) 3 (C) 5 (D) –5 6. Let

x2 + 5 x + 1 a b
3. If = +
( x + 1)( x + 2)( x + 3) x + 1 ( x + 2)( x + 1)
x3 Ax + B Cx + D c
= 2 + 2 +
( x + 1)
2 2
x +1 ( x + 1) 2 ( x + 1)( x + 2)( x + 3)

then A - C is éa b ù
and P=ê ú
(A) 3 ë c -1û
(B) 1 then P is
(C) an even prime number (A) idempotent matrix (B) involuntary matrix
(D) an odd prime number of the form 4n + 1 (C) symmetric matrix (D) scalar matrix
4. If 7. If
5 x2 + 2 A Bx + C 3x 2 1
= + 2 = +
x3 + x x x +1 ( x - a)( x - b) x - a x - b
www.jeeneetbooks.in
472 Chapter 9 Partial Fractions

then the relation between a and b is then


(A) a = 2b (B) b = 2a (A) f (x) > 0 for all real x
(C) a = -2b (D) b = -2a ( B) f (x) = 0 has distinct real roots
(C ) f (x) = 0 has equal roots
8. If
(D) Range of f (x) is 
x2 + 1 a b c
= + - 10. If
(2 + x)(2 - x)( x - 1) 3( x - 1) 4(2 - x) 12( x + 2)
3x a b é a bù
éa b c ù = + and A=ê ú
( x - 6)( x + 3) x - 6 x + 3 ëb a û
and A = êêb c a úú
êë c a búû then A-1 is equal to
é2 -1 ù
then |A| is equal to ê 3ú é 2 -1ù
(A) ê 3 ú (B) ê ú
(A) –34 (B) 34 ê -1 2ú ë -1 2 û
(C) –24 (D) –108 êë 3 3 úû
é -1 2ù é1 2ù
9. If ê
(C) ê 3 3ú ê
(D) ê 3 3ú
ú ú
x4 A B ê2 -1 ú ê2 1ú
= f ( x) + +
( x - 1)( x - 2) x-1 x-2 êë 3 3 úû êë 3 3 úû

Multiple Correct Choice Type Questions


1. If Then
(A) f (x) = 0 has irrational roots
x2 - 3 A Bx + C
= + 2 (B) f (x) = 0 has no real roots
( x + 2)( x + 1) x + 2
2
x +1
(C) f (x) + A + B = 0 has integer roots
then
(D) f (x) + A + B = 0 has no real roots
(A) 5B = 4 (B) 5A = 1
(C) 5C = -8 (D) ABC = -32 4. If

1 A B C D
2. Let = + + +
x3 ( x + 2) x x2 x3 x + 2
x2 + 1 a b c
= + - then
(2 + x)(2 - x)( x - 1) 3( x - 1) 4(2 - x) 12( x + 2)
(A) A + D = 0
and Z1 = a + bi, Z2 = b + ci and Z3 = c + ai where ( B) C + B = 0
i = -1 . Then Z1, Z2, Z3 represent (C ) the number of permutations of the values of A, B,
(A) collinear points C and –D is 12
( B) vertices of an equilateral triangle 1
(D) A + B =
(C ) vertices of an isosceles triangle 8
(D) vertices of a right-angled triangle

3. Let

x4 A B
= f ( x) + +
x - 3x + 2
2
x-1 x-2
www.jeeneetbooks.in
Answers 473

ANSWERS
Single Correct Choice Type Questions
1. (C) 6. (B)
2. (B) 7. (C)
3. (C) 8. (D)
4. (D) 9. (A)
5. (A) 10. (A)

Multiple Correct Choice Type Questions


1. (A), (B), (C) 3. (B), (D)
2. (C), (D) 4. (A), (B), (C)
www.jeeneetbooks.in
www.jeeneetbooks.in

Index
A Convergent series 211 composition 39–40
Crammer’s rule 413–414 domain 38
Cube roots 132 even 53, 57
Addition
of unity 134–135 image 38–39
of complex numbers 106
odd 53–57
of matrices 364–366
Fundamental theorem 462–463
Adjoint 404
Algebraic equations 136–138 D of algebra 138–139
Algebraic forms 128
Antilogarithmic function 89 De Moivre’s theorem 131–133
ARGANDS 112 De Morgan laws 10–11 G
Argument of z 115–119 Degree of a polynomial 460
Arithmetic geometric progression 220 Derangement 294–295 Gauss–Jordan method 393–394, 415
Arithmetic mean 215–216 Determinants 395–409 Geometric mean 219
Arithmetic progression 211–214 Diagonal matrix 363 Geometric progression 217–218
Arrow-diagram form 29 Diagonal of a matrix 362 Graph of a function 49
Difference of two sets 10
Directly similar triangle 120–122
B Discriminant of an equation 137
H
Discriminant of quadratic expression 171
Disjoint sets 7
Bijection 43–44 Harmonic mean 221–224
Distributive laws 8–9
inverse 44–45 Harmonic progression 221
Divergent series 211
Binary relation 30 Homogenous system of linear
Division algorithm for polynomials 461
Binomial coefficients 329–331 equations 416–417
Domain 28
Binomial theorem 322
for positive integral index 322–328
for rational index 329–333
E I
Idempotent matrix 411
C Elementary row (column) matrix 386
Identity matrix 363–364, 372
Elementary operations 385
Elementary transformations 385 Imaginary axis 112
Cardinality 6
Empty set 3 Improper fraction 460
Cartesian product 25
Equal polynomials 460 Inconsistent system 413, 415
Cayley–Hamilton theorem 410
Equal sets 4 Index set 4
Characteristic equation 410
Equality of matrix 361 Index 411
Characteristic polynomial 410
Equivalence class of relation 33–38 Infinite sequence 208
Circular permutations 284–285
Equivalence relations 33 Infinite set 4
Class of sets 4
Equivalent matrix 387 Injection 41
Coefficient matrix 413
Even extension 57 Integral part of a function 46
Cofactor 399–404
Even functions 53, 57 Intersection of sets 6
Column matrix 362
graphs 53 Inverse of a matrix 388, 393–394
Column transformation 385
Exponential and logarithmic Inverse of a relation 32
Combination 286–288
inequalities 92–93 Invertible matrix 388
Common difference 211
Exponential equation 90 Involuntary matrix 412
Common ratio 218
Exponential function 86–87 Irreducible polynomial 463
Complement of a set 12
Complex number 106
purely real and purely imaginary 110
real part and imaginary part 108–109 F L
square root of 134
unimodular 114 Factorization theorem 461 Limit of a sequence 210
Composition of relations 30 Family of sets 4 Linear permutations 279–281
Conjugate 111 Finite sequence 208 Linear term 170
Consistent system 413, 415 Finite set 4 Logarithmic equation 90
Constant polynomial 465 Fractional part of a function 46 Logarithmic function 88–89
Constant sequence 208 Function 38
www.jeeneetbooks.in
476 Index

M Partition 34 Solution of quadratic equation 170


Period of a matrix 412 Square matrix 361
Periodic function 48 Square roots 132
Mathematical operation on complex
Permutation 278–279 of a complex number 134
numbers
fundamental principle of 278–279 Standard binomial expansion 324
addition 106
Plane or complex plane 112 Standard form 170
difference 106
Polar form 128 Subset 5
multiplication 106
Polynomials 460 Successive differences method 225–226
product 106
Power set 5 Surjection 42
sum 106
Progression 211 Symmetric difference 12–13
subtraction 106
Proper fraction 460 Symmetric matrices 379–383
unity 107
Symmetric relation 33–34
zero 107
Symmetric set 53
Matrix/matrices 360
addition of 364–366 Q System of homogenous linear
equations 413
columns 360
Quadratic equation 170 System of linear equations 412
elements or entries 360
Quadratic expression 170 Systems of logarithmic and exponential
equality of 361
Quadratic term 170 equations 91–92
inverse of 388, 393–394
invertible 388 Quotient 461
involuntary 412
multiplication of 368–369 T
order 360 R
polynomial 375 Tabular form 30
rows 360 Range 28 Telescopic series 225
similar 387 Rank 282 Transitive relation 33–34
singular 405 Rational fraction 460 Transpose of a matrix 404
skew-symmetric 379–382 Real axis 112 Triangular matrix (ces) 364, 409
Maximum value of quadratic Real-valued function 45, 56 Trigonometric form 128
equation 178 Reflexive relation 33–34 Trigonometric notation 128
Minimum value of quadratic Relation (s) 25–28 Trivial solution of a system 417
equation 178 inverse of a 32
Minor 398 reflexive 33–34
Modulus of z 113–114 representations of 29–30 U
Multiplication of matrices 368–369 Relatively prime polynomial 462
Multiplicative identity 372 Remainder 461 Ultimately constant sequence 208
Multiplicative inverse 107–108 Root of quadratic equation 170, 172, Unimodular complex number 114
174–175 Union of sets 7
Roots of degree n 132–133 Unit matrix 372
N Roster form 29 Universal set 12
Row matrix 362
n factorial (factorial n) 278 Row transformation 385
Nilpotent matrix 411 V
Nonhomogenous system of linear
equations 413 S Venn diagrams 13–19
Non-singular matrix 388
Sarrus diagram 398

O
Scalar matrix 363
Scalar 366
Z
Sequence of elements 208
Odd extension 57 Zero matrix 362
Series 210
Odd function 53, 57 Zero of quadratic expression 170
Set (s) 2
graphs 54–56 Zero polynomial 460
class of 4
Ordered pairs 25 elements of a set 2
Orthogonal matrix 383–384 indexed family of 4
members of a set 2
Set builder form 3, 29
P Similar matrix 387
Singular matrix 405
Partial fraction 462–463 Skew-symmetric matrices 379–382
of proper fraction 463 Solution of a system 413
www.jeeneetbooks.in
www.jeeneetbooks.in

You might also like